Sei sulla pagina 1di 360

Calculus

C o n c e p t s an d Ap p l i c a t i o n s
Second Edition

Solutions Manual

P a u l A. F o e r s t e r
Project Editor: Josephine Noah

Project Administrator: Shannon Miller

Consulting Editor: Christopher David

Accuracy Checkers: Jenn Berg, Dudley Brooks

Production Director: Diana Jean Ray

Production Editor: Angela Chen

Copyeditor: Margaret Moore

Production Coordinator: Michael Hurtik

Text and Cover Designer: Jenny Somerville

Art Editors: Jason Luz, Laura Murray Productions

Art and Design Coordinator: Kavitha Becker

Cover Photo Credit: Alec Pytlowany/Masterfile

Compositor: Interactive Composition Corporation

Printer: Alonzo Printing

Executive Editor: Casey FitzSimons

Publisher: Steven Rasmussen

© 2005 by Key Curriculum Press. All rights reserved.


Limited Reproduction Permission
The publisher grants the teacher who purchases Calculus: Concepts and Applications Solutions Manual the
right to reproduce material for use in his or her own classroom. Unauthorized copying of Calculus:
Concepts and Applications Solutions Manual constitutes copyright infringement and is a violation of federal
law.

The Geometer’s Sketchpad, Dynamic Geometry, and Key Curriculum Press are registered trademarks
of Key Curriculum Press. Sketchpad is a trademark of Key Curriculum Press. Fathom Dynamic Statistics
is a trademark of KCP Technologies, Inc. All other registered trademarks and trademarks in this book are the
property of their respective holders.

Key Curriculum Press


1150 65th Street
Emeryville, CA 94608
editorial@keypress.com
www.keypress.com

Printed in the United States of America

10 9 8 7 6 5 4 12 11 10 09 08

ISBN: 978-1-55953-657-8
Contents

Chapter 1 Limits, Derivatives, Integrals, and Integrals .................................................... 1


Chapter 2 Properties of Limits ................................................................................................... 9
Chapter 3 Derivatives, Antiderivatives, and Indefinite Integrals .............................. 28
Chapter 4 Products, Quotients, and Parametric Functions ......................................... 51
Chapter 5 Definite and Indefinite Integrals ....................................................................... 82
Chapter 6 The Calculus of Exponential and Logarithmic Functions ..................... 118
Chapter 7 The Calculus of Growth and Decay ............................................................ 139
Chapter 8 The Calculus of Plane and Solid Figures ................................................... 168
Chapter 9 Algebraic Calculus Techniques for the Elementary Functions .......... 213
Chapter 10 The Calculus of Motion—Averages, Extremes, and Vectors ............. 266
Chapter 11 The Calculus of Variable-Factor Products ................................................. 292
Chapter 12 The Calculus of Functions Defined by Power Series ............................. 313

iii
Overview

This Solutions Manual contains the answers to all problems in Calculus: Concepts and
Applications. Solutions or key steps in the solutions are presented for all but the simplest
problems.
In most cases the solutions are presented in the form your students would be expected to
use. For instance, decimal approximations are displayed as exact answers using ellipsis
format for a mathematical-world answer, then rounded to an appropriate number of
decimal places with units of measurement applied for the corresponding real-world
answer. An answer such as f(3) = 13.7569... ≈ 13.8 cm indicates that the precise answer,
13.7569... , has been retained in memory in the student’s calculator without round-off for
possible use in subsequent computations. The ellipses indicate that the student chooses not
to write all the digits on his or her paper.
Because the problems applying to the real world may be somewhat unfamiliar to both you
and your students, fairly complete solutions are presented for these. Often commentary is
included over and above what the student would be expected to write to further guide your
evaluation of students’ solutions, and in some cases reference is provided to later sections
in which more sophisticated solutions appear. Later in the text, the details of computing
definite integrals by the fundamental theorem are omitted because students are usually
expected to do these numerically. However, exact answers such as V = 8π/3 are presented
where possible in case you choose to have your students do the algebraic integration.
Solutions are not presented for journal entries because these are highly individual for each
student. The “prompts” in most problems calling for journal entries should be sufficient to
guide students in making their own responses.
Where programs are called for, you may use as a model the programs in the Instructor’s
Resource Book. Check the publisher’s Web page (see the address on the copyright page of
this manual) for further information on programs for specific models of the graphing
calculator.
If you or your students find any mistakes, please report them to Key Curriculum Press by
sending in the Correction/Comment Form in the back of this book.

Paul A. Foerster

v
Chapter 1—Limits, Derivatives, Integrals, and Integrals

Problem Set 1-1 3. a. Decreasing fast b. Decreasing slowly


1. a. 95 cm 4. a. Decreasing slowly b. Increasing slowly
b. From 5 to 5.1: average rate ≈ 26.34 cm/s 5. a. Increasing fast b. Increasing slowly
From 5 to 5.01: average rate ≈ 27.12 cm/s c. Decreasing slowly d. Increasing fast
From 5 to 5.001: average rate ≈ 27.20 cm/s 6. a. Decreasing fast b. Increasing slowly
So the instantaneous rate of change of d at c. Increasing fast d. Decreasing fast
t = 5 is about 27.20 cm/s.
7. a. Increasing slowly b. Increasing slowly
c. Instantaneous rate would involve division
by zero. c. Increasing slowly
d. For t = 1.5 to 1.501, rate ≈ −31.42 cm/s. 8. a. Decreasing fast b. Decreasing fast
The pendulum is approaching the wall: The c. Decreasing fast
rate of change is negative, so the distance is 9. a. Increasing fast b. Neither increasing
decreasing. nor decreasing
e. The instantaneous rate of change is the limit c. Increasing fast d. Increasing slowly
of the average rates as the time interval 10. a. Decreasing slowly b. Decreasing fast
approaches zero. It is called the derivative.
c. Decreasing fast d. Neither increasing
f. Before t = 0, the pendulum was not yet nor decreasing
moving. For large values of t, the pendulum’s
motion will die out because of friction. 11. a.
2. a. x = 5: y = 305, price is $3.05 T(x) (°C)
x = 10: y = 520, price is $5.20
x = 20: y = 1280, price is $12.80 100

b. x = 5.1, rate ≈ 46.822 ¢/ft


x = 5.01, rate ≈ 46.9820… ¢/ft
50
x = 5.001, rate ≈ 46.9982… ¢/ft
c. 47 ¢/ft. It is called the derivative.
d. x = 10: 44 ¢/ft. x = 20: 128 ¢/ft x (s)
100 200
e. The 20-ft board costs more per foot than the
10-ft board. The reason is that longer boards x = 40: rate ≈ 1.1°/s
require taller trees, which are harder to find. x = 100: rate = 0°/s
x = 140: rate ≈ −0.8°/s
b. • Between 0 and 80 s the water is warming
Problem Set 1-2 up, but at a decreasing rate.
Q1. Power function, or polynomial function • Between 80 and 120 s the water is boiling,
Q2. f (2) = 8 Q3. Exponential function thus staying at a constant temperature.
Q4. g (2) = 9 Q5. • Beyond 120 s the water is cooling down,
h (x ) rapidly at first, then more slowly.
12. a.
v ( x ) (ft/s)
1
x 70
1
60

50
Q6. h (5) = 25 Q7. y = ax2 + bx + c, a ≠ 0
40
Q8. y=x Q9. y = |x| 30

Q10. Derivative 20

10
1. a. Increasing slowly b. Increasing fast x (s)
2. a. Increasing fast b. Decreasing slowly 1 2 3 4 5 6 7 8

Calculus Solutions Manual Problem Set 1-2 1


© 2005 Key Curriculum Press
x = 2: rate  18 (ft/s)/s c. Substituting 1 for t causes division by zero, so
x = 5: rate = 0 (ft/s)/s r(1) is undefined.
x = 6: rate  11 (ft/s)/s Estimate: r approaches the average of r(0.99)
b. Units are (ft/s)/s, sometimes written as ft/s2. and r(1.01), 108.0586… foxes/year. (Actual
The physical quantity is acceleration. is 108.0604… .)
13. a. The instantaneous rate is called the
derivative.
h (x)
f (4.01)  f (4)
18 d. = 129.9697…
0.01
f (4)  f (3.99)
= 131.4833…
0.01
2 x
Instantaneous rate = (129.9697… 
131.4833…)/2 = 130.7265… foxes/year
3 4 7

• Increasing at x = 3 (actual: 130.7287…)


The answer is negative because the number of
• Decreasing at x = 7
foxes is decreasing.
b. h (3) = 17, h(3.1) = 17.19 a(2.1)  a(2)
15. a. Average rate = =
0.19 0.1
Average rate = = 1.9 ft/s
52.9902… mm /h 2
0.1
c. From 3 to 3.01: 200(1.2 t )  200(1.2 2 )
b. r (t) =
average rate =
0. 0199 t2
= 1. 99 ft/s
0.01
r (t ) (mm2/hr)
From 3 to 3.001:
60
0.001999
average rate = = 1.99 ft/s 40
0.001 20
t (mm)
The limit appears to be 2 ft/s. 2

d. h (7) = 9, h(7.001) = 8.993999 r(2) is undefined.


0.006001
Average rate = = 6.001 ft /s c. r(2.01) = 52.556504…
0.001
52.556504…  52.508608… = 0.04789…
The derivative at x = 7 appears to be 6 ft/s. Use the solver to find t when
The derivative is negative because h(x) is r(t) = 52.508608… + 0.01 = 52.518608… .
decreasing at x = 7.
t = 2.002088… , so keep t within
14. a. 0.002 unit of 2.
f (t ) Not much 4
500 16. a. v(x ) = x 3  v(6) = 288
3
4  (6.13  63 )
300
b. 6 to 6.1: average rate = 3 =
Decrease
Increase
0.1
146.4133…
100
t
10 4  (6 3  5. 93 )
5.9 to 6: average rate = 3 =
0.1
y1 (x )  y1(1) 141.6133…
b. Enter y2 =
x 1 Estimate of instantaneous rate is
(146.4133… + 141.6133…)/2 =
t r(t) = y2 (foxes/year)
144.0133… = 452.4312… cm3/cm.
0.97 110.5684… 4  x 3  4  63
c. r (x ) = 3 3
0.98 109.7361… x6
0.99 108.9001… r(x ) (cm3/cm)
1 undefined
1.01 107.2171… 144π

1.02 106.3703…
48π
x
1.03 105.5200…
6
r(6) is undefined.

2 Problem Set 1-2 Calculus Solutions Manual


© 2005 Key Curriculum Press
d. r(6.1) = 146.4133… = 459.9710… change in f(x), then divide. Repeat, using a
r(6.1) is 7.5817… units from the derivative. smaller change in x. See what number these
Use the solver feature to find x if average rates approach as the change in x
r(x) = 144 + 0.1. approaches zero.
x = 6.001326… , so keep x within 0.00132… • The numerical method illustrates the fact that
unit of 6. the derivative is a limit.
17. a. i. 1.0 in./s ii. 0.0 in./s iii. 1.15 in./s 30. Problems 13 and 14 involve estimating the value
b. 1.7 s, because y = 0 at that time of a limit.
18. a. i. 0.395 in./min ii. 0.14 in./min
iii. 0.105 in./min Problem Set 1-3
b. The rate is negative, because y is decreasing Q1. 72 ft2 Q2. y = cos x
as the tire goes down.
Q3. y = 2 x
Q4. y = 1/x
19. a. Quadratic (or polynomial)
Q5. y = x 2
Q6. f(5) = 4
b. f(3) = 30
Q7. Q8.
c. Increasing at about 11.0 (2.99 to 3.01) y y

20. a. Quadratic (or polynomial)


x x
b. f(1) = 12
c. Increasing at about 6.0 (0.99 to 1.01)
21. a. Exponential
b. Increasing, because the rate of change from Q9. Q10. x = 3
y
1.99 to 2.01 is positive.
22. a. Exponential
b. Increasing, because the rate of change from x
3.01 to 2.99 is positive.
23. a. Rational algebraic
1. f(x) = 0.1x2 + 7 2. f(x) = 0.2x2 + 8
b. Decreasing, because the rate of change from
a. Approximately 30.8 a. Approximately 22.2
3.99 to 4.01 is negative.
b. Approximately 41.8 b. Approximately 47.1
24. a. Rational algebraic
b. Increasing, because the rate of change from 7
f (x) 8 f(x)

2.01 to 1.99 is positive.


25. a. Linear (or polynomial)
b. Decreasing, because the rate of change from
4.99 to 5.01 is negative.
26. a. Linear (or polynomial) x
–1 5 6 x
b. Increasing, because the rate of change from –2 3 5
7.99 to 8.01 is positive.
27. a. Circular (or trigonometric)
3. h(x) = sin x 4. g( x) = 2 x + 5
b. Decreasing, because the rate of change from
a. Approximately 2.0 a. Approximately 7.9
1.99 to 2.01 is negative.
b. Approximately 1.0 b. Approximately 12.2
28. a. Circular (or trigonometric)
g ( x)
b. Decreasing, because the rate of change from h(x)

0.99 to 1.01 is negative.


29. • Physical meaning of a derivative: 1
6
instantaneous rate of change
• To estimate a derivative graphically: Draw a
tangent line at the point on the graph and x
measure its slope. 3

• To estimate a derivative numerically: Take a


x
small change in x, find the corresponding –1 1 2

Calculus Solutions Manual Problem Set 1-3 3


© 2005 Key Curriculum Press
5. There are approximately 6.8 squares between the v(3.01) – v(2.99)
d. Rate ≈ = 1.8648K
curve and the x-axis. Each square represents 3.01 − 2.99
(5)(20) = 100 feet. So the distance is about About 1.86 (ft/s)/s
(6.8)(100) = 680 feet. The derivative represents the acceleration.
6. There are approximately 53.3 squares between the 11. From t = 0 to t = 5, the object travels about
curve and the x-axis. Each square represents 11.4 cm. From t = 5 to t = 9, the object travels
(0.5)(10) = 5 miles. So the distance is about back about 4.3 cm. So the object is located about
(53.3)(5) = 266.5 miles. 11.4 − 4.3 = 7.1 cm from its starting point.
tan 1.01 – tan 0.99 12. See the text for the meaning of derivative.
7. Derivative ≈ = 3.42K
1.01 – 0.99 13. See the text for the meaning of definite integral.
8. Derivative = −7 (exactly, because that is the 14. See the text for the meaning of limit.
slope of the linear function)
9. a. Problem Set 1-4
v (t )
100
Q1. y changes at 30 Q2. Derivative ≈ −500
Q3. Q4. f (3) = 9
y
60
x

t
5 8.7 10 Q5. 100 Q6. sin (π/2) = 1
Q7. 366 days Q8. Derivative
The range is 0 ≤ y ≤ 32.5660… .
Q9. Definite integral Q10. f (x) = 0 at x = 4
b. Using the solver, x = 8.6967… ≈ 8.7 s.
1. a.
c. By counting squares, distance ≈ 150 ft. v (t )
The concept used is the definite integral.
v(5.01) − v( 4.99) 20,000
d. Rate ≈ = 3.1107K
5.01 − 4.99
About 3.1 (ft/s)/s t
The concept is the derivative. 30
The rate of change of velocity is called
acceleration. b. Integral ≈ 5(0.5v(0) + v(5) + v(10) + v(15) +
v(20) + v(25) + 0.5v(30)) = 5(56269.45…) =
10. a.
281347.26… ≈ 281,000 ft
v (t)
10 The sum overestimates the integral because
the trapezoids are circumscribed about the
region and thus include more area.
c. The units are (ft/s)(s), which equals feet, so
5
the integral represents the distance the
spaceship has traveled.
d. Yes, it will be going fast enough, because
t
1 2 3 4 5
v(30) = 27,919.04… , which is greater than
27,000.
2. a. v(t) = 4 + sin 1.4t
b. v(4) = 9.3203… ≈ 9.3 ft/s
v (t )
Domain: 0 ≤ t ≤ 4 5

Range: 0 ≤ v(t) ≤ 9.3203…


c. By counting squares, the integral from t = 0
to t = 4 is about 21.3 ft. The units of the
t
integral are (ft/s) · s = ft. The integral tells 3
the length of the slide.

4 Problem Set 1-4 Calculus Solutions Manual


© 2005 Key Curriculum Press
b. A definite integral has the units of the b. T10 = 8.6700…
x-variable times the y-variable. Distance = T20 = 8.6596…
rate × time. Because v(t) is distance/time Τ50 = 8.65672475…
and t is time, their product is expressed in These values overestimate the integral,
units of distance. because the trapezoids are circumscribed
c. See graph in part a. about the region.
Distance ≈ 0.5(0.5v(0) + v(0.5) + v(1) + c. T10: 0.01385… unit from answer
v(1.5) + v(2) + v(2.5) + 0.5v(3)) = T20: 0.003465… unit from answer
0.5(26.041…) = 13.02064… ≈ 13.0 ft T50: 0.0005545… unit from answer
d. v(3) = 3.128… ≈ 3.1 mi/h Tn is first within 0.01 of 8.65617024… when
Maximum speed was 5 mi/h at about 1.12 h. n = 12.
T12 = 8.665795… , which is 0.009624… unit
3. Distance ≈ 0.6(150 + 230 + 150 + 90 + 40 + 0) =
from 8.65617024… .
396 ft
Because Tn is getting closer to the exact
4. Volume ≈ 3(2500 + 8000 + 12000 + 13000 + answer as n increases, Tn is within 0.01 unit
11000 + 7000 + 4000 + 6000 + 4500) = of the answer for all n ≥ 12.
204,000 ft3 9. From the given equation,
5. Programs will vary depending on calculator. See y = ±( 40/110) 110 2 – x 2 . Using the trapezoidal
the program TRAPRULE in the Instructor’s
rule program on the positive branch with n = 100
Resource Book for an example. The program
increments gives 6904.190… for the top half of
gives T20 = 23.819625.
the ellipse. Doubling this gives an area of
6. See the program TRAPDATA in the Instructor’s 13,808.38… cm 2 . The estimate is too low
Resource Book for an example. The program because the trapezoids are inscribed within the
gives T7 = 33, as in Example 2. ellipse. The area of an ellipse is πab, where a
7. a. and b are the x- and y-radii, respectively. So
f (x )
the exact area is π (110)(40) = 4400π =
7 13,823.007… cm 2 , which agrees both with the
answer and with the conclusion that the
trapezoidal rule underestimates the area.
x
10. Integral = 1(0.0 + 2.1 + 7.9 + 15.9 + 23.8 +
29.7 + 31.8 + 29.7 + 23.8 + 15.9 + 7.9 +
1 4

b. T10 = 18.8955 2.1 + 0) = 190.6


T20 = 18.898875 The integral will have the units (in.2)(in.) = in.3,
T50 = 18.89982 representing the volume of the football.
These values underestimate the integral, 11. n = 10: integral ≈ 21.045
because the trapezoids are inscribed in the n = 100: integral ≈ 21.00045
region. n = 1000: integral ≈ 21.0000045
c. T10: 0.0045 unit from the exact answer Conjecture: integral = 21
T20: 0.001125 unit from the exact answer The word is limit.
T50: 0.00018 unit from the exact answer 12. The trapezoidal rule with n = 100 gives
Tn is first within 0.01 unit of 18.9 when integral ≈ 156.0096.
n = 7. Conjecture: integral = 156
T7 = 18.8908… , which is 0.0091… unit 13. If the trapezoids are inscribed (graph concave
from 18.9. down), the rule underestimates the integral.
Because Tn is getting closer to 18.9 as n If the trapezoids are circumscribed (graph concave
increases, Tn is within 0.01 unit of 18.9 for up), the rule overestimates the integral.
all n ≥ 7.
8. a.
g (x )

1 x
Concave down Concave up
1 3 Inscribed trapezoids Circumscribed trapezoids
Underestimates integral Overestimates integral

Calculus Solutions Manual Problem Set 1-4 5


© 2005 Key Curriculum Press
Problem Set 1-5 R3. By counting squares, the integral is
approximately 23.2.
1. Answers will vary.
Distance ≈ 23.2 ft (exact answer: 23.2422…)
Concept: definite integral
R4. a.
Problem Set 1-6
f (x )

Review Problems 5

R1. a. When t = 4, d = 90 − 80 sin [1, 2(4 − 3)] ≈


15.4 ft.
b. From 3.9 to 4: average rate ≈ –40.1 ft/s x

From 4 to 4.1: average rate ≈ −29.3 ft/s 1 4

Instantaneous rate ≈ −34.7 ft/s


The graph agrees with Figure 1-6c.
The distance from water is decreasing, so he is
going down. b. By counting squares, integral ≈ 15.0.
(Exact answer is 15.)
d (5.01) − d ( 4.99) c. T 6 = 0.5(2.65 + 5.575 + 5.6 + 5.375 + 4.9 +
c. Instantaneous rate ≈ ≈ 70.8
0.02 4.175 + 1.6) = 14.9375
d. Going up at about 70.8 ft/s The trapezoidal sum underestimates the
e. Derivative integral because the trapezoids are inscribed in
the region.
R2. a. Physical meaning: instantaneous rate of d. T50 = 14.9991; Difference = 0.0009
change of a function T100 = 14.999775; Difference = 0.000225
Graphical meaning: slope of a tangent line to The trapezoidal sums are getting closer to 15.
a function at a given point Concept: limit
b. x = − 4: decreasing fast R5. Answers will vary.
x = 1: increasing slowly
x = 3: increasing fast
x = 5: neither increasing nor decreasing Concept Problems
c. From 2 to 2.1: C1. a. f (3) = 32 − 7.3 + 11 − 1
52.1 − 52
average rate = = 43.6547K b. f (x) − f (3) = x2 − 7x + 11 + 1 = x 2 − 7x + 12
0.1
c. f ( x ) – f (3) = x – 7 x + 12 = ( x – 4)( x – 3) =
2
From 2 to 2.01:
52.01 − 52 x–3 x–3 x–3
average rate = = 40.5614K
0.01 x − 4, if x ≠ 3
From 2 to 2.001: d. The limit is found by substituting 3 for x
52.001 − 52 in (x − 4).
average rate = = 40.2683K
0.001 Limit = exact rate = 3 − 4 = −1
Differences between average rates and C2. The line through (3, f (3)) with slope −1 is
instantaneous rates, respectively: y = −x + 2.
43.6547… − 40.235947… = 3.4187…
f (x )
40.5617… − 40.235947… = 0.3255…
40.2683… − 40.235947… = 0.03239…
The average rates are approaching the
instantaneous rate as x approaches 2.
The concept is the derivative. 2 x
The concept used is the limit. 3

d. t = 2: 3.25 m/s
The line is tangent to the graph. Zooming in by
t = 18: 8.75 m/s
a factor of 10 on the point (3, 2) shows that the
t = 24: 11.5 m/s
graph becomes straighter and looks almost like
Her velocity stays constant, 7 m/s, from 6 s
the tangent line. (Soon students will learn that
to 16 s. At t = 24, Mary is in her final sprint
this property is called local linearity.)
toward the finish line.

6 Problem Set 1-6 Calculus Solutions Manual


© 2005 Key Curriculum Press
T5. Concept: definite integral
By counting squares, distance ≈ 466.
(Exact answer is 466.3496… .)
2
T6.
3
Speed (ft/s)
25
4 x 2 − 19 x + 21 ( 4 x − 7)( x − 3)
C3. a. f ( x ) = = = 20
x −3 x −3 15
4x − 7, x ≠ 3
When x = 3, 4x − 7 = 4 ⋅ 3 − 7 = 5.
10

5
b. Time (s)

f (x ) (ft) 5 10 15 20 25 30 35 40
6
5.8

5
4.2
T 7 = 5(2.5 + 5 + 5 + 10 + 20 + 25 + 20 + 5) =
4 462.5
Trapezoidal rule probably underestimates the
3 integral, but some trapezoids are inscribed and
some circumscribed.
2

1
T7. Concept: derivative
2.8 3.2 x (s)
Speed (ft/s)
1 2 3 4 5 6 25

c. 5.8 = 4(3 + δ ) − 7 4.2 = 4(3 − δ ) − 7 20

5.8 = 12 + 4δ − 7 4.2 = 12 − 4δ − 7 15

4δ = 0.8 −4δ = −0.8 10


δ = 0.2 δ = 0.2 5

d. 4(3 + δ ) − 7 = 5 + ε
Time (s)

12 + 4δ − 7 = 5 + ε
5 10 15 20 25 30 35 40

4δ = ε
δ = 14 ε Slope ≈ −1.8 (ft/s)/s
There is a positive value of δ, namely 14 ε , for (Exact answer is −1.8137… .)
each positive value of ε, no matter how small Name: acceleration
ε is. T8. The roller coaster is at the bottom of the hill at
e. L = 5, c = 3. “. . . but not equal to 3” is 25 s because that’s where it is going the fastest.
needed so that you can cancel the (x − 3) The graph is horizontal between 0 and 10 seconds
factors without dividing by zero. because the velocity stays constant, 5 ft/s, as the
roller coaster climbs the ramp.
Chapter Test
T9. Distance = (rate)(time) = 5(10) = 50 ft
T1. Limit, derivative, definite integral, indefinite
integral T10. T5 = 412.5; T50 = 416.3118… ;
T100 = 416.340219…
T2. See the text for the definition of limit.
T3. Physical meaning: instantaneous rate T11. The differences between the trapezoidal sum and
the exact sum are:
T4.
y For T5: difference = 3.8496…
6 For T50: difference = 0.03779…
For T100: difference = 0.009447…
3 The differences are getting smaller, so Tn is
getting closer to 416.349667… .
x
2 5

Calculus Solutions Manual Problem Set 1-6 7


© 2005 Key Curriculum Press
T12. From 30 to 31: The differences are getting smaller, so the average
y(31) − y(30) rates are getting closer to the instantaneous rate.
average rate = = −1.9098K
1 y( x ) − y(30)
From 30 to 30.1: T15. Solve = −1.81379936 + 1, getting
x − 30
y(30.1) − y(30) x = 30.092220… . So keep x within 0.092…
average rate = = −1.8246K
0.1 unit of 30, on the positive side.
From 30 to 30.01:
T16. Concept: derivative
y(30.01) − y(30)
average rate = = −1.8148K f ( 4.3) − f (3.7) 35 − 29
0.01 T17. f ′( 4) ≈ = = 10
4.3 − 3.7 0.6
T13. The rates are negative because the roller coaster is
T18. Answers will vary.
slowing down.
T14. The differences between the average rates and
instantaneous rate are:
For 30 to 31: difference = 0.096030…
For 30 to 31.1: difference = 0.010833…
For 30 to 30.01: difference = 0.001095…

8 Problem Set 1-6 Calculus Solutions Manual


© 2005 Key Curriculum Press
Chapter 2—Properties of Limits

Problem Set 2-1 h (x )

8 − 10 + 2 0
1. a. f (2) = = 2
2−2 0
No value for f (2) because of division by zero.
x
b. 2.7 3

x f (x)
There appears to be no limit, because the graph
1.997 2.994
cycles infinitely as it approaches x = 3.
1.998 2.996
1.999 2.998
2 undefined Problem Set 2-2
2.001 3.002 Q1. Q2.
y y
2.002 3.004 8

2.003 3.006 x
π
Yes, f (x) stays close to 3 when x is kept
close to 2, but not equal to 2. x –1
3
c. To keep f (x) within 0.0001 unit of 3, keep
x within 0.00005 unit of 2. To keep f (x)
Q3. Q4.
within 0.00001 unit of 3, keep x within y y
0.000005 unit of 2. To keep f (x) arbitrarily x
close to 3, keep x within 12 that distance 4 –2 2
of 2.
d. The discontinuity can be “removed” by
x
defining f (2) to equal 3. 6
–4

2.
g (x ) Q5. Q6. Trapezoidal rule
3 y

2 4

x
3 x
1

g (x )
Q7. Counting squares
2 Q8. Slope of the tangent line
Q9. Instantaneous rate of change
x Q10. B
3
1. See the text for the definition of limit.
2. f (x) might be undefined at x = c, or might have a
The limit seems to be 2. value at x = c that is different from the limit.
3.
3. Has a limit, 3 4. Has a limit, 2
h (x )
5. Has a limit, 3 6. Has a limit, 5
2 7. Has no limit 8. Has no limit
9. Has a limit, 7 10. Has a limit, 20
x 11. Has no limit 12. Has no limit
3 13. lim f ( x ) = 5. For ε = 0.5, δ ≈ 0.2 or 0.3.
x →3

Calculus Solutions Manual Problem Set 2-2 9


© 2005 Key Curriculum Press
14. lim f ( x ) = 3. For ε = 0.5, δ ≈ 0.8. ∴ 24− x = 1.8
x →2
log 1.8
15. lim f ( x ) = 4. For ε = 0.7, δ ≈ 0.5 or 0.6. x = 4−
x →6 log 2
(The right side is more restrictive.)  log 1.8 
16. lim f ( x ) = 2. For ε = 0.8, δ ≈ 0.7 or 0.8. Max. δ = 4 −  4 −  = 0.84799K
x→4  log 2 
(The left side is more restrictive.)
17. lim f ( x ) = 2. For ε = 0.3, δ ≈ 0.5 or 0.6. d. Because the left side is more restrictive, set
x →5
1 + 24− x = 2 + ε .
(The right side is more restrictive.) ∴ 24− x = 1 + ε
18. lim f ( x ) = 6. For ε = 0.4, δ ≈ 0.1.
x →3 log(1 + ε )
x = 4−
19. a. The graph should match Problem 13. log 2
b. lim f ( x ) = 5
x →3  log(1 + ε )  log(1 + ε )
Max. δ = 4 − 4 − =
log 2 
,
c. Graph is symmetrical about x = 3.  log 2
Let 5 − 2 sin (x − 3) = 5 + 0.5 = 5.5. which is positive for all ε > 0.
∴ sin (x − 3) = −0.25
23. a. The graph should match Problem 17.
x = 3 + sin− 1 (−0.25)
Max. δ = 3 − [3 + sin− 1 (−0.25)] = 0.25268… b. lim f ( x ) = 2
x →5
d. Let 5 − 2 sin (x − 3) = 5 + ε . c. The right side is more restrictive.
∴ sin (x − 3) = −ε/2 Let (x − 5)2 + 2 = 2 + 0.3 = 2.3.
x = 3 + sin− 1 (−ε/2)
Max. δ = 3 − [3 + sin− 1 (−ε/2)] = ∴ x = 5 + 0.3
−sin− 1 (−ε/2) = sin− 1 (ε/2), which is positive Max. δ = (5 + 0.3 ) − 5 = 0.54772 K
for any positive value of ε.
d. Because the right side is more restrictive, set
20. a. The graph should match Problem 14. (x − 5)2 + 2 = 2 + ε .
b. lim f ( x ) = 3 ∴ x = 5+ ε
x →2

c. The graph is symmetrical about x = 2. Max. δ = (5 + ε ) − 5 = ε , which is


Let (x − 2)3 + 3 = 3 + 0.5 = 3.5. positive for all ε > 0.

∴ x = 2 + 3 0.5 24. a. The graph should match Problem 18.


Max. δ = 2 + 3 0.5 − 2 = 3 0.5 = 0.7937K b. lim f ( x ) = 6
x →3
d. Let (x − 2) + 3 = 3 + ε .
3
c. The graph is symmetrical about x = 3.
∴ x = 2 + ε 1/3 Let 6 − 2(x − 3)2/3 = 6 − 0.4 = 5.6.
Max. δ = 2 + ε 1/3 − 2 = ε 1/3, which is ∴ x = 3 + 0.23/2
positive for any positive value of ε. Max. δ = (3 + 0.23/2) − 3 = 0.08944…
21. a. The graph should match Problem 15.
d. Let 6 − 2(x − 3)2/3 = 6 − ε .
b. lim f ( x ) = 4
x →6 ∴ x = 3 + (ε/2)3/2
c. The right side is more restrictive. Max. δ = [3 + (ε/2)3/2] − 3 = (ε/2)3/2, which
Let 1 + 3(7 − x)1/3 = 4 − 0.7 = 3.3. is positive for all ε > 0.
∴ x = 7 − (2.3/3)3
Max. δ = [7 − (2.3/3)3] − 6 = 0.5493… (52 − 6 ⋅ 5 + 13)(5 − 2) (5)(0) 0
25. a. f (2) = = =
d. Because the right side is more restrictive, set 5−2 0 0
1 + 3 3 7 − x = 4 − ε. The graph has a removable discontinuity at
∴ x = 7 − [(3 − ε)/3]3 x = 2.
Max. δ = 7 − [(3 − ε)/3)3] − 6 = 1 − [(3 − ε)/3]3, Limit = 22 − 6(2) + 13 = 5
which is positive for all positive values of ε. b. When f (x) = 5.1, x = 1.951191… .
22. a. The graph should match Problem 16. δ1 = 2 − 1.951191… = 0.048808…
b. lim f ( x ) = 2 When f (x) = 4.9, x = 2.051316… .
x→4
δ2 = 2.051316… − 2 = 0.051316…
c. The left side is more restrictive. ∴ max. δ = 0.048808…
Let 1 + 24− x = 2 + 0.8 = 2.8.

10 Problem Set 2-2 Calculus Solutions Manual


© 2005 Key Curriculum Press
c. e. The limit of the average velocity is the
f (x ) instantaneous velocity.
L=5

Problem Set 2-3


δ δ
Q1. 13
Q2. Q3.
y y

x 3
c=2
x x
26. a. 2 –4
y
Q4. Q5.
8 y y
1
x

x 1
x
2

The graph is linear. Q6. (x − 10)(x + 10) Q7. 75%


There is a removable discontinuity at x = 2. Q8. Product of x and y, where x varies and y may
The limit appears to be 9. vary
4(2 2 ) − 7(2) − 2 0 Q9.
b. f (2) = =
2–2 0 3 1 –8 22 –21
Indeterminate form 3 –15 21
( 4 x + 1)( x − 2)
c. f ( x ) = = 4 x + 1, x ≠ 2 1 –5 7 0
x−2
x 2 − 5x + 7
Limit = 4(2) + 1 = 9
Q10. D
If x ≠ 2, then (x − 2) ≠ 0. Canceling is a
division process, but because (x − 2) ≠ 0, 1.
y
you do not risk dividing by zero. g+h

d. If f (x) = 9.001, x = 2.00025.


If f (x) = 8.999, x = 1.99975. 10

δ1 = 2.00025 − 2 = 0.00025 h
δ2 = 2 − 1.99975 = 0.00025
g
x
Largest number is 0.00025. 2

e. L = 9, c = 2, ε = 0.001, δ = 0.00025
lim f ( x ) = 10, lim g( x ) = 4, and lim h( x ) = 6
d (t ) − d ( 4) 3t 2 – 48 x →2 x →2 x →2
27. a. m(t ) = = ∴ lim f ( x ) = lim g( x ) + lim h( x ), Q .E.D .
t−4 t−4 x →2 x →2 x →2
b. Removable discontinuity at x = 4.
m (t )
x f (x)
30
1.96 9.9640…
1.97 9.9722…
1.98 9.9810…
t
4
1.99 9.9902…
2.00 10
c. Limit = 24 ft/s
3(t − 4)(t + 4) 2.01 10.0102…
d. m(t ) = = 3t + 12, if t ≠ 4
t–4 2.02 10.0209…
3t + 12 = 24.12 ⇒ t = 4.04 2.03 10.0322…
3t + 12 = 23.88 ⇒ t = 3.96 2.04 10.0439…
Keep t within 0.04 s of 4 s.
All these f (x) values are close to 10.

Calculus Solutions Manual Problem Set 2-3 11


© 2005 Key Curriculum Press
2. x y 3 = f (x)
y
g
0.997 2.9739…
9
0.998 2.9825…
1.8 f 0.999 2.9912…
x
3 1 3
1.001 3.0087…
lim f ( x ) = 1.8 and lim g( x ) = 9
x →3 x →3
1.002 3.0174…
∴ lim f ( x ) = 0.2 lim g( x ), Q.E.D.
x →3 x →3 1.003 3.0262…
x f (x)
All these f (x) values are close to 2(1.5) = 3.
2.96 1.75232
3π 
2.97 1.76418 6. 2 3 = 8 and sin  = 0.5
 3.6 
2.98 1.77608
8
2.99 1.78802 r(3) = = 16
0.5
3.00 1.8
3.01 1.81202
x r(x)
3.02 1.82408
2.9997 15.9894…
3.03 1.83618
3.04 1.84832 2.9998 15.9929…
2.9999 15.9964…
All these f (x) values are close to 1.8.
3. 3 16
f (x )
Limit = 7 3.0001 16.0035…
7
3.0002 16.0070…
3.0003 16.0105…
x
3 All these r(x) values are close to 16.
2 3.6
The limit is 7 because f (x) is always close to 7, lim f ( x ) → , so the limit of a quotient
x →3.6 0
no matter what value x takes on. (It shouldn’t
cannot be applied because of division by zero.
bother you that f (x) = 7 for x ≠ 3 if you think of
the definition of limit for a while.) 7. lim f ( x ) = lim x 2 − 9 x + 5
x →3 x →3
4.
f (x) = x = lim x − lim 9 x + lim 5
2
Limit of a sum
x →3 x →3 x →3
Limit = 6 (or difference)
x = lim x ⋅ lim x − 9 lim x + 5
x →3 x →3 x →3
6
Limit of a product,
limit of a constant
= (3)(3) − 9(3) + 5 Limit of x
lim f ( x ) = 6. The y-value equals the x-value. = 9 − 27 + 5 = −13
x →6

5. 8. lim f ( x ) = lim x 2 + 3 x − 6
x →−1 x →−1
y
5 = lim x 2 + lim 3 x − lim 6
y1 x →−1 x →−1 x →−1
Limit of a sum
y1 y2
y2 = lim x ⋅ lim x + 3 lim x − 6
x →−1 x →−1 x →−1
x Limit of a product,
1
limit of a constant
lim y1 = 2, lim y2 = 1.5, and lim y1 ⋅ y2 = 3 = (−1)(−1) + 3(−1) − 6 Limit of x
x →1 x →1 x →1
2(1.5) = 3, ∴ lim y1 ⋅ lim y2 = lim y1 ⋅ y2 = 1 − 3 − 6 = −8
x →1 x →1 x →1

12 Problem Set 2-3 Calculus Solutions Manual


© 2005 Key Curriculum Press
9. r (x ) Proof:
–2
lim f ( x ) = lim ( x 2 + 2 x + 6) Because x ≠ 5
x
x →5 x →5

= lim x + lim (2 x ) + lim 6


2
Limit of a sum
–8 x →5 x →5 x →5

= lim x ⋅ lim x + 2 lim x + 6 Limit of a product,


x →5 x →5 x →5
( −2) − 4( −2) − 12 4 + 8 − 12 0
2 limit of a constant
r(–2) = = = times a function,
( −2) + 2 0 0 limit of a constant
( x – 6)( x + 2)
r( x ) = = x − 6, x ≠ −2 = 5 ⋅ 5 + 2 · 5 + 6 = 41, Q .E .D .
x+2
Limit of x
lim r ( x ) = −2 − 6 = −8
x →−2
12.
f (x )
Proof: 28

lim r ( x ) = lim ( x − 6) Because x ≠ −2


x →−2 x →−2
= lim x + lim (–6) Limit of a sum
x →−2 x →−2
x
= −2 − 6 = −8, Q .E .D . Limit of x, limit of a 3
constant
10. f (x )
13 33 + 32 − 5(3) − 21 27 + 9 − 15 − 21 0
f (3) = = =
3−3 0 0
( x 2 + 4 x + 7)( x – 3)
x f ( x) = = x 2 + 4 x + 7, x ≠ 3
x–3
5
lim f ( x ) = 32 + 4(3) + 7 = 28
52 + 3(5) − 40 25 + 15 − 40 0 x →3
f (5) = = =
5−5 0 0 Proof:
( x + 8)( x – 5) lim f ( x ) = lim ( x 2 + 4 x + 7)
f ( x) = = x + 8, x ≠ 5 x →3 x →3
x–5
Because x ≠ 3
lim f ( x ) = 5 + 8 = 13
x →5 = lim x 2 + lim 4 x + lim 7 Limit of a sum
x →3 x →3 x →3

Proof: = lim x ⋅ lim x + 4 lim x + 7 Limit of a product,


x →3 x →3 x →3
lim f ( x ) = lim ( x + 8) Because x ≠ 5 limit of a constant
x →5 x →5 times a function,
= lim x + lim 8 Limit of a sum limit of a constant
x →5 x →5
= 5 + 8 = 13, Q .E .D . Limit of x, limit of a = 3 ⋅ 3 + 4 ⋅ 3 + 7 = 28, Q .E .D .
constant Limit of x
11. f (x ) 13.
41
f (x )
9

10
x x
5 –1

53 − 3(52 ) − 4(5) − 30
f ( −5) =
5−5
125 − 75 − 20 − 30 0 ( −1)3 − 4( −1) 2 − 2( −1) + 3
= = f ( −1) =
0 0 ( −1) + 1
( x 2 + 2 x + 6)( x – 5)
f ( x) = = x 2 + 2 x + 6, x ≠ 5 −1 − 4 + 2 + 3 0
x–5 = =
0 0
lim f ( x ) = 52 + 2(5) + 6 = 41
x →5

Calculus Solutions Manual Problem Set 2-3 13


© 2005 Key Curriculum Press
( x 2 – 5 x + 3)( x + 1) 15.
f ( x) = = x 2 − 5 x + 3, x ≠ −1
x +1 x f (x)
lim f ( x ) = ( −1) − 5( −1) + 3 = 9
2
4.990 40.8801
x →−1
4.991 40.8921…
Proof: 4.992 40.9040…
lim f ( x ) = lim ( x 2 − 5 x + 3) 4.993 40.9160…
x →−1 x →−1
Because x ≠ −1 4.994 40.9280…
= lim x 2 + lim (–5 x ) + lim 3 4.995 40.9400…
x → –1 x → –1 x → –1
Limit of a sum 4.996 40.9520…
= lim x ⋅ lim x – 5 lim x + 3 4.997 40.9640…
x → –1 x → –1 x → –1
4.998 40.9760…
Limit of a product,
limit of a constant 4.999 40.9880…
times a function, 5 undefined
limit of a constant 5.001 41.0120…
= (−1)(−1) + (−5)(–1) + 3 = 9, Q.E.D.
5.002 41.0240…
Limit of x
5.003 41.0360…
14. 5.004 41.0480…
f (x)
2 x 5.005 41.0600…
–17 5.006 41.0720…
5.007 41.0840…
5.008 41.0960…
5.009 41.1080…

The table shows that f (x) will be within 0.1 unit


2 4 − 11(2 3 ) + 21(2 2 ) − 2 − 10 of lim f ( x ) = 41 if we keep x within 0.008 unit
f (2) = x →5
2−2 of 5.
16 − 88 + 84 − 2 − 10 0
= = 16.
0 0 f (x )

( x – 9 x + 3 x + 5)( x – 2)
3 2 9
f ( x) =
x–2
x
= x − 9 x + 3 x + 5, x ≠ 2
3 2
–1
lim f ( x ) = 2 3 − 9(2 2 ) + 3(2) + 5 = −17
x →2

Proof: When x is close to –1, f (x) is close to 9.


lim f ( x ) = lim ( x 3 – 9 x 2 + 3 x + 5) x 2 − 5 x + 6 ( x − 2)( x − 3) x − 2
x →2 x →2 17. f ( x ) = = =
Because x ≠ 2 x 2 − 6 x + 9 ( x − 3)( x − 3) x − 3
= lim x 3 + lim ( −9 x 2 ) + lim 3 x + lim 5 You cannot find the limit by substituting into
x →2 x →2 x →2 x →2
the simplified form because the denominator still
Limit of a sum
becomes zero.
= lim x ⋅ lim x ⋅ lim x + −
( 9 ) lim x ⋅ lim x
x →2 x →2 x →2 x →2 x →2
x3 − 8 ( x − 2)( x 2 + 2 x + 4)
+ 3 lim x + 5 18. f ( x ) = =
x →2 x − 4x + 4
2
( x − 2)( x − 2)
Limit of a product,
x2 + 2x + 4
limit of a constant =
times a function, x−2
limit of a constant You cannot find the limit by substituting into
= 2 ⋅ 2 ⋅ 2 + (−9)(2 ⋅ 2) + 3 ⋅ 2 + 5 = −17, the simplified form because the denominator still
Q .E .D . Limit of x goes to zero.

14 Problem Set 2-3 Calculus Solutions Manual


© 2005 Key Curriculum Press
19. a. 5(0)1/2 = 0 = v(0) 21. By the symmetric difference quotient,
5(1)1/2 = 5 = v(1) 0.75.01 – 0.7 4.99
derivative ≈ = −0.05994 K .
5(4)1/2 = 10 = v(4) 2(0.01)
5(9)1/2 = 15 = v(9) 22. By the trapezoidal rule with n = 100,
5(16)1/2 = 20 = v(16) integral ≈ 11.8235K .
b. a(9) ≈
v(9.001) – v(9)
= 0.8333101K 23. Prove that lim x n = c n for any positive integer n.
x →c
9.001 – 9
Conjecture: a(9) = 0.83 = 5/6
Proof:
Units of a(t): (mi/h)/s
Anchor:
v(t ) – v( 9) 5t 1/ 2 – 15
c. a(9) = lim = lim
t →9 t–9 t →9 t–9 If n = 1, lim x 1 = c = c1 by the limit of x.
x →c
1/ 2
5(t – 3)
= lim 1/ 2
t →9 (t – 3)(t 1/ 2 + 3) Induction Hypothesis:
5 Assume that the property is true for n = k.
= lim 1/ 2
t →9 t +3 ∴ lim x k = c k
x →c
5 Verification for n = k + 1:
= , which agrees with the conjecture.
6 lim x k +1 = lim ( x k ⋅ x )
x →c x →c
d. Distance = integral of v(t) from 1 to 9. By the
trapezoidal rule with n = 100 increments, = lim x ⋅ lim x = c k ⋅ c
k
By the induction
x →c x →c
integral ≈ 86.6657… . The units are hypothesis
(mi/h) · s. To convert to ft, multiply by 5280 = c k +1
and divide by 3600, getting 127.1111…
(exact: 127 19 ) . The truck went about 127 ft. Conclusion:
2.1 – 2 3 3
∴ lim x n = c n for all integers n ≥ 1, Q.E.D.
20. a. Derivative ≈ = 12.61 x →c
2.1 – 2 24. Answers will vary.
x 3 – 8 ( x – 2)( x 2 + 2 x + 4)
b. = =
x–2 x–2
x 2 + 2 x + 4, provided x ≠ 2. This expression Problem Set 2-4
approaches 12 as x approaches 2.
Q1. Instantaneous rate of change
Proof: Q2. Product of x and y, where x varies and y
can vary
x3 – 8
lim = lim ( x 2 + 2 x + 4) Q3. 0.0005
x →2 x – 2 x →2
Because x ≠ 2 Q4.
= lim x 2 + lim 2 x + lim 4
x →2 x →2 x →2
Limit of a sum
= lim x ⋅ lim x + 2 lim x + 4
x →2 x →2 x →2
Limit of a product,
limit of a constant Q5. Exponential function
= 2 · 2 + 2 · 2 + 4 = 12, Q .E .D . Q6.
Limit of x y = cos x
x
c. The line through point (2, 8) with slope 12 is
y = 12x − 16. The line appears to be tangent
to the graph of f at point (2, 8).

f (x )
Q7. (x + 6)(x − 1) Q8. 53
Q9. 120 Q10. 103
12
1. a. Has left and right limits
8
1 b. Has no limit
x c. Discontinuous. Has no limit
2

Calculus Solutions Manual Problem Set 2-4 15


© 2005 Key Curriculum Press
2. a. Has left and right limits 15. Answers may vary. 16. Answers may vary.
b. Has a limit f (x ) f (x )

c. Discontinuous. No f (3)
3. a. Has left and right limits
b. Has a limit
c. Continuous x
x
6
4. a. Has left and right limits 2

b. Has a limit
c. Continuous 17. Answers may vary. 18. Answers may vary.
f(x)
5. a. Has no left or right limit f (x )

b. Has no limit 10
c. Discontinuous. No limit or f (2) 5 x
6. a. Has left and right limits
–2
b. Has a limit x
c. Continuous (Note that the x-value 5 is not at –2

the discontinuity.)
7. a. Has left and right limits 19. Answers may vary. 20. Answers may vary.
b. Has a limit f (x ) f(x)

c. Discontinuous. f (1) ≠ limit 6

8. a. Has left and right limits 4 5

b. Has no limit
c. Discontinuous. No limit x x
1
9. a. Has left and right limits 3

b. Has a limit
c. Discontinuous. No f (c) 21. Discontinuous at x = −3
10. a. Has left and right limits 22. Discontinuous at x = 11
b. Has no limit 23. Discontinuous at x = π/2 + π n, where n is an
integer
c. Discontinuous. No limit, no f (c)
24. Nowhere discontinuous
11. Answers may vary. 12. Answers may vary.
25.
f (x ) f (x )
f (x )

3
2
1
x

x x 2

3 4
Discontinuous because lim f ( x ) = 2 and f (2) = 3
x →2

13. Answers may vary. 14. Answers may vary. 26.


g (x )
f (x ) f (x )

2
1
x
f (–2)
2
x
x
–2
5 Discontinuous because g(x) has no limit as x
approaches 2

16 Problem Set 2-4 Calculus Solutions Manual


© 2005 Key Curriculum Press
27. 33. a.
s (x ) d (x )

3
x
2
x
2
Discontinuous because s(x) has no limit as x
approaches 2 from the left (no real function b. lim− d ( x ) = 3, lim+ d ( x ) = 3. Limit = 3.
values to the left of x = 2) x →2 x →2

28. Continuous.
p (x ) 34. a.
h ( x)

3
1 2

x x
2 1

Discontinuous because p(x) has no limit as x


approaches 2
29. b. lim− h( x ) = 3, lim+ h( x ) = 2. No limit.
x →1 x →1
h(x ) Not continuous.
35. a.
1 m (x )
9
x
2 7

Discontinuous because there is no value of h(2) x


30. 2
f(x)

b. lim− m( x ) = 9, lim+ m( x ) = 7. No limit.


3 x →2 x →2
Not continuous.
x
2 36. a.
q (x)

Discontinuous because f (x) has no limit as x


approaches 2
2
31.
x
lim− f ( x ) lim+ f ( x ) lim f ( x )
c f (c) x →c x →c x →c Continuous? –1
1 4 2 2 2 removable
b. lim− q( x ) = 2, lim+ q( x ) = 2. Limit = 2.
2 1 1 1 1 continuous x →−1 x →−1
4 5 5 2 none step Continuous.
5 none none none none infinite 37. 9 – 22 = 2k
∴ k = 2.5
32. g (x )

lim− f ( x ) lim+ f ( x ) lim f ( x )


c f (c) x →c x →c x →c Continuous?
5
1 3 2 3 none step
2 1 4 4 4 removable x
3 5 5 5 5 continuous 2
5 5 5 none none infinite

Calculus Solutions Manual Problem Set 2-4 17


© 2005 Key Curriculum Press
38. 0.4(1) + 1 = k(1) + 2 43. Let T(θ ) = the number of seconds it takes to
∴ k = −0.6 cross.
f (x ) 24, if θ = 90°
T(θ ) =  40
, if 0° < θ < 90° or 90° < θ < 180°
1.4
 sin θ
θ
x
1

40
39. (32)k = 3k − 3 θ
∴ k = −1/2. 90

u (x )
44. a. f (x )
1 x
3
4

40. −k + 5 = (−1) k 2 1

∴ k = 5/2
b. f (x) seems to approach 4 as x approaches 1.
v (x )
c. f (1.0000001) = 1.0000001 + 3 + 10− 13 ≈
5 4.0000001, which is close to 4.
d. There is a vertical asymptote at x = 0. You
x
–1
must get x much closer to 1 than x =
1.0000001 for the discontinuity to show up.
45. For any value of c, P(c) is determined by addition
41. a. b − 1 = a(1 − 2)2 ⇒ b − 1 = a and multiplication. Because the set of real
b. a = −1 ⇒ b = 0. Continuous at x = 1. numbers is closed under multiplication and
addition, P(c) will be a unique, real number for
f (x )
any real value x = c. P(c) is the limit of P(x) as
x approaches c by the properties of the limit of a
1 1 x product of functions (for powers of x), the limit
of a constant times a function (for multiplication
a = –1, b = 0 by the coefficients), and the limit of a sum (for
the individual terms). Therefore, P is continuous
c. For example, a = 1 ⇒ b = 2. Continuous for all values of x.
at x = 1. 46. a. lim |sgn x| = 1 but f (0) = 0
x→0
f (x ) lim f ( x ) ≠ f (0), so discontinuous
x→0

b.
1 x g(x )
1
3
e.g., a = 1, b = 2
x
2

42. lim− f ( x ) = lim− k 2 − x 2 = k 2 − 4


x →2 x →2
lim+ f ( x ) = lim+ 1.5kx = 1.5k (2) = 3k
x →2 x →2 c.
For f (x) to be continuous at x = 2, these two h(x )
limits must be equal, so find k such that
k 2 − 4 = 3k
k 2 − 3k − 4 = 0
1
(k − 4)(k + 1) = 0 x

so k = 4 and k = −1 are the two values of k that –1 1

will make f (x) continuous at x = 2.

18 Problem Set 2-4 Calculus Solutions Manual


© 2005 Key Curriculum Press
d. For x > 0, a(x) = x/x = 1 = sgn x. b. lim+ f ( x ) = ∞, lim− f ( x ) = – ∞,
x →3 x →3
For x < 0, a(x) = (−x)/x = −1 = sgn x. lim f ( x ), none, lim f ( x ) = 2,
For x = 0, a(0) is not defined. x →3 x →∞

∴ a(x) = sgn x for all x ≠ 0, Q.E.D. lim f ( x ) = 2


x →−∞
e. 1
f (x ) c. 2 + = 100
x −3
2
1
x = 98
–2
π x −3
1
–2
x–3=
98
1
x = 3 = 3.0102 K
Problem Set 2-5 98
Q1. No limit Q2. 3 x f (x)
Q3. 4 Q4. 3
3.01 102
Q5. 2 Q6. No
3.001 1002
Q7. No Q8. Yes
3.0001 10002
Q9. No Q10. Yes
1. • lim f ( x ) = ∞ • lim− f ( x ) = –4 All of these f (x) values are greater than 100.
x →−∞ x →−3 lim+ f ( x ) = ∞ means that f (x) can be kept
x →3
• lim f ( x ) = 3 • lim f ( x ) = – ∞
x →−3+ x →1 arbitrarily far from zero just by keeping x
• lim f ( x ) = 1 • lim− f ( x ) = ∞ close enough to 3 on the positive side.
x →2 x →3 There is a vertical asymptote at x = 3.
• lim+ f ( x ) = 2 • lim f ( x ) does not 1
x →3 x →∞
d. 2 + = 2.001
exist. x−3
lim g( x ) = 2 lim− g( x ) = 4 1
2. •
x →−∞
• = 0.001
x →−2 x−3
• lim+ g( x ) = –3 • lim− g( x ) = ∞ x − 3 = 1000
x →−2 x →1

• lim g( x ) = 3 • lim− g( x ) = 4 x = 1003


x →2 x →3

• lim g( x ) = −2 x f (x)
x →∞
3. Answers may vary. 4. Answers may vary. 1004 2.00099…
f (x ) f (x ) 1005 2.00099…
1006 2.00099…
x All of these f (x) values are within 0.001 unit
2 of 2. lim f ( x ) = 2 means that you can keep
x →∞
x
2
f (x) arbitrarily close to 2 by making the value
of x arbitrarily large. y = 2 is a horizontal
5. Answers may vary. 6. Answers may vary. asymptote.
f (x ) f (x ) 8. a.
7
g (x )

x x 1
x
–5 π/2

7. a. f (x )

b. lim g( x ) = ∞, lim + g( x ) = −∞
2 x →π / 2 − x →π / 2
x The limit is infinite because |g(x)| can be kept
3
arbitrarily far from zero. You can’t say
lim g( x ) = ∞ because the left and right
x →π / 2

Calculus Solutions Manual Problem Set 2-5 19


© 2005 Key Curriculum Press
limits are not the same (one is positive and the e. The graph suggests that lim r ( x ) = 3.
x→0
other is negative).
(The exact value is e, 2.7182… .)
c. sec x = −1000
10. a. h( x ) = (1 + 1/ x ) x
∴ cos x = −0.001
x = arccos (−0.001) = 1.57179… 3
h(x )

x g(x) 2

1.5717 –1106.5… 1
x
1.5716 –1244.2…
10
1.5715 –1421.1…
All of these f (x) values are less than −1000. b. There is a compromise number (bigger than
lim + g( x ) = −∞ means that arbitrarily far 1, but finite) that wins. (The exact limit is e.)
x →π / 2
g(x) can be kept arbitrarily far from zero in the 11. The limit is infinite. y is unbounded as x
negative direction by keeping x close enough approaches infinity. If there were a number E
to π2 on the positive side. such that log x < E for all x > 0, then you could
The line x = π2 is a vertical asymptote. let x = 102E so that log x = log 102E = 2E, which
9. a. is greater than E, which was assumed to be an
r (x ) upper bound.
3
12. “Wanda, here’s what happens to a fraction when
2 the denominator gets close to zero: 01.1 = 10,
0.0001 = 10, 000, 0.00001 = 100, 000. The answers
1 1

x just keep getting bigger and bigger. When the


5
denominators get bigger and bigger, the fraction
gets closer and closer to zero, like this:
10 = 0.1, 100 = 0.01, 1000 = 0.001.”
1 1 1
b. lim r ( x ) = 2 because (sin x)/x approaches zero.
x →∞

sin (28) 13. a. The definite integral is the product of the


c. r(28) = 2 + = 2.00967K , which is independent and dependent variables. Because
28
distance = (rate)(time), the integral represents
within 0.01 unit of 2.
sin (32) distance in this case.
r(32) = 2 + = 2.01723K , which is
32 b. T9 = 17.8060052…
more than 0.01 unit away from 2. T45 = 17.9819616…
y
T90 = 17.9935649…
r
T450 = 17.9994175…
2.01 c. The exact answer is 18. It is a limit because
1.99
the sums can be made as close to it as you
x like, just by making the number of trapezoids
28 32
large enough (and thus keeping their widths
close to zero). The sums are smaller than the
Keeping r(x) within 0.01 unit of 2 means you
integral because each trapezoid is inscribed
want to keep sinx x < 0.01, or |sin x| < under the graph and thus leaves out a part of
0.01 |x|. You are looking for a large value of its respective strip of the region.
x, so you know x will be positive, so you
want |sin x| < 0.01x. You can’t get rid of the d. Tn is 0.01 unit from 18 when it equals 17.99.
absolute value symbol on the sine because From part b, this occurs between n = 45 and n
sine will keep alternating as x gets larger. = 90. By experimentation,
You know |sin x| ≤ 1 for all values T66 = 17.9897900… and T67 = 17.9900158… .
of x, so you need to make 0.01x > 1, or Therefore, the approximation is within 0.01
x > 100. So D = 100. unit of 18 for any value of n ≥ 67.
An alternative solution is to plot the graph of
d. The line y = 2 is an asymptote. Even though the difference between 18 and Tn as a function
r (x) oscillates back and forth across this line, of the number of increments, n, or to do a
the limit of r(x) is 2 as x approaches infinity, regression analysis to find an equation. The
satisfying the definition of asymptote. best-fitting elementary function is an inverse
power variation function, y = (5.01004…)

20 Problem Set 2-5 Calculus Solutions Manual


© 2005 Key Curriculum Press
(x− 1.48482…). The graph of this function and three Q7. 1 Q8. +
of the four data points are shown here. Use Q9. Indeterminate Q10. C
TRACE or the solver feature of your grapher to
1. IVT applies on [1, 4] because f is a polynomial
find n ≈ 67.
function, and polynomial functions are
y continuous for all x.
f (1) = 18, f (4) = 3
67 ∴ There is a value x = c in (1, 4) for which
0.1
f (c) = 8.
0.01 n Using the intersect or solver feature,
90 c = 1.4349… , which is between 1 and 4.
f (x )

f (1)
14. a. Work = force × distance. Because a definite
integral measures the y-variable times the
8
x-variable, it represents work in this case.
f (4) x
b. By the trapezoidal rule, T10 = 24.147775… 1 c 4
and T100 = 24.004889… . The units are
foot-pounds. 2. IVT applies on [0, 6] because f is a polynomial
function, and polynomial functions are
c. The integer is 24. continuous for all x.
d. By experimentation, T289 = 24.001003… and f (0) = −8, f (6) = −0.224
T290 = 24.000998… . ∴ There is a value x = c in (0, 6) for which
∴ D = 290 f (c) = −1.
15. Length = 100 sec x = 100/cos x Using the intersect or solver feature,
Length > 1000 ⇒ 100/cos x > 1000 c = 5.8751… , which is between 0 and 6.
cos x < 0.1 (because cos x is positive) f (x )
x > cos–1 0.1 (because cos is decreasing) 0 c6 x
–0.224
x > 1.4706289…
π/2 − 1.4706289… = 0.100167…
x must be within 0.100167… radian of π/2.
The limit is (positive) infinity. –8

16. a. f (2) = 5 · 2 · 0 · (1/0), which has the


form 0 · ∞. 3. a. For 1 ≤ y < 2 or for 5 < y ≤ 8, the conclusion
g(2) = 5 · 2 · 0 · (1/0)2, which has the would be true. But for 2 ≤ y ≤ 5, it would be
form 0 · ∞. false because there are no values of x in [1, 5]
h(2) = 5 · 2 · 02 · (1/0), which has the that give these values for f (x).
form 0 · ∞. b. The conclusion of the theorem is true because
1 every number y in [4, 6] is a value of g(x) for
b. f ( x ) = 5 x ( x − 2) ⋅ = 5 x, x ≠ 2
x–2 some value of x in [1, 5].
∴ lim f ( x ) = 10
x →2 4. a. f (2) = 4, f (3) = 8, f (0.5) = 2 = 1.414 K ,
1 5x f ( 5) = 8
g( x ) = 5 x ( x − 2) ⋅ 2 = ,x≠2
( x – 2) x–2
b. f is continuous at x = 3 because it has a limit
∴ lim g( x ) is infinite.
x →2 and a function value and they both equal 8.
1
h( x ) = 5 x ( x − 2 ) 2 ⋅ = 5 x ( x – 2), x ≠ 2 c. f is continuous nowhere else. Because the
x–2 sets of rational and irrational numbers are
∴ lim h( x ) = 0
x →2 dense, there is a rational number between any
c. The indeterminate form 0 · ∞ could approach two irrational numbers, and vice versa. So
zero, infinity, or some finite number. there is no limit of f (x) as x approaches any
number other than 3.
Problem Set 2-6 d. The conclusion is not true for all values of y
between 1 and 4. For instance, if y = 3, then
Q1. 53 Q2. 53
c would have to equal log2 3. But log2 3 is
Q3. Undefined Q4. 5 irrational, so f (c) = 8, which is not between
Q5. Undefined Q6. Does not exist 1 and 4.

Calculus Solutions Manual Problem Set 2-6 21


© 2005 Key Curriculum Press
5. Let f (x) = x2. f is a polynomial function, so it is log 3
continuous and thus the intermediate value • c = log 2 3 = = 1.5849...
log 2
theorem applies. f (1) = 1 and f (2) = 4, so there
is a number c between 1 and 2 such that f (c) = 3. • Using the solver feature, c = 1.5849...
By the definition of square root, c = 3 , Q .E .D . • Using the intersect feature, c = 1.5849...
6. Prove that if f is continuous, and if f (a) is 13. This means that a function graph has a high
positive and f (b) is negative, then there is at least point and a low point on any interval in which
one zero of f (x) between x = a and x = b. the function is continuous.
f (x )
Proof:
f is continuous, so the intermediate value
theorem applies. f (a) is positive and f (b) is
negative, so there is a number x = c between a x
and b for which f (c) = 0. Therefore, f has at least a c1 c2 b
one zero between x = a and x = b, Q.E.D.
7. The intermediate value theorem is called an If the function is not continuous, there may be a
existence theorem because it tells you that a point missing where the maximum or minimum
number such as 3 exists. It does not tell you would have been.
how to calculate that number. f (x)

8. Telephone your sweetheart’s house. An answer to


the call tells you the “existence” of the
sweetheart at home. The call doesn’t tell such
things as how to get there, and so on. Also, x

getting no answer does not necessarily mean that a b

your sweetheart is out.


Another possibility would be a graph with a
9. Let f (t) = Jesse’s speed − Kay’s speed. f (1) =
vertical asymptote somewhere between a and b.
20 − 15 = 5, which is positive. f (3) = 17 − 19 =
−2, which is negative. The speeds are assumed to 14. Prove that if f is continuous on [a, b], the image
be continuous (because of laws of physics), so f of [a, b] under f is all real numbers between the
is also continuous and the intermediate value minimum and maximum values of f (x),
theorem applies. inclusive.
So there is a value of t between 1 and 3 for Proof:
which f (t) = 0, meaning that Jesse and Kay are
going at exactly the same speed at that time. By the extreme value theorem, there are numbers
The existence of the time tells you neither what x1 and x2 in [a, b] such that f (x1) and f (x2) are the
that time is nor what the speed is. An existence minimum and maximum values of f (x) on [a, b].
theorem, such as the intermediate value theorem, Because x1 and x2 are in [a, b], f is continuous on
does not tell these things. the interval whose endpoints are x1 and x2. Thus,
the intermediate value theorem applies on the
10. Let f (x) = number of dollars for x-ounce letter.
latter interval. Thus, for any number y between
f does not meet the hypothesis of the IVT on the
f (x1) and f (x2), there is a number x = c between
interval [1, 9] because there is a step
x1 and x2 for which f (c) = y, implying that the
discontinuity at each integer value of x. There is
image of [a, b] under f is all real numbers
no value of c for which f (c) = 2 because f (x)
between the minimum and maximum values of
jumps from 1.98 to 2.21 at x = 8.
f (x), inclusive, Q.E.D.
11. You must assume that the cosine is function
continuous. Techniques:
• c = cos− 1 0.6 = 0.9272… Problem Set 2-7
• Using the solver feature, c = 0.9272... Review Problems
• Using the intersect feature, c = 0.9272... R0. Answers will vary.
12. You must assume that 2x is continuous. 36 − 51 + 15 0
R1. a. f (3) = =
f (0) = 20 = 1, because any positive number to the 3−3 0
0 power equals 1. Indeterminate form

22 Problem Set 2-7 Calculus Solutions Manual


© 2005 Key Curriculum Press
b. f ( x ) = 4 x – 5, x ≠ 3 You can cancel the (x − 3) because the
y definition of limit says “but not equal to 3.”
9
• lim g( x )
x →3
x
–9 9 = lim x 2 + lim (–10 x ) + lim 2
x →3 x →3 x →3
Limit of a sum
–9 = lim x ⋅ lim x – 10 lim x + 2
x →3 x →3 x →3

At x = 3 there is a removable discontinuity. Limit of a product,


limit of a constant
c. For 0.01, keep x within 0.0025 unit of 3. For
times a function,
0.0001, keep x within 0.000025 unit of 3. To
limit of a constant
keep f (x) within ε unit of 7, keep x within
= 3 · 3 − 10(3) + 2 Limit of x
4 ε unit of 3.
1
= −19, which agrees with the graph.
R2. a. L = lim f ( x ) if and only if for any number
x →c
c. • f (x) = 2x,
ε > 0, no matter how small, x 2 − 8 x + 15 ( x − 3)( x − 5)
there is a number δ > 0 such that if x is g( x ) = =
3− x 3− x
within δ units of c, but x ≠ c, then f (x) is
= −x + 5, x ≠ 3
within ε units of L. lim f ( x ) = 8, lim g( x ) = 2
x →3 x →3
b. lim f ( x ) = 2
x →1
lim f ( x ) does not exist. • p(x) = f (x) · g(x)
x →2
lim p( x ) = 8 ⋅ 2 = 16
lim f ( x ) = 4 x →3
x →3
lim f ( x ) does not exist.
x →4 x p(x)
lim f ( x ) = 3
x →5 2.997 15.9907…
c. lim f ( x ) = 3 2.998 15.9938…
x →2
Maximum δ: 0.6 or 0.7 2.999 15.9969…
d. The left side of x = 2 is the more restrictive. 3 undefined
Let 2 + x – 1 = 3 − 0.4 = 2.6. 3.001 16.0030…
∴ x = 1 + 0.62 = 1.36
3.002 16.0061…
∴ maximum value of δ is 2 − 1.36 = 0.64.
e. Let f ( x ) = 3 − ε . 3.003 16.0092…
2 + x −1 = 3 − ε All these p(x) values are close to 16.
x = (1 − ε )2 + 1
f ( x)
Let δ = 2 − [(1 − ε)2 + 1] = 1 − (1 − ε)2, • r( x ) =
which is positive for all positive ε < 1. If g( x )
ε ≥ 1, simply take δ = 1. Then δ will be 8
lim r ( x ) = =4
positive for all ε > 0. x →3 2
R3. a. See the limit property statements in the text. 20
y

b. •
g (x ) 15
20 f
10
r
x
3 5
g
x
–19
3

• The limit of a quotient property does not


apply because the limit of the denominator d. For 5 to 5.1 s: average velocity = −15.5 m/s.
is zero. f (t ) – f (5)
Average velocity = =
( x − 3)( x 2 − 10 x + 2) t–5
• g( x ) = 35t – 5t 2 – 50 –5(t – 2)(t – 5)
x−3 = =
g(x) = x 2 − 10x + 2, x ≠ 3 t–5 t–5

Calculus Solutions Manual Problem Set 2-7 23


© 2005 Key Curriculum Press
−5(t − 2), for t ≠ 5. Instantaneous velocity = d. f (x )

limit = −5(5 − 2) = −15 m/s.


The rate is negative, so the distance above the
starting point is getting smaller, which 4
means the rock is going down.
2
Instantaneous velocity is a derivative.
x
R4. a. • f is continuous at x = c if and only if 2

1. f (c) exists The left limit is 4 and the right limit is 2, so


2. lim f ( x ) exists
x →c
f is discontinuous at x = 2, Q.E.D.
Let 22 = 22 − 6(2) + k.
3. lim f ( x ) = f (c)
x →c ∴ k = 12
• f is continuous on [a, b] if and only if f is R5. a. lim f ( x ) = ∞ means that f (x) can be kept
x→4
continuous at every point in (a, b), and
arbitrarily far from 0 on the positive side just
lim+ f ( x ) = f ( a) and lim− f ( x ) = f (b).
x→a x→b by keeping x close enough to 4, but not equal
b. to 4.
lim− f ( x ) lim+ f ( x ) lim f ( x ) lim f ( x ) = 5 means that f (x) can be made to
c f (c) x →c x →c x →c Continuous? x →∞
stay arbitrarily close to 5 just by keeping x
1 none none none none infinite
large enough in the positive direction.
2 1 3 3 3 removable
b. • lim f ( x ) does not exist.
3 5 2 5 none step x →−∞

4 3 3 3 3 continuous • lim f ( x ) = 1
x →−2
5 1 1 1 1 continuous • lim− f ( x ) = ∞
x →2
c. i. ii. • lim+ f ( x ) = −∞
y y
x →2

• lim f ( x ) = 2
x →∞

c. f (x) = 6 − 2− x
lim f ( x ) = 6
x →∞
x x
1 2
f (x) = 5.999 = 6 − 2− x
2− x = 0.001
iii. iv. log 0.001
y y x=−
log 2
x = 9.965...

x f (x)
x x
3 4 10 5.999023…
20 5.999999046…
v. vi.
y y
30 5.99999999907…
f (6) All of these f (x) values are within 0.001 of 6.
L d. g(x) = x− 2
lim g( x ) = ∞
x→0
x x
5 6 g(x) = 106 = x− 2
x2 = 10− 6
vii. x = 10− 3
y
5 x g(x)
x 0.0009 1.2345… ⋅ 106
1
–2 0.0005 4,000,000
0.0001 1 ⋅ 108

24 Problem Set 2-7 Calculus Solutions Manual


© 2005 Key Curriculum Press
All of these g(x) values are larger than C2. f (1) = 12 − 6 ⋅ 1 + 9 = 4
1,000,000. As x → 1 from the left, f (x) → 12 + 3 = 4.
e. v(t) = 40 + 6 t As x → 1 from the right, f (x) → 12 − 6 + 9 = 4.
∴ lim f ( x ) = 4 = f (1)
n Trapezoidal Rule x →1
∴ f is continuous at x = 4, Q.E.D.
50 467.9074… For the derivative, from the left side,
100 467.9669… f ( x ) – f (1) x 2 + 3 – 4 ( x – 1)( x + 1)
= = =
200 467.9882… x –1 x –1 x –1
x + 1, x ≠ 1
400 467.9958… ∴ lim− f ′( x ) = 1 + 1 = 2
x →1
The limit of these sums seems to be 468. For the derivative, from the right side,
By exploration, f ( x ) – f (1) x 2 – 6 x + 9 – 4 ( x – 1)( x – 5)
T222 = 467.98995… = = =
x –1 x –1 x –1
T223 = 467.99002… x − 5, x ≠ 1
∴ D = 223 ∴ lim+ f ′( x ) = 1 – 5 = −4
x →1
R6. a. See the text statement of the intermediate So f is continuous at x = 1, but does not have a
value theorem. value for the derivative there because the rate of
The basis is the completeness axiom. change jumps abruptly from 2 to −4 at x = 1. In
See the text statement of the extreme value general, if a function has a cusp at a point, then
theorem. the derivative does not exist, but the function is
The word is corollary. still continuous.
b. f (x) = −x 3 + 5x 2 − 10x + 20 C3. The graph is a y = x2 parabola with a step
f (3) = 8, f (4) = −4 discontinuity at x = 1. (Use the “rise-run”
So f (x) = 0 for some x between 3 and 4 by property. Start at the vertex. Then run 1, rise 1;
the intermediate value theorem. run 1, rise 3; run 1, rise 5; . . . . Ignore the
The property is continuity. discontinuity at first.) To create the
The value of x is approximately 3.7553. discontinuity, use the signum function with
c. f (x ) argument (x − 1). Because there is no value for
f (1), the absolute value form of the signum
function can be used.
3 | x –1|
x
y = x2 + 2 −
x –1
–4
C4. The quantity | f ( x ) – L | is the distance between
f (x) and L. If this distance is less than ε, then
f (−6) = 1 and f (−2) = 5 by tracing on the f (x) is within ε units of L. The quantity |x − c|
graph or by simplifying the fraction to get is the distance between x and c. The right part of
f (x) = x + 7, then substituting. You will not the inequality, |x – c| < δ, says that x is within δ
always get a value of x if y is between 1 and units of c. The left part, 0 < |x − c|, says that x
5. If you pick y = 3, there is no value of x. does not equal c. Thus, this definition of limit is
This fact does not contradict the intermediate equivalent to the other definition.
value theorem. Function f does not meet the
continuity hypothesis of the theorem.
Chapter Test
Concept Problems
C1. y T1. f is continuous at x = c if and only if
1. f (c) exists
h
7 2. lim f ( x ) exists
x →c
f
3. lim f ( x ) = f (c)
g x →c
x f is continuous on [a, b] if and only if f is
4 continuous at all points in (a, b), and
lim+ f ( x ) = f ( a) and lim− f ( x ) = f (b) .
Conjecture: lim f ( x ) = 7 x→a x→b
x→4

Calculus Solutions Manual Problem Set 2-7 25


© 2005 Key Curriculum Press
T2. a. • lim f ( x ) = 3 • lim f ( x ) = 4 = lim x 2 + lim ( −5 x ) + lim 8
x →2 − x →2 + x →3 x →3 x →3
• lim f ( x ) does not exist. • lim− f ( x ) = 2 Limit of a sum
x →2 x →6 = lim x ⋅ lim x + ( −5) ⋅ lim x + 8
x →3 x →3 x →3
• lim f ( x ) = 2 • lim f ( x ) = 2
x →6 + x →6 Limit of a product,
b. f is continuous on [2, 6] because it is limit of a constant
continuous for all values in (2, 6) and times a function,
lim+ f ( x ) = f (2) and lim− f ( x ) = f (6) . limit of a constant
x →2 x →6
= 3 · 3 + (−5) · 3 + 8 Limit of x
T3. See the text statement of the quotient property. = 2, Q .E .D .
T4. a. Left: −4; right: −4
x 2 , x≤2
b. Limit: −4 T11. If k = 1, f ( x ) = 
 x + 1, x > 2
c. Discontinuous
lim− f ( x ) = 4, lim+ f ( x ) = 3
T5. a. Left: none; right: none x →2 x →2
∴ f is discontinuous at x = 3.
b. Limit: none
c. Discontinuous f(x)
5

T6. a. Left: 6; right: 6


b. Limit: 6
c. Continuous
x
T7. a. Left: −2; right: 3 2

b. Limit: none
c. Discontinuous T12. lim− f ( x ) = k ⋅ 2 2
x →2
T8. lim f ( x ) = 2 + k
x →2 +
y
∴ 4k = 2 + k
4
k = 2/3
T13. See graph in T11.
x
4 T14. a. lim T ( x ) = 20
x →∞
From the graph, it appears that if x > 63 ft,
then T(x) is within 1° of the limit.
T9. a. b. The graph of T has a horizontal asymptote at
f (x ) g(x ) T = 20.
b. T = 20 + 8(0.97x) cos 0.5x. The amplitude of
1 the cosine factor is 8(0.97 x ). Make this
x amplitude < 0.1.
x 8(0.97c) = 0.1
0.97c = 0.0125
log 0.0125
c=
c. d. log 0.97
h(x) s (x )
c = 143.8654…
1 ∴ T is within 0.1 unit of 20 whenever
1
x x x > 143.8654… .
–1
1 c. The time of day would be mid-afternoon,
when the temperature of the surface is
highest.
T15. a. Use either TRACE or TABLE to show:
(0 2 − 5 ⋅ 0 + 8)(0 − 3) 0
T10. a. f (3) = = , d(0) = 0, d(10) = 6, d(20) = 14, d(30) = 24,
0−3 0 d(40) = 36, and d(50) = 50.
an indeterminate form d (20.1) − d (20)
b. Average rate = =
b. lim f ( x ) = lim ( x 2 − 5 x + 8), x ≠ 3 20.1 − 20
x →3 x →3
14.0901 – 14
Definition of limit = 0.901 cm/day
“x ≠ c” 20.1 – 20

26 Problem Set 2-7 Calculus Solutions Manual


© 2005 Key Curriculum Press
d (t ) − d (20) Make 4k = 10 − 2k ⇒ k = 5/3. There is a cusp
c. Average rate = = at x = 2.
t − 20
0.01t 2 + 0.5t – 14 0.01(t + 70)(t – 20) f(x)
= = 10
t – 20 t – 20
0.01t + 0.7, t ≠ 20. The limit as t approaches
20 is 0.01(20) + 0.7, which equals
0.9 cm/day. This instantaneous rate is called x
the derivative. 2

d. The glacier seems to be speeding up because


each 10-day period it moved farther than it had T18. h(x) = x3. h(1) = 1 and h(2) = 8, so 7 is between
in the preceding 10-day period. h(1) and h(2). The intermediate value theorem
T16. c(0) = p(0) = 10, so each has the same speed at allows you to conclude that there is a real
t = 0. lim c(t ) = 16. lim p(t ) = ∞. Surprise for number between 1 and 8 equal to the cube root
t →∞ t →∞
Phoebe! of 7.
kx 2 , T19. Answers will vary.
if x ≤ 2
T17. f ( x ) = 
10 – kx, if x > 2
lim f ( x ) = k ⋅ 2 2 = 4 k
x →2 −
lim f ( x ) = 10 − 2 k
x →2 +

Calculus Solutions Manual Problem Set 2-7 27


© 2005 Key Curriculum Press
Chapter 3—Derivatives, Antiderivatives,
and Indefinite Integrals

Problem Set 3-1 Q4.


y
1. The graph is correct.
f (5.1) – f (5) 3.21 – 3
2. Average rate = = = 1
0.1 0.1 x
2.1 km/min
3. Q5. 9x 2 − 42x + 49
y
Q6. “−” sign
Q7. Q8.
y y
2

x
2
5 x
x
3 5

4. The graph of r has a removable discontinuity


at x = 5. Q9. Newton and Leibniz
f (5) − f (5) 0 Q10. D
r (5) = =
5−5 0 1. See the text for the definition of derivative.
f ( x ) − f (5) x 2 − 8 x + 18 − 3 2. Physical: Instantaneous rate of change of the
5. r ( x ) = =
x−5 x−5 dependent variable with respect to the
( x − 5)( x − 3) independent variable
= = x − 3, x ≠ 5 Graphical: Slope of the tangent line to the graph
x−5
of the function at that point
f ′(5) = lim r ( x )
x →5 0.6 x 2 – 5.4
( x − 5)( x − 3) 3. a. f ′(3) = lim
= lim = lim ( x − 3) = 2 x →3 x–3
x →5 x−5 x →5
0.6( x – 3)( x + 3)
The derivative is the velocity of the spaceship, = lim = 3.6
x →3 x–3
in km/min.
b. Graph of the difference quotient m(x)
6. Find the equation of the line through (5, f (5)),
m(x )
or (5, 3), with slope 2.
y − 3 = 2(x − 5) ⇒ y = 2x − 7 3.6

3 x
3

x c., d. Tangent line: y = 3.6x − 5.4


5
f (x )

This line is tangent to the graph of f (x) at (5, 3).


7. As you zoom in, the line and the graph appear to
be the same.
1 x
8. Answers will vary. 3

Problem Set 3-2 – 0.2 x 2 + 7.2


4. a. f ( x ) = – 0.2 x 2 , f ′(6) = lim
x →6 x–6
Q1. Instantaneous rate of change
Q2. x + 9 –0.2( x – 6)( x + 6)
Q3. 18 = lim = −2.4
x →6 x–6

28 Problem Set 3-2 Calculus Solutions Manual


© 2005 Key Curriculum Press
b. 14. The derivative of a constant function is zero.
m (x )
Constant functions are horizontal and don’t
change! The tangent line coincides with the
x graph.
15. a. Find f ′ ( 1) = 2, then plot a line through point
6

–2.4
(1, f (1)) using f ′ ( 1) as the slope. The line is
y = 2x − 1.
b. Near the point (1, 1), the tangent line and the
c., d. Tangent line: y = −2.4x + 7.2 curve appear nearly the same.
c. The curve appears to get closer and closer to
f (x)
x the line.
6 d. Near point (1, 1) the curve looks linear.
–7.2
e. If a graph has local linearity, the graph near
that point looks like the tangent line.
Therefore, the derivative at that point could be
said to equal the slope of the graph at that
point.
x 2 + 5x + 1 + 5 16. a. f ( x) = x 2 + 0.1 (x − 1)2/3
5. f ′(–2) = lim
x →−2 x+2 f ( 1) = 12 + 0.1(1 − 1)2/3 = 1 + 0 = 1, Q .E .D .
( x + 2)( x + 3) The graph appears to be locally linear at
= lim =1
x →−2 x+2 (1, 1), because it looks smooth there.
x 2 + 6 x – 2 + 10 b. Zoom in by a factor of 10,000.
6. f ′(– 4) = lim
x →−4 x+4
( x + 4)( x + 2)
= lim = –2
x →−4 x+4 1

x3 – 4x2 + x + 8 – 6
7. f ′(1) = lim
x →1 x –1
1
( x – 1)( x 2 – 3 x – 2)
= lim = –4
x →1 x –1
c. The graph has a cusp at x = 1. It changes
x3 – x2 – 4x + 6 – 8 direction abruptly, not smoothly.
8. f ′(–1) = lim
x →−1 x +1 d. If you draw a secant line through (1, 1) from a
( x + 1)( x 2 – 2 x – 2) point just to the left of x = 1, it has a large
= lim =1 negative slope. If you draw one from a point
x →−1 x +1
just to the right, it has a large positive slope.
–0.7 x + 2 + 0.1
9. f ′(3) = lim In both cases, the secant line becomes vertical
x →3 x–3 as x approaches 1 and a vertical line has
–0.7( x – 3) infinite slope. So there is no real number
= lim = −0.7
x →3 x–3 equal to the derivative.
1.3 x – 3 – 2.2 17. a.
10. f ′( 4) = lim
x→4 x–4 f (x )
7
1.3( x – 4)
= lim = 1.3 5
x→4 x–4
5–5
11. f ′(–1) = lim =0
x →−1 x + 1 x

–2 + 2 3
12. f ′(3) = lim =0
x →3 x – 3

13. The derivative of a linear function equals the b. First simplify the equation.
slope. The tangent line coincides with the graph  x + 2, if x ≠ 3
f ( x) = 
of a linear function. 7, if x = 3

Calculus Solutions Manual Problem Set 3-2 29


© 2005 Key Curriculum Press
The difference quotient is = lim 0.25( x – 7) = –1, Q .E.D .
x →3
( x + 2) – 7 x – 5
m( x ) = = The tangent line on the graph has slope −1.
x–3 x–3
b.
m (x )
f (x)
5
Draw secant lines
x from here.

c.
x f (x) 2

2.997 667.66…
x
2.998 1001 3

2.999 2001
3.000 undefined As the x-distance between the point and 3
decreases, the secant lines (solid) approach the
3.001 −1999
tangent line (dashed).
3.002 −999
c. The same thing happens with secant lines
3.003 −665.66… from the left of x = 3. See the graph in part b.
The difference quotients are all large positive d.
numbers on the left side of 3. On the right g ( x)
side, they are large negative numbers. For a 4
derivative to exist, the difference quotient Draw secant lines
from here.
must approach the same number as x gets
closer to 3.
18. a.
s (x )
x
3
2

x
1

| sin ( x – 1) | e. A derivative is a limit. Because the left and


b. m( x ) =
x –1 right limits are unequal, there is no derivative
at x = 3.
π
m(x )

1
–6 cos x
6
x f. m( x ) = . By table,
x–3
1

x m(x)
2.9 3.1401…
c. As x approaches 1 from the left, m(x) 2.99 3.1415…
approaches −1. As x approaches 1 from the 3 undefined
right, m(x) approaches 1. Because the left and 3.01 −3.1415…
right limits are unequal, there is no derivative
at x = 1. 3.1 −3.1401…
0.25 x 2 – 2.5 x + 7.25 – 2 Conjecture: The numbers are π and −π.
19. a. f ′( x ) = lim
x →3 x–3 20. From Problem 19, parts b and c, the tangent line
0.25( x – 3)( x – 7) is the limit of the secant lines as x approaches c.
= lim Because the slope of the secant line is the average
x →3 x–3

30 Problem Set 3-2 Calculus Solutions Manual


© 2005 Key Curriculum Press
rate of change of f (x) for the interval from x to c 2.
(or from c to x) and the derivative, f ′(c), is the 15
y
g'
limit of this average rate, the slope of the tangent
line equals f ′(c). x
3

Problem Set 3-3


Q1. 3
Q2. The graph does not have the high and low
y
points that are typical of a cubic function. As x
5 increases, the graph starts to roll over and form
a high point, but it starts going back up again
x
–2
before that happens. This behavior is revealed by
the fact that the derivative is positive everywhere.
Q3. Between x = 0 and x = 1, the derivative reaches
y a low point, indicating that the slope is a
x minimum, but the slope is still positive and
the graph of g is still going up.
3. a.
y
200
h
Q4. 20%
Q5. 3x 2 − 2x − 8 h´
Q6. 25x2 − 70x + 49 x
–2 1 2.5
Q7. log 6
Q8.
b. The h′ graph looks like a cubic function
graph. Conjecture: Seventh-degree function
has a sixth-degree function for its derivative.
Q9. c. By plotting the graph using a friendly window,
then tracing, the zeros of h′ are −2, 1, 2.5.
d. If h′(x) = 0, the h graph has a high point or
a low point. This is reasonable because if
h′(x) = 0, the rate of change of h(x) is zero,
Q10. “lim” is missing. which would happen when the graph stops
x →c

1. a. going up and starts going down, or vice


y versa.
40
f
e. See the graph in part a.
x
4.
–2 2 y
f´ 10 q'
2 x

b. f ′(x) is positive for −2 < x < 2. The graph of


f is increasing for these x-values.
c. f ( x) is decreasing for x satisfying |x| > 2.
f ′(x) < 0 for these values of x. The graph does not have the expected shape for a
d. Where the f ′ graph crosses the x-axis, the quartic function. The two high points and the low
f graph has a high point or a low point. point all appear to occur as a high point at
x = 2. The derivative graph crosses the x-axis just
e. See the graph in part a.
once, at x = 2, indicating that this is the only
f. Conjecture: f ′ is quadratic. place where the function graph is horizontal.

Calculus Solutions Manual Problem Set 3-3 31


© 2005 Key Curriculum Press
5. a. derivative equal the function value. So the base
y
must be greater than 2. By experimenting, 3 is
f
4 too large, but not by much. You can use trial
and error with bases between 2 and 3, checking
f´ the results either by plotting the graph and the
x
5
numerical derivative or by constructing tables.
An ingenious method that some students come
up with uses the numerical derivative and
b. Amplitude = 1, period = 2π = 6.283… numerical solver features to solve
c. The graph of f ′ also has amplitude 1 and nDeriv(bx, x, 1) = b1 at x = 1. The answer is
period 2π. about 2.718281… . (In Section 3-9, students will
d.
y learn that this number is e, the base of natural
logarithms.) The graph of f ( x) = 2.781…x
f
4 g
and its numerical derivative are shown here.

x y

5
f´ and g´

f and f'
The graphs of f and g are the same shape,
spaced 1 unit apart vertically. The graphs of 1 x
f ′ and g′ are identical! This is to be expected 1
because the shapes of the f and g graphs are
the same. 12. Answer will vary depending on calculator.
6.
13. a. Maximum area = (12.01)2 = 144.2401 in.2
y Minimum area = (11.99)2 = 143.7601 in.2
4
f
Range is 143.7601 ≤ area ≤ 144.2401.
Area is within 0.2401 in.2 of the ideal.

x b. Let x be the number of inches.
5 Area = x2.
The right side of 12 is more restrictive, so set
x2 = 144.02.
The function available on the grapher is y = cos x. ∴ x = 144.021/2 = 12.000833…
The amplitude is 1, the period is 2π, and the Keep the tile dimensions within 0.0008 in. of
shape is sinusoidal. cos 0 = 1, and the graph is at 12 in.
a high point, y = 1, when x = 0.
c. The 0.02 in part b corresponds to ε, and the
7. 8.
y y
0.0008 corresponds to δ.
f 14. The average of the forward and backwards
3 3
f' difference quotients equals
x x
4 4
1  f ( x + h) – f ( x ) f ( x ) – f ( x – h) 
+
f´ f 2  h h 
1 f ( x + h) – f ( x – h) 
=  
9. 10. 2 h
y y f ( x + h) – f ( x – h)
5
f f = , Q .E .D .
3 2h
f'
15. a. f ( x) = x 3 − x + 1 ⇒ f ( 1) = 1
f´ x x
1 2
( x 3 – x + 1) – 1
f ′(1) = lim
x →1 x –1
x3 – x x ( x + 1)( x – 1)
11. The derivative for f ( x) = 2x is consistently below = lim = lim
x →1 x – 1 x →1 x –1
that of the function itself. This fact implies that
= lim x ( x + 1) = 2
f ( x) does not increase rapidly enough to make the x →1

32 Problem Set 3-3 Calculus Solutions Manual


© 2005 Key Curriculum Press
f (1.1) – f (1) 1.231 – 1 2. y = 11x8 ⇒ dy/dx = 88x7
b. Forward: = = 2.31
0.1 0.1 3. v = 0.007t− 83 ⇒ dv/dt = −0.581t− 84
f (1) – f (0.9) 1 – 0.829
= = 1.71 x –9 1
Backwards: 4. v( x ) = ⇒ v ′( x ) = – x –10
0.1 0.1 18 2
Symmetric:
5. M(x) = 1215 ⇒ M′ (x) = 0 (Derivative of a
f (1.1) – f (0.9) 1.231 – 0.829 constant)
= = 2.01
2(0.1) 0.2 6. f (x) = 4.7723 ⇒ f ′ (x) = 0 (Derivative of a
The symmetric difference quotient is closer to constant)
the actual derivative because it is the average 7. y = 0.3x 2 − 8x + 4 ⇒ dy/dx = 0.6x − 8
of the other two, and the other two span the
8. r = 0.2x2 + 6x − 1 ⇒ dr/dx = 0.4x + 6
actual derivative.
d
c. f ( 0) = 1 9. (13 – x ) = −1
dx
( x 3 – x + 1) – 1
f ′(0) = lim 10. f (x) = 4.5x 2 − x ⇒ f′ (x) = 9x − 1
x→0 x–0
x3 – x 11. y = x 2.3 + 5x − 2 − 100x + 4 ⇒
= lim = lim ( x 2 – 1) = −1 dy/dx = 2.3x1.3 − 10x− 3 − 100
x→0 x x→0

f (0.1) – f (0) 0.901 – 1 d 2/5 2


d. Forward: = = –0.99 12. ( x – 4 x 2 – 3 x –1 + 14) = x –3/ 5 − 8 x + 3 x –2
0.1 0.1 dx 5
f (–0.1) – f (0) 1.099 − 1 13. v = (3x − 4)2 = 9x2 − 24x + 16 ⇒ dv/dx = 18x − 24
Backwards: = = –0.99 14. u = (5x − 7)2 = 25x 2 − 70x + 49 ⇒
0.1 0.1
Symmetric: du/dx = 50x − 70
f (0.1) – f (–0.1) 0.901 – 1.099 15. f (x) = (2x + 5)3 = 8x3 + 60x2 + 150x + 125 ⇒
= = −0.99 f ′ (x) = 24x2 + 120x + 150
2(0.1) 0.2
16. f (x) = (4x − 1)3 = 64x 3 − 48x 2 + 12x − 1 ⇒
All three difference quotients are equal because
f ′ (x) = 192x2 − 96x + 12
f ( x) changes just as much from −0.1 to 0 as it
does from 0 to 0.1. x2
17. P( x ) = – x + 4 ⇒ P ′( x ) = x – 1
16. 2
x3 x2
h Backwards Forward Symmetric 18. Q( x ) = + – x + 1 ⇒ Q′( x ) = x 2 + x – 1
3 2
0.1 −1.1544… 3.1544… 1
19. f (x) = 7x4
0.01 −3.6415… 5.6415… 1 7( x + h) 4 – 7 x 4
f ′( x ) = lim
0.001 −9 11 1 h→0 h
The backwards difference quotients are becoming = lim (28 x + 42 x 2 h + 28 xh 2 + 7h 3 ) = 28 x 3
3
h→ 0
large and negative, while the forward difference
quotients are becoming large and positive. Their By formula, f ′(x) = 7 ⋅ 4x3 = 28x3, which checks.
average, the symmetric difference quotient, is 20. g(x) = 5x3
always equal to 1. 5( x + h)3 – 5 x 3
17. Answers will vary. g′( x ) = lim
h→0 h
= lim (15 x + 15 xh + 5h 2 ) = 15 x 2
2
h→ 0
Problem Set 3-4
By formula, g ′ (x) = 5 ⋅ 3x2 = 15x2, which checks.
Q1. 9x 2 − 24x + 16
21. v(t) = 10t2 − 5t + 7
Q2. a3 + 3a2b + 3ab2 + b3
[10(t + h)2 − 5(t + h) + 7] − (10t 2 − 5t + 7)
Q3. See the text definition of derivative. v ′(t ) = lim
h→0 h
f ( x + h) – f ( x – h)
Q4. 20th + 10 h 2 – 5h
2h = lim
h→0 h
Q5. No limit (infinite) Q6. log 73 = lim (20t + 10 h – 5) = 20t – 5
h→0
Q7. 3 Q8. Pythagorean theorem
By formula, v ′ (t) = 10 ⋅ 2t − 5 = 20t − 5, which
Q9. 10 Q10. C checks.
1. f ( x) = 5x 4 ⇒ f ′ ( x) = 20x3

Calculus Solutions Manual Problem Set 3-4 33


© 2005 Key Curriculum Press
22. s(t ) = t 4 – 6t 2 + 3.7 d. f (3) = −6.2
[(t + h) 4 − 6(t + h)2 + 3.7] − [t 4 − 6t 2 + 3.7] f ′(3) = 3.8 (by formula)
s ′(t ) = lim
h→0 h f ′(3) ≈ 3.8000004 (depending on grapher)
4t 3 h + 6t 2 h 2 + 4th 3 + h 4 – 12th – 6h 2
= lim The two values of f ′(3) are almost identical!
h→0 h
= lim ( 4t 3 + 6t 2 h + 4th 2 + h 3 – 12t – 6h) 28. a. g(x) = x− 1. Conjecture: g′(x) = −1 ⋅ x− 2.
h→0
Conjecture is confirmed.
= 4t – 12t
3

By formula, s′(t ) = 4t 3 – 6 ⋅ 2t = 4t 3 – 12t , which y


checks. y1
1
23. Mae should realize that you differentiate x
functions, not values of functions. If you 1
substitute a value for x into f (x) = x4, you get y 2 and y3
f (3) = 34 = 81, which is a new function, g(x) =
81. It is the derivative of g that equals zero.
Moral: Differentiate before you substitute for x.
24. a. v(x) = h′(x) = −10x + 20 b. h(x) = x1/ 2. Conjecture: g′(x) = 0.5x− 1/ 2.
Conjecture is confirmed.
b. The book was going down at 10 m/s.
The velocity is −10, so h(x) is decreasing.
y
c. The book was 15 m above where he threw it. 2
y1
d. 2 s. The book is at its highest point when the
velocity is zero. v(x) = 0 if and only if x = 2.
y2 and y3
25.
1 x
y
7

c. e(x) = 2x. Conjecture: e′ (x) = x ⋅ 2x− 1.


f´ Conjecture is refuted!
f x
y y2
9

y1 y3
26.
y 1
3 1 x

g
x 29. f ( x ) = x 1/ 2 + 2 x – 13
6 f ′( x ) = 12 x –1/ 2 + 2, f ′( 4) = 94
g'
Increasing by 9/4 y-units per x-unit at x = 4
30. f (x) = x− 2 − 3x + 11
f ′(x) = −2x− 3 − 3, f ′(1) = −5
Decreasing by 5 y-units per x-unit at x = 1
27. a.
y
31. f (x) = x1.5 − 6x + 30
10 f´ f ′(x) = 1.5x0.5 − 6, f ′(9) = −1.5
Decreasing by 1.5 y-units per x-unit at x = 9
1 x

f 32. f ( x ) = –3 x + x + 1
f ′( x ) = – 23 x –1/ 2 + 1, f ′(2) = –0.0606K
Decreasing by approximately 0.0607 y-unit per
b. The graph of f ′ is shown dashed in part a. x-unit at x = 2
c. There appear to be only two graphs because x3
the exact and the numerical derivative graphs 33. f ( x ) = – x 2 – 3 x + 5, f ′( x ) = x 2 – 2 x – 3
3
almost coincide.

34 Problem Set 3-4 Calculus Solutions Manual


© 2005 Key Curriculum Press
High and low points of the f graph are at the 37. If f (x) = xn, then f′(x) = nxn− 1.
x-intercepts of the f ′ graph. Proof:
y
( x + h) n – x n
f ′( x ) = lim
f´ h→0 h
5
x
x n + nx n−1h + 12 n(n − 1) x n−2 h 2 + L + h n − x n
–1 3 = lim
f h→ 0 h
= lim nx n−1 + n(n − 1) x n−2 h + L + h n−1 
1
h→ 0  2 
x3 = nx n–1 + 0 + 0 + L + 0
34. f ( x ) = – 2 x 2 + 3 x + 9, f ′( x ) = x 2 – 4 x + 3 = nx n –1 , which is from the second term in the
3
binomial expansion of (x + h)n, Q.E.D.
High and low points of the f graph are at the
x-intercepts of the f ′ graph. 38. If yn = u1 + u2 + u3 + L + un , where the ui are
differentiable functions of x, prove that
yn′ = u1′ + u2′ + u3′ + L + un′ for all integers n ≥ 2.
y
15
f
Proof:
f' Anchor: For n = 2, y2 = u1 + u2 .
x
1 3
∴ y2′ = u1′ + u2′ by the derivative of a sum of the
two functions property, thus anchoring the
induction.
35. If f (x) = k ⋅ g(x), then f ′(x) = k ⋅ g′(x). Induction hypothesis:
Proof: Suppose that for n = k > 2,
yk′ = u1′ + u2′ + u3′ + L + uk′ .
f ( x + h) – f ( x )
f ′( x ) = lim Verification for n = k + 1:
h→0 h Let yk +1 = u1 + u2 + u3 + L + uk + uk +1 .
k ⋅ g( x + h ) – k ⋅ g ( x ) Then yk +1 = (u1 + u2 + u3 + L + uk ) + uk +1 , which
= lim
h→0 h is a sum of two terms.
g( x + h) – g( x ) ∴ yk′ +1 = (u1 + u2 + u3 + L + uk )′ + uk′ +1 ,
= lim k ⋅
h→0 h which, by the anchor,
g( x + h) – g( x ) = (u1′ + u2′ + u3′ + L + uk′ ) + uk′ +1
= k ⋅ lim = u1′ + u2′ + u3′ + L + uk′ + uk′ +1 ,
h→0 h
= k ⋅ g ′( x ), Q .E .D . which completes the induction.
Conclusion:
Dilating a function f (x) vertically by a constant k
∴ yn′ = u1′ + u2′ + u3′ + L + un′ for all integers
results in the new function g(x) = k ⋅ f (x). What
n ≥ 2, Q .E .D .
has been shown is that
d d 39. a. f ′( x ) = 3 x 2 – 10 x + 5 ⇒ f ( x ) = x 3 – 5 x 2 + 5 x
( k ⋅ f ( x )) = k ⋅ f ( x) b. g(x) = f (x) + 13 is also an answer to part a
dx dx
because it has the same derivative as f (x). The
That is, dilating a function vertically by a
derivative of a constant is zero.
constant k dilates the derivative function by a
constant factor of k. c. The name antiderivative is chosen because it
is an inverse operation of taking the derivative.
36. If f ( x ) = x 5 , then f ′(c) = 5c 4 .
d d
d. [ g( x )] = [ f ( x ) + C] =
Proof: dx dx
f ( x ) – f (c ) d d d
f ′(c) = lim f ( x) + C = f ( x)
x →c x–c dx dx dx
x 5 – c5 The word indefinite is used because of the
= lim unspecified constant C.
x →c x–c
( x − c)( x 4 + x 3c + x 2 c 2 + xc 3 + c 4 ) 40. a. f ′(x) = 5x 4 ⇒ f (x) = x 5 + C
= lim f(2) = 23 ⇒ (2)5 + C = 23
x →c x−c
C = −9
= lim ( x 4 + x 3c + x 2 c 2 + xc 3 + c 4 ) ∴ f (x) = x 5 − 9
x →c

= c4 + c4 + c4 + c4 + c4 b. f ′(x) = 0.12x2 ⇒ f (x) = 0.04x3 + C


= 5c4, Q .E.D . f (1) = 500 ⇒ 0.04(1)3 + C = 500

Calculus Solutions Manual Problem Set 3-4 35


© 2005 Key Curriculum Press
C = 499.96 5. a. x = – t 3 + 13t 2 – 35t + 27 (See Problem 3.)
∴ f (x) = 0.04x3 + 499.96 v = –3t 2 + 26t – 35, a = –6t + 26
c. f ′( x ) = x 3 ⇒ f ( x ) = 14 x 4 + C b. v(1) = –3 + 26 – 35 = –12
f (5) = 2 ⇒ 14 (5) 4 + C = 2 So x is decreasing at 12 ft/s at t = 1.
C = −154.25 a(1) = –6 + 26 = 20
So the object is slowing down at 20 (ft/s)/s
∴ f ( x ) = 14 x 4 − 154.25
because the velocity and acceleration are in
opposite directions when t = 1.
v(6) = −3(6)2 + 26(6) − 35 = 13
Problem Set 3-5 So x is increasing at 13 ft/s at t = 6.
Q1. No values of t Q2. dy/dx = 10 x a(6) = −6(6) + 26 = −10
−4 So the object is slowing down at 10 (ft/s)/s
Q3. y ′ = −51x Q4. f ′( x ) = 1.7 x 0.7
because the velocity and acceleration are in
Q5. ( d/dx )(3 x + 5) = 3 Q6. f (3) = 45 opposite directions when t = 6.
Q7. f ′(3) = 30 Q8. 45 v(8) = –3(8)2 + 26(8) – 35 = –19
So x is decreasing at 19 ft/s at t = 8.
Q9. ε Q10. C a(1) = –6(8) + 26 = –22
4 2.4
1. y = 5t – 3t + 7t So the object is speeding up at 22 (ft/s)/s
dy because the velocity and acceleration are in the
v= = 20t 3 – 7.2t 1.4 + 7, same directions when t = 8.
dt
a=
dv
= 60t 2 – 10.08t 0.4 c. At t = 7, x has a relative maximum because
dt v(7) = 0 at that point and is positive just
2. y = 0.3t –4 – 5t before t = 7 and negative just after. No, x is
dy dv never negative for t in [0, 9]. It starts out at
v= = –1.2t –5 – 5, a = = 6t –6 27 ft, decreases to just above zero around
dt dt
t = 1.7, and does not become negative until
3. x = −t3 + 13t2 − 35t + 27. The object starts out some time between t = 9.6 and 9.7.
at x = 27 ft when t = 0 s. It moves to the left to
6. a. x = t 4 – 11t 3 + 38t 2 – 48t + 50 (See Problem 4.)
x ≈ 0.15 ft when t ≈ 1.7 s. It turns there and
goes to the right to x = 70 ft when t = 7 s. It v = 4t 3 – 33t 2 + 76t – 48, a = 12t 2 – 66t + 76
turns there and speeds up, going to the left for all b. At t = 1, v(1) = –1 and a(1) = 22. At t = 3,
higher values of t. v(3) = –9 and a(3) = –14 . At t = 5, v(5) = 7
y and a(5) = 46 . The object is slowing down at
Turns at t = 7, x = 76 t = 1 because the velocity and acceleration are
Turns at t = 1.7, x = 0.15 in opposite directions. The object is speeding
up at t = 3 and t = 5 because velocity and
Starts at t = 0, x = 27
acceleration are in the same direction.
x
10 c. v = 0 when t = 1.0475… , 2.3708… ,
or 4.8315… .
4. x = t 4 – 11t 3 + 38t 2 – 48t + 50. The object starts
at x = 50 ft when t = 0 s. It moves to the left d. The displacement is at a maximum or a
to x ≈ 30 ft when t ≈ 1.0 s. Then it moves to the minimum whenever v = 0.
right to x ≈ 34.8 ft when t ≈ 2.4 s. The object
moves to the left again, turning at x ≈ 9.4 ft y

when t ≈ 4.8 s and then moving back to the right


for higher values of t. x

10 t
y
Turns at t ≈ 4.8, x ≈ 9.4
1 2 3 4 5
Turns at t ≈ 2.4, x ≈ 34.8 v

Turns at t ≈ 1.0, x ≈ 30.0

Starts at t ≈ 0, x ≈ 50
x e. a = 0 when t = 1.6413… or 3.8586… .
10
When a = 0, v is at a maximum or minimum

36 Problem Set 3-5 Calculus Solutions Manual


© 2005 Key Curriculum Press
point and the graph of x is at its steepest for 9. a. d (t ) = 18t – 4.9t 2 ⇒ d ′(t ) = 18 – 9.8t
times around these values. d′(1) = 18 – 9.8 = 8.2
y d′(3) = 18 – 9.8 ⋅ 3 = –11.4
d′ is called velocity in physics.
x
b. At t = 1 the football is going up at 8.2 m/s.
10 t At t = 3 the football is going down at
1 2 3 4 5 11.4 m/s. The ball is going up when the
v
a derivative is positive and coming down when
the derivative is negative. The ball is going
7. a.
y
up when the graph slopes up and coming
300 down when the graph slopes down.
d
c. d′( 4) = –21.2, which suggests that the
ball is going down at 21.2 m/s. However,
d´ t
d( 4) = –6.4, which reveals that the ball
10 has gone underground. The function gives
meaningful answers in the real world only if
b. v = d′ = 30 − 2t. Velocity is positive for the domain of t is restricted to values that
0 ≤ t < 15. Calvin is going up the hill for make d(t) nonnegative.
the first 15 s. 10. a.
c. At 15 seconds his car stopped. d(15) = 324, y
so distance is 324 feet.
2 a
d. 99 + 30t – t 2 = 0 ⇒ (33 – t )(3 + t ) = 0 ⇒ t = 33
t
or t = −3. He’ll be back at the bottom when 5 v
t = 33 s.
e. d(0) = 99. The car runs out of gas 99 ft from
the bottom.
8. a. b. The acceleration at the bottom of the swing
y
v = 251 is 0. The acceleration is greatest at either end
200 of its swing.
v
11.
y

a = v' t 2
v
30
t
b. Trace the v′ graph to find a(0) ≈ 32 . The 5
a
acceleration decreases because the velocity is
approaching a constant. In the real world, this
occurs because the wind resistance increases as
the velocity increases. 12. v(t) = 15t0.6 . Because v(t ) = x ′(t ) , x(t) must
c. The limit is 251 ft/s as t approaches infinity. have had t 1.6 in it. The derivative of t1.6 can be
The term 0.88t approaches zero as t gets very assumed to be 1.6t0.6 . So the coefficient of t 1.6
large, leaving only 1 inside the parentheses. must be 15/1.6 , or 9.375. But x(0) was 50.
d. 90% of terminal velocity is 0.9(251) = Thus, x(t) = 9.375t1.6 + 50. The derivative x′(t)
225.9 ft/s. really does equal v(t). Using this equation,
Algebraic solution: x(10) = 9.375(101.6 ) + 50 = 423.225K .
251(1 – 0.88t ) = 225.9 ⇒ –0.88t = –0.1 So the distance traveled is 423.225… − 50 =
373.225… , or about 373 ft.
log 0.1
t= = 18.012394... ≈ 18.0 s 13. The average rate is defined to be the change in
log 0.88
the dependent variable divided by the change in
Numerical solution gives the same answer.
the independent variable (such as total distance
Graphical solution: Trace to v(t) = 225.9.
divided by total time). Thus, the difference
T is between 18 and 18.5.
quotient is an average rate. The instantaneous
e. Find the numerical derivative. rate is the limit of this average rate as the change
v′(18.0123…) ≈ 3.2086… , which is in the independent variable approaches zero.
approximately 10% of the initial acceleration.

Calculus Solutions Manual Problem Set 3-5 37


© 2005 Key Curriculum Press
14. a. m ′′(5) = 14.6299K
y m ′′(10) = 23.5616K
3
Both quantities are in the units ($/yr)yr.
The quantities represent the instantaneous rate of
change of the instantaneous rate of change of the
t amount of money in the account. For example,
10 at t = 5, the rate of increase of the account
v (153.50 $/yr) is increasing at a rate of
14.63 ($/yr)/yr.
20. p ′(7) = − 4.9510K
b. y is a relative maximum when t ≈ 0, 4, p ′(14) = −2.4755K
8, … . ∴ p ′(14) = 12 p ′(7)
y is a relative minimum when t ≈ 2, 6, 10, … . The fact that these derivatives are negative tells
c. The velocity is a relative maximum when us that the amount of nitrogen 17 is decreasing.
t ≈ 3 or 7. The displacement graph at these p ′′(7) = 0.4902K
times appears to be increasing the fastest. p ′′(14) = 0.2451K
d. The equation used in the text is Both quantities are in units (% of nitrogen
π 17/s)/s. The quantities represent the rate of
y = 2 + 0.85t cos t change of the rate of change of the percentage of
2
The student could observe that the period is 4, nitrogen 17 remaining. For example, at t = 7 s,
leading to the coefficient π/2. The amplitude the rate of decrease (−4.95%/s) is changing at a
decreases in a way that suggests an exponential rate of 0.49 (%/s)/s.
function with base close to, but less than, 1. 21.
y
The additive 2 raises the graph up two units,
as can be ascertained by the fact that the graph 2
f f´
seems to converge to 2 as t gets larger. The x
numerical derivative of the function shown in 5

part a agrees with the graph of the velocity.


Note that the actual maximum and minimum
values occur slightly before the values of t
read from the graph in part a. For instance, The graph of the derivative looks like the graph
the maximum near t = 4 is actually at of y = cos (x).
t = 3.9343… . 22.
dy d2y
15. y = 5x 3 ⇒ = 15 x 2 ⇒ 2 = 30 x y
dx dx 2

dy d2y f' f x
16. y = 7x 4 ⇒ = 28 x 3 ⇒ 2 = 84 x 2
dx dx 5

dy
17. y = 9x + x ⇒
2 5
= 18 x + 5 x 4 ⇒
dx
d2y
= 18 + 20 x 3 The graph of the derivative looks like the graph
dx 2 of y = −sin (x).
18. y = 10 x 2 − 15 x + 42 ⇒
dy d2y Problem Set 3-6
= 20 x − 15 ⇒ 2 = 20
dx dx 1.
19. m′(5) = 153.4979K y

3
m′(10) = 247.2100K
These numbers represent the instantaneous rate of
change of the amount of money in the account. x
The second quantity is larger because the money 10

grows at a rate proportional to the amount of


money in the account. Because there is more
money after 10 years, the rate of increase should
also be larger.

38 Problem Set 3-6 Calculus Solutions Manual


© 2005 Key Curriculum Press
2. The graph confirms the conjecture. e. q(x) = 1/(tan x). Inside: tangent. Outside:
y reciprocal.
2
y2 and y 3 y1
f. L(x) = log (sec x). Inside: secant. Outside:
x logarithm.
5 8. Answers will vary.

Problem Set 3-7


3. g(x) = sin 3x Q1. f (c) exists 
Conjecture: g′(x) = 3 cos 3x 
Q2. lim f ( x ) exists  ( Any order is acceptable.)
The graph confirms the conjecture. x →c
Q3. lim f ( x ) = f (c)
3
y x →c 

Q4. No (not continuous) Q5. dy/dx = 16 x –1/ 5
g
x Q6. f (x) = −10x− 3 Q7. Antiderivative
10
Q8.
y = sin x
x

4. h( x ) = sin x 2
Conjecture: h′( x ) = 2 x cos x 2 Q9.
y = cos x
The graph confirms the conjecture. x

y
h'
1 h
Q10. C
x
5 1. a. Let y = f (u), u = g(x).
dy dy du
= ⋅
dx du dx
b. y′ = f ′[ g( x )] ⋅ g′( x )
5. t ( x ) = sin x 0.7 c. To differentiate a composite function,
Conjecture: t ′( x ) = 0.7 x –0.3 cos x 0.7 differentiate the outside function with respect to
The graph confirms the conjecture. the inside function, then multiply by the deriva-
y tive of the inside function with respect to x.
2. f ( x ) = ( x 2 − 1)3
1 t
x a. f ′(x) = 3(x2 − 1)2(2x) = 6x(x2 − 1)2
10
t´ b. (x 2 − 1)3 = x 6 − 3x 4 + 3x 2 − 1,
so f ′(x) = 6x 5 − 12x 3 + 6x.
c. From part a, f ′(x) = 6x(x2 − 1)2 =
6. f (x) = sin [g(x)] 6x(x 4 − 2x 2 + 1) = 6x 5 − 12x 3 + 6x, so the
f is a composite function. two answers are equivalent.
g is the inside function. 3. f ( x ) = cos 3 x ⇒ f ′( x ) = − sin 3 x ⋅ 3 = −3 sin 3 x
sine is the outside function. 4. f ( x ) = sin 5 x ⇒ f ′( x ) = 5 cos 5 x
Differentiate the outside function with respect to
5. g( x ) = cos ( x 3 ) ⇒ g′( x ) = −3 x 2 sin ( x 3 )
the inside function. Then multiply the answer by
the derivative of the inside function with respect 6. h( x ) = sin ( x 5 ) ⇒ h′( x ) = 5 x 4 cos ( x 5 )
to x. 7. y = (cos x)3 ⇒
7. a. f (x) = sin 3x. Inside: 3x. Outside: sine. y′ = 3(cos x)2 ⋅ (−sin x) = −3 cos2 x sin x
b. h(x) = sin3 x. Inside: sine. Outside: cube. 8. f ( x ) = (sin x )5 ⇒
c. g(x) = sin x3. Inside: cube. Outside: sine. f ′( x ) = 5(sin x ) 4 ⋅ cos x = 5 sin 4 x cos x
d. r(x) = 2cos x. Inside: cosine. Outside: 9. y = sin 6 x ⇒ y′ = 6 sin 5 x cos x
exponential. 10. f ( x ) = cos 7 x ⇒
f ′( x ) = 7 cos 6 x ⋅ ( − sin x ) = –7 cos 6 x sin x

Calculus Solutions Manual Problem Set 3-7 39


© 2005 Key Curriculum Press
11. y = −6 sin 3x ⇒ y′ = −18 cos 3x The line is tangent to the graph.
12. f (x) = 4 cos (−5x) ⇒ y
f′ (x) = 4[−sin (−5x)] ⋅ (−5) = 20 sin (−5x) 5

d
13. (cos 4 7 x ) = 4 cos3 7 x ⋅ ( − sin 7 x ) ⋅ 7
dx x
= −28 cos3 7x sin 7x 3

d
14. (sin 9 13 x ) = 9 sin 8 13 x cos 13 x ⋅ 13
dx
= 117 sin8 13x cos 13x
15. f (x) = 24 sin5/3 4x ⇒ 28. y = 7 sin π t + 12t1.2
dy
f′ (x) = 40 sin2/3 4x ⋅ cos 4x ⋅ 4 velocity = = 7π cos π t + 14.4t0.2
dt
= 160 sin2/3 4x cos 4x Yes, there are times when the beanstalk is
16. f (x) = −100 sin6/5 (−9x) ⇒ shrinking. The velocity graph is negative for
brief intervals, and the y-graph is decreasing in
f′ (x) = −120 sin1/5 (−9x) ⋅ cos (−9x) ⋅ (−9)
these intervals.
= 1080 sin1/5 (−9x) cos (−9x)
17. f (x) = (5x + 3)7 ⇒ y and v
f′ (x) = 7(5x + 3)6 ⋅ 5 = 35(5x + 3)6 y

18. f (x) = (x + 8) ⇒
2 9

f′ (x) = 9(x2 + 8)8 ⋅ 2x = 18x(x2 + 8)8


50 v
19. y = (4x3 − 7)− 6 ⇒ t

y′ = −6(4x3 − 7)− 7 ⋅ 12x2 = −72x2(4x3 − 7)− 7 10

20. y = (x 2 + 3x − 7)− 5 ⇒
y′ = −5(x 2 + 3x − 7)− 6 ⋅ (2x + 3) 4π 3 dV
29. a. V = r ⇒ = 4πr 2
= −5(2x + 3)(x2 + 3x − 7)− 6 3 dr
dV/dr is in (cm3/cm), or cm2.
21. y = [cos (x2 + 3)]100 ⇒
y′ = 100 [cos (x2 + 3)]99 ⋅ [−sin (x2 + 3)] ⋅ 2x b. r = 6t + 10
= −200x cos99(x2 + 3) sin (x2 + 3) dr
22. y = [cos (5x + 3)4]5 ⇒ y′ = 5[cos (5x + 3)4]4 ⋅ c. = 6 (not surprising!). Units: cm/min
dt
[−sin (5x + 3)4] ⋅ 4(5x + 3)3 ⋅ 5 = dV dV dr
−100(5x + 3)3 cos4 (5x + 3)4 sin (5x + 3)4 d. = ⋅
dt dr dt
dy
23. y = 4 cos 5x ⇒ = 4(−sin 5x)5 = −20 sin 5x ⇒ When t = 5, r = 40. So
dx
d2y dV
= −20(cos 5x)5 = −100 cos 5x = 4π ( 40 2 ) = 6400π .
dx 2 dr
dV
24. y = 7 sin (2x + 5) ⇒ ∴ = 6400π ⋅ 6 = 38,400π cm 3 /min.
dy dt
= 7 cos (2x + 5)(2) = 14 cos (2x + 5) ⇒ dV/dr has units cm 2, and dr/dt has units
dx
2 cm
d y cm/min, so dV/dt has units cm 2 ⋅ ,
= 14[−sin (2x + 5)](2) = min
dx 2
−28 sin (2x + 5) which becomes cm3/min, Q.E.D. This
matches the commonsense answer that rate of
1
25. f′ (x) = cos 5x ⇒ f (x) = sin 5x + C volume cm 3
5 change of volume is = .
time min
26. f′ (x) = 10 sin 2x ⇒ f (x) = −5 cos 2x + C

27. f (x) = 5 cos 0.2x e. V = (6t + 10)3
f′ (x) = −5 sin 0.2x ⋅ 0.2 = −sin 0.2x 3
f′ (3) = −sin 0.6 = −0.5646… and ∴
dV
= 4π (6t + 10)2 (6) = 24π (6t + 10)2
f (3) = 4.1266… dt
dV
The line has the equation When t = 5, = 24π [6(5) + 10]2 = 38,400π .
y = −0.5646…x + 5.8205… . dt

40 Problem Set 3-7 Calculus Solutions Manual


© 2005 Key Curriculum Press
π
30. a. y(t ) = 25 + 20 cos (t – 3)
u 10
π
b. y ′(t ) = –2π sin (t – 3)
10
π
∆u c. y ′(15) = –2π sin (15 – 3) = 3.69316 K
10
∆x
y(t) is increasing at about 3.7 ft/s.
The fastest that y(t) changes is 2π, or
x
d.
6.28… ft/s. The seat is at y(t) = 25 ft above
∆u does not approach zero as ∆x approaches the ground then.
zero from the left side. (∆u does approach zero 2. a. y = C + A cos B(x − D).
as ∆x approaches zero from the left side.) B = 2π /6 = π /3 rad/s
b. D = phase displacement = 1.3 s
A = 0.5(110 − 50) = 30 cm
u

C = 110 − 30 or 50 + 30, which equals 80 cm.


π
∆u ∴ d = 80 + 30 cos (t – 1.3)
3
π
b. d ′ = –10π sin (t – 1.3)
∆x
x
3
π
c. d′(5) = −10π sin (5 – 1.3) = 21.02135K
3
∆u does approach zero as ∆x approaches zero π
from either side. d′(11) = –10π sin (11 – 1.3) = 21.02135K
3
At both times, the pendulum is moving away
from the wall at about 21.0 cm/s. The
Problem Set 3-8 answers are the same because the times are
Q1. f′(x) = 9x 8 Q2. dy/dx = −3 sin x exactly one period apart.
Q3. y′ = 72x5 (5x6 + 11)1.4 Q4. s′ = 0 π
d. d′(20) = –10π sin (20 – 1.3) = −21.02135K
Q5. 12 Q6. 1 3
The pendulum is moving toward the wall.
Q7. Yes (continuous) Q8. f (x) = −cos x 2
Because the derivative is negative, d is
Q9. Q10. E decreasing, which in this problem implies
4
y
f' motion toward the wall.
e. The fastest is 10π ≈ 31.4 cm/s, when d = 80.
π π
x f. 0 = –10π sin (t – 1.3) ⇒ sin (t − 1.3) = 0
4 3 3
π −1 π
⇒ (t – 1.3) = sin 0 ⇒ (t − 1.3) = 0 + π n
3 3
⇒ t − 1.3 = 3n ⇒ t = 1.3 + 3n.
1. a. The first positive time occurs when n = 0,
d ( x)
45 that is, when t = 1.3 s. When the velocity
is zero, the pendulum is at its maximum
25 height.
Increasing 3. a. The curb has slope (3.25 − 0.75)/44 = 2.5/44.
∴ equation is f (x) = 0.75 + (2.5/44)x.
5 x
3 15 23
b. Sinusoid has period 8 ft, so B = 2π /8 = π /4.
y(t) = C + A cos B(t − D) Amplitude = 0.5(0.75 − 0.25) = 0.25 ft. Low
Vertical displacement = 25 = C end of ramp is a low point on the sinusoid.
Amplitude = 0.5(40) = 20 = A ∴ sinusoidal axis is at y = 0.25 when x = 0
Phase disp. (for cosine) = 3 = D and goes up with slope 2.5/44.
Period = 60/3 = 20, so B = 2π/20 = π/10. Sinusoid is at a low point when x = 0. So
(Note that B is the angular velocity in radians phase displacement is zero if the cosine is
per second.) subtracted.

Calculus Solutions Manual Problem Set 3-8 41


© 2005 Key Curriculum Press
∴ equation is gravitational acceleration, about 9.8 m/s2.
2.5 π Consequently, if the pendulum is 1 meter long,
g( x ) = 0.25 + x − 0.25 cos x
44 4 its period will be 2π 1/9.8 = 2.007K , or about
(There are other correct forms.) 2 s. This is the period for a complete back-and-
2.5 π π
c. g ′( x ) = + sin x forth swing. You must quadruple the length of a
44 16 4 pendulum to double its period. A pendulum hung
2.5 π π
g ′( 9) = + sin (9) = 0.1956 K ft/ft from the ceiling will have a period slow enough
44 16 4 to measure fairly precisely. A good way to get
Going up at about 0.2 vertical ft per more accuracy is to count the total time for ten
horizontal ft swings, then divide by 10. The period is roughly
2.5 π π
g ′(15) = + sin (15) = −0.0820 K ft/ft constant for any (moderate) amplitude, as long as
44 16 4 the amplitude is not too big. This fact is not
Going down at about 0.08 vertical ft per obvious to the uninitiated student and is worth
horizontal ft. A positive derivative implies spending time showing. It is quite dramatic to
g(x) is getting larger and thus the child is watch a pendulum take just as long to make ten
going up. A negative derivative implies g(x) swings with amplitude 2 cm as it does with
is getting smaller and thus the child is going amplitude 20 or 30 cm.
down.
d. By tracing the g ′ graph, maximum value of 6. The following data were computed from actual
g′ (x) is 0.2531… ft/ft (about 14.2° up). sunrise and sunset times for San Antonio for
Minimum is −0.1395… ft/ft (about 7.9° each ten days. You can get similar information
down). for your locality from the local weather bureau or
4. a. Let d = day number and L(d) = number of newspaper office, from the Nautical Almanac
minutes. Office, U.S. Naval Observatory, Washington,
14 hours 3 minutes is 843 minutes. 10 hours D.C., 20390, or from the Internet.
15 minutes is 615 minutes.
∴ amplitude = (1/2)(843 − 615) = 114 min. Day Min Day Min Day Min
Sinusoidal axis is at L(d) = 615 + 114 =
729 min. 0 617 130 811 260 738
Assuming a 365-day year, B = 2π/365. 10 623 140 823 270 720
Phase displacement = 172 20 632 150 833 280 703

∴ L( d ) = 729 + 114 cos ( d – 172) 30 645 160 840 290 686
365
On August 7, d = 219. 40 660 170 842 300 669
2π 50 676 180 842 310 653
L(219) = 729 + 114 cos (219 – 172) =
365 60 693 190 836 320 639
807.67… , or about 13 hours 28 minutes.
228π 2π 70 711 200 828 330 628
b. L ′( d ) = − sin ( d − 172)
365 365 80 729 210 816 340 620
On August 7, d = 219. 90 747 220 803 350 615
228π 2π
L ′(219) = − sin (219 – 172) = 100 764 230 789 360 615
365 365
–1.42009… 110 780 240 772
Rate ≈ –1.42 min/day 120 797 250 755
(Decreasing at about 1.42 min/day)
L(d )
c. The greatest rate occurs when the sine is
1 or −1. 800

Rate is 228π/365 ≈ 1.96 min/day.


1/4 year is about 91 days. So greatest rate 700
occurs at day 172 ± 91, which is day 263 or
day 81 (September 20 or March 22). 600 d
5. In general, the period for a pendulum formed by 0 100 200 300

a weight suspended by a string of negligible The graph shows a good fit to the data. But there
mass is 2π L/g , where L is the length from the is a noticeable deviation in the fall and winter,
pivot point to the center of mass (actually, the here the day is slightly longer than predicted.
center of percussion) of the weight, and g is the The main reason for the discrepancy, apparently,

42 Problem Set 3-8 Calculus Solutions Manual


© 2005 Key Curriculum Press
is the fact that in the fall and winter, Earth is c. Answers will vary according to calculator. For
closer to the Sun and hence moves slightly more the TI-83 in TABLE mode, starting x at 0 and
rapidly through its angle with the Sun than using ∆ x = 10−7 shows that all values round
during the spring and summer. to 1 until x reaches 1.8 × 10−6, which
7. a. registers as 0.999999999999.
y
g h
d. Answer will depend on calculator. For TI-83
in TABLE mode, (sin 0.001)/0.001 is
f 0.999999833333, which agrees exactly with
4
the value published by NBS to 12 places.
x e. If students have studied Taylor series
1
(Chapter 12) before taking this course, they
will be able to see the reason. The Taylor
The limits are all equal to 4. series for sin 0.001 is
b. f (x) < g(x) and lim f ( x ) = lim g( x ) = 4
x →1 x →1 0.0013 0.0015 0.0017
f (x) ≤ h(x) ≤ g(x) 0.001 − + − +L
3! 5! 7!
c.
= 0.00100 00000 00000 00000 000…
x f(x) h(x) g(x)
− 0.00000 00001 66666 66666 666…
0.95 3.795 3.8 3.805 + 0.00000 00000 00000 00833 333…
_______________________________
0.96 3.8368 3.84 3.8432
0.97 3.8782 3.88 3.8818 = 0.00099 99998 33333 34166 666…
10. See the text proof.
0.98 3.9192 3.92 3.9208
11. See the text proof.
0.99 3.9598 3.96 3.9602
12. a. See the text statement of the theorem.
1.00 4 4 4
b. Proof:
1.01 4.0398 4.04 4.0402
Given any ε > 0, there is a δ f > 0 such that
1.02 4.0792 4.08 4.0808
0 < |x − c | < δ f ⇒ |f (x) − L | < ε, because
1.03 4.1182 4.12 4.1218 lim f ( x ) = L. Similarly, there is a δ g > 0
x →c
1.04 4.1568 4.16 4.1632
such that 0 < |x − c | < δ g ⇒ |g(x) − L| < ε.
1.05 4.195 4.2 4.205 Let δ be the smaller of δ f and δ g. Then 0 <
|x − c | < δ ⇒ 0 < |x − c | < δ f = |f (x) − L| <
d. From the table, if x is within 0.02 unit of 2, ε ⇒ f (x) < L + ε , and also 0 < |x − c | <
then both f (x) and g(x) are within 0.1 unit of δ ⇒ 0 < |x − c | < δ g ⇒ |g(x) − L| < ε ⇒
4. From the table, δ = 0.01 or 0.02 will
L − ε < g(x). Then L − ε < g(x) < h(x) < f(x)
work, but 0.03 is too large. All the values of
< L + ε , so |h(x) − L | < ε , so lim h( x ) = L .
h(x) are between the corresponding values of x →c
f (x) and g(x), and the three functions all Q.E.D.
approach 4 as a limit. c. See Figure 3-8c or 3-8d.
sin x 13. a. The limit seems to equal 2.
8. Prove that lim = 0. See the text proof.
x→0 x
b.
9. a. The numbers are correct.
h g
b.
x (sin x)/x 2

0.05 0.99958338541… g h
1
0.04 0.99973335466…
0.03 0.99985000674…
c. See the graph in part b. The lines have
0.02 0.99993333466…
equations g(x) = x + 1 and h(x) = 3 − x.
0.01 0.99998333341…
d. Prove that lim y = 2.
x →1
Values are getting closer to 1.

Calculus Solutions Manual Problem Set 3-8 43


© 2005 Key Curriculum Press
Proof: c. 1061.84/1000 = 1.06184, so the APR for
lim( x + 1) = 1 + 1 = 2 0 to 1 year is approximately 6.184%.
x →1 1127.50/1061.84 = 1.061836… , so the APR
lim(3 − x ) = 3 − 1 = 2
x →1 for 1 to 2 years is approximately 6.184%.
For x < 1, g(x) ≤ y ≤ h(x). 1197.22/1127.50 = 1.06183… , so the APR
∴ the squeeze theorem applies, and lim− y = 2. for 2 to 3 years is approximately 6.184%.
x →1
For x > 1, h(x) ≤ y ≤ g(x). The APR is higher than the instantaneous
∴ the squeeze theorem applies, and lim+ y = 2. rate. Savings institutions may prefer to
x →1 advertise the APR instead of the instantaneous
Because both left- and right-hand limits equal rate because the APR is higher.
2, lim y = 2, Q .E.D .
x →1 2. a. f (t) = 10e− 0.34 t ⇒
e. The word envelope (a noun) is used because f ′ (t) = 10(−0.34)e− 0.34 t = −3.4e− 0.34 t
the small window formed by the two lines f ′ (0) = −3.4
“envelops” (a verb) the graph of the function. f ′ (2) = −1.7224…
1 f ′ (4) = −0.8726…
f. As |x| becomes large, (x − 1) · sin =
x –1 f ′ (6) = −0.4420
sin [1/( x – 1)] Factor of change from 0 to 2:
takes on the form
1/( x – 1) −1.7224…/−3.4 = 0.5066
sin (argument) Factor of change from 2 to 4:
as the argument approaches −0.8726…/−1.7224… = 0.5066
(argument)
zero. Thus the limit is 1 and y approaches Factor of change from 4 to 6:
2 + 1, which equals 3. −0.4420…/−0.8726… = 0.5066
14. Answers will vary. b. f (0) = 10
f (2) = 5.0661…
f (4) = 2.5666…
Problem Set 3-9 f (6) = 1.3002…
Q1. 1 Q2. –sin x Factor of change from 0 to 2:
2 5.0661…/10 = 0.5066
d d
Q3. (cos x ) = ( − sin x ) = − cos x Factor of change from 2 to 4:
dx 2 dx 2.5666…/5.0661… = 0.5066
Q4. y = sin x + C Factor of change from 4 to 6:
Q5. x 8 Q6. x48 1.3002…/2.5666… = 0.5066
Q7. log 32 = 5 log 2 Q8. The factors of change are the same in part a
and part b.
y
c.
y
x
10
f

Q9. Cube function Q10. C 5


f'
1. M(x) = 1000e0.06 x
The values of f are negative because the
a. M′(x) = 1000(0.06)e0.06 x = 60e0.06 x amount of 18-F is decreasing as time goes on.
M′(1) = 63.7101… $/yr
−23.5
M′(10) = 109.3271… $/yr 3. a. A( p) = 63 − 23.5 ln p ⇒ A′( p) =
M′(20) = 199.2070… $/yr p
y
b. M(0) = $1000
M(1) = $1061.84 50 A
M(2) = $1127.50
M(3) = $1197.22 x
10
Increase from year 0 to year 1: $61.84 A´
Increase from year 1 to year 2: $65.66
Increase from year 2 to year 3: $69.72 b. If the pressure is increasing, then the altitude
No, the amount of money in the account does is decreasing. A′(10) = −2.35, so the altitude
not change by the same amount each year. is changing at −2.35 thousand feet/psi. That

44 Problem Set 3-9 Calculus Solutions Manual


© 2005 Key Curriculum Press
is, a change of 1 psi would indicate that the 7. g(x) = −4ecos x ⇒
altitude had decreased by 2.35 thousand feet. g′(x) = −4(ecos x)(−sin x) = 4(sin x) ecos x
The negative sign means that the altitude is 8. h(x) = 8e− sin x ⇒
decreasing. h ′ (x) = 8e− sin x (−cos x) = −8(cos x)e− sin x
c. A′(5) = −4.7 = 4.7 9. y = 2 sin (e4x) ⇒ y′ = 2 cos (e4x) 4e4x = 8e4x cos (e4x)
A′(10) = −2.35 = 2.35 10. y = 6 cos (e− 0.5 x) ⇒
This shows that the altitude is changing faster y′ = 6[−sin (e− 0.5 x)](−0.5) e− 0.5 x = 3e− 0.5 x sin (e− 0.5 x)
at 5 psi than it is at 10 psi. 10 10
11. f ( x ) = 10 ln (7 x ) ⇒ f ′( x ) = ⋅7 =
d. A(p) = 0 7x x
63 − 23.5 ln p = 0 9 9
−23.5 ln p = −63 12. g( x ) = 9 ln 4 x ⇒ g ′( x ) = ⋅4 =
4x x
ln p = 2.6808…
18 54
p = 14.5975… 13. T = 18 ln x 3 ⇒ T ′ = 3 (3 x 2 ) =
The fact that A(p) is negative for all values of x x
p greater than 14.5975… means that if the air 1000 700
14. P = 1000 ln x 0.7 ⇒ P ′ = 0.7 0.7 x −0.3 =
pressure is above 14.5975 psi, then the plane x x
must be beneath sea level. 15. y = 3 ln (cos 5x) ⇒
4. x = 3000e0.05 y 3
y′ = ( − sin 5 x )5 = −15 tan 5 x
a. ln x = ln (3000e0.05 y) cos 5 x
ln x = ln 3000 + ln e0.05 y 16. y = 11 ln (sin 0.2x) ⇒
ln x − ln 3000 = 0.05y 11
y′ = (cos 0.2 x ) 0.2 = 2.2 cot 0.2 x
1
y= (ln x − ln 3000) sin 0.2 x
0.05 17. u = 6 ln (sin x0.5 ) ⇒
y = 20 ln x – 20 ln 3000 6
b. y(3000) = 20 ln 3000 − 20 ln 3000 = 0 u′ = (cos x 0.5 ) 0.5 x −0.5 = 3 x −0.5 cot x 0.5
sin x 0.5
y(4000) = 20 ln 4000 − 20 ln 3000 =
18. v = 0.09 ln (cos x 8 ) ⇒
5.7536…
y(5000) = 20 ln 5000 − 20 ln 3000 = 0.09
v′ = ( − sin x 8 )8 x 7 = −0.72x7 tan x8
10.2165… cos x 8
y(6000) = 20 ln 6000 − 20 ln 3000 =
1 x
13.8629… 19. r ( x ) = ln e x ⇒ r ′( x ) = ⋅e = 1
Number of years to get from $3000 to $4000: ex
Not surprising because we could have first used
5.7536…
the fact that natural log and exp are inverses:
Number of years to get from $4000 to $5000:
r(x) = ln ex = x ⇒ r′ (x) = 1
4.4628…
Number of years to get from $5000 to $6000: 20. c(x) = eln x = x ⇒ c ′ (x) = 1
3.6464… c ′ (2) = 1, c ′ (3) = 1, c ′ (4) = 1
The time intervals decrease as the amount of 21. f (x) = 3x ⇒ f ′ ( x ) = ( ln 3 ) 3 x
money increases because when there is more 22. g (x) = 0.007x ⇒ g′(x) = (ln 0.007) 0.007x
money in the account, it takes less time to
23. y = 1.6cos x ⇒ y′ = (ln 1.6)1.6cos x (−sin x) =
earn the given amount of interest. −ln 1.6 sin x (1.6cos x)
20
c. y = 20 ln x − 20 ln 3000 ⇒ y ′ = yr/$ 24. y = sin 5x ⇒ y′ = cos 5x (ln 5)5x
x
y′(3000) = 0.0066… dy 1 5
25. y = ln x 5 ⇒ = ⋅ 5 x 4 = = 5 x −1 ⇒
y′(4000) = 0.005 dx x 5 x
y′(5000) = 0.004 d2y − 5
= −5 x −2 = 2
y′(6000) = 0.0033… dx 2 x
This shows that the number of years it takes dy d2y
to earn each dollar decreases as the amount of 26. y = e 7 x ⇒ = 7e 7 x ⇒ 2 = 49e 7 x
dx dx
money increases.
27. y = e − 0.7 x
⇒ y′ = −0.7e − 0.7 x
⇒ y″ = 0.49e− 0.7 x
5. f (x) = 5e3x ⇒ f ′ (x) = 15e3x
6. f (x) = 7e− 6x ⇒ f ′ (x) = −42e− 6x

Calculus Solutions Manual Problem Set 3-9 45


© 2005 Key Curriculum Press
28. y = ln 8 x ⇒ y ′ =
1 1
⋅ 8 = = x −1 ⇒ 0.4 x 2 – x − 0.6
c. m( x ) =
8x x x–3
−1
y ′′ = −1x −2 =
m (x )

x2 2

29. f ′ (x) = 12e ⇒ f ( x ) = 6 e 2 x + C


2x
1
30. y′ = 5x ln 5 ⇒ y = 5x + C
x
3

Problem Set 3-10


d. Line: y = 1.4x + 1.4
Review Problems
y
R0. Answers will vary.
R1. a.
4
x Average rate of change from 2 to x
f (2) − f (1.97)
x

1.97 = 11.82 3
0.03
f (2) − f (1.98) e. The line is tangent to the graph.
1.98 = 11.88 f. Yes, f does have local linearity at x = 3.
0.02
Zooming in on the point (3, 5.6) shows that
f (2) − f (1.99)
1.99 = 11.94 the graph looks more and more like the line.
0.01 R3. a.
f (2.01) − f (2) y
2.01 = 12.06 y2
0.01 50
y1
f (2.02) − f (2)
2.02 = 12.12 1 x
0.02
f (2.03) − f (2)
2.03 = 12.18
0.03
b. See the graph in part a.
The derivative of f at x = 2 is approximately 12. c. The y1 graph has a high point or a low point
f ( x ) − f (2) at each x-value where the y2 graph is zero.
b. r ( x ) =
x−2 d.
r(2) is of the form 00 . y
lim r ( x ) appears to be 12.
x →2 p

x 3 − 8 ( x − 2)( x 2 + 2 x + 4)
20
c. r ( x ) = =
x−2 ( x − 2) 1 t
= x 2 + 2 x + 4, x ≠ 2 p´
lim r ( x ) = lim x 2 + 2 x + 4 because x ≠ 2.
x →2 x →2
Take the numerical derivative at t = 3, 6,
∴ lim r ( x ) = 12 and 0.
x →2
p′(3) ≈ −2.688… . Decreasing at about
d. The answers in parts a, b, and c are the same.
2.69 psi/h when t = 3.
f ( x ) − f (c ) p′(6) ≈ −1.959… . Decreasing at about
R2. a. f ′(c) = lim
x →c x–c 1.96 psi/h when t = 6.
b. f (x) = 0.4x − x + 5
2 p′(0) ≈ −3.687… . Decreasing at about
3.69 psi/h when t = 0.
0.4 x 2 − x + 5 − 5.6
f ′(3) = lim The units are psi/h. The sign of the pressure
x →3 x–3 change is negative because the pressure is
( x – 3)(0.4 x + 0.2)
= lim decreasing. Yes, the rate of pressure change is
x →3 x–3 getting closer to zero.
= lim (0.4 x + 0.2) = 1.4 R4. a. See the text for the definition of derivative.
x →3
b. Differentiate

46 Problem Set 3-10 Calculus Solutions Manual


© 2005 Key Curriculum Press
c. If y = x n, then y′ = nx n− 1. e. i. y = −0.01t3 + 0.9t2 − 25t + 250
d. See solution to Problem 35 in Problem Set dy
v= = −0.03t 2 + 1.8t − 25
3-4. dt
e. See the proof in Section 3-4. dv
a= = −0.06t + 1.8
dy dt
f. is pronounced “d y, d x.”
dx ii. a( 15) = −0.06(15) + 1.8 = 0.9 (km/s)/s
d v(15) = −0.03(152) + 1.8(15) − 25
( y) is pronounced “d, d x, of y.”
dx = −4.75 km/s
Both mean the derivative of y with respect to x. The spaceship is slowing down at t = 15
63 4 / 5 because the velocity and the acceleration
g. i. f ( x ) = 7 x 9/ 5 ⇒ f ′( x ) = x have opposite signs.
5
iii. v = −0.03t2 + 1.8t − 25 = 0
x2
ii. g( x ) = 7 x –4 − − x+7⇒ By using the quadratic formula or the
6 solver feature of your grapher,
x
g′( x ) = −28 x –5 − − 1 t = 21.835… or t = 38.164… .
3 The spaceship is stopped at about 21.8
iii. h(x) = 73 ⇒ h′ (x) = 0 and 38.2 seconds.
h. f ′(32) = 635 (32) 4 / 5 = 201.6 exactly. The iv. y = −0.01t3 + 0.9t2 − 25t + 250 = 0
numerical derivative is equal to or very close By using TRACE or the solver feature of
to 201.6. your grapher, t = 50.
v(50) = −10
y
i. 4 Because the spaceship is moving at
f
10 km/s when it reaches the surface, it is
a crash landing!
x
R6. a.
4
f´ y

1 derivative cosine

x
1

dx
R5. a. v = or x ′(t ).
dt
dv d2x b. The graph of the derivative is the same as the
a= or v ′(t ), a = 2 or x ′′(t )
dt dt sine graph but inverted in the y-direction.
d2y Thus, (cos x )′ = − sin x is confirmed.
b. means the second derivative of y with c. −sin 1 = −0.841470984…
dx 2
respect to x. Numerical derivative ≈ −0.841470984…
The two are very close!
y = 10x 4 ⇒ y′ = 40x 3 ⇒ y″ = 120x 2
d. Composite function
c. f ′( x ) = 12 x 3 ⇒ f ( x ) = 3 x 4 + C. f (x) is
the antiderivative, or the indefinite integral, f′(x) = −2x sin (x2)
of f (x). dy dy du
R7. a. i. = ⋅
d. The slope of y = f ( x ) is determined by the dx du dx
value of f ′( x ). So the slope of y = f ( x ) at ii. f (x) = g(h(x)) ⇒ f ′(x) = g ′(h(x)) ⋅ h ′(x)
x = 1 is f ′(1) = −1, at x = 5 is f ′(5) = 3, and iii. The derivative of a composite function is
at x = −1 is f ′( −1) = 0. the derivative of the outside function with
y f f´
respect to the inside function times the
5
derivative of the inside function with
respect to x.
b. See the derivation in the text. This derivation
x constitutes a proof. ∆u must be nonzero
5
throughout the interval.

Calculus Solutions Manual Problem Set 3-10 47


© 2005 Key Curriculum Press
c. i. f (x) = (x2 − 4)3 d. cos x = sin (π /2 − x)
f ′(x) = 3(x 2 − 4)2 ⋅ 2x = 6x(x2 − 4)2 cos′ x = cos (π/2 − x) (−1)
ii. f (x) = x 6 − 12x 4 + 48x 2 − 64 = −sin x, Q .E .D .
f′(x) = 6x 5 − 48x 3 + 96x e. d(t) = C + A cos B(t − D)
Expanding the answer to part i gives C = 180, A = 20
f′(x) = 6x5 − 48x3 + 96x, which checks. D = 0 for cosine because hand starts at a high
d. i. f′(x) = −3x 2 sin x 3 point.
B = 2π/60 = π/30 because period is 60 s.
ii. g′(x) = 5 cos 5x π
iii. h′(x) = 6 cos5 x (−sin x) d (t ) = 180 + 20 cos t
30
= −6 sin x cos5 x 2π π
iv. k ′(x) = 0 d ′(t ) = − sin t
3 30
e. f ′(x) = 12 cos 3x ⇒ f ″ (x) = At 2, t = 10: d′(10) ≈ −1.81 cm/s
12(−sin 3x)3 = −36 sin 3x At 3, t = 15: d′(15) ≈ −2.09 cm/s
f ′(x) = 12 cos 3x ⇒ f (x) = 4 sin 3x + C At 7, t = 35: d′(35) ≈ 1.05 cm/s
f (x) is the second derivative of f (x). At the 2 and 3, the tip is going down, so the
f (x) is the antiderivative, or indefinite distance from the floor is decreasing, which is
integral, of f (x). implied by the negative derivatives. At the 7,
f. W = 0.6x3 and dx/dt = 0.4 the tip is going up, as implied by the positive
dW dW dx derivative.
= ⋅ = 1.8 x 2 ⋅ 0.4 = 0.72 x 2
dt dx dt R9. a. p( x ) = 100e −0.1x ⇒ p ′( x ) = 100( −0.1)e −0.1x
If x = 2, W = 0.6 ⋅ 23 = 4.8 lb = −10e− 0.1 x
dW/dt = 0.72(22) = 2.88 p′(0) = −10
The shark is gaining about 2.88 lb/day. p′(10) = −3.6787…
If x = 10, W = 0.6 ⋅ 103 = 600 lb. p′(20) = −1.3533…
dW/dt = 0.72(102) = 72 The rates are negative because the amount of
The shark is gaining about 72 lb/day. medication in your body is decreasing.
The chain rule is used to get dW/dt from To find the biological half-life, find x such that
dW/dx by multiplying the latter by dx/dt. 1
sin x p( x ) = p(0) = 50
R8. a. lim =1 2
x→0 x 100e −0.1x = 50
1
x (sin x)/x e −0.1x =
2
−0.05
−0.1x = ln  
0.99958338541… 1
−0.04 0.99973335466…  2
−0.03 1
0.99985000674… x = −10 ln
2
−0.02 0.99993333466…
x = 6.9314…
–0.01 0.99998333341… The half-life is 6.9314… h.
0.00 undefined p(2(6.9314…)) = p(13.8629…) =
0.01 0.99998333341… 100e− 0.1(13.8629…) = 25
0.02 0.99993333466… After two half-lives have elapsed, 25% of the
medicine remains in your body.
0.03 0.99985000674…
b. i. f (x) = 5e2x ⇒ f ′(x) = 5(2)e2x = 10e2x
0.04 0.99973335466…
dy
0.05 0.99958338541… ii. y = 7 x ⇒ = (ln 7)7 x
dx
The values of (sin x)/x approach 1 as x d 1
approaches 0. iii. [ln (cos x )] = ( − sin x ) = − tan x
dx cos x
b. See the text for the statement of the squeeze dy 1
iv. y = ln x 8 = 8 ln x ⇒ = 8 ⋅ = 8 x −1 ⇒
theorem. Squeeze (sin x)/x between cos x and dx x
sec x. d2y − 8
= −8 x −2 = 2
c. See the proof in Section 3-8 of the text. dx 2 x
c. f ′( x ) = 12e 3x ⇒ f ( x ) = 4e 3x + C

48 Problem Set 3-10 Calculus Solutions Manual


© 2005 Key Curriculum Press
d. Chapter Test
y y3
y1
T1. See the definition of derivative in Section 3-2
3 y2
or 3-4.
x
3 T2. Prove that if f (x) = 3x4, then f ′ (x) = 12x3.

Proof:
f ( x + h) – f ( x ) 3( x + h) 4 − 3 x 4
y1 = ex is the inverse of y2 = ln x, so y1 is a f ′( x ) = lim = lim
h→ 0 h h→ 0 h
reflection of y2 across the line y = x.
3 x + 12 x h + 18 x h + 12 xh 3 + 3h 4 − 3 x 4
4 3 2 2

Concept Problems = lim


h→0 h
C1. a. f (x) = x7, g(x) = x9. So h(x) = f(x) ⋅ g(x) = x 16. = lim (12 x 3 + 18 x 2 h + 12 xh 2 + 3h 3 ) = 12 x 3 ,
h→ 0
b. h′(x) = 16x15 Q .E .D .
c. f ′(x) = 7x 6, g′(x) = 9x 8. So f ′(x) ⋅ g′(x) = T3. If you zoom in on the point where x = 5, the
63x14 ≠ h ′(x). graph appears to get closer and closer to the
d. h′(x) = f ′ (x) ⋅ g(x) + f (x) ⋅ g′(x) = tangent line. The name of this property is local
7x 6 ⋅ x 9 + x 7 ⋅ 9x 8 = 16x 15 linearity.
x – sin 2 x
C2. a. f ( x ) = . f (0) has the form 0/0, y
sin x
which is indeterminate. f is discontinuous at 5
x = 0 because f (0) does not exist.
b. By graph (below) or by TABLE , f (x) seems to Slope = 2
approach −1 as x approaches zero. Define f (0) x

to be −1. 5

f (x )
5 T4. Amos substituted before differentiating instead
of after. Correct solution is f (x) = 7x ⇒
f ′ (x) = 7 ⇒ f ′ (5) = 7.
x
π
T5. f (x) = (7x + 3)15 ⇒ f ′ (x) = 105(7x + 3)14
T6. g(x) = cos (x5) ⇒ g ′ (x) = −5x 4 sin x 5
d 1
c. Conjecture: The function is differentiable at T7. [ln (sin x )] = ⋅ cos x = cot x
dx sin x
x = 0. The derivative should equal zero
because the graph is horizontal at x = 0. T8. y = 36x ⇒ y′ = (ln 3)36x(6) = 6(ln 3)36x
f ( x ) – f (0) T9. f (x) = cos (sin5 7x) ⇒
d. f ′(0) = lim
h→0 x–0 f ′(x) = −sin (sin5 7x) ⋅ 5 sin4 7x ⋅ cos 7x ⋅ 7
x – sin 2 x = −35 sin (sin5 7x) sin4 7x cos 7x
– (–1)
= lim sin x T10. y = 60x 2/3 − x + 25 ⇒ y′ = 40x− 1/3 − 1
x→0 x
x – sin 2 x + sin x d2y
= lim T11. y = e 9x ⇒
dy
= 9e 9x ⇒ 2 = 81e 9x
x→0 x sin x dx dx
Using TABLE for numerator, denominator, and
T12. y′ ≈ 0.6 (Function is y = −3 + 1.5x, for which
quotient shows that the numerator goes to
the numerical derivative is 0.6081… .)
zero faster than the denominator. For instance,
if x = 0.001, T13. y = 3 + 5x − 1.6
1.1666 K × 10 –9 v(x) = 5(−1.6)x− 2.6 = −8x − 2.6
quotient = = 0.00116 K
9.999K × 10 –7 a(x) = −8(−2.6)x− 3.6 = 20.8x− 3.6
Thus, the limit appears to be zero. (The limit
Acceleration is the second derivative of the
can be found algebraically to equal zero by
displacement function.
l’Hospital’s rule after students have studied
Section 6-5.) T14. f ′ (x) = 72x 5/4 ⇒ f (x) = 32x 9/4

Calculus Solutions Manual Problem Set 3-10 49


© 2005 Key Curriculum Press
T15. f ′ (x) = 5 sin x and f (0) = 13 Proof:
f (x) = −5 cos x + C d x 5 x +h − 5 x
13 = −5 cos 0 + C ⇒ C = 18 (5 ) = lim Definition of derivative.
dx h→ 0 h
f (x) = −5 cos x + 18
5h − 1
T16. f (x) = cos 3x ⇒ f ′ (x) = −3 sin 3x = 5 x lim Factor out 5x.
h→0 h
f ′ (5) = −3 sin 15 = −1.95086… = 5x · ln 5 Evaluate.
Decreasing at 1.95… y-units per x-unit.
T19. v(t) = 251(1 − 0.88 ) t

sin x a(t) = 251[− ln (0.88)] 0.88t = −251(ln 0.88)0.88t


T17. f ( x ) =
x a(10) = −251(ln 0.88)(0.88)(10) = 8.9360…
Numerical derivative gives 8.9360… as well.
f (x )
T20. If the velocity and the acceleration have opposite
1 signs for a particular value of t, then the object is
slowing down at that time.
x T21. a. v(t) = t1.5 + 3 ⇒ a(t) = 1.5t0.5
1
 t 2.5 
b. d (t ) =   + 3t + C
 2.5 
As x approaches zero, f (x) approaches 1. d(1) = 20
The squeeze theorem states: 12.5
+ 3(1) + C = 20
If (1) g(x) ≤ h(x) for all x in a neighborhood of c, 2.5
(2) lim g( x ) = lim h( x ) = L, and (3) f is a 3.4 + C = 20
x →c x →c C = 16.6
function for which g(x) ≤ f (x) ≤ h(x) for all x in ∴ d(t) = 0.4t2.5 + 3t + 16.6
that neighborhood of c, then lim f ( x ) = L.
x →c c. d(9) − d(1) = 120.8
T18. This represents the displacement between the
first and ninth seconds.
5h − 1 2π
h T22. a. c(t ) = 300 + 2 cos t⇒
h 365
–0.0003 1.6090… 4π 2π
c ′(t ) = − sin t
–0.0002 1.6091… 365 365
4π 2π
–0.0001 1.6093… b. c′(273) = − sin  ⋅ 273
365  365 
0 undefined
= 0.03442… ppm/day
0.0001 1.6095…
0.03442 K 1
0.0002 1.6097… c. Rate is (6 × 1015 ) ⋅ ⋅ =
1, 000, 000 24 ⋅ 60 ⋅ 60
0.0003 1.6098…
2390.6627… , which is approximately 2390
ln 5 = 1.6094… . The table shows that tons per second!
5h − 1 T23. Answers will vary.
lim = ln 5.
h→0 h

50 Problem Set 3-10 Calculus Solutions Manual


© 2005 Key Curriculum Press
Chapter 4—Products, Quotients, and Parametric Functions

Problem Set 4-1 6. You’ll see in Section 4-2 that


1. f (x) = 3 cos x ⇒ f ′ (x) = −3 sin x p′(x) = f ′ (x)g(x) + f (x)g′(x).
g(x) = 2 sin x ⇒ g′(x) = 2 cos x ∴ p′(2) = (–3 sin 2)(2 sin 2) + (3 cos 2)(2 cos 2) =
2. –3.9218… , which agrees with Problem 2.
p (x ) You’ll see in Section 4-3 that
6

f ′ ( x ) g( x ) – f ( x ) g ′ ( x )
q ′( x ) =
x [ g( x )]2
10
(–3 sin 2)(2 sin 2) – (3 cos 2)(2 cos 2)
∴ q ′( 2 ) = =
(2 sin 2)2
−1.8141… , which agrees with Problem 3.
p′(2) ≈ −3.9218… Problem Set 4-2
p(x) is decreasing at x = 2 because p ′(2) < 0.
This fact corresponds with the graph, which 3 –1/ 4
Q1. y ′ = x Q2. y′ = 1/x
slopes steeply in the negative direction at x = 2. 4
f ′ (2) ⋅ g′(2) = (−3 sin 2)(2 cos 2) = 2.2704… dy d
So p′ (2) ≠ f ′ (2) ⋅ g′ (2). Q3. = −30(5 x − 7) –7 Q4. (sin 2 x ) = 2 cos 2 x
dx dx
3.
q (x )
Q5. v′ = −3 cos2 t sin t Q6. L′ = 2m + 5
6
Q7. y = sin x + C 3
Q8. y ′ ≈ −3
x Q9. 4 ft/s Q10. B
10
1. f ( x) = x cos x ⇒ f ′ (x) = 3x 2 cos x – x 3 sin x
3

2. f ( x) = x 4 sin x ⇒ f ′ (x) = 4x 3 sin x + x 4 cos x


3. g(x) = x1.5 e2x ⇒ g′ (x) = 1.5x0.5 e2x + 2x1.5 e2x
q is the cotangent function. 4. h(x) = x− 6.3 ln 4x ⇒
q′ (2) = −1.8141… h′ (x) = −6.3x− 7.3 ln 4x + x − 6.3 (1/4x)4
q( x ) is decreasing at x = 2. = −6.3x− 7.3 ln 4x + x − 7.3
f ′ (2)/g′ (2) = (−3 sin 2)/(2 cos 2) = 3.2775… 5. y = x7(2x + 5)10 ⇒
So q′ (2) ≠ f ′ (2)/g′ (2). dy/dx = 7x6(2x + 5)10 + x7(10)(2x + 5)9 ⋅ 2
4. = x6(2x + 5)9(34x + 35)
y
6. y = x (3x + 7)9 ⇒
8

t = 2 here
2
dy/dx = 8x7(3x + 7)9 + x8(9)(3x + 7)8(3)
x
= x7(3x + 7)8 (51x + 56)
3
7. z = ln x sin 3x ⇒
z′ = (1/x) sin 3x + 3 ln x cos 3x
8. v = e5x cos 2x ⇒ v′ = 5e5x cos 2x − 2e5x sin 2x
The geometric figure seems to be an ellipse. 9. y = (6x + 11)4(5x − 9)7 ⇒
5. See graph in Problem 4. y ′ = 4(6x + 11)3(6)(5x − 9)7
∆x = 3 cos 2.1 − 3 cos 1.9 = −0.54466… + (6x + 11)4(7)(5x − 9)6(5)
∆y = 2 sin 2.1 − 2 sin 1.9 = −0.16618… = (6x + 11)3(5x − 9)6(330x + 169)
dy ∆y –0.16618K
≈ = = 0.3051K
dx ∆x –0.54466 K 10. y = (7x – 3)9(6x − 1)5 ⇒
dy/dt 2 cos 2 y′ = 9(7x – 3)8(7)(6x − 1)5
At t = 2, = = 0.3051 K ,
dx/dt –3 sin 2 + (7x − 3)9(5)(6x − 1)4(6)
which agrees with the difference quotient. = (7x − 3)8(6x − 1)4(588x − 153)

Calculus Solutions Manual Problem Set 4-2 51


© 2005 Key Curriculum Press
11. P = (x2 − 1)10(x2 + 1)15 ⇒ Proof:
P′ = 10(x2 − 1)9(2x)(x2 + 1)15 y = uvw = (uv)w
+ (x2 − 1)10(15)(x2 + 1)14(2x) ∴ y′ = (uv)′w + (uv)w′ = (u′v + uv′)w + (uv)w′
= 10x(x2 − 1)9(x2 + 1)14[2(x2 + 1) + 3(x2 – 1)] = u′vw + uv′w + uvw′, Q .E .D .
= 10x(x2 − 1)9(x2 + 1)14(5x2 − 1)
22. If yn = u1u2 u3 …un where u1…un are differentiable
12. P(x) = (x3 + 6)4(x3 + 4)6 ⇒ functions of x, then
P′ (x) = 4(x3 + 6)3(3x2)(x3 + 4)6 yn′ = u1′u2 u3 Kun + u1u2′ u3 Kun
+ (x3 + 6)4 ⋅ 6(x3 + 4)5 ⋅ 3x2 + u1u2 u3′ Kun + L + u1u2 u3 Kun′ K .
= 6x2(x3 + 6)3(x3 + 4)5[2(x3 + 4) + 3(x3 + 6)]
23. z = x 5 cos6 x sin 7x ⇒
= 6x2(x3 + 6)3(x3 + 4)5(5x3 + 26)
z′ = 5x4 cos6 x sin 7x + x5 6 cos5 x (−sin x) ⋅
13. a( t) = 4 sin 3t cos 5t ⇒ sin 7x + x 5 cos6 x ⋅ 7 cos 7x
a′(t) = 12 cos 3t cos 5t + 4 sin 3t(−5 sin 5t) = 5x 4 cos6 x sin 7x − 6x 5 cos5 x sin x sin 7x
= 12 cos 3t cos 5t − 20 sin 3t sin 5t + 7x5 cos6 x cos 7x
14. v = 7 cos 2t sin 6t ⇒
24. y = 4x 6 sin3 x cos 5x ⇒
v′ = −14 sin 2t sin 6t + 7 cos 2t(6 cos 6t)
y′ = 24x5 sin3 x cos 5x + 4x6 3 sin2 x cos x ⋅
= −14 sin 2t sin 6t + 42 cos 2t cos 6t
cos 5x + 4x6 sin3 x(−5 sin 5x)
15. y = cos (3 sin x) ⇒ = 24x5 sin3 x cos 5x + 12x6 sin2 x cos x ⋅
y′ = −sin (3 sin x) · 3 cos x cos 5x − 20x6 sin3 x sin 5x
= −3 sin (3 sin x) cos x 25. y = x4 (ln x)5 sin x cos 2x ⇒
16. y = sin (5 cos x) ⇒ y′ = 4x3(ln x)5 sin x cos 2x + x4 ⋅ 5(ln x)4(1/x) ⋅
y′ = cos (5 cos x) · (−5 sin x) sin x cos 2x + x4(ln x)5 cos x cos 2x
= −5 cos (5 cos x) sin x
+ x4(ln x)5 sin x ⋅ (−2 sin 2x)
17. y = cos e6x ⇒ = 4x3(ln x)5 sin x cos 2x + 5x3(ln x)4 sin x ⋅
dy/dx = 6e6x(−sin e6x) = −6e6x sin e6x ⇒ cos 2x + x4(ln x)5 cos x cos 2x
d2y/dx2 = −36e6x sin e6x − 6e6x(6e6x cos e6x) − 2x4(ln x)5 sin x sin 2x
= −36e6x sin e6x − 36e12x cos e6x
26. u = x 5 e2x cos 2x sin 3x ⇒
18. y = ln (sin x) ⇒ dy/dx = (1/sin x) cos x u′ = 5x 4 e2x cos 2x sin 3x + x5 · 2e2x ·
= (cos x)(sin x)− 1 ⇒ d 2y/dx2 cos 2x sin 3x + x 5 e2x(−2 sin 2x) sin 3x
= (cos x) [−(sin x)]− 2 (cos x) + (−sin x) ⋅ (sin x)− 1 + x 5 e2x cos 2x · 3 cos 3x
− cos 2 x = 5x 4 e2x cos 2x sin 3x + 2x52e2x cos 2x sin 3x
= + 1 = − cot 2 x + 1 = − csc 2 x
− sin 2 x − 2x5 e2x sin 2x sin 3x + 3x5 e2x cos 2x cos 3x
19. z = x3(5x − 2)4 sin 6x ⇒ 27. a. y(t) = 4 + 3e− 0.1 t cos π t
z′ = 3x2(5x − 2)4 sin 6x + x3[(4)(5x − 2)3(5) sin 6x v(t) = y′(t) = 3(−0.1)e− 0.1 t cos π t
+ (5x − 2)4(6 cos 6x)] + 3e− 0.1 t(− π sin π t)
= 3x2(5x − 2)4 sin 6x + 20x3(5x − 2)3 sin 6x = e− 0.1 t(−0.3 cos π t − 3π sin π t )
+ 6x3(5x − 2)4 cos 6x
b. v(2) = e− 0.2 (−0.3 cos 2π − 3π sin 2π)
20. u = 3x5(x2 − 4) cos 10x = e−0.2(−0.3 − 0) = −0.2456…
u′ = 15x4(x2 − 4) cos 10x + 3x5[2x cos 10x There is not a high point at t = 2 because
+ (x2 − 4) ⋅ (−10 sin 10x)] v(2) ≠ 0.
= 15x4 ⋅ (x2 − 4) cos 10x + 6x6 cos 10x v(t) = 0 ⇒ e− 0.1 t(−0.3 cos π t − 3π sin π t) = 0
− 30x5(x2 − 4) sin 10x ⇒ −0.3 cos π t = 3π sin π t ⇒ t = 1.9898…
21. If y = uvw, where u, v, and w are differentiable 28. a. y(t) = t sin t ⇒ v(t) = y′(t) = sin t + t cos t
functions of x, then y′ = u′vw + uv′w + uvw′. Graph confirms Figure 4-2d.

52 Problem Set 4-2 Calculus Solutions Manual


© 2005 Key Curriculum Press
b. v exceeds 25. By the derivative of a product property,
fk′+1 ( x ) = ( x k )′( x ) + ( x k )( x )′ = ( x k )′( x ) + x k .
Substituting for (xk)′ from the induction
25
hypothesis,
fk′+1 ( x ) = ( kx k −1 )( x ) + x k = kx k + x k = ( k + 1) x k =
v
(k + 1)x ( k+ 1 ) − 1, completing the induction.
∴ fn′ ( x ) = nx n−1 for all integers ≥ 1, Q.E.D.
25
32. Way 1: y = (x + 3)8(x − 4)8
c. In 1940, wind-induced vibrations in the y′ = 8(x + 3)7 · (x − 4)8 + (x + 3)8 · 8(x − 4)7
Tacoma Narrows Bridge increased in = 8(x + 3)7(x − 4)7(x + 3 + x − 4)
amplitude until the bridge collapsed. = 8(x + 3)7(x − 4)7(2x − 1)
29. Prove that the derivative of an odd function is an Way 2: y = (x2 − x − 12)8
even function and that the derivative of an even y′ = 8(x 2 − x − 12)7(2x − 1)
function is an odd function. = 8(x + 3)7(x − 4)7(2x − 1), which checks.
33. a.
Proof: f (x)

5
For any function, the chain rule gives f
d x
f ( − x ) = f ′( − x ) ⋅ ( −1) = − f ′( − x ). 1
dx

For an odd function,
d d
f ( − x ) = [– f ( x )] = − f ′( x ).
dx dx b. f ′ (x) = 3x 2 sin x + x 3 cos x
∴ −f ′ (−x) = −f ′ (x) or f ′ (−x) = f ′ (x), The graph in part a is correct.
and the derivative is an even function. c. The numerical derivative graph duplicates the
For an even function, algebraic derivative graph, as in part a, thus
d d
f (− x ) = f ( x ) = f ′( x ). showing that the algebraic derivative is right.
dx dx
34. a.
∴ −f ′ (−x) = f ′ (x) or f ′ (−x) = −f ′ (x),
f (x )
and the derivative is an odd function, Q.E.D.
599,128
30. f (x) = 2 sin x cos x ⇒ 500,000

f ′ (x) = 2 cos x · cos x + 2 sin x(−sin x) x

= 2 cos2 x − 2 sin2 x = 2 cos 2x –1.5


1/9
1.4

g(x) = sin 2x ⇒ g′(x) = 2 cos 2x = f ′ (x), Q.E.D. –500,000

f(0) = 0 and g(0) = 0


∴ f (x) = 2 sin x cos x = sin 2x = g(x), by the
b. f ′ (x) = 4(5x − 7)3(5) · (2x + 3)5
uniqueness theorem for derivatives, Q.E.D.
+ (5x − 7)4 · (5)(2x + 3)4(2)
f (x) = cos2 x − sin2 x ⇒
= 10(5x − 7)3(2x + 3)4[2(2x + 3)
f ′ (x) = 2 cos x(−sin x) − 2 sin x cos x
+ 5x − 7]
= −4 sin x cos x = −3 sin 2x
= 10(5x − 7)3(2x + 3)4(9x − 1)
g(x) = cos 2x ⇒
g′(x) = (−2 sin 2x) = −sin 2x = f ′ (x), Q.E.D. c. f ′ (x) = 0 ⇔ 5x − 7 = 0 or 2x + 3 = 0
f(0) = 1 and g(0) = 1 or 9x − 1 = 0
∴ f (x) = cos2 x − sin2 x = cos 2x = g(x) by the ∴ x = 7/5 = 1.4, or x = −3/2 = −1.5,
uniqueness theorem, Q.E.D. or x = 1/9
See graph in part a.
31. Prove that if fn(x) = xn, then fn′ ( x ) = nx n−1 for all
integers ≥ 1. d. f (1.4) = 0, f (−1.5) = 0, f (1/9) = 599,127.6… .
See graph in part a.
Proof (by induction on n): e. False. The graph may have a point where it
If n = 1, then f1(x) = x1 = x, which implies that levels off and then continues changing in the
f1′( x ) = 1 = 1x 0, which anchors the induction. same direction, as at x = −1.5 in part a.
Assume that for some integer n = k > 1, 35. a. A = L W
fk′ ( x ) = kx k −1 . dA dL
= ⋅W + L ⋅
dW
For n = k + 1, fk+ 1(x) = xk+ 1 = (xk)(x). dt dt dt

Calculus Solutions Manual Problem Set 4-2 53


© 2005 Key Curriculum Press
dW cos 12 x
b. W = 2 + 2 cos t ⇒ = −2 sin t 6. y = ⇒
dt sin 18 x
dL –12 sin 12 x sin 18 x – 18 cos 12 x cos 18 x
L = 3 + 2 sin 2t ⇒ = 4 cos 2t y′ =
dt sin 2 18 x
dA
= ( 4 cos 2t )(2 + 2 cos t ) 3x − 7
dt 7. y = ⇒
+ (3 + 2 sin 2t )( −2 sin t ) 6x + 5
At t = 4, dA/dt = 7.132… , so A is increasing. 3(6 x + 5) – (3 x − 7)(6) 57
y′ = =
At t = 5, dA/dt = −4.949… , so A is (6 x + 5)
2
(6 x + 5)
2

decreasing. 10 x + 9
8. y = ⇒
Problem Set 4-3 5x − 3
10 ⋅ (5 x − 3) − (10 x + 9) ⋅ 5 –75
Q1. 1066x1065 y′ = =
(5 x − 3) 2 (5 x – 3)2
Q2. f (x) = 12x5 + C
(8 x + 1)6 dz
Q3. y′ = 3x2 sin x + x3 cos x 9. z = ⇒
(5 x – 2 ) 9 dx
Q4. dy/dx = −sin (x7) ⋅ 7x6 = −7x6 sin (x7) 6(8 x + 1)5 (8) ⋅ (5 x − 2) 9 − (8 x + 1)6 ⋅ (9)(5 x − 2)8 (5)
Q5. f ′ (x) = 0 (derivative of a constant) =
(5 x − 2)18
Q6. 54e9t (8 x + 1) 5 (120 x + 141)
Q7. See the text for the definition of derivative. =−
(5 x – 2)10
Q8. Instantaneous rate of change at a given x
( 4 x – 1) 7 dA
Q9. (x − 3)4(x − 3 + 2x) = 3(x − 3)4(x − 1) 10. A = 4 ⇒
(7 x + 2) dx
Q10.
7( 4 x − 1)6 ( 4) ⋅ (7 x + 2) 4 − ( 4 x − 1) 7 ⋅ 4( 7 x + 2)3 ( 7)
4 y
=
( 7 x + 2 )8
28( 4 x – 1) ( 7 x + 2)3 [(7 x + 2) − ( 4 x − 1)]
6
=
x ( 7 x + 2 )8
4
28( 4 x – 1)6 ( 7 x + 2)3 (3 x + 3)
=
( 7 x + 2 )8
84( 4 x – 1)6 ( x + 1)
=
x3 3 x 2 sin x – x 3 cos x ( 7 x + 2)5
1. f ( x ) = ⇒ f ′( x ) =
sin 2 x
3 3 3
sin x ex 3 x 2 e x sin x − e x cos x
11. Q = ⇒ Q′ =
x4 4 x 3 cos x + x 4 sin x sin x sin 2 x
2. f ( x ) = ⇒ f ′( x ) =
cos x cos 2 x ln x 4
12. r = ⇒
cos3 x cos x
3. g( x ) = ⇒
ln x 4 x 3 (1/ x 4 )cos x − (ln x 4 )( − sin x )
r′ =
3 cos 2 x ( − sin x ) ⋅ ln x − cos3 x ⋅ (1/ x ) cos 2 x
g ′( x ) =
(ln x )2 ( 4 cos x )/ x + (ln x 4 ) sin x 4 cos x + x ln x 4 sin x
= =
−3 ln x sin x cos 2 x − (cos3 x )/ x cos 2 x x cos 2 x
=
(ln x )2 d
13. (60 x –4 / 3 ) = −80 x −7/3
sin 5 x dx
4. h( x ) = ⇒
e3 x d
14. (24 x –7/ 3 ) = −56 x −10/3
5 sin 4 x cos x e 3 x − sin 5 x ⋅ 3e 3 x dx
h ′( x ) =
(e 3 x ) 2 Problems 15–22 and 25–26 can be done using either
5 sin x cos x − 3 sin 5 x
4
the power rule or the quotient rule.
=
e3 x 12 –36
sin 10 x 15. r ( x ) = = 12 x −3 ⇒ r ′( x ) = −36 x −4 = 4
5. y = ⇒ x3 x
cos 20 x
51 –867
10 cos 10 x cos 20 x + 20 sin 10 x sin 20 x 16. t ( x ) = 17 = 51x −17 ⇒ t ′( x ) = −867 x −18 = 18
y′ = 2
x x
cos 20 x

54 Problem Set 4-3 Calculus Solutions Manual


© 2005 Key Curriculum Press
14 1000
17. v( x ) = = 14(cos 0.5 x ) −1 ⇒ 27. a. v(t ) =
cos 0.5 x 3–t
v′(x) = −14(cos 0.5x)–2(−sin 0.5x)(0.5) v(1) =
1000
= 500 mi/h
7 sin 0.5 x 3 –1
= 1000
cos 2 0.5 x v(2) = = 1000 mi/h
20 3–2
18. a( x ) = = 20(sin x ) −2 ⇒ 1000 1000
sin 2 x v(3) = = . No value for v(3).
a′ (x) = −40(sin x) −3 (cos x) 3–3 0
–40 cos x b. v(t) = 100(3 − t)− 1 ⇒
=
sin 3 x a(t) = −1000(3 − t)− 2 − 1 =
1000
1 −1 (3 – t ) 2
19. r ( x ) = = x −1 ⇒ r ′( x ) = − x −2 = 2
x x 1000
a(1) = = 250 (mi/h)/h
1 –2 (3 – 1)2
20. s( x ) = 2 = x −2 ⇒ s′( x ) = −2 x −3 = 3
x x 1000
a(2) = = 1000 (mi/h)/h
10 (3 – 2 ) 2
21. W ( x) = 3 = 10( x − 1) ⇒
3 5
( x – 1) –5 a(3) =
1000
=
1000
. No value for a(3).
W′(x) = 150x2(x3 − 1)4 (3 – 3)2 0
1
22. T ( x) = = (cos x sin x ) −1 ⇒ c.
cos x sin x a or v
T ′( x ) = −(cos x sin x ) −2 ( − sin x sin x
sin 2 x – cos 2 x a
+ cos x cos x ) = , 2000
v
cos 2 x sin 2 x
which transforms to t
– cos 2 x 3
T ′( x ) = 1 2 = −4 csc 2 x cot 2 x
4 sin 2 x
sin x 1000
23. T ( x) = ⇒ d. = 500 ⇒ 2 = (3 − t )2 ⇒
cos x (3 – t ) 2
(cos x )(cos x ) – (sin x )(– sin x ) 3−t = ± 2 ⇒ t = 3± 2 ⇒
T ′( x ) =
cos 2 x t = 3 − 2 = 1.585K in the domain.
Range is 0 ≤ t < 1.585… .
cos 2 x + sin 2 x 1
= = = sec 2 x 28. a. Because they are walking in the same
cos 2 x cos 2 x direction, their relative rate is the difference
(T is for tangent function.)
cos x (x − 5).
24. C( x ) = ⇒ 300
sin x b. t( x ) = , assuming Willie’s rate is
(– sin x )(sin x ) – (cos x )(cos x ) x–5
C ′( x ) = constant.
sin 2 x
t(6) = 300 s, t(8) = 100 s, t(10) = 60 s,
– sin 2 x – cos 2 x –1 t(5) = 300/0, which is infinite, t(4) = −300,
= 2 = = − csc 2 x
sin x sin 2 x which is not reasonable in the real world,
(C is for cotangent function.) and t(5.1) = 3000 s. A reasonable domain
1 0 – cos x is x > 5.
25. C( x ) = ⇒ C ′( x ) =
sin x sin 2 x c. t(x) = 300(x – 5)− 1
1 cos x −300
=− ⋅ = − csc x cot x t′(x) = −300(x − 5)− 2 =
sin x sin x ( x − 5)2
(C is for cosecant function.) t′(6) = −300 s/(ft/s)
1 d. t′(5) does not exist because of division by
26. S( x ) = = (cos x ) −1 ⇒
cos x zero. More fundamentally, t′(5) does not exist
S′(x) = −(cos x)− 2(−sin x) because t(5) does not exist.
=
sin x
= sec x tan x 3x + 7
cos 2 x 29. f ( x ) = ⇒
2x + 5
(S is for secant function.)

Calculus Solutions Manual Problem Set 4-3 55


© 2005 Key Curriculum Press
3 ⋅ (2 x + 5) – (3 x + 7) ⋅ 2 1 31. If y = xn, where n is a negative integer, then
f ′( x ) = = y′ = nx n− 1.
(2 x + 5)2 (2 x + 5) 2
1
f ′( 4 ) = = 0.005917159K Proof:
169
Let n = −p, where p is a positive integer.
Using 4.1, f ′(4) ≈ 0.005827505… . 1
Using 4.01, f ′(4) ≈ 0.005908070… . ∴ y = x−p = p
x
Using 4.001, f ′(4) ≈ 0.005916249… . 0 ⋅ x p – 1 ⋅ px p–1
∴ y′ = because p is a
f ′(4) (exact) = 0.005917159… x2p
Difference quotients are approaching f ′(4). positive integer.
px p–1
30. a. Sketch. See accurate plot in part b. = − 2 p = − px p−1−2 p = − px − p−1 .
x
x2 – 8 Replacing −p with n gives y′ = nxn− 1, Q .E .D .
b. f ( x ) = ⇒
x–3 32.
2 x ( x – 3) – ( x 2 – 8)(1) x 2 − 6 x + 8 y y
f ′( x ) = =
( x – 3)2 ( x − 3)2 y'

1 y' 1 y
f (x )
x x
f 1 1

5 y

f'
3 x
33. Answers will vary.

y2 and y3 both agree with the graph of f ′.


c. Problem Set 4-4
x f (x) f ′(x) Q1. (sin x)/(tan x) = cos x
Q2. 1/(sec x) = cos x
2.95 −14.05 −399
Q3. sin2 3 + cos2 3 = 1
2.96 −19.04 −624
Q4. f ′ (x) = ex sin x + ex cos x
2.97 −27.363… −1110.11…
cos x + x sin x
2.98 −44.02 −2499 Q5. g ′( x ) =
cos 2 x
2.99 −94.01 −9999
Q6. h ′ (x) = −(15/7)(3x)–12/7
3.00 undefined undefined
Q7. dy/dx = 3(cos x)− 4 sin x
3.01 106.01 −9999
Q8. Limit = −3
3.02 56.02 −2499
Q9. (Function is secant.)
3.03 39.363… −1110.11…
y
3.04 31.04 −624 y
3.05 26.05 −399 y'

1 x
f ( x) changes faster and faster as x approaches
π
3, shooting off to negative infinity as x
approaches 3 from the negative side and to
positive infinity as x approaches 3 from the
positive side. Note that the rates are
symmetrical about x = 3. Q10. C
d. There is a relative minimum at x = 4 and a 1. f (x) = tan 5x ⇒ f ′(x) = 5 sec2 5x
relative maximum at x = 2. 2. f (x) = sec 3x ⇒ f ′(x) = 3 sec x tan x
2 2 − 6(2) + 8 y = sec x7 ⇒ y′ = 7x6 sec x7 tan x7
f ′( 2 ) = =0 3.
(2 − 3)2
4. z = tan x9 ⇒ z′ = 9x8 sec2 (x9)
4 2 − 6( 4) + 8
f ′( 4 ) = =0
( 4 − 3)2

56 Problem Set 4-4 Calculus Solutions Manual


© 2005 Key Curriculum Press
5. g(x) = cot e11x ⇒ g ′ (x) = −11e11x csc2 (e11x) 22. m(x) = cot2 x − csc2 x = −1 ⇒ m′(x) = 0
6. h(x) = csc e10x ⇒ h ′ (x) = −10e10x csc (e10x) cot (e10x) (The differentiation formulas give the same.)
7. r(x) = ln (csc x) ⇒ 23. A(x) = sin x 2 ⇒ A′(x) = cos x 2 · 2x = 2x cos x 2
r ′( x ) =
1
(−csc x cot x) = −cot x 24. f (x) = cos x3 ⇒ f ′ (x) = −sin x 3 · 3x 2
csc x = −3x 2 sin x 3
8. p(x) = ln (cot x) ⇒ 25. F(x) = sin2 x ⇒ F ′ (x) = 2 sin x cos x
1 1 26. g(x) = cos3 x ⇒
p ′( x ) = ( − csc 2 x ) =
cot x cos x sin x g′(x) = 3 cos2 x · (−sin x) = −3 cos2 x sin x
9. y = tan5 4x ⇒ 27. y = tan x ⇒ dy/dx = sec2 x ⇒
(d/dx)(y) = 5 tan4 4x · sec2 4x · 4 d2y/dx2 = 2 sec x(sec x tan x) = 2 sec2 x tan x
= 20 tan4 4x sec2 4x 28. y = sec x ⇒ y′ = sec x tan x ⇒
10. y = tan7 9x ⇒ y″ = (sec x tan x) · tan x + sec x · sec2 x
(d/dx)(y) = 7 tan6 9x · sec2 9x · 9 = sec x tan2 x + sec3 x
= 63 tan6 9x sec2 9x
cos x
11. (d/dx)(sec x tan x) = sec x tan x · tan x + 29. y = cot x = ⇒
sin x
sec x · sec2 x = sec x tan2 x + sec3 x – sin x ⋅ sin x – cos x ⋅ cos x
12. (d/dx)(csc x cot x) = −csc x cot x · cot x + y′ =
sin 2 x
csc x · (−csc2 x) = −csc x cot2 x − csc3 x –1
= = − csc 2 x or:
13. y = sec x csc x ⇒ sin 2 x
y′ = sec x tan x · csc x + sec x · (−csc x cot x) 1
y= = ( tan x ) −1 ⇒
= sec2 x − csc2 x tan x
14. y = tan x cot x = 1 for all x ⇒ y′ = 0 y′ = −1 ⋅ (tan x)− 2 ⋅ sec2 x = −csc2 x
tan x 1
15. y = = sec x ⇒ y′ = sec x tan x 30. y = csc x = = (sin x ) –1 ⇒
sin x sin x
cot x − cos x
16. y = = csc x ⇒ y′ = −csc x cot x y′ = −(sin x ) −2 cos x = = − csc x cot x
cos x sin 2 x
5 ln 7 x 31. a. See graph in part b.
17. y = ⇒
cot 14 x b. f (x) = tan x ⇒ f ′(x) = sec2 x. Predicted graph
5 ⋅ 7 ( 71x )cot 14 x − 5 ln 7 x ( −14 csc 2 14 x ) should be close to actual one.
y′ =
cot 2 14 x y
(5 cot 14 x )/ x + 70 ln 7 x csc 2 14 x f´
=
cot 2 14 x 1
f
x
5 70 ln 7 x 1
= +
x cot 14 x cos 2 14 x
4 csc 10 x
18. y = ⇒
e 40x
tan 1.01 – tan 0.99
4( −10 csc10 x cot 10 x )e − 4 csc10 x ( 40e
40x 40x
) c. = 3.42646416 K
y′ = 40x 2(0.01)
(e ) 2
tan′ 1 = sec2 1 = (1/cos 1)2 = 3.42551882…
−40 csc 10 x cot 10 x − 160 csc 10 x Difference quotient is within 0.001 of actual.
=
e 40x 32. a. f (x) = sec x ⇒ f ′ (x) = sec x tan x ⇒
19. w = tan (sin 3x) ⇒ f ′ (1) = sec 1 tan 1 = 2.8824…
w′ = sec2 (sin 3x) · 3 cos 3x b.
= 3 sec2 (sin 3x) · cos 3x
y
20. t = sec (cos 4x) ⇒ y1

t′ = sec (cos 4x) tan (cos 4x) · (−4 sin 4x)


5 y2

= −4 sec (cos 4x) tan (cos 4x) sin 4x


21. S(x) = sec2 x − tan2 x = 1 ⇒ S′(x) = 0
x
(The differentiation formulas give the same.) 1

Calculus Solutions Manual Problem Set 4-4 57


© 2005 Key Curriculum Press
The formula is confirmed by the fact that the cannot be continuous. (Some texts restrict the
line is tangent to the graph. range of the inverse cosecant to 0 ≤ y ≤ π/2 so
c. that the function will be continuous, but doing
y
so throws away the other half of the possible
f f'
values.)
1
x 7. sin (sin− 1 0.3) = 0.3
π/2
8. cos− 1 (cos 0.8) = 0.8
9. y = sin− 1 x ⇒ sin y = x ⇒ cos y · y′ = 1 ⇒
1 1
y′ = = , Q .E .D .
If f ′ (x) is negative, the graph of f is cos y 1 – x2
decreasing.
33. a. y/10 = tan x ⇒ y = 10 tan x, Q .E .D .
1
b. y′ = 10 sec2 x. At x = 1, y′ = 10 sec2 1 = x

34.2551… . y is increasing at about y

34.3 ft/radian. √1 – x 2
π
(34.2551K) = 0.5978… ft/degree
180 [Because sin y = (opposite leg)/(hypotenuse), put
c. y = 535 ⇒ x = tan− 1 53.5 = 1.55210… x on the opposite leg and 1 on the hypotenuse.
∴ y′ = 10 sec2 1.55210… = 28632.5… Adjacent leg = 1 – x 2 , and cos y =
y is increasing at about 28,632.5 ft/radian. (adjacent)/(hypotenuse).]
opposite side y
34. a. tan x = = 10. y = cos− 1 x ⇒ cos y = x ⇒ −sin y · y′ = 1 ⇒
adjacent side 500 1 1
∴ y = 500 tan x, Q.E.D. y′ = − =− , Q .E .D .
sin y 1 – x2
b. dy/dt = 500 sec2 x · dx/dt
c. dx/dt = 0.3 rad/s
At y = 300, x = tan− 1 (300/500) = 0.5404… 1
√1 – x 2
∴ dy/dt = 500 (sec2 0.5404…)(0.3) y
= 500(1.36)(0.3) = 204 ft/s x

35. a. y = sin x + C
b. y = − 12 cos 2 x + C [Because cos y = (adjacent leg)/(hypotenuse), put
c. y = 13 tan 3 x + C x on the adjacent leg and 1 on the hypotenuse.
d. y = − 14 cot 4 x + C Opposite leg = 1 – x 2 , and sin y =
e. y = 5 sec x + C (opposite)/(hypotenuse).]
36. Answers will vary. 11. y = csc− 1 x ⇒ csc y = x ⇒ −csc y cot y · y′ ⇒
1 1
y′ = − =− if x > 0
csc y cot y x x2 – 1
Problem Set 4-5
If x < 0, then y is in Quadrant IV (see Fig-
Q1. sin′ x = cos x Q2. cos′ x = −sin x ure 4-5d). So both csc y and cot y are negative,
Q3. tan′ x = sec x 2
Q4. cot′ x = −csc2 x and thus their product must be positive.
Q5. sec′ x = sec x tan x Q6. csc′ x = −csc x cot x ∴ y′ = −
1
, Q .E .D .
Q7. f ′ (1) is infinite. Q8. f ′ (3) is undefined. | x | x2 – 1
Q9. f ′ (4) = −1 Q10. f ′ (6) = 0
1. See Figure 4-5d. 2. See Figure 4-5d. x
1
3. See Figure 4-5d. 4. See Figure 4-5d. y
5. The principal branch of the inverse cotangent √x 2 – 1
function goes from zero to π so that the function
will be continuous. [Because csc y = (hypotenuse)/(opposite leg), put
6. There are no values of the inverse secant for x x on the hypotenuse and 1 on the opposite leg.
between −1 and 1, so the inverse secant function

58 Problem Set 4-5 Calculus Solutions Manual


© 2005 Key Curriculum Press
Adjacent leg = x 2 – 1, and csc y = x and cot y = 16. y = tan− 1 (ln x) ⇒ tan y = ln x ⇒
(adjacent)/(opposite).] sec2 y · y′ = l/x ⇒
1 1 1
12. y = cot− 1 x ⇒ cot y = x ⇒ −csc2 y · y′ = 1 ⇒ y′ = = 2 =

y′ = −
1
=−
1
2 =−
1
, Q .E .D .
x sec 2 y
x 1 + ln 2 x (
x (1 + ln 2 x )
)
csc 2 y
1+ x 2
(1+ x2
)
√ 1 + ln2x
ln x
√1 + x 2 y
1 1
y
x
x x 1
17. y = sec −1 ⇒ sec y = ⇒ sec y tan y ⋅ y′ = ⇒
[Because cot y = (adjacent leg)/(opposite leg), 3 3 3
put x on the adjacent leg and 1 on the opposite 1 1
y′ = = , if x > 0
leg. Hypotenuse = 1 + x 2 , and csc y = 3 sec y tan y 2
3⋅ (
x
3 x – 9) / 3
(hypotenuse)/(opposite).]
Problems 13−18 are shown done “from scratch,” as If x < 0, then y is in Quadrant II, where both
in Example 1. If students practice doing them this sec y and tan y are negative. So their product is
way, they will not be dependent on memorized positive.
formulas. Problem 13 shows how an alternate 3
∴ y′ =
solution could be found using the formulas and the 2
chain rule. |x| x –9
13. y = sin− 1 4x ⇒ sin y = 4x ⇒ cos y · y′ = 4 ⇒
4 4
y′ = =
x
√x 2 – 9
cos y 1 – 16 x 2 y
3

1
4x
x x
y 18. y = csc −1 ⇒ csc y = ⇒
10 10
√1 – 16x 2
1
− csc y cot y ⋅ y ′ = ⇒
Alternate solution by application of the formula: 10
1 4 1 1
y = sin −1 4 x ⇒ y ′ = ⋅4= y′ = − =−
2
1 – (4 x ) 1 – 16 x 2 10 csc y cot y x x 2 – 100
10 ⋅
14. y = cos− 1 10x ⇒ cos y = 10x ⇒ 10 100
10 10 If x < 0, then y is in Quadrant IV, where both
−sin y · y′ = 10 ⇒ y′ = − =− csc y and cot y are negative. So their product is
sin y 1 – 100 x 2 positive.
10
1
∴ y′ = −
√1 – 100x 2 | x | x 2 – 100
y
10x

x
10
15. y = cot −1 e 0.5x ⇒ cot y = e 0.5x ⇒ y
− csc 2 y ⋅ y ′ = 0.5e 0.5x ⇒ √x 2 – 100
0.5e 0.5x 0.5e 0.5x 0.5e 0.5x
y′ = − =− 2 =−
( )
1+ ex
2 For Problems 19−24, a solution is shown using the
csc y
1+ ex
appropriate formula.
19. y = cos− 1 5x2
√ 1 + ex 1 10 x
y′ = − ⋅ 10 x = −
2 2
y
1
1 – (5 x ) 1 – 25 x 4
e 0.5x 20. f (x) = tan− 1 x3
1 3x 2
f ′( x ) = ⋅ 3 x 2
=
1 + ( x 3 )2 1+ x6
Calculus Solutions Manual Problem Set 4-5 59
© 2005 Key Curriculum Press
21. g(x) = (sin− 1 x)2 d. Maximum is between x = 38 and 39.
−1 1
g ′( x ) = 2 sin x ⋅ θ
1 – x2 0.5

22. u = (sec− 1 x)2


1
u ′ = 2 sec −1 x ⋅
| x | x2 – 1 x
40 100
23. v = x sin− 1 x + (1 − x2)1/ 2
1 1
v ′ = 1 ⋅ sin −1 x + x ⋅ + (1 – x 2 ) −1/2 ⋅ ( −2 x )
1– x 2 2 27.
x x x Num. Deriv.* Alg. Deriv.
= sin −1 x + − = sin −1 x
1– x 2
1– x 2
−0.8 −1.666671… −1.666666…
The surprise is that you now have seen a formula −0.6 −1.250000… −1.25
for the antiderivative of the inverse sine function.
−0.4 −1.091089… −1.091089…
24. f (x) = cot− 1 (cot x) = x ⇒ f ′ (x) = 1 (Surprise!!)
−0.2 −1.020620… −1.020620…
Application of the formulas gives the same
result. 0 −1.000000… −1
25. a. tan θ = x/100, so θ = tan− 1 (x/100), Q.E.D. 0.2 −1.020620… −1.020620…
dθ 1 1 100 0.4 −1.091089… −1.091089…
b. = ⋅ =
dx 1 + ( x/100)2 100 10000 + x 2 0.6 −1.250000… −1.25
dθ dθ dx 100 dx 0.8 −1.666671… −1.666666…
= ⋅ = ⋅
dt dx dt 10000 + x 2 dt *The precise value for the numerical
c. If x = 500 ft and dθ/dt = −0.04 rad/s, then derivative will depend on the tolerance to
100 dx which the grapher is set. The values given
−0.04 = 2 ⋅
10000 + 500 dt by numerical derivative and the formula are
dx (–0.04)(260000) very close.
= = −104
dt 100 dy 1
The truck is going 104 ft/s. 28. a. y = sec −1 x ⇒ =
dx | x | x 2 – 1
104(3600/5280) = 70.909… ≈ 71 mi/h
dy 1
26. a. θ = tan− 1 (50/x) − tan− 1 (30/x) or At x = 2, = = 0.288675K .
θ = cot− 1 (x/50) − cot− 1 (x/30) dx | 2 | 3
The inverse tangent equation has the The answer is reasonable because the graph
advantage that the function appears on the slopes up at x = 2, with slope significantly
calculator. The inverse cotangent equation has less than 1.
the advantage that x is in the numerator of the b. At x = 2, y = sec− 1 2 = cos− 1 (1/2) =
argument, which makes the chain rule less 1.04719… .
complicated to use.) d
(sec y) = sec y tan y
dy
dθ –50 x –2 –30 x –2
b. = 2 − At y = 1.047… ,
dx 1 + (50/ x ) 1 + (30/ x )2 d
–50 30 (sec y) = (sec 1.047K)(tan 1.047K) =
= 2 + 2 dy
x + 2500 x + 900 3.464101… .
–20 x 2 + 30000 c. The answer to part b is the reciprocal of the
= 2
( x + 2500)( x 2 + 900) K =
1
answer to part a. That is, 3.464101
0.288675… . Thus, the derivative of the
c. dθ/dx = 0 ⇒ −20x2 + 30000 = 0 ⇒ inverse secant at x = c is the reciprocal of
20x2 = 30000 the derivative of the secant at y = sec− 1 c.
x = ± 1500 = ±38.729K 29. a. y = sin− 1 x ⇒ sin y = x ⇒ cos y · y′ = 1 ⇒
About 38.7 ft 1
y′ = , Q .E .D .
cos y

60 Problem Set 4-5 Calculus Solutions Manual


© 2005 Key Curriculum Press
1 1 14. a.
b. y ′ = = = 1.25
cos(sin –1 x ) cos(sin –1 0.6) f (x)

1 1 1
y′ = = = = 1.25, Q .E.D . 4

1– x 2
1 – 0.6 2 0.8
d
c. y = f −1 ( x ) ⇒ f ( y) = x ⇒ f ′( y) ⋅ ( y) = 1 ⇒ x
dx
–2
d 1 d 1
( y) = ⇒ [ f –1 ( x )] = ,
dx f ′( y ) dx f ′[ f −1 ( x )]
Q .E .D . b. f ( x) = x 2
d. f ( x) = x3 + x = 10 ⇒ (x − 2)(x2 + 2x + 5) = 0 15. a.
⇒ x = 2 (only)
f (x )

∴ h(10) = 2
Because h(x) = f −1(x) and f ′(x) = 3x 2 + 1,
1 1 1
h ′(10) = = = = 113/ .
f ′[h(10)] f ′(2) 3 ⋅ 2 2 + 1 x
6
30. The inverse trig cofunctions, cos− 1, cot− 1, and
csc− 1, are the ones whose derivatives are preceded
by a minus sign. ( x − 6)( x + 1)
b. f ( x ) =
x−6
Problem Set 4-6 16. a.
f (x )
Q1. See the text for the definition of continuity.
Q2. See the text for the definition of derivative.
2
Q3. y′ = −6x − 2 + C Q4. cos′ x = −sin x
Q5. dy/dx = sec2 x (
Q6. 1 | x | x 2 − 1 ) x

1
Q7. f ′ (x) = 4x 3; f ″ (x) = 12x2; f ″ (2) = 48
Q8. dy/dx = 15x2(x3 + 1)4
 x 2 ( x − 1) , if x ≠ 1
Integral ≈ 5.4 (Function is y = 2− x.) 
Q9. b. f ( x ) =  x − 1
Q10. E 5, if x = 1
1. Continuous 2. Neither 17. a.
3. Neither 4. Both f (x )

5. Neither 6. Neither
7. Both 8. Neither x
9. Neither 10. Neither —5

11. Continuous 12. Both


For Problems 13−20, sample answers are given.
Equations do not necessarily correspond to the graphs
shown.  x, if x ≤ −5
b. f ( x ) = 
13. a. 3 x, if x > −5
f (x )
18. a.
5 f (x )

3
x
3

b. f ( x) = x + 2 –1

Calculus Solutions Manual Problem Set 4-6 61


© 2005 Key Curriculum Press
b. f ( x) = (x + 1) 2/3 + 3 b = 1 − a = 1 − (−1.5) ⇒ b = 2.5
19. a. f (x )

f (x )
7 1
x
1
x
4

f is differentiable at x = 1.
–( x – 3)2 + 7, if x ≥ 2
26. f ( x ) =  3
 x 2 − 9, if x < 4 ax + b, if x < 2
b. f ( x ) = 
11 − x, if x ≥ 4 For f to be continuous at x = 2,
20. a. No such function lim− ( ax 3 + b) = lim+ [ −( x – 3) 2 + 7] ⇒
x →2 x →2
f(x )
a ⋅ 23 + b = 6 ⇒ 8a + b = 6 ⇒ b = 6 − 8a
Not possible. For f to be differentiable at x = 2,
Differentiability
implies lim− 3ax 2 = lim+ [ −2( x – 3)] ⇒ 3a ⋅ 2 2 = 2 ⇒
continuity. x →2 x →2
a = 1/6
x b = 6 − 8(1/6) ⇒ b = 14/3
f (x )
6
b. No such function
21. Continuous 22. Both
x
f (x ) f (x )
2

4
f is differentiable at x = 2.
 ax 2 + 10,
x
if x < 2
3 27. f ( x ) =  2
x  x − 6 x + b, if x ≥ 2
2
For f to be continuous at x = 2,
lim− ( ax 2 + 10) = lim+ ( x 2 − 6 x + 6) ⇒ 4 a + 10 =
23. Both 24. Neither x →2 x →2

f (x )
4 – 12 + b ⇒ b = 4a + 18
y
For f to be differentiable at x = 2,
lim− 2 ax = lim+ (2 x – 6) ⇒ 2 a ⋅ 2 = 2 ⋅ 2 – 6 ⇒
x →2 x →2
x x a = −0.5
1 π/2 b = 4(–0.5) + 18 ⇒ b = 16
f (x )

10

x ,3
if x < 1
25. f ( x ) = 
a( x – 2) + b, if x ≥ 1
2
x

For f to be continuous at x = 1, 2

lim− x 3 = lim[ +
a( x − 2 ) 2 + b ] ⇒ f is differentiable at x = 2.
x →1 x →1

1 = a(1 − 2) + b ⇒ a + b = 1 ⇒ b = 1 − a
2
 a/ x, if x ≤ 1
28. f ( x ) = 
For f to be differentiable at x = 1,  12 − bx , if x > 1
2

lim− 3 x 2 = lim+ 2 a( x − 2) ⇒ 3 = 2 a(1 − 2) ⇒ For f to be continuous at x = 1,


x →1 x →1

a = −1.5

62 Problem Set 4-6 Calculus Solutions Manual


© 2005 Key Curriculum Press
lim a/ x = lim+ (12 – bx 2 ) ⇒ a/1 = 12 – b ⋅ 12 ⇒
x →1−
f (x )
x →1
a + b = 12
For f to be differentiable at x = 1, 0.5

lim− − ax −2 = lim+ −2 bx ⇒ − a ⋅ 1−2 = −2 b ⋅ 1 ⇒


x →1 x →1
a = 2b 3 x

∴ 2b + b = 12 ⇒ b = 4
a=2·4⇒a=8 f is differentiable at x = 2π/3.
f (x )
ax 3 + bx 2 + cx + d , if 0 ≤ x ≤ 0.5
31. a. y = 
10  x + k, if x > 0.5
For y to contain the origin,
x a ⋅ 03 + b ⋅ 02 + c ⋅ 0 + d = 0 ⇒ d = 0
1
For y′ = 0 at x = 0, y′ = 3ax 2 + 2bx + c ⇒
0 = 3a ⋅ 02 + 2b ⋅ 0 + c ⇒ c = 0
f is differentiable at x = 1. For y′ = 1 at x = 0.5, y′ = 3ax2 + 2bx + c ⇒
1 = 3a(0.5)2 + 2b(0.5) + c ⇒ 1 = 0.75a + b ⇒
e ax , if x ≤ 1 b = 1 – 0.75a
29. f ( x ) = 
b + ln x, if x > 1 For y ′′ = 0 at x = 0.5, y ′′ = 6 ax + 2 b ⇒
For f to be continuous at x = 1, 0 = 3a + 2b
lim− e ax = lim+ (b + ln x ) ⇒ e a = b Solve for a and b:
x →1 x →1
3a + 2(1 – 0.75a) = 0 ⇒ 1.5a = –2 ⇒
For f to be differentiable at x = 1,
a = – 4/3 b = 2
lim− ae ax = lim+ (1/ x ) ⇒ ae a = 1
x →1 x →1 b. For the function to be continuous,
Solve by grapher: a = 0.5671… and
b = 1.7632… lim ( − 43 x 3 + 2 x 2 ) = lim + ( x + k ) ⇒
x →0.5− x →0.5

f (x ) − 43 (0.5)3 + 2(0.5)2 = 0.5 + k ⇒


k = – 16 = −0.1666 K
2 32. Equation of the linear part of the fork is
y – 20 = 5(x – 10) ⇒ y = 5x – 30
x
ax 3 + bx, if x < 10
1 ∴y = 
5 x − 30, if x ≥ 10
f is differentiable at x = 1.
For y to be continuous at x = 10,
a sin x, if x < 2π /3 a ⋅ 10 3 + b ⋅ 10 = 5 ⋅ 10 – 30
30. f ( x ) =  bx
e , if x ≥ 2π /3 1000a + 10b = 20 ⇒ b = 2 – 100a
For y to be differentiable at x = 10,
For f to be continuous at x = 2π/3,
3a ⋅ 10 2 + b = 5
lim − a sin x = lim + e bx ⇒ 300a + (2 – 100a) = 5
x →( 2π / 3) x →( 2π / 3)
200a = 3 ⇒ a = 3/200
a 3 2e 2πb / 3 b = 2 – 100(3/200) ⇒ b = 0.5
= e 2πb / 3 ⇒ a =
2 3
For f to be differentiable at x = 2π/3,  2 x−2
x − , if x ≠ 2
lim − a cos x = lim + be bx ⇒ 33. f ( x ) =  x−2
x→( 2π /3) x →( 2π /3) 4, if x = 2
−a
= be 2πb / 3 ⇒ a = –2 be 2πb/3 Simplifying the equation for f (x) gives
2
2e 2πb/3 2 −1  x 2 + 1, if x < 2
So = –2 be 2πb/3 ⇒ = –2 b ⇒ b =
3 3 3 
f ( x ) =  x 2 − 1, if x > 2
2 −2π /( 3 3 )
= –0.5773… and a = e = 0.3446 K 4, if x = 2
3 

Calculus Solutions Manual Problem Set 4-6 63


© 2005 Key Curriculum Press
Taking the derivative for each branch gives e. A regulation baseball diamond has the
2 x, if x < 2 pitcher’s mound 60.5 feet from home plate.
 Substituting zero for t gives d(0) = 60.5,
f ′( x ) = 2 x, if x > 2
confirming that the pitcher was on the mound
undefined, if x = 2
 at that time.
Taking the left and right limits gives 35. a. y = mx + b ⇒ y′ = m, which is independent
lim− f ′( x ) = 2 ⋅ 2 = 4; lim+ f ′( x ) = 2 ⋅ 2 = 4. of x.
x →2 x →2
Using the definition of derivative, taking the ∴ linear functions are differentiable for all x.
x2 +1− 4 1 ∴ linear functions are continuous for all x.
limit from the left, f ′( x ) = lim− → ,
x →2 x−2 0 b. y = ax 2 + bx + c ⇒ y′ = 2ax + b, which
which is infinite. The same thing happens from exists for all x by the closure axioms.
the right. As the following graph shows, the ∴ quadratic functions are differentiable for
secant lines become vertical as x approaches 2 all x.
from either side. ∴ quadratic functions are continuous for all x.
f (x ) Secant c. y = 1/x = x–1 ⇒ y′ = –x–2, which exists for
slope
becomes all x ≠ 0 by closure and multiplicative inverse
infinite.
axioms.
4 ∴ the reciprocal function is differentiable for
all x ≠ 0.
∴ the reciprocal function is continuous for all
x x ≠ 0.
2
d. y = x ⇒ y′ = 1, which is independent of x.
∴ the identity function is differentiable for
Thus, f is not differentiable at x = 2, even all x.
though the right and left limits of f ′ (x) are equal ∴ the identity function is continuous for
to each other. The function must be continuous all x.
if it is to have a chance of being differentiable.
e. y = k ⇒ y′ = 0, which is independent of x.
  0.5 − t  ∴ constant functions are differentiable for
60.5  0.5 + t  , if t ≤ 0.5
 all x.
34. a. d (t ) =  ∴ constant functions are continuous for all x.
150  2 − 1 , if t ≥ 0.5
  t 36. See text proof.
−2
−60.5(0.5 + t ) , if t < 0.5
d ′(t ) =  Problem Set 4-7
150t −2, if t > 0.5
The inequality signs must be < and > because Q1. y′ = 243x1214 Q2. dy/dx = 2/(x–1)2
although the function is defined at x = 0.5, Q3. f ′ (x) = 1 + ln x Q4. y′(x) = 5e5x cos e5x
the derivative is not. Q5. (d/dx)(y) = 3x2, x ≠ 0; d2y/dx2 = 6x, x ≠ 0
b. d ′(1) = 150(1) –2 = 150 ⇒ d is continuous at
Q6. y′ = 0 Q7. θ ′ = −1/ 1 − x 2
x = 1 because it is differentiable there.
c. lim − d ′(t ) = −60.5(0.5 + 0.5) −2 = −60.5 Q8. v(t) is decreasing at t = 5.
x →0.5
Q9. Q10. E
lim d ′(t ) = 150(0.5) −2 = 600
x →0.5+
y'
As the ball was about to be hit, it was
approaching the plate at 60.5 ft/s. 1 x
Just after the ball was hit, it was going away π/2

from the plate at 600 ft/s. y

d. Function d is not differentiable at t = 0.5


because d′ (t) approaches different limits from
both sides of x = 0.5. 1. x = t4 , y = sin 3t
Function d is continuous at t = 0.5 because dy dy/dt 3 cos 3t d 2 y d  3 cos 3t 
= = ⇒ =
you get zero as the limit of d(t) as t dx dx/dt 4t 3 dx 2 dx  4t 3 
approaches zero from either left or right.

64 Problem Set 4-7 Calculus Solutions Manual


© 2005 Key Curriculum Press
( −9 sin 3t )( dt/dx ) ⋅ 4t 3 − 3 cos 3t (12t 2 )( dt/dx ) 4. a. x = t 2 , y = t 3
=
( 4t 3 ) 2 t x y

−36t 3 sin 3t − 36t 2 cos 3t dx –3 9 –27


= ÷ –2 4 –8
16t 6 dt
–1 1 –1
−36t 3 sin 3t − 36t 2 cos 3t
= 0 0 0
64t 9 1 1 1
−9t sin 3t − 9 cos 3t 2 4 8
=
16t 7 3 9 27
2. x = 6 ln t, y = t 3 y
b.
2 2
dy dy/dt 3t d y
= = = 0.5t 3 ⇒ 2
dx dx/dt 6/t dx
d dx
= (0.5t 3 ) = 1.5t 2 ( dt/dx ) = 1.5t 2 ÷
dx dt 5
x
2
1.5t
= = 0.25t 3
5

6/t
3. a. x = 2 + t, y = 3 – t 2

t x y
–3 –1 –6
dy dy/dt 3t 2
–2 0 –1 c. = = = 1.5t
dx dx/dt 2t
–1 1 2 If t = 1, dy/dx = 1.5 and (x, y) = (1, 1).
0 2 3 Line through (1, 1) with slope 1.5 is tangent
1 3 2 to the graph. See graph in part b.
2 4 –1 d. x = t 2 ⇒ t = x 1/ 2 ⇒ y = ( x 1/ 2 )3 ⇒ y = x 1.5
The name semicubical is picked because 1.5
3 5 –6
is half of 3, the exponent for a cubic function.
b. The name parabola is used because the equation
y looks similar to y = x2 for a parabola.
e. By direct differentiation, dy/dx = 1.5x0.5 .
2
x
At (x, y) = (1, 1), dy/dx = 1.5 ⋅ 10.5 = 1.5,
3 which agrees with part c.
dy/dx = 1.5x0.5 = 1.5(t2)0.5 = 1.5t, which
agrees with part c.
5. a. The graph confirms the figure in the text.
dy 5 cos t −5
b. = = cot t
dx −3 sin t 3
dy dy/dt −2t If t = π /4, x = 3 2 /2 and y = 5 2 /2.
c. = = = −2t c.
dx dx/dt 1 (x, y) = (2.121… , 3.535…)
If t = 1, dy/dx = –2 and (x, y) = (3, 2). dy −5 π
Line through (3, 2) with slope –2 is tangent = cot = −5/3
dx 3 4
to the graph. See part b. y
d. x = 2 + t ⇒ t = x – 2 ⇒ y = 3 – (x – 2)2 5

This is the Cartesian equation of a parabola


because only one of the variables is squared. x
e. By direct differentiation, dy/dx = –2( x – 2). –3 3

At (x, y) = (3, 2), dy/dx = –2(3 – 2) = –2,


which agrees with part c. –5
dy/dx = –2(x – 2) = –2(2 + t – 2) = –2t,
which agrees with part c. The line is tangent to the graph.

Calculus Solutions Manual Problem Set 4-7 65


© 2005 Key Curriculum Press
d. False. The line from (0, 0) to (2.1… , 3.5…) 7. a. x = 6 + 5 cos t, y = 3 + 5 sin t
does not make an angle of 45° with the y
x-axis. (This shows that the t in parametric
functions is not the same as the θ in polar dy /dx is
infinite
coordinates.) 3 here.

e. The tangent line is horizontal if dy/dx = 0. x

∴ cos t = 0 and sin t ≠ 0.


6

This happens at t = π/2, 3π/2, … .


Points are (0, 5), (0, –5). dy 5 cos t
b. = ⇒ dy/dx = − cot t
Tangent line is vertical if dy/dx is infinite. dx −5 sin t
∴ sin t = 0 and cos t ≠ 0. c. dy/dx = 0 if cos t = 0 and sin t ≠ 0.
This happens at t = 0, π , 2π , … . ∴ t = 0.5π , 1.5π , 2.5π , …
Points are (3, 0), (–3, 0). See graph in part c. dy/dx is infinite if sin t = 0 and cos t ≠ 0.
f. x/3 = cos t ⇒ (x/3)2 = cos2 t ∴ t = 0, π , 2π , …
y/5 = sin t ⇒ (y/5)2 = sin2 t At a point where dy/dx is infinite, dx/dt must
Adding left and right sides of the equations be zero. This happens where t = π /2 ± nπ ,
gives (x/3)2 + (y/5)2 = cos2 t + sin2 t. so dy/dx = 5 cos t = 0 at those points. See
∴ (x/3)2 + (y/5)2 = 1, which is a standard form graph in part a.
of the equation of an ellipse centered at the x−6 y−3
origin, with x-radius 3 and y-radius 5. d. = cos t and = sin t
5 5
6. a. The graph confirms the figure in the text.
 x − 6  +  y − 3  = cos 2 t + sin 2 t
2 2

dy 24 sin 2 t cos t sin t  5   5 


b. = =− = – tan t
dx 24 cos t ( − sin t )
2
cos t
 x − 6  +  y − 3 = 1
2 2
∴ dy/dx = –tan t
 5   5 
c. If t = 1, x = 8 cos3 1 = 1.2618… , and This is an equation of a circle centered at
y = 8 sin3 1 = 4.7665… , (6, 3) with radius 5.
(x, y) = (1.2618… , 4.7665…). e. The 6 and 3 added in the original equations
At t = 1, dy/dx = –tan 1 = –1.5574… . are the x- and y-coordinates of the center.
y The coefficients, 5, for cosine and sine in the
8
original equations are the x- and y-radii,
respectively. Because the x- and y-radii are
x equal, the graph is a circle.
–8 8 8. x = cos2 t, y = sin2 t
dy 2 cos t ( − sin t )
= = –1 (cos t ≠ 0, sin t ≠ 0)
–8 dx 2 sin t cos t

The line is tangent to the graph. y

d. dx/dt = –24 cos2 t sin t 1


dy/dt = 24 sin2 t cos t
The cusps occur where t is a multiple of π/2.
At each such value, dx/dt and dy/dt equal zero. x

t = 0 gives the cusp at (8, 0). 1

lim ( dy/dx ) = lim (– tan t ) = – tan 0 = 0


t→0 t→0
The graph is a line segment with a slope of –1.
So the graph becomes horizontal at (8, 0). x + y = cos2 t + sin2 t ⇒ x + y = 1
t = π/2 gives the cusp at (0, 8).
This is the equation of a line with slope –1,
lim ( dy/dx ) = lim ( − tan t ), which is infinite. confirming what was observed on the graph.
t →π /2 t →π /2
So the graph becomes vertical at (0, 8). The parametric equations restrict the ranges of x
and y to the first quadrant, no matter what is the
e. x/8 = cos3 t ⇒ (x/8)2/3 = cos2 t domain of t. This is true because cos2 t and sin2 t
y/8 = sin3 t ⇒ (y/8)3/2 = sin2 t are never negative.
∴ (x/8)2/3 + (y/8)2/3 = cos2 t + sin2 t The Cartesian equation allows
⇒ x 2/3 + y 2/3 = 4 –∞ < x < ∞ and –∞ < y < ∞.

66 Problem Set 4-7 Calculus Solutions Manual


© 2005 Key Curriculum Press
9. a. The grapher confirms the figure in the text. 2. Use the double argument properties on
dy/dx:
dy 2 cos t − 2 cos 2t cos t − cos 2t
b. = = dy cos t – (2 cos 2 t – 1)
dx −2 sin t − 2 sin 2t − sin t − sin 2t =
dx –(sin t + 2 sin t cos t )
c. Cusps occur where both dx/dt and dy/dt = 0. (1 – cos t )(1 + 2 cos t ) 1 – cos t
A graphical solution shows that this occurs at = = ,
–(sin t )(1 + 2 cos t ) – sin t
t = 0, t = 2π /3, t = 4π /3, t = 2π , … .
(A cusp could also happen if dx/dt = 0 and which approaches − 3 as t → 2π/3.]
dy/dt ≠ 0, but for this figure there is no such 10. a. The grapher confirms the figure in the text.
place.) dy 4 a cos t (– sin t )
b. = = –2 cos3 t sin t
dx /dt or dy /dt
dx 2 a sec 2 t
(The answer is independent of a.)
dx /dt
4 a 2 sin 2 t 4 a 2 (1 – cos 2 t )
t c. x 2 = 4 a 2 tan 2 t = =
0 __
2 π 4π
__ 2π cos 2 t cos 2 t
3 3
y = 2a cos t ⇒ cos t = y/(2a)
2 2

4 a 2 [1 – y/(2 a)] 4 a 2 (2 a − y)
∴ x2 = =
dy /dt

y/(2 a) y
x2y = 8a3 – 4a2y ⇒ (x2 + 4a2)y = 8a3 ⇒
At t = 0, 2π, … , the tangent appears to
8a 3
be horizontal. At t = 2π /3, 4π /3, 8π /3, y= 2
10π /3, … , there appears to be a tangent x + 4a 2
216
line but not a horizontal one. a = 3⇒ y = 2
A numerical solution shows the following x + 36
values as t approaches 2π/3: d. y = 8a 3 ( x 2 + 4 a 2 ) −1 ⇒
dy –16 a 3 x
= −8a 3 ( x 2 + 4 a 2 ) −2 ⋅ 2 x = 2
t dy/dx dx ( x + 4a 2 )2
e. At t = π/4, x = 2a tan (π/4) = 2a.
2π /3 – 0.1 –1.547849…
From part d,
2π/3 – 0.01 –1.712222…
dy –16 a 3 (2 a) –32 a 4
2π/3 – 0.001 –1.730052… = 2 2 = = –1/2
dx [(2 a) + 4 a ]
2
64 a 4
2π/3 indeterminate
From part b,
2π/3 + 0.001 –1.734052… dy
= –2 cos3 (π /4) sin (π /4)
2π /3 + 0.01 –1.752225… dx
2 π /3 + 0.1 –1.951213… = –2( 2 /2)3 ( 2 /2) = –1/2, which agrees.
At t = π/4, x = 2a tan (π/4) = 2a = 6 and
dy/dx seems to be approaching about –1.732 y = 2a cos2 (π/4) = 2a(1/2) = a = 3.
as t approaches 2π/3. A line through (6, 3) with slope –1/2 is
tangent to the graph at that point.
[The exact answer is − 3, which students
will be able to prove easily with l’Hospital’s y
10
rule after they have studied Section 6-5. Joan
Gell and Cavan Fang have shown clever
5 t = π/4
trigonometric transformations that “remove”
the removable discontinuity and lead to the x
same answer. These are –15 –10 –5 0 5 10 15

1. Use the sum and product properties on –5

dy/dx:
dy 2 sin 1.5t sin 0.5t 11. a. x = cos t + t sin t
= y = sin t – t cos t
dx −2 sin 1.5t cos 0.5t
The grapher confirms the figure in the text.
= − tan 0.5t if dx/dt ≠ 0 [Note: In the derivation of these equations
As t → 2π /3, dy/dx → – tan(π /3) = – 3 . from the geometric definition of involute,

Calculus Solutions Manual Problem Set 4-7 67


© 2005 Key Curriculum Press
x = cos t + t cos (t – π/2) π
2
π
t⇒
x – 25 x – 25 
y = sin t + t sin (t – π /2) d. = sin = sin 2 t
15 30  15  30
(cos t, sin t) is the point of tangency of the π
2
π
t⇒
y – 20 y – 20 
string. = cos = cos 2 t
15 30  15  30
Because the circle is a unit circle, the length
π π
of the string is also t, the central angle in Because sin 2 t + cos 2 = 1,
radians. 30 30
2 2
The string makes an angle of (t – π/2) with  x – 25  +  y – 20  = 1.
the positive x-axis so that  15   15 
(t cos (t – π/2), t sin (t – π/2)) is a vector This is an equation of a circle centered at
representing the unwound string. (25, 20) with radius 15, confirming that the
The cofunction properties and odd-even path really is a circle.
properties from trig are used to simplify the 13. The actual solutions will vary depending on the
equations so that the calculus will be easier.] period of the pendulum, as determined by the
dy cos t – [cos t + t (– sin t )] length of the string. The following solution
b. = supposes that the period turns out to be 3.1
dx – sin t + (sin t + t cos t )
t sin t seconds.
= = tan t 2π 2π
t cos t x = 30 cos t y = 20 sin t
3.1 3.1
c. At t = π, dy/dt = tan π = 0. The string will 2π
( 40π /3.1) cos t
be pointing straight up from the x-axis. The dy
= 3.1 = – 2 cot 2π t
diagram shows that the tangent to the graph is dx –(60π /3.1) sin 2π t 3 3.1
horizontal at this point. 3.1
y
( x, y)
At t = 5, x ≈ –22.8, y ≈ –13.0,
and dy/dx ≈ –0.78.
String If the measurements have been accurate, the
1
t=π pendulum will be above the coin when t = 5.
x
1 14. The graph looks like an ellipse that moves in the
x-direction as t increases. Because y starts at a
high point and varies between 5 and 1, the ellipse
has center at y = 3 and y-radius 2. Thus, an
12. a. x starts at a middle point and increases.
equation for y would be y = 3 + 2 cos t.
y starts at a high point and decreases.
x starts at 0 and increases. If the ellipse had
∴ x = 25 + 15 sin Bt
x-radius 0.5, an equation for x would be
y = 20 + 15 cos Bt
x = 0.5 sin t. The graph of this ellipse is
The period is 60 seconds.
So B = 2π /60 = π /30 y
π 5
∴ x = 25 + 15 sin t
30
π
y = 20 + 15 cos t
30 x

π π 10
b. dx/dt = cos t
2 30
π π The graph seems to move over 1 unit to the right
dy/dt = – sin t each cycle. Thus, if t increases by 2π, x increases
2 30
by 1. The equations are thus
At t = 5,
x = t/(2π ) + 0.5 sin t, y = 3 + 2 cos t
π π
dx/dt = cos = π 3 /4 = 1.3603K The graph here duplicates the one in the text.
2 6
π π y
dy/dt = – sin = –π /4 = –0.7853K 5
2 6
c. The slope of the circular path is dy/dx.
At t = 5,
x
dy – π /4
= = –1 / 3 = –0.5773K 10
dx π 3 /4

68 Problem Set 4-7 Calculus Solutions Manual


© 2005 Key Curriculum Press
To locate “interesting” features, d. n = 1. (x = cos t, y = sin t)
dy dy/dt –2 sin t
= = . 1 y
dx dx/dt 1/(2π ) + 0.5 cos t
For horizontal tangents, dy/dt = 0 and dx/dt ≠ 0.
∴ 2 sin t = 0 ⇔ t = 0 + πn (n an integer) 1
x

Thus, x = 0, 0.5, 1, 1.5, … .


For vertical tangents, dx/dt = 0 and dy/dt ≠ 0.
∴ 1/(2π) + 0.5 cos t = 0 ⇔ cos t = –1/π
Solving numerically for t gives
t = 1.8947… + 2π n or 4.3884… + 2πn. n = 2. (x = cos 2t, y = sin t)
For crossing points, x = 0.5, 1.5, 2.5, …
from symmetry on the graph. If x = 0.5, then 1 y

1/(2π )t + 0.5 sin t = 0.5.


Solving numerically for the value of t closest to x
0, t = 0.8278… . 1

y(0.8278…) = 3 + 2 cos 0.8278… = 4.3529…


A crossing point is (0.5, 4.3529…) at t =
0.8278… .
15. a. The grapher confirms the figure in the text. If n = 1, the graph is a circle.
b. (x = cos 4t, y = sin t) If n = 2, the graph is a parabola.
e. Jules Lissajous (1822–1880) lived in France.
1 y
Nathaniel Bowditch (1773–1838) lived in
Massachusetts.
x
1

Problem Set 4-8


Q1. y′ = 2001x2000 Q2. y′ = ln (2001)2001x
If n is an even number, the graph comes to Q3. 5 Q4. f ′(u) = –csc2 u
endpoints and retraces its path, making two Q5. product Q6. 1/(1 + 9x2)
complete cycles as t goes from 0 to 2π. Q7. x3 + C Q8. Instantaneous rate
If n is an odd number, the graph does not Q9. Q10. –2.4033… ft/s
come to endpoints. It makes one complete
4 y
cycle as t goes from 0 to 2π.
c. i. (x = cos 5t, y = sin t)
x
y' 6
1 y

x
1
1. x 3 + 7y 4 = 13 ⇒ 3x 2 + 28y 3y′ = 0 ⇒
3x 2
y′ = –
28 y 3
ii. (x = cos 6t, y = sin t) 15 x 4
2. 3x 5 − y 4 = 22 ⇒ 15x 4 − 4y 3y′ = 0 ⇒ y′ =
4 y3
1 y
1
3. x ln y = 104 ⇒ 1 · ln y + x ⋅ ⋅ y′ = 0 ⇒
y
x
− y ln y
1 y′ =
x
4. y = 213 ⇒ p xy′ + yex = 0 ⇒ y′ = –y
ex

Calculus Solutions Manual Problem Set 4-8 69


© 2005 Key Curriculum Press
5. x + xy + y = sin 2x ⇒ y′(–x – x sin xy) = y + y sin xy ⇒
1 + y + xy′ + y′ = 2 cos 2x ⇒ y(1 + sin xy) y
y′(x + 1) = 2 cos 2x – 1 – y ⇒ y′ = ⇒ y′ = –
x (–1 – sin xy) x
2 cos 2 x – 1 – y
y′ = 17. sin y = x ⇒ cos y · y′ = 1 ⇒ y′ = sec y
x +1
18. cos y = x ⇒ –sin y · y′ = 1 ⇒ y′ = –csc y
6. cos xy = x – 2y ⇒
(–sin xy) ( y + xy′) = 1 – 2y′ ⇒ 19. csc y = x ⇒ –csc y cot y · y′ = 1 ⇒
y′(–x sin xy + 2) = 1 + y sin xy ⇒ y′ = –sin y tan y
1 + y sin xy 20. cot y = x ⇒ –csc2 y · y′ = 1 ⇒ y′ = –sin2 y
y′ =
2 – x sin xy 21. y = cos− 1 x ⇒ cos y = x ⇒ –sin y · y′ = 1 ⇒
7. x0.5 – y0.5 = 13 ⇒ 1 1
y′ = – =–
0.5x–0.5 – 0.5y–0.5 y′ = 0 ⇒ y′ = y 0.5 /x0.5 sin y 1 – x2
8. x1.2 + y 1.2 = 64 ⇒ 1.2x0.2 + 1.2y0.2 y′ = 0 ⇒
y′ = –x 0.2 /y0.2 1
√1 – x 2
9. e xy = tan y ⇒ exy(1 · y + x · y′) = y′sec2 y ⇒ ye xy y
+ xy′e xy = y′sec2 y ⇒ xy′e xy – y′sec2 y = –ye xy x
− ye xy
⇒ y′(xexy – sec2 y) = –yexy ⇒ y′ =
xe xy − sec 2 y 1 1
22. y = ln x ⇒ e y = x ⇒ e y · y′ = 1 ⇒ y′ = =
10. ln (xy) = tan x ⇒ tan (ln xy) = x ⇒
–1 ey x
 1 23. y = x 11/5 ⇒ y 5 = x 11 ⇒ 5y 4 · y′ = 11x 10 ⇒
sec (ln xy) ·   (1 · y + y′x) = 1 ⇒ y + y′x =
2
11x 10 11x 10 11x 10 11 6 / 5
 xy  y′ = = = = x ,
xy cos (ln xy) y′x = xy cos2 (ln xy) – y ⇒
2 5y 4 5( x 11/ 5 ) 4 5 x 44 / 5 5
xy cos 2 (ln xy) − y which is the answer obtained using the derivative
y′ = of a power formula, Q.E.D.
x
24. Prove that if y = xn, where n = a/b and a and b
11. (x y ) = x – y ⇒
3 4 5
are integers, then y′ = nan− 1.
5(x 3y 4)4(3x 2y 4 + x 3 · 4y 3y′) = 1 – y′ ⇒
y′(20x15y19 + 1) = 1 – 15x14y20 ⇒ Proof:
1 – 15 x 14 y 20
y′ = y = x n = x a/ b ⇒ y b = x a.
1 + 20 x 15 y19 Because a and b are integers,
12. (xy)6 = x + y ⇒ byb− 1 y′ = ax a− 1
6(xy)5(y + xy′) = 1 + y′ ⇒ ax a –1 ax a –1 ax a –1 a a−1−( a− a/b )
y′ = b –1 = = a – a/b = x
y′(6x6y5 – 1) = 1 – 6x5y6 ⇒ by a/b b –1
b( x ) bx b
1 – 6 x 5 y6 a
y′ = 6 5 = x a/b –1 = nx n –1, Q .E.D .
6x y – 1 b
13. cos 2 x + sin2 y = 1 ⇒ 25. a. x 2 + y 2 = 100
2 cos x · (–sin x) + 2 sin y · cos y · y′ = 0 ⇒ At (–6, 8), (–6)2 + 82 = 100, which shows
cos x sin x that (–6, 8) is on the graph, Q.E.D.
y′ =
cos y sin y b. x 2 + y 2 = 100 ⇒ 2x + 2y · dy/dx = 0 ⇒
14. sec2 y – tan2 x = 1 ⇒ dy/dx = –x/y
2 sec y · sec y tan y · y′ – 2 tan x · sec2 x = 0 ⇒ At (–6, 8), dy/dx = –(–6)/8 = 0.75.
sec 2 x tan x A line at (–6, 8) with slope 0.75 is tangent to
y′ = the graph, showing that the answer is
sec 2 y tan y
reasonable.
15. tan xy = xy ⇒
10 y
(sec 2 xy) · (y + xy′) = y + xy′ ⇒
y′(x sec2 xy – x) = y – y sec2 xy ⇒
y(1 – sec 2 xy) y
y′ = 2 ⇒ y′ = – x
x (sec xy – 1) x 10

16. cos xy = xy ⇒
(–sin xy) · (y + xy′) = y + xy′ ⇒

70 Problem Set 4-8 Calculus Solutions Manual


© 2005 Key Curriculum Press
c. x = 10 cos t y = 10 sin t b. y = x: x 3 + x 3 = 64 ⇒ x 3 = 32 ⇒
dy 10 cos t cos t x = 3.1748…
= =–
dx –10 sin t sin t dy/dx = –x2/y2 = –x2/x2 = –1
At x = –6, t = cos (–0.6).
–1
c. y = (64 – x 3)1/3
sin [cos–1 (–0.6)] = 0.8 As x becomes infinite, (64 – x3)1/3 gets closer
dy –0.6 to (–x3)1/3, which equals –x. The graph has
∴ =– = 0.75,
dx 0.8 a diagonal asymptote at y = –x, and
which agrees with part b, Q.E.D. dy/dx → –1.
26. a. x 2 – y 2 = 36 d. By analogy with the equation of a circle, such
At (10, –8), 102 – (–8)2 = 36, which shows as x 2 + y 2 = 64
that (10, –8) is on the graph, Q.E.D. 28. a. First simplify the equation.
b. x2 – y2 = 36 ⇒ 2x – 2y · dy/dx = 0 ⇒ [(x – 6)2 + y2][(x + 6)2 + y2] = 1200
dy/dx = x/y (x – 6)2(x + 6)2 + (x – 6)2y2 + (x + 6)2y2 + y4
At (10, –8), dy/dx = 10/(–8) = –1.25. = 1200
A line at (10, –8) with slope –1.25 is tangent (x2 – 36)2 + (x2 – 12x + 36 + x2 + 12x + 36)y2
to the graph, showing that the answer is + y4 = 1200
reasonable. x − 72x 2 + 1296 + 2x 2y 2 + 72y 2 + y 4 = 1200
4

10 y x 4 − 72x 2 + 2x 2y 2 + 72y 2 + y 4 = −96


Differentiate the simplified equation
implicitly.
x 4x 3 − 144x + 4xy 2 + 4x 2y · dy/dx
10 + 144y · dy/dx + 4y3 · dy/dx = 0
(4x y + 144y + 4y3) · dy/dx = −4x3
2

+ 144x − 4xy 2
dy – x 3 + 36 x – xy 2
c. x = 6 sec t y = 6 tan t = 2
dx x y + 36 y + y 3
dy 6 sec t tan t tan t
= = At x = 8: (4 + y2)(196 + y2) = 1200
dx 6 sec 2 t sec t 784 + 200y2 + y4 = 1200
At x = 10, t = ±sec (10/6).
–1
y 4 + 200y 2 − 416 = 0
tan [±sec–1 (10/6)] = ±8/6. –200 ± 41664
Choose the negative value because y < 0. y2 = = 2.058806 K or
dy –10/6 2
∴ = = –1.25, −202.0…
dx 8/6 y = ±1.4348542… (No other real solutions)
which agrees with part b, Q.E.D. At (8, 1.434…), dy/dx = −1.64211… .
27. a. x 3 + y 3 = 64 ⇒ 3x 2 + 3y 2 · dy/dx = 0 ⇒ At (8, –1.434…), dy/dx = 1.64211… .
dy/dx = –x2/y2 Both answers agree with the moderately steep
x = 0: y 3 = 64 ⇒ y = 4 negative and positive slopes, respectively.
∴ dy/dx = –0/16 = 0
y
The tangent is horizontal (see the next graph). 5

x = 2: 8 + y 3 = 64 ⇒ y 3 = 56 ⇒
y = 3.8258… x

∴ dy/dx = –22/(3.8258…)2 = –0.2732… 10

The tangent line has a small negative slope,


which agrees with the graph.
x = 4: 64 + y 3 = 64 ⇒ y = 0
∴ dy/dx = –42/0, which is infinite. b. At the x-intercepts, y = 0.
The tangent line is vertical. ∴ (x − 6)2 (x + 6)2 = 1200
10 y
(x2 − 36)2 = 1200
x = ± 36 ± 1200 = ±8.4048K or
±1.1657…
x Derivative appears to be infinite at each
10 x-intercept.
At x = 36 + 1200 = 8.4048K ,

Calculus Solutions Manual Problem Set 4-8 71


© 2005 Key Curriculum Press
dy –(8.4 K)3 + 36(8.4 K) – (8.4 K)(0) A = πr 2 ⇒
dA
= 2πr
dr

dr
=
6
=
dx (8.4 K)2 (0) + 36(0) + 0 3 dt dt dt πr
896.29K
= , which is infinite, as conjectured. dr /dt
0
c. From part a,
x 4 − 72x 2 + 2x 2y 2 + 72y 2 + y 4 = −96 ⇒
y 4 + (2x 2 + 72)y 2 + (x 4 − 72x 2 + 96) = 0
1
r
y2 = 3

−(2 x 2 + 72) ± (2 x 2 + 72)2 − 4(1)( x 4 − 72 x 2 + 96)


2 dr 2
= = 0.6366 K mm/h when r = 3 mm.
y = − x − 36 ± 144 x 2 – 1200
2 2
dt π
dr
Only the positive part of the ambiguous varies inversely with the radius.
sign ± gives real solutions for y. dt
dr dV
y = ± – x 2 – 36 + 144 x 2 – 1200 2. Know: = 2 cm/s. Want: .
dt dt
Plot the graph letting y1 equal the positive 4 dV dr
branch and y2 equal the negative branch. The V = πr 3 ⇒ = 4πr 2
3 dt dt
graph is as in the text. The two loops may dV
not appear to close, depending on the window = 72π = 226.1946 K cm 3 /s at r = 3 cm
dt
you use for x. dV
= 288π = 904.7786 K cm 3 /s at r = 6 cm
d. Repeating the algebra of parts a and c with dt
1400 in place of 1200 gives
dV/dt
y = ± – x 2 – 36 + 144 x 2 – 1400
Plot the graph as in part a. The two ovals in
the original graph merge into a single closed 500
figure resembling an (unshelled) peanut.
r
y 3 6
5

x The graph shows that the larger the balloon gets,


10
the faster Phil must blow air to maintain the
2 cm/s rate of change of radius.
dA da
3. Know: = −144 cm 2 /s. Want: .
dt dt
e. The two factors in the equation 1
[(x − 6)2 + y2][(x + 6)2 + y2] = 1200 A = πab and a = 2b ⇒ A = πa 2
2
are the squares of the distances from (x, y) to dA da da 1 dA
the points (6, 0) and (−6, 0), respectively. = πa ⇒ =
dt dt dt πa dt
The product of the distances is 1200, a da 6
constant. b = 12 ⇒ a = 24 ⇒ = − = −1.9098K
dt π
≈ −1.91 cm/s
The length of the major axis is 2a, so the major
Problem Set 4-9
axis is decreasing at 12/π cm/s.
Q1. y 2 + 2xyy′ Q2. Implicit differentiation dK dm
4. Know: = 100, 000 MJ/s; = −20 kg/s.
Q3. Product rule Q4. Chain rule dt dt
Q5. Speeding up Q6. smaller dV
Want: . (Note: 1 megaJoule—MJ—is the
1 dx dt
Q7. cos x − x sin x Q8. energy required to accelerate a 1-kg mass by
x dt
1 km/s through a distance of 1 km; it can be
Q9. −2e + xe
−x −x
Q10. E expressed 1 MJ = 1 kg · km2/s2.)
dA dr
1. Know: = 12 mm2/h. Want: . 1
K = mV 2 ⇒
dK 1 2 dm
= V + mV
dV

dt dt 2 dt 2 dt dt

72 Problem Set 4-9 Calculus Solutions Manual


© 2005 Key Curriculum Press
dV 1 dK V dm dH H dW H dL
= − =− ⋅ − ⋅
dt mV dt 2 m dt dt W dt L dt
dV 100000 10(–20) 20 20
= − = 2.02 (km/s)/s =− (0.1) − 2 (–0.3)
dt 5000 ⋅ 10 2 ⋅ 5000 LW 2 LW
5. Let y = Milt’s distance from home plate. dH 20 ⋅ 0.1 20 ⋅ 0.3
Let x = Milt’s displacement from third base. b. =− + 2 = 0.02
dt 5 ⋅ 22 5 ⋅2
dx dy
Know: = −20 ft/s. Want: . Depth is increasing at 0.02 m/s.
dt dt
dy dx 8. Let L = distance between spaceships.
y = x + 90 ⇒ 2 y
2 2 2
= 2x dx dy
dt dt Know: = 80 km/s; = −50 km/s.
dy x dx –20 x dt dt
⇒ = ⋅ = dL
dt y dt x 2 + 90 2 Want: .
dt
dy /dt dL dx dy
L2 = x 2 + y 2 ⇒ 2 L = 2x + 2y
10 dt dt dt
dL 1
x ⇒ = (80 x – 50 y)
90 dt x 2 + y2
dL 1 –200
= (80 ⋅ 500 – 50 ⋅ 1200) = =
dt 1300 13
−15.3846…
dy
At x = 45, = −8.944 K ≈ −8.9 ft/s Distance is decreasing at about 15.4 km/s.
dt
(exact: −4 5 ). 9. a. Let x = distance from bottom of ladder to
dy wall. Let y = distance from top of ladder to
At x = 0, = 0 ft/s, which is reasonable because floor.
dt
dx dy
Milt is moving perpendicular to his line from 20 2 = x 2 + y 2 ⇒ 0 = 2 x + 2y ⇒
home plate. dt dt
dy x dx
6. Let y = displacement from stern to dock along =−
dt y dt
pier. Let x = displacement from bow to pier
along dock. Note that the velocity of the weight is −dy/dt,
dy dx so
Know: = −3 m/s. Want: . x dx
dt dt v=
2 dt
x + y = 200
2 2 2
400 – x
dx dy dx y dy 3y 4 6
2x + 2y =0⇒ =− = b. v = ⋅ ( −3) = − = −0.6123K ft/s
dt dt dt x dt 200 2 – y 2 384 4
dx 360 dx 40
At y = 120, = = 2.25 m/s. c. Here x = 20, = 2, so v = → ∞ (!!)
dt 160 dt 0
dx/dt 10. a.

10
W
D
1200 in.2

L
y
120 200
dL dW
Know: . Want: .
dt dt
7. a. Let L = length. Let W = width. Let H = depth dL dW
(meters). LW = 1200 ⇒ ⋅W + L⋅ =0
dt dt
dW dL −W
Know: = 0.1 m/s; = −0.3 m/s. ⇒
dW
= ⋅
dL
=−
1
W2 ⋅
dL
dt dt dt 1200/W dt 1200 dt
dH
Want: . 1
dt b. −2 = − W 2 (6) ⇒ W = 20 in. ⇒
LWH = 20 ⇒ 1200
dL dW dH L = 60 in.
⋅ WH + L ⋅ ⋅ H + LW ⋅ =0
dt dt dt

Calculus Solutions Manual Problem Set 4-9 73


© 2005 Key Curriculum Press
dD dL dW 1 dV 2 dr 1 dh
c. D2 = L2 + W 2 ⇒ 2 D = 2L + 2W V = πr 2 h ⇒ = πr h + πr 2
dt dt dt 3 dt 3 dt 3 dt

dD
=
1 L dL
+W
dW  dV 2 1
= π (8)(3)(7) + π (8) ( −6) =
2

dt 2
L + W2  dt dt  dt 3 3
−16π ft3/min = −50.2654…
At L = 60 and W = 20, Volume is decreasing at about 50.3 ft3/min.
dD 1
= [60(6) + 20( −2)] 13. a. Let ω = angular velocity in radians per day.
dt 20 + 60 2
2
2π 2π
320 ωE = , ωM =
= = 1.6 10 = 5.0596 K 365 687
4000 dθ
= ω E − ω M = 2π 
1 1 
Diagonal is increasing at about 5.06 in./min. − =
dt  365 687 
11. a. Let h = depth of water. Let r = radius of water 644π
at surface. Let V = volume of water. = 0.008068K ≈ 0.00807 rad/day
dh dV 250755
Know: = 5 m/h. Want: . −1
b. T = 
dt dt 1 1 
− = 778.7422 K ≈
1
V = πr 2 h  365 687 
3 778.7 days
r 3 3
By similar triangles, = ⇒r= h The next time after 27 Aug. 2003 when the
h 5 5 two planets will be closest is 779 days later,
2
1 3  3 3
∴ V = π h h = πh on 14 Oct. 2005 (or 15 Oct., if the planets
3 5  25 were aligned later than about 6:11 a.m. back
dV 9 2 dh on 27 Aug. 2003). Because the actual orbits
= πh
dt 25 dt of Earth and Mars are not as simple as
dV 81 previously assumed, the actual closest
At h = 3, = π = 16.2 π = 50.8938K ≈
dt 5 distances are not always the same. In fact, the
3
50.9 m /h. approach on 27 Aug. 2003 was the closest
dV dh one in nearly 60,000 years! Nor is the period
b. i. Know: = −2 m3/h. Want: .
dt dt between close approaches quite so simple.
dV 9 dh dh 25 dV The next close approach will actually be on
= πh 2 ⇒ =
dt 25 dt dt 9πh 2 dt 30 Oct. 2005, not 15 Oct.
dh –50 c. By the law of cosines,
= = −0.1105K ≈
dt 144π D 2 = 932 + 1412 − 2 · 93 · 141 cos θ
−0.11 m/h at h = 4 m
D = 28530 – 26226 cos θ million mi
dh
ii. → −∞ as h → 0 m dD 26226 sin θ dθ
dt d. =
dV dt 2 28530 – 26226 cos θ dt
c. i. Know: = k h.
26226 ⋅ 
dt 1 1 
dV – ⋅ 2π sin θ
= −0.5 at h = 4 ⇒ k = −0.25  365 687 
dt = million mi/day
2 28530 – 26226 cos θ
dV
= −0.25 h
1, 000, 000 ⋅ 26226 ⋅ 
dt 1 1 
– ⋅ 2π sin θ
dV  365 687 
ii. = −0.25 0.64 = −0.2 m 3 /h =
dt 24 ⋅ 2 28530 – 26226 cos θ
at h = 0.64 m
1, 092, 750, 000 ⋅ 
1 1 
− π ⋅ sin θ
dV
= −0.2 at h = 0.64 m ⇒  365 687 
iii.
dt = mi/h
28530 − 26226 cos θ
dh 25
= (–0.2) = −0.4317K ≈ To find out how fast D is changing today,
dt 9π (0.64)2
first determine how many days after 27 Aug.
−0.43 m/h
2003 it is today, then multiply that number
12. Let h = altitude. Let r = radius. Let V = volume dθ  1 1 
by = − 2π to find θ, then
of cone. dt  365 687 
dh dr dV
Know: = −6 ft/min; = 7 ft/min. Want: . substitute θ into the previous expressions.
dt dt dt

74 Problem Set 4-9 Calculus Solutions Manual


© 2005 Key Curriculum Press
dD dl
e. To maximize , plot the variable part of = −1.9963… units/s at x = −5 units.
dt dt
dD sin θ dl
, y= . = 2.6610… units/s at x = 2 units.
dt 28530 − 26226 cos θ dt
y
The length of AB is at a minimum when dl/dt = 0.
0.01
Use your grapher to solve 0.8e0.8x + 2x = 0.
At x = −0.3117… , the length of AB stops
θ
decreasing and starts increasing.
π 2π

Problem Set 4-10


From the graph, it is clear that the maximum Review Problems
occurs well before θ = π/2 (90°). Using the
R0. Answers will vary.
maximize feature, the maximum occurs at
θ ≈ 0.8505… , or 48.7…°. R1. a. x = g(t) = t3 ⇒ g′(t) = 3t2
(The exact value is cos− 1 (93/141). One can y = h(t) = cos t ⇒ h′(t) = −sin t
find this by finding (d/dt)(dD/dt) and setting it If f (t) = g(t) · h(t) = t3 cos t, then, for example,
equal to zero. One can also see this by f ′(1) = 0.7794… by numerical differentiation.
decomposing Earth’s motion vector into two g′(1) · h′(1) = 3(12) · (−sin 1) = −2.5244…
components—one toward/away from Mars and ∴ f ′(t) ≠ g′(t) · h′(t), Q.E .D.
the other perpendicular to the first. The rate of b. If f (t) = g(t)/h(t) = t3/cos t, then, for example,
change in D is maximized when all of Earth’s f ′(1) = 8.4349… by numerical differentiation.
motion is along the Earth-to-Mars component, g′(1)/h′(1) = 3(12)/(−sin 1) = 3.5651…
which occurs when the Earth-Mars-Sun ∴ f ′(t) ≠ g′(t)/h′(t), Q.E .D.
triangle has a right angle at Earth. c. y = cos t
In this case, cos θ = 141
93
.) x = t3 ⇒ t = x1/3 ⇒ y = cos (x1/3 )
f. θ =   2π t if t = days since
1 1 dy 1
− = − sin ( x 1/ 3 ) ⋅ x −2 / 3
 365 687  dx 3
27 Aug. 2003. dy 1
At x = 1, = −sin 1 ⋅ = −0.280490 K .
dx 3
 1 
D = 28530 – 26226 cos 
1 
– 2π t  If x = 1, then t = 11/3 = 1.

 365 687   dy/dt – sin t – sin 1
∴ = = = −0.280490 K ,
D dx/dt 3t 2 3
which equals dy/dx, Q.E .D.
200
R2. a. If y = uv, then y′ = u′v + uv′.
b. See the proof of the product formula in the
t text.
c. i. f (x) = x7 ln 3x ⇒
1000

3
The graph is not a sinusoid. The high and low f ′(x) = 7x6 ln 3x + x 7 ⋅ = 7x6 ln 3x + x6
3x
points are not symmetric.
ii. g(x) = sin x cos 2x ⇒
14. As B moves from negative values of x to positive g′(x) = cos x cos 2x − 2 sin x sin 2x
values of x, the length of AB decreases to about
iii. h(x) = (3x − 7)5(5x + 2)3
0.56 unit, then begins to increase when the x-
h′(x) = 5(3x − 7)4(3) · (5x + 2)3
value of point B passes about −0.3.
+ (3x − 7)5(3)(5x + 2)2(5)
Let l = length of AB.
dx dl = 15(3x − 7)4(5x + 2)2(5x + 2
Know: = 2 units/s. Want: . + 3x − 7)
dt dt
dl = 15(3x − 7)4(5x + 2)2(8x − 5)
l = e 0.8 x + x 2 ⇒ iv. s(x) = x8e− x ⇒ s′(x) = −x8e− x + 8x7e− x
dt
d. f (x) = (3x + 8)(4x + 7)
= (e 0.8 x + x 2 ) −1/ 2 ⋅  2 x + 0.8e 0.8 x 
1 dx dx
2  dt dt  i. f ′(x) = 3(4x + 7) + (3x + 8)(4) = 24x + 53
ii. f (x) = 12x2 + 53x + 56
0.8e 0.8 x + 2 x
= f ′(x) = 24x + 53, which checks.
e 0.8 x + x 2

Calculus Solutions Manual Problem Set 4-10 75


© 2005 Key Curriculum Press
u′ v – uv ′ The values get closer to 3.4255… as x
R3. a. If y = u/v, then y′ = .
v2 approaches 1 from either side, Q.E.D.
b. See proof of quotient formula in text. R4. a. i. y = tan 7x ⇒ y′ = 7 sec2 7x
sin 10 x ii. y = cot (x4) ⇒ y′ = −4x3 csc2 (x4)
c. i. f ( x ) = ⇒
x5
iii. y = sec e x ⇒ y′ = e x sec e x tan e x
10 cos 10 x ⋅ x 5 – sin 10 x ⋅ 5 x 4
f ′( x ) = iv. y = csc x ⇒ y′ = −csc x cot x
x 10
10 x cos 10 x – 5 sin 10 x b. See derivation in text for tan′x = sec2 x.
=
x6 c. The graph is always sloping upward, which
is connected to the fact that tan′ x equals the
(2 x + 3)9
ii. g( x ) = ⇒ g ′( x ) square of a function and is thus always
(9 x – 5) 4 positive.
9(2 x + 3)8 ⋅ 2(9 x − 5) 4 − (2 x + 3)9 ⋅ 4(9 x − 5)3 ⋅ 9
= y
(9 x − 5)8
18(2 x + 3)8 (5 x – 11)
= 1 x
(9 x – 5)5 ␲

iii. h(x) = (100x3 − 1)− 5 ⇒


h ′(x) = −5(100x3 − 1)− 6 · 300x2
= −1500x2(100x3 − 1)− 6
d. f (t) = 7 sec t ⇒ f ′(t) = 7 sec t tan t
d. y = 1/x 10
As a quotient: f ′ (1) = 20.17…
0 ⋅ x 10 – 1 ⋅ 10 x 9 –10 f ′ (1.5) = 1395.44…
y′ = = 11 = −10 x −11
x 20 x f ′ (1.57) = 11038634.0…
As a power: There is an asymptote in the secant graph at
y = x − 10 t = π /2 = 1.57079… . As t gets closer to this
y′ = −10x − 11, which checks. value, secant changes very rapidly!
sin x 3
e. t ( x ) = = tan x R5. a. i. y = tan −1 3 x ⇒ y′ =
cos x 1 + 9x 2
cos x cos x – sin x (– sin x ) d 1
t ′( x ) = ii. (sec –1 x ) =
cos 2 x dx | x | x2 – 1
cos 2 x + sin 2 x 1
= = = sec 2 x –2 cos –1 x
cos 2 x cos 2 x iii. c( x ) = (cos −1 x )2 ⇒ c′( x ) =
t ′ (1) = sec2 1 = 3.4255… 1 – x2
1
f. m( x ) =
t ( x ) – t (1) tan x – tan 1
= b. y = sin −1 x ⇒ y ′ =
x –1 x –1 1− x2
m (x) y
␲/2
3.42...

x
1
1
x
1

x m(x) 1
y′(0) = = 1 , which agrees with the
0.997 3.40959… 1 – 02
0.998 3.41488… graph.
1 1
0.999 3.42019… y′(1) = = , which is infinite.
1 undefined 1–1 2 0
1.001 3.43086… The graph becomes vertical as x approaches 1
1.002 3.43622… from the negative side. y′(2) is undefined
because y(2) is not a real number.
1.003 3.44160…
R6. a. Differentiability implies continuity.

76 Problem Set 4-10 Calculus Solutions Manual


© 2005 Key Curriculum Press
b. i. Answers may vary. ii. Answers may vary. 6t ( dt/dx ) ⋅ 2e 2 t − 3t 2 ⋅ 4e 2 t ( dt/dx )
=
f (x ) f (x ) (2e 2 t ) 2
3t − 3t 2 dx 3t − 3t 2
= ÷ =
e2t dt 2e 4 t
x x
c c
b. x = (t/π) cos t
y = (t/π ) sin t
dy dy/dt (1/π ) sin t + (t/π )(cos t )
iii. No such function. iv. Answers may vary. = =
dx dx/dt (1/π ) cos t + (t/π )(– sin t )
f (x ) sin t + t cos t
=
cos t – t sin t
Where the graph crosses the positive x-axis,
t = 0, 2π , 4π , 6π , … .
x
If t = 6π , x = 6 and y = 0.
c
∴ (6, 0) is on the graph.
c. i. If t = 6π, then
f (x ) dy sin 6π + 6π cos 6π 0 + 6π
= = = 6π .
dx cos 6π – 6π sin 6π 1– 0
2
So the graph is not vertical where it crosses
the x-axis. It has a slope of 6π = 18.84… .
x
1 c. At a high point, y is a maximum and x is zero.
Use cosine for y and sine for x.
ii. f is continuous at x = 1 because right and For y, the sinusoidal axis is at 25 ft.
left limits both equal 2, which equals f (1). For x, the sinusoidal axis is at 0 ft.
Both x and y have amplitude 20 ft, the radius
iii. f is differentiable. Left and right limits
of the Ferris wheel.
of f ′ (x) are both equal to 2, and f is
The phase displacement is 3 seconds.
continuous at x = 2.
The period is 20 seconds, so the coefficient
sin –1 x, if 0 ≤ x ≤ 1 of the arguments of sine and cosine is
d. g( x ) =  2
 x + ax + b, if x ≤ 0 2 π /20 = π /10.
π
(1 − x ) −1/2, if 0 < x < 1 x = 20 sin (t – 3)
g ′( x ) =  10
2 x + a, if x < 0 π
lim− g( x ) = 0 + 0 a + b = b y = 25 + 20 cos (t – 3)
x→0 10
lim g( x ) = sin −1 0 = 0 π
x→0 +
dx/dt = 2π cos (t – 3)
10
∴b=0 π
lim– g′( x ) = 0 + a = a dy/dt = −2π sin (t – 3)
x→0 10
lim g′( x ) = 1−1/2 = 1 When t = 0, dy/dt = 5.0832… .
x→0 +
The Ferris wheel is going up at about
∴a=1
5.1 ft/s.
When t = 0, dx/dt = 3.6931… .
g (x )

1 The Ferris wheel is going right at about


x 3.7 ft/s.
0 1 dy dy/dt
=
dx dx/dt
dy/dx will be infinite if dx/dt = 0 and
The graph appears to be differentiable and dy/dt ≠ 0.
π
continuous at x = 0. dx/dt = 0 if 2π cos (t – 3) = 0 .
10
dy dy/dt 3t 2
R7. a. x = e 2 t, y = t 3 ⇒ = = ⇒ π π
dx dx/dt 2e 2 t (t – 3) = + π n (where n is an integer)
10 2
d 2 y d  3t 2  t = 8 + 5n
=  
dx 2 dx  2e 2 t  The first positive time is t = 8 s.

Calculus Solutions Manual Problem Set 4-10 77


© 2005 Key Curriculum Press
R8. a. y = x 8/5 ⇒ y 5 = x 8 At z = 200, x = 200 2 – 70 2 = 30 39
8x 7 8x 7 8 3/ 5
5 y 4 y′ = 8 x 7 ⇒ y′ = 4 = 8/ 5 4 = x dz 20 ⋅ 30 39
5y 5( x ) 5 = = 3 39 = 18.7349...
dt 200
Using the power rule directly:
The glass moves at the same speed as the
y = x 8/5 ⇒ y′ = 85 x 3/ 5 tablecloth, or about 18.7 cm/s, which is about
b. y3 sin xy = x4.5 ⇒ 1.3 cm/s slower than Rover.
3y2y′ · sin xy + y3(cos xy)(y + xy′) = 4.5x3.5
Concept Problems
y′[3y2 sin xy + xy3 cos xy]
C1. a. Let (x, y) be the coordinates of a point on the
= 4.5x 3/5 − y 4 cos xy
tangent line.
dy 4.5 x 3.5 – y 4 cos xy
y′ = = 2 y – y0
= m ⇒ y = m( x − x 0 ) + y0
dx 3 y sin xy + xy 3 cos xy x – x0
c. i. 4y 2 − xy 2 = x 3 ⇒ b. Substituting (x1, 0) for (x, y) gives
8yy′ − y 2 − x · 2yy′ = 3x 2 y
0 = m( x1 − x 0 ) + y0 ⇒ x1 = x 0 − 0 , Q .E.D .
y′(8y − 2xy) = 3x 2 + y 2 m
dy 3 x 2 + y 2 c. The tangent line intersects the x-axis at (x2, 0).
y′ = = Repeating the above reasoning with x2 and x1
dx 8 y – 2 xy
in place of x1 and x0 gives
At (2, 2), dy/dx = 2. At (2, −2), dy/dx = −2. y
Lines at these points with these slopes are x 2 = x1 − 1
m
tangent to the graph (see diagram). Because y1 = f (x1) and m = f ′(x1),
y
5 f ( x1 )
x 2 = x1 − , Q .E .D .
f ′( x1 )
x
d. Programs will vary according to the kind of
2 5 grapher used. The following steps are needed:
• Store f (x) in the Y= menu.
• Input a starting value of x.
• Find the new x using the numerical
ii. At (0, 0), dy/dx has the indeterminate form derivative.
0/0, which is consistent with the cusp. • Display the new x.
iii. To find the asymptote, solve for y. • Save the new x as the old x and repeat.
(4 − x)y 2 = x 3
For f (x) = x2 − 9x + 14, the program should
x3
y2 = give x = 2, x = 7.
4–x
e. For g(x) = x3 − 9x2 + 5x + 10, first plot the
As x approaches 4 from the negative side,
graph to get approximations for the initial
y becomes infinite. If x > 4, y2 is negative,
values of x.
and thus there are no real values of y.
Asymptote is at x = 4. g (x )

R9.
20
x
z 1
70

Let x = Rover’s distance from the table.


Let z = slant length of tablecloth. Run the program three times with x0 = −1, 1,
dx dz and 8. The values of x are
Know: = 20 cm/s. Want: at z = 200.
dt dt x = −0.78715388…
z = x + 70
2 2 2
x = 1.54050386…
dz dx
2z = 2x x = 8.24665002…
dt dt
dz x dx 20 x The answers are the same using the built-in
= =
dt z dt z solver feature. The same preliminary analysis
is needed to find starting values of x.

78 Problem Set 4-10 Calculus Solutions Manual


© 2005 Key Curriculum Press
f. f (x) = sec x − 1.1 a

Starting with x0 = 1, it takes seven iterations 500,000


to get x = 0.429699666… . θ
C2. a. The connecting rod, the crankshaft, and the 3

y-axis form a triangle with angle φ = θ − π /2


included between sides of 6 cm and
(y − 8) cm.
Solving graphically and numerically,
8 a < −980 for θ ∈ (0.2712… , 2.8703…). The
piston is going down (v < 0) for
θ ∈ (π /2, 3π /2).
y–8
20 So the piston is going down with acceleration
φ
θ greater than gravity for θ between π /2 and
2.8703… .
6

By the law of cosines, Chapter Test


20 2 = (y − 8)2 + 62 − 2 · 6 ·
T1. y = uv ⇒ y′ = u′v + uv′
(y − 8) cos (θ − π /2)
 u + ∆u − u 
202 = (y − 8)2 + 62 − 12(y − 8) sin θ  v + ∆v v (v + ∆v)v 
T2. y ′ = lim  ⋅
(y − 8)2 − 12 sin θ (y − 8) − 364 = 0 ∆x →0
 ∆x (v + ∆v)v 
Solve for y − 8 using the quadratic formula.  
y − 8 = 6 sin θ + (36 ⋅ sin 2 θ + 364) . (The  (u + ∆u)v − u(v + ∆v) 
= lim  
solution with the negative radical gives a
∆x →0  ∆x (v + ∆v)v 
triangle below the origin, which has no real-  uv + ∆uv − uv − u∆v 
life meaning.) = lim  
∆x →0  ∆x (v + ∆v)v 
y = 8 + 6 sin θ + 2 9 ⋅ sin 2 θ + 91  1 ∆u ⋅ v − u∆v 
= lim  ⋅ 
dy dθ 18 sin θ cos θ dθ ∆x →0  ( v + ∆v )v ∆x 
b. v = = 6 cos θ +
9 ⋅ sin 2 θ + 91 dt  ∆ ∆ 
⋅ v − u 
dt dt 1 u v
= lim 
dθ 9 sin 2θ dθ ∆x →0  ( v + ∆v )v  ∆x ∆x  
v = 6 cos θ +
1  du u ′v − uv ′
v−u  =
dt 9 ⋅ sin θ + 91 dt
2 dv
= 2 
d2y v dx dx  v2
c. a = [Because as ∆x → 0, u/x and ∆v/∆x become
dt 2

2 du/dx and dv/dx and v → 0, so (v + ∆v)v → v 2 ].
= −6 sin θ  
 dt  T3. cot x =
cos x
sin x
91 cos 2θ – 9 sin 4 θ  dθ 
2
+ 18 − sin x sin x − cos x cos x
(9 sin 2 θ + 91)3/ 2  dt  =
sin 2 x
 91 cos 2θ – 9 sin 4 θ   dθ  2 −(sin 2 x + cos 2 x )
= 18 ⋅ – 6 sin θ   =
1
= − 2 = − csc 2 x
 (9 sin 2 θ + 91)3/ 2  dt sin 2 x sin x
(There are many other correct forms of the T4. y = sin −1 x ⇒ sin y = x ⇒ y′ cos y = 1 ⇒
answer, depending on how you use the 1
double-argument properties and Pythagorean y′ = cos 2 y + sin 2 y = 1 ⇒ cos 2 y = 1 − sin 2 y ⇒
cos y
properties from trigonometry.)
1
Note that the angular velocity is constant at cos y = 1 − sin 2 y = 1 − x 2 ⇒ y =
6000π radians per minute, so 1− x2
dθ dy dy/dt 4t 3 d2y d
= 100π rad/s. T5. = = = 2t 2 2 = (2t 2 ) =
dt dx dx/dt 2t dx dx
d. See the graph. Note that a line at a = −980 dx 4t
is so close to the x-axis that it does not 4t ( dt/dx ) = 4t ÷ = =2
dt 2t
show up.

Calculus Solutions Manual Problem Set 4-10 79


© 2005 Key Curriculum Press
T6. c(x) = cot 3x T15. y = 4 sin− 1 (5x3)
c′(x) = −3 csc2 3x, which is negative for all 1 60 x 2
y′ = 4 ⋅ ⋅ 15 x 2 =
permissible values of x. 1 – (5 x 3 ) 2 1 – 25 x 6
c′(5) = −3 csc2 15 = −3/sin2 15 = −7.0943… T16. 9x 2 − 20xy + 25y 2 − 16x + 10y − 50 = 0 ⇒
c(t) is decreasing at about 7.1 y-units/x-unit. 18x − 20y − 20xy ′ + 50yy ′ − 16 + 10y ′ = 0
T7. f (x) = sec x ⇒ f ′ (x) = sec x tan x y ′(−20x + 50y + 10) = −18x + 20y + 16
f ′ (2) = sec 2 tan 2 = 5.25064633… dy –18 x + 20 y + 16
y′ = =
Use m(x) for the difference quotient. dx –20 x + 50 y + 10
1/cos x – 1/cos 2 –9 x + 10 y + 8
m( x ) = =
x–2 –10 x + 25 y + 5
If x = −2, then
x m(x)
36 + 40y + 25y 2 + 32 + 10y − 50 = 0
1.997 5.28893631… 25y 2 + 50y + 18 = 0
1.998 5.27611340… Solving numerically gives
1.999 5.26335022… y = −0.4708… or y = −1.5291… , both of
2.000 undefined which agree with the graph.
2.001 5.23800134… (Solving algebraically by the quadratic formula,
2.002 5.22541482… y = −1 ± 7 /5 , which agrees with the numerical
2.003 5.21288638… solutions.)
At (−2, −0.4708…), dy/dx = 1.60948… .
T8. Answers may vary.
At (−2, −1.5291…), dy/dx = −0.80948… .
f (x ) The answers are reasonable, because lines of
7
these slopes are tangent to the graph at the
respective points, as shown here.
5 y

x
–2 1 2

–2 x
T9. f (x) = mx + b 5

f ′ (x) = m for all x


∴ f is differentiable for all x.
∴ f is continuous for all x, Q.E.D.
T10. f (x) = sec 5x ⇒ f ′(x) = 5 sec 5x tan 5x  x 3 + 1, if x ≤ 1
T17. f ( x ) = 
T11. y = tan7/3 x ⇒ y′ = 37 tan4/3 x a( x – 2) + b, if x > 1
2

T12. f (x) = (2x – 5)6(5x – 1)2 3 x 2 , if x < 1


f ′ (x) = 6(2x – 5)5(2) ⋅ (5x – 1)2 f ′( x ) = 
+ (2x – 5)6 ⋅ 2(5x – 1) ⋅ 5 2 a( x – 2), if x > 1
= 2(2x – 5)5(5x – 1)[6(5x – 1) + 5(2x – 5)] For equal derivatives on both sides of x = 1,
= 2(2x – 5)5(5x – 1)(40x – 31) lim– f ′( x ) = 3 ⋅ 12 = 3
x →1
e3 x lim f ′( x ) = 2 a(1 − 2) = −2 a
T13. f ( x ) = ⇒ x →1+
ln x
∴ −2 a = 3 ⇒ a = −1.5
3e 3 x ln x − e 3 x (1/ x ) 3 xe 3 x ln x − e 3 x
f ′( x ) = = For continuity at x = 1,
(ln x )2 x (ln x )2
lim– f ( x ) = 13 + 1 = 2
T14. x = sec 2t x →1

y = tan 2t3 lim f ( x ) = a(1 − 2) 2 + b = a + b


x →1+
dy dy/dt sec 2 2t 3 ⋅ 6t 2 3t 2 sec 2 2t 3 ∴a + b = 2
= = =
dx dx/dt sec 2t tan 2t ⋅ 2 sec 2t tan 2t Substituting a = −1.5 gives b = 3.5.

80 Problem Set 4-10 Calculus Solutions Manual


© 2005 Key Curriculum Press
The graph shows differentiability at x = 1. T19. cot = adjacent/opposite = x/5 = cot− 1 (x/5)
dθ 1 1 1
f (x )
T20. =− ⋅ =−
dx 1 + ( x/5)2 5 5 + 5( x 2 / 25)
1 5
2 =− =−
5 + ( x 2 / 5) 25 + x 2
x
dx
1 T21. = −420 mi/h
dt
dθ dθ dx 5 2100
Values of b other than 3.5 will still cause the T22. = ⋅ =− ⋅ ( −420) =
two branches to have slopes approaching 4 dt dx dt 25 + x 2 25 + x 2
as x approaches 1 from either side as long as T23. The plane is changing fastest when x approaches
a = −1.5. However, f will not be continuous, zero, when the plane is nearest the station.
and thus will not be differentiable, as shown y

here for b = 4.5.


f (x )

25
x
2 30

x
T24. Answers will vary.
1

T18. y = x 7/3 ⇔ y 3 = x 7
3y 2y′ = 7x 6
7x 6 7 x 6 7 7
y′ = 2 = = x 6−14 / 3 = x 4 / 3
3y 3 ( x 7/ 3 ) 2 3 3
This answer agrees with y′ = nxn− 1. 4/3 is
7/3 − 1.

Calculus Solutions Manual Problem Set 4-10 81


© 2005 Key Curriculum Press
Chapter 5—Definite and Indefinite Integrals

Problem Set 5-1 Q7. 1 Q8. That constant.


1. f (1000) = 20 + (0.000004)(10002) = 24 $/ft Q9. 0 Q10. B
f (4000) = 20 + (0.000004)(40002) = 84 $/ft 1. f (x) = 0.2x4 ⇒ f ′ (x) = 0.8x3 ⇒ f ′ (3) = 21.6;
The price increases because it is harder and slower f (3) = 0.2(34) = 16.2
to drill at increasing depths. ∴ y − 16.2 = 21.6(x − 3) ⇒ y = 21.6x − 48.6
2. T6 = 500(24 + 29)/2 + 500(29 + 36)/2
+ 500(36 + 45)/2 + 500(45 + 56)/2 x f (x) y Error
+ 500(56 + 69)/2 + 500(69 + 84)/2 3.1 18.47042 18.36 0.11042
= 13,250 + 16,250 + 20,250 + 25,250 3.001 16.22161… 16.2216 0.0000108…
+ 31,250 + 38,250 = 144,500 2.999 16.17841… 16.1784 0.0000107…
About $144,500, an overestimate because the 2. g (x) = sec x ⇒ g ′ (x) = sec x tan x ⇒
trapezoids are circumscribed above the curve.
(Note that T6 can be found more easily by first g ′ (π /3) = 2 3 = 3.464…
factoring out the 500, then adding the function g (π /3) = sec (π /3) = 2
values.) Linear function is y − 2 = 2 3 (x − π /3) ⇒
3. R6 = 500(26.25) + 500(32.25) + 500(40.25) + y = 2 3 (x − π /3) + 2.
500(50.25) + 500(62.25) + 500(76.25) = 143,750
About $143,750 dx f (x) y Error
(Note that R6 can be found more easily by first 0.04 2.15068… 2.13856… 0.01212…
factoring out the 500, then adding the function
values.) − 0.04 1.87184… 1.86143… 0.01041…
R6 is close to T6. (They differ by less than 1%.) 0.001 2.003471… 2.003464… 7.01 × 10− 6
4. T100 = 144,001.8, T500 = 144,000.072 3. a. f (x) = x 2 ⇒ f ′ (x) = 2x ⇒ f ′ (1) = 2
Conjecture: Exact value is $144,000. Tangent line: y − 1 = 2(x − 1) ⇒ y = 2x − 1
5. g (x) = 20x + 13 (0.000004)x 3 + C The graph shows a zoom by factor of 10.
g (4,000) − g (1,000) = (165,333.3333… + C) −
(21,333.3333… + C) = 144,000,
which is the conjectured value of the definite
integral! 1
The other name for antiderivative is indefinite
graph tangent line
integral. 1
6. a. f (x) = x 7 + C b. y = − cos x + C
c. u = 0.5e + C
2x
d. v = 321 (4x + 5)8 + C Local linearity describes the property of the
function because if you keep x close to 1 (in
the “locality” of 1), the curved graph of the
Problem Set 5-2 function looks like the straight graph of the
tangent line.
Q1. Answers may vary. b.
y Area = product
x f (x) y Error, f (x) − y
of x and y
0.97 0.9409 0.94 0.0009
x
0.98 0.9604 0.96 0.0004
a b 0.99 0.9801 0.98 0.0001
1 1 1 0
Q2. Instantaneous rate of change 1.01 1.0201 1.02 0.0001
Q3. f ′ (x) = −(ln 2)2− x Q4. y = sin x + C 1.02 1.0404 1.04 0.0004
Q5. y′ = 4 m/s Q6. sec x tan x (derivative) 1.03 1.0609 1.06 0.0009

82 Problem Set 5-2 Calculus Solutions Manual


© 2005 Key Curriculum Press
The table shows that for x-values close to 1 (the V ≈ 43 π (63) + 4.32π = 288π + 4.32π =
point of tangency), the tangent line is a close
292.32π ≈ 918.350 mm3
approximation to the function values.
4
4. f (x) = x2 − 0.1(x − 1)1/3 Actual volume is V = π (6.033) =
3
Zooming in on (1, 1) shows that the graph goes
vertical at x = 1. This observation is confirmed 292.341636π ≈ 918.418 mm3.
algebraically. ∆V = 43 π (6.033 ) − 43 π (6 3 ) = 4.341636π ≈
f ′ (x) = 2x − (1/3)(0.1)(x − 1) − 2/3 13.640 mm3
f ′ (1) = 2 − (1/3)(0.1)(0) − 2/3 , which is infinite. Error is 292.32π − 292.341636π = − 0.021636π,
or about 0.068 mm3 too low.
7. a. (6000 · 0.05)/365 = 0.8219… , or about
1
82 cents.
graph b. m = 6000e (0.05/365)t ⇒
tangent line
1 dm = 6000(0.05/365)e(0.05/365)t dt
Substituting t = 0 and dt = 1 gives dm =
f does not have local linearity at x = 1. Because 0.8219… , the same as part a.
the slope of the graph becomes infinite, no linear Substituting t = 0 and dt = 30 gives dm =
function can approximate the graph there. If f is 24.6575… ≈ $24.66.
differentiable at x = c, then f is locally linear Substituting t = 0 and dt = 60 gives dm =
there. The converse is also true. If f is locally 49.3150… ≈ $49.32.
linear at x = c, then f is differentiable there. c. t = 1: ∆m = 6000e (0.05/365)(1) − 6000 =
5. a. Let A be the number of radians in θ degrees. 0.8219… , almost exactly equal to dm.
By trigonometry, tan A =
x
⇒ t = 30: ∆m = 6000e (0.05/365)(30) − 6000 =
100 24.7082… , about 5 cents higher than dm.
x t = 60: ∆m = 6000e (0.05/365)(60) − 6000 =
A = tan− 1 .
100 49.5182… , about 20 cents higher than dm.
Because 1 radian is 180/π degrees, As t increases, dm is a less accurate
180 x approximation for ∆m.
θ= tan− 1 , Q .E . D .
π 100 8. a. dS = − 1.636 dt
180 1 1 March 11: dS = − 1.636(10) = − 16.36
b. dθ = · 2 · dx
π 1 + ( x / 100) 100 minutes
1.8 / π Sunrise time ≈ 6:26 − 0:16 = 6:10 a.m.,
= dx
1 + ( x / 100)2 which agrees with the tabulated value.
March 21: dS = − 1.636(20) = − 32.72 minutes
x = 0: dθ = 0.5729… dx
Sunrise time ≈ 6:26 − 0:33 = 5:53 a.m.,
x = 10: dθ = 0.5672… dx
which agrees with the tabulated value.
x = 20: dθ = 0.5509… dx
b. By September 1, t = 185, giving
c. At x = 0, θ = 0. For x = 20, dx = 20.
dS = − 1.636(185) = − 302.66, or 5:04 hours.
∴ θ ≈ (0 + 0.5729…)(20) = 11.459…°
The actual value is (180/π)(tan− 1 0.2) = So the predicted sunrise time would be
11.309… . 6:26 − 5:04 = 1:22 a.m. Because the sunrise
reaches its earliest in mid-June, the time
The error is 0.1492…°, which is about 1.3%.
predicted by dS is not reasonable.
d. 0.5729… is approximately 0.5, so multiplying
by it is approximately equivalent to dividing 9. y = 7x3 ⇒ dy = 21x2 dx
by 2. For a 20% grade, this estimate gives 10°, 10. y = − 4x11 ⇒ dy = − 44x10 dx
compared to the actual angle of 11.309…°, an
error of about 11.6%. For a 100% grade, this 11. y = (x4 + 1)7 ⇒ dy = 28x3(x4 + 1)6 dx
estimate gives 50°, compared to the actual 12. y = (5 − 8x)4 ⇒ dy = − 32(5 − 8x)3 dx
angle of 45°, an error of about 11.1%.
13. y = 3x2 + 5x − 9 ⇒ dy = (6x + 5) dx
6. dV = 4π r 2 dr
dr = 0.03 and r = 6, so dV = 4π (62)(0.03) = 14. y = x2 + x + 9 ⇒ dy = (2x + 1) dx
4.32π ≈ 13.57 mm3 15. y = e − 1.7x ⇒ dy = − 1.7e− 1.7x dx

Calculus Solutions Manual Problem Set 5-2 83


© 2005 Key Curriculum Press
15 1 −2 / 3 5 ∴ ∆y = − 0.679… − (− 0.866…)
16. y = 15 ln x 1/3 ⇒ dy = ⋅ x = dx
x 1/ 3 3 x = 0.186439…
17. y = sin 3x ⇒ dy = 3 cos 3x dx d. 0.15 is (fairly) close to 0.186439... .
18. y = cos 4x ⇒ dy = − 4 sin 4x dx
19. y = tan3 x ⇒ dy = 3 tan2 x sec2 x dx Problem Set 5-3
20. y = sec3 x ⇒ dy = 3 sec3 x tan x dx
Q1. Antiderivative = x3 + C
21. y = 4x cos x ⇒ dy = (4 cos x − 4x sin x) dx
Q2. Indefinite integral = (1/6)x6 + C
22. y = 3x sin x ⇒ dy = (3 sin x + 3x cos x) dx
Q3. y′ = 3x 2 Q4. y′ = ln 3(3x)
23. y = x 2 /2 − x/4 + 2 ⇒ dy = (x − 1/4) dx
Q5. dy = ln 3(3x) dx Q6. y ′′ = 5 x 4
24. y = x 3 /3 − x/5 + 6 ⇒ dy = (x 2 − 1/5) dx
Q7. Integral = sin x + C Q8. y′ = − sin x
− sin (ln x )
25. y = cos (ln x) ⇒ dy = dx Q9. 1 Q10. E
x

1
26. y = sin (e0.1 x) ⇒ dy = 0.1e0.1 x cos (e0.1 x) dx 1. x 10 dx = x 11 + C
11
27. dy = 20x3 dx ⇒ y = 5x4 + C

1 21
28. dy = 36x4 dx ⇒ y = 7.2x5 + C 2. x dx =
20
x +C
21
29. dy = sin 4x dx ⇒ y = − (1/4) cos 4x + C

4
3. 4 x −6 dx = − x −5 + C
30. dy = cos 0.2x dx ⇒ y = 5 sin 0.2x + C 5


dy = (0.5x − 1)6 dx ⇒ y = (2/7)(0.5x − 1)7 + C −7 3 −6
31. 4. 9 x dx = − x + C
32. dy = (4x + 3)− 6 dx ⇒ y = (− 1/20)(4x + 3)− 5 + C 2
33. dy = sec2 x dx ⇒ y = tan x + C 5. ∫ cos x dx = sin x + C
dy = csc x cot x dx ⇒ y = − csc x + C
∫ sin x dx = − cos x + C
34. 6.
35. dy = 5 dx ⇒ y = 5x + C
∫ 4 cos 7x dx = 7 sin 7x + C
4
36. dy = − 7 dx ⇒ y = − 7x + C 7.
37. dy = (6x2 + 10x − 4) dx ⇒ y = 2x3 + 5x 2 − 4x + C
∫ 20 sin 9 x dx = − 9 cos 9 x + C
20
38. dy = (10x2 − 3x + 7) dx ⇒ 8.
y = (10/3)x3 − (3/2)x 2 + 7x + C
∫ 5e dx = 0.3 e + C
0.3 x 5 0.3 x
9.
39. dy = sin5 x cos x dx ⇒ y = (1/6) sin6 x + C
dy = sec7 x tan x dx = sec6 x(sec x tan x dx) ⇒
∫ 2e dx = −200e + C
40. −0.01 x −0.01 x
10.
y = (1/7) sec7 x + C
4m
41. a. y = (3x + 4)2(2x − 5)3 ⇒
y′ = 2(3x + 4)(3)(2x − 5)3
11. ∫ 4 m dm =
ln 4
+C

8.4 r
+ (3x + 4)2 ⋅ 3(2x − 5)2 ⋅ 2
= 6(3x + 4)(2x − 5)2[2x − 5 + 3x + 4]
12. ∫ 8.4 r dr =
ln 8.4
+C

∫ ∫
1
∴ dy = 6(3x + 4)(2x − 5)2(5x − 1) dx 13. ( 4v + 9)2 dv = ( 4v + 9)2 ( 4 dv)
4
b. dy = 6(7)(− 3)2(4)(− 0.04) = − 60.48 1
= ( 4 v + 9) 3 + C
c. x = 1 ⇒ y = (7)2(− 3)3 = − 1323 12
x = 0.96 ⇒ y = − 1383.0218…
∫ ∫
1
14. (3 p + 17)5 dp = (3 p + 17)5 (3 dp)
∴ ∆y = − 1383.0218… − (− 1323) 3
1
= − 60.0218… = (3 p + 17) 6 + C
18
d. − 60.48 is close to − 60.0218… .
∫ ∫
1
42. a. y = sin 5x ⇒ dy = 5 cos 5x dx 15. (8 − 5 x )3 dx = − (8 − 5 x )3 ( −5 dx )
5
b. dy = 5 cos (5π /3) ⋅ 0.06 = 0.15 1
= − (8 – 5 x ) 4 + C
c. x = π /3 ⇒ y = sin (5π /3) = − 3 /2 20
= − 0.86602… ∫ (20 − x ) dx = (−1)∫ (20 − x ) (−dx )
4 4
16.
x = π /3 + 0.06 ⇒ y = − 0.679585565… 1
= − (20 – x ) 5 + C
5

84 Problem Set 5-3 Calculus Solutions Manual


© 2005 Key Curriculum Press
c. g (4) − g (1) = 6.4 + 4 + C − 0.1 − 1 − C =
∫ (sin x ) cos x dx = 7 sin x + C
6 1 7
17.
9.3, which is about equal to the definite
integral! It is also interesting that the constant
∫ (cos x ) sin x dx = − 9 cos x + C
8 1 9
18. C drops out.
35. Prove that if f and g are functions that can be
∫ cos θ sin θ dθ = − 5 cos θ + C
4 1 5
19. integrated, then

20. ∫ sin θ cosθ dθ = 6 sin θ + C


5 1 6 ∫ [ f ( x ) + g( x )] dx = ∫ f ( x ) dx + ∫ g( x ) dx.
Proof:
∫ ( x + 3x − 5) dx = 3 x + 2 x − 5x + C
2 1 3 3 2
21.
Let h( x ) = ∫ f ( x ) dx + ∫ g( x ) dx.
∫ ( x − 4 x + 1) dx = 3 x − 2 x + x + C
2 1 3 2
22. By the derivative of a sum property,

∫ ∫
d d
h ′( x ) = f ( x ) dx + g( x ) dx.
∫ ( x + 5) dx = ∫ ( x + 15x + 75x + 125) dx
2 3 6 4 2
23. dx dx
1 7 By the definition of indefinite integral applied
= x + 3x 5 + 25x 3 + 125x + C twice to the right side of the equation,
7
h ′ (x) = f (x) + g (x).
∫ (x ∫
− 6)2 dx = ( x 6 − 12 x 3 + 36) dx
3
24. By the definition of indefinite integral applied in
1 the other direction,
= x 7 − 3x 4 + 36x + C
7

h( x ) = [ f ( x ) + g( x )] dx
∫ sec x tan x dx = e +C
sec x sec x
25. e By the transitive property, then,

26. ∫e
tan x
sec 2 x dx = e tan x + C ∫ [ f ( x ) + g( x )] dx = ∫ f ( x ) dx + ∫ g( x ) dx,
Q .E .D .
∫ sec x dx = tan x + C
2
27.
36. Calvin says ∫ x cos x dx = x sin x + cos x + C.
∫ csc x dx = − cot x + C
2
28. Phoebe checked this by differentiating:
d
∫ tan
1 8 ( x sin x + cos x + C )
29. 7
x sec 2 x dx = tan x + C
8 dx
= 1 ⋅ sin x + x ⋅ (cos x) − sin x + 0 = x cos x
∫ cot
1
30. 8
x csc 2 x dx = − cot 9 x + C
9 By the definition of indefinite integral, she knew

∫ csc ∫
that Calvin was right.
31. 9
x cot x dx = csc8 x (csc x cot x dx )
37. a.
1
= − csc9 x + C C v (t)
9

∫ sec ∫
1.5 12.25
32. 7
x tan x dx = sec 6 x (sec x tan x dx )
2.5 16.25
1
=sec7 x + C 3.5 22.25
7
Sum: 50.75
33. v (t) = 40 + 5 t = 40 + 5t1/ 2
Integral ≈ 50.75

10 3/ 2
D(t ) = ( 40 + 5t 1/ 2 ) dt = 40t + t +C b.
3
c v (t)
10 3/ 2
D (0) = 0 ⇒ 0 = 40 ⋅ 0 + 0 +C⇒C=0
3 1.25 11.5625
10 3/ 2 1.75 13.0625
∴ D (t) = 40t + t
3 2.25 15.0625
D (10) = 505.4092... ≈ 505 ft 2.75 17.5625
34. a. f (x) = 0.3x 2 + 1 3.25 20.5625
T100 = 9.300135
3.75 24.0625

b. g( x ) = (0.3 x 2 + 1) dx = 0.1x 3 + x + C Sum: 101.8750
Integral ≈ (101.8750)(0.5) = 50.9375

Calculus Solutions Manual Problem Set 5-3 85


© 2005 Key Curriculum Press
3 2

∫ ∫
c. As shown in Figures 5-3c and 5-3d, the 3. 3 x dx 4. 2 x dx
Riemann sum with six increments has −1 −1
smaller regions included above the graph and
smaller regions excluded below the graph, so c f (c) c f (c)
the Riemann sum should be closer to the − 0.75 0.43869… − 0.75 0.59460…
integral.
− 0.25 0.75983… −0.25 0.84089…
d. Conjecture: Exact value is 51. 0.25 1.31607… 0.25 1.18920…
By the trapezoidal rule with n = 100,
0.75 2.27950… 0.75 1.68179…
integral ≈ 51.00045, which agrees with the
conjecture. 1.25 3.94822… 1.25 2.37841…
e. The integral is the product of v (t) and t, and 1.75 6.83852… 1.75 3.36358…
thus has the units (ft/min)(min), or ft. So the 2.25 11.84466… Sum = 10.04849…
object went 51 ft. Average velocity = 51/3 = 2.75 20.51556… R6 = (0.5)(10.04…)
17 ft/min. Sum = 47.94108… = 5.024249…
38. Answers will vary. R8 = (0.5)(47.94…)
= 23.97054…
Problem Set 5-4 2 1

Q1. y′ = sin x + x cos x


5. ∫ sin x dx
1
6. ∫ cos x dx
0

Q2. tan x + C c f (c) c f (c)


Q3. f ′(x) = sec2 x
1.1 0.891207… 0.1 0.995004…
Q4. (1/4)x4 + C 1.3 0.963558… 0.3 0.955336…
Q5. z′ = − 7 sin 7x 1.5 0.997494… 0.5 0.877582…
Q6. − cos u + C 1.7 0.991664… 0.7 0.764842…
Q7. Limit = 8 1.9 0.946300… 0.9 0.621609…
Q8. Sum = 4.790225… Sum = 4.214375…
y R5 = (0.2)(4.79…) R5 = (0.2)(4.21…)
7
= 0.958045… = 0.842875…
1.2


x
4
7. tan x dx
0.4
L4 = 0.73879… , U 4 = 1.16866…
Q9. If a + b = 5, then a = 2 and b = 3.
M 4 = 0.92270… , T4 = 0.95373…
Q10. No ∴ M4 and T4 are between L4 and U4, Q.E.D.
3

∫ 10/x dx:
4 6
1. ∫1
x 2 dx 2. ∫
2
x 3 dx 8.
1
L4 = 9.5, U 4 = 12.8333…
c f (c) c f (c)
M 4 = 10.89754… , T4 = 11.1666…
1.25 1.5625 2.25 11.390625 ∴ M4 and T4 are between L4 and U4, Q.E.D.
1.75 3.0625 2.75 20.796875 5

2.25 5.0625 3.25 34.328125 9. ∫ ln x dx is underestimated by the trapezoidal


1
2.75 7.5625 3.75 52.734375 rule and overestimated by the midpoint rule.
3.25 10.5625 4.25 76.765625
y y
3.75 14.0625 4.75 107.171875 2 2
Sum = 41.8750 5.25 144.703125
x
R6 = (0.5)(41.875)
x
5.75 190.109375 5 5
= 20.9375 Sum = 638.000000
R8 = (0.5)(638) = 319

86 Problem Set 5-4 Calculus Solutions Manual


© 2005 Key Curriculum Press
2
c. Un = (3/n)(1 ⋅ 3/n)2 + (3/n)(2 ⋅ 3/n)2
10. ∫0
e x dx is overestimated by the trapezoidal rule
+ (3/n)(3 ⋅ 3/n)2 + ⋅ ⋅ ⋅ + (3/n)(n ⋅ 3/n)2
and underestimated by the midpoint rule.
d. U n = (3/n)3(12 + 22 + 32 + ⋅ ⋅ ⋅ + n 2)
= (3/n)3(n/6)(n + 1)(2n + 1)
y y

= (4.5/n2)(n + 1)(2n + 1)
4 4
U100 = (4.5/1002)(101)(201) = 9.13545, which
x x
is correct.
1 1 e. Using the formula, U1000 = 9.013504… ,
which does seem to be approaching 9
11. a. h (x) = 3 + 2 sin x
n + 1 2n + 1
For an upper sum, take sample points at x f. Un = 4.5 ⋅ ⋅
equals 1, π/2, 2, 3, 4, and 6. n n
= 4.5(1 + 1/n)(2 + 1/n)
b. For a lower sum, take sample points at x
As n approaches infinity, 1/n approaches zero.
equals 0, 1, 3, 4, 3π/2, and 5.
∴ Un approaches 4.5(1 + 0)(2 + 0),
c. U6 = 1[h (1) + h (π/2) + h (2) + h (3) + h (4) which equals 9, exactly!
+ h (6)] = 21.71134… 2
L 6 = 1[h (0) + h (1) + h (3) + h (4) + h (3π/2)
+ h (5)] = 14.53372…
15. ∫0
x 3 dx
Find an upper sum using the sample points
12. Programs will vary depending on the type of
1 ⋅ 2/n, 2 ⋅ 2/n, 3 ⋅ 2/n, . . . , n ⋅ 2/n.
grapher used. See the program in the Programs
Un = (2/n)(1 ⋅ 2/n)3 + (2/n)(2 ⋅ 2/n)3
for Graphing Calculators section of the
Instructor’s Resource Book. + (2/n)(3 ⋅ 2/n)3 + ⋅ ⋅ ⋅ + (2/n)(n ⋅ 2/n)3
4 = (2/n)4(13 + 23 + 33 + ⋅ ⋅ ⋅ + n 3)
13. a. For ∫ 1
x 2 dx, the program should give the
= (2/n)4[(n/2)(n + 1)]2
values listed in the text. = 4/n2 ⋅ (n + 1)2 = 4(1 + 1/n)2
b. L100 = 20.77545, L500 = 20.955018. lim Un = 4(1 + 0)2 = 4
n→∞
Ln seems to be approaching 21.
c. U100 = 21.22545, U500 = 21.045018. Problem Set 5-5
Un also seems to be approaching 21.
10 t
f is integrable on [1, 4] if Ln and Un have the Q1. x 2/2 + 2x + C Q2. +C
same limit as n approaches infinity. ln 10
d. The trapezoids are circumscribed around the Q3. − cot x + C Q4. − csc x cot x
region under the graph and thus contain more 5(ln x ) 4
Q5. +C Q6.
area (see left diagram). For rectangles, the x
“triangular” part of the region that is left out y
has more area than the “triangular” part that is
added, because the “triangles” have equal bases
but unequal altitudes (see right diagram). x
1 2
y y
Trapezoid Rectangle
includes leaves out
more area. more area. Q7. Answers may vary. Q8. Answers may vary.
y y
5
x x
1 x
x

3 3 2

∫x
2
14. a. dx
0
Q9. No limit (infinite) Q10. D
U 100 = 9.13545, L100 = 8.86545
1. See the text for the statement of the mean value
Conjecture: Integral equals 9 exactly.
theorem.
b. The sample points will be at the right of each
2. See the text for the statement of Rolle’s
interval, 1 ⋅ 3/n, 2 ⋅ 3/n, 3 ⋅ 3/n, . . . ,
theorem.
n ⋅ 3/n.

Calculus Solutions Manual Problem Set 5-5 87


© 2005 Key Curriculum Press
3. g (x) = 6/x; [1, 4] h ′ (x) = − (1/2)x− 1 / 2
g (x ) ∴ (− 1/2)c− 1/2 = − 1/4 ⇒ c = 4
6 Tangent at x = 4 parallels the secant line.
7. f (x) = x cos x on [0, π /2]
2 f (x )
x 1
1 c 4
x
6/4 – 6 π
_
m= = −3/2
0 c1
2
4 –1
g ′ (x) = − 6x− 2
∴ − 6c− 2 = − 3/2 ⇒ c = 2
Tangent at x = 2 parallels the secant line. f ′ (x) = cos x − x sin x
∴ f is differentiable for all x.
4. f (x) = x4; [− 1, 2]
x cos x = 0 ⇔ x = 0 or cos x = 0
f (x ) cos x = 0 at ± π /2 + 2π n, where n is an integer
∴ hypotheses are met on [0, π/2].
10 Using the solver feature, f ′ (c) = 0 at
c = 0.86033… .
x Horizontal line at x = 0.86033… is tangent.
8. f (x) = x 2 sin x
–1 c 2

16 – 1 f (x )
m= =5
2 – (–1)
1
g ′ (x) = 4x3 x
0 c π
∴ 4c = 5 ⇒ c = 3 5 / 4 = 1.077K
3

Tangent at x = 1.077… parallels the secant line.


π
5. c( x ) = 2 + cos x; 0, 
 2 f ′ (x) = 2x sin x + x 2 cos x
c (x ) ∴ f is differentiable for all x.
x 2 sin x = 0 ⇔ x = 0 or sin x = 0
sin x = 0 at x = 0 + π n , where n is an integer
Interval: [0, π]
1 Using the solver feature, f ′ (c) = 0 at
x
c = 2.28892… .
π /2
Horizontal line at x = 2.288… is tangent.
c

cos (π /2) – cos 0 9. f (x) = (6x − x 2) 1/2


m= = −2/π = 0.6366 K
π /2 – 0 f (x )
c′(x) = − sin x 3
∴ − sin c = − 2/π ⇒ c = 0.69010…
Tangent at x = 0.690… parallels the secant line.
6. h( x ) = 5 − x ; [1, 9] x
0 c 6
h (x)
5
f ′( x ) = 12 (6 x – x 2 ) −1/ 2 ⋅ (6 − 2 x )
∴ f is differentiable on (0, 6).
f is continuous at x = 0 and x = 6.
x (6x − x 2)1/2 = 0
1 c 9
x(6 − x) = 0 ⇒ x = 0 or 6
Interval: [0, 6]
m=
2–4
= −1/4 (6c − c2)− 1/2(3 − c) = 0 ⇒ c = 3
9 –1 Horizontal line at x = 3 is tangent.

88 Problem Set 5-5 Calculus Solutions Manual


© 2005 Key Curriculum Press
10. f (x) = x 4/3 − 4x 1/3 intervals for which 1 is an endpoint, such
as [1, 2].
f (x )
The conclusion is true if the instantaneous
velocity, d′ (t), ever equals the average velocity.
1 c x The average velocity equals
0 4
d (2) – d ( 0)
m= = 28.5 ft/s for [0, 2],
2
d (2) – d (0.5)
m= = 57.111K ft/s for [0.5, 2],
1.5
f ′( x ) = 43 x 1/ 3 − 43 x −2 / 3 d (2) – d (1)
∴ f is differentiable for all x ≠ 0. m= = 100 ft/s for [1, 2].
1
f is continuous at x = 0. Between t = 0 and t = 1, d′ (t) is negative.
f (x) = 0 ⇒ x 4/3 − 4x 1/3 = 0 ⇒ Above t = 1, d′ (t) = 200t− 2.
x 1/3 (x − 4) = 0 ⇒ x = 0 or 4 For d′(c) = 28.5 ft/s,
Interval: [0, 4] 200c− 2 = 28.5 ⇒ c = 2.649… .
4
3 c1/ 3 − 43 c −2 / 3 = 0 But 2.649… is outside (0, 2), so the conclusion
4
c −2 / 3 (c − 1) = 0 ⇒ c = 1 is not true. See the left graph.
For d′ (c) = 57.111… ft/s,
3
Horizontal line at x = 1 is tangent.
200c− 2 = 57.111… ⇒ c = 1.871… .
11. a. d(t) = 1000(1.09t ) Because 1.871… is in (0.5, 2), the conclusion is
d(50) = 1000(1.0950) = 74,357.520… true. See the right graph.
= $74,357.52 (Surprising!) d (t ) d (t )

74357.5K – 1000
b. Average rate is = 200 200
50 c is outside
(0, 2).
c is in
(0.5, 2).
1467.150… ≈ $1,467.15 per year.
100 100
c. d′ (t) = ln (1.09)1000(1.09)t
d′ (0) ≈ $86.18 per year t t
d′ (50) ≈ $6,407.96 per year 0.5 1 2 c? 0.5 1 c? 2

The average of these is $3,247.07 per year,


which does not equal the average in part b. For d′ (c) = 100, 200c− 2 = 100 ⇒ c = 1.414… .
d. Solving 1000(1.90)t ln 1.09 = Because 1.414… is in (1, 2), the conclusion is
true, as is guaranteed by the mean value theorem.
1000(1.09)50 − 1000
algebraically gives The fact that the conclusion is true when the
50 hypotheses are met illustrates the fact that the
1.09 − 1
50
(1.09)t = hypotheses are sufficient. The fact that the
50 ln 1.09 conclusion can be true even if the hypotheses are
1.09 50 − 1 not met proves that the hypotheses are not
⇒ t ln 1.09 = ln necessary.
50 ln 1.09
= ln (1.0950 − 1) − ln 50 − ln (ln 1.09) ⇒ 13. See Figure 5-5d. 14. Answers may vary.
ln (1.09 50 − 1) − ln 50 – ln (ln 1.09)
f (x )
t=
ln 1.09
= 32.893K years. x
a b
This time is not halfway between 0 and 50.
 1– t
43  1 + t  , if t ≤ 1
12. d (t ) =  15. Answers may vary. 16. Answers may vary.
200 1 –  , if t ≥ 1
1
  t f (x ) f (x )

The hypotheses of the mean value theorem do not


apply on any interval that contains t = 1 as an a b x

interior point, such as [0, 2] and [0.5, 2], because d


x
d is not differentiable there. The hypotheses do a b
apply on any interval not containing 1 and on

Calculus Solutions Manual Problem Set 5-5 89


© 2005 Key Curriculum Press
17. Answers may vary. f ′ (x) never equals 0. f ′ (x) never equals 0.
f (x ) f (x ) f (x )
5

1
x
1 2

x
x
5
a c b

18. Michel Rolle (1652− 1719) lived in France. 27. f (x) = 1 − (x − 3)2/3 28. f ( x ) =
Sources will vary. x 3 – 6 x 2 + 11x – 6
19. f (x) = x 2 − 4x 20. f (x) = x 2 − 6x + 5 x–2
f (1) = − 3 ≠ 0 f (2) = − 3 ≠ 0 f is not differentiable f is not continuous or
Conclusion is not true. Conclusion is not true. at x = 3. differentiable at 2.
f ′ (2) = 0, but 2 is not f ′ (3) = 0, but 3 is not Conclusion is Conclusion is
in the interval (0, 1). in the interval (1, 2). not true. not true.
f (x ) f (x ) f ′ (x) never equals 0. There is no point at x = 2
x to draw the tangent line.
x
0 1 f (x )
1 2 f (x )

1 1
–3
x
–3
1 3
x
No point of
21. f (x) = x − 4x f (x) = x − 6x + 5
2 2 2 4 tangency
22.
f (2) = − 4 ≠ 0 f (4) = − 3 ≠ 0
x 3 – 7 x 2 + 13 x – 6
Conclusion is not true. Conclusion is true. 29. g( x ) =
f ′ (2) = 0, but 2 is not in f ′ (3) = 0, and 3 is in x–2
the open interval (0, 2). the interval (1, 4). ( x – 2)( x 2 – 5 x + 3)
= = x 2 − 5 x + 3, x ≠ 2
f (x ) f (x ) x–2
1 1
x c x Thus, g is discontinuous at x = 2, and the
0 2 1 4
hypotheses of the mean value theorem are not
met. The conclusion is not true for [1, 3],
because the tangent line would have to contain
(2, g (2)), as shown in the left graph. The
conclusion is true for (1, 5), because the slope of
23. f (x) = x 2 − 4x 24. f (x) = |x − 2| − 1 the secant line is 1, and g ′ (x) = 1 at x = 3, which
f (3) = − 3 ≠ 0 f is not differentiable is in the interval (1, 5). See the right graph.
at x = 2. g (x ) g (x )
Conclusion is true. Conclusion is not
3 3
true.
f ′ (2) = 0, and 2 is in f ′ (x) never equals 0. 1 2 3 x 1 2 x
3 5
the interval (0, 3).
f (x ) f (x )

c x
0 3 1
x 30. h (x) = x 2/3 ⇒ h ′ (x) = (2/3)x− 1 / 3
1 3 ∴ h is differentiable for all x ≠ 0.
–3 h ′ (0) would be 0− 1/3 = 1/(01/3) = 1/0, which is
infinite. The hypotheses of the mean value
theorem are met on the interval [0, 8], because
25. f (x) = 1/x 26. f (x) = x − [x] the function need not be differentiable at an
f (0) does not exist. f is discontinuous at endpoint. The hypotheses are not met on [− 1, 8],
1 and 2. because the point x = 0 where h is not
Conclusion is Conclusion is differentiable is in the open interval (− 1, 8). To
not true. not true. see if the conclusion of the mean value theorem

90 Problem Set 5-5 Calculus Solutions Manual


© 2005 Key Curriculum Press
is true anywhere, find the slope of the secant line 33. a. f (x) = 25 − (x − 5)2 + 4 cos 2π (x − 5)
(see next graph). The graph agrees with Figure 5-5l.
4 –1 b. f ′ (x) = − 2(x − 5) − 4 sin 2π (x − 5) ⋅ 2π =
m= = 1/3
8 – (–1) − 2x + 10 − 8π sin 2π (x − 5) f ′ (5) = 0
The tangent line has slope h ′ (c) = 1/3. Therefore, Because the derivative at x = 5 is 0, the
(2/3)c− 1/3 = 1/3 ⇒ c− 1/3 = 1/2 ⇒ c1/3 = 2 ⇒ c = 8. tangent line at x = 5 is horizontal. This is
So the conclusion of the mean value theorem is consistent with x = 5 being a high point on
not true because 8 is at the endpoint of the the graph.
interval, not in the open interval (− 1, 8). c.
h(x) m (x )
y1
4
20
y2
x
4
x
–1 8

3 x – 3, if x ≥ 3
31. a. f ( x ) =  x m (x) x m (x)
 x + 3, if x < 3
f (x ) 3.0 2 5.5 − 16.5
3.5 6.8333… 6.0 −1
6 4.0 1 6.5 − 6.833…
x 4.5 16.5 7.0 −2
3
5.0 no value

b. f is continuous at x = 3 because the right and The difference quotient is positive when x is
left limits both equal 6. f is not differentiable less than 5 and negative when x is greater
at x = 3 because the left limit of f ′ (x) is 1 than 5.
and the right limit is 3. d. In the proof of Rolle’s theorem, the left limit
c. f is not differentiable at x = 3, which is in (1, 6). of the difference quotient was shown to be
The secant line has slope 11/5. The tangent line positive or zero and the right limit was
has slope either 1 or 3, and thus is never 11/5. shown to be negative or zero.
d. f is integrable on [1, 6]. The integral equals The unmentioned hypothesis is
41.5, the sum of the areas of the two trapezoids differentiability on the interval (a, b). The
shown in this diagram. function f is differentiable. Because there is a
f (x ) value of f ′(5), both the left and right limits
15
of the difference quotient must be equal. This
number can only be zero, which establishes
6
the conclusion of the theorem. The
4 conclusion of Rolle’s theorem can be true
x even if the hypotheses aren’t met. For
1 3 6
instance, f (x) = 2 + cos x has zero derivatives
every π units of x, although f (x) is never
32. a. f (t) = number of miles in t hours
equal to zero.
t = number of hours driven
For the mean value theorem to apply on f ( 4.5) – f (2) 5.5 – 4
34. a. m = = = 0.6
[a, b], f must be differentiable on (a, b) and 2.5 2.5
continuous at t = a and t = b. g (x) − 4 = 0.6(x − 2) ⇒
b. The 60 mi/h equals the slope of the secant g (x) = 0.6x + 2.8
line. Therefore, there must be a tangent line Your graph should agree with Figure 5-5m.
at some value t = c in (a, b) with slope equal
b. f ′ (x) = 1 − π sin π x
to 60. This tangent line’s slope is the
instantaneous speed at t = c. Therefore, the Using the solver feature, f ′(c) = 0.6 at
speed was exactly 60 at some time between c = 2.0406… , 2.9593… , and 4.0406… ,
t = a and t = b, Q .E.D . all of which are in (2, 4.5).

Calculus Solutions Manual Problem Set 5-5 91


© 2005 Key Curriculum Press
c. h (x) = f (x) − g (x) ∴ f ′(c) − g ′(c) = 0
∴ f ′(c) = g ′(c)
∴ f ′(c) = the slope of the secant line, Q.E.D.
y
y
5 2
y1
35. The hypotheses of the mean value theorem
state that f should be differentiable on the open
x
interval (a, b) and continuous at x = a and x = b.
2 y3
If f is differentiable on the closed interval [a, b],
it is automatically continuous at x = a and x = b,
because differentiability implies continuity.
For c1 = 2.0406… :
36. a. h (x) = f (x) − g (x)
x h(x) The mean value theorem applies to h because
both f and g are given to be differentiable, and
1.7906… –0.2925… a linear combination of differentiable
1.8406… –0.1865… functions is also differentiable.
1.8906… –0.1022… b. By the mean value theorem, there is a number
1.9406… –0.0411… c in (a, b) for which
1.9906… –0.0041… h( b ) − h( a )
2.0406… 0.0081… h ′( c ) = .
b−a
2.0906… –0.0039… If f (a) = g (a) + D1 and f (b) = g (b) + D2,
2.1406… –0.0397… then h (a) = D1 and h (b) = D2.
2.1906… –0.0977… D – D1
∴ h ′( c ) = 2
2.2406… –0.1760… b–a
2.2906… –0.2723… c. If D1 ≠ D2, then h ′ (c) ≠ 0.
But h ′ (x) = f ′ (x) − g ′ (x) by the derivative of
For c = 2.9593… : For c = 4.0406… : a sum, and thus h ′(x) = 0 for all x in the
domain.
x h(x) x h(x) ∴ h ′(c) = 0, which contradicts h ′(c) ≠ 0.
2.7093… –1.3274… 3.7906… 0.5075… So the supposition that D1 ≠ D2 is false,
meaning that D1 and D2 are equal, Q.E.D.
2.7593… –1.4237… 3.8406… 0.6134…
37. By the definition of antiderivative (indefinite
2.8093… –1.5021… 3.8906… 0.6977…
2.8593… –1.5601… 3.9406… 0.7588… ∫
integral), g( x ) = 0 dx if and only if g ′ (x) = 0.

2.9093… –1.5959… 3.9906… 0.7958… Any other function f for which f ′ (x) = 0 differs
from g (x) by a constant. Thus the antiderivative
2.9593… –1.6081… 4.0406… 0.8081…
of zero is a constant function, Q.E.D.
3.0093… –1.5958… 4.0906… 0.7960…
38. f (x) = (cos x + sin x)2, and g (x) = sin 2x
3.0593… –1.5589… 4.1406… 0.7602…
3.1093… –1.4979… 4.1906… 0.7022… y

3.1593… –1.4136… 4.2406… 0.6239… f


1
3.2093… –1.3077… 4.2906… 0.5276… x
1
g
h(c1) = h(2.0406…) = 0.0081…
So h(c1) is an upper bound for h(x).
h (c2) = h (2.9593…) = − 1.60811669…
So h (c2) is a lower bound for h (x). x f (x) g (x)
h (c3) = h (4.0406…) = 0.808116698… 0 1 0
So h (c3) is an upper bound for h (x).
1 1.9092… 0.9092…
d. f meets the hypotheses of the mean value 2 0.2431… − 0.7568…
theorem, because f is differentiable for all x.
3 0.7205… − 0.2794…
h (x) = f (x) − g (x)
∴ h ′(x) = f ′(x) − g ′(x) 4 1.9893… 0.9893…
∴ h ′(c) = f ′(c) − g ′(c)
For each of the values of c in part b, h ′(c) = 0. In each case, f (x) = g (x) + 1.

92 Problem Set 5-5 Calculus Solutions Manual


© 2005 Key Curriculum Press
Proof: k x f ( x)
(cos x + sin x) = cos x + 2 cos x sin x + sin x
2 2 2
1 4 1.25
= 2 cos x sin x + 1 = sin 2x + 1, Q .E .D .
2 5 0.89442719…
39. The hypotheses of Rolle’s theorem say that f
is differentiable on the open interval (a, b). 3 6 0.68041381…
Because differentiability implies continuity, 4 7 0.53994924…
f is also continuous on the interval (a, b). 5 8 0.44194173…
Combining this fact with the hypothesis of
continuity at a and at b allows you to conclude Sum = 3.80673199…
that the function is continuous on the closed U5 = (1)(3.80673199…) = 3.80673199…
interval [a, b].
40. The intermediate value theorem applies to
continuous functions, whereas the mean value k x f ( x)
theorem applies to differentiable functions. Both 1 5 0.89442719…
are existence theorems, concluding that there
is a value x = c in the open interval (a, b). For 2 6 0.68041381…
the intermediate value theorem, f (c) equals a 3 7 0.53994924…
pre-selected number v between f ( a) and f ( b). 4 8 0.44194173…
For the mean value theorem, f ′ (c) equals the
slope of the secant line connecting (a, f ( a)) 5 9 0.37037037…
and (b, f ( b)). Sum = 2.92710236…
41. Answers will vary. L5 = (1)(2.92710236…) = 2.92710236…

Average = (U5 + L5)/2 = 3.36691717… .


Problem Set 5-6 Average overestimates the integral,
Q1. r′(x) = m(x) 3.33333… .
Q2. See the text for the definition of derivative. This fact is consistent with the fact that the
graph is concave up.
Q3. Increasing at 6 units/unit
d. Use sample points at the midpoints.
Q4. dy = sec x tan x dx
M10 = 3.32911229…
Q5. y′ = 8x(x 2 + 3)3
M100 = 3.33329093…
Q6. d2z/dz2 = −25 sin u
M1000 = 3.33333290…
Q7. f ′(x) = 0 Sums are converging toward 10/3.
Q8. 4.5 1.5
Q9. See Figure 5-5b. 2. I = ∫0
sin x dx = − cos 1.5 − ( − cos 0)
Q10. E = 0.92926279…
9 Using sample points at the midpoints,
∫ 10 x
−1.5
1. a. I = dx
4 M10 = 0.93013455…
= (−10/0.5)(9− 0.5 ) − (−10/0.5)(4− 0.5 ) M100 = 0.92927151…
= −20/3 + 20/2 = 10/3 = 3.33333… M1000 = 0.92926288…
The +C and −C add up to zero.
Integral = 0.92926279…
b.
y The sums are converging toward the integral.
The rectangle and the region differ by the two
“triangular” regions. Because the sample point
1
is at the midpoint of the subinterval, the
“triangles” have equal bases. Because the graph
x
4 9
is concave down, the “triangle” below the
horizontal line has a larger altitude, and
thus a larger area, than the one above the line.
c. Pick sample points at left ends of subintervals
So the rectangle includes more area on the left
for U5 and at right ends for L5.

Calculus Solutions Manual Problem Set 5-6 93


© 2005 Key Curriculum Press
than it leaves out on the right, and thus A(u + ∆u) − A(u)
overestimates the integral. c. h(u) < < h(u + ∆u)
∆u
y But the limits of h(u) and h(u + ∆u) both equal
Rectangle
includes h(u) because h is continuous and h(u) is
more area.
independent of ∆u. Therefore, by the squeeze
theorem,
A(u + ∆u) – A(u)
x lim = h(u). But the limit on
∆u→0 ∆u
the left is defined to be dA/du.
3. See the statement of the fundamental theorem in
∴ dA/du = h(u), Q.E.D.
the text.
d. dA = h(u) du
∴ A(u) = ∫ h(u) du = ∫ u1/2 du = (2/3)u3/2 + C
4. See the work in the text preceding the proof of
the fundamental theorem for a derivation of a
∴ A(u) = (2/3)u3/2 − 16/3
Riemann sum that is independent of the number
of subintervals. e. A(9) = 12 23 , which agrees with M 10 = 12.667… .
5. See the text proof of the fundamental theorem. (Note also that A(9) < M10, which is expected
because the graph of h is concave down.)
6. If c is picked as the point in (a, b) where the
9. a. Answers may vary. b. Answers may vary.
mean value theorem is true for g( x ) = ∫ f ( x ) dx, f (x ) f (x )

then the exact integral equals


g(b) – g( a)
⋅ (b − a), which equals g(b) − g(a).
(b – a)
x x
7. v(t) = 100 − 20(t + 1)1/2
8
Distance = ∫ [100 – 20(t + 1)
1/ 2
] dt c. Answers may vary. d. Answers may vary.
0
8 f (x ) f (x )
40
= 100t −(t + 1)3/ 2
3 0
40 3/ 2 40 3/ 2
= 800 − ( 9) − 0 + (1)
3 3
x x
1
= 453 ft
3
8. a. h(x) = x 1/2 e. Answers may vary. f. Answers may vary.
f (x ) f (x )

k x f ( x)
1 4.25 2.0615528…
2 4.75 2.1794494… x x
3 5.25 2.2912878…
4 5.75 2.3979157… 10. Answers will vary.
5 6.25 2.5
6 6.75 2.5980762… Problem Set 5-7
7 7.25 2.6925824… Q1. 1
6 x6 + C Q2. 18 (3 x + 7) + C
1 6

8 7.75 2.7838821… Q3. − 13 x −3 + C Q4. 6 sin x + C


1 6

9 8.25 2.8722813… Q5. 1


sin 5 x + C Q6. x + C
5
10 8.75 2.9580398…
Q7. tan x + C Q8. y″ = −1/x 2
Sum = 25.3350679…
Q9. definite Q10. indefinite
M10 = (0.5)(25.3350679…) = 12.66753… 4 4


1 1 1
b. h(u)∆u and h(u + ∆u)∆u are terms in a lower 1. x dx = x 3
2
= (64) − (1) = 21
1 3 1 3 3
sum and an upper sum, respectively, because 5
5


1 4 1 1 609 1
h(x) is increasing. 2. x 3 dx = x = (625) − (16) = = 152
∴ h(u)∆u < A(u + ∆u) – A(u) < h(u + ∆u)∆u 2 4 2 4 4 4 4

94 Problem Set 5-7 Calculus Solutions Manual


© 2005 Key Curriculum Press
π /3 π /3


3 3


1 (sec x tan x + sin x ) dx = sec x − cos x
3. (1 + 3 x )2 dx =
(1 + 3 x )3 16.
0
−2 9 −2 0

1 1 1
= (1000) − ( −125) = 125 =2− − 1 + 1 = 1.5
9 9 2
ln 4 ln 4


4 1 2x 1 2 ln 4 1 0
e 2 x dx = = − e = 7.5
4


1 17. e e
4. (5 x – 2) dx = (5 x – 2)3
2
0 2 0 2 2
−1 15 −1
ln 3


ln 3
=
1
[5832 – (–343)] =
6175 1235
= = 411
2 18. e − x dx = − e − x = − e − ln 3 + e 0
0 0
15 15 3 3
1 2
8 8 = − +1 =
∫ 60 x dx = 36 x = 36(32 − 1) = 1116
2/3 5/ 3
5. 3 3
1 1 2 1 4 2
4 4
19. ∫ sin 3 x cos x dx =
4
sin x
∫ 24 x
1 1
6. 3/ 2
dx = 9.6 x 5/ 2 = 9.6(32 − 1) = 297.6
1 1
=
(sin 4 2 – sin 4 1) = 0.045566 K
1

8 8 4
7. ∫ 5 dx = 5x = 40 − 10 = 30 3 3


1
2 2 20. (1 + cos x ) 4 sin x dx = − (1 + cos x )5
50
−3 5 −3
50
8. ∫20
dx = x
20
= 50 − 20 = 30 1 1
= − (1 + cos 3)5 + [1 + cos(–3)]5 = 0
5 5
0 0


1 3 3 2
9. ( x 2 + 3 x + 7) dx =
x + x + 7x or: Integral equals zero because an odd function is
−2 3 2 −2 integrated between symmetric limits.

= 0 −  − + 6 − 14 =
8 32 2 0.2 0.2


= 10 1
 3  3 3 21. cos 3 x dx = sin 3 x
0.1 3 0.1
0 0


1 3 1
10. ( x 2 + 4 x + 10) dx =
x + 2 x 2 + 10 x = (sin 0.6 – sin 0.3) = 0.0897074 K
−3 3 −3 3
0.4
= 0 − (−9 + 18 − 30) = 21 0.4


1
22. sin 2 x dx = − cos 2 x
1 1 1 2
∫ ∫
0 0
11. 4 x + 5 dx = ( 4 x + 5)1/ 2 ( 4 dx )
−1 4 −1 1
= − (cos 0.8 – cos 0) = 0.1516466K
1 2 1
1 13 1 2
= ⋅ ( 4 x + 5)3/ 2 = (27 – 1) = =4 5 5

3
4 3 −1
3
6 3 3 23. ∫−5
( x 7 – 6 x 3 + 4 sin x + 2) dx = 2 ∫ 2 dx
0

∫ ∫
1 5
12. 2 x + 10 dx = (2 x + 10) (2 dx ) 1/ 2
= 2( 2 x ) = 20 − 0 = 20
−3 2 −3 0
1 1

∫ ∫ cos x dx
3
1 2 1
= ⋅ (2 x + 10)3/ 2 = (64 – 8) =
56
= 18
2 24. (cos x + 10 x 3 – tan x ) dx = 2
−1 0
2 3 −3 3 3 3 1
π
= 2 sin x = 2 sin 1 − 2 sin 0 = 1.682941K
π 0
13. ∫ 4 sin x dx = −4 cos x = −4( −1) + 4(1) = 8 1


−2
0 0 25. x dx has no value because y = x− 2 has a
−1
π /2 π /2
14. ∫−π / 2
6 cos x dx = 6 sin x
−π / 2
= 6(1) − 6( −1) = 12 vertical asymptote at x = 0, which is within the
interval.
2


or: Integral of an even function between 26. x dx has no value because the integrand is
symmetric limits. −2

π /2 π /2 not a real number for negative values of x.


2 ∫ 0
6 cos x dx = 12 sin x
0
= 12(1) − 0 = 12 27. Integral = −(area) 28. Integral = area
f (x ) f (x )
π /3

∫ (sec x + cos x ) dx
2 3 6 x 1
15.
π /6
x
π /3
= tan x + sin x = 3 + 3 /2 − 1/ 3 − 1/2 5 7
-5
π /6

= (7/6) 3 − 1/2 = 1.52072K

Calculus Solutions Manual Problem Set 5-7 95


© 2005 Key Curriculum Press
29. Integral ≠ area 30. Integral ≠ area 39. Statement:
“If f ( x) < g(x) for all x in [a, b],
f (x ) f (x ) b b

∫ ∫ g( x ) dx. ”
1
1
then f ( x ) dx <
a a
7 x
Converse:
0
b b

∫ f ( x ) dx < ∫ g( x ) dx,
8 x
12
“If
a a

then f ( x) < g(x) for all x in [a, b].”


a b The converse can be shown to be false by any
31. ∫b
f ( x ) dx = − ∫ a
f ( x ) dx = −7 counterexample in which the area of the region
under the g graph is greater than the area under
b b the f graph, but the g graph touches or crosses
32. ∫ 4 f ( x ) dx = 4∫
a a
f ( x ) dx = 4(7) = 28
the f graph somewhere in [a, b]. One
c b c
counterexample is
33. ∫ g( x ) dx = ∫ g( x ) dx + ∫ g( x ) dx
a a b
f ( x) = 1.5 and g(x) = 2 + cos x on [0, 2π].

= 12 + 13 = 25 y
3
c g
34. ∫a
f ( x ) dx cannot be determined. f

x
c c 2π

∫ ∫ g( x ) dx cannot be determined.
0
35. f ( x ) dx +
a a

b 4


4


1 3
36. [ f ( x ) + g( x )] dx = 40. x 2 dx = x +C
a 1 3 1
b b

∫a
f ( x ) dx + ∫ g( x ) dx = 7 + 12 = 19
a
1 1
= ( 4 3 ) + C − (13 ) − C = 21
3 3
37.
The two C’s will always cancel, so it is not
y
necessary to write them.
f (3) = 7

y = f(x )
Problem Set 5-8
Q1. 30x2.4 + C
y = f' (x)
Q2. 30(42.4 − 1) = 805.72…
x
Q3. y′ = −1/ 1 – x 2
5 10
Q4. f ′ (x) = 3x2 sin x + x3 cos x
Q5.
f (x ) and f' (x )
38.
f
y f (1) = 8
1 f' x
8 1
y = f (x )

Q6. Yes, continuous


Q7. Increasing at x = 7
x Q8. f ( a) = f ( b) = 0
7
y = f' ( x ) Q9. v(9) = 450 ft/s
Q10. a(9) = 25 (ft/s)/s

96 Problem Set 5-8 Calculus Solutions Manual


© 2005 Key Curriculum Press
1. a. 3. a.
v y
100 (x , y)
(t, v )

50 5 dx

dt x
2
t
1 t=a t=b
b. dA = y dx = 10e0. 2x dx
dy = v dt = (55 + 12t0.6 ) dt 2 2

b
c. ∫ 10e 0.2 x dx = 50e 0.2 x = 50e 0.4 − 50
∫ (55 + 12t
0
b. Displacement = 0.6
) dt 0
2

∫ 10e
a
1
d. 0.2 x
dx = 24.59123K
1

∫ (55 + 12t
0
0.6
) dt = 55t + 7.5t 1.6
0 0 The region is approximately a trapezoid with
= 55 + 7.5 − 0 − 0 = 62.5 mi height 2 and bases 10 and y(2). y(2) =
2 2 14.9182… , so the area of the trapezoid is
∫ 1
(55 + 12t 0.6 ) dt = 55t + 7.5t 1.6
1
2/2(10 + 14.9182…) = 24.9182… .
4.
= 110 + 22.735… − 55 − 7.5 ≈ 70.2 mi y ( x, y )
2 2

∫ (55 + 12t ) dt = 55t + 7.5t 1.6


0.6
c.
0 0
4 dx

= 110 + 22.735… − 0 − 0 = 132.735… ,


which equals the sum of the two integrals 3 x
above.
b

∫ (55 + 12t ) dt = 300 dA = (6x − x2 ) dx


0.6
d.
0 6 6


1 1
55t
b
+ 7.5t 1.6 0 = 300 (6 x − x 2 ) dx = 3 x 2 − x 3 = 3 ⋅ 6 2 − ⋅ 6 3 = 36
0 3 0 3
55b + 7.5b = 300 1.6
The area of the circumscribed rectangle is 6 ⋅ 9 =
b ≈ 4.13372… ≈ 4.134 h 54. The area of the parabolic region is two-thirds
y
this area.
300 5. a. dA = [x + 2 − (x2 − 2x − 2)] dx
The top and bottom of the strip are not
horizontal, so the area of the strip is slightly
x different from dA. As dx approaches zero, the
4.134 differences in height at different values of x in
the strip become smaller, so the difference
between dA and the area of the strip gets
e. v(4.13372…) = 55 + 12(4.13372…)0.6 =
smaller.
83.1181… . At the end of the trip, you were
going about 83 mi/h. b. y1 = y2 ⇒ x + 2 = x 2 − 2x − 2 ⇒
2. 0 = x 2 − 3x − 4 ⇒ 0 = (x − 4)(x + 1) ⇒
v x = 4 or x = −1
4 4


1 3
( − x 2 + 3 x + 4) dx = − x 3 + x 2 + 4 x
10 ( t, v ) −1 3 2 −1

dv
 1 3 3 2
= − ⋅4 + ⋅4 + 4⋅4 
t  3 2 

− − ⋅ ( −1)3 + ⋅ ( −1)2 + 4 ⋅ ( −1)


10 1 3
dy = v dt = 15e −0.1t dt  3 2 
125
20 20 = = 20.83333K
∫0
15e −0.1t dt = −150e −0.1t
0
= −150e −2 + 150 6
c. R100 = 20.834375 (Checks)
= 129.6997… ft

Calculus Solutions Manual Problem Set 5-8 97


© 2005 Key Curriculum Press
6. 9. a.
y T
( x, y )
1 1

dx 20 (x, T )

( x, y ) dx
2 x x
π⁄4 0.5 1

The curves intersect at x = π /4. b. dD = T dx = [20 − 12 cos 2π(x − 0.1)] dx


dA = (cos x − sin x) dx 0.5

π /4 π /4 c. D = ∫ [20 – 12 cos 2π ( x – 0.1)] dx



0
(cos x − sin x ) dx = (sin x + cos x ) 0.5
0 0 = 20 x − (6/π ) sin 2π ( x − 0.1) 0

= 2 −1 = 10 − (6 /π) sin 0.8π − 0 + (6 /π) sin (−0.2π)


= 7.75482… ≈ 7.75 degree-days
7. a. d. From noon to midnight,
F
6 1
(x , F ) D= ∫ 0.5
[20 – 12 cos 2π ( x – 0.1)] dx
1
dx
= 20 x − (6/π ) sin 2π ( x − 0.1)
0.5
x
= 20 − (6 /π) sin 1.8π − 10 + (6 /π) sin 0.8π
= 12.24517… ≈ 12.25 degree-days
0 9

The total number of degree-days is


b. dW = F dx = 0.6x dx D = 7.75482… + 12.24517… =
9
9 20 degree-days.
W= ∫
0
0.6 x dx = 0.3 x 2
0
Note that this answer can be found more
easily by observing that in one full cycle of a
= 24.3 − 0 = 24.3 inch-pounds
sinusoid, there is just as much area above the
c. The region under F from x = 0 to x = 9 is a sinusoidal axis, T = 20, as there is below it.
triangle with base 9 and height F(9) = 5.4. So So the average temperature difference for the
the area is 1/2 ⋅ 9 ⋅ 5.4 = 24.3. day is 20 degrees, making the number of
d. dW is found by multiplying F by dx. F is degree-days for one day equal to 20.
measured in pounds, and dx is measured in
inches, so the units of dW are 10. a.
(pounds)(inches), or inch-pounds. C

1000
8.
F

( x, F ) dT
T
30
10 30
dx

x
5 dH = C dT
= (−0.016T 3 + 0.678T 2 + 7.45T + 796) dT
π
dW = 50 cos 30


x dx
20 H= (–0.016T 3 + 0.678T 2
10
10 π π 10 + 7.45T + 796) dT

1000
50 cos x dx = sin x
0 20 π 20 0 = −0.004T 4 + 0.226T 3 + 3.725T 2
30
1000 π 1000 1000 + 796T 10 = −3240 + 6102 + 3352.5
=
sin − sin 0 =
π 2 π π + 23880 + 40 − 226 − 372.5 − 7960
= 318.3098… = 21,576 Btu
The midpoint Riemann sum R100 gives b. (2000)(21576) = 43,152,000 Btu
318.313… , which is close to the answer found The property is the integral of a constant
using integration.

98 Problem Set 5-8 Calculus Solutions Manual


© 2005 Key Curriculum Press
30
The graph intersects the x-axis at x = 1 and x = 5.
times a function. That is, ∫
10
2000C dT =
y = −x 2 + 6x − 5 = −(x − 1) ( x − 5) = 0 ⇒ x = 1,
30


5, which confirms the graphical solution.
2000 C dT .
10 dA = (y − 0) dx = (−x2 + 6x − 5) dx
11. a. P 5 5


1
A= (– x 2 + 6 x – 5) dx = − x 3 + 3 x 2 − 5 x
1 3 1
(x , P )
125 1 2
1000 =− + 75 − 25 + − 3 + 5 = 10
dx 3 3 3
x
0 100 200
14. y
(x, 0) x
b. dC = P dx = (100 + 0.06x ) dx 2
–2 3

b b
C= ∫0
(100 + 0.06 x 2 ) dx = 100 x + 0.02 x 3
0

= 100b + 0.02b 3 − 0 − 0 (x, y )


–6
∴ C = 100b + 0.02b3
c. b = 100: C = 100(100) + 0.02(1003) = The graph intersects the x-axis at x = −2
$30,000
and x = 3.
b = 200: C = 100(200) + 0.02(2003) = y = x 2 − x − 6 = (x + 2)(x − 3) = 0 ⇒ x = −2,
$180,000 3, which confirms the graphical solution.
For 100 m to 200 m, the cost should be dA = (0 − y) dx = (−x2 + x + 6) dx
180,000 − 30,000 = $150,000. 3 3


1 1
As a check, A= (– x 2 + x + 6) dx = − x 3 + x 2 + 6 x
−2 3 2 −2
200 200


100
(100 + 0.06 x 2 ) dx = 100 x + 0.02 x 3
100
= −9 +
9
2
8
+ 18 − − 2 + 12 = 20
3
5
6
= 100(200) + 0.02(2003) − 100(100)
− 0.02(1003) = 150000 15. 4
y

200 100 200

∫ ∫ ∫
(x , y ) (0, y )
Thus, P dx = P dx + P dx,
0 0 100
which shows that the sum of integrals with
the same integrand applies. x
12. Using trapezoids, the area is approximately –2

10(0/2 + 38 + 50 + 62 + 60 + 55 + 51 + 30 +
3/2) = 3475 ft2. The fundamental theorem cannot The graph intersects the y-axis at y = 1
be used because the function is specified only by and y = 4.
data, not by an equation whose antiderivative can x = (y − 1) ( y − 4) = 0 ⇒ y = 1, 4, which
be found. confirms the graphical solution.
Plan of attack for area problems: dA = (0 − x) dy = −(y − 1) ( y − 4) =
• Do geometry to get dA in terms of sample (−y2 + 5y − 4) dy
4
point (x, y). 4


1 5
A= (– y 2 + 5 y – 4) dy = − y 3 + y 2 − 4 y
• Do algebra to get dA in terms of one variable. 1 3 2 1

• Do calculus to sum the dA’s and take the limit 64 1 5 1


(i.e., integrate). =− + 40 − 16 + − + 4 = 4
3 3 2 2
13. y
y
4 (x , y ) 16. 5

(x, y )
(0, y )

x x

1 (x, 0) 5 5
–1

Calculus Solutions Manual Problem Set 5-8 99


© 2005 Key Curriculum Press
The graph intersects the y-axis at y = −1 The graphs intersect at x = −2 and x = 2.
and y = 5. 0.5x 2 + 2x = −x 2 + 2x + 6 ⇔ 1.5x 2 = 6 ⇔ x =
x = 5 + 4y − y 2 = (1 + y) ( 5 − y) = 0 ⇒ y = −2, 2, which confirms the graphical solution.
−1, 5, which confirms the graphical solution. dA = (y2 − y1) dx = (−1.5x2 + 6) dx
dA = (x − 0) dy = (5 + 4y − y2) dy 2 2

5 5
A= ∫
−2
(–1.5 x 2 + 6) dx = −0.5 x 3 + 6 x
−2

1
A= (5 + 4 y – y 2 ) dy = 5 y + 2 y 2 − y 3
−1 3 −1 = −4 + 12 − 4 + 12 = 16
125 1 20.
= 25 + 50 − + 5 − 2 − = 36
3 3 y

17.
y 5 (x, y1 )

(x, y2)

x
2
0 5
x (x, y2 )
–1 4

(x, y1 ) The graphs intersect at x = 0 and x = 5.


0.2x 2 + 3 = x 2 − 4x + 3 ⇔ 0.8x 2 − 4x = 0 ⇔
The graphs intersect at x = −1 and x = 4. 0.8x(x − 4) = 0 ⇔ x = 0, 5, which confirms the
x 2 − 2x − 2 = x + 2 ⇔ x 2 − 3x − 4 = 0 ⇔ x = graphical solution.
−1, 4, which confirms the graphical solution. dA = (y1 − y2) dx = (−0.8x2 + 4x) dx
dA = (y2 − y1) dx = (−x2 + 3x + 4) dx 5 5


4 3
4 4 A= (–0.8 x 2 + 4 x ) dx = − x + 2x2

1 3 15
A= ( − x 2 + 3 x + 4) dx = − x 3 + x 2 + 4 x 0 0
−1 3 2 −1
500 2
=− + 50 + 0 − 0 = 16
64 1 3 5 15 3
=− + 24 + 16 − − + 4 = 20
3 3 2 6 21.
18. y

y
(x, y1 )
10
2
(x, y1)
4 x x
–2 0 5
(x , y 2 )
(x, y2 )
The graphs intersect at x = 0 and x = 5.
dA = (y1 − y2) dx = (2e0. 2x − cos x) dx
The graphs intersect at x = −2 and x = 4.
5 5
−2x + 7 = x 2 − 4x − 1 ⇔ x 2 − 2x − 8 = 0 ⇔
(x + 2)(x − 4) = 0 ⇔ x = −2, 4, which confirms
A= ∫
0
(2e 0.2 x − cos x ) dx = 10e 0.2 x − sin x
0

the graphical solution. = 10e − sin 5 − 10 + 0 = 18.1417…


dA = (y1 − y2) dx = (−x2 + 2x + 8) dx 22.
4 4 y


1
A= (– x 2 + 2 x + 8) dx = − x 3 + x 2 + 8 x
−2 3 −2
(x, y2 )

64 8
=− + 16 + 32 − − 4 + 16 = 36
3 3 (x, y1 )
x
19. 1
y (x , y 2 )
6

dA = (y2 − y1) dx = (e2x − sec2 x) dx


( x, y1 ) 1
1

–2 2
x A= ∫ 0
(e 2 x – sec 2 x ) dx = 0.5e 2 x − tan x
0

= 0.5e − tan 1 − 0.5 + 0 = 1.6371…


2

100 Problem Set 5-8 Calculus Solutions Manual


© 2005 Key Curriculum Press
23. 2 2


1 4
A= ( x 3 – 3 x 2 + 4) dx = x − x3 + 4x
y −1 4 −1
4
1 3
(x 1 , y ) = 4−8+8− −1+ 4 = 6
( x 2 , y) 4 4
1 26.
x
–2 2 y

(x, y2 )
The graphs intersect at y = 1 and y = 4. 2
Write y = x + 3 as x = y − 3. (x, y1 )
x
y − 3 = −y 2 + 6y − 7 ⇒ y 2 − 5y + 4 = 0 ⇒ –1 8
(y − 1)(y − 4) = 0 ⇒ y = 1, 4, which confirms
the graphical solution.
dA = (x2 − x1) dy = (−y2 + 5y − 4) dy The graphs intersect at x = −1 and x = 8.
4 4 x 2/3 = (x + 1)1/2 + 1 ⇒ x = −1, 8 numerically,
∫ (– y
1 5
A= 2
+ 5 y – 4) dy = − y 3 + y 2 − 4 y which confirms the graphical solution.
1 3 2 1
Or x2/ 3 − 1 = (x + 1)1/ 2 ⇒ (x2/ 3 − 1)2 = x + 1.
=−
64 1 5
+ 40 − 16 + − + 4 = 4
1 Write t = x1/ 3, so (t2 − 1)2 = t3 + 1 ⇒
3 3 2 2 t4 − t3 − 2t2 = t2(t + 1)(t − 2) = 0 ⇒
24. t = 0, −1, 2 ⇒ x = t3 = 0, −1, 8.
y
But x = 0 is extraneous from the irreversible step
5
of squaring both sides. So x = −1, 8.
(x2 , y) (x1 , y) dA = (y 2 − y 1) dx = [(x + 1)1/2 + 1 − x 2/3] dx
x
8
8
A= ∫−1
[( x + 1)1/ 2 + 1 – x 2 / 3 ] dx
8
–5 2 3
= ( x + 1)3/ 2 + x − x 5/ 3
3 5 −1
The graphs intersect at y = −5 and y = 5.
Write y = −2x 1 + 11 as x 1 = 5.5 − 0.5y. 96 3 1
= 18 + 8 − − 0 +1− = 7
5 5 5
5.5 − 0.5y = 0.25y 2 − 0.5y − 0.75 ⇒
0.25y2 = 6.25 ⇒ y = −5, 5, which confirms the 27. Wanda: You can always tell the right way
graphical solution. because the altitude of the strip should be
dA = (x1 − x2) dy = (−0.25y2 + 6.25) dy positive. This will happen if you take
(larger value) minus (smaller value). In this case,
5 5


1 3 25 if you slice vertically, it’s line minus curve
A= (–0.25 y 2 + 6.25) dy = − y + y
−5 12 4 −5 (see graph).
125 125 125 125 2
=− + − + = 41
y

12 4 12 4 3 line
25.
y curve
(x , y1 ) x
4

x
–1 2
(x , y 2 )
For curve minus line, you’d get the opposite of
the right answer. Note that if you slice
horizontally, it would be curve minus line.
The graphs intersect at x = −1 and x = 2. 28. a. Peter: Horizontal slicing would be awkward
x 3 − 4x = 3x 2 − 4x − 4 ⇒ x 3 − 3x 2 + 4 = because for some values of y the length of the
(x + 1) ( x − 2)2 = 0 ⇒ x = −1, 2, which strip would be given by line minus curve, but
confirms the graphical solution. in others it would be boundary minus curve,
and yet elsewhere it would be curve minus
dA = (y1 − y2) dx = (x3 − 3x2 + 4) dx
curve. If you use vertical slices, the length

Calculus Solutions Manual Problem Set 5-8 101


© 2005 Key Curriculum Press
of the strip will always be line minus curve. For y = 7 cos 5x, width is 1/5 as much and
(See graphs.) altitude is 7 times as much.
y y
∴ A = (2)(1/5)(7) = 2.8
(x, y1)
31.
x x y
(x, y2 )
8

(x, y 2)

(x, y1 )

b. Peter: In the graph on the right, y1 − y2 will


x
–2 3
be positive. Because y2 is negative, you will
get (pos.) − (neg.), which is equivalent to
(pos.) + (pos.). Thus, the altitude for the strip The graphs intersect at x = −2 and x = 3.
is positive. dA = (y2 − y1) dx = (−x2 + x + 6) dx
3 3


29. 1 1
y A= (– x 2 + x + 6) dx = − x 3 + x 2 + 6 x
(x, ah2)
−2 3 2 −2

9 8 5
= −9 + + 18 − − 2 + 12 = 20 = 20.8333K
2 3 6
x R 10 = 20.9375
–h h
(x , y )
R100 = 20.834375
R1000 = 20.83334375
The graph shows the parabolic region from The Riemann sums seem to be approaching the
x = −h to x = h and a strip from the graph to a exact answer.
horizontal line at y = ah2.
dA = (ah2 − y) dx = (ah2 − ax2) dx 32.
t (x )
h
A = ( ah 2 – ax 2 ) dx = 2  ah 2 x − ax 3 
h


1
−h  3  0 10

= 2 a  h 3 − h 3  = ah 3
1 4
 3  3 x
Area of rectangle = 2h(ah2) = 2ah3 π 2π 3π 4π
area of region ( 4/3)ah 3 2
∴ = = , Q. E . D .
area of rectangle 2 ah 3 3 t′(x) = 1 + cos x
The graph shows y = 67 − 0.6x and the line
2
t′(x) = 0 ⇒ cos x = −1 ⇒
y = 7, with a circumscribed rectangle. x = π + 2π n = … , π , 3π , 5π , …
67
y t′(x) is never negative, so t′(x) does not change
signs. These points are plateau points.

y=7
x
Problem Set 5-9
1 3 1 2 4 7/4
7 = 67 − 0.6x 2 ⇒ 0.6x 2 = 60 ⇒ x = ±10 Q1. x + x + x+C Q2. x +C
3 2 7
Rectangle has width 10 − (−10) = 20 and length
67 − 7 = 60. Area of region = 23 (20)(60) = 800. 2 −1/ 3 1
Q3. y′ = x Q4. − e −3 x + C
30. dA = sin x dx 3 3
π
π Q5. −csc x + C Q6. x− 1
A= ∫
0
sin x dx = − cos x
0
= −( −1) + 1 = 2, which
Q7. See Section 5-4. Q8. mean value
is a rational number. Q9. Q10.
y y y
7 y
y = 7 cos 5x
y = cos x (x, y )
1
x x 1
x
x
π 4 1
–π/10 π/10

102 Problem Set 5-9 Calculus Solutions Manual


© 2005 Key Curriculum Press
4 4

∫ ∫ (16 x
Plan of attack for volume problems: V= π ( 4 x − x 2 )2 dx = π 2
– 8 x 3 + x 4 ) dx
• Do geometry to get dV in terms of sample 1 1
4
=π  x3 − 2x4 + x5
point (x, y). 16 1
= 30.6π = 96.132 …
• Do algebra to get dV in terms of one variable.  3 5  1

• Do calculus to add up the dV’s and take the limit The midpoint Riemann sum R100 gives
(i.e., integrate). 96.1341… , which is close to the answer found
using integration.
1. a. dV = π x 2 dy
y = 9 − x2 ⇒ x2 = 9 − y 5. y = x 1.5 is rotated about the x-axis.
dV = π (9 − y) dy y
27
9 9
b. V = ∫ π (9 − y) dy = π (9 y − 0.5y )
2
0 0

= π (81 − 40.5) − π (0 − 0) = 40.5π (x, y )

= 127.2345…
x
c. R100 = 127.2345… (Checks.) 9
1

d. Volume of circumscribed cylinder is 9(π ⋅ 32)


= 254.4690… . Half of this is 127.2345… ,
equal to the volume of the paraboloid.
2. dV = π x2 dy
y = 10 − 2x ⇒ x = 5 − 0.5y
∴ dV = π (5 − 0.5y)2 dy dV = π y 2 dx = π x3 dx
10 10 9 9

∫ ∫
2 1 4
V= π (5 − 0.5 y)2 dy = − π (5 − 0.5 y)3 V= π x 3 dx = πx = 1640π = 5152.2119…
0 3 0 1 4 1

2 2 250π 6. y = ln x ⇒ x = ey is rotated about the y-axis.


= − π (0) + π (125) = = 261.7993…
3 3 3 y
1 1
V cylinder = π r2h, so of that is V = πr 2 h.
3 3 1
1
Here, r = 5 and h = 10, so V = π (52 )(10) = (x, y )
3 x

250π 1
, as found by integrating.
3
3. a. dV = π y 2 dx = π (3e− 0.2 x)2 dx = 9π e− 0.4 x dx dV = π x 2 dy = π e2y dy
5 5

∫ 9πe
−0.4x
dx = −22.5πe −0.4x 1 π 2y 1
π 2

b.
0 0 V= πe 2 y dy = e = (e − 1) = 10.0359K
0 2 0 2
= −22.5π e− 2 + 22.5π e0 = 61.1195…
The midpoint Riemann sum R100 gives 7. y = x 3/4 ⇒ x = y 4/ 3 is rotated about the y-axis.
61.1185… , which is close to the answer y
found using integration.
c. Slice perpendicular to the axis of rotation, so 8
slice vertically if rotating about the x-axis and
(x, y )
horizontally if rotating about the y-axis. 1
x
4. y = 4x − x2 is rotated about the x-axis. 1 16

y
(x, y ) dV = π x 2 dy = π y 8/3 dy
8 8


3 6141
V = πy 8/3 dy = π ⋅ y11/3 = π
x 1 11 1 11
= 1753.8654…
1 4

8. y = x14 and y = 8x2, intersecting at (0, 0) and


(2, 16), are rotated about the y-axis. Area of cross
dV = π y 2 dx = π (4x − x2)2 dx section is π x12 − π x 22 .

Calculus Solutions Manual Problem Set 5-9 103


© 2005 Key Curriculum Press
1 y
x 1 = y 1/ 4, and x 2 = y 4
8 (x2 , y)

dy = π  y1/2 −
1 2
∴ dV = π ( x12 − x 22 ) y dy
 64 
3
(x 1 , y )

π  y1/2 −
16 1 2
V= ∫0 
y dy
64 
16
= π  y 3/2 −
2 1 3 64
y π = 67.0206 K =
3 192  0 3
The midpoint Riemann sum R100 gives V ≈ x
1
67.0341… , which is close to the answer found
using integration.
dV = π ( x 22 − x12 ) dy = π [( 4 − y) − ( 4 − y) 2 ] dy
9. y1 = e0.4 x and y2 = x + 1, from x = 0 to x = 3, are
= π (−y2 + 7y − 12) dy
rotated about the x-axis. 4
Area of cross section is πy22 − πy12 . V= ∫ π (− y + 7 y − 12) dy
2
3
dV = π ( y22 − y12 ) dx = π [( x + 1) 2 − e 0.8 x ] dx 4
= π  − y 3 + y 2 − 12 y
1 7
3
 3 
V= ∫ π [( x + 1) − e 0.8 x ] dx 2
2
3
0
= π− + 36
64 63
3 + 56 − 48 + 9 −
= π  ( x + 1)3 − 1.25e 0.8 x   3 
1 2
3  1
0
= π = 0.523598…
= π  − 1.25e 2.4 − + 1.25
64 1 6
 3 3  12. y = ax2 ⇒ x = (y/a)1/2, from (0, 0) to (r, h), is
= π (22.25 − 1.25e 2.4 ) = 26.6125… ft 3 rotated about the y-axis.
y
The midpoint Riemann sum R100 gives h
V = 26.6127… , which is close to the answer
found using integration.
10. y1 = x1/3 and y2 = 10e− 0.1 x are rotated about the (x, y )
x-axis. Only the back half of the solid is shown. x
r

y
dV = π x 2 dy = π (y/a) dy = (π /a)y dy
10 h h


1 1
(x, y2) V = (π /a) y dy = (π /a) ⋅ y 2 = (π /a)(h) 2 − 0
0 2 0 2
(x, y1)
Because y = ax2, h = ar2.
x
1 1
0 8 ∴ V = (π /a)( ar 2 )2 = π ar 4
2 2
Volume of circumscribed cylinder is
V c = π r2h = π r2(ar2) = π ar4.
Thus, the volume of the paraboloid is half the
volume of the circumscribed cylinder, Q.E.D.
dV = π ( y22 − y12 ) dx = π (100e −0.2 x − x 2/3 ) dx 13. a. y = 0.3x1.5 is rotated about the x-axis.
y
8

∫ π (100e
2
−0.2 x
V= − x ) dx
2/3 (x, y )
0 x
−0.2 x 8
= π ( −500e − 0.6 x ) 5/3 4
0
−1.6
= π ( −500e + 480.8) = 1193.3394 …
11. y = 4 − x 1 ⇒ x 1 = 4 − y, and y = 4 − x 22 ⇒ x2 = dV = π y 2 dx = π (0.3x1.5 )2 dx = π (0.09x3) dx
4 4
4 – y , intersecting at x = 0 and x = 1, are
rotated about the y-axis. Only the back half of
V= ∫
0
π (0.09 x 3 ) dx = 0.0225πx 4
0

the solid is shown. = 5.76π = 18.09557…

104 Problem Set 5-9 Calculus Solutions Manual


© 2005 Key Curriculum Press
b. R10 = 5.7312π
1
V= ∫ (x − x 2 )2 dx
1/ 5

R100 = 5.75971…π 0

R1000 = 5.7599971…π = ∫ (x
1
2/5
− 2 x 11/ 5 + x 4 ) dx
Values are getting closer to V = 5.76π. 0
1
14. y = 4 − x 2 ⇒ x = (4 − y)1/2 dy =  x 7/5 − x 16/5 + x 5 
5 5 1 81
= = 0.2892 K
Inner radius is 3 − x; outer radius is 3. 7 8 5  0 280
dV = π[32 − (3 − x)2] dy 17. Cross sections perpendicular to y-axis are squares
= π{9 − [3 − (4 − y)1/2]2} dy of edge 2x, where (x, y) is a sample point on the
= π [6(4 − y)1/2 − 4 + y] dy line in the xy-plane.
4 y

∫ π [6(4 − y)
15
V= 1/2
− 4 + y] dy y = –(15/4)x + 15
0
4
= π [ −4( 4 − y)3/2 − 4 y + 0.5 y 2 ] (x, y)
0
= π (0 − 16 + 8 + 32 + 0 − 0) = 24π
= 75.3982… x
15. y = 4 − x2 is rotated about the line y = −5. Only 4

the back half of the solid is shown.


Equation of line is
15 4
4
y y = − x + 15 ⇒ x = 4 − y.
(x, y ) 4 15
2
 4 
dV = (2 x ) dy = 4 4 − y dy
2
2 x
 15 
2
= 64 1 − y dy
1
y = –5  15 
2 3 15
64 1 − y dy = −320 1 − y
15 1 1

0  15   15 
0

= 320 cm3
The circumscribed rectangular box has volume
l · w · h = 8 · 8 · 15 = 960 = 3V, so the pyramid
is 1/3 the volume of the circumscribed rectangular
solid, Q.E.D.
dV = π[(y + 5)2 − 52] dx The volume of a pyramid is one-third the volume
= π[(9 − x2)2 − 52] dx = π (56 − 18x2 + x4) dx of the circumscribed rectangular box, just as the
2 volume of a cone is one-third the volume of the
V= ∫ π (56 − 18x + x 4 ) dx
2
0 circumscribed cylinder.
= π (56 x − 6 x 3 + 0.2 x 5 )
2
18. Center line: y = 0.2x2
0
Upper bound: y = 0.16x2 + 1
= π (112 − 48 + 6.4 − 0 + 0 − 0) Radius of circular cross section is 1 − 0.04x2.
= 70.4π = 221.168… The tip of the “horn” is where 0.2x2 = 0.16x2 + 1
16. Cross sections perpendicular to the x-axis are with x ≥ 0, which is at x = 5.
squares with side length (y2 − y1). The curves dV = π (1 − 0.04x2)2 dx
intersect at (0, 0) and (1, 1). = π (1 − 0.08x2 + 0.0016x4) dx
5
V= ∫ π (1 − 0.08x + 0.0016 x 4 ) dx
2
y
0
5
= π x −
0.08 3 0.0016 5 
1 x + x
 3 5  0

= π  5 − + 1 − 0 + 0 − 0
1
x 10
 3 
8
dV = (y2 − y1)2 dx = (x1/5 − x2)2 dx = π = 8.3775… ≈ 8.4 cm 3
3

Calculus Solutions Manual Problem Set 5-9 105


© 2005 Key Curriculum Press
19. a. y = x 0.6
1
2b √3
Pick sample point (x, y) on the curve within
the slice. One leg of the isosceles triangle is
y, so the other leg is also equal to y.
60°
1 1
dV = y 2 dx = x 1.2 dx 1
2b
2 2 b

4 1 2.2 4

1 1.2 1 1 1 3
b. V = x dx = x = ⋅ 4 2.2 − 0 dV = bh dx = ( y2 − y1 ) ( y2 − y1 ) dx
0 2 4.4 0 4. 4 2 2 2
= 4.7982… 3 3
= ( y2 − y1 )2 dx = (2 − x 2 − x 2 )2 dx
The midpoint Riemann sum R100 gives 4 4
4.7981… , which is close to the answer found 3
using integration. = (2 − 2 x 2 )2 dx
4
c. If the cross sections were squares, they would 1 3
have twice the area of the triangles, so dV
1
V= ∫
0 4
(2 − 2 x 2 )2 dx
would be twice as much and V = ⋅ 4 2.2 = 3 1
9.5964… .
2.2 =
4 0 ∫
( 4 − 8 x 2 + 4 x 4 ) dx

20. y = ex, y = 3, and x = 0. Cross sections 3 8 4 1


= 4x − x3 + x5
perpendicular to the x-axis are rectangles with 4  3 5 0
height equal to 4 times the base. Each base has 3 8 4 
length (3 − y). = 4− + −0+0−0
4  3 5 
8 3
= = 0.9237...
15
22. y = ln x and y = 1. Cross sections perpendicular
y
to the y-axis are rectangles with height equal to
2
1/2 the base. Each base has length x.
1 x
y
5
dV = (3 − y)[ 4(3 − y)] dx
= (3 − e x )[ 4(3 − e x )] dx = 4(3 − e x )2 dx
ln 3 ln 3
V= ∫0
4(3 − e x )2 dx = 4 ∫ 0
(9 − 6e x + e 2x ) dx
1
x

ln 3
= 4 9 x − 6e x + e 2x 
1
dV = x ⋅ 12 x dy = 12 x 2 dy
 2 0
Solve y = ln x for x, to get x = e y.
= 4 9 ln 3 − 6 ⋅ 3 + ⋅ 9 − 0 + 6 − 
1 1
dV = 12 (e y )2 dy = 12 e 2 y dy
 2 2
11 1 2y 1 1 2 1
= 7.5500 … V= ∫2
0
e 2 y dy =
4
e = e − = 1.5972 K
4 4
21. y = x2 and y = 2 − x2, intersecting at x = 1. 0

Cross sections perpendicular to the x-axis are 23. a. Line has equation y = 12 x, 0 ≤ x ≤ 6.
equilateral triangles. Each base has length (y2 − y1). b. The log has radius = 6, so the circle is
x2 + z2 = 36, or z = 36 – x 2 = (36 − x 2 )1/2 .
1
y c. dV = y ⋅ 2z ⋅ dx = x · 2(36 − x 2)1/2 ⋅ dx
1 2
= (36 − x2)1/2 (x dx)
6
V= ∫ (36 – x ) ( x dx )
2 1/2
x
1 0
6
= − ∫ (36 – x ) ( −2 x dx )
1 2 1/2
Using properties of special right triangles, you 2 0
can find that an equilateral triangle with 1 2 6

3 =− ⋅ (36 – x 2 )3/2 = 72 in.3


base b has height b. 2 3 0
2

106 Problem Set 5-9 Calculus Solutions Manual


© 2005 Key Curriculum Press
24. The points (0, 0) and (r, h) in xy-coordinates are Slicing perpendicular to the y-axis as in Problem
on the line running up the top surface, so the 26 gives dV = πx2 dy = π(r2 − y2) dy.
h
line is y = x. The circle forming the boundary
r
π (r 2 − y 2 ) dy = π  r 2 y − y 3 
r


1
r V=
for the bottom surface has radius = r and center −r  3  −r
(0, 0) in xz-coordinates, so the circle is x2 + z2 =
= π  r 3 − r 3  − π  − r 3 + r 3  = πr 3, Q.E.D.
1 1 4
r2, or z = r 2 – x 2 . The slab at x = x0 is  3   3  3
hx 28. The graph shows slices perpendicular to x-axis
rectangular of height y = 0 , width
r with sample points (x, y) and (x, z).
2 z = 2 r 2 – x 02 , and thickness dx, so y
b (x, y )
2 hx 2
dV = r − x 2 dx, and
r
x
2h r
V=
r 0 ∫
x r 2 − x 2 dx c (x, z )
a

h r 2
=− ∫(r − x 2 )1/ 2 ( −2 x dx ) z
r 0
r
2 h 2 h 2 2 2
= − ⋅ (r 2 − x 2 )3/2 = − ⋅ (0 3/ 2 − r 3 ) Equation of ellipsoid is   +   +   = 1.
x y z
3 r 0 3 r  a  b  c
2 2 For a fixed value of x, the x-term will be
= r h
3 constant. Subtracting this term from both sides
25. A cone of radius r and altitude h can be generated of the equation gives an equation of the form
2 2
by rotating about the x-axis the line  y  +  z  = k 2 , where k2 = 1 − (x/a)2.
r
y = x from x = 0 to h.  b  c
h Dividing both sides by k2 gives
2 2
y  y  +  z  = 1. Thus, the y- and z-radii are
(x , y )  kb   kc 
r
kb and kc, which have the original ratio b/c.
x
h Therefore, each elliptical cross section is similar
to the ellipse at the yz-plane, Q.E.D.
dV = π yz dx
Because z = (c/b)y, dV = π (c/b)y2 dx.
The ellipse in the xy-plane (z = 0) has equation
r2 2 2 2
dV = πy 2 dx = π x dx  x  +  y  = 1, from which y2 = (b/a)2(a2 − x2).
h2  a  b
h
πr 2 h πr 2 1

1 ∴ dV = π (c/b)(b/a)2(a2 − x2) dx
V= x 2 dx = 2 ⋅ x 3 = πr 2 h, Q .E.D .
h2 0 h 3 0 3 a

26. a. Equation of circle in xy-plane is x 2 + y 2 = 100. V= ∫ −a


π (c/b)(b/a)2 ( a 2 − x 2 ) dx
dV = πx2 dy = π(100 − y2) dy a
= π (c/b)(b/a)2  a 2 x − x 3 
1
 3 
10
V= ∫
−10
π (100 − y 2 ) dy
4 4
−a

10
= π (c/b)(b/a) ⋅ a 3 = πabc
2
= π 100 y − y 3 
1
3 3
 3  −10
Note that the volume formula for a sphere is a
= π 1000 − (1000) + 1000 − (1000)
1 1 special case of the volume formula for an
 3 3  ellipsoid in which a = b = c = r, the radius of the
4 sphere.
= π (1000) cm 3
3 29.
b. Formula: V = 43 πr 3 = 43 π 10 3 = 43 π (1000) cm3, 50 + 2 L

which agrees with the answer by calculus. L L

y
27. Sphere can be generated by rotating about the
y-axis the circle x2 + y2 = r2. 50

Calculus Solutions Manual Problem Set 5-9 107


© 2005 Key Curriculum Press
Note that the top of each isosceles trapezoidal 7.
cross section has length 50 + 2L yards, where y

y 52π
= tan (52°) ⇒ L = y cot (52°) = y cot . 2
L 180
So each slab is dV = 2 (50 + 2 L + 50) y dx;
1

52π x
dV = 50 y + y 2 cot dx, and
180
π
52π 
19
V= ∑  50 y k + yk2 cot
180 
⋅ 30 ∫0
π sin 2 x dx = 4.9348K
k =0
We cannot compute this integral algebraically
= 1, 649, 443.6 K ≈ 1, 649, 443 yd 3 . because we do not know an antiderivative for
Cost = 12 ⋅ 1,649,443.6… ≈ $19,793,324 sin 2 x.
8.
y
Problem Set 5-10 2

1 3 x
Q1. x + x+C Q2. 24 5
3

∫ sec x dx = tan x + C
2
Q3. Q4. 2 sec2 x tan x
5
Q5. Answers may vary. Q6. See graph in Q5.
∫ π (ln x ) dx = 14.6673K
2
2
v
We cannot compute this integral algebraically
(t, v ) because we do not know an antiderivative for
(ln x)2.
dt t
9. a.
Si x
2
Q7. See graph in Q5. Q8. d(disp) = v dt
b
Q9. ∫ v dt
a
Q10. A
–20
x
20
1.4
1. ∫ cos x dx ≈ 0.6899295233K
0.3 –2

∫ ( x – 3x + 5) dx = 13.5
2
2. b. (sin x)/(x) approaches 1 as x approaches zero.
1
3 c. Answers will vary depending on the grapher
∫ 2 dx ≈ 10.0988652K
x
3. used. The TI-83 gives Si 0.6 = 0.58812881
0
1.4 using TRACE or 0.588128809608 using TABLE,
4. ∫ tan x dx ≈ 1.76714178K
0.1
both of which are correct to as many decimal
1.4 1.4 places as the NBS values.
5. ∫0.3
cos x dx = sin x
0.3
= sin 1.4 − sin 0.3 = d. By TABLE, Si x seems to be oscillating between
about 1.53 and 1.61 when x is between 20
0.6899295233… and 30. The limit is somewhere between
For the ten digits of the answer shown by these two numbers, say about 1.57. The
calculator, there is no difference between this actual limit is π /2, which equals 1.570796… .
solution and the solution to Problem 1. e.
4 Si x
2
( x − 3 x + 5) dx =  x 3 − x 2 + 5 x 
4


2 1 3
6.
1 3 2  1 x

=  ( 4)3 − ( 4)2 + 5( 4)


1 3 –20 20

3 2 
−  (1)3 − (1)2 + 5(1) = 13.5
–2
1 3
3 2  The f graph is positive when x is between −π
There is no difference between this answer and and π (as well as elsewhere), and has its
the solution to Problem 2. greatest values there, which agrees with the

108 Problem Set 5-10 Calculus Solutions Manual


© 2005 Key Curriculum Press
large positive slope of the Si x graph in this c. The answer by Simpson’s rule should be
region. Each place where the Si x graph has a closer, because the graph is represented by
high or low point, the f ( x) graph has a zero, curved segments instead of straight ones.
corresponding to the zero slope of the Si x 12. (Data and CAT scans for this problem were
graph. So f ( x) is the derivative of Si x. provided by Dr. James Stewart of San Antonio.)
2 x −t2

8
10. a. erf x =
π 0
e dt a. ∫0
A dD ≈ (0.8/3)(6.8 + 4 ⋅ 6.8 + 2 ⋅ 20.1

The integrand e − t is an even function + 4 ⋅ 25.3 + 2 ⋅ 29.5 + 4 ⋅ 34.6 + 2 ⋅ 38.4


2

integrated between symmetrical limits. Thus, + 4 ⋅ 33.9 + 2 ⋅ 15.8 + 4 ⋅ 6.1 + 2.3)


rather than using the entire interval [−x, x], = (0.8/3)(643.5) = 171.6 cm3
one may find the integral on [0, x] and double b. The mass will be 171.6 g, which is within
the result. the normal range of 150 to 200 g.
b. 13. a.
y = erf x
Force (lb)
1

300

x
Distance (in.)
2
0.5

c. By TABLE, values of erf x are


b. Let F = force, x = displacement, W = work.
x erf x 0.5

1 0.842700792…
W= ∫ 0
F dx

2 0.995322265… ≈ (0.05/3)(0 + 4 ⋅ 120 + 2 ⋅ 240 + 4 ⋅ 360


+ 2 ⋅ 370 + 4 ⋅ 330 + 2 ⋅ 290 + 4 ⋅ 280
3 0.999977909…
+ 2 ⋅ 270 + 4 ⋅ 270 + 190)
4 0.999999984… = (0.05/3)(7970) = 132.8333…
The values approach 1, meaning that the ≈ 132.8 inch-pounds
fraction of the data between −x and x is 14. Let C = heat capacity (Btu/lb mole)/ºF,
virtually 100% when x is beyond 4. T = temperature (ºF), H = heat (Btu/lb-mole).
d. Answers will vary depending on the Approximate values of C to the nearest 0.02 are
grapher used. The TI-83 gives erf 0.5 = from the given figure.
0.52049987781… using TABLE, which is
correct to as many decimal places as the NBS T C Simpson’s factor
value.
500 8.44 1
e.
y = erf x
1000 9.24 4
1
1500 10.08 2
2000 10.84 4
2500 11.48 2
x
3000 11.98 4
2 3500 12.36 2
The slope of y = erf x appears to equal about 4000 12.68 4
1 when x = 0 and decreases toward zero as x 4500 12.94 1
increases, which agrees with the graph of f. 4500


500
12 H= C dT ≈
[8.44 + (9.24)( 4)
11. a. ∫0
(speed) dt ≈ [(2/60)/3](33 + 4 ⋅ 25 + 2 ⋅ 27 500 3
+ (10.08)(2) + (10.84)(4) + (11.48)(2)
+ 4 ⋅ 13 + 2 ⋅ 21 + 4 ⋅ 5 + 9)
+ (11.98)(4) + (12.36)(2)
= (1/90)(310) = 3.444… ≈ 3.4 nautical miles
+ (12.68)(4) + 12.94]
b. T 6 = (1/30)(33 ⋅ 0.5 + 25 + 27 + 13 + 21 + 5 500
+ 9 ⋅ 0.5) = (268.18) = 44696.6666 K ≈ 44,697 Btu
3
= (1/30)(112) = 3.7333… ≈ 3.7 nautical miles
The answers students get will vary slightly.

Calculus Solutions Manual Problem Set 5-10 109


© 2005 Key Curriculum Press
15. a. Simpson’s rule will give a more accurate d. f (2) = 0.6931…
answer because the function y = sin x is f ( 3) = 1.0986…
approximated better by quadratic functions
f ( 6) = 1.7917…
than by straight lines.
f ( 2) + f ( 3) = f ( 2 ⋅ 3). This is a property of
b. S4 = (1/3)(π /4)[sin 0 + 4 sin (π /4) + logarithmic functions.
2 sin (π/2) + 4 sin (3π /4) + sin π] = 2.0045…
T4 = (1/2)(π /4)[sin 0 + 2 sin (π/4) +
2 sin (π /2) + 2 sin (3π /4) + sin π] = 1.8961… Problem Set 5-11
π π


Review Problems
sin x dx = − cos x = cos π − cos 0 = 2
0 0
R0. Answers will vary.
Simpson’s rule does give a better R1. a. The width of each region is 4. So
approximation of the integral because S4 is
closer to 2 than is T4. T3 = (4/2)[v(4) + 2v(8) + 2v(12) + v(16)] =
2[22 + 2(26.9705…) + 2(30.7846…) + 34] =
16. Programs will vary depending on the type of 343.0206… . T3 underestimates the integral
grapher used. See the program in the Programs because v(t) is concave down, so trapezoids
for Graphing Calculators section of the are inscribed under the curve.
Instructor’s Resource Book.
b. R3 = 4[v(6) + v(10) + v(14)] = 4(24.6969… +
17. Using a Simpson’s rule program, the mass of the 28.9736… + 32.4499…) = 344.4821…
spleen is 171.6 cm3.
This Riemann sum is close to the trapezoidal-
2 − x2
18. Enter Y1 = e . A Simpson’s rule program rule sum.
π c. T50 = 343.9964… , and T100 = 343.9991…
gives S50 = 0.5204998781… and S100 = Conjecture: The exact value of the integral
0.5204998778… . There is little difference is 344.
between the two estimations, and both are close d. g ( t) = 10t + 4t1.5
to the tabulated value.
g(16) − g(4) = 344
19. a.
y
This is the value the trapezoidal-rule sums are
approaching.
R2. a. The slope of the linear function is the same
2 as the slope of the curve at x = 1. So the
slope is
1 x t f ( x) = sin π x ⇒ f ′(x) = π cos π x ⇒
f ′ ( 1) = π cos π = − π
As x varies, the area beneath the curve y = 1/t At x = 1, y = sin π = 0
from t = 1 to t = x varies also.
y − 0 = − π (x − 1) ⇒ l(x) = − π x + π
t0
b. Using the power formula on t −1 dt gives ∫ 0
.
1
y
l (x)
Division by zero is undefined, so this
approach does not work. f (x)
x
c. Graph Y1 = fnint(x− 1, x, 1, x). (Entries may 1

be different for different calculators.) The


graph looks like y = ln x. The value of f (x)
is negative for x < 1 because for these values
the lower limit of integration is larger than the l (x )
f (x )
upper limit, resulting in negative values for dx.
0.2
x
y
0.8 1 1.2

1
x
5

As you zoom in, you see that f ( x) is very


close to the line l(x) for values near x = 1.

110 Problem Set 5-11 Calculus Solutions Manual


© 2005 Key Curriculum Press
For x = 1.1, the error is sin [π ( 1.1)] − iii. M6 = 2.209073…
[−π ( 1.1) + π] = 0.0051… iv. T6 = 2.359018…
For x = 1.001, the error is sin [π ( 1.001)] − d. U6 L6
[−π(1.001) + π] = 5.1677… × 10− 9 .
y y
5 5
b. i. y = csc5 2x ⇒ dy = −10 csc5 2x cot 2x dx
ii. y = x 5 /5 − x − 3/3 ⇒ dy = (x4 + x− 4) dx
iii. y = (7 − 3x)4 ⇒ dy = −12(7 − 3x)3 dx x x
0.2 1.4 0.2 1.4
iv. y = 5e− 0. 3x ⇒ dy = −1.5e− 0. 3x dx
M6 T6
1
v. y = ln (2 x ) ⇒ dy = 4
⋅ 4(2 x )3 ⋅ 2 dx y y
(2 x ) 4 5 5

= 4/x dx; or y = ln ( 2x)4 = 4 ln ( 2x) ⇒


1
dy = 4 ⋅ ⋅ 2 dx = 4/x dx x x
2x 0.2 1.4 0.2 1.4

c. i. dy = sec x tan x dx ⇒ y = sec x + C e.


1
ii. dy = (3 x + 7)5 dx ⇒ y = (3 x + 7)6 + C y

18
iii. dy = 5 dx ⇒ y = 5x + C f (x)

iv. dy = 0.2e− 0.2 x ⇒ y = −e− 0.2 x + C


x
6x
v. dy = 6 x dx ⇒ y = +C
ln 6
d. i. y = (2x + 5)1/2 ⇒ dy = (2x + 5)− 1/2 dx R5. a. The hypothesis is the “if ” part of a theorem,
and the conclusion is the “then” part. (Hypo-
ii. x = 10 and dx = 0.3 ⇒ means “under,” and -thesis means “theme.”)
dy = 25− 1/2 ⋅ 0.3 = 0.06 π
b. d (t ) = 20 + 3 sin t
iii. ∆y = (2 ⋅ 10.3 + 5)1/2 − (2 · 10 + 5)1/2 4
= 0.059644… d (2) – d ( 0)
Average velocity = = 1.5 m/s
iv. 0.06 is close to 0.059644… . 2–0
π
R3. a. See the text for the definition of indefinite Instantaneous velocity = d′(t) = 0.75π cos t
4
integral. π
d ′(c) = 0.75π cos c = 1.5
∫ 12 x
4
b. i. 2/3
dx = 7.2 x 5/3 + C π
c = cos− 1 (2/π) = 0.880689…
4
∫ sin
1
ii. 6
x cos x dx = sin 7 x + C c = 1.12132… ≈ 1.12 s
7
c. g(x) = x 4/3 − 4x 1/3 = x 1/3(x − 4)

∫ (x
1 3
iii. 2
− 8 x + 3) dx = x − 4 x 2 + 3x + C g(x) = 0 ⇒ x = 0 or x = 4. Interval is [0, 4].
3
g′(x) = (4/3)x1/3 − (4/3)x− 2/3 = (4/3)x− 2/3(x − 1)
∫ 12e dx = 4e 3x + C g′(c) = 0 ⇒ c = 1
3x
iv.
At x = 0, g′(0) takes the form 1/0, which is
7x
∫ 7 dx = ln 7 + C
x infinite.
v.
Thus, g is not differentiable at x = 0.
R4. a. See the text for the definition of integrability. However, the function need not be
differentiable at the endpoints of the interval,
b. See the text for the definition of definite
just continuous at the endpoints and
integral.
differentiable at interior points.
1.4
c. ∫0.2
sec x dx d. For a function to be continuous on a closed
interval, the limit needs to equal the function
i. U6 = 2.845333… value only as x approaches an endpoint from
ii. L6 = 1.872703… within the interval. This is true for function f

Calculus Solutions Manual Problem Set 5-11 111


© 2005 Key Curriculum Press
at both endpoints, but not true for function g R3 = (1.513…)1.5 + (2.508…)1.5 + (3.505…)1.5
at x = 2. The graphs show that the conclusion = 12.4, which is the exact value of the
of the mean value theorem is true for f but integral.
not for g. 3 3

f (x ) Tangent parallels secant


b. ∫ −1
(10 – x 2 ) dx = 10 x − (1/3) x 3
−1
= 30 − 9 − (−10) + (−1/3) = 92/3 = 30.6666…
4
Secant c. T100 = 30.6656, which is close to 92/3.
d. M 10 = 30.72
x M 100 = 30.6672
2 c 7 M 1000 = 30.666672
g (x )
No tangent
These Riemann sums are approaching 92/3.
parallels 5
5

∫ x −2 dx = − x −1 = −5 −1 + 1−1 = 4/5
secant
4 Secant R7. a. i.
1 1
4

∫ (x + 3) 5 ( x dx )
2
ii.
x 3
4

∫ (x
2 7
= (1/2) 2
+ 3)5 (2 x dx )
(Middle branch has the equation y = 1.4 . ( x−2 ) 3
4
Point c = 4.4825… .) = (112
/ )( x 2 + 3)6 3
e. g is the linear function containing the points = (1/12)(19)6 − (1/12)(12)6
(a, f (a)) and (b, f (b)). h is the function h (x) = = 3,671,658.08…
f ( x) − g(x). Thus, h(a) = h ( b) = 0, satisfying π
π
one hypothesis of Rolle’s theorem. The other
two hypotheses are satisfied because f and g
iii. ∫ 0
(sin x – 5) dx = −cos x − 5 x
0
are differentiable and continuous at the = −cos π − 5π + cos 0 + 0 = 2 − 5π
appropriate places, and a difference of ln 5 ln 5
differentiable and continuous functions also
has these properties. The c in (a, b) for which
iv. ∫ 0
4e 2 x dx = 2e 2 x
0
= 2e 2 ln 5 − 2e 0 = 48

h′(c) = 0 turns out to be the c in (a, b) for 4 x 4


34 31

3 78
which f ′ (c) equals the slope of the secant line, v. 3 x dx = = − =
1 ln 3 ln 3 ln 3 ln 3
g′(c), which equals [f (b) − f (a)]/(b − a). 1
= 70.9986…
f.
f (x ) b.
8 y

x
1 –5

Points are 18 , 14 , 83 , 12 , 85 , 43 , and 87 . Integral is negative, because each y-value in


g. If r′(x) = s′(x) for all x in an interval, then the Riemann sum is negative.
r(x) = s(x) + C for some constant C. 10 10
c. ∫ ( 4 sin x + 6 x 7 – 8 x 3 + 4) dx = 2 ∫ 4 dx

−10
R6. a. g( x ) = x 1.5 dx = 0.4 x 2.5 + C 10
0

= 8x = 80
Without loss of generality, let C = 0. 0
g(2) – g(1) d. Total area = sum of two areas
g′(c1 ) = = 1.862741K
2 –1 f (x )

∴ c11.5 = 1.862741… ⇒
c1 = (1.862741…)1/1.5 = 1.513915927…
Similarly, c2 = 2.50833898… .
c3 = 3.505954424… x
4 a c b
For ∫ 1
x 1.5 dx,

112 Problem Set 5-11 Calculus Solutions Manual


© 2005 Key Curriculum Press
R8. a. c. y = x 1 + 2 ⇒ x 1 = y − 2
v 1
(t, v)
y = 3x 2 − 6 ⇒ x 2 = y + 2
3
Graphs intersect at y = 6. Diameter of circular
dt t
cross section is (x2 − x1).
dV = π [0.5(x2 − x1)]2 dy
dy = v dt = 150t0.5 dt 2
π  1 π
2

=  y + 2 − ( y − 2) dy =  4 − y dy
9 9 2
y= ∫ 150t dt = 100t 1.5 = 2700 ft 4  3  4 3 
0.5
0

0 6

For [0, 4], y = ∫ 150t


4
0.5
dt = 800.
V= ∫ dV ≈ 25.1327… (exactly 8π)
0
0
9
The right circular cone of altitude 6 and
For [ 4, 9], y = ∫ 150t 0.5
dt = 2700 − 800 radius 2 also has volume 13 π ⋅ 2 2 ⋅ 6 = 8π .
4
10
= 1900.
9
R10. a. ∫ log x dx = 6.0913K
∫ v(t ) dt = 2700 = 800 + 1900
1
So
0 The integral is reasonable because counting
4 9 squares gives approximately 6.
= ∫ v(t ) dt + ∫ v(t ) dt.
0 4 v (t )
b. dA = x dy 1
y = ln x ⇒ x = e y t
1 10
dA = ey dy
ln 4 ln 4

∫0
e y dy = e y
0
= e ln 4 − e 0 = 4 − 1 = 3
b. dW = v · y · dx = (1000 + 50x)(4 − 0.2x2) dx
R9. a. y = e , from x = 0 to x = 4, is rotated about
3

∫ (1000 + 50 x )(4 − 0.2 x ) dx = 10, 897.5


0.2 x
2
the x-axis. 0
5

∫ v(t ) dt = 1/3(0.2)[29 + 4(41) + 2(50) + 4(51)


y
(x, y )
c.
3
1
x
+ 2(44) + 4(33) + 2(28) + 4(20)
0 4 + 2(11) + 4(25) + 39] = 67.6
Values of velocity are more likely to be
connected by smooth curves than by straight
dV = π y2 dx = πe0.4 x dx lines, so the quadratic curves given by
4 4 Simpson’s rule will be a better fit than the
V= ∫ 0
πe 0.4 x dx = 2.5πe 0.4 x
0
straight lines given by the trapezoidal rule.

= 2.5π (e − 1) = 31.0470…
1.6

b. y = x10.25 and y = x2, intersecting at (0, 0) and Concept Problems


(1, 1) in Quadrant I, is rotated about the
y-axis. Only the back half of the solid is C1. a.
shown. f (b)
5
y
(x 1 , y ) π/2
b
1 –5 5

(x 2 , y )
–5
x
1

b.
y = x10.25 ⇒ x1 = y 4 f (b)
y = x2 ⇒ x2 = y
dV = π ( x 22 − x12 ) dy = π ( y 2 − y 8 ) dy 1
b
1
V = π ( y 2 − y 8 ) dy = π  y 3 − y 9 
1


1 1 3
0 3 9  0
2
= π = 0.6981K
9

Calculus Solutions Manual Problem Set 5-11 113


© 2005 Key Curriculum Press
The two lines are horizontal asymptotes. y4 is Induction hypothesis:
the inverse of y1.
Assume that for some integer n = k > 1,
c. The graph of f in part a is a reflection of S(k) = (k/6)(k + 1)(2k + 1).
the graph of y = tan x in part b across the line Verification for n = k + 1:
y = x. Function f seems to be the inverse S( k + 1) = 0 2 + 12 + L + k 2 + ( k + 1)2
tangent function, f (b) = tan− 1 b. (In Chapter 9,
= (0 2 + 12 + L + k 2 ) + ( k + 1)2
students will learn that this is actually true.)
= (k/6)(k + 1)(2k + 1) + (k + 1)2
C2. f (x) = ax2 + bx + c
y
= [(k + 1)/6][k(2k + 1) + 6(k + 1)]
= [(k + 1)/6][2k2 + 7k + 6]
= [(k + 1)/6][(k + 2)(2k + 3)]
= [(k + 1)/6][(k + 1) + 1] ⋅
[2(k + 1) + 1],
x which is the formula with (k + 1) in place of
d k e
k, thus completing the induction.
f (d) = ad2 + bd + c ∴ S(n) = (n/6)(n + 1)(2n + 1) for any positive
f (e) = ae2 + be + c integer n, Q.E.D.
ae 2 + be + c − ( ad 2 + bd + c) d. S(n) = 0 3 + 13 + 2 3 + 33 + L + n 3
∴m =
e−d S(0) = 0
a( e 2 − d 2 ) + b ( e − d ) S(1) = 0 + 1 = 1
= = a( e + d ) + b
e−d S(2) = 0 + 1 + 8 = 9
f ′(x) = 2ax + b ⇒ f ′(k) = 2ak + b S(3) = 0 + 1 + 8 + 27 = 36
∴ 2ak + b = a(e + d) + b S(4) = 0 + 1 + 8 + 27 + 64 = 100
2ak = a(e + d) (The answers are perfect squares!)
k = (1/2)(e + d) Assume that S(n) = an4 + bn3 + cn2 + dn + e.
∴ k is at the midpoint of [d, e], Q.E.D. 0=0+0+0+0+e
C3. S(n) = 0 2 + 12 + 2 2 + 32 + L + n 2 1=a+b+c+d+e
9 = 16a + 8b + 4c + 2d + e
a. S(0) = 0, S(1) = 1, S(2) = 5, S(3) = 14
36 = 81a + 27b + 9c + 3d + e
S(n) = an3 + bn2 + cn + d
100 = 256a + 64b + 16c + 4d + e
0=0+0+0+d
Solving this system gives
1=a+b+c+d
a = 1/4, b = 1/2, c = 1/4, d = 0, e = 0.
5 = 8a + 4b + 2c + d
∴ S(n) = (1/4)n4 + (1/2)n3 + (1/4)n2
14 = 27a + 9b + 3c + d
= (1/4)n2(n2 + 2n + 1)
Solving this system gives a = 1/3, b = 1/2,
= [(n/2)(n + 1)]2,
c = 1/6, d = 0.
which agrees with the observation that S(n) is
∴ S(n) = (1/3)n3 + (1/2)n2 + (1/6)n
a perfect square.
= (n/6)(n + 1)(2n + 1) By equation,
b. By equation, S(5) = [(5/2)(6)]2 = 225
S(4) = (4/6)(5)(9) = 30 S(6) = [(6/2)(7)]2 = 441
S(5) = (5/6)(6)(11) = 55 By addition,
By addition, S(5) = 03 + 13 + 23 + 33 + 43 + 53 = 225,
S(4) = 0 + 1 + 4 + 9 + 16 = 30, which checks. which checks.
S(5) = 0 + 1 + 4 + 9 + 16 + 25 = 55, which S(6) = 03 + 13 + 23 + 33 + 43 + 53 + 63 =
checks. 441, which checks.
S(1000) = (1000/6)(1001)(2001) = π
333,833,500 C4. a. ∫ 0
4 sin x sin 10 x dx
c. Prove that S(n) = (n/6)(n + 1)(2n + 1) for any π

positive integer n. = ∫ [–2 cos 11x + 2 cos (–9 x )] dx


0
π
Proof (by induction on n): 2 2
=− sin 11x − sin ( −9 x )
Anchor: 11 9 0

For n = 1, S(n) = (1/6)(2)(3) = 1, the correct = −0 − 0 + 0 + 0 = 0, Q .E .D .


answer, which anchors the induction. There is just as much area below the x-axis as
there is above it, so the integral is 0.

114 Problem Set 5-11 Calculus Solutions Manual


© 2005 Key Curriculum Press
π
b. From part a, 0 ≤ U n − Ln ≤ ||P|| (1.29 − 1.21).
b. ∫0
4 sin x sin nx dx
As ||P|| approaches zero, the rightmost
π

∫ {−2 cos [(1 + n) x] + 2 cos [(1 − n) x]} dx


member of the inequality goes to zero. By the
=
0 squeeze theorem, lim (Un − Ln ) = 0, which
π P →0
–2 2
= sin [(1 + n) x ] + sin [(1 − n) x ] implies lim Un = lim Ln . So f is integrable
1+ n 1– n 0 P →0 P →0
–2 2 on [1, 9] by the definition of integrability,
= sin (1 + n)π + sin (1 − n)π
1+ n 1– n Q .E .D .
–2 2 c. Prove that g(x) = 1/x is integrable on [1, 4].
− sin 0 − sin 0
1+ n 1– n
Proof:
If n is an integer, the first two terms will
involve sines of integer multiples of π, and Partition the interval [1, 4] into n
are thus equal to 0. The last two terms are 0 subintervals whose widths are not necessarily
unless n = ±1. Thus, the integral equals 0 for equal. Let ||P|| be the norm of the partition.
any integer n > 1, Q.E.D. Pick sample points ck at the left end of each
subinterval. Because g (x) is decreasing on
C5. a. Algebraic solution:
[1, 4], the high points are located at the left
Pick sample points ck at the right end of each ends of the subintervals and the low points
subinterval. Because f (x) is increasing on the are at the right ends (graph).
interval [1, 9], the high points of f (x) are
y
located at the right ends of the subintervals 1
and the low points are at the left ends.
n n
∴ Un = ∑
k =1
f (ck )∆x k and Ln = ∑ f (c
k =1
k −1 ) ∆x k

Note that c0 = 1 and cn = 9. Subtracting gives


x
1 4
n
Un − Ln = ∑ [ f (c ) − f (c
k k −1 )]∆x k By algebraic or graphical reasoning as in
k =1 part a, Un − Ln ≤ ||P||(1 − 1/4). As ||P||
= [ f (c1 ) − f (c0 )]∆x1 + [ f (c2 ) − f (c1 )]∆x 2 + L approaches zero, Un − Ln is squeezed to zero.
+ [ f ( cn) − f ( cn− 1)]∆xn Thus, Un and Ln approach the same limit,
≤ [ f ( c1) − f ( c0)] ||P|| + [ f ( c2) − f ( c1)] ||P|| which implies that g is integrable on [1, 4],
+ L + [ f (cn ) − f (cn−1 )] || P || Q .E .D .
= [ f (c1 ) − f (c0 ) + f (c2 ) − f (c1 ) + L + f (cn ) d. This reasoning cannot be applied directly to
− f ( cn− 1)] ||P|| h (x) = sin x on the interval [0, 3] because
= [ f ( cn) − f ( c0)] ||P|| = ||P||(1.29 − 1.21) h (x) is both increasing and decreasing on
∴ Un − Ln ≤ ||P||(1.29 − 1.21), Q.E.D. different parts of the interval. Thus, the high
Graphical solution: points are not always at the same end of the
The difference Un − Ln is equal to the area of subinterval and the high point at π/2 may not
the spaces between the lower and upper be at either end of a subinterval (graph).
rectangles in Figure 5-11g. Imagine these y = sin x
1
spaces moved over to the left so that they
align at x = 1 (graph). The spaces can be
circumscribed with a rectangle of base ||P||
and altitude (1.29 − 1.21). Thus, Un − Ln ≤
x
||P|| (1.29 − 1.21), Q.E.D. 0 π/2 3
f (x )
1.29 Slide them over. The reasoning could be applied indirectly by
first splitting the interval [0, 3] into [0, π /2]
and [π /2, 3] so that h(x) is increasing on one
and decreasing on the other.

1.21 Chapter Test


x
T1. Indefinite integral:
1 Norm ∆x 9

∫ f ( x ) dx if and only if g′(x) = f (x).


Norm = largest
g( x ) =

Calculus Solutions Manual Problem Set 5-11 115


© 2005 Key Curriculum Press
−2 −2
T2. Definite integral:

10 3
T12. (12 x 3 + 10 x 2 ) dx = 3 x 4 + x
Let Ln and Un be lower and upper sums of f (x) 2 3 2
on the interval [a, b]. Then 80 80 1
b = 48 − − 48 − = −53
∫ f ( x ) dx = lim Ln = lim Un , 3
d
3 3

∫ [( x )
n→∞ n→∞
T13. π
a
1
2
− ( x 2 )2 ] dy
provided the two limits are equal. c

T3. Fundamental theorem: T14. The slope of the linear function is the same as
If f is an integrable function, and the slope of the curve at x = 1. So the slope is
found by y = x3 ⇒ y′ = 3x2 ⇒ y′(1) = 3.
g( x ) = ∫ f ( x ) dx, then At x = 1, y = 1.
b y − 1 = 3(x − 1) ⇒ y = 3x − 2
∫a
f ( x ) dx = g(b) − g( a).
y

T4. The function f is continuous on the interval


1
[3, 8] and differentiable on (3, 8). It does not
matter that it is not differentiable at the
endpoints. x
T5. 1

f (x )

1.2

1 (1, 1)

x
3 c 8 1 1.2

As you zoom in on (1, 1), you see that the graph


T6. Hypotheses: f (a) = f (b) = 0 of y = x3 is locally linear.
Differentiable on (a, b). 3 3
Continuous at x = a and x = b.
Conclusion: There is a c in (a, b) such that
T15. a. ∫ 0
12e 0.25 x dx = 48e 0.25 x
0

f ′ (c) = 0. = 48e 0.75 − 48e 0 = 53.6160 K


f(x ) b. M 50 = 53.6154…
T50 = 53.6170…
c x
S50 = 53.61600081…
a b The midpoint Riemann sum error is
0.000502646712… .
The trapezoidal-rule error is 0.0010052962… .
The midpoint Riemann sum error is half of
T7. the trapezoidal-rule error, because
f (x ) 2(0.000502646712…) = 0.0010052962… .
The Simpson’s rule error is
0.000000015077… , which is much smaller
than the error for the other two methods.
x
T16. a.
y
3 8
1 y = cos x
T8. y = esin x ⇒ dy = cos x esin x dx (x, y )

∫ 0.1 dx = ln 0.1 + C
x 0.1
T9.
x
π

∫ (4 x
1
T10. 3
+ 13)5 ( x 2 dx ) = ( 4 x 3 + 13)6 + C 1 2 1
72 b. dV = y dx ⇒ dV = cos 2 x dx
4
2 2
4 x3 4 3 13 π /2 1
∫ x dx = = − = 21

2
T11. V= cos 2 x dx
1 3 1
3 3 0 2

116 Problem Set 5-11 Calculus Solutions Manual


© 2005 Key Curriculum Press
c. This integral cannot be evaluated algebraically g( 4) – g(1) 6.4 – 0.1
c. m = = = 2.1
because we do not know an antiderivative for 4 –1 3
cos2 x. g′(x) = f ( x) = 0.3x 2
π /2 1
V=
0 ∫ 2
cos 2 x dx = 0.3926 K ∴ 0.3c2 = 2.1 ⇒ c = 7 = 2.6457513…
In the right graph in part b, the tangent line at

T17. a. g( x ) = 0.3 x 2 dx = 0.1x 3 + C = 0.1x 3 x = 2.64… is parallel to the secant line from
x = 1 to x = 4.
b. d. f ( 2.645…) = 0.3(2.645…)2 = 2.1 (exactly)
f (x ) g(x ) The point is (2.645… , 2.1).
Area of region under graph equals area of
5 5
rectangle, as shown in the graph on the left in
part b.
x x T18. Answers will vary.
1 2.64... 4 1 2.64... 4

Calculus Solutions Manual Problem Set 5-11 117


© 2005 Key Curriculum Press
Chapter 6—The Calculus of Exponential
and Logarithmic Functions
1
Problem Set 6-1 Q5. differentiable Q6. y′ =
1 – x2
1. The integral would be 1
0 P 0 , which involves
division by zero. Q7. y′ = − csc x cot x Q8. Riemann sum
Q9. indefinite integral, or antiderivative
N Integral
1000 0 Q10. log 12
1500 0.4054… 0. Answers will vary.
2000 0.6931… 1. y = ln 7x ⇒ y′ = 1/(7x) · 7 = 1/x
2500 0.9162… 2. y = ln 4x ⇒ y′ = 1/(4x) · 4 = 1/x
500 −0.6931…
100 −2.3205… 3. f (x) = ln x5 ⇒ f ′ (x) = 1/(x5) · 5x4 = 5/x
4. f (x) = ln x3 ⇒ f ′ (x) = 1/(x3) · 3x2 = 3/x
Integral
2
5. h (x) = 6 ln x− 2 ⇒ h′ (x) = 6/(x− 2) · (−2x− 3) = −12/x
1 6. g (x) = 13 ln x− 5 ⇒
N
1000 2000 g′ (x) = 13/(x− 5) · (−5x− 6) = −65/x
–1
7. r(t) = ln 3t + ln 4t + ln 5t ⇒
–2
r′ (t) = 1/(3t) · 3 + 1/(4t) · 4 + 1/(5t) · 5 = 3/t
8. v (z) = ln 6z + ln 7z + ln 8z ⇒
The graph resembles a logarithmic function. v′ (z) = 1/(6z) · 6 + 1/(7z) · 7 + 1/(8z) · 8 = 3/z
10
10
y = (ln 6x)(ln 4x) ⇒

9.
2. 0.05 dt = 0.05t = 0.5 − 0 = 0.5, Q .E.D .
0 0 y′ = 1/(6x) · 6 · (ln 4x) + (ln 6x)[1/(4x) · 4]
ln 24 x 2
0.5 is between 0.4054… and 0.6931… , the = (1/x)(ln 4x + ln 6x) or
values of the left integral for N = 1500 and x
N = 2000. 10. z = (ln 2x)(ln 9x) ⇒
N 1 z′ = 1/(2x) · 2 · (ln 9x) + (ln 2x)[1/(9x) · 9]
By solver,
1000 P ∫
dP = 0.5 when
= (1/x)(ln 9x + ln 2x) or
ln 18 x 2
N = 1648.7212… , or about 1649 people. x
3. At 20 years, the integral on the right equals 1. At ln 11x
11. y = ⇒
0 years, the integral equals 0. Solving for N at ln 3 x
these times gives N = 2718.2818… for 20 years, 1/(11x ) ⋅ 11 ⋅ (ln 3 x ) – (ln 11x )1/(3 x ) ⋅ 3
and N = 1000 (as expected!) for 0 years. y′ =
(ln 3 x )2
Population ln 3 x – ln 11x ln(3/11)
3000 = 2 or
x (ln 3 x ) x (ln 3 x )2
2000
ln 9 x
1000 12. y= ⇒
ln 6 x
1/(9 x ) ⋅ 9 ⋅ (ln 6 x ) – (ln 9 x ) ⋅ 1/(6 x ) ⋅ 6
Time (yr)
5 10 15 20 y′ =
(ln 6 x )2
The graph resembles an exponential function. ln 6 x – ln 9 x ln(2/3)
= or
4. ln 1648.7212… − ln 1000 = 0.5, exactly. This x (ln 6 x )2 x (ln 6 x )2
is the value of the integral on the left!
13. p = (sin x)(ln x) ⇒
p′ = (cos x)(ln x) + (sin x)(1/x)
Problem Set 6-2 14. m = (cos x)(ln x) ⇒
1 0.7 m′ = (−sin x)(ln x) + (cos x)(1/x)
Q1. x +C Q2. 9 15. y = cos (ln x) ⇒ y′ = −sin (ln x) · (1/x)
0.7
Q3. f ′ ( x) = −2 cos x sin x Q4. continuous 16. y = sin (ln x) ⇒ y′ = cos (ln x) · (1/x)

118 Problem Set 6-2 Calculus Solutions Manual


© 2005 Key Curriculum Press
17. y = ln (cos x), where cos x > 0 ⇒ 4 4

y′ = (1/cos x) · (−sin x) = −tan x (Surprise!) 39. ∫0.5


(1/w ) dw = ln | w |
0.5
= ln 4 − ln 0.5
18. y = ln (sin x), where sin x > 0 ⇒ = ln 8 = 2.079441…
y′ = (1/sin x) · (cos x) = cot x (Surprise!) 10 10
19. T (x) = tan (ln x) ⇒ T ′ (x) = sec2 (ln x) · (1/x) 40. ∫0.1
(1/v) dv = ln | v |
0.1
= ln 10 − ln 0.1
20. S (x) = sec (ln x) ⇒
= ln 100 = 4.605170…
S′ (x) = sec (ln x) · tan (ln x) · (1/x)
−3 −3
21. y = (3x + 5)−1 ⇒
y′ = −(3x + 5)−2 · 3 = −3(3x + 5)−2
41. ∫−0.1
(1/ x ) dx = ln | x |
− 0.1
= ln | −3 | − ln | −0.1 |

22. y = (x3 − 2)−1 ⇒ = ln 3 − ln 0.1 = ln 30 = 3.401197…


y′ = −(x3 − 2)−2 · 3x2 = −3x2(x3 − 2) −4 −4

23. y = x 4 ln 3x ⇒
42. ∫−0.2
(1/ x ) dx = ln | x |
−0.2
= ln | −4 | − ln | −0.2 |
y′ = 4x 3 ln 3x + x 4 · 1/(3x) · 3 = ln 4 − ln 0.2 = ln 20 = 2.995732…
= 4x 3 ln 3x + x 3
x 1/ 2 dx 2 9 1
9

∫ ∫
3 1/ 2
24. y = x 7 ln 5x ⇒ 43. 3/ 2 = 3/ 2 ⋅ x dx
4 1+ x 3 4 1+ x 2
y′ = 7x 6 ln 5x + x 7 · 1/(5x) · 5 2 9
2
= 7x 6 ln 5x + x 6 = ln| 1 + x 3/ 2 | = (ln 28 – ln 9) = 0.756653K
3 4 3
25. y = ln (1/x) ⇒ y′ = 1/(1/x) · (−x −2) = −1/x
x −1/ 3 dx 3 8 1
8

∫ ∫
2
26. y = ln (1/x4) ⇒ y′ = 1/(1/x)4 · (−4x−5) = −4/x 44. 2/3 = ⋅ x −1/ 3 dx
1 2+x 2 1 2 + x 2/3 3
27. ∫ 7/x dx = 7 ln | x | + C 3 8
3
= ln | 2 + x 2 / 3 | = (ln 6 – ln 3) = 1.5 ln 2
2 2
∫ 5/x dx = 5 ln | x | + C
1
28. = 1.039720…


dx 1
∫ 1/(3x ) dx = 3 ln | x | + C
1
29. 45. (ln x )5 = (ln x )6 + C
x 6

∫ ∫
ln x dx 1
∫ 1/(8x ) dx = 8 ln | x | + C
1 46. dx = (ln x )1 = (ln x )2 + C
30.
x x 2
x

31. ∫
x2
x +5
3 dx =
1

1
3 x3 + 5
(3 x 2 dx ) 47. f ( x ) = ∫ cos 3t dt ⇒ f ′( x ) = cos 3x
2
x
1
= ln | x 3 + 5 | + C 48. f ( x ) = ∫ (t + 10t – 17) dt ⇒
2
5
3
x5 f ′( x ) = x 2 + 10 x − 17
∫ ∫
1 1
32. dx = (6 x 5 dx )
6
6 x6 – 4 d  x


x –4
49.  tan 3 t dt  = tan 3 x
1 dx  2 
= ln | x 6 – 4 | + C
6 d  x

33. ∫
x5
dx = −
1

1
(–6 x 5 dx )
50. 
dx  ∫ −1
2 t dt  = 2 x

9 – x6 6 9 – x6
x2


1 51. f ( x ) = 3t dt ⇒ f ′( x ) = 2 x ⋅ 3 x
2
= − ln | 9 – x 6 | + C
6 1

x3 cos x
g( x ) = ∫
∫ ∫
1 1
34. 4 dx = − (–4 x 3 dx ) 52. t dt ⇒ g′( x ) = cos x ⋅ ( − sin x )
10 – x 4 10 – x 4 0
1 3 x −5
= − ln | 10 – x 4 | + C 53. h( x ) = ∫ 1 + t 2 dt ⇒
4 0


sec x tan x dx
35. = ln | 1 + sec x | + C h′( x ) = 3 1 + (3 x – 5)2
1 + sec x
x3
sec 2 x dx
36. ∫ 1 + tan x
= ln | 1 + tan x | + C 54. p( x ) = ∫–1
(t 4 + 1) 7 dt ⇒ p′( x ) = ( x 12 + 1) 7 ⋅ 3 x 2


cos x dx 3 3
37.
sin x
= ln | sin x | + C 55. ∫1
(5 / x ) dx = 5 ln | x |
1
= 5 ln 3 − 5 ln 1 = 5 ln 3

∫ ∫
sin x dx – sin x dx
38. =− = − ln | cos x | + C = 5.493061…
cos x cos x

Calculus Solutions Manual Problem Set 6-2 119


© 2005 Key Curriculum Press
Midpoint Riemann sum: M100 = 5.492987… b. F

Trapezoidal rule: T100 = 5.493209…


30
Numerical integration: 5.493061…
(h, F )
56. Answers will vary.
57. a. By finding areas, g (0) ≈ −2.7, g (1) = 0, dh
h

g(2) = 1, g(3) ≈ 0.3, g(4) ≈ −0.3, g(5) ≈ 0.7, 10 20

g (6) ≈ 3.3, g (7) = 6, and g (8) = 7.


c. Work equals force times displacement.
y Because the force varies, a definite integral
7 must be used.
6 g
5 d. The work done compressing the air a small
4 amount, dh, is approximately equal to the
3
force at the sample point (h, F ) times dh
2
1 (see part b).
x
–1
1 2 3 5 6 7 8 dW = F dh = (600/h − 30) dh
10


–2
–3 ∴W= (600/h – 30) dh
20
10
x 2 −1 = 600 ln | h | − 30 h
b. h( x ) = ∫ f (t ) dt ⇒ h ′( x ) = f ( x − 1) ⋅ 2x ⇒
2 20
1 = 600 ln 10 − 300 − 600 ln 20 + 600
h′(2) = f (3) ⋅ 4 = −1 ⋅ 4 = −4 = −115.8883… ≈ −116 inch-pounds
58. a. By finding areas, g (0) = −6, g (1) = −2.5, This number is negative because each value of
g (2) = 0, g (3) = 1.5, g (4) = 2, g (5) = 1.5, dh is negative and F is positive, making their
product negative.
g (6) = 0.75, g (7) = 0.5, g (8) = 0.75,
e. Distance is measured in inches, force is
g (9) = 1.5, and g (10) = 2.75. measured in pounds, and we are finding their
y
product.
4
3 g
61. a. d(f ) = a + b ln f
2 0 = a + b ln 53, 10 = a + b ln 160
1
x
10 = b ln 160 − b ln 53 ⇒
1 2 3 4 5 7 8 9 10 10
–1 b= = 9.050741...
–2 ln 160 – ln 53
–3
a = −9.050741… ln 53 = −35.934084…
–4
–5
d( f ) = –35.934084… + 9.050741… ln f
–6
b.
2x f d cm d′ (part c)
b. h( x ) = ∫
2
f (t ) dt ⇒ h′( x ) = f (2 x ) ⋅ 2 ⇒
53 0 0.1707…
h′(4) = f (8) ⋅ 2 = (0.5)(2) = 1
N
60 1.1227… 0.1508…
N
59. ∫1000
(1/ P) dP = ln | P |
1000
= ln N − ln 1000 70
80
2.5197…
3.7265…
0.1292…
0.1131…
10 10

∫0
0.05 dt = 0.05t
0
= 0.5 100
120
5.7461…
7.3962…
0.0905…
0.0754…
ln N − ln 1000 = 0.5 140 8.7914… 0.0646…
N
ln = 0.5 160 10.0 0.0565…
1000
N The measured distances will vary. They
= e 0.5 should be close to the calculated distances.
1000
N = 1000e0.5 ≈ 1648.721… c. d′ ( f ) = b/f = 9.050…/f. See table in part b.
≈ 1649 people d. d′ ( f ) is in cm/10 kHz.
e. d′ ( f ) decreases as f gets larger; this is
60. a. F + 30 = k/h consistent with the spaces between the
0 + 30 = k/20 ⇒ k = 600 numbers getting smaller as f increases.
∴ F + 30 = 600/h ⇒ F = 600/h − 30

120 Problem Set 6-2 Calculus Solutions Manual


© 2005 Key Curriculum Press
62. a. ln 2 = 0.693147… 3. ln 2001 − ln 667 = 7.60140… − 6.50279… =
ln 3 = 1.098612… 1.09861…
ln (2001/667) = ln 3 = 1.09861…
ln 6 = 1.791759…
4. ln 1001 − ln 77 = 6.90875… − 4.34380… =
ln 2 + ln 3 = ln 6
2.56494…
Conjecture: ln (ab) = ln a + ln b ln (1001/77) = ln 13 = 2.56494…
b. ln (10/2) = ln 5 = 1.609437… 5. 3 ln 1776 = 3(7.48211…) = 22.44635…
ln 10 = 2.302585… ln (17763) = ln 5,601,816,576 = 22.44635…
ln 2 = 0.693147… 6. 4 ln 1066 = 4(6.97166…) = 27.88667…
ln (10/2) = ln 10 − ln 2 ln (10664) = ln 1,291,304,958,736 = 27.88667…
Conjecture: ln (a/b) = ln a − ln b 7. See the text for the proof of the uniqueness
theorem.
c. ln (210) = ln 1024 = 6.931471…
8. See the text for the proof.
ln 2 = 0.6931471…
9. Prove that ln (a/b) = ln a − ln b for
ln (210) = 10 ln 2
all a > 0, b > 0.
Conjecture: ln (ab ) = b ln a
Proof:
ln 2
d. = 2.30258… Let f (x) = ln (x/b), g(x) = ln x − ln b for x, b > 0
log 2 Then f ′ (x) = (b/x)(1/b) = 1/x, and
ln 3 g′ (x) = (1/x) − 0 = 1/x.
= 2.30258…
log 3 ∴ f ′ (x) = g′ (x) for all x > 0.
They seem to be the same. f (b) = ln (b/b) = ln 1 = 0
ln 10 = 2.30258… g(b) = ln b − ln b = 0
1
= 2.30258… ∴ f (b) = g (b).
log e ∴ f (x) = g (x) for all x > 0 by the uniqueness
log 4 = 0.60205… and theorem.
ln 4 1.3862 K ∴ ln (x/b) = ln x − ln b for all x > 0.
= = 0.60205K ∴ ln (a/b) = ln a − ln b for all a > 0 and b > 0,
ln 10 2.30258K
Q .E .D .
63. Answers will vary. 10. Prove that ln (ab) = b ln a for all a > 0 and all b.
Proof:
Problem Set 6-3 Let f (x) = ln (xb); g (x) = b ln x for x > 0.
1 Then f ′ (x) = 1/(xb) · bxb− 1 = b/x and
Q1. y′ = 1/(1 + x 2) Q2. ln | 4 x + 1 | + C g′ (x) = b (1/x) = b/x.
4
∴ f ′ (x) = g′ (x) for all x > 0.
Q3. 1 Q4. 1/4 f (1) = ln (1b) = ln 1 = 0
Q5. 35 Q6. 8 g(1) = b ln 1 = b · 0 = 0
Q7. ∴ f (1) = g (1).
y ∴ f (x) = g (x) for all x > 0 by the uniqueness
1
theorem.
x ∴ ln (xb) = b ln x for all x > 0.
2
∴ ln (ab) = b ln a for all a > 0 and all b, Q.E.D.
11. Prove that ln (a/b) = ln a − ln b for
all a > 0, b > 0.
Q8. f is differentiable on (a, b). Proof:
Q9. f is continuous at x = a and x = b. ln (a/b) = ln (a · b− 1) = ln a + ln b− 1 =
Q10. B ln a + (−1) ln b = ln a − ln b
∴ ln (a/b) = ln a − ln b, Q.E.D.
1. ln 6 + ln 4 = 1.79175… + 1.38629… =
3.17805… 12. Example: ln (2 + 3) = ln 5 = 1.60943…
ln 24 = 3.17805… ln 2 + ln 3 = 0.69314… + 1.09861… =
1.79175…
2. ln 5 + ln 7 = 1.60943… + 1.94591… =
∴ ln (2 + 3) ≠ ln 2 + ln 3.
3.55534…
∴ ln (a + b) = ln a + ln b is false, Q.E.D.
ln 35 = 3.55534…

Calculus Solutions Manual Problem Set 6-3 121


© 2005 Key Curriculum Press
13. See the text definition of ln x. d d
25. (ln x 3 x ) = (3 x ln x ) =
ln x log a x dx dx
14. log b x = = 3x
ln b log a b 3 ln x + = 3 ln x + 3
x
15. f (x) = log3 x ⇒ f ′ (x) = 1/(x ln 3) d d
26. (ln 5sec x ) = (sec x ln 5)
f ′ (5) = 0.182047… dx dx
The graph shows a tangent line with a small = ln 5 sec x tan x
positive slope. 27. a. y = 7 · (2 − 0.9x)
dy/dx = 7(−0.9x)(ln 0.9)
f (x )
dy/dx = 0.737523…(0.9x)
x = 0: dy/dx = 0.737… mi/h
1
x x = 1: dy/dx = 0.663… mi/h
5 x = 5: dy/dx = 0.435… mi/h
x = 10: dy/dx = 0.257… mi/h
The lava is slowing down.
b. y/7 = 2 − 0.9x
0.9x = 2 − y/7
16. f (x) = log0.8 x ⇒ f ′ (x) = 1/(x ln 0.8)
x ln 0.9 = ln (2 − y/7)
f ′ (4) = −1.120355…
x = (1/ln 0.9)[ln (2 − y/7)]
The graph shows a tangent line with slope ≈ −1.
⋅ − 
dx 1 1
c. = (1/ln 0.9) ⋅
f (x ) dy 2 – y/7  7 
dx 9.491221K
5
4 x
=
dy 14 – y
10
y = 10: dx/dy = 2.372… h/mi
d. If x = 10, then y = 7(2 − 0.910), so
dx 9.491221K
= = 3.888651… .
dy 14 – 7(2 – 0.910 )
17. g (x) = 8 ln (x5) = 40 ln x ⇒ g′ (x) = 40/x e. 3.88… is the reciprocal of 0.257… , the
value of dy/dx when x = 10, not when y = 10.
18. h (x) = 10 ln (x0.4 ) = 4 ln x ⇒ h′ (x) = 4/x
28. a. 1000(1.06t) = M ⇒ 1.06t = M/1000 ⇒
1 log1.06 1.06t = log1.06 (M/1000) ⇒
19. T(x) = log5 (sin x) ⇒ T ′( x ) = ⋅ cos x
sec x ⋅ ln 5 t = log1.06 (M/1000)
T′ (x) = (cot x)/(ln 5) 1 1
b. dt/dM = ⋅
20. R (x) = log4 (sec x) ⇒ (ln 1.06)( M/1000) 1000
1 tan x 1
R′( x ) = ⋅ sec x tan x = =
sec x ⋅ ln 4 ln 4 M ln 1.06
21. p (x) = (ln x)(log5 x) ⇒ 1
c. If M = 1000, dt/dM = =
p′ (x) = (1/x) · (log5 x) + (ln x) · [1/(x ln 5)] 1000 ln 1.06
1 ⋅ ln x ln x 2 ln x 0.01716… yr/$. At this rate, with $1000 in
= + =
x ln 5 x ln 5 x ln 5 the account, it would take 0.017… year, or
about 6 days, to earn a dollar of interest.
ln x ln 3 ln 3 1
22. q(x) = (log9 x)/(log3 x) = ⋅ = = d. dt/dM gets smaller as M increases; more
ln 9 ln x 2 ln 3 2
interest is earned when M is larger, so it takes
∴ q′ (x) = 0 because q(x) is constant.
less time to accumulate $1000.
 x3  29. The intersection point is at x = 2.7182818… ,
23. f ( x ) = ln   = ln x − ln sin x
3
 sin x  which is approximately e.
= 3 ln x − ln sin x ⇒ f ′ (x) = 3/x − cot x 30. Answers will vary.
24. f (x) = ln (x4 tan x) = ln x4 + ln (tan x)
= 4 ln x + ln (tan x) ⇒ Problem Set 6-4
4 sec 2 x 4 1 Q1. y′ = 3/x Q2. (−1/10)(5x)− 2 + C
f ′( x ) = + = +
x tan x x sin x cos x Q3. 15 ln 5 x + C Q4. y′ = −1/ 1 – x 2

122 Problem Set 6-4 Calculus Solutions Manual


© 2005 Key Curriculum Press
ln 23 between 5 and 10 years, which agrees with
Q5. 5 tan (5x) Q6.
ln 17 the increasing derivatives shown in part a.
Q7. 36 Q8. 8 m′(t ) 1000(1.06)t (ln 1.06)
c. = = ln 1.06
Q9. B Q10. E m( t ) 1000(1.06)t
0. Answers will vary. ∴ m′ (t)/m(t) = ln 1.06, a constant
d. m(1) = 1060.00. So you earn $60.00.
1. a. R(t) = aekt
The rate starts out at only $58.27/year but
60,000 = aek·0 ⇒ a = 60,000
has increased enough by year’s end to make
2,400,000 = aek·2 = 60,000e2k
the total for the year equal to $60.00.
40 = e2k ⇒ 2k = ln 40 ⇒
k = (ln 40)/2 = 1.844… 4. d(t) = 200t · 2− t ⇒ ln d(t) = ln 200t − t ln 2
1
(Store 1.844… without round-off as k.) ⋅ d ′(t ) = 1/t − ln 2 ⇒
∴ R(t) = 60,000e1.844…t d (t )
b. R ( 5 ) = 60,000e1.844…(5) = 607,157,310.7… d ′(t ) = (200t ⋅ 2 − t )(1/t − ln 2)
About 607 million rabbits. d ′ (1) = (200 · 2− 1)(1 − ln 2) = 30.685…
d ′ (2) = (400 · 2− 2)(1/2 − ln 2) = −19.314…
c. 2 = 60,000e1.844…t ⇒ 1/30,000 = e1.844…t ⇒
So the door is opening at about 30.7°/s at
−ln 30,000 = 1.844…t ⇒ t = −5.589…
1 second and closing at about 19.3°/s at
So the first pair of rabbits was introduced
2 seconds, which agrees with the graph.
about 5.6 years earlier, or in about 1859.
2. a. v (t) = 20,000e− 0.1 t d (t )

v(0) = 20,000e0 = 20,000 100

$20,000 when built.


b. v (10) = 20,000e− 1 = 7357.588…
v(11) = 20,000e− 1.1 = 6657.421… t
At 10 years, value is $7357.59. 1 2

At 11 years, value is $6657.42.


So depreciation is 7357.59 − 6657.42 = The widest opening occurs when d ′ (t) = 0.
$700.17. Solving numerically for t in
(200t · 2− t)(1/t − ln 2) = 0,
c. v′ (t) = −2000e− 0.1 t
t = 1.44269… .
v′ (10) = −2000e− 1 = −735.758… , d(1.44269…) = 106.147…
or about $736 per year. So the widest is about 106° at t ≈ 1.4 s.
This rate is higher than the actual depreciation n
5. e = lim 1 +  and e = lim(1 + n)1/ n
1
in part b because the latter rate is an average
n→∞  n n→0
for the year. The rate at the end will be lower
than 736 to give the average of 700. When you substitute ′ for n in the first equation,
you get the indeterminate form 1∞ . When you
d. 5,000 = 20,000e− 0.1 t substitute 0 for n in the second equation, you
0.25 = e− 0.1 t also get the indeterminate form 1∞ .
ln (0.25) = −0.1t
n (1 + 1/n)n
t = (ln 0.25)/(−0.1) = 13.8629… ≈
14 yr. 100 2.70481…
3. a. m(t) = 1000(1.06)t 1000 2.71692…
ln m(t) = ln 1000 + t ln 1.06 10000 2.71814…
1/m(t) · m′ (t) = 0 + ln 1.06
m′ (t) = m (t) · ln 1.06 n (1 + n)1/ n
m′ (t) = 1000(1.06)t (ln 1.06) 0.01 2.70481…
m′ (0) = 58.27 $/yr 0.001 2.71692…
m′ (5) = 77.98 $/yr
0.00001 2.71826…
m′ (10) = 104.35 $/yr
6. y = 17e ⇒ y′ = −85e− 5x
− 5x
b. m(0) = $1000.00
m(5) = $1338.23 7. y = 667e− 3x ⇒ y′ = −2001e− 3x
m(10) = $1790.85 8. h(x) = x3ex ⇒ h′ (x) = 3x2ex + x3ex = x2ex (3 + x)
The rates are increasing. $338.23 is earned 9. g (x) = x− 6ex ⇒
between 0 and 5 years; $452.62 is earned g′ (x) = −6x− 7ex + x− 6ex = x− 7ex (−6 + x)

Calculus Solutions Manual Problem Set 6-4 123


© 2005 Key Curriculum Press
10. r(t) = et sin t ⇒ r′ (t) = et sin t + et cos t 35. y = x ln x ⇒ ln y = ln x · ln x ⇒ ln y = (ln x)2 ⇒
y ′ = 2 ln x ⋅ ⇒ y ′ = 
s (t) = et tan t ⇒ s′ (t) = et tan t + et sec2 t 1 1 2 ln x  ln x
11. ⋅ x = 2 ln x · xln x−1
y x  x 
12. u = 3exe− x = 3 ⇒ u′ = 0
1
13. v = e4xe− 4x = 1 ⇒ v′ = 0 36. y = (csc 5x)2x ⇒ ln y = 2x ln (csc 5x) ⇒ y′ =
y
ex e x ln x − e x (1/ x )
14. y= ⇒ y′ = 1
ln x (ln x )2 2 ln (csc 5 x ) + 2 x (–5 csc 5 x cot 5 x ) ⇒
csc 5 x
ln x (1/ x )e x − ln x ⋅ e x y′ = (csc 5x)2x [2 ln (csc 5x) − 10x cot 5x]
15. y = x ⇒ y′ =
e e2 x 37. y = (cos 2x)3x ⇒ ln y = 3x ln (cos 2x) ⇒
16. y = 4esec x ⇒ y′ = 4esec x sec x tan x 1 1
y′ = 3 ln (cos 2 x ) + 3 x (–2 sin 2 x ) ⇒
17. y = 7ecos x ⇒ y′ = −7ecos x sin x y cos 2 x
18. y = 3 ln e 2x = 6x ln e = 6x ⇒ y′ = 6 y′ = (cos 2x)3x [3 ln (cos 2x) − 6x tan 2x]
38. Two solution methods are possible.
19. y = 4 ln e 5x = 4 · 5x = 20x ⇒ y′ = 20 Differentiate directly:
20. y = (ln e3x)(ln e4x) = 3x · 4x = 12x2 ⇒ y′ = 24x 5x + 2
y = ln ⇒
21. y = (ln e− 2x)(ln e5x) = −2x · 5x = −10x2 ⇒ 7x – 8
y′ = −20x 7 x – 8  5(7 x – 8) – (5 x + 2) ⋅ 7 
y′ =
22. g(x) = 4eln 3x = 4 · 3x = 12x ⇒ g′ (x) = 12 5 x + 2  (7 x – 8)2 

23. h (x) = 6eln 7x = 6 · 7x = 42x ⇒ h′ (x) = 42 =
–54
24. y = ex + e− x ⇒ y′ = ex − e− x (5 x + 2)(7 x – 8)
Or simplify using properties of logarithms first:
25. y = ex − e− x ⇒ y′ = ex + e− x
3 3 3 y = ln (5x + 2) − ln (7x − 8) ⇒
26. y = e 5 x ⇒ y ′ = e 5 x ⋅ 15 x 2 = 15 x 2 e 5 x 5 7 –54
5 5 5 y′ = − =
27. y = 8e x ⇒ y ′ = 8e x ⋅ 5 x 4 = 40 x 4 e x 5 x + 2 7 x – 8 (5 x + 2)(7 x – 8)
28. f (x) = 0.42x ⇒ ln f (x) = 2x ln 0.4 ⇒ 39. Two solution methods are possible.
1 Differentiate directly:
f ′( x ) = 2 ln 0.4 ⇒ f ′( x ) = 0.4 2 x ⋅ 2 ln 0.4 y = ln [(4x − 7)(x + 10)]
f ( x) 1
y′ = ⋅ [ 4( x + 10) + ( 4 x − 7) ⋅ 1]
29. f (x) = 10− 0.2 x ⇒ ln f (x) = −0.2x ln 10 ⇒ ( 4 x – 7)( x + 10)
1
f ′( x ) = −0.2 ln 10 ⇒ 8 x + 33
=
f ( x) ( 4 x – 7)( x + 10)
f ′( x ) = 10 −0.2 x ( −0.2 ln 10) Or simplify using properties of logarithms first:
30. g (x) = 4(7x) ⇒ ln g (x) = ln 4 + x ln 7 ⇒ y = ln (4x − 7) + ln (x + 10)
1 4 1 8 x + 33
g ′( x ) = ln 7 ⇒ g ′( x ) = 4(7 x ) ln 7 y′ = + =
g( x ) 4 x − 7 x + 10 ( 4 x – 7)( x + 10)
31. h (x) = 1000(1.03x) ⇒ ln h (x) = ln 1000 + 40. y = (2x + 5)3 4 x − 1 ⇒
1 1
x ln 1.03 ⇒ h ′( x ) = ln 1.03 ⇒ ln y = 3 ln (2x + 5) + ln ( 4 x − 1) ⇒
h( x ) 2
h ′( x ) = 1000(1.03x) ln 1.03 1 6 2
y′ = + ⇒
y 2x + 5 4x −1
32. c (x) = x5 · 3x ⇒ ln c (x) = 5 ln x + x ln 3 ⇒
y′ = 
1 6 2 
c ′( x ) = 5/ x + ln 3 ⇒ + [(2 x + 5)3 4 x − 1 ]
c( x )  2 x + 5 4 x − 1
c ′( x ) = x5 · 3x(5/x + ln 3) (28 x + 4)(2 x + 5)2
=
33. m(x) = 5x · x 7 ⇒ ln m(x) = x ln 5 + 7 ln x ⇒ 4x −1
1 (10 + 3 x )10
m ′( x ) = ln 5 + 7/ x ⇒ 41. y = ⇒ ln y = 10 ln (10 + 3 x ) −
m( x ) ( 4 − 5 x )3
m′ (x) = 5x · x7(ln 5 + 7/x) 1
3 ln ( 4 − 5 x ) ⇒ y ′ =
30
+
15

34. y = (ln x)0.7 x ⇒ ln y = 0.7x ln (ln x) ⇒ y 10 + 3 x 4 − 5 x
15  (10 + 3 x )10
y′ = 
1 1 30
y ′ = 0.7 ln (ln x ) + 0.7 x ⋅ ⇒ +
y x ln x  10 + 3 x 4 − 5 x  ( 4 − 5 x )3
 0.7  (270 − 105 x )(10 + 3 x )9
y ′ = 0.7 ln (ln x ) + ⋅ (ln x ) 0.7 x =
 ln x  (4 − 5x )4

124 Problem Set 6-4 Calculus Solutions Manual


© 2005 Key Curriculum Press
d  x
 Step 4: Write division as multiplication by the
 ∫ 10 dt  = 10 x
t
42.
dx  3  reciprocal; distribute division over addition.
Step 6: 1/x does not depend on h, so it is a
d  x

43. 
dx  ∫ 3
ln t dt  = ln x

“constant” with respect to h.
Step 7: Logarithm of a power, applied in reverse.
d  4x

44. 
dx  ∫ 5
log 2 t dt  = 4 log 2 ( 4 x )

Step 9: The expression in parentheses has the form
(1 + n)1/n, whose limit is e as n approaches zero.
d  x2  63. Answers will vary.
45. 
dx  ∫ 6.3
ln (cos t ) dt  = 2 x (ln cos x 2 )

64. Answers will vary.

d2 d2 d  5 5
46. (ln x 5
) = 2 (5 ln x ) = =− 2 Problem Set 6-5
dx 2
dx dx  x  x
n
Q2. e = lim 1 +  or
d 2 7x d 1
Q1. e ≈ 2.71828
47. 2 (e ) = (7e 7 x ) = 49e 7 x n→∞  n
dx dx
e = lim(1 + n)1/ n

1
48. e 5 x dx = e 5 x + C n→0
5 Q3. 1 Q4. x


1
49. e 7 x dx = e 7 x + C Q5. x Q6. e
7 Q7. (ln x)/(ln b) Q8. ex
72 x −e + C
−x
50. ∫ 72 x dx =
2 ln 7
+C Q9.
2 sin 5 x 0
Q10. E

1. lim →
1.05 x

x→0 3x 0
51. 1.05 x dx = +C
ln 1.05 10 cos 5 x 10
= lim =
∫ 6e dx = 6e + C
x x x→0 3 3
52.
y

∫ e dx = 5e + C
0.2 x 0.2 x
53.

∫ e cos x dx = e + C
sin x sin x
54. 1
1 x

∫ e sec x dx = e + C
tan x 2 tan x
55.

∫ e dx = ∫ x dx = 4 x + C
3 ln x 1 3 4
56. 4 tan 3 x 0
2. lim →
x→0 5x 0
∫ 60e dx = 60∫ 5x dx = 150 x + C
ln 5 x 2
57. 2
12 sec 3 x 12
= lim =
x→0
∫ (1 + e ) e dx = 102 (1 + e ) + C
1 2 x 50 2 x 2 x 51 5 5
58.
y

∫ (1 − e ) e dx = − 404 (1 − e ) + C
1
4 x 100 4 x 4 x 101
59.
1 1 x
2 2

∫ (e
−x −x
60. x
− e ) dx = e + e x
0 0
−2
= e + e − 1 − 1 = 5.524391...
2

Numerically: integral ≈ 5.524391... (Checks.) tan x 0


2
3. lim →
2 x→0 x 0
61. ∫−1
(e x + e – x ) dx = (e x − e – x )
−1 sec 2 x
= lim =1
= e2 − e−2 − e−1 + e1 = 9.604123… x→0 1
Numerically: integral ≈ 9.604123... (Checks.) sin x 0
4. lim →
62. Step 2: Definition of derivative. x→0 x 0
Step 3: Logarithm of a quotient, applied in cos x
= lim = 1, a “well-known” limit.
reverse. x→0 1

Calculus Solutions Manual Problem Set 6-5 125


© 2005 Key Curriculum Press
1 − cos x 0 x3 ∞

x →
5. lim 16. lim
x→0 x2 0 x →∞ e ∞
sin x 0 3x 2 ∞
= lim → = lim x →
x→0 2 x 0 x →∞ e ∞
6x ∞
= lim
cos x 1
= = lim x →
x→0 2 2
x →∞ e ∞
= lim x = 0  Form: .
6 6
x2 0  ∞
6. lim → x →∞ e
x→0 cos 3 x − 1 0
3 x + 17 3 3
2x 0 17. lim = lim =
= lim → x →∞4 x – 11 x→∞ 4 4
x →0 −3 sin 3 x 0
2 – 7x –7 7
2 2 18. lim = lim =−
x →∞ 3 + 5 x
= lim =− x →∞ 5 5
x→0 −9 cos 3 x 9
x 3 – 5 x 2 + 13 x – 21 ∞
sin x 0 19. lim →
7. lim+ 2 → x →∞ 4 x 3 + 9 x 2 – 11x – 17 ∞
x→0 x 0
3 x 2 – 10 x + 13 ∞
cos x = lim →
= lim+ =∞ x →∞ 12 x + 18 x – 11
2

x→0 2x
1 − cos x 0 6 x – 10 ∞
= lim →
8. lim 2 → x →∞24 x + 18 ∞
x→0 x + x 0
6 1
sin x = lim =
= lim =0 x →∞ 24 4
x→0 1 + 2 x

−∞ 3x 5 + 2 ∞
9. lim+
ln x
→ 20. lim →
x →0 1/ x ∞
5
x →∞ 7 x – 8 ∞
15 x 4 15 3
x −1 = lim 4 = lim =
= lim+ = lim ( − x ) = 0 x →∞ 35 x x →∞ 35 7
x→0 − x −2 x→0 +
21. L = lim+ x x → 0 0
e3 x x→0
10. lim 2 = ∞  Form is .
1
ln x −∞
x→0 x  0 ln L = lim+ ( x ln x ) = lim+ –1 →
x→0 x→0 x ∞
ex − e 0 −1
x
11. lim → = lim+ = lim ( − x ) = 0
x →1 5 ln x 0 x→0 − x −2 x→0 +
ex e ∴ L = e0 = 1
= lim −1 =
x →1 5 x 5
22. L = lim+ (sin x )sin x → 0 0
ln x − x + 1 0 x→0
12. lim 2 → ln sin x −∞
x →1 x − 2 x + 1 0 ln L = lim+ sin x (ln sin x ) = lim+ →
x −1 − 1 0 x→0 x→0
csc x ∞
= lim → 1/(sin x ) ⋅ cos x 1
x →1 2 x − 2 0 = lim+ = lim+
− x −2 1 x→0 − csc x cot x x →0 − csc x
= lim =−
x →1 2 2 = lim+ ( − sin x ) = 0
x→0
3x + 5 11
13. lim = = −26.43297K ∴ L = e0 = 1
x →2 cos x cos 2
23. L = lim − (sin x ) tan x → 1∞
x →π / 2
= ∞  Form is
tan x tan 2 
14. lim
x →2 x−2  0 
. ln L = lim − tan x (ln sin x )
x →π / 2
ln sin x 0
e ∞ x
= lim − →
15. lim 2 → x →π /2 cot x 0
x →∞ x ∞
(1/sin x ) ⋅ cos x
ex ∞ = lim −
= lim → x →π /2 − csc 2 x
x →∞ 2 x ∞
ex − cos x sin 2 x
= lim =∞ = lim −
x →π / 2 sin x
x →∞ 2

126 Problem Set 6-5 Calculus Solutions Manual


© 2005 Key Curriculum Press
= lim − ( − cos x sin x ) = 0 cos x − 1 0
x →π / 2 = lim →
x→0 x cos x + sin x 0
∴ L = e0 = 1
− sin x
= lim =0
24. L = lim+ x 1/( x −1) → 1∞ x →0 − x sin x + 2 cos x
x →1
ln L = lim+ [1/( x − 1) ⋅ ln x ] π π
x →1 31. f ( x ) = sec 2 x − tan 2 x. Where secant and
2 2
ln x 0 1/ x
= lim+ → = lim+ =1 tangent are defined, the Pythagorean properties
x →1 x −1 0 x→1 1 tell us that f (x) = 1.
∴ L = e1 = e
f (x )
25. L = lim(1 + ax )1/ x → ∞ 0 ( Note: a ≥ 0.)
x →∞
ln (1 + ax ) ∞ 1
ln L = lim [1/ x ⋅ ln (1 + ax )] = lim →

x
x →∞ x →∞ x –1 1 3 5
1/(1 + ax ) ⋅ a a
= lim = lim =0
x →∞ 1 x →∞ 1 + ax

∴ L = e0 = 1
26. L = lim (1 + ax )1/ x → 1∞ 32. Using l’Hospital’s rule leads to
x→0 sec x sec x tan x
ln (1 + ax ) 0 lim = lim
ln L = lim [1/ x ⋅ ln (1 + ax )] = lim → x →π /2 tan x x →π /2 sec 2 x
x→0 x→0 x 0
1/(1 + ax ) ⋅ a tan x sec 2 x
= lim = lim
a
=a = lim = lim
x →π /2 sec x x →π /2 sec x tan x
x→0 1 x →0 1 + ax

∴ L = ea sec x
= lim , the original expression!
x →π /2 tan x
27. L = lim+ x 3/(ln x ) → 0 0
x→0
Using tan x = (sec x)/(csc x), the expression
ln L = lim+ [3/(ln x ) ⋅ ln x ] = lim+ 3 = 3
x→0 x→0 reduces to
∴ L = e = 20.08553...
3 sec x
lim = lim csc x = 1
x →π /2 (sec x )/(csc x ) x →π /2
28. L = lim+ (7 x )5/(ln x ) → 0 0
x→0
33. L = lim+ x k /(ln x ) → 0 0
ln L = lim+ [5/(ln x ) ⋅ ln(7 x )] x→0
x→0
5 ln (7 x ) −∞ ln L = lim+ [k/(ln x ) ⋅ ln x ] = lim+ k = k
= lim+ → x→0 x→0
x→0 ln x −∞ ∴ L = ek
5 ⋅ [1/(7 x )] ⋅ 7 The graph turns out to be a horizontal line,
= lim+ =5
x→0 1/ x y = ek, defined for x > 0.
∴ L = e 5 = 148.4131...
y
y = ek
29. lim  − x  → ∞ − ∞
1 1
x→0  x e − 1
e –1− x
x
0
= lim x → x
x →0 x (e – 1) 0
ex – 1 0
= lim x →
x →0 1(e – 1) + x ⋅ e
x
0
By the definition of a power,
ex – 1 0
= lim x → x k /( ln x ) = ( x k )1/ln x = (e k ln x )1/ln x = e k
x →0 e – 1 + xe
x
0
ex 1 g( x ) 0.3 x 2 – 2.7
= lim x x =
34. a. f ( x ) = =
x →0 e + e + xe
x
2 h( x ) 0.2 x 2 – 2 x + 4.2
g(3) = 0.3(9) − 2.7 = 0,
1 1 
30. lim  −  →∞−∞ h(3) = 0.2(9) − 2(3) + 4.2 = 0, Q .E.D .
x→0  x sin x 
b. g′ (x) = 0.6x ⇒ g′ (3) = 1.8
sin x − x 0 h′ (x) = 0.4x − 2 ⇒ h′ (3) = −0.8
= lim →
x→0 x sin x 0

Calculus Solutions Manual Problem Set 6-5 127


© 2005 Key Curriculum Press
Tangent lines at (3, 0) have these equations. c.
For g: y1 = 1.8(x − 3) m(t),
For h: y2 = −0.8(x − 3) t m(t), Annual Continuous Difference
y 1.8( x – 3)
c. 1 = = −2.25, for x ≠ 3. 5 1,338.23 1,349.86 11.63
y2 –0.8( x – 3) 20 3,207.14 3,320.12 112.98
g′(3) 1.8 50 18,420.15 20,085.54 1,665.38
= = −2.25, which equals y1/y2,
h′(3) –0.8
d. For 7% interest, compounded continuously,
Q .E .D .
m (t) = 1000e0.0 7t .
d. Because the ratio g (x)/h(x) approaches the 36. a. f (x) = x n, g (x) = ln x, h (x) = ex
ratio y1/y2 as x approaches 3, and because
f ( x) xn ∞
y1/y2 equals g′ (3)/h′ (3) for all x ≠ 3, the ratio lim = lim →
g (x)/h(x) also approaches g′ (3)/h′ (3) as x
x →∞ g( x ) x →∞ ln x ∞
approaches 3. This is what l’Hospital’s rule nx n –1
= lim = lim nx n = ∞, if n > 0
concludes. x →∞ 1/ x x →∞

If g (3) or h (3) were any number other than 0, ∴ a power function is higher-order than the
the canceling of the (x − 3)’s in part c could natural log function.
not be done, and the ratio would almost
f ( x) xn ∞
certainly not equal 1.8/(−0.8). lim = lim x →
x →∞ h( x ) x →∞ e ∞
e. The graph shows a removable discontinuity at
(3, −2.25): nx n –1 ∞
= lim → , if n − 1 > 0
x →∞ e
x

f (x )
Eventually, the exponent of the power will
become zero, in which case the limit takes the
1 x
form constant/∞, which is 0.
3 ∴ a power function is lower-order than an
exponential function.
Using “<” to represent “is lower-order than,”
natural log < power < exponential.
35. a. For yearly compounding, m(t) = ln 3 x x 100
1000(1 + 0.06)t. For semiannual compound- b. i. lim 5 = 0 ii. lim 0.01x = 0
x →∞ x x →∞ e
ing, m ( t) = 1000(1 + 0.06/2)2t because there
e 0.3 x
are two compounding periods per year, each iii. lim =∞
x →∞ 100 ln x
of which gets half the interest rate.
b. m(t) = 1000(1 + 0.06/n)nt x 1
iv. lim = lim =0
x →∞ x x →∞ x
lim m(t ) = lim 1000(1 + 0.06/n) nt
n→∞ n→∞
ex
0.2 x = lim e =∞
0.8 x
= 1000 lim (1 + 0.06/n) nt v. lim
x →∞ e x →∞
n→∞
37. Answers will vary.
Let L = lim (1 + 0.06/n) nt .
n→∞

ln L = lim [nt ln (1 + 0.06/n)] Problem Set 6-6


n→∞

ln (1 + 0.06/n) 0 1. y = ln (3x + 4) ⇒ y′ = 3/(3x + 4)


= lim → 2. y = ln (3x 5) = ln 3 + 5 ln x ⇒ y′ = 5/x
n→∞ 1/(nt ) 0
3. y = ln (e 3x) = 3x ⇒ y′ = 3
1/(1 + 0.06/n) ⋅ ( −0.06n −2 )
= lim 4. y = ln (sin 4x) ⇒ y′ =
4 cos 4 x
= 4 cot 4x
n→∞ − n −2 /t sin 4 x
= lim
0.06t
= 0.06t 5. y = ln (cos5 x) = 5 ln (cos x) ⇒
n→∞ (1 + 0.06/n) −5 sin x
y′ = = −5 tan x
∴ L = e 0.06 t ⇒ lim m(t ) = 1000e 0.06 t cos x
n→∞ 6. y = ln (e5) = 5 ⇒
When interest is compounded continuously, y′ = 0 (Derivative of a constant!)
m(t) = 1000e0. 06t .

128 Problem Set 6-6 Calculus Solutions Manual


© 2005 Key Curriculum Press
7. y = ln [cos (tan x)] ⇒ 29. y = xx = (eln x)x = ex ln x ⇒
y′ =
– sin (tan x )
⋅ sec2 x = −tan (tan x) sec2 x y′ = xx (ln x + 1) (See Problem 28.)
cos (tan x ) 30. y = x ln x − x ⇒ y′ = ln x + x · (1/x) − 1 = ln x
1
8. y = ln x 2 – 2 x + 3 = ln ( x 2 − 2 x + 3) ⇒ (Note: This answer reveals that the integral of
2 ln x is x ln x − x.)
 1 2x − 2 x −1
y′ = = 31. y = ex(x − 1) ⇒ y′ = ex(x − 1) + ex · 1 = xex
 2 x2 − 2x + 3 x2 − 2x + 3
1 1
9. y = cos (ln x) ⇒ y′ = −(1/x) sin (ln x) 32. y = (e x + e – x ) ⇒ y′ = (e x – e – x )
2 2
10. y = sin x · ln x ⇒ y′ = cos x · ln x + (1/x) sin x
1 1
11. y = e7x ⇒ y′ = 7e7x 33. y = (e x – e – x ) ⇒ y′ = (e x + e – x )
3 3 2 2
12. y = e x ⇒ y′ = 3 x 2 e x
5
(Problems 32 and 33 are the hyperbolic cosine
13. y = e 5 ln x = e ln x = x 5 ⇒ y ′ = 5 x 4 and sine functions, respectively. See Chapter 8.)
14. y = ecos x ⇒ ex
y′ = ecos x · (−sin x) = −ecos x sin x 34. y = ⇒
1+ ex
15. y = cos (ex) ⇒ y′ −sin (ex) · ex = −ex sin ex
e x ⋅ (1 + e x ) – e x ⋅ (e x ) ex
16. y = (cos3 x)(e3x) ⇒ y′ = =
y′ = 3 cos2 x (−sin x) · e3x + cos3 x · e3x · 3 (1 + e x )2 (1 + e x )2
= −3e3x cos2 x sin x + 3e3x cos3 x 35. y = 5x ⇒ ln y = x ln 5 ⇒ (1/y)y′ = ln 5 ⇒
= 3e3x cos2 x (−sin x + cos x) (Factoring y′ = y ln 5 = 5x ln 5
optional) ln x 1/ x 1
5
17. y = e x ⇒ y′ = 5 x 4 e x
5
36. y = log5 x = ⇒ y′ = =
x x
ln 5 ln 5 x ln 5
18. y = e e ⇒ y′ = e e ⋅ e x ln x
19. sin y = ex ⇒ cos y · y′ = ex ⇒ 37. y = x − 7 log2 x = x − 7 · ⇒
ln 2
ex ex 1 
y′ = = –7 x –8 ⋅ ln x + x –7 ⋅ 
(See sketch.) 1
y′ =
cos y 1 – e2 x ln 2  x
–8
x
1 = (–7 ln x + 1)
sin y = e x ln 2
y
38. y = 2− x cos x ⇒
y′ = 2− x (−ln 2) · cos x + 2− x (−sin x)
(Showing cos y = 1 – e 2 x )
= −2− x(ln 2 · cos x + sin x) (Factoring optional)
20. y = e x · ln x ⇒ 39. y = e− 2x ln 5x ⇒
y′ = ex · ln x + ex · (1/x) = ex (ln x + 1/x) y′ = −2e− 2x · ln 5x + e− 2x · (1/x)
x = e− 2x (−2 ln 5x + 1/x)
21. y = ∫ 1/t dt ⇒ y′ = 1/x
1
7x 1
22. tan y = e ⇒ sec y · y′ = e ⇒
x 2 x 40. y = ⇒ y′ = ⋅ 7 x ln 7 = 7 x
ln 7 ln 7
ex ex
y′ = = (See sketch.) log 3 x 1
sec y 1 + e 2 x
2
41. y = = log e x = ln x ⇒ y ′ =
log 3 e x
√1 + e 2x log10 x 1
y
ex
42. y = = log e x = ln x ⇒ y ′ =
log10 e x
1
ln x
23. y = ln (e ln x) = ln x ⇒ y′ = 1/x 43. y = (log 8 x )(ln 8) = ⋅ ln 8 = ln x ⇒
24. y = 2x ⇒ ln y = x ln 2 ⇒ (1/y)y′ = ln 2 ⇒ ln 8
y′ = y ln 2 = 2x ln 2 1
y′ =
x x
25. y = e x ln 2 = e ln 2 = 2 x ⇒
y′ = 2x ln 2 (See Problem 24.) 44. y = (log4 x)10 ⇒
2 1 10(log 4 x )9
26. y = e2 ln x = e ln x = x 2 ⇒ y′ = 2x y′ = 10(log4 x)9 · =
x ln 4 x ln 4
27. y = x 2 ⇒ y′ = 2x
7 ln x 7
28. y = ex ln x (which equals xx) ⇒ 45. y = log5 x 7 = 7 log5 x = ⇒ y′ =
y′ = ex ln x [ln x + x (1/x)] = xx (ln x + 1) ln 5 x ln 5

Calculus Solutions Manual Problem Set 6-6 129


© 2005 Key Curriculum Press
46. y = tan ex ⇒ y′ = sec2 ex · ex = ex sec2 ex
47. y = esin x ⇒ y′ = esin x cos x
69. ∫ (3/x ) dx = 3 ln | x | + C
x 2
2

∫ 4 dx = ln 4 = ln 4 = 8.656170K
4 12
48. y = ln csc x ⇒
x
70.
1
1
y′ = (1/csc x) · (−csc x cot x) = −cot x

∫ (ln x ) x dx = 10 (ln x ) + C
1 1
49. y = 35 ⇒ y′ = 0 (Derivative of a constant!) 71. 9 10

50. y = ln (cos2 x + sin2 x) = ln 1 = 0 ⇒ y′ = 0


51. y = sin x ⇒ y′ = cos x 72. ∫ cos x dx = sin x + C
∫ e dx = ∫ x dx = 2 x + C
1 1
52. y = sin− 1 x ⇒ y′ = 73. ln x 2
2
1– x

∫ ln (e ) dx = ∫ 3x ln e dx = ∫ 3x dx = 2 x
3
53. y = csc x ⇒ y′ = −csc x cot x 74. 3x 2
+C
1
54. y = tan− 1 x ⇒ y′ =
1+ x2 75. ∫ 0 dx = C (Integral of zero is a constant.)
55. y = tan x ⇒ y′ = sec2 x
56. y = cot x ⇒ y′ = −csc 2 x
76. ∫ cos x sec x dx = ∫ 1 dx = x + C
∫ sec 2 x dx = 2 ∫ sec 2 x (2 dx )
1
∫e
1 4x 77.
57. 4x
dx = e +C
4
1
∫e dx = e 4 x + C = ln | sec 2 x + tan 2 x | + C
4
58. 2

∫ ∫
1
∫ x e dx = 4 ∫ e (4 x
3 x4 1 x4 1 x4 tan 3 x dx =
59. 3
dx ) = e +C 78.
3
tan 3 x (3 dx )
4
1
∫ cos x ⋅ e dx = ∫ e = ln | sec 3 x | + C
sin x sin x
60. (cos x dx )
3

∫ ∫
= e sin x + C 1
79. cot 4 x dx = cot 4 x ( 4 dx )
5 4
∫ ∫
(ln x ) 1 1
61. dx = (ln x )5 dx = (ln x )6 + C 1
x x 6 = ln | sin 4 x | + C
4
∫5 ∫
dx = e x ln 5 dx
x
62.
∫ ∫
1
80. csc 5 x dx = csc 5 x (5 dx )
5
= (1/ln 5) ∫ e x ln 5
ln 5 dx 1
= − ln | csc 5 x + cot 5 x | + C
5
1 x ln 5 5x
= e +C= +C 1 − cos x 0
ln 5 ln 5 81. lim →
x→0 x 0
5x
∫e ∫
dx = 5 x dx = +C sin x
x ln 5
63.
ln 5 = lim =0
x→0 1

(See Problem 62.) x 0


82. lim →
x →0 1 – cos x 0

1 x 1
64. (e + e – x ) dx = (e x – e – x ) + C
2 2 1
= lim = ∞ (Reciprocal of Problem 81.)
x1 x →0 sin x
65. ∫t
1
dt = ln x (By definition!)
83. lim
x
=
π /2
=
π
x →π / 2 1 – cos x 1 – cos (π /2) 2
∫e
−x −x
66. dx = − e +C
π
= ∞  Form: 
x
84. lim
x
1 + cos x
x →π  0
∫2
2
67. x
dx = +C
ln 2 5 x – sin 5 x 0
85. lim 3 →
x→0 x 0
3x
68. ∫ ( x −0.2 + 3 x ) dx = 1.25 x 0.8 +
ln 3
+C
= lim
5 – 5 cos 5 x
2 →
0
x→0 3x 0

130 Problem Set 6-6 Calculus Solutions Manual


© 2005 Key Curriculum Press
= lim
25 sin 5 x

0 3 – 3e 3 x 0
= lim 3x →
x→0 6x 0 x→0 3(e 3 x – 1) + 3 x ⋅ 3e 0
125 cos 5 x 125 –9 e 3 x
= lim = = 20.8333K 1
x→0 6 6 = lim 3x = −
x→0 9e 3 x + 9e + 27 xe
3x
2
86. lim(1 + 0.03/ x ) x → 1∞
x →∞

Let L = lim(1 + 0.03/ x ) x. Problem Set 6-7


x →∞
Review Problems
ln L = ln lim (1 + 0.03/ x ) x = lim ln(1 + 0.03/ x ) x
x →∞ x →∞
R0. Answers will vary.
= lim [ x ln (1 + 0.03/ x )]
x →∞
R1. a. dM/dt = 0.06M ⇒ M − 1 dM = 0.06 dt
ln (1 + 0.03 x –1 ) 0 x 5
= lim
x →∞ x –1

0
∴ ∫
100
M −1 dM = ∫ 0.06 dt, Q.E.D.
0

1/(1 + 0.03/ x ) ⋅ (–0.03 x –2 ) 5

∫ 0.06 dt = 0.06t
5
= lim = 0.03 = 0.3
x →∞ – x –2 0 0

∴ L = e0.03 = 1.03045… b. Solving numerically for x in


x
87. lim (1 + 0.03 x ) →∞
∫ M −1 dM = 0.3
1/ x 0
x →∞ 100

Let L = lim(1 + 0.03 x )1/ x . gives x ≈ 134.9858… .


x →∞
c. There will be $134.99 in the account, so the
ln L = lim [(1/ x ) ln (1 + 0.03 x )] → 0 ⋅ ∞ interest will be $34.99.
x →∞

ln (1 + 0.03 x ) ∞ R2. a. Integrating x− 1 by the power rule results in


= lim → x –1+1
x →∞ x ∞ division by zero: + C.
1/(1 + 0.03 x ) ⋅ 0.03 −1 + 1
= lim =0 x
x →∞

∴ L = e0 = 1
1 b. If g( x ) = ∫a
f (t ) dt and f (x) is continuous in
a neighborhood of a, then g′ (x) = f (x).
2x ∞ x1
88. lim 2 →
x →∞ x ∞ ln x =
1 t
dt ∫
x
2 ln 2 ∞ d  x1  1

d
= lim → (ln x ) =  dt  =
x →∞ 2x ∞ dx dx  1 t  x
x 2
2 (ln 2) c. i. y = (ln 5x)3 ⇒ y′ = (3/x)(ln 5x)2
= lim =∞
x →∞ 2 ii. f (x) = ln x 9 = 9 ln x ⇒ f ′ (x) = 9/x
2x iii. y = csc (ln x) ⇒
or: lim 2 = ∞ by (exponential)/(power)
x →∞ x y′ = −csc (ln x) cot (ln x) · (1/x)
89. lim (0.5 x )3/( 2− x ) → 1∞ x2
x →2
Let L = ln (0.5x)3/(2 − x) .
iv. g( x ) = ∫ 1
csc t dt ⇒ g ′( x ) = 2 x csc x 2

∫ ∫ sec x sec x tan x dx


sec x tan x 1
ln L = lim  ⋅ ln 0.5 x  → ∞ ⋅ 0
3 d. i. dx =
x →2  2 – x  sec x
= ln | sec x | + C
3 ln 0.5 x 0
= lim → –3 10


x →22–x 0 –3
ii. dx = 10 ln | x | –2
3/(0.5 x ) ⋅ 0.5 3 x –2
= lim =− = 10 ln | –3 | −10 ln | −2 |
x →2 –1 2
= 10(ln 3 − ln 2) = 4.054651…
∴ L = e− 3/2 = 0.22313…
∫ ∫
1
iii. x 2 ( x 3 − 4) −1 dx = ( x 3 − 4) −1 (3 x 2 dx )
90. lim  3 x –  ⇒∞−∞
1 1 3
x→0  e – 1 3x  1
= ln | x 3 – 4 | + C
3 x – (e 3 x – 1) 0 3
= lim 3x →
x→0 3 x (e – 1) 0

Calculus Solutions Manual Problem Set 6-7 131


© 2005 Key Curriculum Press
e. By finding areas, h (1) = −2.5, h (2) = 0, ii.
h (6) = 2.7, h (10) = 0, and h (11) = −2.5. y

y y = h (x )
4 10

x or t x
1 2 6 11
–2

y = f (t )

–4 iii.
y
f. i. y (100) ≈ 70 names; 70% remembered
y(1) = 1 name; 100% remembered 1
x
101
ii. y′ = 5
100 + x
y′ (100) = 101/(200) = 0.505 names/person
y′ (1) = 101/101 = 1 name/person
iii. Paula has probably not forgotten any
names as long as x − y < 0.5. After b. i. f (x) = x1.4 e5x ⇒
meeting 11 people, she remembers about f ′ (x) = 1.4x0.4 e5x + 5x1.4 e5x
10.53… ≈ 11 names, but after meeting ii. g (x) = sin (e− 2x) ⇒ g′ (x) = −2e− 2x cos(e− 2x)
12 people, she remembers about 11.44… ≈ d ln x d
11 names. iii. (e ) = ( x) = 1
dx dx
R3. a. i. See the text for the definition of logarithm. iv. y = 100x ⇒ y′ = (ln 100)100 x
ii. See the text for the definition of ln x. v. f (x) = 3.7 · 100.2 x ⇒
iii. See the text for the statement of the f ′ (x) = 0.74 ln 10 · 100.2 x
uniqueness theorem.
v i . r(t) = t tan t ⇒ ln r = tan t ln t ⇒
iv. See the text for the proof.
1 tan t
v. See the solution to Problem 10 in r ′ = sec 2 t ln t + ⇒
r t
Lesson 6-3.
r ′ = t tan t  sec 2 t ln t +
tan t 
b. i. e = lim (1 + n)1/ n or e = lim (1 + 1/n) n 
n→0 n→∞ t 
ln x c. y = (5x − 7)3 (3x + 1)5 ⇒
ii. log b x =
ln b ln y = 3 ln (5x − 7) + 5 ln (3x + 1) ⇒
1 15 15
ln x 1 y′ = + ⇒
c. i. y = log 4 x = ⇒ y′ = y 5x − 7 3x + 1
ln 4 x ln 4
y′ = 
15 15 
+ (5 x − 7)3 (3 x + 1)5
ii. f ( x ) = log 2 (cos x ) =
ln (cos x )
⇒  5 x − 7 3 x + 1
ln 2 = (120x − 90)(5x − 7)2 (3x + 1)4

∫ 10e
1 −2 x
dx = −5e −2 x + C
f ′( x ) = ⋅ ( − sin x ) d. i.
(cos x )(ln 2)

∫e sin x dx = − e cos x + C
cos x
tan x ii.
=−
ln 2 2
2
iii. y = log 5 9 x = x log 5 9 ⇒ y ′ = log 5 9 iii. ∫
−2
e −0.1x dx = −10e −0.1x
−2
R4. a. i. −0.2
= −10e + 10e 0.2
= 4.02672 K
y
0.2x

∫ 10
10
iv. 0.2x
dx = +C
10 0.2 ln 10
e. i. The exposure is the product of C (t) and t,
x where C (t) varies. Thus, a definite integral
2 must be used.

132 Problem Set 6-7 Calculus Solutions Manual


© 2005 Key Curriculum Press
x

∫ 150e 2x2 − 3 ∞
−0.16 t
ii. E( x ) = dt = 937.5( − e −0.16 x + 1) R5. a. lim 2 →
x →∞ 7 − 5 x −∞
0
E(5) = 937.5(−e− 0.8 + 1) =
4x 2
516.25… ppm · days E (10) = = lim =−
x →∞ −10 x 5
937.5(−e− 1.6 + 1) = 748.22… ppm · days
As x grows very large, E (x) seems to x − cos x + 1
2
0
b. lim →
approach 937.5. x→0 e − x −1
x
0
iii. E ′ (x) = 150e− 0.16 x = C (x) 2 x + sin x 0
= lim →
E ′ (5) = 67.39… ppm (or ppm · days x→0 ex − 1 0
per day) 2 + cos x 2 + 1
E′ (10) = 30.28… ppm = lim = =3
x→0 ex 1
f. i. From Figure 6-7d, the maximum
c. lim x 3e − x → ∞ ⋅ 0
concentration is about 150 ppm at about x →∞
2 hours. (These values can be found more x3 ∞
precisely by setting the numerical or = lim x →
x →∞ e ∞
algebraic derivative equal to zero, solving
to get t = −1/ln 0.6 = 1.9576… . Then 3x 2

= lim x →
C (1.9576…) = −200/(e ln 0.6) = x →∞ e ∞
144.0332… .) 6x ∞
= lim x →
ii. C (t) = 200t · 0.6t x →∞ e ∞
C′ (t) = 200t · 0.6t ln 0.6 + 200 · 0.6t 6
= lim x = 0 (Form: 6/∞)
x →∞ e
C′ (1) = 200 · 0.61(ln 0.6 + 1) = 58.70…
C′(5) = 200 · 0.65(5 ln 0.6 + 1) d. L = lim x tan (πx/2 ) → 1∞
x →1
= −24.16… < 0 ln L = lim [tan (π x/2) ⋅ ln x ]
x →1
C(t) is increasing at about 58.7 ppm/h
when t = 1 and decreasing at about ln x 0
= lim →
24.2 ppm/h when t = 5. The concentration x →1 cot (π x/2) 0
is increasing if C′ (t) is positive and 1/ x 1 −2
decreasing if it is negative. = lim = =
x →1 −(π /2)csc π x/2 −π /2 π
2

iii. Solving 50 = 200t · 0.6t numerically for t


∴ L = e− 2/ π = 0.529077…
gives t ≈ 0.2899… and t ≈ 6.3245… .
So C(t) > 50 for 6.3245… − 0.2899… = e. lim 3 x 4 = 48
x →2
6.03… , or about 6 hours.
f. lim (tan 2 x − sec 2 x ) = lim (1) = 1
iv. C 1(t) = 200t · 0.3t x →π /2 x →π /2

C (t ) g. Examples of indeterminate forms:


0/0, ∞/∞, 0 · ∞, 00, 1∞ , ∞ 0, ∞ − ∞
R6. a. i. y = ln (sin4 7x) = 4 ln sin 7x ⇒
100

y′ = 4(1/sin 7x) · cos 7x · 7 = 28 cot 7x


50
t

1 ii. y = x − 3e2x ⇒
y′ = −3x − 4 · e2x + x− 3 · 2e2x
From the graph, the maximum is about = x − 4e2x (2x − 3)
60 ppm around t = 1. (Exactly, t = −1/ln 0.3 =
0.8305… , for which C (0.8305…) = iii. y = cos (2x) ⇒ y′ = −sin (2x) · 2x ln 2
4 ln x 4
−200/(e ln 0.3) = 61.11092… ≈ 61.1 ppm.) iv. y = log 3 x 4 = ⇒ y′ =
Repeating the computations of part iii ln 3 x ln 3
gives C (t) > 50 for 0.409… < t < 1.473… ,
∫e
−1.7 x
or for about 1.06 hours. b. i. dx = ( −1 / 1.7) e −1.7 x + C
In conclusion, the concentration peaks
∫2
sec x
sooner at a lower concentration and stays ii. sec x tan x dx
above 50 ppm for a much shorter time. = ∫e ln 2 sec x
sec x tan x dx
ln 0.5 = −0.025t

Calculus Solutions Manual Problem Set 6-7 133


© 2005 Key Curriculum Press
x

∫ 150e 2x2 − 3 ∞
−0.16 t
ii. E( x ) = dt = 937.5( − e −0.16 x + 1)
0 R5. a. lim 2 →
x →∞ 7 − 5 x −∞
E(5) = 937.5(−e− 0.8 + 1) =
4x 2
516.25… ppm · days E(10) = = lim =−
x →∞ −10 x 5
937.5(−e− 1.6 + 1) = 748.22… ppm · days
As x grows very large, E(x) seems to x − cos x + 1
2
0
b. lim →
approach 937.5. x→0 e − x −1
x
0
iii. E′ (x) = 150e− 0.16x = C (x) 2 x + sin x 0
= lim →
E′ (5) = 67.39… ppm (or ppm · days x→0 ex − 1 0
per day) 2 + cos x 2 + 1
E′ (10) = 30.28… ppm = lim = =3
x→0 ex 1
f. i. From Figure 6-7d, the maximum c. lim x 3e − x → ∞ ⋅ 0
concentration is about 150 ppm at about x →∞
2 hours. (These values can be found more x3 ∞
precisely by setting the numerical or = lim x →
x →∞ e ∞
algebraic derivative equal to zero, solving
to get t = −1/ln 0.6 = 1.9576… . Then 3x 2

= lim x →
C (1.9576…) = −200/(e ln 0.6) = x →∞ e ∞
144.0332… .) 6x ∞
= lim x →
ii. C (t) = 200t · 0.6t x →∞ e ∞
C′ (t) = 200t · 0.6t ln 0.6 + 200 · 0.6t 6
= lim x = 0 (Form: 6/∞)
x →∞ e
C′ (1) = 200 · 0.61(ln 0.6 + 1) = 58.70…
C′(5) = 200 · 0.65(5 ln 0.6 + 1) d. L = lim x tan (πx/2 ) → 1∞
x →1
= −24.16… < 0 ln L = lim [tan (π x/2) ⋅ ln x ]
x →1
C(t) is increasing at about 58.7 ppm/h
when t = 1 and decreasing at about ln x 0
= lim →
24.2 ppm/h when t = 5. The concentration x →1 cot (π x/2) 0
is increasing if C′ (t) is positive and 1/ x 1 −2
decreasing if it is negative. = lim = =
x →1 −(π /2)csc π x/2 −π /2 π
2

iii. Solving 50 = 200t · 0.6t numerically for t


∴ L = e− 2/ π = 0.529077…
gives t ≈ 0.2899… and t ≈ 6.3245… .
So C(t) > 50 for 6.3245… − 0.2899… = e. lim 3 x 4 = 48
x →2
6.03… , or about 6 hours.
lim (tan x − sec x ) = lim ( −1) = −1
2 2
iv. C1(t) = 200t · 0.3t f.
x → π /2 x → π /2

C (t )
g. Examples of indeterminate forms:
0/0, ∞/∞, 0 · ∞, 00, 1∞ , ∞ 0, ∞ − ∞
100
R6. a. i. y = ln (sin4 7x) = 4 ln sin 7x ⇒
50
t y′ = 4(1/sin 7x) · cos 7x · 7 = 28 cot 7x
1 ii. y = x− 3e2x ⇒
From the graph, the maximum is about y′ = −3x− 4 · e 2x + x− 3 · 2e2x
60 ppm around t = 1. (Exactly, t = −1/ln 0.3 = x− 4e2x (2x − 3)
= 0.8305… , for which C (0.8305…) = iii. y = cos (2x ) ⇒ y′ = −sin (2x ) · 2x ln 2
−200/(e ln 0.3) = 61.11092… ≈ 61.1 iv. y = log 3 x 4 =
4 ln x
⇒ y′ =
4
ppm.) ln 3 x ln 3
Repeating the computations of part iii
∫e
−1.7 x
gives C (t) > 50 for 0.409… < t < 1.473… b. i. dx = ( −1 / 1.7) e −1.7 x + C
, or for about 1.06 hours.
∫2
sec x
In conclusion, the concentration peaks ii. sec x tan x dx
sooner at a lower concentration and stays
= ∫e ln 2 sec x
sec x tan x dx
above 50 ppm for a much shorter time.
ln 0.5 = −0.025t

Calculus Solutions Manual Problem Set 6-7 133


© 2005 Key Curriculum Press
C4. a. Suppose there is a number M > 0 such that Chapter Test
ln x ≤ M for all x > 0. Let x = e M +1. Then x1
ln x = ln e M +1 = (M + 1) ln e = M + 1 > M. T1. ln x = ∫
1 t
dt
This contradicts ln x ≤ M for all x > 0. Thus n
T2. e = lim 1 +  or e = lim (1 + n)1/n
the supposition is false, and there can be no 1
such number M that is an upper bound for n→∞  n n→0

ln x, Q .E .D . x

b. If M were a lower bound for ln x, then −M


T3. If g (x) = ∫a
f (t ) dt and f ( t ) is continuous in a

would be an upper bound for ln (1/x), but neighborhood of a, then g′ (x) = f ( x ) .


part a shows no such number can exist. T4. If (1) f ′ ( x ) = g′ (x) for all x in the domain and
c. ln′ x = 1/x, which shows that ln is (2) f ( a) = g ( a) for some a in the domain, then
differentiable for all x > 0. Thus, ln is f ( x ) = g ( x ) for all x in the domain.
continuous for all x > 0 because T5. Prove that ln x = loge x for all x > 0.
differentiability implies continuity.
Proof:
d. Because ln is continuous for all x > 0, the
intermediate value theorem applies. Thus, if k Let f ( x ) = ln x, and g (x) = loge x.
is between ln a and ln b, there is a number c f ′ ( x ) = 1/x and
between a and b such that ln c = k. g′ (x) = (1/x) · loge e = (1/x) · 1 = 1/x
∴ f ′ ( x ) = g′ (x) for all x > 0
y = ln x f ( 1 ) = ln 1 = 0 and g (1) = loge 1 = 0
∴ f ( 1 ) = g (1)
ln b
∴ by the uniqueness theorem,
k f ( x ) = g (x) for all x > 0.
ln a
x ∴ ln x = loge x for all x > 0, Q .E .D .
a c b
T6. f ( x ) = ln (x3ex )
1
a. f ′( x ) = 3 x ⋅ (3 x 2 e x + x 3e x ) = 3/ x + 1
e. Part a shows k cannot be an upper bound for x e
ln, so there must be some b > 0 such that b. f ( x ) = 3 ln x + x ln e = 3 ln x + x ⇒
ln b > k. Similarly, part b shows k is not f ′ ( x ) = 3/x + 1 (Checks.)
a lower bound, so some a > 0 exists for
T7. y = e2x ln x3 = 3e2x ln x ⇒
which ln a < k. By part d there is some
number c between a and b such that ln c = k, y′ = 6e 2 x ⋅ ln x + 3e 2 x ⋅ (1 / x )
Q .E .D . = 3e 2 x (2 ln x + 1/x )
f. The domain of ln is the positive reals, and the T8. v = ln (cos 10x) ⇒
range is all reals; the domain of the inverse to v′ = 1/(cos 10x) · (−10 sin 10x) = −10 tan 10x
ln (i.e., exp) is the range of ln (i.e., all reals), T9. f ( x ) = (log2 4x)7 = [(ln 4x)/(ln 2)]7 ⇒
and the range of the inverse is the domain of f ′ ( x ) = 7[(ln 4x)/(ln 2)]6 · [(1/4x) · 4 · (1/ln 2)]
ln (i.e., positive reals). 7(log 2 4 x )6
=
4 x2 x ln 2
C5. a. g( x ) = ∫ x2
sin t dt = − ∫4
sin t dt ⇒
T10. t ( x ) = ln (cos2 x + sin2 x) = ln 1 = 0 ⇒
g′ (x) = −2x sin x2 t′ (x) = 0
tan x
b. g( x ) = ∫ x2
sin t dt
T11. p( x ) = ∫
1
ln x
e t sin t dt ⇒
4 tan x
= ∫ x 2
sin t dt + ∫ 4
sin t dt ⇒ p′ (x) = e ln x sin ln x · 1/x = sin ln x
g′ (x) = −2x sin x2 + sin (tan x) sec2 x
∫e
1 5x
T12. 5x
dx = e +C
v( x ) 5
c. g( x ) = ∫ f (t ) dt ⇒

u( x ) 1
T13. (ln x )6 ( dx/x ) = (ln x ) 7 + C
g′ (x) = f (v(x)) · v′ (x) − f ( u ( x ) ) · u′ (x) 7
C6. log cabin

1
(or log cabin + C, which equals “houseboat”) T14. sec 5 x dx = ln | sec 5 x + tan 5 x | + C
5

Calculus Solutions Manual Problem Set 6-7 135


© 2005 Key Curriculum Press
2 1 x2 But h ′ ( x ) = f ′ ( x ) − g′ (x), which equals 0 for

1
T15. 5 x dx = 5 = (25 – 1)
0 ln 5 0 ln 5 all values of x.
= 14.9120… ∴ h ′ (c) = 0
5 – 3x −∞
T16. lim → This result thus contradicts the mean value
x →∞ ln 4 x ∞
–3 theorem, Q.E.D.
= lim = lim(–3 x ) = −∞
x →∞ [1/( 4 x )] ⋅ 4 x →∞ T22. a. F ( x ) = 60e0.1 x ⇒ F ′ ( x ) = 6e0.1 x so
T17. L = lim − (tan x ) cot x → ∞ 0 F ′ ( 5 ) = 6e0.5 = 9.8923… lb/ft
x →π / 2
F ′ ( 1 0 ) = 6e = 16.3096… lb/ft
ln L = lim − [cot x ⋅ ln ( tan x )] → 0 ⋅ ∞
x →π / 2 b. Work equals force times displacement. But the
ln (tan x ) ∞ force varies at different displacements. Thus, a
= lim − →
x →π / 2 tan x ∞ definite integral has to be used.
(1 / tan x ) ⋅ sec 2 x c.
= lim − = lim − cot x = 0 F
x →π / 2 sec 2 x x →π / 2
∴L=e =1 0

100 (x, F )
T18. a.
y
5 dx x
4 g
5
3
2
1 t or x
dW = F dx = 60 e0.1 x dx
1 2 3 4 5 6 7 8 9 10 5

∫ 60e
-1
-2
W= 0.1 x
dx
f 0
-3 5
-4 = 600e 0.1x = 600(e 0.5 − 1)
-5 0
W ≈ 389.23 ft-lb
3 x −5
b. h( x ) = ∫2
f (t ) dt ⇒ h ′( x ) = f (3 x − 5) ⋅ 3, T23. Answers will vary.
h′(3) = f (9 − 5) ⋅ 3 = f ( 4) ⋅ 3 = 1 ⋅ 3 = 3
x1 1.8 1 Problem Set 6-8
T19. ln x = ∫1 t t
dt, so ln 1.8 =
dt ∫1
Cumulative Review, Chapters 1–6
M4 = 0.2 
1 1 1 1 
+ + + = 0.58664 K 1. f ( x ) = 2x
 1.1 1.3 1.5 1.7 
2 3.1 – 2 2.9
From calculator, ln 1.8 = 0.58778… . f ′(3) ≈ = 5.549618K
0.2
x2 x2
2. There are about 10.0 squares, each 20 units.
T20. g( x ) = ∫ sin t dt = − cos t 50


2
2
∴ g( x ) dx ≈ 200
= −cos x2 + cos 2 ⇒ g′(x) = 2x sin x2 10

d x
2 (Function is g( x ) = 2 + 0.1x + sin x, so exact
g ′( x ) =
dx 2 ∫
sin t dt = 2 x sin x 2
answer is 200.)
15

T21. Let h ( x ) = f ( x ) − g (x). 3. L = lim f ( x ) if and only if for any ε > 0, there is
x →c
Then h ( a) = f ( a) − g (a) = 0 and a δ > 0 such that if x is within δ units of c but
h ( b ) = f ( b ) − g (b) ≠ 0. not equal to c, f ( x ) is within ε units of L.
h( b ) – h( a )
∴ ≠0 4. Answers may vary.
b–a
By the mean value theorem, there is a number c f (x )
4
between a and b such that
x
h( b ) – h( a )
h ′( c ) =
3
.
b–a
∴ h′ (c) ≠ 0

136 Problem Set 6-8 Calculus Solutions Manual


© 2005 Key Curriculum Press
f ( x + h) – f ( x ) 12. Optional graph showing upper sum:
5. f ′ (x) = lim
h→0 h
y=x2
f ( x ) − f (c )
or f ′(c) = lim
x →c x−c
10
6. f (x) = x 3
( x + h)3 − x 3
f ′( x ) = lim x
h→0 h 1 4

x + 3 x 2 h + 3 xh 2 + h 3 – x 3
3
= lim 4
h→0
2
h
= lim(3 x + 3 xh + h 2 ) = 3 x 2 , Q.E.D.
∫1
x 2 dx
h→0
U 6 = 0.5(1.52 + 22 + 2.52 + 32 + 3.52 + 42)
7. f (x) = ⇒ f ′ (x) = 3x 2
x3 = 24.875
f ′ (5) = 3·52 = 75
13. M 10 = 20.9775
5.013 – 4.99 3
f ′ (5) ≈ = 75.0001 M 100 = 20.999775
0.02 Sums seem to be approaching 21.
5.0013 – 4.999 3
f ′ (5) ≈ = 75.000001 1
0.002 ∫
14. a. cos 5 x sin x dx = − cos 6 x + C
6
The symmetric differences are getting closer to
75 as ∆ x gets closer to zero. b. ∫ (1/x ) dx = ln | x | + C
8. f (x) = 3 x – 5 = (3 x − 5) 1/ 2

1
f ′ (x) = (3 x – 5) −1/ 2 ⋅ 3 = 1.5(3 x − 5) −1/ 2
c. ∫ tan x dx = ln | sec x | + C = − ln | cos x | + C
2
f ′ (7) = 1.5(21 − 5)−1/2 = 1.5/4 = 0.375 = 3/8 d. ∫ sec x dx = ln | sec x + tan x | + C
9. Line with slope of 3/8 is tangent to the graph at 1
∫ (3x − 5) dx = 3 ∫ (3x − 5) (3 dx )
1/ 2 1/ 2
x = 7. e.
f (x )
1 2 2
= ⋅ (3 x – 5)3/ 2 + C = (3 x – 5)3/ 2 + C
5
8
3 3 9
4 1 3 4 64 1
3

x
15. ∫1
x 2 dx =
3
x
1
= − = 21,
3 3
5 which agrees with the conjecture in Problem 13.
10. a. y = e 2x cos 3x ⇒ 16. The graph shows a tangent line at x = c parallel
y′ = 2 e2x cos 3x − 3e2x sin 3x to the secant line.
ln x f (x )
b. q( x ) = ⇒ q ′( x ) =
tan x
(tan x )/x − ln x sec x
2
1 ln x
2
= − 2 x
tan x x tan x sin x a c b
d2 x d
c. 2 (5 ) = [(ln 5)5 x ] = (ln 5)2 5 x
dx dx Statement:
11. For the function to be differentiable, If f is differentiable on (a, b) and continuous at
lim− ( ax 2 + 1) = lim+ ( − x 2 + 6 x + b) and x = a and x = b, then there is a number x = c in
x →2 x →2
lim 2 ax = lim+ ( −2 x + 6) . (a, b) such that f ′( x ) =
f (b) – f ( a)
.
x →2 − x →2
b–a
1 17. y = x9/7
4a + 1 = 8 + b and 4a = 2 ⇒ a = and b = −5
2 y7 = x9
y
7y6 y′ = 9x 8
9x8 9x8 9 9 9
y′ = 6 = = x 8−54 / 7 = x 2 / 7 = x 9/ 7−1
2 7y 7( x 9/ 7 )6 7 7 7
as from the derivative of a power formula
x
2

Calculus Solutions Manual Problem Set 6-8 137


© 2005 Key Curriculum Press
18. If x− 1 were the derivative of a power, then the 26. L = lim (1 + n)1/n → 1∞
power would have to be x0. But x0 = 1, so its n→0

ln L = lim  ln (1 + n) → ∞ ⋅ 0
derivative equals 0, not x− 1. Thus, x− 1 is not the 1
derivative of a power, Q.E.D. n→0  n 
tan x ln (1 + n) 0
19. f ( x ) = ∫1
cos 3t dt ⇒ = lim
n→0 n

0
f ′ (x) = cos (3 tan x) · sec2 x 1/(1 + n)
x = lim =1
∫ (1/t ) dt ⇒ f ′( x ) = 1/x, Q.E.D.
n→0 1
20. f ( x ) =
1 ∴ L = e1 = e, Q.E.D.
21. Prove ln xa = a ln x for any constant a and all dx dy
x > 0. 27. Know: = −30 ft/s, = 40 ft/s
dt dt
dz
Proof: Want: when x = 200 and y = 100
Let f (x) = ln xa and g (x) = a ln x. dt
1 1 a x2 + y2 = z2
Then f ′( x ) = a ⋅ ax a−1 = a ⋅ = and dx dy dz
x x x 2x + 2y = 2z
1 a dt dt dt
g ′( x ) = a ⋅ = .
x x When x = 200 and y = 100, z = 50, 000 = 100 5 .
∴ f ′ (x) = g′ (x) for all x > 0
f (1) = ln (1a) = ln 1 = 0 and g(1) = a ln 1 = 0 dz
2(200)( −30) + 2(100)( 40) = 2 ⋅ 100 5
∴ f (1) = g (1) dt
∴ f (x) = g (x) for all x > 0, and thus dz −20
ln xa = a ln x for all x ≥ 0, Q .E .D . = = −8.94427K ft/s
dt 5
22. x = 5 cos t, y = 3 sin t
The distance z is decreasing.
dy dy/dt 3 cos t 3
∴ = = = − cos t 5
dx dx/dt –5 sin t 5 28. ∫2
f (x ) dx ≈ (1/3)(0.5)(100 + 4 · 150 + 2 · 170 +
23. At t = 2, (x, y) = (5 cos 2, 3 sin 2) 4 · 185 + 2 · 190 + 4 · 220 + 300)
= (−2.08… , 2.72…). = (1/3)(0.5)(3340) = 556 23
dy 3
At t = 2, = − cot 2 = 0.2745… .
dx 5 29. Area of cross section = πy2
The graph shows that a line of slope 0.27… Because the end of the radius is on a line through
at point (−2.08… , 2.72…) is tangent to the the origin with slope r/h, y = (r/h)x.
curve. πr 2
∴ Area = π [(r/h) x ]2 = 2 x 2
y h
Area
3

x
5 (x, Area)

dx x
h

24. y = tan− 1 t dV = (Area) dx


dy 1 h πr 2
v= = = (1 + t 2 ) −1
h
dt 1 + t 2
dv 2t
∴V= ∫
0
( Area ) dx = ∫
0 h2
x 2 dx

a= = −1(1 + t 2 ) −2 ⋅ 2t = − πr 2 1 3 h 1 r 2 3 1
dt (1 + t 2 )2 = ⋅ x = π (h − 0 3 ) = πr 2 h, Q .E.D .
3x
e –1 0 h2 3 0 3 h2 3
25. lim → 30. Answers will vary.
x →0 sin 5 x 0
3e 3 x 3
= lim =
x →0 5 cos 5 x 5

138 Problem Set 6-8 Calculus Solutions Manual


© 2005 Key Curriculum Press
Chapter 7—The Calculus of Growth and Decay

Problem Set 7-1 Q9. Q10.


y y
1. D (0) = 500 3
D (10) = 895.4238482… x
x
D (20) = 1603.567736… 4
3

2. D ′ (t) = 500(ln 1.06)(1.06t ) $/yr


D ′ (0) = 29.13445406… 1. a. B = number of millions of bacteria;
D ′ (10) = 52.17536994… t = number of hours
D ′ (20) = 93.43814108…
The rate of change, in $/yr, increases as the ∫ ∫
dB/dt = kB ⇒ dB/ B = k dt ⇒ ln | B| = kt + C
amount in the account increases. | B| = e kt +C = e kt ⋅ e C ⇒ B = C1e kt
D′(t ) 500 ⋅ (ln 1.06) ⋅ (1.06 t ) b. 5 = C1e k⋅0 ⇒ C1 = 5
3. R(t ) = =
D(t ) 500 ⋅ (1.06 t ) 7 = 5e3k ⇒ ln (7/5) = 3k
= ln 1.06 = 0.0582689081… 1 7
R(0) = ln 1.06 ⇒ k = ⋅ ln = 0.112157K
3 5
R(10) = ln 1.06 t/3
R(20) = ln 1.06 ∴ B = 5e (1/3) ln( 7/5)t = 5  
7
= 5e 0.112157Kt
4. The percent interest rate stays the same:  5
approximately 5.83%. c.
B
5. f (x) = a ⋅ bx ⇒
f ′(x) = a ⋅ (ln b) ⋅ bx = (ln b)(a ⋅ bx )
= (ln b) ⋅ f (x)
So f ′(x) is directly proportional to f (x). t
5
6. See Problem 11 in Section 7-2. 10

d. B = 5(7/5)24/3 = 73.78945…
About 74 million
Problem Set 7-2
e. 1000 = 5(7/5)t/3 ⇒ ln (1000/5) = t/3 ⋅ ln (7/5)
Q1. Q2.
3 ln 200
t= = 47.24001K
y y

ln (7/5)
1
1 About 47 hours after start, so in a little less
x x
than 2 days
2. a. N = number of units of radiation from N17;
Q3. Q4. t = number of seconds
y y

x

dN /dt = kN ⇒ dN / N = k dt ∫
1 x
⇒ ln |N| = kt + C
| N | = e kt +C ⇒ N = C1e kt
b. 3 × 1017 = C 1ek·0 ⇒ C 1 = 3 × 1017
Q5. Q6. 5.6 × 1013 = 3 × 1017e60k
y y
⇒ ln(1.866 K × 10 −4 ) = 60 k
x x ⇒ k = −0.143103…
∴ N = 3 × 1017 e −0.143103Kt
c.
N
Q7. Q8. 3 × 1017
y y
3
x

x
t

Calculus Solutions Manual Problem Set 7-2 139


© 2005 Key Curriculum Press
d. t = 5(60) = 300 s 5. a. dC/dt = kC
N = 3 × 1017 e ( −0.143103K)(300 ) = 0.067991K
It will not be safe because 0.067… > 0.007. b. ∫ dC/C = ∫ k dt ⇒ ln |C| = kt + D
3. a. F = number of mg; t = number of minutes ⇒ | C | = e kt + D ⇒ C = D1e kt
∫ ∫
dF/dt = kF ⇒ dF/ F = k dt ⇒ ln | F| = kt + C 0.00372 = D1e k⋅0 ⇒ D1 = 0.00372
0.00219 = 0.00372e8k
| F| = e kt +C ⇒ F = C1e kt
⇒ ln (0.00219/0.00372) = 8k
50 = C 1ek·0 ⇒ C 1 = 50 ⇒ k = (1/8) ⋅ ln (219/372) = −0.0662277…
30 = 50e20k ⇒ ln (30/50) = 20k ∴ C = 0.00372e− 0.0662277…t
⇒ k = (1/20) ⋅ ln (0.6) = −0.025541…
F = 50e ln ( 0.6 )t / 20 = 50(0.6)t / 20 = 50e −0.025541Kt c. Either: C = 0.015
⇒ 0.015 = 0.00372e− 0.0662277…t
b.
F
ln 4.0322… = −0.0662277…t
t = −21.05… , which is before the poison was
50
inhaled,
or: t = −20 ⇒ C = 0.00372e− 0.0662277…( − 20)
C = 0.0139… , which is less than 0.015
t ∴ the concentration never was that high.
50
d.
c. F = 50(0.6)(60/20) = 10.8 mg (exactly) P
100
d. 0.007 = 50(0.6)t/20
⇒ ln (0.007/50) = ln (0.6)t/20
⇒ t = 347.4323… 50

About 5 h 47 min t

4. a. V = number of dollars trade-in value; 20,000

t = number of months from the present

∫ ∫
dV /dt = kV ⇒ dV /V = k dt ⇒ ln |V | = kt + C e. (1/2)(0.00372) = 0.00372e− 0.0662277…t
ln (1/2) = −0.0662277…t ⇒ t = 10.4661…
|V | = e kt +C ⇒ V = C1e kt About 10.5 hours
b. 4200 = C1e k⋅0 ⇒ C1 = 4200 6. a. dP/dt = kP
4700 = 4200e ( k )( −3) ⇒ ln( 4700/4200) = −3k
k = (−1/3) ln (4700/4200) = −0.037492…
∴ V = 4200e −0.037492Kt
b. ∫ dP/P = ∫ k dt ⇒ ln | P | = kt + C
⇒ | P| = e kt +C ⇒ P = C1e kt
c.
V
100 = C1e k⋅0 ⇒ C1 = 100
50 = 100e5750k ⇒ ln 0.5 = 5750k
⇒ k = −0.0001205473…
∴ P = 100e− 0.0001205473…t
4200
t c. P = 100e( − 0.0001205473…)(4000) = 61.74301…
–30 30 About 61.7%
d. 48.37 = 100e −0.0001205473Kt
d. At 1 year after V = 4700, t = 9 months. ln 0.4837 = −0.0001205473…t
V = 4200e ( − 0.037492…)(9) = 2997.116… t = 6024.939…
About $3000 The wood is about 6025 years old. For 1996,
e. 1200 = 4200e− 0.037492…t the flood would have been 1996 − (−4004) =
⇒ ln (1200/4200) = −0.037492…t 6000 years ago, so the wood is old enough.
⇒ t = (−1/0.037492…) ⋅ ln (1200/4200) e.
= 33.4135…
P
About 33 months from the present 100

f. 31 months before V = 4700, t = −34. 61.7


∴ V = 4200e ( −0.037492K)( −34 ) = 15026.795K 50

About $15,000
t
g. The difference between $16,000 and $15,000 20,000
4000 5750
is the dealer’s profit.

140 Problem Set 7-2 Calculus Solutions Manual


© 2005 Key Curriculum Press
7. dM/dt = kM ⇒ M = Cekt by the techniques in dy
9. = 0.3 y
Problems 1–6, where C is the initial investment. dx
∴ M varies exponentially with t.
∫ ∫
dy
Let i = the interest rate as a decimal. = 0.3 dx
y
dM/dt = Ck ⋅ ekt
ln |y| = 0.3x + C
At t = 0, dM/dt = Ci.
∴ Ci = Ck ⋅ e0 ⇒ k = i ⇒ M = Ce it | y | = e 0.3 x +C = e 0.3 x ⋅ e C
Examples: y = ±eC ⋅ e0.3 x = C1e0.3 x
−4 = C1e0 ⇒ C1 = −4, showing that C1 can be
$1000 at 7% for 5 yr: $1419.07
negative.
$1000 at 7% for 10 yr: $2013.75
∴ y = −4e0.3 x
$1000 at 14% for 5 yr: $2013.75
$1000 at 14% for 10 yr: $4055.20 y
20
Leaving the money in the account twice as long 10
has the same effect as doubling the interest rate. x
Doubling the amount invested obviously doubles 5
the money at any particular time, but that doesn’t –10

tell us how that compares with doubling the time –20


or the interest rate.
Algebraically, Cei ·2t = Ce2i ·t shows that doubling dy
the time is equivalent to doubling the interest 10. = −0.2 y
dx
rate. Solving Ce2it > 2Ceit gives Ce2it − 2Ceit > 0
∫ ∫
⇒ Ceit(eit − 2) > 0 ⇒ eit > 2 (because Ceit > 0, so dy
= −0.2 x dx
C > 0, being an investment) ⇒ it > ln 2. So y
doubling either the time or the interest rate will ln |y| = −0.2x + C
always eventually yield more than doubling the | y | = e −0.2 x +C = e −0.2 x ⋅ e C
investment, once t is high enough. For example, y = ±eC ⋅ e− 0.2 x = C1e− 0.2 x
at 7%, 0.07t > ln 2 ⇔ t > (ln 2)/0.07 = 30 = C1e− 1.4
9.9021… ⇒ t, so by 9 years 11 months,
30
doubling the time or interest rate will yield more C1 = = 7.3979K
than doubling the investment. e1.4
∴ y = 7.3979…e− 0.2 x
8. Assume an investment of $1000 at 7% per year.

∫ ∫
For 5 years, as in Problem 7:
11. dy/dx = ky ⇒ dy/y = k dx ⇒ ln | y | = kx + C1
Annually: M = 1000(1.07)5 = $1402.55
Quarterly: M = 1000(1.0175)20 = $1414.78 | y | = e kx +C1 ⇒ y = Ce kx
Monthly: M = 1000(1.00583…)60 = $1417.63 y(0) = Ce k⋅0 ⇒ C = y(0) ⇒ y = y0 e kx , Q.E.D.
Daily: M = 1000(1.0001917808…)1825 =
$1419.02 Problem Set 7-3
Continuously (Problem 7): $1419.07
Note that compounding continuously is only Q1. Cekx Q2. (kx2)/2 + C
5 cents better than compounding daily for a Q3. kx + C Q4. −cos x + C
$1000 investment in 5 years!
Q5. 1/ 1 – x 2 Q6. 5 cos x
M = M 0(1 + k/n)nt
Let L = lim (1 + k/n) nt → 1∞. Q7. tan x
n→∞
ln L = lim [nt ⋅ ln (1 + k/n)] → ∞ ⋅ 0 Q8.
n→∞ y' or y
t ⋅ ln (1 + k/n) ∞
= lim → y
n→∞ n –1 ∞ 1
x
1 −2 1
t⋅ ⋅ ( − kn ) y'
1 + k /n
= lim
n→∞ − n −2
kt Q9. lim Ln = lim Un
= lim = kt ∆x →0 ∆x →0
n→∞ 1 + k /n

∴ L = ekt Q10. B
∴ lim M = M0 e kt , which is the continuous 1. a. dM/dt = 100 − S
n→∞
compounding equation. b. S = kM ⇒ dM/dt = 100 − kM

Calculus Solutions Manual Problem Set 7-3 141


© 2005 Key Curriculum Press
− k dM
∫ 100 − kM = ∫ dt ⇒ − k ∫ 100 − kM = ∫ dt
dM 1 The graph is almost straight. The $100/day
c.
deposits far exceed the interest for the first few
1 years.
⇒ − ln |100 − kM | = t + C
k M

⇒ |100 − kM| = e− kte− kC


⇒ 100 − km = C 1e− kt ⇒ kM = 100 − C 1e− k t 50,000
1
⇒ M = (100 – C1e – kt )
k t

Substitute M = 0 when t = 0. 500

1
0 = (100 – C1e 0 ) ⇒ C1 = 100 Make a table of M and dM/dt for various
k numbers of years. Neglect leap years.
100
∴M = (1 – e – kt ) Years M dM/dt
k
d. k = 0.02 ⇒ M = 5000(1 – e–0.02 t) 0 0 100.00
e. 1 37865 107.57
M 10 537540 207.51
5000
20 1652980 430.60

After 1 year, the $100/day is putting more into


t
the account. After 10 years, the interest has
30 60 90 started putting in more than the $100/day. After
20 years, the interest puts in about $331 a day,
while the winnings still put in only $100 a day.
f. t = 30: $2255.94 ($3000 in, $744.06 spent)
As t approaches infinity, the amount in the
t = 60: $3494.03 ($6000 in, $2505.97 spent)
account becomes infinite!
t = 90: $4173.51 ($9000 in, $4826.49 spent)
g. t = 365: (365.23 or 366 could be used.) 3. a. E = RI + L(dI/dt)
M = 5000(1 – e− 7.30 ) = 4996.622… b. L dI/dt = E – RI
≈ $4996.62 in the account L − R dI
∫ ∫ ∫ ∫
L dI
dM/dt = 100 − 0.02(4996.622…) = 0.06755… = dt ⇒ − = dt
E – RI R E − RI
Increasing at about $0.07 per day L
h. lim M = lim 5000(1 – e −0.02 t ) ⇒ – ln | E – RI | = t + C
t →∞ t →∞ R
= 5000(1 − 0) = 5000 ⇒ | E – RI | = e − ( R/ L )t e − ( R/ L )C
1
2. dM/dt = 100 + kM (k = daily interest rate) ⇒ E − RI = C1e − ( R/ L )t ⇒ I = [ E – C1e –( R/ L )t ]
R
∫ ∫ ∫ ∫
dM 1 k dM
= dt ⇒ = dt ⇒ Substitute I = 0 when t = 0.
100 + kM k 100 + kM 1
1 0 = ( E – C1e 0 ) ⇒ C1 = E
ln |100 + kM | = t + C ⇒ |100 + kM | = e kt e kC ⇒ R
k E
∴ I = [1 – e –( R/ L )t ]
100 + km = C 1e kt ⇒ kM = −100 + C 1e kt ⇒ R
1
M = (C1e kt − 100) c. I =
110
[1 – e –(10 / 20 )t ]
k 10
Substitute M = 0 when t = 0. I = 11(1 – e− 0.5 t)
1
0 = (C1e 0 – 100) ⇒ C1 = 100 I
k
11
100 kt
∴M = (e – 1)
k
Let k = 0.0002 (0.02% per day).
∴ M = (500000)(e0.0002 t – 1) t
5 10

142 Problem Set 7-3 Calculus Solutions Manual


© 2005 Key Curriculum Press
d. i. I = 11(1 – e− 0.5) = 4.3281… ≈ 4.33 amps 0.99 = 1 – e− 0.02t
ii. I = 11(1 – e–5) = 10.9258… ≈ 10.93 amps e− 0.02t = 0.01
iii. lim I = lim 11(1 – e −0.5t ) = 11(1 – 0) −0.02t = ln 0.01
t →∞ t →∞
t = −50 ln 0.01 = 230.258…
= 11 amps
About 230 seconds
e. I = 0.95(11) = 10.45
10.45 = 11(1 – e −0.5t ) dV
5. a. = kV 1/2
dt
0.95 = 1 – e −0.5t
∫V ∫
−1/2
e −0.5t = 0.05 b. dV = k dt ⇒
−0.5t = ln 0.05 kt + C 
2
2V 1/2 = kt + C ⇒ V = 
t = −2 ln 0.05 = 5.9914…  2 
About 6 seconds V varies quadratically with t.
4. a. R = C(dT/dt) + hT c. Initial conditions t = 0; V = 196;
dV/dt = −28:
b. C dT/dt = R – hT k⋅0+C
1961/2 = ⇒ C = 28
∫ C dT/( R − hT ) = ∫ dt 2
and −28 = k ⋅ 1961/ 2 ⇒ k = −2
C − h dT
− ∫ R − hT = ∫ dt  −2 t + 28 
2

h ∴ V =  ⇒ V = (14 − t ) 2
 2 
C
− ln | R − hT | = t + D d. False. dV/dt = 2t − 28, so the water flows
h
− ( h / C )t − ( h / C ) D
out at 28 ft3/min only when t = 0. For
| R − hT | = e e instance, at t = 5, dV/dt = −18, which
R − hT = D1e –( h /C )t means water flows out at only 18 ft3/min.
T = (1/ h)[ R – D1e –( h /C )t ] So it takes longer than 7 min to empty
Substitute T = 0 when t = 0. the tub.
1 e. 0 = (t − 14)2 ⇒ the tub is empty at
0 = ( R – D1e 0 ) ⇒ D1 = R
h t = 14 min.
R f.
∴ T = [1 – e –( h / C )t ]
h V
− (0.04/ 2)t
c. T = (50/0.04)[1 – e ]
T = 1250(1 – e− 0.02t )
100
d.
T t
1250 14

g. See the solution to Problem C4 in Problem


t
Set 7-7.
100 200
6. The following data were gathered in the author’s
e. Use TRACE or TABLE. class in December 1994. Times t are in seconds
and volumes V are in mL. Note that a burette
t = 10: T = 226.586… ≈ 227° reads the amount of fluid delivered, so you
t = 20: T = 412.099… ≈ 412° must subtract the reading from 50 to find the
t = 50: T = 790.150… ≈ 790° volume remaining. Use food coloring in the
t = 100: T = 1080.830… ≈ 1081° water to make the liquid level easier to read.
t = 200: T = 1227.105… ≈ 1227° Read from the bottom of the meniscus (the
f. lim T = lim 1250(1 – e −0.02 t ) = 1250(1 – 0) curved surface of the liquid).
t →∞ t →∞
= 1250°
g. T = 0.99(1250) = 1237.5
1237.5 = 1250(1 – e− 0.02t )

Calculus Solutions Manual Problem Set 7-3 143


© 2005 Key Curriculum Press
Seconds Reading Volume position of the vertex can be used to predict the
position of the stopcock and the time when the
0 0 50 fluid would all be gone if the burette were of
10 2.4 47.6 uniform diameter all the way down to the
20 4.4 45.6 stopcock. For the preceding data, the vertex is at
−0.20964 K
30 6.4 43.6 t=− ≈ 500 s
(2)(0.000209255K)
40 8.5 41.5
V ≈ −3.0 mL
50 10.5 39.5
So the stopcock should be found at a point
60 12.4 37.6 corresponding to about 3 mL below the bottom
70 14.3 35.7 mark.
80 16.1 33.9 7. a. n = 1, k = 1, C = −3:
90 17.8 32.2 ∫ ∫
∴ dy/dx = y ⇒ dy/ y = dx ⇒ ln | y | = x − 3
100 19.9 30.1 ⇒ |y| = e x−3 = exe− 3 ⇒ y = ±0.04978…ex
110 21.2 28.8 y
120 22.8 27.2
1
130 24.5 25.6 x
140 25.6 24.4 1

150 27.4 22.6


160 28.6 21.4
170 30.0 20.0
b. n = 0.5, k = 1, C = −3:
∫ ∫
180 31.3 18.7
∴ dy/dx = y 0.5 ⇒ y −0.5 dy = dx
190 32.6 17.4
1
200 33.8 16.2 ⇒ 2 y 0.5 = x − 3 ⇒ y = ( x − 3)2
4
210 35.1 14.9 Note: x ≥ 3 because y0.5 is a positive number.
220 36.4 13.6
y
230 37.4 12.6
240 38.5 11.5
1
250 39.5 10.5
260 40.6 9.4 x
3
M M M
320 46.1 3.9
360 49.3 0.7 c. n = −1 ⇒ dy/dx = ky −1 ⇒ y dy = k dx ∫ ∫
1
Using quadratic regression with these data, ⇒ y 2 = kx + C ⇒ y = ± 2 kx + 2C
2
V = 0.000209255…t2 − 0.20964…t + 49.54… . k = 1, C = −3 ⇒ y = ± 2 x − 6
The data and the equation can be plotted on the
y
grapher, as shown. 5

V
50 x
3

t
100
n = −2 ⇒ dy/dx = ky −2 ⇒ y 2 dy = k dx ∫ ∫
The volume does seem to vary quadratically with 1 3
time. Because there is still fluid in the burette ⇒ y = kx + C ⇒ y = 3 3kx + 3C
3
when V = 0, the graph crosses the t-axis, unlike
the graphs in Problem 5 and Example 1. The k = 1, C = −3 ⇒ y = 3 3 x − 9

144 Problem Set 7-3 Calculus Solutions Manual


© 2005 Key Curriculum Press
5
y
⇒ | M − kB| = e − kct e − kcC
⇒ M − kB = C1e − kct
x
1
3 ⇒B= ( M − C1e − kct )
k
Use the initial condition B = 0 when t = 0.
0 = (1/k) ( M − C 1e0) ⇒ C 1 = M

∫ ∫
dy
d. For n > 1, = ky n ⇒ y − n dy = k dx M
∴ B = (1 – e – kct )
dx k
− ( n −1)
y
⇒− = kx + C because n > 1, Use the initial condition kB = 80 when
n −1 B = 1000.
−1
so y = 80 = k(1000) ⇒ k = 0.08
n −1 ( n − 1) ⋅ ( kx + C )
Use the initial condition dB/dt = 500 when t = 0.
which has a vertical asymptote at x = −C/k
From dB/dt = c (M − kB), 500 = c (M − 0) ⇒
because the denominator equals zero for this
c = 500/M.
point.
∴ particular equation is
Note that the radical will involve a ± sign
when the root index is even (for example, B = ( M/0.08)[1 − e −0.08/( 500 / M )t ]
when n is odd). B = 12.5 M[1 − e − ( 40 / M )t ]
For n = 2, k = 1, C = −3: y = −( x − 3) −1 Assume various values of M:
y
M = 1000: B = 12500(1 − e− 0.04 t)
2
M = 5000: B = 62500(1 − e− 0.008 t )
M = 10000: B = 125000(1 − e− 0.004 t )
x
3
B
M = 10000
100,000
M = 5000
−1
For n = 3, k = 1, C = −3: y = M = 1000
± 2x − 6 t
250 500
y
2

x As shown on the graph, the sales start out


3
increasing at the same rate (500 bottles/day). As
t increases, the number of bottles/day increases,
approaching a steady state equal to 12.5M.
To find the break-even time, first find the total
Note that the graph shows two branches. number of bottles sold as a function of time. B is
dy in bottles per day, so the total sales in x days,
e. For n = 0, = ky 0 = k , so y = kx + C,
dx T(x), is
x
a linear function. For k = 1, C = −3, y = x − 3. T ( x) = ∫
0
B dt.
y Use, for example, M = $10,000/day.
x

∫ 125000(1 − e
−0.004 t
T ( x) =
2
) dt
0
x
x
3 = 125000[t + (1/0.004)e −0.004 t ] 0

= 125000( x + 250e −0.004 x − 0 − 250)


= 125000[ x − 250(1 − e −0.004 x )]
8. dB/dt = c ( M − kB), where k and c are constants. For selling prices of $0.25 and $0.50/bottle, the
1 − k dB total numbers of dollars are
∫ ∫ ∫ ∫
dB
= c dt ⇒ − = c dt
M − kB k M − kB D25(x) = 31250[x − 250(1 − e− 0.004 x)]
1 D50(x) = 62500[x − 250(1 − e− 0.004 x)]
⇒ − ln | M − kB | = ct + C
k

Calculus Solutions Manual Problem Set 7-3 145


© 2005 Key Curriculum Press
The total amount spent on advertising is M ⋅ x, T t
or A(x) = 10000x.
The three graphs can be plotted by grapher. For 140° 39 min
$0.25/bottle, the break-even time is 207 days. 155° 61 min
For $0.50/bottle, the break-even time is 90 days 160° 72 min
(less than half!). 170° 130 min
180° Never!
Dollars (millions) $0.25/bottle
$0.50/bottle
2
The limit of T as t increases is 70 +
1
102.26…(1 + 0), which equals 172.26…°. Thus,
the temperature never reaches 180°. When the
x
heater turns off, the differential equation becomes
90 207
dT
= − kh (T − 70) ⇒ T = 70 + C2 e − kht .
dt
9. The differential equation is dT/dt = k(1200 − L), Using T = 160 at time t = 0 when the heater
and L = h (T − 70), where h is a proportionality turns off, T = 70 + 90e− 0.02933…t.
constant. To find the time taken to drop to 155°, substitute
∴ dT/dt = k(1200 + 70h − hT ) 155 = 70 + 90e −0.02933Kt .
Solving numerically or algebraically gives
∫ dT/(1200 + 70h − hT ) = ∫ k dt t = 1.9… . Thus, it takes only 2 minutes for the
temperature to drop 5°! By contrast, from the
(−1/h) ln |1200 + 70h − hT | = kt + C
preceding table, it takes 11 minutes
ln |1200 + 70h − hT | = −kht − h C (t = 61 to t = 72 in the table) to warm back up
|1200 + 70h − hT | = e− kht ⋅ e− hC from 155° to 160°.
1200 + 70h − hT = C 1e− kht ⇒ The design of the heater is inadequate because it
hT = 1200 + 70h − C 1e− kht takes much longer to warm up by a certain
T = 1200/h + 70 − (C 1/h) ⋅ e− kht amount than it does to cool back down again.
Use T = 70 when t = 0. Near 172°, a slight increase in the thermostat
70 = 1200/h + 70 − C 1/h ⋅ e− kh⋅ 0 setting for the heater makes a great increase in
the time taken to reach that setting. For instance,
⇒ C 1 = 1200
it takes an hour (72 minutes to 130 minutes) to
∴ T = 1200/h + 70 − 1200/h ⋅ e− kht warm the 10 degrees from 160° to 170°. These
T = 70 + (1200/h)(1 − e− kht) inadequacies could be corrected most easily by
Substitute t = 0, L = 0, and dT/dt = 3 into the adding more insulation. The resulting decrease in
original differential equation. h would make the heater cool more slowly, heat
3 = k(1200 − 0) ⇒ k = 0.0025 up faster, and reach the 180 degrees it currently
∴ T = 70 + (1200/h)(1 − e− 0.0025 ht) cannot reach. Decreasing h would also reduce the
Using T = 96 when t = 10, power consumption.
26 = (1200/h)(1 − e− 0.025 h). 10. a. dP/dT = kP/T 2
Solving numerically gives h ≈ 11.7347… .
∫ ∫
dP/P = k dT /T 2 ⇒ ln | P | = − kT −1 + C
∴ equation is T ≈ 70 + 102.26…(1 − e− 0.02933…t ). ⇒ | P| = e − k /T + C
⇒ P = C1e − k / T
Time data for various temperatures can be found
b. 0.054 = C1e − k /293
by grapher or by substituting for T and
solving for t. 3.95 = C1e − k /343
(3.95 / 0.054) = e ( − k /343+ k /293)
T
ln (3.95/0.054) = −k/343 + k/293
Never reaches 180°
k = [ln (3.95 / 0.054)]/( −1/ 343 + 1/ 293)
170°
= 8627.812641K
160°
140°
100
From ln |P| = −kT − 1 + C,
C = ln 0.054 + 8627.812641…/293.
t
39 72 130
C = 26.52768829… ⇒ C1 = e26.52768829…
P = e 26.52768829…e −8627.812641…/T
P = e ( 26.52768829K−8627.812641K/T )

146 Problem Set 7-3 Calculus Solutions Manual


© 2005 Key Curriculum Press
c. So the new equation models the data above
Temperature T P Actual* the melting point, but not below it.
d. Using the equation for liquid naphthalene,
10 283 0.0190… 0.021
760 = e(18.33140949… − 5734.569702…/ T ) ,
20 293 0.054 0.054 ln 760 = 18.33140949… − 5734.569702…/T
30 303 0.142… 0.133 5734.569702 K
40 313 0.354… 0.320 T = = 490.214 K
18.33140949K – ln 760
50 323 0.832… 0.815 About 490 K, or 217°C (actual: 218°C)
60 333 1.85… 1.83 e. Answers will vary.
70 343 3.95 3.95
80 353 8.05… 7.4
(melting Problem Set 7-4
point)
Q1. y ′ = 5x 4 Q2. y ′ = 5x ln 5
90 363 15.7… 12.6
Q3. (1/8)x + C
8
Q4. 7 x/ln 7 + C
100 373 29.8… 18.5
Q5. y ′ = −y/x or y ′ = −3x − 2
110 383 54.6… 27.3 Q6. 87.5
200 473 3972.1… 496.5 Q7. Q8.
*Source: Lange’s Handbook of Chemistry, 1952, p. 1476. y y

The function models the data well up to the


melting point, but not above it. The 1 1
x x
differences between the predicted and actual 1 1
answers are most likely due to the fact that
naphthalene changes from solid to liquid at
Q9. g(5) − g(1) Q10. E
80°C; the constants for solid and liquid
naphthalene differ. 1. a. dy/dx = x/(2y)
Use initial conditions for T = 90, 110 as in At (3, 5), dy/dx = 3/10 = 0.3.
part b to get a better equation for the liquid: At (−5, 1), dy/dx = −5/2 = −2.5.
12.6 = C1e− k/ 363 On the graph, the line at (3, 5) slopes upward
496.5 = C1e− k/ 473 with a slope less than 1. At (−5, 1) the line
k = [ln (496.5/12.6)]/(−1/473 + 1/363) slopes downward with a slope much steeper
= 5734.569702… than −1.
C = ln 12.6 + 5734.569702…/363 b. The figure looks like one branch of a
= 18.33140949… hyperbola opening in the y-direction. (The
⇒ C1 = e18.33140949… lower branch shown on the graph is also part
∴ P = e(18.33140949… − 5734.569702…/ T ) of the solution, but students would not be
With the new equation, expected to find this graphically.)
Temperature T P Actual y

10 283 0.144… 0.021


20 293 0.289… 0.054 (3, 5)
5
30 303 0.551… 0.133
40 313 1.01… 0.320 (–5, 1) (1, 2)
(5, 1) x
50 323 1.78… 0.815 5
60 333 3.03… 1.83
70 343 5.01… 3.95
80 353 8.05… 7.4
(melting
point)
90 363 12.6 12.6
100 373 19.2… 18.5 c. See graph in part b. The figure looks like the
110 383 28.7… 27.3 right branch of a hyperbola opening in the
x-direction. (The left branch is also part of the
200 473 496.5 496.5

Calculus Solutions Manual Problem Set 7-4 147


© 2005 Key Curriculum Press
solution, but students would not be expected 1 2
y2 = − x +C
to find this graphically.) 2

∫ ∫
dy x 1 1
d. = ⇒ 2 y dy = x dx ⇒ y 2 = x 2 + C ( −1)2 = − (1)2 + C ⇒ C = 1.5
dx 2 y 2 2
x = 5, y = 1 ⇒ C = 1 − 12.5 = −11.5 y = 1.5 − 0.5x 2
2

By algebra, x2 − 2y2 = 23. This is the y = − 1.5 − 0.5 x 2


particular equation of a hyperbola opening in
the x-direction, which confirms the (Use the negative square root because of the
observations in part c. initial condition.)
2. At (3, 3), dy/dx = 0.1(3) = 0.3, which is The graph agrees with part b.
reasonable because the slope is positive and less From the next-to-last line, add 0.5x2 to both
than 1. At (0, −2), dy/dx = 0.1(−2) = −0.2, which sides, getting 0.5x2 + y2 = 1.5, which is the
is reasonable because the slope is negative and equation of an ellipse because x2 and y2 have
less than 1 in absolute value. The next graph the same sign but unequal coefficients.
shows the two particular solutions. For the first,
dy
y (6) ≈ 4.0. For the second, y (6) ≈ −3.6. 4. a. At (3, 1), = 3(1 − 1) = 0.
dx
dy
= 0.1y ⇒ y = C1e 0.1x dy
dx At (1, 2), = 1(1 − 2) = −1.
For (3, 3), the particular solution is dx
y = 2.2224…e0.1 x, giving y(6) = 4.0495… . dy
At (0, − 1), = 0(1 + 1) = 0.
For (0, −2), the particular solution is dx
y = −2e0.1 x, giving y(6) = −3.6442… .
y
Both graphical answers are close to these actual 3
answers.
2
y
4
3 1

2 x
1 –3 –2 –1 1 2 3
x
–2 –1 1 2 3 4 5 6 –1
–1
–2
–3
–4
b. See the graph from part a. Both graphs have a
horizontal asymptote at y = 1.
dy 3 dy
3. a. At (3, 2), =− = −0.75. c. = x (1 − y)
dx (2)(2) dx
dy 1 dy
At (1, 0), =− , which is infinite. = x dx
dx (2)(0) 1– y
y

∫ 1 – y = ∫ x dx
2 dy
1
x −ln |1 − y| = 0.5x 2 + C
1 − y = ± e −0.5 x ⋅ e − C
2
–3 –2 –1 1 2 3
–1
y = 1 + C1e −0.5 x (C1 can be positive or
2

–2 negative.)
−1 = 1 + C 1e 0 ⇒ C 1 = −2
b. See the graph from part a. The figures
∴ y = 1 − 2e −0.5 x
2

resemble half-ellipses.
dy x The grapher confirms the graph in part b.
c. =−
As | x | → ∞, e −0.5 x → 0. So y → 1,
2
dx 2y
2y dy = − x dx which agrees with the horizontal asymptote
at y = 1.

2 y dy = − x dx ∫
148 Problem Set 7-4 Calculus Solutions Manual
© 2005 Key Curriculum Press
5. 9. a.
y
y
2

1.5
(3, 2)
1

0.5 x
x
–2 –1.5 –1 –0.5 0.5 1 1.5 2
–0.5 (1, –2)

–1

–1.5

–2
dy
b. = −0.2 xy
dx
6.
y Evidence: At (1, 1) the slope was given to
be −0.2, which is true for this differential
2

1.5
equation. As x or y increases from this
1
point, the slope gets steeper in the negative
0.5
x
direction, which is also true for this
–2 –1.5 –1 –0.5 0.5 1 1.5 2 differential equation. In Quadrants I and III the
–0.5 slopes are all negative, and in Quadrants II
–1 and IV they are all positive. (Note: The
algebraic solution is y = Ce −0.1x .)
2
–1.5

–2
10. a. Initial condition (0, 2)

7. y

y
2

1.5 5

1
(0, 2)
0.5
x x
5
–2 –1.5 –1 –0.5 0.5 1 1.5 2
–0.5 (0,–2.5)

–1 (0, –5)
–1.5

–2

8.
y
b. See the graph in part a with initial condition
2
(0, −5). The graph goes toward −∞ in the
1.5 y-direction instead of toward +∞.
1 c. If a ruler is aligned with the slope lines, the
0.5 lines that form a straight line are the ones
x
–2 –1.5 –1 –0.5 0.5 1 1.5 2
crossing the y-axis at −2.5 with slope −1/2.
–0.5 (In courses on differential equations, students
–1 will learn that the given equation is a first-
–1.5 order linear equation that can be solved using
–2 an integrating factor. The general solution is
y = Ce0.2 x − 0.5x − 2.5. For C = 0, the curve
is the line y = −0.5x − 2.5, which intersects
the y-axis at (0, −2.5).)

Calculus Solutions Manual Problem Set 7-4 149


© 2005 Key Curriculum Press
11. a. Initial condition (0, 2) The graph shows this velocity for times
P above about 15 seconds.
(0, 18)
d. See the graph in part b with initial condition
(5, 0). The graph is identical to the one in
part b except shifted 5 seconds to the right.
This behavior is to be expected because the
10.5 differential equation is independent of t.
e. See the graph in part b. This graph decreases
to the terminal velocity because the diver
starts out going faster.
f. Similarities include: Both models have a
(0, 2) (4, 2) horizontal asymptote that the particular
t
solutions approach from above or below.
Both models decrease rapidly and gradually
b. See the graph in part a with initial condition level off for values above the asymptotic
(4, 2). The graph is the same as that in part a limit.
but shifted over 4 months. This behavior is Differences include: For values below the
to be expected because dP/dt depends only on asymptotic limit, one model starts with rapid
P, not on t, and both initial conditions have increase and gradually slows its growth,
the same value of P. whereas the other starts with a slow increase
c. See the graph in part a with initial condition that becomes more rapid growth before
(0, 18). The population is decreasing to the slowing toward the asymptote.
same asymptote, P = 10.5, as in parts a mg
13. a. ma = 2 By hypothesis
and b. r
d. The asymptote at P = 10.5 indicates that the dv g dv
= 2 Divide by m; a = .
island can sustain only 1050 rabbits. If the dt r dt
dv dr g
population is lower than that, it increases. If ⋅ = Chain rule
the population is higher than that, it dr dt r 2
dv g dr
decreases. The number 10.5 is a value of P ⋅v = 2 v= (r = distance)
that makes dP/dt equal zero. Note that there is dr r dt
another asymptote at P = 0, which also dv g
= 2 Divide by v.
makes dP/dt equal zero. dr r v
12. a. dv/dt = 32.16 − 0.0015v2 dv
b. (5, 2) = −1.2488
The slope at (5, 120) appears to be about 1, dr
but dv/dt actually equals 32.16 − dv
(1, 10) = −6.244
0.0015(120)2 = 10.56. The answers are dr
dv
different because the graph is scaled by a (10, 4) = −0.1561
factor of 10. dr
These slopes agree with those shown.
b. Initial condition (0, 0)
c. Initial condition (r, v) = (1, 10)
v
(0,180) From the graph, the velocity is zero at r ≈ 5.
So the spaceship is about 4 Earth radii, or
146.4... about 25,000 km, above the surface.
v
(5,120)
(1,18)

14.11...

(1,12)
50
(1,10) (2,10)

t 6.12...
(0, 0) (5, 0)
5 4.37...
(10,4)
c. Terminal velocity occurs when dv/dt = 0. (5,2)
0 = 32.16 − 0.0015v2 r
v = (32.16/0.0015)1/2 = 146.424… ≈ 146 ft/s

150 Problem Set 7-4 Calculus Solutions Manual


© 2005 Key Curriculum Press
The precise value of r can be found Q7.
y
algebraically.

∫ ∫
dv –62.44 –62.44
= ⇒ v dv = dr 1
dr r 2v r2 y' y'
x

v 2 62.44 1
⇒ = +C
2 r
For the solution through (1, 10), C = 50 − Q8. 3x 2y 5 + 5x 3y 4y′ = 1 + y′
62.44 = −12.44, so the ship starts falling Q9. continuous Q10. A
when v = 0 at r = 62.44/12.44 ≈ 5. 1. a. dy = −(x/y) dy
d. See the graph in part c with initial condition For (1, 3), dy = −(1/3)(0.5) = −0.1666… ,
(r, v) = (1, 12). The graph levels off between so new y ≈ 3 − 0.1666… = 2.8333… at
4 and 5 km/s. The precise value of v can be x = 1.5.
found algebraically. For (1.5, 2.8333…), dy =
v 2 62.44 −(1.5/2.8333…)(0.5) = −0.2647… ,
C = 72 − 62.44 = 9.56 ⇒ = + 9.56
2 r so new y ≈ 2.8333… − 0.2647… = 2.5686…
Because r > 0, v is never zero, so the at x = 2.
spaceship never stops and falls back.
As r approaches infinity, v2/2 approaches x y
9.56, and thus v approaches
0 3.2456…
(2)(9.56) = 4.37… km/s.
0.5 3.1666…
e. See the graph in part c with initial condition
1 3
(r, v) = (1, 18). The graph levels off at
v ≈ 14 km/s. Here the spaceship loses about 1.5 2.8333…
4 km/s of velocity, whereas it loses 7 or 2 2.5686…
8 km/s when starting at 12 km/s. Both cases 2.5 2.1793…
lose the same amount of kinetic energy,
3 1.6057…
which is proportional to v2 (the change in v2
is the same in both cases). The precise value The Euler’s y-values overestimate the actual
of v can be found algebraically as in part d. values because the tangent lines are on the
For the solution through (1, 18), convex side of the graph and the convex side
C = 162 − 62.44 = 99.56. As r → ∞, is upward.
v → (2)(99.56) = 14.11… km/s.
b. dy = −(x/y) dy
f. See the graph in part c with initial condition
(r, v) = (2, 10). The graph levels off at ∫ y dy = − ∫ x dx
about 6 km/s, so the spaceship does escape. 0.5y 2 = −0.5x 2 + C
Alternatively, note that the solution through 0.5(32) = −0.5(12) + C ⇒ C = 5
(2, 10) lies above the solution through 0.5y 2 = −0.5x 2 + 5
(1, 12). The precise value of v can be found y = 10 − x 2 (Use the positive square root.)
algebraically as in parts d and e. For the
solution through (2, 10), C = 50 − 31.22 = At x = 3, y = 10 – 32 = 1.
18.78. As r → ∞, v → (2)(18.78) = The particular solution stops at the x-axis
6.12… km/s. because points on the circle below the x-axis
would lead to two values of y for the same
14. See the Programs for Graphing Calculators
value of x, making the solution not a
section of the Instructor’s Resource Book.
function.
The Euler’s value of 1.6057… overestimates
the actual value by 0.6057… .
Problem Set 7-5 2. a. dy = (x/y) dy
Q1. k y Q2. y = Ce3x For (1, 2), dy = (1/2)(0.5) = 0.25, so new
y ≈ 2 + 0.25 = 2.25 at x = 1.5.
Q3. 4.8 Q4. 100
For (1.5, 2.25), dy = (1.5/2.25)(0.5) =
Q5. −ln |1 − v| + C Q6. sec x tan x 0.3333… , so new y ≈ 2.25 + 0.3333… =
2.5833… at x = 2.

Calculus Solutions Manual Problem Set 7-5 151


© 2005 Key Curriculum Press
y
x y 4

3
0 1.6071…
2
0.5 1.75 1
x
1 2 1 2 3 4

1.5 2.25
You cannot tell whether the last value of y is an
2 2.5833… overestimate or an underestimate because the
2.5 2.9704… convex side of the graph is downward in some
places and upward in other places.
3 3.3912…
4. dx = 0.3. Make a table showing values of dy =
The Euler’s y-values underestimate the actual 0.3(dy/dx) and new y = old y + dy.
values because the tangent lines are on the
convex side of the graph and the convex side x dy/dx dy y
is downward. The error is greater at x = 0
because the graph is more sharply curved 1 −3 −0.9 2
between x = 0 and x = 1 than it is between 1.3 −2 −0.6 1.1
x = 1 and x = 3.
1.6 −1 −0.3 0.5
b. dy = x/y dy
1.9 0 0 0.2
∫ ∫
y dy = x dx
2.2 1 0.3 0.2
0.5y = 0.5x 2 + C
2
2.5 2 0.6 0.5
0.5(22) = 0.5(12) + C ⇒ C = 1.5
0.5y 2 = 0.5x 2 + 1.5 2.8 3 0.9 1.1
y = x 2 + 3 (Use the positive square root.) 3.1 4 1.2 2
At x = 0, y = 0 + 3 = 3 = 1.7320 … . 3.4 5 1.5 3.2
The particular solution stops at the x-axis 3.7 6 1.8 4.7
because points on the circle below the x-axis
would lead to two values of y for the same 3.9 7 2.1 6.5
value of x, making the solution not a y
function. 5
4
The Euler’s value of 1.6071… underestimates 3
the actual value by 0.1249… unit. 2

3. dx = 0.2. Make a table showing values of dy = 1


x
0.2(dy/dx) and new y = old y + dy. 1 2 3 4

x dy/dx dy y The approximate values of y underestimate the


actual values of y because the convex side of the
2 3 0.6 1 graph is down.
2.2 5 1.0 1.6 5. See the Programs for Graphing Calculators
section of the Instructor’s Resource Book.
2.4 4 0.8 2.6
6. See the Programs for Graphing Calculators
2.6 1 0.2 3.4 section of the Instructor’s Resource Book.
2.8 −3 −0.6 3.6 dy
7. a. and b. = −0.2 xy
3 −6 −1.2 3.0 dx

3.2 −5 −1.0 1.8 y

3.4 −3 −0.6 0.8 (3, 2)

x
3.6 −1 −0.2 0.2
3.8 1 0.2 0.0 (1, –2)

4 2 0.4 0.2

152 Problem Set 7-5 Calculus Solutions Manual


© 2005 Key Curriculum Press
dy putting the graph into a region of the slope
8. a. and b. = −0.1x + 0.2 y
dx field from which the spacecraft would not
escape Earth’s gravity.
y
d. Let v1 be the initial velocity at r = 1.
Solving for C gives
5 0.5v12 = 62.44 + C
(0, 4)
C = 0.5v12 − 62.44
(0, 2)
5 x If v1 < 2(62.44), then C is negative,
making v = 128.88r + C an imaginary
–1

c. When the graph is observed, the slope lines number when r is large enough. If
seem to follow a straight path using (0, 2.5) v1 > 2(62.44), then C is positive, making v
as an initial condition. Euler’s method a positive real number for all positive
confirms this. values of r. (The asymptote is v = C .)
(In differential equations, students will learn
how to solve such first-order linears by 10. a. v(2) = 61.6831… , v(4) = 106.2850… ,
multiplying both sides by the integrating v(6) = 129.7139… , v(8) = 139.9323… ,
factor e− 0.2x . The general solution is v(10) = 143.9730… , v(20) = 146.4066…
y = Ce 0.2x + 0.5x − 2.5. For C = 0, the These values will be overestimates because
particular solution is y = 0.5x + 2.5.) the graph is concave down (convex side up),
so the Euler’s tangent lines will be above the
9. a. Using dr = 0.6, v(13.6) ≈ 0.1414… and actual graph, as in the next graph.
v(14.2) ≈ −1.2900… , so the spacecraft seems
to reverse direction somewhere between these 200 v

two values of r, as shown in the graph in


part b.
b. Using dr = 0.1, v(20) ≈ 4.5098… , and the Euler
values are leveling off, as shown in the
graph. 100 Actual

20 v

t
10
10 20
Actual

b. v(2) = 157.7979… , v(4) = 150.5128… ,


dr = 0.1 v(6) = 147.9234… , v(8) = 146.9777… ,
dr = 0.6
r v(10) = 146.6290… , v(20) = 146.4254…
10 20 These values will be underestimates because
the graph is concave up (convex side down),
dv –62.44
c. = so the Euler’s tangent lines will be below the
dr r 2v actual graph.
∫ v dv = −62.44 ∫ r
−2
dr dv
c. = 0 ⇔ 0.0015v 2 = 32.16 ⇔
−1
0.5v = 62.44r + C
2 dt
0.5(12)2 = 62.44(1) −1 + C ⇒ C = 9.56 v=
32.16
= 146.42404 K(store)
0.5v 2 = 62.4r −1 + 9.56 0.0015
v = 124.88r –1 + 19.12 The terminal velocity is about 146.4 ft/s.
When r = 20, v = 5.0362… . d. The table shows the values of v and the
Because the graph is concave up (convex side errors, Euler minus actual. The errors increase
down), the Euler’s solution underestimates only for a while, then approach zero because
the actual velocity. The first increment, where both the Euler’s solution and the actual
the graph is so steep, makes a large error that solution approach the same asymptote. (It is
accumulates as the iterations continue, not always true that values farther from the

Calculus Solutions Manual Problem Set 7-5 153


© 2005 Key Curriculum Press
initial condition have a greater error in their 12. Answers will vary.
Euler’s approximation.)

t Euler’s v Actual v Error Problem Set 7-6


Q1. definition of definite integral
2 61.6831… 60.4791… 1.2040…
Q2. fundamental theorem of calculus
4 106.2850… 103.3298… 2.9552…
Q3. definition of indefinite integral
6 129.7139… 126.8383… 2.8756…
Q4. the intermediate value theorem
8 139.9323… 137.9573… 1.9749…
Q5. Rolle’s theorem
10 143.9730… 142.8466… 1.1264…
Q6. the mean value theorem
20 146.4066… 146.3792… 0.0274…
Q7. the chain rule Q8. general
The graph in part a shows the Euler’s Q9. particular Q10. initial
solutions from parts a and b, and the actual
1. a. dB/dt is proportional to B, which means that
solution from part c, thus confirming
the larger the population is, the faster it
graphically the numerical answers to this
grows. But dB/dt is also proportional to
problem.
(30 − B)/30, which means that the closer B is
11. a. For x ≤ 5, the radicand 25 − x2 is non- to 30, the slower it grows. dB/dt > 0 when
negative, giving a real-number answer for y. 0 < B < 30 because when the population is
For x > 5, the radicand is negative, giving no less than 30 million the population will
real solution. increase until it reaches the carrying capacity.
b. The slope field shows that the graph will be dB/dt < 0 when B > 30 because when the
concave up (convex side down), making the population is greater than 30 million, the
Euler’s tangent lines lie below the graph, population will decrease until it reaches the
leading to an underestimate. carrying capacity.
At x = 4.9, y = −0.6 25 – 4.9 2 = −0.5969… . b.
B
The Euler’s solution at x = 4.9 is
40

−0.8390… , which is an underestimate


because −0.8390… < −0.5969… but is 30

reasonably close to the actual value. Actual


20
c. The Euler’s solutions for the given points are
Euler
x y 10

5.1 −0.3425… t
10 20 30 40
5.2 0.1935…
5.3 −0.7736… For the initial condition (0, 3), the population
grows, leveling off at B = 30. For the initial
6.6 26.9706… condition (10, 40), the population drops
(9)(5.1) because it is starting out above the maximum
From 5.1 to 5.2, dy = − (0.1) sustainable value (carrying capacity).
(25)(–0.3425K)
= 0.5360… , indicating that the graph is still c.
taking upward steps. t B
(9)(5.2)
From 5.2 to 5.3, dy = − (0.1) 0 3
(25)(0.1935K)
= −0.9672… , indicating that the graph takes 10 13.8721…
a relatively large downward step. The sign 20 26.4049…
change in dy happens whenever the prior 30 29.5565…
Euler’s y-value changes sign. The graph starts 40 29.9510…
over on another ellipse representing a different
particular solution. See the graph in part b. The graph shows that
d. Euler’s method can predict values that are the Euler’s points and graphical solution are
outside the domain, which are inaccurate. close to each other.

154 Problem Set 7-6 Calculus Solutions Manual


© 2005 Key Curriculum Press
d.
dB
= 0.21B ⋅
30 – B ln |y| − ln |120 − y| = 0.9x + C
dt 30 −ln |y| + ln |120 − y| = −0.9x − C
30 120 – y
dB = 0.21 dt = C1e −0.9 x C1 = ± e − C
B(30 – B) y
Separate the variables. 115
= C 1e0 ⇒ C 1 = 23
 +
1 1  dB = 0.21 dt 5
 B 30 – B  Substitute the initial
By partial fractions condition (0, 5) to
(see Example 1). find C1.
ln |B| − ln |30 − B| = 0.21t + C 120 – y
Why the “−” sign? = 23e −0.9 x
y
−ln |B| + ln |30 − B| = −0.21t − C
To simplify later 120 − y = 23 ye −0.9 x
steps. 120
y=
30 – B
= C1e −0.21t 1 + 23e –0.9 x
B The graph confirms that the particular
C1 = ± e − C solution follows the slope lines.
27
= C1e 0 ⇒ C1 = 9 y (houses)
3
Substitute the initial 100

condition (0, 3) to
find C1.
30 – B
= 9e −0.21t
B 50
30 − B = 9 Be −0.21t
30 Solve for B explicitly
B =
1 + 9e −0.21t in terms of t.
30 x (years)
At t = 20, B = = 26.4326 K .
1 + 9e –4.2 5 10

The Euler’s value, B ≈ 26.4049… , is very


close to this precise value. c. 70% of 120 is 84.
120
d  dB  84 = ⇒ x ≈ 4.4253K , or about
e. = −0.014 B + 30(0.007) 1 + 23e –0.9 x
dB  dt  4 years 5 months Solve numerically
Derivative = 0 if −0.014B + 30(0.007) = 0, for x.
which is true if and only if B = 15. 1 lot left means 119 lots built on.
This value is halfway between B = 0 and 120
B = 30. 119 = ⇒ x ≈ 8.7940 … , or about
30 1 + 23e –0.9 x
15 = ⇒ t ≈ 10.4629… 8 years 10 months.
1 + 9e –0.21t
dy 120 – y 0.9
The point of inflection is (10.4629… , 15). d. = 0.9 y ⋅ = (120 y – y 2 )
dx 120 120
2. a. A logistic function is reasonable because the d  dy  0.9
number of houses grows at an increasing rate = (120 – 2 y)
dy  dx  120
for a while, then slows down as the number
approaches 120, the “carrying capacity” of the The derivative is zero if 120 − 2y = 0, which
subdivision. is true if and only if y = 60. This value is
halfway between y = 0 and y = 120.
dy 120 – y If y < 60, the derivative is positive, so dy/dx
b. = 0.9 y ⋅
dx 120 is increasing. If y > 60, the derivative is
120 negative, so dy/dx is decreasing. Therefore,
dy = 0.9 dx dy/dx is a maximum when y = 60, and the
y(120 – y)
number of houses is increasing the most
1 1  rapidly at this point of inflection.
 +  dy = 0.9 dx
 y 120 – y 

Calculus Solutions Manual Problem Set 7-6 155


© 2005 Key Curriculum Press
dy M–y 178
3. a. = ky ⋅ 89 =
dx M 1 + 16.8e −0.05297Kx
k k 0.5 Solving numerically gives x ≈ 53.2574… ,
0.5 = (10)( M – 10) ⇒ = or on the 54th day.
M M 10( M – 10)
dy M−y
k
1.1 = (24)( M – 24) ⇒
k
=
1.1 4. = ky ⋅
M M 24( M – 24) dx M

∫ ∫
M dy
0.5 1.1 = k dx
∴ = y( M − y )
10( M − 10) 24( M − 24) 1 1 
Eliminate k by ∫ +
 y M − y
 dy = k dx ∫
equating the two
See Section 9-7 for a
values of k/M.
quick way to resolve
12(M − 24) = 11.0(M − 10)
into partial fractions.
12M − 11M = 288 − 110 ⇒ M = 178
ln |y| − ln |M − y| = kx + C
Solve for M.
The differential of the
k 0.5 89 second denominator
= ⇒k= =
178 10(178 − 10) 1680 is −dy.
0.05297… (Store this.) −ln |y| + ln |M − y| = −kx − C
Ajax expects to sell 178,000 CDs based on M−y
ln = − kx − C
this mathematical model. y
dy 89 178 − y M−y
b. = ⋅y⋅ = e − kx −C = e − C ⋅ e − kx
dx 1680 178 y
M−y
= ± e − C ⋅ e − kx = C1e − kx C1 = ± e − C
y (thousand CDs)
200
178 y
M − y = C 1ye − kx
y + C 1ye− kx = M
M M
y= − kx =
1 + ae − kx
100
1 + C1e
a = C 1, Q .E .D .
5. a. At t = 5.5, F ≈ 1.7869… ≈ 2 fish left.
x (days) At t = 5.6, F ≈ −11.0738… , meaning no
50 100 fish are left.
The fish are predicted to become extinct in
The slope field has horizontal slope lines at
just over 5.5 years.
about y = 178, thus confirming M = 178.
F (fish)
M
c. The general solution is y = . Part b
1 + ae − kx 1000
Substitute M = 178 and k = 89/1680 =
0.05297… and the initial condition y = 10 at
x = 0.
178 Parts c and d
10 = ⇒ a = 16.8 500
1 + ae 0
178
The equation is y =
Part a, dt = 0.1
. 200
1 + 16.8e −0.05297Kx t (years)
See the graph in part b. The graph follows the 5 10
slope lines.
d. At x = 50, y = 81.3396… . b. See the graph in part a with initial condition
At x = 51, y = 83.6844… . (3, 1200), showing that the fish population
83.6844… − 81.3396… = 2.35447… will decrease because the initial condition is
They expect to sell about 2354 CDs on the above the 1000 maximum sustainable.
51st day. c. See the graph in part a with initial condition
e. The point of inflection is halfway between (0, 300), showing that the population rises
y = 0 and y = 178, that is, at y = 89. slowly at first, then faster, eventually

156 Problem Set 7-6 Calculus Solutions Manual


© 2005 Key Curriculum Press
slowing down as the population approaches The logistic function fits reasonably well (as
the 1000 maximum sustainable (carrying shown in this graph), especially if you use
capacity). several values of the maximum number of
d. Let F = y + 200. people as shown in the table.
dy y 800 − y
= 130 ⋅ ⋅
N
25
dt 200 1000 20

∫ ∫
200000 15
dy = 130 dt
y(800 − y) 10
5 x
 250 250 
∫
 y
+  dy = 130 dt
800 − y  ∫ 1 2 3 4 5 6 7 8

See Section 9-7 for 7. a. and b.


quick partial fractions.
Year P ∆P/∆t (∆P/∆t)/P
250 ln |y| − 250 ln |800 − y| = 130t + C
Why “−” ? 1940 131.7
800 − y 1950 151.4 2.38 0.01571…
= e − (13/25)t ⋅ e − C
y 1960 179.3 2.59 0.01444…
800 − y 1970 203.2 2.36 0.01161…
= C1e − (13/25)t
y 1980 226.5 2.275 0.01004…
Substitute y = 100 (F = 300) when t = 0. 1990 248.7
800 − 100
= C1e 0 ⇒ C1 = 7 You can’t find ∆P/∆t for 1940 and 1990
100
800 − y because you don’t know values of P both
= 7e − (13/25)t before and after these values.
y
c. Using linear regression on the values of
800
y= (∆P/∆t)/P without round-off gives
1 + 7e − (13/25)t
1 ∆P
800 ≈ 0.02802596 … − 0.0000792747… P.
F= + 200 P ∆t
1 + 7e − (13/25)t
The correlation coefficient is r = −0.98535… .
See the graph in part a, showing that the For the other types of regression:
sketch from part c reasonably approximates r = −0.978… for logarithmic
this precise algebraic solution. r = −0.981… for exponential
6. Answers will vary. Here is a typical run with a r = −0.971… for power
class of 25 people. Thus, a linear function fits best because r is
closest to −1.
x N
1 dP
d. ≈ 0.02802596 … − 0.0000792747… P
0 1 P dt
1 3 dP
⇒ = P(0.02802596 … − 0.0000792747… P)
2 6 dt
3 13 e.
4 21 500
P

5 25
6 25 Stable population at 353.5 million

7 25
8 25

Logistic regression gives


t
25.5083K –50 0 50 100
N=
1 + 43.1120 K e −1.3032Kx

Calculus Solutions Manual Problem Set 7-6 157


© 2005 Key Curriculum Press
f. h. See the graph in part e. Data do follow the
solution.
Year t Euler Actual* Euler** i. Sample answer: The predicted populations
1890 −50 44.6… 62.9 46.1… agree fairly well with the data for the six
given years. The fit is exact for 1940 because
1900 −40 56.9… 76.0 58.3…
this point was used as an initial condition.
1910 −30 71.7… 92.0 72.9… For the other five years, the predicted
1920 −20 89.2… 105.7 90.1… populations are a bit higher than the actual
1930 −10 109.3… 122.8 109.8… population.
1940 0 131.7 131.7 131.7 j. Actual data are given in the table in part f.
1950 10 155.4… 151.4 155.0… k. The predicted population for 2010 from part f
is 286.1… million. Using 486.1 million as
1960 20 180.1… 179.3 179.2…
an initial condition in 2010 gives the
1970 30 204.7… 203.2 203.5… following predictions:
1980 40 228.2… 226.5 226.9…
Year t Euler
1990 50 249.9… 248.7 248.8…
2000 60 269.3… 281.4 268.6 2010 70 486.1…
2010 70 286.1… 2020 80 444.5…
2020 80 300.2… 2030 90 417.7…
2030 90 311.8… 2040 100 399.7…
2040 100 321.1… 2050 110 387.1…
*Data from The World Book Encyclopedia .
The logistic model predicts that the
**Note that although linear regression gives the
population will drop, approaching the
“best” fit for (∆ P/∆ t ) /P versus P, actually plotting
the graph shows that the data point for 1960 is ultimate value of 353.5 million from above.
considerably out of line. This behavior shows up in the slope field of
part e because the slopes are negative for
(∆ P/∆ t)/P
1950
populations above 353.5.
0.015
1960 1 A B A(10 – y) + By
0.014
8. a. = + =
0.013 y(10 – y) y 10 – y y(10 – y)
0.012
1970
0.011 The numerator of the first fraction must
1980 P
150 200
equal that of the last fraction for all values
of y. That is, 1 = 10A − Ay + By. The
Using the two endpoints, 1950 and 1980, constant and linear coefficients on the left
gives (∆P/∆t)/P = 0.002716… − must equal the corresponding ones on the
0.00007557…P. Using this equation gives right. Thus, 1 = 10A and 0 = −Ay + By.
populations much closer to the actual ones So A = B = 0.1.
for the given years, as shown in the  0.1 0.1 
∫ y(10 – y) dy = ∫ 
1
rightmost column of the table in part f. This b. +  dy
y 10 – y 
is, of course, no guarantee that the later
1 1 
model fits any better in the future than the
former one. ∫
= 0.1  −  dy, which equals 3 dx.
 y y – 10  ∫
g. The population growth rate is zero ∴ 0.1(ln |y| − ln |y − 10|) = 3x + C
if dP/dt = 0. y y – 10
Let ln = − ln = 30 x + 10C
y – 10 y
P(0.02802596… − 0.0000792747…P) = 0.
P = 0 or P = (0.02802596…)/ y – 10 10
= 1− = e − ( 30 x +10 C )
(0.0000792747…) = 353.5… y y
Predicted ultimate population ≈ 353.5 million
10
Differential equation: P = 353.5… makes = 1 ± e −10 C e −30 x
dP/dt = 0. y
Graph: P = 353.5… is a horizontal 10
y= , where k = ± e −10 C , Q .E.D .
asymptote. 1 + ke –30 x

158 Problem Set 7-6 Calculus Solutions Manual


© 2005 Key Curriculum Press
dP dF dF/dt – k2 F + k4 RF
c. = P(0.02802 … − 0.00007927… P) 12. = =
dt dR dR/dt k1 R – k3 RF
= 0.00007927…P(353.5… − P) The dt cancels out.
∫ ∫
1
dP = 0.00007927K dt 13. R = 70, F = 15
P(353.5K – P)
dF –15 + 0.025 ⋅ 1050 11.25
1 −  dP ⇒ = = = 0.4017K

1 1 70 – 0.04 ⋅ 1050
= dR 28
353.5K  P P – 353.5K
14. The slope at (70, 15) is about 0.4.
= 0.00007927K ∫ dt F
ln |P| − ln |P − 353.5…|
= 353.5…(0.00007927…t + C)
50
P P – 353.5K
ln = − ln
P – 353.5K P
= 0.02802…t + 353.5…C
P – 353.5K
= e − ( 0.02802…t +353.5…C ) (70, 15)
R
P
353.5K 100
= 1 + ke 0.02802…t
P dR dF
353.5K At R = 70, F = 15, = 28, and = 11.25,
P= dt dt
1 + ke –0.02802...t which are both positive. So both populations are
For the initial condition t = 0, P = 131.7, increasing and the graph starts up and to the
353.5K right.
k= − 1 = 1.684 … .
131.7 15. The populations vary periodically and the graph
d. is cyclical. The fox population reaches its
Year t Algebraic Euler Actual maximum 1/4 cycle after the rabbit population
reaches its maximum.
1940 0 131.7 131.7 131.7
16. Neither population changes when dR/dt =
1950 10 155.5… 155.4… 151.4 dF/dt = 0.
1960 20 180.2… 180.1… 179.3 dF/dt = 0 ⇔ F = 0 or R = 1/0.025 =
1970 30 204.7… 204.7… 203.2 40 (4000 rabbits)
dR/dt = 0 ⇔ R = 0 or F = 1/0.04 = 25 foxes
1980 40 228.2… 228.2… 226.5
17. Assume that dF/dt still equals −F + 0.025RF.
1990 50 249.8… 249.9… 248.7
dF dF/dt – F + 0.025 RF
= =
The two methods of evaluating the dR dR/dt R – 0.04 RF – 0.01R 2
mathematical model agree almost perfectly. dF 11.25
However, the fact that they agree with each R = 70 and F = 15 ⇒ = = −0.5357K
dR −21
other is no guarantee that they will fit the real 18.
world as closely as they match each other. F

dR dR
9. = k1 R ⇒ = k1dt ⇒ ln | R| = k1t + C
dt R 50
⇒ | R| = e e ⇒ R = C1e k1t
C k1t

R is increasing because k1 > 0.


dF dF
10. = − k2 F ⇒ = − k2 dt (70, 15)
dt F R
⇒ ln | F| = − k2 t + C
100
⇒ | F| = e C e − k2t ⇒ F = C2 e − k2t
Note that the slope at (70, 15) is now negative.
F is decreasing because −k2 < 0.
19. The populations now spiral to a fixed point. The
dR
11. = k1 R − k3 RF rabbit population stabilizes at the same value as
dt in Problem 16, R = 40 (4000 rabbits), which is
dF surprising. The stable fox population decreases
= − k2 F + k4 RF
dt from 25 to 15.

Calculus Solutions Manual Problem Set 7-6 159


© 2005 Key Curriculum Press
20. Assume that dF/dt still equals = −F + 0.025RF.
∫V ∫
dV
b. = k dt
dF dF/dt – F + 0.025 RF
= = ln |V| = kt + C
dR dR/dt R – 0.04 RF – 0.01R 2 – 10
dF 11.25 | V | = e kt +C = e C ⋅ e kt
R = 70 and F = 15 ⇒ = = −0.3629K
dR −31 V = C 1e kt
21. C1 can be positive or negative, so the
F absolute value sign is not needed for V. In the
real world, V is positive, which also makes
50
the absolute value sign unnecessary.
c. 400 = Ce k·0 ⇒ C = 400
(70, 30) 500 = 400e k⋅ 40
ln 1.25
(70, 15) ⇒k= = 0.005578…
R 40
100 V = 400e0.005578…t
d. 750 = 400e0.005578…t
Note that the slope at (70, 15) is about −0.4. ln 1.875
⇒t = = 112.68… ≈ 113 s
22. The fox and rabbit populations spiral toward a 0.005578...
fixed point. Again, and even more surprisingly,
∫y ∫
−1/ 2
the rabbits stabilize at R = 40 (4000). But the R3. a. dy = 6 dx ⇒ y = (3 x + C )2
stable fox population is reduced to 8 or 9. Along
the way, the model shows that the foxes are b. y = (3x − 4) (y = (3x − 14)2 does not work
2

reduced to about 1, thus becoming in danger of because at (3, 5), dy/dx = −30 but 6y1/2 = 30.)
extinction! c.
23. See the graph in Problem 21 with initial y (3, 25)
condition (70, 30). With this many foxes and
hunters chasing rabbits, the rabbits become
extinct. At this point, the foxes have been
10
reduced to just 5. After the rabbits become
extinct, the foxes decrease exponentially x
with time, eventually becoming extinct 1 2 3

themselves.
d. At x = 2, y′ = 12 and y = 4.
See graph in part c.
Problem Set 7-7 A line through (2, 4) with slope 12 is tangent
Review Problems to the graph.
R0. Answers will vary. e. i. dN/dt = 100 − kN

∫ ∫
R1. P(t) = 35(0.98 t ) dN
= dt
P′(t) = 35(0.98t) ln 0.98 100 – kN
−(1/k) ln |100 − kN| = t + C
t P(t) P′(t) P′(t)/P(t) Using (0, 0) gives −(1/k) ln 100 = C.
Substituting this value for C gives
0 35 −0.7070… −0.2020… −(1/k) ln |100 − kN| = t − (1/k) ln 100.
10 28.5975… −0.5777… −0.2020… ln |100 − kN| − ln 100 = −kt
20 23.3662… −0.4720… −0.2020… ln |1 − (k/100)N| = −kt
1 − ( k / 100) N = e − kt
P′(t ) 35(0.98t ) ln 0.98 N = (100 / k )(1 − e − kt )
= = ln 0.98 Using (7, 600) and solving numerically
P( t ) 35(0.98t )
= −0.2020… , which is a constant, Q.E.D. gives k ≈ 0.045236.
∴ N = 2210.6…(1 − e− 0.045236 t)
R2. a. V = speed in mi/h; t = time in s
ii. t = 30: About 1642 names
dV
= kV
dt

160 Problem Set 7-7 Calculus Solutions Manual


© 2005 Key Curriculum Press
iii. lim N = 2210.6 …(1 − 0) = 2210.6 … Table with initial condition (1, 9), ∆x = 1:
t →∞
The brain saturates at about 2211 names. x y (∆x = 1) y (∆x = 0.1)
iv. Let dN/dt = 30. 1 9 9
70
30 = 100 − kN ⇒ N = = 1547.4 … 2 7.227… 7.707…
k 3 6.205… 6.949…
names. Substituting this for N gives
4 5.441… 6.413…
1547.4 K = 2210.6 K(1 − e −0.045236 t ).
5 4.794… 5.999…
1547.4 K
e −0.045236 t = 1 − = 0.3 (exactly) 6 4.200… 5.662…
2210.6 K
ln 0.3 7 3.616… 5.377…
t= = 26.6 … ≈ 27 days 8 3.007… 5.130…
–0.04523K
or: 9 2.326… 4.910…
30 = N(t) − N(t − 1) 10 1.488… 4.712…
= 2210.6 K[ − e −0.045236 t + e −0.045236( t −1) ] 11 0.2185… 4.529…
12 −8.091… 4.359…
= 2210.6 Ke −0.045236 t ( t −1) ( − e −0.045236 + 1)
13 4.199…
⇒ t ≈ 27 days
14 4.045…
dy 20
R4. a. =− + 0.05 y 15 3.896…
dx xy 16 3.750…
At (2, 5), dy/dx = −1.75.
17 3.604…
At (10, 16), dy/dx = 0.675.
The slopes at (2, 5) and (10, 16) agree with 18 3.457…
these numbers. 19 3.306…
b. Initial conditions (1, 8) and (1, 12) 20 3.150…
M M
y
28.9 0.1344…
(10, 16) 29 −0.3810…
y
(1, 12)

(1, 10)

(1, 8)

5
(1, 9)
(2, 5)

x
∆x = 0.1
5 5
∆x = 1

The solution containing (1, 8) crosses the


x
x-axis near x = 7, converges asymptotically 5
to the y-axis as x approaches zero, and is
symmetric across the x-axis. The solution For ∆x = 1, the graph crosses the x-axis at
containing (1, 12) goes to infinity as x goes about x = 11.
to infinity. b. See the table in part a for ∆x = 0.1.
c. See the graph in part b with initial condition See the graph in part a.
(1, 10). The solution containing (1, 10) c. The accuracy far away from the initial
behaves more like the one containing condition is very sensitive to the size of the
(1, 12), although a slight discrepancy in increment. For instance, in part a the first
plotting may make it seem to go the step takes the graph so far down that it
other way. crosses the x-axis before running off the
edge of the grid. The greater accuracy with
dy 20 ∆x = 0.1 shows that the graph actually does
R5. a. =− + 0.05 y
dx xy not cross the x-axis before x = 20.

Calculus Solutions Manual Problem Set 7-7 161


© 2005 Key Curriculum Press
d. Continuing the computations in part c, the (Note that the general solution to the differ-
graph crosses the x-axis close to x = 28.9. ential equation is (x − 6)2 + 2(y − 7)2 = C,
See the table in part a. and the specific solution for the given initial
R6. a. condition is (x − 6)2 + 2(y − 7)2 = 0, whose
graph is a single point.)
y (hundred beavers)
e. Initial condition (9, 7)
10
y

5 (6, 7) (9, 7) (15, 7) (19, 7)

x (years) x
5 10 5

Suddenly there are too many predators for the


The population is decreasing because it is
number of prey, so the yak population
above the maximum sustainable, 900 beavers
declines. Because y is decreasing from (9, 7),
(y = 9). By Euler’s method, y ≈ 9.3598… , or
the graph follows a clockwise path.
about 936 beavers, at x = 3 years.
f. See the graph in part e with initial condition
b. See the graph in part a with initial condition
(19, 7). The graph crosses the x-axis at
(3, 100), showing that the population is
x ≈ 14.4, indicating that the yaks are hunted
expected to increase slowly, then more
to extinction. (The Xaltos would then starve
rapidly, then more slowly again, leveling off
or become vegetarian!)
asymptotically toward 900. This happens
because the initial population of 100 is below g. See the graph in part e with initial condition
the maximum sustainable. (15, 7). The graph never crosses the x-axis,
but crosses the y-axis at y ≈ 2.3, indicating
dy 9– y
c. = 0.6 y ⋅ that the yak population becomes so sparse
dx 9 that the predators become extinct. (The yak
9 Substitute into the population would then explode!)
y=
1 + ae –0.6 x general equation.
9 Substitute the initial
1=
1 + ae –1.8 condition (3, 1). Concept Problems
a = 8e1.8 = 48.3971… Solve for a. dy
C1. a. = k ⋅ y1/2 ⇒ y −1/2 dy = k dx ⇒
9 9 dx
y= =
1 + 8e1.8e −0.6 x 1 + 48.3971K e –0.6 x 2 y1/2 = kx + C, so y = [0.5( kx + C )]2 .
The point of b. The differential equation would have to
inflection is halfway become y1/3 after it is integrated. So the
between the original equation would have to contain
asymptotes at y = 0 y −2 /3 after the variables have been separated.
and y = 9. dy
Conjecture: = ky 2/3
9 Substitute 4.5 for y. dx
4.5 =
1 + 8e1.8e −0.6 x c. Confirmation:
x = ln (8e1.8 )/0.6 = 6.4657… ≈ 6.5 yr dy
= ky 2 /3 ⇒ y −2 /3 dy = k dx ⇒
dx
dy –0.5( x – 6) 3y1/ 3 = kx + C ⇒ y = [(1/3)(kx + C)]3, a
d. =
dx ( y – 7) cubic function, Q.E.D.
dy = 0 when x = 6, and dx = 0 when y = 7. So dy
d. For n ≠ 0, = k ⋅ y ( n−1)/n ⇒
the stable point is (6, 7), corresponding to the dx
present population of 600 Xaltos natives and y − ( n−1)/n dy = k dx ⇒ ny1/n = kx + C ⇒
7000 yaks. y = [(1/n)(kx + C)]n

162 Problem Set 7-7 Calculus Solutions Manual


© 2005 Key Curriculum Press
dy Limit is 10, indicating maximum possible
For example: = y 7/8 ⇒ y −7/8 dy = dx
dx population.
⇒ 8 y1/8 = x + C ⇒ y = [(1/8)( x + C )]8 b. a = 421.3692… , c = 0.7303036… , and
C2. a. k = 0.01589546… , either by twice taking
logarithms as suggested, or by this method:
Ticket Price People
Taking ln once ⇒ ln a − ce− kt = ln P, so
2.00 460 ln a − ce10k = ln 179
2.50 360 ln a − c = ln 203
ln a − ce −10 k = ln 226
3.00 320 Then substituting ln a = c + ln 203 into the
4.00 260 first and third equations gives
4.50 140 c(1 − e10k) = ln 179 − ln 203
5.50 120 c(1 − e −10 k ) = ln 226 − ln 203
6.00 80 Substituting c(1 − e −10 k ) = c(e10 k − 1)e −10 k =
− e −10 k (ln 179 − ln 203) into the previous
b. equation yields
N
ln 226 − ln 203 ln 226 − ln 203
e −10 k = − =
500
ln 179 − ln 203 ln 203 − ln 179
1  ln 226 − ln 203 
so k = − ln   = 0.01589K .
10  ln 203 − ln 179 
Then find c using c(1 − e −10 k ) = ln 226 −
ln 203 and find a using 203 = ae − c .
P c = 0.7303… and a = 421.3692…
6
g (t )
431.3...

Function behaves (more or less) linearly.


Let N = number of tickets and P = number of
$/ticket. 100
By linear regression, N ≈ −90.83P + 605.4, t
100
with correlation coefficient r = −0.9747… .
c. Let M = total number of dollars. Note that this model predicts an ultimate
M ≈ P ⋅ N = P(−90.83P + 605.4) population of lim P(t ) ≈ 421 million.
t →∞
M ≈ −90.83P 2 + 605.4P
c. Now a = 551.1655… , c = 0.9988291… ,
d. Maximize M: M′ ≈ −181.66P + 605.4 k = 0.01186428… , and the ultimate
605.4 population is lim P(t ) ≈ 551 million. Thus,
M′ = 0 ⇔ P ≈ = 3.332… t →∞
181.66 the Gompertz model is quite sensitive to a
Maximum M at P ≈ 3.332… because M′ small change in initial conditions. The
changes from positive to negative there (or predicted ultimate population increased by
because the graph of M is a parabola opening 130 million with only a 1 million change in
downward). one data point!
Charge $3.30 or $3.35. C4. dV/dt = −2V1/ 2 + F, where F is a constant.

∫ ∫
dV
e. M has a local maximum at this price because = dt
charging more than the optimum price F − 2V 1/2
The integral on the right is not the integral of
reduces attendance enough to reduce the total
the reciprocal function because the numerator
amount made, whereas charging less than the
cannot be made the differential of the
optimum price increases attendance, but not
denominator. A slope field gives information
enough to make up for the lower price per
about the solutions. The following graph is for
ticket.
−0.5 t F = 20 ft3/min flowing in. (The dashed line
C3. a. g(t ) = 10e −0.8e shows the solution with F = 0, the original
The graph does look like Figure 7-7e. condition.) Starting with 196 ft3 in the tub, the
−0.5 t −0.5 t
lim 10e −0.8e = 10e limt →∞ e volume levels off near 100 ft3. Starting below
t →∞
100 ft3, the volume would increase toward 100.
= 10e −0.8⋅0 = 10

Calculus Solutions Manual Problem Set 7-7 163


© 2005 Key Curriculum Press
V underestimate. (Or: The convex side of the graph
is down, so the Euler’s tangent lines are below
196 F = 20
the actual graph.)
F=0
T6. General logistic differential equation:
dy M−y
14 t = ky ⋅
dx M
dy
If the inflow rate is too high, the tub will T7. = 0.4 y
overflow. The next graph is for F = 40 ft3/min. dx

∫ ∫
dy
In this case, the stable volume is above the = 0.4 dx
initial 196 ft3. y
V ln |y| = 0.4x + C
F = 40 |y| = eCe0.4 x
196
y = C1e0.4 x
−5 = C 1 e 0.4(0) = C 1
F=0 y = −5e0.4 x

∫ ∫
14 t dy
T8. = 12 y1/2 ⇒ y −1/2 dy = 12 dx ⇒
dx
It is possible to antidifferentiate the left side by 2 y1/ 2 = 12 x + C
the algebraic substitution method of Problem
dP
Set 9-11, Problems 101–106. The general T9. a. = kP ⇒ P = Ce kt
solution is dt
F P = 3000 at t = 0 ⇒ P = 3000ekt
t + C = − ln ( F − 2V 1/2 ) − V 1/2
2 b. P = 2300 at t = 5 ⇒
and the particular solution for V = 196 at t = 0 is 1 2300
k = ln = −0.05314 K ⇒
F F − 28 5 3000
t − 14 = − ln − V 1/2
2 F − 2V 1/2 P(25) = 794.6…
Unfortunately, it is difficult or impossible to Phoebe will not quite make it because the
solve for V. The volume will asymptotically pressure has dropped just below 800 psi by
approach F2/4, overflowing the tub if F2/4 > tub time t = 25.
capacity. or:
800 = 3000e −0.05314Kt
Chapter Test 1 800
t= ln = 24.87K
−0.05314 K 3000
dy
T1. = ky Phoebe will not quite make it because the
dx
pressure has dropped to 800 just before t = 25.
T2. Solving a differential equation means finding the
T10. a. y = number of grams of chlorine dissolved
equation of the function whose derivative appears
t = number of hours since chlorinator was
in the differential equation.
started
T3. The general solution involves an arbitrary dy
constant of integration, C. A particular solution = 30 − ky
dt
has C evaluated at a given initial condition.
∫ ∫
dy
= dt
T4. 30 – ky
y 1
5 − ln |30 − ky| = t + C
k
ln |30 − ky| = −kt + C 1
x 30 − ky = C 2e− k t
5 y = 0 when t = 0 ⇒ C 2 = 30
∴ ky = 30(1 − e− kt)
30
(0, –4)
y= (1 – e – kt )
k
The rate of escape is ky = 13 when y = 100.
T5. The concave side of the graph is up, so the So k = 0.13.
30
actual graph curves up from the Euler’s tangent ∴y = (1 – e –0.13t ) = 230.7K(1 − e −0.13t )
lines, making the Euler’s method values an 0.13

164 Problem Set 7-7 Calculus Solutions Manual


© 2005 Key Curriculum Press
b. 200 = 230.7…(1 − e −0.13t ) The graph starts going downward and to the
200 right from (80, 700) because the coyote
e −0.13t = 1 − = 0.1333K
230.7K population is relatively high, thus decreasing
ln 0.1333K the number of roadrunners.
t= = 15.499K≈ 15.5 hr
–0.13 b. There can be two different values for the
dy 16 – y roadrunner population for a particular coyote
T11. a. = 0.5 y ⋅
dx 16 population because the two events happen at
16 16 two different times. For example, coyotes are
y= ⇒2= ⇒a=7
1 + ae –0.5 x 1 + ae 0 increasing from 80 when there are 700 road-
16 runners, but later they are decreasing from 80
y=
1 + 7e –0.5 x when there are about 200 roadrunners.
b. At x = 0, y = 2: T13. Answers will vary.
dy = 0.5(2)(16 − 2)/(16)(0.1) = 0.0875
At x = 0.1, y ≈ 2 + 0.0875 = 2.0875,
so dy = 0.5(2.0875)(16 − 2.0875)/(16)(0.1) = Problem Set 7-8
0.09075… .
At x = 0.2, y ≈ 2.0875 + 0.09075… = Cumulative Review, Chapters 1–7
2.17825… .
16 1.
The precise solution is y = =
1 + 7e –0.1 200
v(t)

2.18166… , which is greater than ( t , v ( t ))


2.17825… , as expected because the
graph is concave up (convex side downward).
16
c. 4 = ⇒ x = [ln (3/7)]/−0.5 = t
1 + 7e –0.5 x 8
1.6945…
About 1 month 21 days v(t) dt represents the distance traveled in time dt.
d.
y (hundred lilies) 2. Definite integral
8

∫ (t – 21t 2 + 100t + 80) dt


3
3.
0

1 4 8
= t − 7t 3 + 50t 2 + 80t 0
4
= 1280 mi
4. M 100 = 1280.0384
M1000 = 1280.000384
The Riemann sums seem to be approaching 1280
x (months)
as n increases. Thus, the 1280 that was found by
purely algebraic methods seems to give the
The graph shows that the number of lilies is correct value of the limit of the Riemann sum.
expected to decrease toward 1600 (y = 16) 5.
v(t)
because of overcrowding. 200
T12. a. ( t , v ( t ))

R (roadrunners)

t
(80, 700)
8

6. Any Riemann sum is bounded by the


corresponding lower and upper sums. That is,
L n ≤ R n ≤ U n.
By the definition of integrability, the limits of Ln
and Un are equal to each other and to the definite
C (coyotes)
integral. By the squeeze theorem, then, the limit
of Rn is also equal to the definite integral.

Calculus Solutions Manual Problem Set 7-8 165


© 2005 Key Curriculum Press
7. Definition: dz dx dy
18. z 2 = x 2 + y 2 ⇒ 2 z = 2x + 2y
b dt dt dt
∫a
f ( x ) dx = lim Ln = lim Un
∆x →0 ∆x →0
dz 1
= [0.3 x + 6e –0.5 x ⋅ (–0.9e –0.5 x )]
provided that the two limits are equal. dt z
Fundamental theorem: If f is integrable on At x = 2, z = 2 2 + 2.2072 K2 = 2.9786 … ,
[a, b] and g( x ) = f ( x ) dx, ∫ so
dz
=
1
dt 2.9786 K
(0.6 – 0.7308K) = −0.04391K .
b
then ∫ a
f ( x ) dx = g(b) − g( a). ∴ z is decreasing at about 0.044 unit per second.
dm
= km
x
Or: If F( x ) = ∫ a
f (t ) dt, then F ′( x ) = f ( x ). 19.
dt

∫ ∫
dm
8. Numerically, the integral equals 1280. By 20. = k dt ⇒ ln |m | = kt + C ⇒
m
counting, there are approximately 52 squares.
| m | = e kt +C ⇒ m = C1e kt
Thus, the integral ≈ 52(25)(1) = 1300.
v( 4.1) – v(3.9) (mi/min ) 21. Exponentially
9. v ′( 4) ≈ = −19.9
0.2 min 22. General
v ′( 4 ) ≈
v( 4.01) – v(3.99)
= −19.9999
(mi/min) 23. 10000 = C1e0 ⇒ C1 = 10000
0.02 min 10900 = 10000e k⋅1
f ( x ) – f (c ) ⇒ k = ln 1.09 ⇒ m = 10000e ln(1.09) t
10. f ′(c) = lim or
x →c x–c = 10000(1.09)t
f ( x + ∆x ) – f ( x )
f ′( x ) = lim 24. False. The rate of increase changes as the amount
∆x →0 ∆x
in the account increases. At t = 10,
11. v′(t) = 3t 2
− 42t 100 ⇒ v′(4) = −20
+ m = 10000(1.09)10 ≈ 23673.64.
12. Slowing down. v′(4) < 0 and v(4) = 208 > 0 ⇒ The amount of money would grow by
velocity is positive but decreasing ⇒ speed is $13,673.64, not just $9,000.
42


slowing down. 25. By Simpson’s rule, y dx
13. The line has slope −20, and passes through 30
2
(4, 208). The line is tangent to the graph. ≈ (74 + 4 ⋅ 77 + 2 ⋅ 83 + 4 ⋅ 88 + 2 ⋅ 90
3
+ 4 ⋅ 91 + 89) = 1022.
v(t)
5
200
Slope
= –20 26. By symmetric difference quotient, at x = 36
–100
90 – 83
y′ ≈ = 1.75.
2( 2 )
100
27. If f is differentiable on (a, b) and continuous at
x = a and x = b, and if f ( a) = f ( b) = 0, then there
t is a number x = c in (a, b) such that f ′ (c) = 0.
0 5 10
28.
f(x)
14. Acceleration tangent
f (b )
15. At a maximum of v(t), v′(t) will equal zero.
42 ± 42 2 – 4 ⋅ 3 ⋅ 100
3t2 − 42t + 100 = 0 ⇔ t = f (a )
secant
6 x
a c b
t = 3.041… or 10.958…
So the maximum is not at exactly t = 3. 29.
16. v″(t) = 6t − 42 f(x) and f´(x)

dx dy dz
17. Know: = 0.3. Want: and . 5
dt dt dt
dy dx
y = 6e −0.5 x ⇒ = −3e −0.5 x ⇒ = −0.9e −0.5 x f f´
dt dt x
dy
At x = 2, = −0.9e −1 = −0.3310 K . 0 5 10
dt f´

y is decreasing at about 0.33 unit per second.

166 Problem Set 7-8 Calculus Solutions Manual


© 2005 Key Curriculum Press
30.
∫ y dy = 0.25 ∫ x dx ⇒ 0.5y = 0.125 x 2 + C1
2
f(x)
36.
⇒ x 2 − 4y 2 = C
Initial condition: (10, 4)
2 100 − 64 = C ⇒ C = 36
x
∴ x 2 − 4 y 2 = 36 ⇒ y = ±0.5 x 2 – 36
1
37. x = 10.5: y = 0.5 10.52 – 36 = 4.30842 …
Step discontinuity at x = 1. dy 10
38. At (10, 4), = 0.25 ⋅ = 0.625.
31. g(x) = x1/ 3(x − 1) dx 4
1 1
g′( x ) = x 13/ + x −2/3 ( x − 1) = x −2/3 ( 4 x − 1) Using ∆x = 0.5, y(10.5) ≈ 4 + (0.625)(0.5) =
3 3 4.3125, which is close to the exact value of
g′(0) is undefined because 0 −2 /3 takes on the form 4.30842… .
1 / 0 2 /3 or 1/0. d 3x 2
39. (sin –1 x 3 ) =
g (x ) dx 1 – x6
1 40. x = ln (cos t) and y = sec t
1
x dx/dt = ⋅ ( − sin t ) = − tan t
–1 1
cos t
dy/dt = sec t tan t
dy dy/dt sec t tan t
= = = − sec t = − y
32. e0.2 x = 0.6x ⇒ x ≈ 3.0953… (Store as a.) dx dx/dt – tan t
Or x ≈ 7.5606… (Store as b.)

dx 1
dA = (0.6x − e0.2 x) dx 41. = − ln |4 − 3 x | + C
b 4 – 3x 3
A= ∫ (0.6 x – e ) dx ≈ 0.8787K
0.2x
a
42. h( x ) = 5 = e x⋅ln 5 ⇒ h′( x ) = ln 5 e 5⋅ln 5 = 5 x ln 5
x

(Integrate numerically.) sin 5 x + cos 3 x – 5 x – 1 0


43. lim →
33. dV = π [(e0.2 x)2 − (0.6x)2] dx x→0 x2 0
a 5 cos 5 x – 3 sin 3 x – 5 0
V =π ∫ (e – 0.36 x 2 ) dx ≈ 8.0554K = lim →
0.4x
0 x→0 2x 0
(Integrate numerically.) –25 sin 5 x – 9 cos 3 x
= lim = −4.5
dy x x→0 2
34. = 0.25
dx y sin 5 x + cos 3 x – 5 x – 1
44. y = , showing a
Initial conditions: (0, 3) and (10, 4) x2
removable discontinuity at (0, −4.5).
y
y
5
5
(10, 4) x
(0, 3)
x –1 1
(0, –4.5)
5 –5

45. Answers will vary.


35. See the graph in Problem 34. Any initial
condition for which y = 0.5x, such as (2, 1), 46. Answers will vary.
gives the asymptote.

Calculus Solutions Manual Problem Set 7-8 167


© 2005 Key Curriculum Press
Chapter 8—The Calculus of Plane and Solid Figures

Problem Set 8-1 Problem Set 8-2


1. f (x) = x 3 − 6x 2 + 9x + 3 Q1. Q2.
f ′ (x) = 3x 2 − 12x + 9 y y
y 1
x
f´ f
1
1
x
1
x
1 2 3 4 Q3. Q4.
y y
1
g (x) = x 3 − 6x 2 + 15x − 9 x
1
x
g ′ (x) = 3x 2 − 12x + 15 1 1

y
f

Q5. Q6.
y y
x
1 2 3 4 1
x
1
1 x

h (x) = x − 6x + 12x − 3
3 2 1

h ′ (x) = 3x 2 − 12x + 12 Q7. Q8.


y y y
f f´
1 1
x x
1 1
x
1 2 3 4

Q9. Q10.
Positive derivative ⇒ increasing function y y

Negative derivative ⇒ decreasing function 1 x 1


x
Zero derivative ⇒ function could be at a high 1 2
point or a low point, but not always.
2. The functions have vertex points at values of x
where their derivatives change sign. If the 1.
derivative is never zero, as for function g, the max.
function graph has no vertex points. If the f (x )
f´(x) + 0 –
derivative is zero but does not change sign, as
x 2
for function h, the function graph just levels
off, then continues in the same direction, with f (x )
no p.i.

no vertex. f´´(x) – – –

3. g ′′ (x) = (d/dx)(3x2 − 12x + 15) = 6x − 12 x 2

h ′′ (x) = (d/dx)(3x2 − 12x + 12) = 6x − 12 2.


All the second derivatives are the same!
min.
4. The curves are concave up where the second f ( x)
derivative is positive and concave down where the f' ( x) – 0 +
x 2
second derivative is negative.
no p.i.
5. Points of inflection occur where the first
f(x )
derivative graph reaches a minimum. f"(x) + 0 +
Points of inflection occur where the second x 2
derivative graph crosses the x-axis.

168 Problem Set 8-2 Calculus Solutions Manual


© 2005 Key Curriculum Press
3. 9.
plateau no max./min.
f (x ) f (x )
f´(x) + 0 + f´(x) – undef. –
x 2 x 2

p.i. no p.i.
f (x ) f (x )
f´´(x) – 0 + f´´(x) – undef. +
x 2 x 2

4. 10.
max. no max.min.
f (x) f (x )
f' (x) + undef. – f'(x ) + undef. –

x 2 x 2

no p.i. no p.i.
f (x ) f (x )
f"(x ) + undef. + f"(x ) + undef. +

x 2 x 2

5. 11.
min. max. min. max.
f (x )
f (x )
f´(x) – undef. +
f´(x) + 0 – 0 + 0 –
x 2
x –2 1 3
no p.i.
f (x )
p.i. p.i.
f´´(x) 0 undef. 0
f (x )
x 2
f´´(x) – 0 + 0 –
6. x –1 2

no max./min.
f (x ) f (x )
f'(x ) – undef. –
x 2

p.i.
f (x )
f"(x ) – undef. + x
x 2 –2 –1 1 2 3

7. 12.
no max./min. min. plateau max.
f (x )
f(x )
f´(x) –
f'(x) – 0 + 0 + 0 –
x 2
x –3 –1 3
p.i.
f (x )
f´´(x) + – p.i. p.i. p.i.

x 2 f (x )
f"(x) + 0 – 0 + 0 –
8.
x –2 –1 2
no max./.min.
f (x )
f (x )
f'(x ) –
x 2

p.i.
f (x )
f"(x) – 0 +
x
x 2
–3 –2 –1 2 3

Calculus Solutions Manual Problem Set 8-2 169


© 2005 Key Curriculum Press
13. 16.
max. plateau min. max. min. max.

f (x ) f (x )
f´(x) + ∞ – 0 – f' (x ) e.p. + 0 – 0 + e.p.
x –2 1 x 1 2 6 7

no p.i. p.i. no p.i.

f (x ) f(x)

f´´(x) + ∞ + 0 – f"(x) e.p. – 0 zero 0 + e.p.


x –2 1 x 1 3 5 7

f (x ) f (x )

x x
–2 1 1 2 3 5 6 7

17.
14. 3
y

max. none max. f´

f(x )
f´ x
f'(x) – 0 + ∞ + 0 –
3 6
x 2 3 4
f f´

p.i. –3
f (x )
f"(x ) + ∞ –
18.
y
4
x 3

f (x ) f f' f'
x
4 8
f'

x
–4
2 3 4
19.
y
15.
4
min. max. min. f
f (x )
f´(x) e.p. + 0 zero 0 – e.p.
f´ f´
x –1 1 3 5
x
4 8
no p.i.

f (x )
f´´(x) e.p. – 0 zero 0 – e.p.

x –1 1 3 5
20.
y
f (x )
4

2 f

f' x
x
4
–1 1 3 5

170 Problem Set 8-2 Calculus Solutions Manual


© 2005 Key Curriculum Press
21. f (x) = 3ex − xex f (x )

f ′ (x) = 3ex − ex − xex = ex (2 − x)


1
f ′ (2) = e2 (2 − 2) = 0 ⇒ critical point at x = 2 x

f ′′ (x) = 2ex − ex − xex = ex (1 − x) 2

f ′′ (2) = e2 (1 − 2) = −7.3890... < 0


∴ local maximum at x = 2
f (x )
The graph confirms a maximum at x = 2.

5
25. f (x) = (x − 2)3 + 1
f ′ (x) = 3(x − 2)2
x f ′ (2) = 3(2 − 2)2 = 0 ⇒ critical point at x = 2
1 2 3 f ″ (x) = 6(x − 2)
f ″ (2) = 6(2 − 2) = 0, so the test fails.
The graph confirms a maximum at x = 2. f ′ (x) goes from positive to positive as
x increases through 2, so there is a plateau
π
22. f ( x ) = − sin x at x = 2.
4
π π f (x )
f ′( x ) = − cos x
4 4
π π
f ′(2) = − cos (2) = 0 ⇒ critical point
4 4 1
x
at x = 2 2
π2 π
f ′′( x ) = sin x
16 4
The graph confirms a plateau at x = 2.
π 2
π
f ′′(2) = sin (2) = 0.6168K > 0 26. f (x) = (2 − x)4 + 1
16 4
f ′ (x) = −4(2 − x)3
∴ local minimum at x = 2
f ′ (2) = −4(2 − 2)3 = 0 ⇒ critical point at x = 2
f (x ) f ″ (x) = 12(2 − x)2
1
f ″ (2) = 12(2 − 2)2 = 0, so the test fails.
x f ′ (x) changes from negative to positive as
2 x increases through 2, so there is a local
–1
minimum at x = 2.

f (x )

The graph confirms a minimum at x = 2.


23. f (x) = (2 − x)2 + 1
f ′ (x) = −2(2 − x) 1 x
f ′ (2) = −2(2 − 2) = 0 ⇒ critical point at x = 2 2
f ″ (x) = 2 ⇒ f ″ (2) = 2 > 0
∴ local minimum at x = 2
The graph confirms a minimum at x = 2.
f (x )
27. a. f (x) = 6x5 − 10x3
f ′ (x) = 30x4 − 30x2 = 30x2(x + 1)(x − 1)
f ′ (x) = 0 ⇔ x = −1, 0, or 1 (critical points
1
x for f (x))
2
f ′′( x ) = 120 x 3 − 60 x = 60 x ( 2 x + 1)( 2 x − 1)
f ″ (x) = 0 ⇔ x = 0, ± 1/2 (critical points for
The graph confirms a minimum at x = 2.
f ′ (x))
24. f (x) = −(x − 2)2 + 1
b. The graph begins after the f-critical point at
f ′ (x) = −2(x − 2)
x = −1; the f ′ -critical point at x = − 1/2 is
f ′ (2) = −2(2 − 2) = 0 ⇒ critical point at x = 2
f ″ (x) = −2 ⇒ f ″ (2) = −2 < 0 shown, but is hard to see.
∴ local maximum at x = 2 c. f ′ (x) is negative for both x < 0 and x > 0.

Calculus Solutions Manual Problem Set 8-2 171


© 2005 Key Curriculum Press
28. a. f (x) = 0.1x 4 − 3.2x + 7 c. There is no inflection point at x = 0 because
f ′ (x) = 0.4x 3 − 3.2 = 0.4(x − 2)(x 2 + 2x + 4) concavity is down for both sides, but there is
x2 + 2x + 4 has discriminant = 22 − 4 · 4 < 0, an inflection point at x = 1.
so f ′ (x) = 0 ⇔ x = 2 (critical point for f (x)).
32. a. f ( x ) = x 1.2 − 3 x 0.2
f ″ (x) = 1.2x2
f ″ (x) = 0 ⇔ x = 0 (critical point for f ′ (x)) f ′( x ) = 1.2 x 0.2 − 0.6 x −0.8 = 0.6 x −0.8 (2 x − 1)
f ′ (x) = 0 ⇔ x = 0.5, and f ′ (x) is undefined
b. f ″ (x) does not change sign at x = 0.
at x = 0 (critical points for f (x)).
( f ″ (x) ≥ 0 for all x)
f ′′( x ) = 0.24 x −0.8 + 0.48 x −1.8 = 0.24 x −1.8 ( x + 2)
c. f ″ (c) = 0, but f ′ (c) ≠ 0
f ″ (x) = 0 ⇔ x = −2 (critical point for f ′ (x);
29. a. f ( x ) = xe − x f ′ (0) is undefined, so f ′ has no critical point
f ′( x ) = − xe − x + e − x = e − x (1 − x ) at x = 0).
f ′ (x) = 0 ⇔ x = 1 (critical point for f (x)) b. f (0) = 01.2 − 3 ⋅ 00.2 = 0 has only one value.
f ′′( x ) = xe − x − 2e − x = e − x ( x − 2) c. Curved concave up because f ′′(x) > 0 for
f ″ (x) = 0 ⇔ x = 2 (critical point for f ′ (x)) x < −2
b. Because f (x) approaches its horizontal 33. a. f (x) = −x 3 + 5x 2 − 6x + 7
asymptote (y = 0) from above, the graph must
f (x )
be concave up for large x; but the graph is
concave down near x = 1, and the graph is 7
smooth; somewhere the concavity must
change from down to up.
c. No. e − x ≠ 0 for all x, so xe − x = 0 ⇔ x = 0. x
1
30. a. f (x) = x 2 ln x
f ′ (x) = x + 2x ln x = x (1 + 2 ln x)
Maximum (2.5, 7.6), minimum (0.8, 4.9),
f (x) and f ′ (x) are undefined at x = 0, so
points of inflection (1.7, 6.3)
f ′( x ) = 0 ⇔ ln x = −0.5 ⇔ x = e −0.5 =
No global maximum or minimum
0.6065… (critical point for f (x)).
f ″ (x) = 3 + 2 ln x b. f ′(x) = −3x 2 + 10x − 6
f ′′( x ) = 0 ⇔ ln x = −1.5 ⇔ x = e −1.5 = 1
f ′( x ) = 0 ⇔ x = (5 ± 7 ) = 2.5485K or
0.2231… (critical point for f ′ (x)). 3
0.7847…
ln x 1/ x
b. lim+ x 2 ln x = lim+ = lim 5
x→0 x→0 x –2 x → 0 + – 2 x – 3 f ′′( x ) = −6 x + 10; f ′′( x ) = 0 ⇔ x = =
3
= lim+ − 0.5 x 2 = 0 by L’Hospital’s rule. 1.666…
x→0

lim x 2 ln x does not exist because x2 ln x is c. f ′′(0.7847…) = −6(0.7847…) + 10 =


x→0 − 5.2915… > 0, confirming local minimum.
undefined for x < 0.
d. Critical and inflection points occur only
c. All critical points from part a appear, where f, f ′, or f ′′ is undefined (no such points
although the inflection point at x = e −1.5 is exist) or is zero (all such points are found
hard to see on the graph. above).
31. a. f (x) = x 5/3 + 5x 2/3 34. a. f (x) = x 3 − 7x 2 + 9x + 10
5 10 5
f ′( x ) = x 2/3 + x −1/3 = x −1/3 ( x + 2) f (x )
3 3 3
f ′ (x) = 0 ⇔ x = −2, and f ′ (x) is undefined
at x = 0 (critical points for f (x)). 10

10 −1/3 10 −4/3 10 −4/3


f ′′( x ) = x − x = x ( x − 1) x
9 9 9 1
f ″ (x) = 0 ⇔ x = 1 (critical point for f ′(x);
f ′ (0) is undefined, so f ′ has no critical point
at x = 0). Maximum (0.8, 13.2), minimum (3.9, −2.1),
b. The y-axis (x = 0) is a tangent line because points of inflection (2.3, 5.6)
the slope approaches −∞ from both sides. No global maximum or minimum

172 Problem Set 8-2 Calculus Solutions Manual


© 2005 Key Curriculum Press
b. f ′(x) = 3x 2 − 14x + 9 b. f ′(x) = 5(x − 1)4
1 f ′(x) = 0 ⇔ x = 1; f ′(x) is undefined ⇔
f ′( x ) = 0 at x = (7 ± 22 ) = 3.896 K or
3 x = −1, 3.
0.769… f ′′(x) = 20(x − 1)3;
7
f ′′( x ) = 6 x − 14; f ′′( x ) = 0 at x = = 2.333K f ′′(x) = 0 ⇔ x = 1; f ′′(x) is undefined ⇔
3 x = −1, 3.
c. f ′′(0.769…) = 6(0.769…) − 14 = c. f ′′(1) = 20(1 − 1)3 = 0, so the test fails.
−9.3808… < 0, confirming local maximum. d. Critical and inflection points occur only
d. Critical and inflection points occur only where f, f ′, or f ′′ is undefined (only at
where f, f ′, or f ′′ is undefined (no such points endpoints) or is zero (all such points are found
exist) or is zero (all such points are found above).
above). 37. f (x) = ax3 + bx2 + cx + d; f ′(x) = 3ax2 + 2bx + c;
35. a. f (x) = 3x 4 + 8x 3 − 6x 2 − 24x + 37, f ′′(x) = 6ax + 2b ⇒ f ′′(x) = 0 at x = −b/(3a)
x ∈ [−3, 2] Because the equation for f ′′(x) is a line with
f (x ) nonzero slope, f ′′(x) changes sign at x = −b/(3a),
80 so there is a point of inflection at x = −b/(3a).
38. f (x) may not have a local maximum or
minimum (if f ′(x) is never zero); if this is not
x
the case, then the maximum and minimum occur
–3 2
where f ′(x) = 3ax2 + 2bx + c = 0, at
–2 b ± 4b 2 – 4 ⋅ 3a ⋅ c – b b 2 – 3ac
Maximum (−3, 82), (−1, 50), (2, 77), x= = ± ,
minimum (−2, 45), (1, 18), points of 6a 3a 3a
inflection (−1.5, 45.7), (0.2, 32.0) and the maximum and minimum occur at
Global maximum at (−3, 82) and global b 2 – 3ac /(3a) units on either side of the
minimum at (1, 18) inflection point −b/(3a) (see Problem 33).
b. f ′(x) = 12x 3 + 24x 2 − 12x − 24 39. f (x) = ax3 + bx2 + cx + d
= 12(x + 2)(x − 1)(x + 1) f ′(x) = 3ax2 + 2bx + c; f ′′(x) = 6ax + 2b
f ′(x) = 0 ⇔ x = −2, −1, 1 Points of inflection at (2, 3) ⇒ f ′′(3) = 0 ⇒
f ′(x) is undefined ⇔ x = −3, 2. 18a + 2b = 0
f ′′(x) = 36x 2 + 48x − 12 = 12(3x 2 + 4x − 1); Maximum at (5, 10) ⇒ f ′(5) = 0 ⇒ 75a + 10b +
1 c=0
f ′′( x ) = 0 ⇔ x = − (2 ± 7 ) = 0.2152 …
3 (3, 2) and (5, 10) are on the graph ⇒
or −1.5485… 27a + 9b + 3c + d = 2.
f ′′(x) is undefined ⇔ x = −3, 2. 125a + 25b + 5c + d = 10
c. f ′′(−2) = 12[3(4) + 4(−2) − 1] = 36 > 0, Solving this system of equations yields
confirming local minimum. 1 9 15 5
f ( x) = − x 3 + x 2 − x − .
d. Critical and inflection points occur only 2 2 2 2
where f, f ′, or f ′′ is undefined (only at f (x ) (5, 10)
endpoints) or is zero (all such points are found
above). 5

f (x) = (x − 1)5 + 4, x ∈ [−1, 3]


(3, 2)
36. a. x
3 5
f (x )

10
x
The graph confirms maximum (5, 10) and points
–1 1 3
of inflection (3, 2).
40. f (x) = ax3 + bx2 + cx + d
f ′(x) = 3ax2 + 2bx + c; f ′′(x) = 6ax + 2b
Maximum (3, 36), minimum (−1, −28), Points of inflection at (2, 7) ⇒ f ′′(2) = 0 ⇒
plateau and points of inflection (1, 4) 12a + 2b = 0
Global maximum at (3, 36) and global Maximum at (−1, 61) ⇒ f ′(−1) = 0 ⇒
minimum at (−1, −28) 3a − 2b + c = 0

Calculus Solutions Manual Problem Set 8-2 173


© 2005 Key Curriculum Press
(2, 7) and (−1, 61) are on the graph ⇒ d.
8a + 4b + 2c + d = 7. f (x )

−a + b − c + d = 61
Solving this system of equations yields
f (x) = x 3 − 6x 2 − 15x + 53.
f (x )
80 x
c

x
e.
–1 2
f ( x)

(Locally
The graph confirms maximum (−1, 61) and constant)

points of inflection (2, 7).


41. a. f (x) = x 3 ⇒ f ′(x) = 3x 2 c
x

f ′(−0.8) = 1.92
f ′(−0.5) = 0.75
f ′(0.5) = 0.75 44. f (x) = 10(x − 1)4/3 + 2
f ′(0.8) = 1.92 f (1) = 2, so f (1) is defined.
b. The slope seems to be decreasing from −0.8 40
f ′( x ) = ( x – 1)1/ 3
to −0.5; f ″(x) = 6x < 0 on −0.8 ≤ x ≤ −0.5, 3
which confirms that the slope decreases. The f ′(1) = 0, so f is differentiable at x = 1.
40
slope seems to be increasing from 0.5 to 0.8; f ″( x ) = ( x – 1) −2 / 3
f ″(x) = 6x > 0 on 0.5 ≤ x ≤ 0.8, which 9
40 40
confirms that the slope increases. f ″(1) has the form (0 –2 / 3 ) or (1/0) , so
c. The curve lies above the tangent line. 9 9
f ″(1) is infinite.
42. Ima could notice that y ′ = 0 at x = 0 There seems to be a cusp at (1, 2), but zooming
(or y ′ = 3 at x = ±1), so the graph could not in on this point reveals that the tangent is
possibly be a straight line with slope = 1. actually horizontal there.
43. a.
f (x ) f (x )

x
x
1
c

See Problem 20 in Problem Set 10-6 for


b. calculation of curvature.
f (x ) 45. f (x) = e0.06 x, f ′(x) = 0.06e0.06 x,
f ″(x) = 0.0036e0.06 x
g (x) = 1 + 0.06x + 0.0018x2 + 0.000036x3
g ′(x) = 0.06 + 0.0036x + 0.000108x2
g ″(x) = 0.0036 + 0.000216x
c
x
f (0) = 1 and g (0) = 1
f ′(0) = 0.06 and g ′(0) = 0.06
f ″(0) = 0.0036 and g ″(0) = 0.0036
c. (In fact, f ′″(0) = g ′″(0).)
f (x )
But f (10) = e0.6 = 1.822… ≠ g (10) = 1.816;
f ′(10) = 0.109… ≠ g ′(10) = 0.1068.
Because f (x) > 0 for all x, f has no x-intercept.
But g (0) = 1 and g (−100) = −23.
By the intermediate value theorem, g (x) = 0
c
x
somewhere between x = −100 and x = 0, meaning
that g does have an x-intercept.

174 Problem Set 8-2 Calculus Solutions Manual


© 2005 Key Curriculum Press
( x – 1)3 sin 1 + 2, if x ≠ 1 1. Let x = total width of pen, y = length of pen.
46. f ( x ) =  x –1 Domains: 0 ≤ x ≤ 300, 0 ≤ y ≤ 200
2, if x = 1 Maximize A(x) = xy.
2
2.01 f (x )
2 x + 3 y = 600 ⇒ y = 200 − x
3
2 2
2 ∴ A( x ) = 200 x − x
3
1.99 A (x)
1 max.

1
lim f ( x ) = lim ( x – 1)3 ⋅ sin + 2 = 2 = f (1)
x →1 x →1 x –1
(The limit of the first term is zero because x

(x − 1)3 approaches zero and the sine factor is 150 300

bounded.)
∴ f is continuous at x = 1. The graph shows a maximum at x ≈ 150.
f ( x ) – f (1) 4
f ′(1) = lim Algebraically, A′( x ) = 200 − x.
x →1 x –1 3
[( x – 1)3 sin(1/( x – 1))] + 2 – 2 A′(x) = 0 ⇔ x = 150, confirming the graph.
= lim 2
x →1 x –1 x = 150 ⇒ y = 200 − ⋅ 150 = 100
1 3
= lim ( x – 1) sin
2
=0 Make the total width 150 ft and length 100 ft.
x →1 x –1
(x − 1)2 → 0 and the sine factor is bounded. (Note: The maximum area was not asked for.)
∴ f ′(1) = 0 2. a. Let x = width of a room across the front,
y = depth of a room from front to back.
2.001
Domains: x ≥ 0, y ≥ 0
Minimize P (x) = 12x + 7y.
xy = 350 ⇒ y = 350x − 1
2

∴ P (x) = 12x + 2450x− 1


1.999
0.9 1 1.1
P (x )

The graph is zoomed in by a factor of 10 both


ways. The graph does appear to be locally linear
at x = 2. Although the sine factor makes an
infinite number of cycles in any neighborhood of x

x = 1, the (x − 1)3 factor approaches zero so 14.28...

rapidly that the graph is “flattened out.” The


name pathological is used to describe the fact that The graph shows a minimum at x ≈ 14.
the graph makes an infinite number of cycles in a Algebraically, P ′ (x) = 12 − 2450x− 2.
bounded neighborhood of x = 1. P ′ (x) = 0 ⇔ 2450x− 2 = 12 ⇔
47. Answers will vary. x = ± 2450/12 = ±35/ 6 = ±14.288…
Minimum is at x = 35/ 6 , y = 10 6 =
Problem Set 8-3
24.49… .
Q1. y′ = −3(3 x + 5) −2 Q2. ln |x + 6| + C Make rooms 14.3 ft across and 24.5 ft deep.
2 b. For 10 rooms, P (x) = 20x + 11y =
Q3. − x −5/ 3 Q4. 3x 1/3 + C
3 20x + 3850x− 1.
Q5. − x −1 + C Q6. x + C P ′ (x) = 20 − 3850x− 2 = 0 at x = 192.5
Q7. ln |sin x| + C
Minimum at x = 192.5 = 13.87… ,
Q8. Q9.
1
y y" y = 350/ 192.5 = 25.22 …
x
1
x Make rooms 13.9 ft across and 25.2 ft deep.
1 2 For 3 rooms, P (x) = 6x + 4y = 6x + 1400x− 1.
P ′(x) = 6 − 1400x− 2 = 0 at x = 1400/6 =
Q10. D 10 7/3

Calculus Solutions Manual Problem Set 8-3 175


© 2005 Key Curriculum Press
Minimum at x = 10 7/3 = 15.27… , A(159.154…) = 79577.471…
Minimum area at r = 70.012… ,
y = 5 21 = 22.91…
s = 1000/(4 + π) = 140.024…
Make rooms 15.3 ft across and 22.9 ft deep. For square, 4(140.024…) ≈ 560.
3. a. Let x = width of rectangle, 2x = length of For circle, 2π(70.012…) ≈ 440.
rectangle, y = width of square. Use 440 yd for square and 560 yd for circle.
A rect = 2x 2, A sq = y 2 (You could build a square corral with side 140
For minimal rectangle, 2x2 ≥ 800 ⇒ x ≥ 20. around the circular fence of radius 70 to
For minimal square, y2 ≥ 100 ⇒ y ≥ 10. enclose a total area of only 19,607 yd2 , but
Perimeter P = 6x + 4y = 600 ⇒ Big Bill might not like your solution!)
y = 150 − 1.5x b. The graph of A versus r shows that the
∴ 150 − 1.5x ≥ 10 ⇒ x ≤ 140/1.5 = maximum area occurs at the largest possible
93.3333… circle. Big Bill should use all 1000 yards for
Domain: 20 ≤ x ≤ 93.3333… the circular fence and not build a corral.
b. Total area A(x) = 2x2 + y2 5. a. Let x = length of square base, z = height
= 2x2 + (150 − 1.5x)2 of box.
= 22500 − 450x + 4.25x2
Domain of x: 0 ≤ x ≤ 120 = 10.954 …
A(x) Maximize V(x) = x2z.
20,000
Area = x2 + 4xz = 120 ⇒ z = 30/x − x/4
∴ V(x) = 30x − x3/4
V max.
x
20 93.3

c. The graph shows a maximum at endpoint x


x = 93.3333… . 0 6.32... 10.95...
A′(x) = −450 + 8.5x
A′(x) = 0 ⇔ x = 450/8.5 = 52.9411… The graph shows a maximum at x ≈ 6.3.
Because A(52.9…) is a minimum, the V′(x) = 30 − 3x 2/4 = 0 at x = ± 40
maximum occurs at an endpoint.
A(20) = 15200, A(93.3333…) = x = − 40 is out of the domain.
17522.2222… Critical points at x = 0, x = 40 , x = 120
Greatest area ≈ 17,522 ft2 V (0) = 0, V ( 120 ) = 0
4. a. Let r = radius of circle, s = width of square V( 40 ) = 20 40 = 126.49…
Diameter ≥ 50 ⇒ r ≥ 25 Maximum at x = 40 = 6.324 … ,
Circumference ≤ 1000 ⇒ 2πr ≤ 1000 ⇒
r ≤ 500/π z = 40 /2 = 3.162 …
Domain of r: 25 ≤ r ≤ 500/π = 159.154… Make the box 6.32 cm square by 3.16 cm
Minimize A(r) = πr2 + s2. deep.
Perimeter 2πr + 4s = 1000 ⇒ s = 250 − πr/2 b. Conjecture: An open box with square base of
∴ A(r) = πr2 + (250 − πr/2)2 side length x and fixed surface area A will
A have maximal volume when the base length
max.
is twice the height, which occurs when
x = A/3 (see the solution to Problem 8b).
min. 6. a. Domain of x is 0 ≤ x ≤ 6.
r
b. V(0) = 0 cm2
25 70 160
V(1) = 180 cm2
V(2) = 256 cm2 (largest volume for an integer
The graph shows minimum area at x ≈ 70.
value of x)
A′(r) = 2πr + 2(250 − πr/2)(−π/2)
V(3) = 252 cm2
A′(r) = 0 ⇔ 2πr − π(250 − πr/2) = 0 ⇒
V(4) = 192 cm2
r = 500/(4 + π) = 70.012…
V(5) = 100 cm2
A(25) = 46370.667…
V(6) = 0 cm2
A(70.012…) = 35006.197…

176 Problem Set 8-3 Calculus Solutions Manual


© 2005 Key Curriculum Press
c. V(x) = (20 − 2x)(12 − 2x)x A – 2 z(–2 z + 4 z 2 + A )
= 240x − 64x 2 + 4x 3 y=
–2 z + 4 z 2 + A + 2 z
V (x )
= −2 z + 4 z 2 + A
200
Therefore, x = y for maximum volume,
Q .E .D .
x b. Let x = y. Maximize V = xyz = x 2z.
6 Fixed area A = xy + 2xz + 2yz = x2 + 4xz
⇒ z = A/(4x) − x/4
The graph shows a maximum at x ≈ 2.4. ∴ V = (A/4)x − x3/4
V′(x) = 240 − 128x + 12x2 = 0 at
dV
x = (128 ± 4864 )/24 = 2.427… or 8.239… = ( A/4) − 3 x 2 /4 = 0 at x = ± A/3
dx
x = 8.239… is out of the domain. dV/dx goes from positive to negative at
V(2.427…) = 262.68… is a maximum x = A/3 ⇒ maximum at x = A/3.
because it is positive and V(0) = V(6) = 0.
Maximum volume ≈ 262.7 cm2 at z = A/( 4 A/3 ) − A/3 /4 =
1
A/3 = x
1
x ≈ 2.43 cm 2 2
7. Let x = length, y = depth, C(x) = total cost. c. For the maximal box in part b, the depth is
Domains: x > 0, y > 0 half the length of the base. Thus, the box is
Area of bottom = 5x short and fat. This makes sense because the
Total area of sides is (10 + 2x)y. problem is equivalent to maximizing the
Minimize C(x) = 10(5x) + 5(10 + 2x)y. volume of two open boxes with the second
Volume = 72 ⇒ 5xy = 72 ⇒ box placed upside-down on the first. The
y = 72/(5 x ) = 14.4 x −1 resulting single closed box will have
maximum volume when it is a cube, which
∴ C( x ) = 50 x + 5(10 + 2 x )(14.4 x −1 )
will happen if each open box is half a cube.
C( x ) = 50 x + 720 x −1 + 144
9. For y = ex, minimize D( x ) = x 2 + y 2 =
C

x 2 + e2 x .

D (x)

x
3.794... 1

The graph shows a minimum at x ≈ 3.8.


x
–0.4263...
C′(x) = 50 − 720x− 2 = 0 ⇔ x = ± 72/5 =
±3.7947…
The graph shows a minimum at x ≈ −0.43.
x = −3.7947… is out of the domain.
Minimum is at x = 3.7947 because C′(x) 1 2
D′( x ) = ( x + e 2 x ) −1/2 (2 x + 2e 2 x )
changes from negative to positive there. 2
C(3.7947…) = 120 10 + 144 ≈ 523.47 D′(x) = 0 ⇔ 2x + 2e2x = 0 ⇔ x = −e2x
Minimum cost is $523.47. Because x appears both algebraically and
8. a. Maximize V = xyz. exponentially, there is no analytic solution.
Fixed area A = xy + 2xz + 2yz Solving numerically gives x ≈ −0.4263. By
graphing D(x), D(−0.4263) is a minimum.
⇒ y = (A − 2xz)/(x + 2z)
Closest point to the origin is
Axz – 2 x 2 z 2 (x, y) = (−0.4263… , 0.6529…).
∴V =
x + 2z 10. Minimize A(r) = πr2 + 2rx, r ≥ 20.
dV –2 z 2 x 2 – 8z 3 x + 2 Az 2 2π r + 2x = 400 ⇒ x = 200 − π r
=
dx ( x + 2 z )2 x ≥ 100 ⇒ r ≤ 100/π
dV ∴ domain is 20 ≤ r ≤ 100/π.
= 0 at x = −2 z + 4 z 2 + A A(r) = π r2 + 2r(200 − π r) = 400r − π r2
dx

Calculus Solutions Manual Problem Set 8-3 177


© 2005 Key Curriculum Press
A (r )
L( x ) = ( x + 7)2 + ( y + 5)2
10,000
Maximize L2(x) = (x + 7)2 + (y + 5)2.
Using similar triangles, y/7 = 5/x ⇒ y =
35/x.
r ∴ L2(x) = (x + 7)2 + (35/x + 5)2
20 31.83... L 2(x) = x 2 + 14x + 49 + 1225/x 2 + 350/x + 25
The graph shows a minimum at endpoint x = 20. L
A′ = 400 − 2π r
A′ = 0 ⇔ r = 200/π = 63.6… (out of domain)
20

A′ > 0 for all r in the domain.


∴ minimum occurs at left end of domain, r = 20.
x
x = 200 − 20π = 137.168… 5.59...
Make radius of semicircles 20 m and straight
sections 137.17 m.
The graph shows a minimum of L(x) at x ≈ 5.6.
11. ( L2 ( x ))′ = 2 x + 14 − 350 x −2 − 2450 x −3
By numerical solution, (L2)′ = 0 at x ≈ 5.5934… .
(Exact answer is x = 3 175 .)
But a minimum distance L in the hall implies
L y
8 that the maximal ladder that will go through the
hall is at x = 5.5934… .
x–1 1
L2(5.5934…) = 285.3222…
x
L(5.5934…) = 16.8914…
No ladder longer than 16.8 ft (rounded down) can
L( x ) = x 2 + y 2 .
pass through the hall.
Domains: x ≥ 1, y ≥ 8 13. Let r = radius, h = height.
Minimize L 2(x) = x2 + y2. V = π r2h
Using similar triangles, =
y 8
⇒y=
8x
. 2r + 2h = 1200 ⇒ h = 600 − r
x x –1 x –1 ∴ V = πr2(600 − r) = π(600r2 − r3)
64 x 2
∴ L2(x) = x2 + V
( x – 1)2
L

10
r
400

x
1 5 The graph shows a maximum at r ≈ 400.
V′ = π(1200r − 3r2)
The graph shows a minimum of L(x) at x ≈ 5. V′ = 0 ⇔ r = 0 or r = 400
128 x From graph, maximum is at r = 400.
(L2)′(x) = 2x −
( x – 1)3 h = 600 − 400 = 200
128 x Maximum volume occurs with rectangle
(L2)′(x) = 0 ⇔ 2 x = ⇔ 400 mm wide (radius), 200 mm high.
( x – 1)3
x = 0 (out of domain) or (x − 1)3 = 64 ⇔ x = 5 14. Rotating a square does not give the maximum
volume. The solution to Problem 13 gives a
By graph, L(x) is a minimum at x = 5.
counterexample. Repeating the calculations with
Shortest ladder has length L(5) = 5 5 ≈ 11.18 ft. perimeter P instead of 1200 gives r = (1/3)P and
12. Let x and y be the segments shown. h = (1/6)P, showing that the proportions for
maximum volume are with radius twice the
L
y height.
15. a. Let r = radius, h = height.
5
7 x
V = πr2h = π(3.652)(10.6) = 141.2185π
= 443.6510… cm3

178 Problem Set 8-3 Calculus Solutions Manual


© 2005 Key Curriculum Press
b. A = 2πrh + 2πr2 C ″ (r ) = 16k + 564.874πr −3 > 0 for all r > 0,
V = πr2h = 141.2185π ⇒ h = 141.2185/r2 so this is a local minimum.
∴ A = 2πr(141.2185/r2) + 2πr2 If the normal can is the cheapest to make,
A = 2π (141.2185r −1 + r 2 ) 282.437π 282.437π
then 3.65 = 3 ⇒k=
c. A
16k 16(3.65)3
= 1.1404… .
To minimize the area (not the cost) of the
can, minimize 8r 2 + 2πrh = 8r 2 + 282.437πr −1.
500

C ′(r ) = 16r – 282.437πr −2 = 0 ⇒


r
282.437π
4.13... r=3 = 3.8126 cm
16
The graph shows a minimum at x ≈ 4.1. 141.2185
h= 2 = 9.7099 K cm.
A′ = 2π ( −141.2185r −2 + 2 r )
A′ = 2π/r2(−141.2185 + 2r3)
(
3 282.437π /16
)
A′ = 0 ⇔ r3 = 70.60925 ⇒ The proportions of this can are closer to those
of the normal can.
r = 3 70.60925 = 4.1332 … c. If the metal for the ends can be cut without
Minimum at r = 4.1… because A′ goes from waste, then it takes π(r + 0.6)2 to make each
negative to positive. end and (2πr + 0.5)h to make the sides, so
h = 141.2815/(3 70.60925 )2 = 23 70.60925 minimize
= 8.2664… C(r) = 2π(r + 0.6)2 + (2πr + 0.5)h
Radius ≈ 4.1 cm, height ≈ 8.3 cm = 2π (r + 0.6)2 + 141.2185(2πr + 0.5)r −2
Because height = 2 × radius, height = diameter. C ′(r ) = 4π (r + 0.6) − 282.437πr −2
So minimal can is neither tall and narrow nor − 141.2185r −3
short and wide. C′(r) = 0 at r ≈ 3.9966 by graphing calculator.
d. Normally proportioned can is taller and C ″ (r ) = 4π + 564.874πr −3 + 423.6555r −4 > 0
narrower than minimal can. For normal can, for all r > 0, so this is a minimum point.
A = 2π(3.65)(10.6) + 2π(3.65)2 = Minimal can has r ≈ 3.9966… ,
326.8041… . h ≈ 8.8411… cm.
For minimal can, A = 2π(4.13…)(8.26…) + But if the metal for the ends is cut from
2π(4.13…)2 = 322.014… . squares, then it takes 4(r + 0.6)2 to make each
Difference is 4.78… cm2. end and (2πr + 0.5)h to make the sides, so
Percent: (4.789…)(100)/326.80… = 1.465… minimize:
≈ 1.5% of metal in normal can C(r) = 8(r + 0.6)2 + (2πr + 0.5)h
e. Savings = (0.06)(20 × 106)(0.01465…)(365) = = 8(r + 0.6)2 + 141.2185(2πr + 0.5)r −2
6.419… × 106, or about $6.4 million! C ′(r ) = 16(r + 0.6) − 282.437πr −2
− 141.2185r −3
16. a. C(r ) = 2πr 2 k + 2πrh = 2πr 2 k + 282.437πr −1
C′(r) = 0 at r ≈ 3.6776… by graphing
C ′(r ) = 4πrk − 282.437πr −2 calculator.
= 4πr −2 ( kr 3 − 70.60925) C ′′(r ) = 16 + 564.874πr −3 + 423.6555r −4 > 0
C′(r) = 0 at r = 3 70.60925/k for all r > 0, so this is a minimum point.
Minimal can has r ≈ 3.6776… ,
C ″ (r ) = 4πk + 564.874πr −3 > 0 for all r > 0,
h ≈ 10.4411… .
so this is a local minimum. This is close to the normal can!
If the normal can is the cheapest to make,
17. a. Volume of cup = π(2.5)2 · 7 = 43.75π
then 3.65 = 3 70.60925/k ⇒ Let r = radius of cup, h = height of cup.
k = 70.60925(3.65) −3 = 1.4520 … . Minimize A(r) = πr2 + 2πrh.
This is reasonable because metal for the ends πr 2 h = 43.75π ⇒ h = 43.75r −2
is cut into circles, so some must be wasted. ∴ A(r ) = πr 2 + 87.5πr −1
b. Now it takes (2r)2 cm2 of metal to make each A

end of the can, so the function to minimize is


C(r ) = 8r 2 k + 2πrh = 8r 2 k + 282.437πr −1 .
C ′(r ) = 16rk − 282.437πr −2 100

282.437π r
C ′(r ) = 0 at r = 3 3.52...
16k

Calculus Solutions Manual Problem Set 8-3 179


© 2005 Key Curriculum Press
The graph shows a minimum at r ≈ 3.5 cm. A (x )

A′(r ) = 2πr − 87.5πr −2 = 2πr −2 (r 3 − 43.75) 1


A′(r) = 0 at r = 3 43.75 = 3.5236… .
There is a minimum at x = 3.5236… because
A(r) goes from decreasing to increasing. x
(See graph.) 1

h = 43.75( 43.75) −2/3 = 3 43.75 = r


The graph shows a maximum at x ≈ 0.86.
Minimal cup has r ≈ 3.52 cm, h ≈ 3.52 cm. A′(x) = 2 cos x − 2x sin x
b. Ratio is d : h = 2r : h = 2 : 1. A′(x) = 0 when x = cot x.
c. Current cup design uses π(2.5)2 + π · 5 · 7 = Solving numerically gives x ≈ 0.8603… .
41.25π = 129.59… cm2 = 0.012959… m2 per A(0) = A(π/2) = 0; A(0.8603…) = 1.1221…
cup, which costs Maximum area = 1.1221…
(300,000,000)(0.012959…)(2.00) 20.
≈ $7,775,441.82 per year. 200
Minimal cup design uses 3π(43.75)2/3 =
117.01… cm2 = 0.011701… m2 per cup, 50 x y

which costs (300,000,000)(0.011701…)(2.00)


Street
≈ $7,021,141.88 per year.
Switching to minimal cup design would Let x = width of store, y = length of store.
save 754,299.93 ≈ $754,000 per year in Minimize C(x) = 100x + 80(x + 2y).
paper costs (about 10% of the current annual xy = 4000 ⇒ y = 4000 x −1
paper bill), but would likely result in loss of C( x ) = 180 x + 640000 x −1
sales because a cup of that shape is hard to y ≤ 200 ⇒ x ≥ 20, so domain of x is
drink from. 20 ≤ x ≤ 50.
d. Let r = radius of cup, h = height of cup. Graph shows minimum at x endpoint x = 50.
πr 2 h = V ⇒ h = (V /π )r −2 C (x )

Minimize A(r ) = πr 2 + 2πrh = πr 2 + 2Vr −1 .


50,000

A′(r ) = 2πr − 2Vr −2 = 0 at r = 3 V /π


A′′(r ) = 2π + 4Vr −3 > 0 for all r > 0, so this is
a minimum. x

Minimal cup has r = 3 V/π , 20 50

h = (V /π )(V /π ) −2/3 = 3 V /π = r. C ′( x ) = 180 − 640000 x −2 = 0


18. a. A = yz = (30 + 0.2x)(40 − 0.2x) at x =
80 5
= 59.628… , outside the domain.
A(x) = 1200 + 2x − 0.04x2 3
Left rectangle: A(0) = 1200 in.2 C(20) = $35,600.00; C(50) = $21,800.00
Right rectangle: A(100) = 1000 in.2 Minimum cost is at x = 50, y = 4000/50 = 80.
b. A(80) = 1104 in.2 Bill should build the store 50 ft × 80 ft.
c. A (x ) 21. a. A = 0.5xy = 0.5x cot x
x 0
1000 lim A = lim →
x→0 x →0 2 tan x 0
1 1
= lim 2 =
x x →0 2 sec x 2
25 100 b. Domain of x is 0 < x ≤ π /2.
A (x )
The graph shows a maximum at x ≈ 25.
A′(x) = 2 − 0.08x = 0 at x = 25. 0.5

Critical points at x = 0, 25, 100


A(25) = 1225 in.2; A(0) = 1200 in.2;
A(100) = 1000 in.2 (from part a)
x
π/2
Maximum area at x = 25 in., minimum area for
x = 100 in. The graph shows that the area approaches a
19. Maximize A(x) = 2xy = 2x cos x. maximum as x approaches the endpoint x = 0
Use 0 ≤ x ≤ π/2 for the domain of x. from the positive side.

180 Problem Set 8-3 Calculus Solutions Manual


© 2005 Key Curriculum Press
1 The graph shows a maximum at x ≈ 1.
A′ ( x ) =
(cot x – x csc 2 x )
2 P′(x) = 4 − 4x = 0 at x = 1
A′(x) = 0 when x = cos x sin x or P(0) = 18; P(1) = 20; P(3) = 12
2x = 2 sin x cos x = sin 2x, Maximal rectangle has width = 2,
which happens at x = 0. length = 9 − 1 = 8.
A(π/2) = 0, so the “maximum” occurs at x = 0. c. No. The maximum-area rectangle is 2 3 by 6.
But x = 0 is not in the domain; A(x) can get The maximum-perimeter rectangle is 2 by 8.
arbitrarily close to 1/2, but never achieve it.
24. a. Maximize V(x) = π x2y = π x2(9 − x2) =
22. y 9 π x 2 − π x 4.
Domain: 0 ≤ x ≤ 3
(x, y)
V(x )

50
x
x 3

Domain of x is 0 ≤ x ≤ 3. x

Maximize A = 0.5(3 − x) ( y) = 0.5(3 − x)ex = 0 2.121... 3

1.5ex − 0.5xex.
The graph shows a maximum at x ≈ 2.1.
A (x )
4 V ′( x ) = 18πx − 4πx 3 = 0 at x = 0, ± 4.5 .
− 4.5 is out of the domain.
V (0) = V (3) = 0, V ( 4.5 ) = 20.25π =
63.6172…
x
0 2 3 Maximum is at x = 4.5 , y = 9 − 4.5 = 4.5.
Maximal cylinder has radius = 4.5 =
The graph shows a maximum at x ≈ 2. 2.12132… and height = 4.5.
A′(x) = 1.5ex − 0.5ex − 0.5xex = 0.5ex(2 − x)
b. Maximize L(x) = 2π xy = 2π x(9 − x2) =
A′(x) = 0 at x = 2, confirming the graph.
18π x − 2π x 3 .
A′(x) > 0 for x < 2, and A′(x) < 0 for x > 2,
confirming maximum point at x = 2. L (x )

Maximum area A(2) = e2/2 = 3.69452… . 50


23. a. Maximize A(x) = 2xy = 2x(9 − x2) =
18x − 2x 3.
Domain: 0 ≤ x ≤ 3 x
0 1.732... 3
A(x )
20
The graph shows a maximum at x ≈ 1.7.
L ′( x ) = 18π − 6πx 2 = 0 at x = ± 3 .
− 3 is out of the domain.
x
L(0) = L(3) = 0; L( 3 ) = 12π 3 = 65.2967K
0 1.732 3
Maximum is at x = 3 , y = 9 − 3 = 6.
The graph shows a maximum at x ≈ 1.7. Maximal cylinder has radius = 3 =
A′( x ) = 18 − 6 x 2 = 0 at x = ± 3 = ±1.732… 1.7320… and height = 6.
−1.732 is out of the domain. c. Maximize A(x) = 2πx2 + 2π xy = 2π x 2 +
A(0) = A(3) = 0; A( 3) = 12 3 = 20.7846 K 2π x(9 − x 2) = 2π x 2 + 18π x − 2π x 3 .
Maximal rectangle has width = 2 3,
A ( x)
length = 9 − 3 = 6.
b. Maximize P(x) = 4x + 2y = 4x + 18 − 2x2. 50
P(x )
20
x
0 2.097... 3

x The graph shows a maximum at x ≈ 2.1.


0 1 3 A′(x) = 18π + 4π x − 6π x 2

Calculus Solutions Manual Problem Set 8-3 181


© 2005 Key Curriculum Press
1± 2 7 Maximal cylinder has radius = 8.1649… ,
A′(x) = 0 at x = = 2.0971K or 20 3
3 height = = 11.5470 K , and volume =
−1.430… 3
−1.430… is out of the domain. 2418.39… .
A(0) = 0; A(2.0971…) = 88.2727… ; c. Height = radius ⋅ 2
A(3) = 18π = 56.5486…
4 4000π
1+ 2 7 Volume of sphere Vs = π ⋅ 1000 =
Maximal cylinder has radius = = 3 3
3
4000π 3
52 – 4 7 Volume of maximal cylinder Vc =
2.0971… and height = = 9
9
4.6018… . ∴ Vc = Vs / 3
d. No. The maximum-volume cylinder has 26. Let r = radius of cone, h = height.
dimensions different from both of the Lateral area A(r) = π r · (slant height) = πr r 2 + h 2
maximum-area cylinders in parts b and c. 1
V = πr 2 h = 5π ⇒ h = 15r −2
e. No. Rotating the maximum-area rectangle 3
does not produce the maximum-volume ∴ A(r ) = πr r 2 + 225r –4
cylinder. But it produces the cylinder with h ≥ 2r ⇒ 2r ≤ 15r− 2
maximum lateral area. Domain of r is 0 < r ≤ 3 7.5 = 1.9574 K .
f. If y = a2 − x2, the paraboloid has radius = a.
V = π x2(a2 − x2) = π (a2x2 − x4) A(r )

V′ = π (2a 2x − 4x3)
V ′ = 0 ⇔ x = 0 or x = ± a/ 2 .
V is maximum at x = a/ 2. 20
r
For the cylinder of maximum volume,
1 1.957...
(cylinder radius):(paraboloid radius) = 1/ 2,
a constant. The graph shows a minimum of A(r) at endpoint
Note: This ratio is also constant (1/ 3 ) for the r = 1.957… .
cylinder of maximum lateral area, but is not Minimize A 2 (r ) = π 2 (r 4 + 225r −2 ).
constant for the cylinder of maximum total ( A 2 (r ))′ = π 2 ( 4r 3 − 450 r −3 ) = 0 at r = 6 112.5 =
area.
2.1971… , which is out of the domain.
25. a. x 2 + y 2 = 100, 0 ≤ x ≤ 10 A(1.9574…) = 26.915… , lim+ A(r ) = ∞.
r→0
Maximize V ( x ) = πx 2 ⋅ y = 2πx 2 100 – x 2 . Minimal cone has radius = 3 7.5 = 1.9574 …
b. V ( x) and height = 2 r = 23 7.5 = 3.9148K .
2000 Make r ≈ 1.96 ft and h ≈ 3.91 ft.
27. a. Lateral area L(x) = 2π x y
Domains: 0 ≤ x ≤ 5 and 0 ≤ y ≤ 7
x Equation of element of cone is
0 8.16... 10 7
y = − x + 7 ⇒ y = −1.4 x + 7.
5
The graph shows a maximum volume at ∴ L(x) = 2πx(−1.4x + 7) = 2π(−1.4x2 + 7x)
x ≈ 8.2.
–2πx 3 L (x )
V ′( x ) = + 4π x 100 – x 2 50
100 – x 2
–6π x 3 + 400π x
=
100 – x 2 x
0 2.5 5
200 10 6
V ′( x ) = 0 at x = 0, = = 8.1649K
3 3
The graph shows a maximum of L(x) at
V(0) = V(10) = 0
x ≈ 2.5.
 10 6  4000π 3 L′(x) = 2π (−2.8x + 7)
V = = 2418.399K
L′(x) = 0 at x = 2.5.
 3  9

182 Problem Set 8-3 Calculus Solutions Manual


© 2005 Key Curriculum Press
L′(x) goes from positive to negative at rh
Maximum area at x = if h ≥ 2r;
x = 2.5. 2(h – r )
∴ maximum lateral area at radius x = 2.5 cm. x = r otherwise.
b. Total area A(x) = 2πxy + 2πx2 c. From part b, the maximal cylinder degenerates
= 2πx(−1.4x + 7) + 2πx2 to two circular bases if the radius of the cone
A(x) = 2π(7x − 0.4x2) is at least half the height.
A (x )
29. Maximize V = π y2x.
150 Ellipse equation is (x/9)2 + (y/4)2 = 1, from
which y2 = (16/81)(81 − x2).
∴ V = (16π/81)(81x − x3)
x Domain: 0 ≤ x ≤ 9
0 5
V
150
The graph shows a maximum at endpoint
x = 5.
A′(x) = 2π(7 − 0.8x) = 0 at x = 8.75, out of
domain. x
∴ maximum is at an endpoint, x = 5. 0 5.196... 10
A(0) = 0; A(5) = 2π(52) = 50π = 157.07…
Maximum area is with the degenerate cylinder The graph shows a maximum V at x ≈ 5.2.
consisting only of the top and bottom, radius V′ = (16π/81)(81 − 3x2) = (16π/27)(27 − x2)
5 and height 0. V′ = 0 at x = ± 27 = ±5.196 …
28. a. Let r = radius of cone, h = height of cone −5.196… is out of the domain.
(constants). V (0) = V (9) = 0; V ( 27 ) = 32π 3 = 174.1…
Let (x, y) be a sample point on cone element. At x = 5.196… , y2 = (16/81)(81 − 27) =
Domain of x is 0 ≤ x ≤ r. 32/3 ⇒ y = 32/3 = 3.2659…
L(x) = 2πxy. ∴ maximum volume ≈ 174.1 cm3 at radius ≈
Equation of element of cone is 3.27 m and height ≈ 5.20 m.
y = (−h/r)x + h.
∴ L(x) = 2πx[(−h/r)x + h] = 2πh(−x2/r + x) 30. Maximize C(y) = πy2x, the volume of the cylinder.
L′(x) = 2πh(−2x/r + 1) The parabola has an equation of the form
L′(x) = 0 at x = r/2. x = ay 2 + 16.
L′(x) goes from positive to negative at 0 = a ⋅ 16 + 16 ⇒ a = −1 ⇒ x = 16 − y2
x = r/2. V(y) = π y2(16 − y2) = π (16y2 − y4)
∴ maximum lateral area at radius x = r/2. Domain: 0 ≤ y ≤ 4
b. A(x) = 2πxy + 2πx2 F (y )

= 2πx[h − (h/r)x] + 2πx2 300

A(x) = 2π[(1 − h/r)x2 + hx]


A′(x) = 2π[2(1 − h/r)x + h] = 0 at
–h
x= y
2(1 – h/r ) 0 2.828... 4
–h rh
A′(x) = 0 at x = =
2 – 2 h/r 2(h – r ) The graph shows a maximum V(y) at y ≈ 2.8.
If h ≤ 2r, then A′(x) ≠ 0 for all x ≤ r, so in C′(y) = π (32y − 4y 3) = 4π y(8 − y 2) = 0 at
this case the critical points are the endpoints, y = 0, ± 8 .
x = 0, r. y = − 8 is out of the domain.
A(0) = 0; A(r) = 2πr2 C(0) = C( 4) = 0, C( 8 ) = 64π = 201.0619…
rh
If h ≥ 2r, then 0 ≤ ≤ r, so this is a Maximum C(y) at y = 8.
2( h – r )
 rh  πrh 2 At y = 8 , x = 8.
critical point; A = . Maximal cylinder has radius = 8 ≈ 2.83 m,
 2( h – r )  2( h – r )
A′(x) goes from positive to negative at height = 8 m, and volume = 64π ≈ 201.1 m3.
rh Maximize F(y), the volume of the frustum.
x= . Note that Vf = (1/3)πh(R2 + r2 + Rr), where
2(h – r )

Calculus Solutions Manual Problem Set 8-3 183


© 2005 Key Curriculum Press
Vf = volume of frustum, h = height of frustum, false because f (c) could be a local minimum
R = larger radius, and r = smaller radius. or a plateau point.
1 32. a. Let x = length of corral (parallel to wall), y =
∴ F( y) = πx (16 + y 2 + 4 y)
3 width of corral (perpendicular to wall).
1 A = xy
= π (16 − y 2 )( y 2 + 4 y + 16)
3 If x ≤ 600, then 1000 = x + 2y ⇔
1 y = 500 − 0.5x.
F( y) = π (256 + 64 y − 4 y 3 − y 4 )
3 If x ≥ 600, then 1000 = x + 2y + (x − 600) ⇔
y = 800 − x.
F (y )
500 x – 0.5 x 2, x ≤ 600
300 ∴ A= 2
800 x – x , x > 600
A
150,000
y
0 1.821... 4

The graph shows a maximum F(y) at y ≈ 1.8.


x
1
F ′( y) = π (64 − 12 y 2 − 4 y 3 ) 500

3
F ′(y) = 0 ⇔ 64 − 12y 2 − 4y 3 = 0 The graph shows a maximum A at x ≈ 500.
Solving numerically for y close to 1.8 gives 500 – x, x < 600
y ≈ 1.8216… . A′ = 
800 – 2 x, x > 600
Substituting y = 1.8216… gives
For x < 600, A′ = 0 ⇔ x = 500.
x = 16 − y 2 ≈ 12.6816… .
For x > 600, A′ = 0 ⇔ x = 400 (out of the
1
F(1.8216 K) = π x (16 + y 2 + 4 y) ≈ domain).
3 A′ is undefined at the cusp, x = 600.
353.318… . Maximum at x = 500 because graph is
Maximal frustum has radii = 4 m and ≈1.82 m, parabola opening downward.
height ≈ 12.68 m, and volume ≈ 353.3 m3. Or: Check the critical points.
The maximal frustum contains ≈ 152.3 m3 more A(500) = 500(500) − 0.5(500)2 = 125,000
than the maximal cylinder, about 75.7% more. A(600) = 500(600) – 0.5(600)2 = 120,000 ft2
31. a. If f (c) is a local maximum, then Maximum area is 125,000 f t 2 at x = 500 ft.
f (x) − f (c) ≤ 0 for x in a neighborhood of c. b. If x ≤ 400, then 1000 = x + 2y ⇔
For x to the left of c, x − c < 0. y = 500 − 0.5x.
f ( x ) – f (c ) If x ≥ 400, then 1000 = x + 2y + (x − 400)
Thus, ≥ 0 (neg./neg.) and
x–c ⇔ y = 700 − x.
f ( x ) – f (c )
f ′(c) = lim− ≥ 0. 500 x – 0.5 x 2, x ≤ 400
x→0 x–c ∴ A=
For x to the right of c, x − c > 0.
2
700 x – x , x > 400
f ( x ) – f (c )
Thus, ≤ 0 (neg./pos.) and A
150,000
x–c
f ( x ) – f (c )
f ′(c) = lim+ ≤ 0.
x→0 x–c
Therefore, 0 ≤ f ′(c) ≤ 0.
Because f ′(c) exists, f ′(c) = 0 by the squeeze
x
400
theorem, Q.E.D.
b. If f is not differentiable at x = c, then f ′(c) The graph shows a maximum A at the cusp,
does not exist and thus cannot equal zero. x = 400.
Without this hypothesis, the reasoning in 500 – x, x < 400
A′ = 
part a shows only that f ′(x) changes sign at 700 – 2 x, x > 400
x = c. There could be a cusp, a removable For x < 400, A′ = 0 ⇔ x = 500 (out of the
discontinuity, or a step discontinuity at x = c. domain).
c. The converse would say that if f ′(c) = 0, then For x > 400, A′ = 0 ⇔ x = 350 (out of the
f (c) is a local maximum. This statement is domain).

184 Problem Set 8-3 Calculus Solutions Manual


© 2005 Key Curriculum Press
Maximum area is at the cusp, x = 400. Q7. Q8. Sample answer:
A = 700(400) − 4002 = 120,000 y y
Maximum area is 120,000 f t 2.
c. If x ≤ 200, then 1000 = x + 2y ⇔
x

y = 500 − 0.5x. 1
x
If x ≥ 200, then 1000 = x + 2y + (x − 200) ⇔ 2

y = 600 − x.
Q9. tan x + C Q10. B
500 x – 0.5 x 2, x ≤ 200
∴ A= 1. a. y = 4 − x 2
2
600 x – x , x > 200 dV = 2πxy ⋅ dx = 2π (4x − x3) dx
A b. 0 = 4 − x 2 = (2 − x)(2 + x) at x = ±2
150,000
2
V = 2π ( 4 x − x 3 ) dx = 2π  2 x 2 − x 4 
2


1
0  4  0
= 8π = 25.1327…
x
c. y = 4 − x 2 ⇒ x 2 = 4 − y
300
Upper bound of solid is at y = 4.
The graph shows a maximum A at x ≈ 300. dV = πx2 dy = π (4 − y) dy
4 4
500 – x, x < 200 V= ∫ π (4 − y) dy = π (4 y − 0.5y ) =
2
A′ = 
600 – 2 x, x > 200
0 0

8π = 25.1327… , which is the same answer


For x < 200, A′ = 0 ⇔ x = 500 (out of the
as by cylindrical shells in part b.
domain).
For x > 200, A′ = 0 ⇔ x = 300. 2. a. Height of cylinder = 8 − x
A′ is undefined at the cusp, x = 200. b. y = x 2/3 ⇒ x = y 3/2
Maximum area is at x = 300 because graph is dV = 2π (8 − x)y dy = 2π (8 − y3/2)y dy
a parabola opening downward. = 2π (8y − y 5/2) dy
Or: Check critical points. c. At x = 8, y = 82/3 = 4.
A(300) = 600(300) − 3002 = 90,000
V = 2π (8 y − y 5/2 ) dy = 2π  4 y 2 − y 7/2 
4 4


2
A(200) = 500(200) − 0.5(200)2 = 80,000 ft2
0  7 
Maximum area is 90,000 ft2 at x = 300 ft. 0
384
33. Answers will vary. = π = 172.3387…
7
d. dV = π y 2 dx = π x 4/3 dx
Problem Set 8-4 8 8


3 384
V = π x 4/3 dx = π x 7/3 = π=
0 7 0 7
Q1. Q2.
y y
172.3387… , which is the same as the
volume by cylindrical shells in part c.
x x 3. The graph shows y = −x2 + 4x + 3, from x = 1 to
x = 4, sliced parallel to the y-axis, with sample
point (x, y), rotated about the y-axis, showing
back half of solid only.
Q3. Q4. y
(x, y)
y y

x x

1 x
1 4

Q5. Q6.
dV = 2πxy ⋅ dx = 2π (−x 3 + 4x 2 + 3x) dx
y y
4
V= ∫ 2π (− x + 4 x 2 + 3 x ) dx
3
x x 1
≈ 268.6061… (exactly 85.5π)

Calculus Solutions Manual Problem Set 8-4 185


© 2005 Key Curriculum Press
y
Circumscribed hollow cylinder of radii 1 and 6
4 and height 7 has volume π(42 − 12) ⋅ 7 = (0, y )
(x, y )
329.8… , which is a reasonable upper bound for
the calculated volume.
4. The graph shows y = x2 − 8x + 17, from x = 2 to
x = 5, sliced parallel to the y-axis, with sample –1 x

point (x, y), rotated about the y-axis, showing


back half of solid only.

5
(x, y)

dV = 2π y(0 − x) ⋅ dy = 2π (−y3 + 10y2 − 24y) dy


6

∫ 2π (− y
x
2 5
V= 3
+ 10 y 2 − 24 y) dy
4

≈ 41.8879…  exactly
40 
π
dV = 2πxy ⋅ dx = 2π (x3 − 8x2 + 17x) dx  3 
5 Circumscribed hollow cylinder of radii 4 and 6
V= ∫ 2π ( x − 8 x 2 + 17 x ) dx and height 1 has volume π(62 − 42) ⋅ 1 =
3
2
62.83… , which is a reasonable upper bound for
≈ 117.8097… (exactly 37.5π)
the calculated volume.
Circumscribed hollow cylinder of radii 2 and 5
and height 5 has volume π(52 − 22) ⋅ 5 = 7. Figure 8-4h shows y = x3, intersecting the line
329.8… , which is a reasonable upper bound y = 8 at x = 2 and the line x = 1. Rotate about
for the calculated volume. Assuming that the the y-axis. Slice parallel to the y-axis. Pick
part of the solid above y = 2 could be fit into sample points (x, y) on the graph and (x, 8) on
the “trough,” the volume is approximately the line y = 8.
π(52 − 22) ⋅ 2 = 131.9… , which is close to the dV = 2π x(8 − y) ⋅ dx = 2π (8x − x4) dx
2
V= ∫ 2π (8x − x
calculated volume. 4
) dx
1
5. The graph shows x = −y2 + 6y − 5, intersecting
≈ 36.4424… (exactly 11.6π)
y-axis at y = 1 and y = 5, rotated about the
Circumscribed hollow cylinder of radii 2 and 1
x-axis, showing back half of solid only.
and height 7 has volume π(22 − 12) ⋅ 7 = 65.9… ,
which is a reasonable upper bound for the
y
5 calculated volume.
(x, y)
(0, y )
8. The graph shows y = 1/x, intersecting line y = 4
at x = 0.25 and the line x = 3, rotated about the
1
x y-axis, showing back half of solid only.
4
y
(x, 4)
y=4

(x, y )
x

dV = 2π y(x − 0) ⋅ dy = 2π (−y + 6y − 5y) dy 3 2 0.25 3

5
dV = 2πx ⋅ (4 − y) · dx = 2π (4x − 1) dx
V= ∫ 2π (− y + 6 y 2 − 5 y) dy
3
3
V= ∫ 2π ( 4 x − 1) dx
1

≈ 201.0619… (exactly 64π) 0.25

Circumscribed hollow cylinder of radii 1 and 5 ≈ 95.0331… (exactly 30.25π)


and height 4 has volume π(52 − 12) ⋅ 4 = Circumscribed hollow cylinder of radii 0.25 and 4
301.5… , which is a reasonable upper bound for and height 3.7 has volume π(32 − 0.252)(3.7) =
the calculated volume. 103.8… , which is a reasonable upper bound for
6. The graph shows x = y2 − 10y + 24, intersecting the calculated volume.
y-axis at y = 4 and 6, rotated about the x-axis, 9. Figure 8-4i shows y = 1/x2, intersecting the line
showing back half of solid only. x = 5 at y = 0.04 and the line y = 4. Rotate

186 Problem Set 8-4 Calculus Solutions Manual


© 2005 Key Curriculum Press
y
about the x-axis. Slice parallel to the x-axis. Pick (x 2 , y )
2
sample points (x, y) on the graph and (5, y) on (x1, y)

the line x = 5. x
dV = 2π y(5 − x) ⋅ dy = 2π (5y − y1/2 ) dy 4 8
4
V= ∫
0.04
2π (5 y − y1/2 ) dy

≈ 217.8254 … (exactly 69.336π )


Circumscribed cylinder of radius 4 and height 4.5 dV = 2π y(x2 − x1) dy = 2π (2y2 + 4y − y4) dy
has volume π ⋅ 42 ⋅ 4.5 = 226.1… , which is a 2
V= ∫ 2π (2 y + 4 y − y 4 ) dy
2
reasonable upper bound for the calculated 0
volume.
≈ 43.5634 K  exactly 13 π 
13
10. The graph shows y = x2/3 , intersecting the line  15 
y = 1 and intersecting the line x = 8 at y = 4,
Circumscribed cylinder of radius 2 and height 8
rotated about the x-axis, showing back half of has volume π ⋅ 22 ⋅ 8 = 100.5… , which is a
solid only. reasonable upper bound for the calculated
y
volume.
4 13. Figure 8-4k shows y = x3/2 , from x = 1 to x = 4.
(x, y ) (8, y )
Rotate about the line x = 5. Slice parallel to the
1
x y-axis. Pick sample point (x, y).
1 8 dV = 2π(5 − x)y ⋅ dx = 2π(5x3/2 − x5/2 ) dx
4
V= ∫ 2π (5x − x 5/2 ) dx
3/2
1

≈ 161.5676 …  exactly 51 π 
3
 7 
dV = 2π y(8 − x) ⋅ dy = 2π (8y − y5/2 ) dy
4 Circumscribed cylinder of radius 4 and height 8
V= ∫ 2π (8y − y
5/2
) dy has volume π(42) ⋅ 8 = 402.1… , which is a
1
reasonable upper bound for the calculated
≈ 149.0012 …  exactly 47 π 
3 volume.
 7  14. The graph shows y = x− 2, from x = 1 to x = 2,
Circumscribed hollow cylinder of radii 1 and 4 rotated about the line x = 3, showing back half of
and height 7 has volume π(42 − 12) ⋅ 7 = solid only.
329.8… , which is a reasonable upper bound for
the calculated volume. y
(x, y )

11. Figure 8-4j shows y1 = x2 − 6x + 7 and y2 =


x + 1, intersecting at (1, 2) and (6, 7). Rotate 1

about the y-axis. Slice parallel to y-axis. Pick x


sample points (x, y1) and (x, y2). 1 2 3
dV = 2πx ⋅ (y2 − y1) ⋅ dx = 2π(−x3 + 7x2 − 6x) dx
6
V= ∫ 2π (− x + 7 x 2 − 6 x ) dx
3
1 dV = 2π (3 − x ) y ⋅ dx = 2π (3 x −2 − x −1 ) dx
2
≈ 458.1489…  exactly 145 π  ∫ 2π (3x
5 V= −2
− x −1 ) dx
 6  1

Circumscribed hollow cylinder of radii 1 and 6 ≈ 5.0696 … (exactly π (3 − 2 ln 2))


and height 7 has volume π(62 − 12) ⋅ 7 = Circumscribed hollow cylinder of radii 1 and 2
769.6… , which is a reasonable upper bound for and height 1 has volume π(22 − 12) ⋅ 1 = 9.4… ,
the calculated volume. which is a reasonable upper bound for the
12. The graph shows y = x11/3 ⇒ x1 = y 3 and y = calculated volume.
0.5x2 − 2 ⇒ x2 = 2y + 4, intersecting at (8, 2) 15. The graph shows y1 = x4 and y2 = 5x + 6,
in Quadrant I and bounded by the x-axis, rotated intersecting at x = −1 and x = 2, rotated about
about the x-axis, showing back half of solid the line x = 4, showing back half of solid
only. only.

Calculus Solutions Manual Problem Set 8-4 187


© 2005 Key Curriculum Press
16
y 18. The graph shows y1 and y2 as described in
Problem 17, but rotated about the line y = −1,
showing back half of solid only. Slicing
(x, y2) perpendicular to the x-axis is appropriate
because slicing parallel to it would give strips
of length (curve) minus (curve) at some values
(x, y1 ) of y and (curve) minus (other curve) at other
values of y.
x
–1 2 4 y
(x, y1)
dV = 2π(4 − x)(y2 − y1) dx
= 2π (4 − x)(5x + 6 − x4) dx (x, y2 )
2
V= ∫ 2π ( 4 − x )(5 x + 6 − x ) dx 4
1
−1 x

≈ 390.1858… (exactly 124.2π ) 0 3


y = –1
Circumscribed hollow cylinder of radii 2 and 5
and height 16 has volume π(52 − 22) ⋅ 16 =
1055.5… , which is a reasonable upper bound for
the calculated volume.
16. The graph shows y1 = x = x 1/2 and y 2 = 6 − x,
intersecting at x = 4 in Quadrant I and bounded
dV = π [(y1 + 1)2 − (y2 + 1)2] dx
by the line x = 1, rotated about the line x = −1,
= π [(−x2 + 4x + 2)2 − (1.4x + 1)2] dx
showing back half of solid only.
Limits of integration are 0 to b, where
y
b = 3.3740… , as in Problem 17.
5 b

∫ π [(− x
(x, y2 )
V= 2
+ 4 x + 2)2 − (1.4 x + 1)2 ] dx
0
≈ 181.0655…
1 (x, y1 )
x Circumscribed hollow cylinder of radii 2 and 6
–1 1 4
and height 3.4 has volume π(62 − 22) · 3.4 =
dV = 2π(x + 1)(y2 − y1) dx 341.8… , a reasonable upper bound for the
= 2π (x + 1)(6 − x − x 1/2) dx calculated volume.
4
V= ∫ 2π ( x + 1)(6 − x − x 19. Slice perpendicular to the y-axis. Pick sample
1/2
) dx
1 points (x, y) on the graph of y = x3 and (1, y)
on the line x = 1.
≈ 109.5368K  exactly 34 π 
13
 15  y = x 3 ⇒ x = y 1/3; y 1/3 = 1 at y = 1
Circumscribed hollow cylinder of radii 2 and 5 dV = π ( x2 − 12) dy = π (y2/3 − 1) dy
and height 4 has volume π(52 − 22) ⋅ 4 =
8
V= ∫ π (y − 1) dy ≈ 36.4424 …
2/3
263.8… , which is a reasonable upper bound for 1
the calculated volume. (exactly 11.6π ), which agrees with the answer to
17. Figure 8-4l shows y1 = −x2 + 4x + 1 and y2 = Problem 7.
1.4x, intersecting at x = 0 and x = 3.3740… 20. See the graph for Problem 8. Slice
(store as b). Rotate about the line x = −2. Slice perpendicular to the y-axis. Pick sample points
parallel to the y-axis. Pick sample points (x, y1) (x, y) on the graph of y = 1/x and (3, y) on the
and (x, y2). line x = 3.
dV = 2π (x + 2) ⋅ (y1 − y2) dx dV = π (32 − x 2 ) dy = π (9 − y −2 ) dy
= 2π (x + 2)(−x2 + 4x + 1 − 1.4x) dx 4
b V= ∫ π (9 − y −2 ) dy ≈ 95.0331… (exactly
V= ∫ 0
2π ( x + 2)( − x 2 + 4 x + 1 − 1.4 x ) dx 1/ 3
30.25π ) , which agrees with the answer to
≈ 163.8592… Problem 8.
Circumscribed hollow cylinder with radii 2 and
21. The graph shows y = x1/3, from x = 0 to x = 8,
5.4 and height 4 has volume π(5.42 − 22)4 =
rotated about the x-axis, showing back half of
316.1… , a reasonable upper bound for calculated solid only.
volume.

188 Problem Set 8-4 Calculus Solutions Manual


© 2005 Key Curriculum Press
y The integral can be found algebraically
2 (x, y )
(8, y ) using the Pythagorean properties from
x trigonometry.
8 sin3 t = (1 − cos2 t) sin t = sin t − cos2 t sin t
0 0
V= ∫ π
−45π sin t dt − ∫ π
45π cos 2 t ( − sin t dt )
0
y = x 1/3 ⇒ x = y 3 = 45π cos t − 15π cos t 3
π
dV = 2π y ( 8 − x) ⋅ dy = 2π ( 8y − y4) dy = 45π − 15π − (−45π ) + (−15π ) = 60π
2 c. Slice the region parallel to the line x = 7 and
V = 2π (8 y − y 4 ) dy = 2π  4 y 2 − y 5 
2


1
0  5  0
rotate about that line. Pick sample points
(x, y) and (x, −y) on the upper and lower
= 2π ( 16 − 6.4) = 19.2π = 60.3185789… branches.
R8 = 19.3662109… π = 60.8407460… dV = 2π ( 7 − x)[y − (−y)] dx
R100 = 19.2010666… π = 60.3219299… = 4π ( 7 − 5 cos t)(3 sin t)(−5 sin t dt)
R1000 = 19.2000106… π = 60.3186124… = −60π ( 7 − 5 cos t)(sin2 t) dt
Rn is approaching 19.2π as n increases. Limits of integration are t = π to t = 0, as in
22. a. y = sin x from x = 0 to x = 2, rotated about part b.
the y-axis, as in Figure 8-4m. Slice parallel V ≈ 2072.6169… (exactly 210π 2, using the
to the y-axis. Pick sample point (x, y) on half-argument properties for sin2 t, as in
the graph. Problem 16 of Problem Set 5-9, or by using
dV = 2π xy ⋅ dx = 2π x sin x dx integration by parts as in Chapter 9).
2
V= ∫ 2πx sin x dx ≈ 10.9427…
0
24. Answers will vary.
numerically (exactly 2π ( sin 2 − 2 cos 2),
integrating by parts).
b. The integrand, x sin x, is a product of two Problem Set 8-5
functions, for which the antiderivative cannot Q1. Q2.
be found using techniques known so far. y y
23. a. x = 5 cos t, dx = −5 sin t dt 16 16

y = 3 sin t, dy = 3 cos t dt
Slice parallel to the x-axis, then rotate about x x
the x-axis. Pick sample points (−x, y) at the 0 4 1 4

left end of the strip and (x, y) at the right end.


dV = 2π y [ x − (−x)] ⋅ dy = 4π xy dy 4 1 34
= 4π ( 5 cos t)(3 sin t)(3 cos t dt)
Q3. A = ∫
1
x 2 dx Q4. A = x
3 1
= 180π cos2 t sin t dt Q5. A = 21
Limits of integration are y = 0 to y = 3.
Q6.
At y = 0, t = 0. At y = 3, t = π /2.
π /2 y
V= ∫0
180π cos 2 t sin t dt 16

π /2
= −60π cos3 t 0 x
= −60π ( 0 − 1) = 60π = 188.4955… 4

b. Slice the region in Quadrant I perpendicular to


the x-axis, then rotate about the x-axis. Pick 4 π 44
sample point (x, y) on the graph. Q7. V = ∫
1
2πx 3 dx Q8. V = x
2 1
dV = π y 2 dx = π ( 3 sin t)2(−5 sin t dt)
Q9. V = 127.5π Q10. E
= −45π sin3 t dt
Limits of integration are from x = −5 to 1. a.
x = 5. y

At x = −5, t = π . At x = 5, t = 0.
0
V= ∫π
−45π sin 3 t dt ≈ 188.4955…
(exactly 60π ) , which agrees with the volume 1
x
found in part a. 0 2

Calculus Solutions Manual Problem Set 8-5 189


© 2005 Key Curriculum Press
5 5
b. L ≈ ∑n =1
(0.4)2 + [e 0.4 n – e 0.4( n –1) ]2 b. L ≈ ∑
n =1
(0.3)2 + [sec 0.3n – sec 0.3 (n – 1)]2

= 6.7848… = 13.7141…
c. dy = e x dx c. dy = sec x tan x dx
dL = dx 2 + dy 2 = 1 + e 2 x dx dL = dx 2 + dy 2 = 1 + tan 2 x sec 2 x dx
1/5

L= ∫
2

0
1 + e dx ≈ 6.7886 … numerically
2x L= ∫
0
1 + tan 2 x sec 2 x dx ≈ 13.7304 …
numerically
2. a. 5. a.
y y
9

1 1 x
x –1 6
0 3

5
b. dy = (2x − 5) dx
b. L ≈ ∑n =1
(0.6)2 + [2 0.6 n – 2 0.6( n –1) ]2
dL = dx 2 + dy 2 = 1 + (2 x – 5)2 dx
= 7.7853… 6

c. dy = (2x ln 2) dx L= ∫
1
1 + (2 x – 5)2 dx ≈ 15.8617K

dL = dx 2 + dy 2 = 1 + (2 x ln 2)2 dx c. Low point is (2.5, −3.25). Chords from


(1, −1) to (2.5, −3.25) and from (2.5, −3.25)
3
L= ∫ 0
1 + (2 x ln 2)2 dx ≈ 7.7920 K to (6, 9) have combined length 7.3125 +
162.3125 = 15.4 … , which is a reasonable
numerically
lower bound for L.
3. a.
6. a.
y
y
4

10

x x
0 1.5 0 4

5
b. L ≈ ∑n =1
(0.3)2 + [tan 0.3n – tan 0.3(n – 1)]2 b. dy = (4 − 2x) dx
dL = dx 2 + dy 2 = 1 + ( 4 – 2 x )2 dx
= 14.4394… 4
c. dy = sec2 x dx L= ∫
0
1 + ( 4 – 2 x )2 dx ≈ 9.2935K

dL = dx 2 + dy 2 = 1 + sec 4 x dx c. Chords from (0, 0) to (2, 4) and from


1.5 (2, 4) to (4, 0) have combined length
L= ∫ 0
1 + sec 4 x dx ≈ 14.4488K
2 20 = 8.9442 … , which is a reasonable
numerically lower bound for L.
4. a. 7. a.
y 16 y

10

1 x x
0 1.5 –1 2

190 Problem Set 8-5 Calculus Solutions Manual


© 2005 Key Curriculum Press
b. dy = −4x3 dx b. dy = (sin x + x cos x) dx
dL = dx 2 + dy 2 = 1 + 16 x 6 dx dL = dx 2 + dy 2
2
L= ∫ −1
1 + 16 x 6 dx ≈ 18.2470 K = 1 + (sin x + x cos x )2 dx

c. Chords from (−1, 15) to (0, 16) and (0, 16) to L= ∫


0
1 + (sin x + x cos x )2 dx ≈ 54.1699K
(2, 0) have combined length 2 + 260 = c. Eight chords of ∆x = π/2 extend from middle
17.5… , which is a reasonable lower bound to high to middle to low points on the graph.
for L. Lengths sum to 52.6109… , a reasonable
8. a. lower bound for L.
y 11. a.
y
50
10

x
–1 9

x
0 1.5

b. dy = (3x2 − 18x + 5) dx
b. dy = sec2 x dx
dL = dx 2 + dy 2
dL = dx 2 + dy 2 = 1 + sec 4 x dx
= 1 + (3 x 2 – 18 x + 5) 2 dx
1.5

L= ∫
9

−1
1 + (3 x – 18 x + 5) dx ≈ 219.4873K
2 2 L= ∫
0
1 + sec 4 x dx ≈ 14.4488K

c. Distance between the endpoints is


c. Using five chords with ∆x = 2, L ≈
14.1809… , which is a reasonable lower
204.4605… , which is a reasonable lower
bound for L.
bound for L.
9. a. 12. a.
y y

10

1 x
x
0 1.5
1 e

b. dy = 2 ln x ⋅ x −1 dx = 2 x −1 ln x dx b. dy = sec x tan x dx
dL = dx 2 + dy 2 = 1 + (2 x –1 ln x )2 dx dL = 1 + (sec x tan x )2 dx
1.5


e
L= ∫ 0.1
1 + (2 x –1 ln x )2 dx ≈ 7.6043… L=
0
1 + (sec x tan x )2 dx ≈ 13.7304K

c. Chords from x = 0.1 to x = 1 and from x = 1 c. The distance between the endpoints is
to x = e have combined length 7.3658… , 13.2221… , which is a reasonable lower
which is a reasonable lower bound for L. bound for L.
10. a. 13. a.
y
y 5
5

x
x
0 5 4π
5

Calculus Solutions Manual Problem Set 8-5 191


© 2005 Key Curriculum Press
b. dx = −15 cos2 t sin t dt, dy = 15 sin2 t cos t dt 2π 1 – cos 2 t
dL = dx 2 + dy 2
= 10 2 ∫ 0 1 + cos t
dt


= (–15 cos 2 t sin t )2 + (15 sin 2 t cos t )2 dt |sin t |
= 10 2 dt

0 1 + cos t
L= ∫ (–15 cos 2 t sin t )2 + (15 sin 2 t cos t ) 2 dt π
0

≈ 30
= 20 2 ∫ 0
(1 + cos t ) −1/2 (sin t dt )

π
To see why the answer is so simple, = −40 2 (1 + cos t )1/2 0
transform the radicand and use the
fundamental theorem. = −40 2 (1 – 1) + 40 2 (1 + 1)1/2 = 80
1/2


c. Maximum/minimum values of y are
L= 225(sin t cos t )2 (cos 2 t + sin 2 t ) dt
0 ±7.5 3 . Circle of radius 7.5 3 has
2π circumference 15π 3 = 81.6209K .
= 7.5 ∫ 0
(2 sin t cos t )2 dt
15. a.
2π y
= 7.5 ∫ 0
sin 2 2t dt 4

2π π /2 x
= 7.5 ∫ 0
|sin 2t | dt = 7.5 ⋅ 4 ∫
0
sin 2t dt 4

π /2
= 30   ( − cos 2t )
1
= 30 (exactly!)
 2 0
b. dx = (−5 sin t + 5 sin 5t) dt
c. Circle of radius 5 (i.e., x = 5 sin t, y = dy = (5 cos t − 5 cos 5t) dt
5 cos t) has circumference 10π = 31.4152… ,
which is close to the calculated value of L. dL = dx 2 + dy 2 =
14. a. (–5 sin t + 5 sin 5t ) 2 + (5 cos t – 5 cos 5t ) 2 dt
y 2π
10 L= ∫
0
(–5 sin t + 5 sin 5t ) 2 + (5 cos t – 5 cos 5t ) 2 dt
x ≈ 40
5
To see why the answer is so simple,
transform the radicand and use the
fundamental theorem.

b. dx = 5(−2 sin t + 2 sin 2t) dt L=5 ∫


0
2 – 2 sin t sin 5t – 2 cos t cos 5t dt
dy = 5(2 cos t − 2 cos 2t) dt 2π

dL = dx + dy = 2 2 =5 2 ∫ 0
1 – cos 4t dt (using cos ( A − B))

2π 1 – cos 2 4t
[5(–2 sin t + 2 sin 2t )]2 + [5(2 cos t – 2 cos 2t )]2 dt =5 2 ∫ 0 1 + cos 4t
dt
L=
2π 2π


|sin 4t |

0
[5(–2 sin t + 2 sin 2t )]2 + [5(2 cos t – 2 cos 2t )]2 dt =5 2
0 1 + cos 4t
dt

≈ 80 π /4
To see why the answer is so simple, = 40 2 ∫ 0
(1 + cos 4t ) −1/2 (sin 4t dt )
transform the radicand algebraically and π /4
use the fundamental theorem. = −20 2 (1 + cos 4t )1/2 0

L = 10 ∫ 0
2 – 2 sin t sin 2t – 2 cos t cos 2t dt = −0 + 20 2 ⋅ 2 = 40


c. Maximum/minimum values of x, y are
= 10 2 ∫ 0
1 – cos t dt (using cos ( A − B)) ±3 3. Circle of radius 3 3 has circumference
32.6483… , which is close.

192 Problem Set 8-5 Calculus Solutions Manual


© 2005 Key Curriculum Press
16. a. c. Distance between endpoints is
y 1.006944 K = 1.00346 K , which is a
5 reasonable lower bound for L.
x 19. a.
5
y

t = 4π

b. dx = (−sin t + sin t + t cos t) dt = t cos t dt


5

dy = (cos t − cos t + t sin t) dt = t sin t dt 1 8


x

dL = dx 2 + dy 2 = (t cos t )2 + (t sin t )2 dt
= |t | dt = t dt (because t ≥ 0) b. dy = 2 x −1/ 3 dx

dL = dx 2 + dy 2 = 1 + 4 x –2/3 dx


L= t dt = 0.5t 2 = 8π 2 = 78.9568K
0 0
8
L= ∫ 1 + 4x
–2/3
c. Circle of radius 4π = 12.5663… would have dx
1
circumference = 8π 2. 8

17. a. = ∫ ( x + 4) 2/3 1/2


( x −1/3 dx )
1
y

( x + 4)1/2  x −1/3 dx 
3 8 2/3

2
=
30

2 1  3 
8
3 2
= ⋅ ( x 2/3 + 4)3/2
2 3 1
x
0 4 = 8 8 − 5 5 = 11.4470 K
c. Distance between endpoints is 130 =
b. dy = 6x1/2 dx
11.4017… , which is a reasonable lower
dL = dx 2 + dy 2 = 1 + 36 x dx bound for L.
4 4 20. a.
∫ ∫ (1 + 36 x )
1
L= 1 + 36 x dx = 1/2
(36 dx )
0 36 0 y
1 4 1
= (1 + 36 x )3/2 0 = (1453/ 2 – 1)
54 54
= 32.3153… 5

c. The chord connecting the endpoints has length x


32.2490… , which is a reasonable lower 0 3
bound for L.
18. a. 1 2
b. dy = ( x + 2)1/ 2 2 x dx = x ( x 2 + 2)1/ 2 dx
y 2
1 dL = dx 2 + dy 2 = 1 + x 2 ( x 2 + 2) dx

= 1 + x 4 + 2 x 2 dx = (1 + x 2 ) dx
3 1 3

x
1 2 L= (1 + x 2 ) dx = x + x 3 = 12
0 3 0
−2
b. dy = ( x /4 − x ) dx
2
c. Distance between endpoints is 11.6123… ,
which is a reasonable lower bound for L.
dL = dx + dy = 1 + ( x /4 – x ) dx
2 2 2 –2 2

21. Construct an x-axis at water level and a y-axis


= 1 + x 4 /16 – 1/2 + x –4 dx through the vertex of the parabola.
= ( x 2 /4 + x –2 )2 dx = | x 2 /4 + x −2 | dx y

∫ ( x /4 + x
750
L= 2 –2
) dx (because integrand > 0)
1
220 x
2 1
= x 3 /12 − x −1 1 = 1 = 1.0833K –2100 0 2100
12

Calculus Solutions Manual Problem Set 8-5 193


© 2005 Key Curriculum Press

General equation is y − 220 = ax2.
Substitute (2100, 750) for (x, y).
L= ∫ 0
(–100 sin t )2 + (50 cos t )2 dt

750 − 220 = a ⋅ 2100 2 ⇒ a =


53 ≈ 484.4224 … ≈ 484.4 m
441000
53 24.
Equation of parabola is y = x 2 + 220 . y
441000 5
106
dy = x dx
441000 x
–8 8
dL = dx 2 + dy 2 = 1 + (106/441000)2 x 2 dx
2100
L= ∫ –2100
1 + (106/441000)2 x 2 dx –5

≈ 4372.0861… numerically ≈ 4372 feet. dx = −16 sin 2t, dy = 5 cos t


The answer is reasonable because the 4200 feet
between supports is a lower bound for L. dL = dx 2 + dy 2
22. y = 0.2(e x + e − x ), dy = 0.2(e x − e − x ) dx = (–16 sin 2t )2 + (5 cos t )2 dt
dL = dx + dy 2 2
Curve appears to have length

= 1 + 0.04(e x – e – x )2 dx
4
L= ∫ 0
(–16 sin 2t )2 + (5 cos t )2 dt
L= ∫−4
1 + 0.04(e x – e – x )2 dx = 68.7694…
Length should be less than the lengths of three
≈ 24.1722 … ≈ 24.2 ft circumscribing segments, 16 + 16 + 10 = 42.
The parabola with vertex (0, 0.4) and endpoints The discrepancy is explained by the fact that the
(±4, 0.2(e4 + e− 4)) = (±4, 10.9232…) has equation parabola is traced twice as t goes from 0 to 2π.
y = ax2 + 0.4. Substituting (4, 10.9232…) gives Actual length ≈ (0.5)(68.7694…) = 34.384… ,
10.9232… = 16a + 0.4 ⇒ a = 0.6577… . for which 42 is a reasonable upper bound.
y = 0.6577… x2 + 0.4 ⇒ dy = 1.3154… x dx
2
dL = dx 2 + dy 2 = 1 + 1.7303... x 2 dx 25. 9 x 2 = 4 y 3 ⇔ x = ± y 3/2 .
3
4
L= ∫−4
1 + 1.7303K x 2 dx ≈ 23.2193… ≈ 23.2 ft, 3
y

which is about a foot shorter than the catenary,


as shown by graph:
y
10 x
0 3 4

dx = y 1/2 dy
dL = dx 2 + dy 2 = ( y + 1)1/2 dy
x
3 3

∫ ( y + 1)
2
–4 4
L= 1/2
dy = ( y + 1)3/2
0 3 0
23. Outer ellipse:
x = 120 cos t, dx = −120 sin t dt =4
2
= 4.6666 …
y = 100 sin t, dy = 100 cos t dt 3
dL = dx 2 + dy 2 26. x 2 = y 3 ⇔ x = ±y 1.5
2x dx = 3y2 dy ⇒ 4x2 dx2 = 9y4 dy2
= (–120 sin t )2 + (100 cos t )2 dt 9

⇒ 4 y 3 dx 2 = 9 y 4 dy 2 ⇒ dx 2 = y dy 2

4
L= (–120 sin t )2 + (100 cos t )2 dt
0 Note that dy < 0 between (−1, 1) and (0, 0):
≈ 692.5791… ≈ 692.6 m y
Inner ellipse:
x = 100 cos t, dx = −100 sin t dt 4

y = 50 sin t, dy = 50 cos t dt
dL = dx 2 + dy 2 1
x
–1 8
= (–100 sin t ) + (50 cos t ) dt
2 2

194 Problem Set 8-5 Calculus Solutions Manual


© 2005 Key Curriculum Press
For x in [−1, 0], x = −y 1.5 , dx = −1.5y0.5 dy, Doubling A doubles the amplitude of the
dL = − dx + dy = − 2.25 y + 1 dy.
2 2 sinusoid. However, it less than doubles the
length of the sinusoid for much the same reason
For x in [0, 8], x = y1.5 , dx = 1.5y0.5 dy,
that doubling one leg of a right triangle does not
dL = dx 2 + dy 2 = 2.25 y + 1 dy. double the hypotenuse. In the limit as A
0 4 approaches infinity, doubling A approaches
L= ∫
1
− 2.25 y + 1 dy + ∫
0
2.25 y + 1 dy doubling the length.
8 0
8 4 30. x = cos t, dx = −sin t dt
=− (2.25 y + 1)3/2 + (2.25 y + 1)3/2 y = A sin t, dy = A cos t dt
27 1 27 0

=
8
(–1 + 3.25 + 10 – 1) = 10.5131…
3/2 3/2 dL = dx 2 + dy 2 = sin 2 t + A 2 cos 2 t dt
27 The entire ellipse is generated as t increases from
t 1
27. x = cos t, dx = (cos t – t sin t ) dt 0 to 2π .
π π 2π
t 1
y = sin t, dy = (sin t + t cos t ) dt
L= ∫ 0
sin 2 t + A 2 cos 2 t dt
π π
dL = dx 2 + dy 2 A L
1
= (cos t – t sin t )2 + (sin t + t cos t )2 dt 0 4 (a double line segment)
π
1 1 6.283185… (= 2π)
= 1 + t 2 dt
π 2 9.688448…
The curve crosses the x-axis exactly when sin t 3 13.364893…
= 0, when t is a multiple of π. There are seven
crossings after the beginning, so t should run Doubling A doubles one axis of the ellipse
between 0 and 7π. To check this, note that the without changing the other axis. That is why the
curve ends at (−7, 0), so solve (t/π) cos t = −7 length does not double when A doubles. The
with t = nπ ⇒ (nπ /π ) cos nπ = −7 ⇒ reasoning is similar to that in the solution to
n cos n π = −7 ⇒ n = 7 ⇒ 0 ≤ t ≤ 7π . Problem 29.
1 7π 31. The function y = ( x − 2) −1 has a vertical
L=
π 0 ∫1 + t 2 dt ≈ 77.6508…
asymptote at x = 2, which is in the interval
The integral can be evaluated algebraically by [1, 3]. So the length is infinite. Mae’s partition
trigonometric substitution as in Section 9-6, of the interval skips over the discontinuity, as
giving shown in the graph.
∫ 1 + t 2 dt = t t 2 + 1 + ln t + 1 + t 2  + C.
1
2   ( ) 25
y

28. x = r cos t, dx = −r sin t dt


y = r sin t, dy = r cos t dt Mae's
error
dL = dx + dy = r sin t + r cos t dt
2 2 2 2 2 2

x
= r dt (for r ≥ 0) 1 2 3
The range 0 ≤ t ≤ 2π generates the entire circle.
2π 2π


32. The sample points are all of the form (n/2,
Circumference = r dt = rt = 2πr, Q .E.D .
0 0 sin nπ), which all lie on the x-axis and therefore
29. y = A sin x, dy = A cos x dx fail to measure the wiggly bits.

dL = dx 2 + dy 2 = 1 + A 2 cos 2 x dx y
Amos's sample points

Pick a convenient interval for x such as [0, 2π]. 1



L= ∫
0
1 + A 2 cos 2 x dx
0
x
10

A L
0 6.283185… (= 2π)
The length of the curve is 40 times the length of
1 7.640395… the part from x = 0 to x = 0.25 (by symmetry),
2 10.540734… so Amos could use five subintervals of [0, 0.25]
3 13.974417… to estimate the length of half of one arch, then

Calculus Solutions Manual Problem Set 8-5 195


© 2005 Key Curriculum Press
multiply his answer by 40 to find the total b. dy = cos x dx
length. dL = dx 2 + dy 2 = 1 + cos 2 x dx
33. See the Programs for Graphing Calculators
section of the Instructor’s Resource Book. dS = 2πy ⋅ dL = 2π sin x 1 + cos 2 x dx
π

Problem Set 8-6


S= ∫ 0
2π sin x 1 + cos 2 x dx ≈ 14.4235…
c. The circumscribed cylinder of length π
Q1. 1 + 9 x 4 dx Q2. 1 + sec 4 x dx and radius 1 has lateral area = 2π 2 =
19.7392… , which is a reasonable upper
1 6
Q3. sin x + C Q4. 156 bound for S.
6
3. The graph shows y = ln x, from x = 1 to x = 3,
Q5. xex + ex rotated about the x-axis.
Q6. Maximum y = 7 (at x = 1)
y
f ( x + ∆x ) – f ( x )
Q7. f ′( x ) = lim 1 (x, y )
∆x →0 ∆x
f ( x ) – f (c ) x
or f ′(c) = lim 1 3
x →c x–c
Q8. Instantaneous rate of change
1
Q9. ln | sec 2 x + tan 2 x | + C dy = x −1 dx
2
Q10. D dL = dx 2 + dy 2 = 1 + x –2 dx
1. a. The graph shows y = 0.5x2, from x = 0 to
x = 3, rotated about the y-axis. dS = 2π y ⋅ dL = 2π ln x 1 + x –2 dx
3
y
S= ∫ 2π ln x
1
1 + x –2 dx ≈ 9.0242 …

4. The graph shows y = ln x, from x = 1 to x = 3,


(x, y ) rotated about the y-axis, showing back half of
1
x
surface only.
0 3
y

dy = x dx
1
dL = dx 2 + dy 2 = 1 + x 2 dx
(x, y ) x
dS = 2πx ⋅ dL = 2πx 1 + x 2 dx 1 3
3
S= ∫ 2πx
0
1 + x 2 dx ≈ 64.1361…

b. The inscribed cone of height 4.5 and radius dL = 1 + x −2 dx, from Problem 3.
3 has lateral surface area = πrL =
dS = 2π x ⋅ dL = 2π x 1 + x –2 dx
π · 3 · 32 + 4.52 = 50.9722 … , which is
3
a reasonable lower bound for S.
3
S= ∫ 2π x
1
1 + x −2 dx ≈ 28.3047K
c. S = ∫ π (1 + x
2 1/2
0
) (2 x dx ) 5. The graph shows y = 1/ x = x −1,from x = 0.5 to
2 3
2 x = 2, rotated about the y-axis.
π (1 + x 2 )3/2 = π (10 10 − 1)
=
3 0 3 2
y

= 64.1361… , agreeing with the answer (x, y )

found numerically.
x
2. a. The graph shows y = sin x, from x = 0 to 2
x = π, rotated about the x-axis.
y
(x, y )
1
x dy = − x −2 dx
0 π
dL = dx 2 + dy 2 = 1 + x –4 dx

196 Problem Set 8-6 Calculus Solutions Manual


© 2005 Key Curriculum Press
dS = 2πx ⋅ dL = 2πx 1 + x –4 dx 3

2
S= ∫ 2πx 1 + (–3 x 2 + 10 x – 8)2 dx

0
S= 2πx 1 + x –4 dx ≈ 15.5181K
0.5 ≈ 58.7946…
6. The graph shows y = 1/x = x , from x = 0.5 to −1 9. The graph shows y = x = x 1/2, from x = 0 to
x = 2, rotated about the x-axis. x = 1, rotated about the x-axis.
y
y
2 1 (x, y )
(x, y )
x x
2 0 1

dL = 1 + x –4 dx, from Problem 5.


dy = 0.5 x −1/ 2 dx
dS = 2πy ⋅ dL = 2πx –1
1+ x –4
dx
2 dL = dx 2 + dy 2 = 1 + 0.25 x –1 dx

−1
S= 2πx 1+ x –4
dx ≈ 15.5181K
0.5 dS = 2πy ⋅ dL = 2πx 1/ 2 1 + 0.25 x –1 dx
(Note that surfaces 5 and 6 are congruent.)
= 2π x + 0.25 dx = 2π ( x + 0.25)1/ 2 dx
7. The graph shows y = x3, from x = 0 to x = 2,
1 4π 1
rotated about the y-axis.
y
S= ∫ 0
2π ( x + 0.25)1/2 dx =
3
( x + 0.25)3/2
0

8 4π
= (1.253/ 2 − 0.125) = 5.3304 K
3
10. The graph shows y = x3, from x = 1 to x = 2,
rotated about the x-axis, showing back half of
surface only.
(x, y )
8 y
x
0 2

dy = 3x2 dx
(x, y )
dL = dx 2 + dy 2 = 1 + 9 x 4 dx
dS = 2πx ⋅ dL = 2πx 1 + 9 x 4 dx 1
x
2 1 2
S= ∫ 2π x
0
1 + 9 x 4 dx ≈ 77.3245K

8. The graph shows y = −x3 + 5x2 − 8x + 6, from


x = 0 to x = 3, rotated about the y-axis.
y
6

dy = 3x2 dx
(x, y )
dL = dx 2 + dy 2 = 1 + 9 x 4 dx
x
dS = 2π y ⋅ dL = 2π x3 (1 + 9x4)1/2 dx
2


3
S= 2πx 3 (1 + 9 x 4 )1/2 dx
1

dy = (−3x2 + 10x − 8) dx π 2
dL = dx 2 + dy 2
=
18 1 ∫
(1 + 9 x 4 )1/ 2 (36 x dx )

π 2 2
= ⋅ (1 + 9 x 4 )3/ 2
= 1 + (–3 x 2 + 10 x – 8)2 dx 18 3 1
dS = 2π x ⋅ dL π
= (1453/ 2 – 10 3/ 2 ) = 199.4804 K
= 2π x 1 + (–3 x 2 + 10 x – 8)2 dx 27
Graph intersects x-axis where y = 0. 11. The graph shows y = x 4 / 8 + x −2 / 4, from x = 1
−x 3 + 5x 2 − 8x + 6 = −(x − 3)(x 2 − 2x + 2) = 0 to x = 2, rotated about the x-axis, showing back
at x = 3. side of surface only.

Calculus Solutions Manual Problem Set 8-6 197


© 2005 Key Curriculum Press
2
y
dL = (1 + x2) dx, from Problem 20 in Section 8-5
dS = 2π x ⋅ dL = 2π ( x + x3) dx
x 3
2π ( x + x 3 ) dx = π  x 2 + x 4 
3


1
1 2 S=
0  2  0
= 49.5π = 155.5088…
dy = (x3/2 − x− 3/2) dx = 0.5(x3 − x− 3) dx 14. The graph shows y = 2x1/3, from x = 1 to
x = 8, rotated about the y-axis, showing back
dL = dx 2 + dy 2 = 1 + 0.25( x 3 – x –3 )2 dx half of surface only.
= 1 + 0.25 x 6 – 0.5 + 0.25 x –6 dx y
−3
= 0.25( x + x ) dx = 0.5( x + x ) dx
3 –3 2 3

dS = 2π y ⋅ dL
4

= 2π ( x 4/8 + x − 2/4)[0.5(x3 + x− 3)] dx 2


(x, y )

π x
= ( x 7 + 3 x + 2 x –5 ) dx 1 8
8
π 2 7
S=
8 1 ∫
( x + 3 x + 2 x –5 ) dx
2 −2/3
π 1
2 dy = x dx
=  x 8 + x 2 – x –4 
3 1 3
8 8 2 2  1
4 –4 / 3
π dL = dx 2 + dy 2 = 1 +
=  32 + 6 –
1 1 3 1 155 x dx
– – + =4 π 9
8  32 8 2 2  256
= 14.4685… 4 –4 / 3
dS = 2π x ⋅ dL = 2π x 1 + x dx
12. The graph shows y = x2, from x = 0 to x = 2, 9
rotated about the y-axis. 8

∫x
4 –4 / 3
y
S = 2π 1+ x dx
1 9
4
1/ 2
x 1/ 3  x 4 / 3 + 
8


4
= 2π dx
1  9
(x, y )
1/2
8
 x 4/3 + 4 

3 4 1/3
= π ⋅
x
x dx
0 2
2 1  9 3
dy = 2 x dx 3/2 8
= π  x 4/3 + 
4
dL = dx 2 + dy 2 = 1 + 4 x 2 dx  9 1

dS = 2π x ⋅ dL = 2π x 1 + 4 x dx 2
π
2 = (1483/2 − 133/2 ) = 204.0435K
∫ x 1 + 4x
S = 2π 2
dx 27
0 1 1
π 2 π 2 15. The graph shows y = x 3 + x −1 , from x = 1 to
= ∫ (1 + 4 x ) 2 1/ 2
(8 x dx ) = (1 + 4 x 2 )3/2 3 4
4 0 6 0 x = 3, rotated about the line y = −1, showing
π back half of surface only.
= (173/ 2 − 1) = 36.1769K
6
1 y
13. The graph shows y = ( x 2 + 2)3/ 2 , from x = 0 to 9
3
x = 3, rotated about the y-axis. (x, y)

y
12

x
1 3

(x, –1)

(x, y )

1 x
0 3

198 Problem Set 8-6 Calculus Solutions Manual


© 2005 Key Curriculum Press
dx =  x 2 − x −2  dx
1 1 1

 4  b. i. S0,1 = ∫ 10π dx = 10πx


0 0
= 10π
2
dL = dx 2 + dy 2 = 1 +  x 2 − x −2  dx
1 2 2

 4  ii. S1,2 = ∫
1
10π dx = 10πx
1
= 10π
1 1
= 1 + x 4 − + x −4 dx 3 3
2 16
2
iii. S2,3 = ∫
2
10π dx = 10πx
2
= 10π

=  x 2 + x −2  dx =  x 2 + x −2  dx
1 1 4
4
 4 
dS = 2π ( y + 1) ⋅ dL
 4  iv. S3,4 = ∫
3
10π dx = 10πx
3
= 10π

= 2π  x 3 + x −1 + 1  x 2 + x −2  dx
5 5


1 1 1
v. S4,5 = 10π dx = 10πx = 10π
3 4  4  4 4

= 2π  x 5 + x 2 + x + x −2 + x −3  dx
1 1 1 1 c. The two features exactly balance each other.
3 3 4 16  The area of a zone of a sphere is a function
1 of the height of the zone only, and is
x + x −2 + x −3  dx
3

∫ 3x
1 1 1
S = 2π 5
+ x2 + independent of where the zone is located
1 3 4 16 
on the sphere.
3
= 2π  x 6 + x 3 + x 2 − x −1 −
1 1 1 1 1 −2  18. Suppose that the sphere is centered at the origin,
x
 18 3 6 4 32  1 as in Problem 17. The equation of a great
5 circle in the xy-plane is x2 + y2 = r2, from
= 101 π = 318.1735K
18 which y = r 2 − x 2 = (r 2 − x 2 )1/2 .
1 1 dy = −x(r2 − x2)− 1/2dx
16. The graph shows y = x 3 + x −1 , from x = 1 to
3 4 dL = dx 2 + dy 2 = 1 + x 2 (r 2 − x 2 ) −1 dx
x = 3, rotated about line x = 4.
dS = 2πy ⋅ dL
y
9
(x, y) = 2π (r 2 − x 2 )1/ 2 1 + x 2 (r 2 − x 2 ) −1 dx

(4, y ) = 2π r 2 − x 2 + x 2 dx = 2πr dx (if r > 0)


r r

x
S= ∫
−r
2πr dx = 2πrx
−r
= 4πr 2 , Q .E.D .
1 3 4
19. Pick a sample point in the spherical shell at
radius r from the center. Surface area at the
dL =  x 2 + x −2  dx, from Problem 15
1
  sample point is 4π r2. Volume of shell is
4
approximately (surface area)(thickness).
dS = 2π ( 4 − x ) ⋅ dL = 2π ( 4 − x ) x 2 + x −2  dx
1
dV = 4πr 2 ⋅ dr
 4  R
R


4 4

= 2π 4 x − x + x − x dx
2 3 −2 1 −1  V= 4π r 2 dr = π r 3 = π R 3 , Q .E .D .
 4  0 3 0 3

S = 2π  4 x 2 − x 3 + x −2 − x −1  dx
3


1 4 dV
20. V = πr 3 ⇒ = 4πr 2 = S, Q.E.D.
1  4  3 dr


3 dV
= 2π  x 3 − x 4 − x −1 − ln | x| or: V = S dr ⇒ = S by the definition of
4 1 1
3 4 4  1
dr
indefinite integral.
= 2π 15 − ln 3 = 94.6164 K
1 1
21. y = ax2, dy = 2ax dx
 3 4 
dL = dx 2 + dy 2 = (1 + 4 a 2 x 2 )1/ 2 dx
17. a. x 2 + y 2 = 25 ⇒ y = 25 − x 2 dS = 2πx ⋅ dL = 2πx (1 + 4 a 2 x 2 )1/ 2 dx
2 −1/ 2
dy = − x (25 − x ) dx r
S = 2π ∫ x(1 + 4a x )
2 2 1/ 2
dx
2 −1
dL = dx + dy = 1 + x (25 − x ) dx
2 2 2 0
π r

4a ∫
dS = 2π y ⋅ dL = (1 + 4 a x )
2
2 2 1/ 2
(8a 2 x dx )
0
= 2π 25 − x 2 1 + x 2 (25 − x 2 ) −1 dx π r
π
= (1 + 4 a 2 2 3/2
x ) = 2 [(1 + 4 a 2 r 2 )3/ 2 − 1]
= 2π 25 − x + x dx = 10π dx
2 2
6a 2
0 6a

Calculus Solutions Manual Problem Set 8-6 199


© 2005 Key Curriculum Press
22. Let h be the height of the paraboloid from the inappropriate for finding the arc length of an
vertex to the center of the base. Because h is the ellipse.
value of y when x = r, h = ar2. Substituting into For the surface area, however, the offending
the formula for S from Problem 21 gives denominator cancels out, giving
π dS = 0.24π 252 – 16 x 2 dx, which is defined
S = 2 [(1 + 4 ah)3/ 2 − 1]
6a at x = ±5.
Let a = 1 and evaluate S for various h. Find the 24. a. x = 35 sec t, dx = 35 sec t tan t dt
zone areas by subtracting. Use the TABLE feature. y = 100 + 80 tan t, dy = 80 sec2 t dt
y = 0 ⇔ 100 + 80 tan t = 0 ⇒ tan t = −5/4
h S Zone t = tan −1 ( −5 / 4)
π Radius at base is x = 35 sec [tan −1 ( −5/4)] =
0 (0) N.A.
6 56.0273… ≈ 56.0 ft.
π π b. At top, t = 0.5.
1 (10.1803K) (10.1803K) Radius: x = 35 sec 0.5 = 39.8822… ≈ 39.9 ft
6 6
Height: y = 100 + 80 tan 0.5 = 143.7041…
π π
2 (26) (15.8196 K) ≈ 143.7 ft
6 6
c. From the information given in parts a and b,
π π it can be assumed that −π /2 < t < π /2.
3 ( 45.8721K) (19.8721K)
6 6 dx
Minimize x: = 35 sec t tan t = 0 at t = 0
π π dt
4 (69.0927K) (23.2206 K) (or, because cos t has a max at t = 0,
6 6
sec t = 1/cos t has a minimum there).
π π
5 (95.2340 K) (26.1412 K) Minimum radius = 35 ft
6 6 Height = y = 100 + tan 0 = 100 ft
π π
6 (124) (28.7659K) d. dL = dx 2 + dy 2
6 6
= 352 sec 2 t ⋅ tan 2 t + 80 2 sec 4 t dt
The property is not true for paraboloids. The
areas of zones of equal height are greater if the dS = 2π x ⋅ dL
zone is farther away from the vertex.
= 2π (35 sec t ) 352 sec 2 t ⋅ tan 2 t + 80 2 sec 4 t dt
23. x = 5 cos t, dx = −5 sin t dt
y = 3 sin t, dy = 3 cos t dt 0.5

3 y
S= ∫
tan −1 ( −5/4 )
dS ≈ 37, 756.5934 … ≈ 37, 757 ft 2
(x, y ) 4
e. Volume ≈ S ⋅ = 12, 585.5311… ft 3 ≈
12
x
5
466.1307… yd3
25. From Figure 8-6m, a circle of radius L has area
πL2 and circumference 2πL. The circumference of
the cone’s base is 2πR. The arc length of the
dL = dx 2 + dy 2 = ( −5 sin t )2 + (3 cos t )2 dt sector of the circle of radius L must be equal to
dS = 2πy ⋅ dL this, so the sector is (2πR)/(2πL) = R/L of the
circle and has surface area S = πL2(R/L) = πRL,
= 2π (3 sin t ) ( −5 sin t )2 + (3 cos t )2 dt Q .E .D .
π
S= ∫0
6π sin t ( −5 sin t ) 2 + (3 cos t )2 dt 26. S = π RL − π rl
The objective is to get the lateral area in terms of
≈ 165.7930 … the slant height of the frustum, L − l.
From ( x/5)2 + ( y/3)2 = 1, y = ±0.6 25 − x 2 .
S = πR  L − ⋅ l
r
Using the upper branch of the graph,  R 
dy = −0.6 x (25 − x 2 ) −1/ 2 dx.
= πR  L − ⋅ l , because
l r l
= .
dL = dx 2 + dy 2 = 1 + 0.36 x 2 (25 − x 2 ) −1 dx  L  R L
At x = ±5, dL involves division by zero, which πR 2 2
= (L – l )
is awkward, and makes the Cartesian equation L

200 Problem Set 8-6 Calculus Solutions Manual


© 2005 Key Curriculum Press
πR c. dr = 3 cos θ dθ
= ( L + l )( L – l )
L dL = dr 2 + (r dθ )2
= π  R + R ⋅  ( L − l)
l
 L = (3 cos θ )2 + ( 4 + 3 sin θ )2 dθ

= π  R + R ⋅  ( L − l)

r L= ∫
0
dL ≈ 28.8141K
R
4. a. r = 5 − 3 cos θ. The calculator graph confirms
= π ( R + r)(L − l) that the text figure is traced out once as θ
R + r increases from 0 to 2π.
= 2π  ( L − l ), Q .E.D . 1
 2  b. dA = (5 – 3 cos θ )2 dθ
2

Problem Set 8-7


A= ∫
0
dA ≈ 92.6769… (exactly 29.5π )

c. dr = 3 sin θ dθ
Q1. 15x 2 − 14x + 4 Q2. 12(4x − 9)2 dL = dr 2 + (r dθ )2
Q3. 3 sin2 x cos x Q4. 3 sec 3x tan 3x
−x
= (3 sin θ )2 + (5 – 3 cos θ )2 dθ
Q5. − e Q6. −1/x 2

Q7. ln |x| + C Q8.


1 2
x +C
L= ∫
0
dL ≈ 34.3136 …
2
5. a. r = 7 + 3 cos 2θ. The calculator graph
Q9. 3x + C Q10. x + C confirms that the text figure is traced out once
1. a. r = 10 sin θ ⇒ dA = 50 sin2 θ dθ as θ increases from 0 to 2π.

A= ∫0
50 sin 2 θ dθ ≈ 157.0796 … 1
b. dA = (7 + 3 cos 2θ )2 dθ
2
(exactly 50π ) 2π

b. The area of the circle is π ⋅ 5 = 25π. 2 A= ∫


0
dA ≈ 168.0752 … (exactly 53.5π)
The calculated area is twice this because the c. dr = −6 sin 2θ dθ
circle is traced out twice as θ increases from 0
to 2π. Although r is negative for π < θ < 2π, dL = dr 2 + (r dθ )2
dA is positive because r is squared.
= (–6 sin 2θ )2 + (7 + 3 cos 2θ )2 dθ
2. a. r = 10 sin θ ⇒ dr = 10 cos θ dθ 2π

dL = dr + (r dθ )
2 2 L= ∫
0
dL ≈ 51.4511…

= 100 cos 2 θ + 100 sin 2 θ dθ = 10 dθ 6. a. r = 8 cos 2θ. The calculator graph confirms
that the text figure is traced out once as
2π 2π
θ increases from 0 to 2π.
L= ∫0
10 dθ = 10θ
0
= 20π
1
b. dA = (8 cos 2θ )2 dθ
The circumference is 2π ⋅ 5 = 10π. The 2


calculated length is twice this value because
A= dA ≈ 100.5309K (exactly 32π )
the circle is traced out twice as θ increases 0
from 0 to 2π. The calculus of this section c. dr = −16 sin θ dθ
always gives the dynamic answer as the
dL = dr 2 + (r dθ )2
distance traveled by a point on the curve as
θ increases from one value to another. This = (–16 sin 2θ )2 + (8 cos 2θ )2 dθ
path length does not necessarily equal the 2π
length of the curve. L= ∫
0
dL ≈ 77.5075K
3. a. r = 4 + 3 sin θ. The calculator graph confirms 7. a. 5 = 5 + 5 cos θ. The calculator graph
that the text figure is traced out once as θ confirms that the text figure is traced out once
increases from 0 to 2π. as θ increases from 0 to 2π.
1
b. dA = ( 4 + 3 sin θ )2 dθ 1
b. dA = (5 + 5 cos θ )2 dθ
2 2
2π 2π
A= ∫0
dA ≈ 64.4026 … (exactly 20.5π) A= ∫
0
dA ≈ 117.8097K (exactly 37.5π )

Calculus Solutions Manual Problem Set 8-7 201


© 2005 Key Curriculum Press
c. dr = −5 sin θ dθ 1
b. dA = ( 4 sec θ – 4 cos θ )2 dθ
2
dL = dr 2 + (r dθ )2 1

= (–5 sin θ ) + (5 + 5 cos θ ) dθ


2 2 A= ∫ −1
dA ≈ 4.5557K
2π (exactly 16 tan 1 − 24 + 4 sin 2)
L= ∫ 0
dL = 40 (exactly)
c. dr = 4(sec θ tan θ + sin θ) dθ
10
8. a. r = . The calculator graph dL = dr 2 + (r dθ )2
3 – 2 cos θ
confirms that the text figure is traced out once = 4 (sec θ tan θ + sin θ )2 + (sec θ – cos θ )2 dθ
as θ increases from 0 to 2π.
1
L= ∫ dL ≈ 10.9534 K
2
1 10 
b. dA =   −1
2  3 – 2 cos θ 
2π 11. r = 49 cos 2θ
A= ∫ 0
dA ≈ 84.2977K (exactly 12 5 π ) r = 0 ⇔ 2θ = cos −1 0 = ± π /2 + 2π n (n an integer)
–20 sin θ θ = ± π /4 + π n
c. dr = dθ The right-hand loop corresponds to
(3 – 2 cos θ )2
nonnegative values of the integrand,
dL = dr 2 + (r dθ )2 − π /4 ≤ θ ≤ π /4.
1
dA = ( 49 cos 2θ ) dθ
2 2
 –20 sin θ   10 
=  (3 – 2 cos θ )2  +  3 – 2 cos θ  dθ 2
    π /4 π /4


1
2π A= ( 49 cos 2θ ) dθ = 12.25 sin 2θ
L= ∫0
dL ≈ 33.0744 K − π /4 2 −π /4

= 24.5
9. a.
Area of both loops is 49.
1
12. The graph of r = csc θ + 4 shows a closed loop
from θ ≈ 3.4 to θ ≈ 6.
1
5

r = sin 3θ makes one complete cycle as θ 5

increases from 0 to π.
1
b. dA = (sin 3θ )2 dθ
2 The graph passes through the pole where r = 0.
π
csc θ + 4 = 0 ⇔ θ = csc −1 ( −4) =
A= ∫ 0
dA ≈ 0.7853K (exactly 0.25π )
sin −1 ( −0.25) = −0.2526 … + 2π n or
c. dr = 3 cos 3θ dθ [π − (−0.2526…)] + 2π n
dL = dr 2 + (r dθ )2 Desired range is 3.3942… ≤ θ ≤ 6.0305… .
= (3 cos 3θ )2 + (sin 3θ )2 dθ 1
dA = (csc θ + 4)2 dθ
π 2
L= ∫ dL ≈ 6.6824 K 6.0305K

10. a.
0 A= ∫
3.3942K
dA ≈ 8.4553…

13. r1 = 4 + 4 cos θ and r2 = 10 cos θ intersect where


4 4 + 4 cos θ = 10 cos θ
θ = cos −1 (2/3) = ±0.8410 … + 2π n.
(The graphs also touch at the pole, but not for
the same value of θ. For the cardioid,
4

θ = π + 2πn. For the circle, θ = π /2 + 2π n.)


Region outside the cardioid and inside

202 Problem Set 8-7 Calculus Solutions Manual


© 2005 Key Curriculum Press
the circle is generated as θ goes from
−0.841… to 0.841… . 4

1
dA = (r22 – r12) dθ 10
2
1
= [(10 cos θ )2 − ( 4 + 4 cos θ )2 ] dθ
2
0.841K
A= ∫
−0.841K
dA ≈ 18.8863K r = 4 + 6 cos θ = 0 ⇔ cos θ = −2/3
−1
θ = cos −1 ( −2/3) = ±2.3005… + 2πn
(exactly 26 cos (2/3) − (4/3) 5) 1
dA = ( 4 + 6 cos θ )2 dθ
14. r1 = 5 and r2 = 5 − 5 cos θ intersect at θ = −π /2 2
and π /2. The outer loop is swept out as θ increases from
−2.3005… to 2.3005.
2.3005K


5
A1 = dA ≈ 105.0506 K
–2.3005K

5
The inner loop is generated as θ increases from
2.3005… to 3.9826… .
3.926K
A2 = ∫2.3005K
dA ≈ 1.7635K
Area of the region between the loops is
1 A 1 − A 2 ≈ 103.2871… .
dA = (r12 – r22 ) dθ
2 17. a.
1
= [52 − (5 − 5 cos θ )2 ] dθ 4
2
Integrate from −π /2 to π /2, because in Quadrants
II and III the cardioid lies outside the circle. 2
π /2
A= ∫
− π /2
dA ≈ 30.3650 K (exactly 50 − 6.25π )

15. a. r = 0.5θ. The graph starts at θ = 0 and makes


three revolutions, so θ increases from 0 to 6π. dr = −2.5θ −1.5 dθ
dr = 0.5 dθ dL = dr 2 + (r dθ )2 = 6.25θ –3 + 25θ –1 dθ

dL = dr 2 + (r dθ )2 = 0.52 + (0.5θ )2 dθ ∫
π /2
dL ≈ 31.0872 …

L= ∫ 0
dL ≈ 89.8589… b. The graph shows sectors of central angles 1,
2, and 3 radians.
1 1
b. dA = (0.5θ )2 dθ = θ 2 dθ
2 8
Area swept out for third revolution in
Quadrant I is
4.5π 4.5π


1 2 1 217 3
A3 = θ dθ = θ 3 = π
4π 8 24 4π 192
Area swept out for second revolution in
Quadrant I is 1 2
2.5π 2.5π
Area of sector is A(θ ) = r θ.

1 2 1 61 3 2
A2 = θ dθ = θ 3 = π 1 2
2π 8 24 2π 192 A(1) = (5) (1) = 12.5
2
Area of region between second and third 1
revolution in Quadrant I is A3 = A2 = A(2) = (3.5355...)2 (2) = 12.5
2
13 3
π = 25.1925… . 1
A(3) = (2.8867K)2 (3) = 12.5
16 2
16. The graph of r = 4 + 6 cos θ shows a closed loop 1
from θ ≈ 2.3 to θ ≈ 4.0. In general, A(θ ) = (5θ –1/ 2 )2 (θ ) = 12.5, which
2
is independent of the value of θ.

Calculus Solutions Manual Problem Set 8-7 203


© 2005 Key Curriculum Press
18. The graph shows r = sec θ and a segment from circles of radius 10.6 cm. L ≈ 800(2π · 10.6) =
θ = 0 to 1.5. 16,960π cm, which is very close to the calculated
16,960.0002…π cm.
(The integral can be evaluated algebraically by
10 the tangent trigonometry substitution from
Chapter 9. The result, 16,960.00021…π, is
remarkably close both to the numerical answer
and to the sum of the lengths of the 800 circles
1 of average radius 10.6 cm.)
100
The point with polar coordinates (r, θ) has xy- 20. a. r = = 100(3 − 2 cos θ ) −1
3 – 2 cos θ
coordinates x = r cos θ, y = r sin θ. The graph
given by r = sec θ can be written 1
dA = [100(3 – 2 cos θ ) –1 ]2 dθ
x = r cos θ = sec θ cos θ = 1 2
y = r sin θ = sec θ sin θ = tan θ = 5000(3 − 2 cos θ ) −2 dθ
(i.e., −∞ < y < ∞). Thus, this graph is the 0.2
line x = 1. A= ∫ 0
dA ≈ 974.3071… ≈ 974 (kilo-mi)2
By calculus, the segment from θ = 0 to θ = 1.5
θ
has length as follows:
dr = sec θ tan θ dθ
b. Solving ∫0.8
5000(3 − 2 cos t ) −2 dt =
974.3071… gives θ ≈ 1.88976… .
dL = dr 2 + (r dθ )2
c. P = ka1.5
= (sec θ tan θ )2 + sec 2 θ dθ (27.3)(24) = k(240)1.5
= sec θ tan 2 θ + 1 dθ = sec 2 θ dθ k = 0.17622…
1.5 1.5 d. The major axis of the spaceship’s orbit is
L= ∫
0
sec 2 θ dθ = tan θ
0
120 thousand miles, so a = 60.
P = k · 601.5 = 81.9 hours (precise answer)
= tan 1.5 − 0 = 14.1014…
e. The total area of the ellipse is
As shown above, y = tan θ.

At θ = 1.5, y = tan 1.5, confirming the calculus.
19. A typical record has grooves of inner radius
A= ∫ 0
5000(3 − 2 cos θ ) −2 dθ
= 8429.7776… (kilo-mi)2
6.6 cm and outer radius 14.6 cm, and takes
about 24 minutes to play. There are thus Fraction of area from θ = 0 to θ = 0.2 is
(33.333…)(24) or about 800 grooves in a space (974.3071…)/(8429.7776…) = 0.1155… .
of (14.6 − 6.6) or 8.0 cm. Thus, the grooves This fraction is the same as the fraction of the
decrease in radius by about 8.0/800 = 0.01 cm period. Thus, the time is 0.1155…(81.9) =
per revolution. A simple equation of the spiral is 9.4659… hours to go from θ = 0 to θ = 0.2,
0.01 0.005 and the same for θ to go from 0.8 to
r= θ= θ
2π π 1.88976… .
which assumes that the grooves start at the center f. dr = −100(3 − 2 cos θ ) −2 ⋅ (2 sin θ ) dθ
and have a pitch of 0.01 cm. The innermost
actual groove is at θ = 6.6π/0.005 = 1320π, = −200 sin θ (3 − 2 cos θ ) −2 dθ
and the outermost groove is at θ = 14.6π/0.005 dL = dr 2 + (r dθ )2
= 2920π .
dr = −(0.005/π) dθ = [( −200 sin θ (3 − 2 cos θ ) −2 ) 2
dL = dr 2 + (r dθ )2 + (100(3 − 2 cos θ ) −1 )2 ]1/2 dθ
From θ = 0 to θ = 0.2,
= (0.005/π ) 2 + [(0.005/π )θ ]2 dθ 0.2

=
0.005
1 + θ 2 dθ
L= ∫0
dL ≈ 20.2228K ≈ 20.2 kilo-mi.
π From θ = 0.8 to θ = 1.88976… ,
2920 π
L= ∫
1320 π
dL ≈ 53, 281.4120 … cm
L= ∫
1.88K
dL ≈ 56.7896 K ≈ 56.8 kilo-mi.
= 16,960.0002…π cm 0.8

Rough check: Average radius = 10.6 cm g. Average speed from θ = 0 to θ = 0.2 is


L should equal approximately the sum of 800 20.2228K
= 2.1363K , or about 2136 mi/h.
9.4659K

204 Problem Set 8-7 Calculus Solutions Manual


© 2005 Key Curriculum Press
Average speed from θ = 0.8 to dr dx dy
56.7896 K e. r 2 = x 2 + y 2 ⇒ 2 r = 2x + 2y
θ = 1.88976… is = 5.9993K , dθ dθ dθ
9.4659K dr dx dy
or about 5999 mi/h. ⇒r =x +y
dθ dθ dθ
h. When the spaceship is farthest from Earth, its Substitute these expressions in parts d and e
radial velocity (toward the Earth) is zero. As into the top and bottom of the expression in
it proceeds in its orbit, it can be thought of as part c to show the property.
falling toward the Earth, thus picking up
r a – a cos θ 1 – cos θ
speed. The reverse is true on the other side of f. tan ψ = = = =
the Earth, where it is moving away and is dr/dθ a sin θ sin θ
thus being slowed by gravity. tan θ/2, using the half-angle formula. Then
21. a. Count 5 spaces to the right and about 7.5
ψ = θ/2 + nπ. But 0 ≤ θ ≤ 2π, and 0 ≤ ψ ≤ π ,
which implies n = 0, so ψ = θ/2.
spaces down from the given point.
Slope ≈ −1.5. r dr
g. tan ψ = ⇒ = const ⋅ r ⇒ r = Cekθ
b. r = θ dr/dθ dθ
x = θ cos θ ⇒ dx = dθ · cos θ − θ sin θ dθ dr dr/dθ
Note that = kCe kθ = kr ⇒ k = =
y = θ sin θ ⇒ dy = dθ · sin θ + θ cos θ dθ dθ r
dy dy/dθ sin θ + θ cos θ 1
= cot ψ .
= = tan ψ
dx dx/dθ cos θ – θ sin θ
At θ = 7, dy/dx = −1.54338… , thus Equations for the spiral will vary.
confirming the answer found graphically.
22. a. x = r cos θ, y = r sin θ
⇒ y/x = sin θ /cos θ = tan θ Problem Set 8-8
slope =
r sin Review Problems
r cos
R0. Answers will vary.
= tan
r R1. a.
r sin
50 y f
r cos
g

b. The slope of any line is tan φ, where φ is the h

angle between the x-axis and the line. x


And, because the tangent line has slope –10
3

dy dy/dθ
= ( by the chain rule),
dx dx/dθ b. f ′ (x) = 3x 2 − 18x + 30; f ″(x) = 6x − 18
dy/dθ g′ (x) = 3x2 − 18x + 27; g″(x) = 6x − 18
tan φ = .
dx/dθ h′ (x) = 3x2 − 18x + 24; h″(x) = 6x − 18
tan φ – tan θ
c. tan ψ = tan (φ − θ) = c. h′ (x) = 3(x − 2)(x − 4) = 0 at x = 2 and 4
1 + tan φ tan θ
h″(2) = −6 < 0, so h has a local maximum
dy/dθ y at x = 2.

dx/dθ x  x dx/dθ  h″(4) = 6 > 0, so h has a local minimum
= ⋅
dy/dθ y  x dx/dθ  at x = 4.
1+ ⋅
dx/dθ x
d. g′ (x) = 3(x − 3)2 = 0 only at x = 3.
dy dx
x –y g′ (x) > 0 on both sides of x = 3, so this is
= d θ d θ neither a maximum nor a minimum point.
dx dy
x +y
dθ dθ e. From the graphs, each point of inflection
d. dx/dθ = −r sin θ; appears at x = 3. Because each second
dy dx derivative equals 6x − 18, each one equals
dy/dθ = r cos θ x −y
dθ dθ zero when x = 3.
= r cos θ ⋅ (r cos θ ) − r sin θ ⋅ ( − r sin θ )
= r2 cos2 θ + r2 sin2 θ = r2

Calculus Solutions Manual Problem Set 8-8 205


© 2005 Key Curriculum Press
R2. a. Minimum at (0, 0). Maximum at (2, 0.5413…).
f (x )
no max.
or min. f ″(x) = 0 at x = 2 + 2 = 3.4142 … and at
f´(x) + undef. + x = 2 − 2 = 0.5857…
x 2
f ″(x) changes sign at each of these x-values,
p.i.
which implies points of inflection at
f (x )
+ undef. –
(0.5857… , 0.1910…), (3.4142… ,
f´´(x)
x 2
0.3835…).
R3. a. Let x = width of a cell, y = length of the cell.
b.
f (x ) xy = 10 ⇒ y = 10 x −1 ; 0 < x
Minimize L( x ) = 12 x + 7 y = 12 x + 70 x −1 .
The graph shows minimum L (x) at x ≈ 2.4.
L (x )
x
–2 1 3 5
100
2 –2 −4/3
c. i. f ′( x ) = x −1/3 − 1, f ′′( x ) = x
3 9 x
ii. Zooming in shows that there is a local 2

minimum cusp at (0, 0) and a local


L ′( x ) = 12 − 70 x −2
maximum with zero derivative at x ≈ 0.3.
f (x ) L ′( x ) = 0 at x = 70/12 = 2.4152 …
0.5
At x = 70/12 , y = 10 12/70 = 4.1403… .
Overall length of battery is 6(2.4152…) =
x 14.4913… .
0 1
Optimal battery is about 14.5″ by 4.1″,
which is longer and narrower than the typical
Algebraically, f ′ (x) = 0 at x = (2/3)3 = battery, 9″ by 6.7″. Thus, minimal wall
8/27, and f ′ (x) is undefined at x = 0, thus length does not seem to be a major
locating precisely the minimum and consideration in battery design.
maximum found by graphing. b. The graph shows y = 8 − x3, from x = 0 to
Because there are no other critical values x = 2, with rectangle touching sample point
of x, there are no other maximum or (x, y) on the graph, rotated about the y-axis,
minimum points. generating cubic paraboloid and inscribed
cylinder.
iii. f ″(x) is undefined at x = 0, and f ″(x) < 0
everywhere else; f ″ never changes sign, so 8
y

there are no inflection points.


iv. f (0) = 0, f (8/27) = 4/27, f (5) = −2.0759… (x, y )
Global maximum at (8/27, 4/27).
Global minimum at (5, −2.0759…).
d. f (x )
1
x
2

x
Domain of x is 0 ≤ x ≤ 2.
2
Maximize V (x) = π r2h = π x 2y = 8π x2 − π x 5 .
The graph shows that V (x) has a maximum at
x ≈ 1.5.
The graph shows that f ( x ) = x 2 e − x has local
minimum at x = 0, local maximum at x ≈ 2, V(x )

and points of inflection at x ≈ 3.4 and at 30

x ≈ 0.6.
f ′( x ) = 2 xe − x − x 2 e − x = x (2 − x )e − x
f ″ ( x ) = 2e − x − 4 xe − x + x 2 e − x = (2 − 4 x + x 2 )e − x x
f ′ (x) = 0 at x = 0, 2 0 2

206 Problem Set 8-8 Calculus Solutions Manual


© 2005 Key Curriculum Press
V ′ (x) = 16π x − 5π x 4 = π x(16 − 5x3) ii. The graph shows the region described in
V ′( x ) = 0 at x = 0 and x = 3 16/5 = 1.4736 … part i, rotated about the x-axis, showing
back half of solid only.
Maximal rectangle has x = 3 16/5 = 1.4736 … , y
y = 8 − 16/5 = 4.8. (x, 4)

R4. a. The graph shows y = x11/3 and y = x 22 ,


4
(x, y )

intersecting at (0, 0) and (1, 1), rotated about


the x-axis, sliced parallel to the x-axis, x
showing back half of solid only. –2 2

y
(x 1 , y )
1

(x 2 , y )
dV = π (42 − y2) dx = π (16 − x4) dx
x 2
0 1 V= ∫ –2
dV ≈ 160.8495… (exactly 51.2π)
(Cylindrical shells can also be used.)
iii. The graph shows the region described in
part i, rotated about the line y = 5,
showing back half of solid only.
x 1 = y 3 , x 2 = y 1/2
y
dV = 2π y (x 2 − x 1) · dy = 2π y (y 1/2 − y 3) dy
1
V= ∫ dV ≈ 1.2566… (exactly 0.4π )
0

b. The graph shows y1 and y2 as in part a, but (x, 4)


5
sliced perpendicular to the x-axis, generating
plane washer slices. 4
(x, y )
y

1 (x, y1 ) x
–2 2

(x, y2 )
x dV = π [(5 − y)2 − 12] dx
0 1 = π [(5 − x2)2 − 1] dx
V = dV ≈ 174.2536 …  exactly 55 π 
2


7
–2  15 
(Cylindrical shells can also be used.)
iv. The graph shows the region described in
dV = π ( y12 − y22 ) dx = π ( x 2/3 − x 4 ) dx part i, rotated about the line x = 3,
1 showing back half of solid only.
V= ∫ dV ≈ 1.2566… (exactly 0.4π), which is
0
y
(x, 4)
the same answer as in part a, Q.E.D. 4

c. i. The graph shows y = x2 and y = 4,


intersecting at (2, 4) and (−2, 4), rotated (x, y ) (3, y)
x
about the y-axis, showing back half of –2 2 3
solid only.
dV = 2π (3 − x) · (4 − y) · dx
y
(x, 4) = 2π (3 − x)(4 − x2) dx
2
4 V= ∫ –2
dV ≈ 201.0619… (exactly 64π)

(x, y ) (Washers can also be used.)


x R5. a. y = x 2 from x = −1 to x = 2.
–2 2
dy = 2x dx
dV = 2π x (4 − y) · dx = 2π x (4 − x2) dx dL = dx 2 + dy 2 = 1 + (2 x )2 dx
V ≈ 25.1327… (exactly 8π) 2

(Disks can also be used.) L= ∫ –1


dL ≈ 6.1257…

Calculus Solutions Manual Problem Set 8-8 207


© 2005 Key Curriculum Press
b. y = x3/2 from x = 0 to x = 9. 3 8 1/2
 x 4 / 3 + 1   4 x 1/3 dx 
dy = 1.5x1/2 dx =
2
π ∫ 0 9  3 
dL = dx 2 + dy 2 = 1 + (1.5 x 1/2 )2 dx 3/2 8
= π  x 4/3 + 
1
9
 9
L= ∫ (1 + 2.25x ) dx
1/2
0
0
π
1 9
= (1453/ 2 – 1) = 203.0436 K
2.25 ∫
= (1 + 2.25 x ) 1/2
(2.25 dx ) 27
0
2 9 The disk of radius 8 has area 64π =
= (1 + 2.25 x )3/2 201.0619… , so the answer is reasonable.
6.75 0
2 b. The graph shows y = tan x, from x = 0 to
= (21.25 – 1) = 28.7281…
3/ 2
x = 1, rotated about the line y = −1, showing
6.75
Distance between the endpoints is the back half of the solid only.
10 2 + 26 2 = 27.8567… , so the answer y

is reasonable. 1
(x, y)
c. x = t cos π t ⇒ dx = (cos π t − π t sin π t ) dt x
y = t sin π t ⇒ dy = (sin π t + π t cos π t ) dt 0 1
The graph shows t increases from 0 to 4.
(x,–1)
4 y

x
4

dL = dx 2 + dy 2 dy = sec2 x dx
dL = dx 2 + dy 2 = 1 + sec 4 x dx
= (cos π t – π t sin π t )2 + (sin π t + π t cos π t )2 dt
dS = 2π (y + 1) · dL
= 1 + (π t )2 dt
4
= 2π ( tan x + 1) 1 + sec 4 x dx
L= ∫
0
1 + π 2 t 2 dt ≈ 25.7255K
S=
1

∫ dS ≈ 20.4199K
0
R6. a. The graph shows y = x1/3 , from x = 0 to
x = 8, rotated about the y-axis, showing the
back half of the solid only. R7. a. r = θ ⇒ dr = d θ
y dL = dr 2 + (rdθ )2 = 1 + θ 2 dθ
5π / 2
(0, y) 2
(x, y )
x
L= ∫ 0
dL ≈ 32.4706 K
1 2 1
b. dA = r dθ = θ 2 dθ . Area of the region
0 8

2 2
between the curves equals the area traced out
1 −2/3 from t = 2π to t = 5π /2 minus the area traced
dy = x dx out from t = 0 to t = π /2.
3 5π / 2 1 π /2 1

dL = dx 2 + dy 2 = 1 +  x –2/3  dx
1
2 A=
2π ∫ 2
θ 2 dθ −
0 2 ∫
θ 2 dθ
3  5π /2 π /2
1 1
2
= θ3 − θ3
dS = 2πx ⋅ dL = 2πx 1 +  x –2/3  dx
1 6 2π 6 0
3  1 3 7.5 3
= π (2.53 − 2 3 − 0.53 + 0 3 ) = π
2 6 6
1 +  x –2/3  dx
8 1
S = 2π ∫x
0 3  = 38.7578…
1/2
x 1/3  x 4/3 +  dx
8 1
= 2π ∫
0  9

208 Problem Set 8-8 Calculus Solutions Manual


© 2005 Key Curriculum Press
y

Concept Problems
C1. a. The graph of µ(t) = 130 − 12T + 15T 2 − 4T 3 f

from T = 0 to T = 3 shows maxima at T = 0 1 g


x
and T ≈ 2.0 and minima at T ≈ 0.5 and 0 1
T = 3.
µ (T ) 2
 π 
C4. a. y = 3 + 1.25 1 + cos  ( x – 5) 
130
 3 
y′ =
 π  π π
T 2.5 1 + cos  ( x – 5)  − sin  ( x – 5) 
0 3   3    3  3
–2.5π  π   π 
To maximize µ(T): = 1 + cos  3 ( x – 5)  sin  3 ( x – 5) 
3   
µ′ (T ) = −12 + 30T − 12T 2
= −6(2T − 1)(T − 2) –2.5π
2
dL = 1 +  (1 + cos χ )sin χ  dx
1  3 
µ′ (T ) = 0 at T = , 2
2
 where χ temporarily stands for π ( x – 5)
µ(0) = 130; µ   = 127.25;
1  
3
 2 7.5
µ(2) = 134; µ(3) = 121 L= ∫
5
dL ≈ 5.7726…
Maximum viscosity occurs at T = 2, or 200°.
b. y ′′ =
b. Minimum viscosity occurs at endpoint, –2.5π dχ
T = 3, or 300°. [(1 + cos χ ) cos χ + ( − sin χ ) sin χ ]
3 dx
c. –2.5π π
= (2 cos 2 χ − 1 + cos χ ) ⋅
C2. The graph of f (x) = (x − 1)4 + x shows that the 3 3
graph straightens out at x = 1 but does not –2.5π 2

change concavity. = (cos χ + 1)(2 cos χ – 1)


9
y y ″ = 0 ⇔ cos χ = −1 or cos χ = 0.5
χ = π + 2π n or χ = ± π /3 + 2π n
x = 8 + 6n, 4 + 6n, or 6 + 6n
The only zero of y ′′ within the domain is
1 x = 6, so the point of inflection must be
x
at x = 6.
1
c. dS = 2π (x − 4) dL, where dL is as in part a.
7.5

f ′ (x) = 4(x − 1)3 + 1; f ″(x) = 12(x − 1)2, S= ∫ 5


dS ≈ 78.2373…
so f ′ (1) = 1 and f ″(1) = 0. 2
5π 
f ″(x) > 0 for all x ≠ 1. In particular, f ″(x) does not d. x = 7.5 ⇒ y = 3 + 1.25 1 + cos
change sign at x = 1. Thus, the graph is straight  6 
at x = 1, but not horizontal. Zooming in on (1, = 3 + 1.25(1 − 3 /2)2
1) shows that the graph resembles y = x when x = 3 + 1.25(1.75 − 3 ) = 3.0224 …
is close to 1, although it is actually concave up
dV = 2π (x − 4) · (y − 3.0224…) · dx
slightly. 7.5
V= ∫ 5
2π ( x − 4)1.25[(1 + cos χ )2 − (1.75 − 3 )] dx
C3. The graphs of f (x) = x2/3 and g( x ) = x −1/3 show a = 58.8652 …
cusp at x = 0 for function f and a vertical
asymptote at x = 0 for function g.
C5. The 2000 World Almanac and Book of Facts
lists the area of Brazil as 3,286,478 square miles.
Individual answers will vary.

Calculus Solutions Manual Problem Set 8-8 209


© 2005 Key Curriculum Press
C6. Let the cylinder lie on the x-axis and the hole lie Because the spike goes from x = 0 to x = 2 /2,
on the y-axis so that the z-axis is perpendicular 2 /2

to both the cylinder and the hole. The cylinder is Vs = ∫ 0


dVs ≈ 0.0109642 … .
thus described by y2 + z2 ≤ 25, and the hole by (The integral can be evaluated algebraically using
x 2 + z2 ≤ 9. trigonometry substitution, as in Chapter 9. The
Slice the hole with planes perpendicular to the 11 2 π 1
z-axis. Then for −3 ≤ z ≤ 3, the cross section at exact value is Vs = − − .)
12 4 2
z of the hole is a rectangle with height The 24 spikes (3 for each of the eight corners) are
2 y = 2 25 – z 2 and width 2 x = 2 9 – z 2 . identical.
Area of cross-section rectangle is Thus, the total volume remaining is
4 225 – 34 z 2 + z 4 , V = 8Vc + 24Vs
so dV = 4 225 – 34 z 2 + z 4 dz.  11 2 π 1 
= 8(1 − 2 /2)3 + 24  – – 
Thus, the volume of the hole (and thus of the  12 4 2
uranium that once filled the hole) is
3 = 8 + 8 2 − 6π = 0.46415… cm 3
V= ∫ −3
4 225 – 34 z 2 + z 4 dz
= 269.3703… cm3 Chapter Test
According to the CRC Handbook, the density
of uranium is 19.1 g/cm3. So the mass of the T1.
uranium drilled out is y
m = (269.3703…)(19.1) = 5144.97… g.
Value is 200(5144.97…) ≈ $1,029,000. 3

C7. Draw x- and y-axes with origin at the center of


x
the circle on one face of the cube.
0 1 2 3 4
y

T2.
1
y
(x, y ) 6
Function
5

4
x
1 3

The solid remaining consists of eight identical 1


corner pieces. Each corner piece consists of a cube x
and three identical spikes. The spikes have square 1 2 3 4 5 6 7
cross sections when sliced perpendicular to the
appropriate axes. The hole perpendicular to the Derivative
xy-plane cuts a circle in that plane with equation
x2 + y2 = 1. The cube shown in the diagram
T3. A = xy = x(500 − 0.5x) = 500x − 0.5x2
begins at x = y so that 2x2 = 1,
A′ = 500 − x
from which x = 2 /2. Each cube is thus
A′ = 0 ⇔ x = 500
(1 − 2 /2) cm on a side, and thus has volume A′ goes from positive to negative at x = 500
Vc = (1 − 2 /2)3 = 0.0251262 … cm 3 . ⇒ local maximum at x = 500.
Consider the leftmost spike in the preceding A(0) = A(1000) = 0 ⇒ global maximum
diagram. Pick a sample point (x, y) on the part at x = 500.
of the circle in that spike. The cross section Maximum at x = 500, y = 250.
perpendicular to the x-axis for this spike is a
square of side (1 − y) = 1 − 1 – x 2 . Thus,
dVs = (1 − 1 – x 2 )2 dx.

210 Problem Set 8-8 Calculus Solutions Manual


© 2005 Key Curriculum Press
b T11. Vcyl = π · 22 · 8 = 32π
T4. V = 2π ∫a
x ( y1 − y2 ) dx
Vsolid 19.2π
= = 0.6
1 Vcyl 32π
T5. a. dA = r 2 dr
2 So, V solid = 0.6V cyl .
b. dL = ( dr ) + (r dθ ) 2 2 T12. a.
y
c. dL = dx 2 + dy 2 2

d. dS = 2π ( x − 1) dL x
T6. f (x) = x 3 − 7.8x 2 + 20.25x − 13 5

f ′ (x) = 3x2 − 15.6x + 20.25


= 3(x − 2.5)(x − 2.7)
f ′ (x) changes from positive to negative at
x = 2.5 and from negative back to positive at b. dx = −5 sin t dt, dy = 2 cos t dt
x = 2.7. So there is a local maximum at x = 2.5
dL = dx 2 + dy 2
and a local minimum at x = 2.7.
f ″(x) = 6x − 15.6 = 6(x − 2.6) = (–5 cos t )2 + (2 sin t )2 dt
f ″(x) = 0 at x = 2.6 2π
f ′ (2.6) = −0.03, so the graph is not horizontal at
the inflection point.
L= ∫
0
dL ≈ 23.0131…

T7. y = x3 ⇒ dy = 3x2 dx c. Slicing perpendicular to the x-axis generates


circular slices of radius y, where sample point
dL = dx 2 + dy 2 = 1 + (3 x 2 )2 dx (x, y) is on the upper branch of the ellipse.
2 dV = π y2 dx = 4π sin2 t (−5 sin t dt)
L= ∫
0
1 + 9 x 4 dx = 8.6303… = −20π sin3 t dt
Leftmost slice is at t = π, and rightmost slice
T8. dS = 2π x · dL = 2π x 1 + 9 x 4 dx is at t = 0.
2
S= ∫ 2π x
0
1 + 9 x 4 dx = 77.3245… V= ∫
π
0
−20π sin 3 t dt = 83.7758…
T9. V(x) = π x 2(8 − y) = 8π x2 − π x 5 = 26.6666 …π (numerically)
The graph shows a maximum V (x) at x ≈ 1.5. V can be evaluated algebraically by
40
y transforming two of the three sin t factors
into cosines.
0
V= ∫
π
−20π (1 − cos 2 t ) sin t dt
0 0
= ∫ −20π sin t dt + ∫ 20π cos
x 2
0 1 2
t sin t dt
π π
0
=  20π cos t − π cos3 t  = 26 π
20 2
V′(x) = 16π x − 5π x 4 = 0 at x = 0 or 3.21/3
 3  π 3
V(0) = V(2) = 0
V(3.2 1/3) = 4.8 · 3.22/3π > 0, so this is a The x-radius is 5, and the y-radius is 2.
4 4
maximum. π (x-radius)(y-radius)2 = π (5)(2)2 =
Maximal cylinder has V = 4.8 · 3.22/3π cm3 = 3 3
2
32.7459… cm3. 26 π , Q .E.D .
3
T10. Slicing parallel to the y-axis generates cylindrical
(In general, if a = x-radius and b = y-radius,
shells of radius x extending from the sample
the parametric functions are x = a cos t, y =
point (x, y) to the line y = 8.
b sin t. Repeating the preceding algebraic
dV = 2π x · (8 − y) · dx = 2π x (8 − x3) dx 4
2 2 solution gives V = πab 2 .)

V= 2π (8 x − x 4 ) dx = 2π ( 4 x 2 − 0.2 x 5 ) 3
0 0 T13. r = 5e0.1 θ
= 19.2π = 60.3185… dr = 0.5e0.1 θ dθ

Calculus Solutions Manual Problem Set 8-8 211


© 2005 Key Curriculum Press
dL = dr 2 + (rdθ )2 minus the area swept out for the second
revolution. In Quadrant I, the third revolution
= (0.5e 0.1θ )2 + (5e 0.1θ )2 dθ extends from θ = 4π to θ = 4.5π and the second
= e 0.1θ 25.25 dθ revolution extends from θ = 2π to θ = 2.5π .
The spiral starts at r = 5 = 5e0.1·0 and makes three 4.5π 2.5π

complete revolutions, so 0 ≤ θ ≤ 6π.


A= ∫4π
12.5e 0.2θ dθ − ∫2π
12.5e 0.2θ dθ
6π 6π 4.5π 2.5π
= 62.5e 0.2θ 4π − 62.5e 0.2θ 2π
L= ∫
0
e 0.1θ 25.25 dθ = 10e 0.1θ 25.25
0 = 62.5(e0.9 π − e0.8 π − e0.5 π + e0.4 π )
= 10 25.25 (e 0.6π − 1) = 280.6961… = 203.7405…
1 T15. Answers will vary.
T14. dA = r 2 dθ = 12.5e 0.2θ dθ
2
The area between the second and third revolutions
equals the area swept out for the third revolution

212 Problem Set 8-8 Calculus Solutions Manual


© 2005 Key Curriculum Press
Chapter 9—Algebraic Calculus Techniques
for the Elementary Functions

Problem Set 9-1


π /2
2. ∫ x cos 3x dx u=x dv = cos 3x dx


1
1. V = 2π x ⋅ cos x dx ≈ 3.5864 … du = dx v = sin 3 x
0 3
2. f (x) = x sin x ⇒ f ′(x) = x cos x + sin x

1 1
= x sin 3 x − sin 3 x dx
∫ ∫ ∫
3 3
3. f ′( x ) dx = x cos x dx + sin x dx 1 1
= x sin 3 x + cos 3 x + C
∫ x cos x dx = ∫ f ′( x ) dx − ∫ sin x dx
3 9
4.
∫ xe u=x dv = e4x dx
4x
3. dx
= f (x) + cos x + C (by definition of indefinite
1 4x
integral) du = dx v= e
= x sin x + cos x + C 4


π /2 1 4x 1 4x
= xe −
5. V = 2π ∫ 0
x cos x dx 4
1
4
1
e dx

= 2π x sin x + 2π cos x π0 /2 = xe 4 x − e 4 x + C
4 16
= π 2 − 2π
∫ 6 xe
−3 x
4. dx u = 6x dv = e− 3x dx
6. V = π2 − 2π = 3.5864… , which is the same as
the approximation, to the accuracy shown. 1
v = − e −3 x
du = 6 dx
7. The method involves working separately with the 3
= (6 x ) − e −3 x  − (6) − e −3 x  dx

different “parts” of the integrand. The function 1 1
f (x) = x sin x was chosen because one of the  3   3 
terms in its derivative is x cos x, which is the 2
= −2 xe −3 x − e −3 x + C
original integrand. See Section 9-2. 3

∫ ( x + 4)e
−5 x
5. dx u=x+4 dv = e− 5x dx
Problem Set 9-2
1
1 11 du = dx v = − e −5 x
Q1. y ′ = x sec x + tan x
2
Q2. x +C 5
11

1 1
= −( x + 4) ⋅ e −5 x + e −5 x dx
1 5 5
Q3. Q4. sin 3 x + C
3 4 1 1
y = − e −5 x − xe −5 x − e −5 x + C
5 5 25
21 −5 x 1 −5 x
= − e − xe + C
x
25 5

∫ ( x + 7)e u=x+7 dv = e2x dx


2x
6. dx

Q5. 5 cos2 5x − 5 sin2 5x 1 2x


Q6.
y
du = dx v= e
2


1 1 2x
x = ( x + 7) ⋅ e 2 x − e dx
2 2
7 1 1
= e 2 x + xe 2 x − e 2 x + C
2 2 4
13 2 x 1 2 x
f ( x + h) – f ( x ) = e + xe + C
Q7. r′(x) = t (x) Q8. lim 4 2
h→0 h
∫x u = ln x dv = x 3 dx
3
Q9. ≈ 110/6 Q10. C 7. ln x dx

∫ x sin x dx u=x dv = sin x dx 1 4


1. du = x− 1 dx v= x
4
du = dx v = −cos x

1 4 1 3
= x ln x −

x dx
= − x cos x − ( − cos x ) dx 4 4
1 1
= −x cos x + sin x + C = x 4 ln x − x 4 + C
4 16

Calculus Solutions Manual Problem Set 9-2 213


© 2005 Key Curriculum Press
∫x u = ln 3x dv = x 5 dx ∫x e
5 3 2x u dv
8. ln 3 x dx 1. dx
1 6 x3 +
e 2x
du = x− 1 dx v= x 3x2
1 2x
6 – 2e


1 6 1 1 2x
= x ln 3 x − x 5 dx 6x + 4e
6 6 1 2x
1 1 6 6 8e
= x 6 ln 3 x − x +C –
1 2x
6 36 0 +
16 e

∫x e u = x2 dv = ex dx
2 x
9. dx
1 3 2x 3 2 2x 3 2x 3 2x
du = 2x dx v = ex = x e − x e + xe − e + C
2 4 4 8
= x e − 2 xe dx ∫
2 x x

u = 2x dv = ex dx
∫x
5 −x
2. e dx u dv
du = 2 dx v = ex x5 e –x
= x 2 e x −  2 xe x − 2e x dx  ∫
+
5x 4 –e –x

= x2ex − 2xex + 2ex + C 20x 3 +
e –x
60x 2 –e –x
∫x

10. 2
sin x dx u = x2 dv = sin x dx 120x + e –x
du = 2x dx v = −cos x 120 – –e –x


= − x cos x − ( −2 x cos x ) dx
2 0 + e –x

u = 2x dv = −cos x dx = −x5e− x − 5x4e− x − 20x3e− x − 60x2e− x − 120xe− x


du = 2 dx v = −sin x − 120e− x + C

= − x 2 cos x − −2 x sin x − ( −2 sin x ) dx  ∫


 
∫x
4 dv
3. sin x dx u
= −x cos x + 2x sin x + 2 cos x + C
2
x4 + sin x
4x 3 –cos x
∫ ln x dx

11. u = ln x dv = dx 12x 2 + –sin x
24x cos x
du = x −1
dx v=x 24

sin x
+


= x ln x − x ⋅ x −1 dx 0 –cos x

= x ln x − x + C = −x4 cos x + 4x3 sin x + 12x2 cos x


− 24x sin x − 24 cos x + C

Problem Set 9-3


∫x
2 u dv
4. cos x dx
x2 + cos x
1 6 2x sin x
Q1. r +C Q2. 2m cos 2m + sin 2m –
6 2 + –cos x
1 3 0 – –sin x
Q3. tan x + C Q4. ( x + 11)6 + C
18
= x2 sin x + 2x cos x − 2 sin x + C
1 4
Q5. x + 11x + C Q6. 1
4
∫x
5
Q7. 1/2 5. cos 2 x dx u dv
x5 + cos 2x
b

∫ [ f ( x)
1
Q8. V = π 2
− g( x )2 ] dx 5x 4 – 2 sin 2x
a 1
20x 3 – 4 cos 2x
Q9. Q10. B +
1
y
60x 2 –
– 8 sin 2x
4 1
120x + 16 cos 2x
1
120 – 32 sin 2x
1
x 0 + – 64 cos 2x
2

214 Problem Set 9-3 Calculus Solutions Manual


© 2005 Key Curriculum Press

1 5 5 5 34 3x
= x sin 2 x + x 4 cos 2 x − x 3 sin 2 x ⇒ e cos 5 x dx
2 4 2 25
15 15 15 1 3
− x 2 cos 2 x + x sin 2 x + cos 2 x + C = e 3x sin 5 x + e 3x cos 5 x + C1
4 4 8 5 25

⇒ e 3x cos 5 x dx
6. ∫ x 3 sin 5 x dx u
x3
dv
sin 5x =
5 3x 3
e sin 5 x + e 3x cos 5 x + C
+
1 34 34
3x 2 – 5 cos 5x

1
∫e
6x +
– 25 sin 5x
10. 4x
sin 2 x dx u dv
1
6 – 125 cos 5x e 4x + sin 2x
1 1
0 4e 4x – 2 cos 2x
+
625 sin 5x –
1
16e 4x + – 4 sin 2x
1 3 2 6
= − x 3 cos 5 x + x sin 5 x + x cos 5 x

5 25 125 1
= − e 4x cos 2 x + e 4x sin 2 x − 4 e 4x sin 2 x dx
6 2
− sin 5 x + C
625

⇒ 5 e 4x sin 2 x dx
1
∫ e sin x dx = − e 4x cos 2 x + e 4x sin 2 x + C1
x u dv
7.
ex + sin x 2
ex
ex

+
–cos x
–sin x ∫
⇒ e 4x sin 2 x dx
1 4x 1
=− e cos 2 x + e 4x sin 2 x + C
10 5

= − e x cos x + e x sin x − e x sin x dx

∫ ∫x
7 u dv
⇒ 2 e sin x dx
x 11. ln 3 x dx
ln 3x + x 7
= − e x cos x + e x sin x + C1 1/x
1 8
8x
-----------------

⇒ e x sin x dx 1 – 8 x7
1
1
1 1 0 + 64 x 8
= − e x cos x + e x sin x + C
2 2
1 8 1 8
= x ln 3 x − x +C
∫e
x u dv 8 64
8. cos x dx
ex + cos x
ex
∫x
– sin x 5 u dv
ex + –cos x 12. ln 6 x dx
ln 6x + x 5
1 6
1/x 6x

= e x sin x + e x cos x − e x cos x dx -------------------------
1 – 6x5
1


1
⇒ 2 e cos x dx
x
0 + 36 x 6
= ex sin x + ex cos x + C 1

1 6 1 6
⇒ e x cos x dx = x ln 6 x − x +C
6 36
1 x 1
= e sin x + e x cos x + C

ln 7 5
2 2 13. x 4 ln 7 dx = x + C (ln 7 is a constant!)
5


cos 5 7x
∫e 14. e 7x cos 5 dx = e +C
3x u dv
9. cos 5 x dx 7
e 3x + cos 5x

3e 3x
1 1
– 5 sin 5x 15. sin 5 x cos x dx = sin 6 x + C
1 6
9e 3x + – 25 cos 5x
∫ ∫
1
16. x (3 − x ) dx = −
2 2/3
(3 − x 2 )2/3 ( −2 x dx )
1 3 2
= e 3x sin 5 x + e 3x cos 5 x 3
5 25 = − (3 – x 2 )5/3 + C
10

9
− e 3x cos 5 x dx
25

Calculus Solutions Manual Problem Set 9-3 215


© 2005 Key Curriculum Press
∫ x ( x + 5) ∫ x (ln x )
3 1/2 u dv 3 u dv
17. dx 23. dx
x3 +
(x + 5) 1/2 (ln x) 3 + x
2 1 2
3x 2 3(x + 5)
3/2 3 (ln x)2/x 2x

4 --------------------------
6x 5/2 1
+ 15(x + 5) 3 (ln x) 2 – 2 x
8 7/2 1 2
6 105(x + 5)
– 6 (ln x)/x 4x
+ 16 9/2 --------------------------
0 945(x + 5) 1
6 ln x + 4 x
1 2
2 3 4 6/x 8x
= x ( x + 5)3/2 − x 2 ( x + 5)5/2 --------------------------
1
3 5 6 – 8x
16 32 1 2
+ x ( x + 5) − 7/2
( x + 5)9/2 + C 0 +
16 x
35 315
1 2 3
= x (ln x )3 − x 2 (ln x )2
∫x ∫
2 − x dx = x 2 (2 − x )1/2 dx
2
18. 2 4
u dv 3 3
x2 (2 – x)1/2 + x 2 ln x − x 2 + C
+
2
4 8
2x – 3(2 – x)3/2

∫ x (ln x ) dx
4 5/2 3 2
2 + 15(2 – x) 24. u dv

8
– 105(2 – x)7/2 (ln x) 2 x3
0 +
1 4
2(ln x)/x 4x
--------------------------
1 3
2 ln x 4x
2 8 –
= − x 2 (2 − x )3/2 − x (2 − x )5/2 2/x
1 4
16 x
3 15 --------------------------
16 1
− (2 – x ) + C
7/2 2 + 16 x 3
105 1
0 – 64 x 4

∫ ln x ∫
dx = 5 ln x dx = 5 x ln x − 5 x + C
5
19.
1 4 1 1 4
= x (ln x )2 − x 4 ln x + x +C
4 8 32
∫e ∫
7 2
20. ln 7 x
dx = 7 x dx = x +C
∫ x (x
2 25. 3 2
+ 1) 4 dx u dv
x2 + x(x 2 + 1)4
∫x e
5 x2 u dv 1 2 5
21. dx 2x 10 (x + 1)
x4 + xe x 2 ----------------------------------
1
1 x2 2 5
4x 3 2e 5 – x(x + 1)
-------------------------- 1 2 6
12 (x + 1)
0 +
2x 2 – xe x 2
1 x2
4x 2e
-------------------------- 1 2 2 1
2 + xe x 2 = x ( x + 1)5 − ( x 2 + 1)6 + C
1
10 60
0 – 2 e x2
∫x ∫
x 2 − 3 dx = x 3 ( x 2 − 3)1/2
3
26.
1 4 x2 2 2
= x e − x 2e x + e x + C u dv
2 x2 x(x 2 – 3) 1/2
+
1 2 3/2
2x 3(x – 3)
∫x e
3
22. 5 x
dx u dv -----------------------------------
2
x(x 2 – 3) 3/2
x3 +
x 2 ex 3 3 –
+ 1 2
0 5/2
1 x3 5(x – 3)
3x 2 3e
---------------------------
x 2 ex 3 1 2 2 2
1 – = x ( x − 3)3/2 − ( x 2 – 3)5/2 + C
1 3 15
0 +
3 ex3

∫ cos
2 u dv
1 3 1 3 27. x dx
= x 3e x − e x + C cos x + cos x
3 3 –sin x – sin x

216 Problem Set 9-3 Calculus Solutions Manual


© 2005 Key Curriculum Press
∫ ∫ log ∫ ln x dx
= cos x sin x − ( − sin 2 x ) dx 1
32. 10 x dx =
ln 10
= cos x sin x + ∫ (1 − cos x ) dx
2
1
= ( x ln x – x ) + C
= cos x sin x + x − ∫ cos x dx 2 ln 10
1
= x log10 x − x+C
⇒ 2 ∫ cos x dx = cos x sin x + x + C
2
1 ln 10

⇒ ∫ cos x dx = cos x sin x + x + C ∫ sin x dx = − cos x + C


2 1 1 33.
2 2
34. ∫ cos x dx = sin x + C
28. ∫ sin 2 0.4x dx u dv
sin 0.4x
0.4 cos 0.4x –
+ sin 0.4x
–2.5 cos 0.4x
35. ∫ csc x dx = − ln | csc x + cot x | + C
36. ∫ sec x dx = ln | sec x + tan x | + C

= −2.5 sin 0.4 x cos 0.4 x + cos 2 0.4 x dx

= −2.5 sin 0.4 x cos 0.4 x + ∫ (1 − sin 0.4 x ) dx


2
37. ∫ tan x dx = − ln | cos x | + C
= −2.5 sin 0.4 x cos 0.4 x + ∫ dx − ∫ sin 0.4 x dx2
38. ∫ cot x dx = ln | sin x | + C
∫ x cos x dx
2
39.
⇒ 2 ∫ sin 0.4 x dx
2
For the first integral, Wanda integrated cos x and
= −2.5 sin 0.4x cos 0.4x + x + C 1 differentiated x2, but in the second integral she


⇒ sin 2 0.4 x dx ∫
plans to differentiate cos x dx and integrate 2x,
effectively canceling out what she did in the first
= −1.25 sin 0.4x cos 0.4x + 0.5x + C part. She will get

∫x ∫
cos x dx = x 2 sin x − x 2 sin x + x 2 cos x dx,
2
29. ∫ sec 3 x dx u
sec x +
dv
sec2 x which is true but not very useful!
sec x tan x – tan x
∫x
2
40. cos x dx


= sec x tan x − sec x tan 2 x dx Amos’s choice of u and dv transforms

∫ ∫
1 1
x 2 cos x dx into x 3 cos x + x 3 sin x dx,
= sec x tan x − ∫ sec x (sec x − 1) dx
2
3 3
which is more complicated than the original
= sec x tan x − ∫ sec x dx + ln | sec x + tan x |
3
expression.
41. After two integrations by parts,
⇒ 2 ∫ sec x dx 3


e x sin x dx
= sec x tan x + ln | sec x + tan x | + C 1

= − e x cos x + e x sin x − e x sin x dx

⇒ sec x dx 3
but after two more integrations,

∫ e sin x dx = −e
1 1 x x
cos x + e x sin x + e x cos x
= sec x tan x + ln | sec x + tan x | + C
2 2

− e x sin x + e x sin x dx
30. ∫ sec 2 x tan x dx Two integrations produced the original integral
with the opposite sign (which is useful), and two

1
= (sec x )1 ⋅ (sec x tan x dx ) = sec 2 x + C more integrations reversed the sign again to give
2
the original integral with the same sign (which
∫ ∫
1
31. log 3 x dx = ln x dx is not useful).
ln 3

∫ cos ∫
1 1
= ( x ln x – x ) + C 42. 2
x dx = (1 + cos 2 x ) dx
ln 3 2
1
=  x + sin 2 x  + C
= x log 3 x − x+C 1 1
ln 3 2 2 

Calculus Solutions Manual Problem Set 9-3 217


© 2005 Key Curriculum Press
By the double-argument properties from
trigonometry,
47. For integration by parts, ∫ u dv = uv − ∫ v du.
1 Applying limits of integration gives
x + sin 2 x  + C = ( x + sin x cos x ) + C
1 1
u=b
  d b

∫ ∫ v du
2 2 2
u dv = uv −
which is equivalent to the answer in Problem 27 c u =a a
found using integrating by parts. b

43.
y
= (bd − ac) − ∫ v du
a
The quantity (bd − ac) is the area of the
1
“L-shaped” region, which is the area of the larger
(1,1/e)
rectangle minus the area of the smaller one.
x
Thus, the integral of u dv equals the area of the
3
L-shaped region minus the area represented by the
integral of v du.

y′ = −xe − x + e− x = e− x(1 − x)
Critical points at x = 0, 1, 3; maximum at
48. ∫ ln ax dx = ∫ (ln a + ln x ) dx
= x ln a + x ln x − x + C
x = 1.
3 = x ln ax − x + C
∫ xe
−x
A= dx

∫ sin
0
7 u dv
49. x dx
= ( − xe − x − e − x )
3
0 sin 6 x + sin x
= −3e− 3 − e− 3 + 1 = −4e− 3 + 1 = 0.8008… 6 sin 5 x cos x – –cos x
44. y = 12x2e− x
Area from x = 0 to x = b is
b

= − sin 6 x cos x + 6 sin 5 x cos 2 x dx

A(b) = ∫ 12 x e = − sin x cos x + 6 ∫ sin x (1 − sin x ) dx


2 −x 6 5 2
dx
0

= −12 x 2 e − x − 24 xe − x − 24e − x = − sin x cos x + 6 ∫ sin x dx − 6 ∫ sin x dx


b 6 5 7
0
= −12b2e− b − 24be− b − 24e− b + 24 7 ∫ sin x dx = − sin x cos x + 6 ∫ sin x dx
7 6 5

The first two terms approach zero as b → ∞


∫ sin x dx = − 7 sin x cos x + 7 ∫ sin x dx
by L’Hospital’s rule. The third term also 7 1 6 6 5

approaches 0.
∴ lim Ab = 24 The fractions are 1/(old exponent) and
b→∞ (old exponent − 1)/(old exponent). The new
45. y = ln x exponent is 2 less than the old exponent. So
dV = πy2 dx = π (ln x)2 dx

1
sin 7 x dx = − sin 6 x cos x
5 7
V= ∫ π (ln x ) dx u dv
2
6 1 4
sin 3 x dx  ∫
(ln x)2 + 1 4
1
+ – sin x cos x +
2 (ln x)/x x 7 5 5 
-----------------------
2 ln x – 1 1 6
= − sin x cos x −
6 4
sin x cos x
2/x x 7 35
-----------------------
+  – sin 2 x cos x + sin x dx 
2 + 1

24 1 2
0 – x 35  3 3 
5 1 6
= π x (ln x )2 − 2π x ln x + 2π x 1
= − sin x cos x −
6 4
sin x cos x
7 35
= 5π ( ln 5)2 − 10π ln 5 + 10π − 0 + 0 − 2π 8 16
= 15.2589… − sin 2 x cos x − cos x + C
35 35

46. Consider u dv, and write dv = v + C. Then ∫ 50. Answers will vary.

∫ u dv = u(v + C) − ∫ (v + C) du
= uv + Cu − ∫ v du − ∫ C du
Problem Set 9-4
= uv + Cu − ∫ v du − Cu = uv − ∫ v du
Thus, the constant cancels out later, Q.E.D.
Q1. uv − v du ∫

218 Problem Set 9-4 Calculus Solutions Manual


© 2005 Key Curriculum Press
Q2. Q3.
∫ tan x dx = ∫ tan x tan
20 18 2
4. x dx
y y

= ∫ tan x (sec x − 1) dx
3
18 2
x 1 x

= ∫ tan x sec x dx − ∫ tan

18 2 18
x dx

= tan x − ∫ tan x dx
1 19 18

1 19
Q4. y′ = 1 + ln 5x Q5. sin 6 x + C
6
Q6. ln |x| + C ∫ sec
13 u dv
5. x dx
sec 11 x + sec 2 x
Q7. 11 sec 10 x sec x tan x – tan x
y

1
3
x

= sec11 x tan x − 11 sec11 x tan 2 x dx

= sec 11
x tan x − 11 ∫ sec 11
x (sec 2 x − 1) dx

= sec11 x tan x − 11 ∫ sec 13



x dx + 11 sec11 x dx
1
Q8. Q9. ln | sec x + tan x | + C
1+ x2

12 sec x dx = sec x tan x + 11 sec11 x dx ∫
13 11

Q10. D
∫ ∫
1 11
sec x dx =
13
sec11 x tan x + sec11 x dx

12 12
1. sin 9 x dx u dv
sin 8 x + sin x
sin7
∫ csc
100
8 x cos x – –cos x 6. x dx
u dv

= − sin 8 x cos x + 8 sin 7 x cos 2 x dx csc 98 x
–98 csc 97 x csc x cot x –
+ csc 2 x
–cot x
= − sin 8
x cos x + 8 ∫ sin 7
x (1 − sin 2 x ) dx

= − sin 8 x cos x + 8 ∫ sin



7
x dx − 8 sin 9 x dx ∫
= − csc 98 x cot x − 98 csc 98 x cot 2 x dx

9 ∫ sin x dx = − sin x cos x + 8 ∫ sin x dx


9 8 7 = − csc 98
x cot x − 98 ∫ csc 98
x (csc 2 x − 1) dx

x cot x − 98 ∫ csc ∫
∫ sin x dx = − 9 sin x cos x + 9 ∫ sin x dx
9 1 8 8 7 = − csc 98 100
x dx + 98 csc 98 x dx

∫ x dx = − csc x cot x + 98 csc 98 x dx ∫


100 98
99 csc
2. ∫ cos10 x dx u dv
∫ ∫
1 98
cos 9 x + cos x csc100 x dx = − csc 98 x cot x + csc 98 x dx
–9 cos 8 x sin x – sin x 99 99

∫ cos
n


7. x dx u dv
= cos 9 x sin x + 9 cos8 x sin 2 x dx cos n – 1 x cos x
+
–(n –1) cosn – 2 x sin x
= cos x sin x + 9 ∫ cos x (1 − cos x ) dx
9 8 2
– sin x

= cos x sin x + 9 ∫ cos x dx − 9 ∫ cos x dx



= cos n−1 x sin x + (n − 1) cos n−2 x sin 2 x dx
9 8 10

10 ∫ cos x dx = cos x sin x + 9 ∫ cos x dx


10 9 8
= cos x sin x + (n − 1) ∫ cos x (1 − cos x ) dx
n −1 n−2 2

∫ cos x dx = 10 cos x sin x + 10 ∫ cos x dx


1 9
= cos x sin x + (n − 1) ∫ cos x dx
n −1 n−2
10 9 8

−(n − 1) ∫ cos x dx n

∫ cot x dx = ∫ cot x cot x dx


12 10 2
3.

= ∫ cot x (csc x − 1) dx
10 2 n ∫ cos x dx = cos x sin x + (n − 1) ∫ cos x dx
n n −1 n−2

∫ cos x dx = n cos x sin x + n ∫ cos x dx


1 n –1
= ∫ cot x csc x dx − ∫ cot x dx
10 2 10 n n −1 n−2

= − cot x − ∫ cot x dx
1 11 10
11

Calculus Solutions Manual Problem Set 9-4 219


© 2005 Key Curriculum Press
∫ sin ∫
= sec n−2 x tan x − (n − 2) sec n−2 x tan 2 x dx
n u dv
8. x dx
sin n –1 x + sin x
(n –1) sinn –2 x cos x –cos x = sec x tan x − (n − 2) ∫ sec x (sec x − 1) dx
– n−2 n−2 2


= − sin n−1 x cos x + (n − 1) sin n−2 x cos 2 x dx = sec x tan x − (n − 2) ∫ sec x dx
n−2 n

= − sin x cos x + (n − 1) ∫ sin x (1 − sin


n −1 n−2 2
+ (n − 2)∫ sec x dx
x ) dx n−2

= − sin x cos x + (n − 1) ∫ sin x dx


n −1 n−2

(n − 1)∫ sec x dx n

− (n − 1) ∫ sin x dx n

= sec x tan x + (n − 2)∫ sec x dx


n−2 n−2

n ∫ sin x dx = − sin x cos x + (n − 1)∫ sin


n n −1 n−2
x dx
∫ sec x dx
n

∫ sin x dx = − n sin x cos x + n ∫ sin


n 1 n –1 n −1 n−2
x dx

n –1 ∫
1 n−2n–2 n−2
= sec x tan x + sec x dx
∫ tan x dx = ∫ tan x tan x dx
n n−2 2
9. n –1

= ∫ tan x (sec x − 1) dx ∫ sin x dx


n−2 5
2 13.

= ∫ tan x sec x dx − ∫ tan x dx


n−2 2 n−2
= − sin 4 x cos x +  − sin 2 x cos x − cos x  + C
1 4 1 2
5 5 3 3 
tan x − ∫ tan x dx
1 n −1 n−2
= 1 4 8
n –1 = − sin 4 x cos x − sin 2 x cos x − cos x + C
5 15 15
∫ cot x dx = ∫ cot x cot x dx
n n−2 2
10.
∫ cos
5
14. x dx
= ∫ cot x (csc x − 1) dx
n−2 2

cos 4 x sin x +  cos 2 x sin x + sin x  + C


1 4 1 2
=
= ∫ cot x csc x dx − ∫ cot x dx
n−2 2 n−2
5 53 3 
1 4 8
cot x − ∫ cot x dx
1 n −1 n−2 = cos 4 x sin x + cos 2 x sin x + sin x + C
=−
n –1 5 15 15

∫ csc x dx ∫ cot
n 6
11. 15. x dx

= − cot 5 x − − cot 3 x − ( − cot x − x ) + C


u dv 1 1
csc n – 2x + csc 2 x 5  3 
–(n – 2) csc n – 3x csc x cot x 1 1 3
–cot x = − cot x + cot x − cot x − x + C
– 5
5 3

∫ ∫ tan
7
= − csc n−2 x cot x − (n − 2) csc n−2 x cot 2 x dx 16. x dx
1 
= − csc x cot x − (n − 2)∫ csc x (csc x − 1) dx = tan 6 x −  tan 4 x −  tan 2 x + ln | cos x |  + C
n−2 n−2 2 1 1
6  4  2  
= − csc x cot x − (n − 2)∫ csc x dx
n−2 n 1 1 1
= tan x − tan x + tan x + ln | cos x | + C
6 4 2
6 4 2
+ (n − 2)∫ csc x dx n−2


1 2
17. sec 4 x dx = sec 2 x tan x + tan x + C
(n − 1)∫ csc x dx n 3 3


1 2
= − csc x cot x + (n − 2)∫ csc x dx
n−2 n−2 18. csc x dx = − csc x cot x − cot x + C
4 2
3 3
19. a. y = cos x is on top; y = cos3 x is in the
∫ csc x dx
n
middle; y = cos5 x is on the bottom.

n –1 ∫
1 n–2n−2 n−2 b. For y = cos x, area ≈ 2.0000… .
=− csc x cot x + csc x dx
n –1 For y = cos3 x, area ≈ 1.3333… .
∫ sec x dx For y = cos5 x, area ≈ 1.06666… .
n
12.
π /2 π /2
u
sec n – 2x +
dv
sec 2 x
c. A1 = ∫
– π /2
cos x dx = sin x
− π /2
(n – 2) sec n – 3 x sec x tan x – tan x sin (π/2) − sin (−π/2) = 2

220 Problem Set 9-4 Calculus Solutions Manual


© 2005 Key Curriculum Press
π /2 ε π ε
A3 = ∫– π /2
cos3 x dx Note that if
4
< x < , then cos x < cos
2 4
π /2
1 2 ε
= cos 2 x sin x + sin x ⇒ cos N x < .
3 3 − π /2 2π
1 2 Now, for any n > N,
= cos 2 (π /2) sin (π /2) + sin (π /2) π /2


3 3
1 2 cos n x dx
− cos ( −π /2) sin ( −π /2) − sin ( −π /2)
2 − π /2
3 3 ε /4 π /2
2
3
2 4
= 0 + − 0 + = = 1.3333…
3 3
=2 ∫ 0
cos n x dx + 2 ∫ ε /4
cos n x dx.
ε /4 ε /4 ε
∫ ∫
2 cos n x dx < 2 dx = (cos n x < 1) .
Observe that A3 = A1 . But 2
3 0 0 2
π /2 π /2 π /2
A5 = ∫– π /2
cos 5 x dx And 2 ∫
ε /4
cos n x dx < 2 ∫ ε /4
cos N x dx (n > N )
π /2 π /2 π /2 ε ε π /2 ε N ε 
∫ ∫ ∫
1 4
= cos 4 x sin x + cos3 x dx <2 dx < dx = cos x < .
5 − π /2 5 − π /2 ε /4 2π π 0 2 2π 
π /2
=0+
4
5
4 4 4 2
A3 = ⋅ = ⋅ ⋅ 2 =
5 3 5 3
16
15
So ∫ − π /2
cos n x dx
ε /4 π /2
= 1.066666 …
4
Observe that A5 = A3 .
=2 ∫ 0
cos n x dx + 2 ∫ε /4
cos n x dx < ε .
5 π /2

d. Based on the graphs, the area under cos x


∴ lim
n→∞ − π /2 ∫ cos n x dx = 0, Q .E.D .
should be greater than that under cos3 x,
∫ cos x dx = ∫ cos x cos x dx
5 4
which in turn is greater than the area under 20.
cos5 x. This is exactly what happens with the
calculated answers: A1 > A3 > A5. = ∫ (1 − sin x ) cos x dx2 2

e.
y = ∫ (1 − 2 sin x + sin x ) cos x dx
2 4
y = cos100 x

= ∫ cos x dx − 2 ∫ sin x cos x dx


1 2

+ ∫ sin x cos x dx
x 4
–0.5π 0.5π

2 1
= sin x − sin 3 x + sin 5 x + C
π /2 3 5
f. Yes, lim ∫ cos n x dx = 0. π /2
n→∞ – π /2

Following the pattern in part c, for odd n,


∴ A5 = ∫ − π /2
sin 5 x dx
π /2
= 2  sin x − sin 3 x + sin 5 x 
(n – 1)(n – 3)(n – 5)K( 4)(2)(1) 2 1
An = ⋅ 2,
(n)(n – 2)(n – 4)K(5)(3)  3 5  0

the denominator gets large faster than the 4 2 16


=2− + = = 1.0666 … , which agrees with
numerator. However, because both go to 3 5 15
infinity, this observation is not decisive. the answer from Problem 19.
The following epsilon proof by Cavan Fang
∫ sec
establishes the fact rigorously, using the 3 u dv
21. x dx
definition of limit in the form “For any sec x + sec 2 x
ε > 0, there is an N > 0 such that whenever sec x tan x – tan x
n > N, A n < ε .”
Proof: ∫
= sec x tan x − sec x tan 2 x dx
Pick 0 < ε < 2π .
ε = sec x tan x − ∫ sec x (sec x − 1) dx 2

Then 0 < cos < 1, so there exists N > 0


4
N ε ε = sec x tan x − ∫ sec x dx + ∫ sec x dx 3

such that cos < .


4 2π

Calculus Solutions Manual Problem Set 9-4 221


© 2005 Key Curriculum Press
1

2 sec 3 x dx = (sin 3 ax + 2 sin ax – 2 sin ax cos 2 ax )
3
= sec x tan x + ln |sec x + tan x| + C 1 1
= [sin 3 ax + 2 sin ax (1 – cos 2 ax )]
∫ sec
3
x dx 3
1
1 1 = [sin 3 ax + 2 sin ax (sin 2 ax )]
= sec x tan x + ln |sec x + tan x| + C 3
2 2 = sin3 ax
Note that the answer is half the derivative of
∴ ∫ sin
3
secant plus half the integral of secant. ax dx

∫ sin 1
n
22. ax dx =− (cos ax )(sin 2 ax + 2) + C, Q .E.D .
3a
u dv
sin n –1 ax + sin ax 24. Use integration by parts, or use the technique of
1 Problem 20, as shown here.
a(n – 1) sin n – 2ax cos ax – – a cos ax

∫ cos ax dx = ∫ cos ax cos ax dx


3 2
1
= − sin n−1 ax cos ax
a = ∫ (1 − sin ax ) cos ax dx
2


n−2
+ (n − 1) sin 2
ax cos ax dx
= ∫ cos ax dx − ∫ sin ax cos ax dx 2

1
= − sin n−1 ax cos ax 1 1
a = sin ax − sin 3 ax + C

a 3a
+ (n − 1) sin n−2 ax (1 − sin 2 ax ) dx
1
= (sin ax )(3 – sin 2 ax ) + C

1 3a
=− sin n−1 ax cos ax + (n − 1) sin n−2 ax dx
a 1
= (sin ax )(2 + cos 2 ax ) + C, Q .E .D .

− (n − 1) sin n ax dx 3a
Or: Differentiate, as in the alternate solution for

n sin n ax dx Problem 23.


1
=− sin n−1 ax cos ax + (n − 1) sin n−2 ax dx
a Problem Set 9-5
∫ sin Q1. f ′(1) = −4 g′(2) = 1/2
n Q2.
ax dx
Q3. h′(3) = −12 Q4. t′(4) = π/24

1 n –1
=− sin n−1 ax cos ax + sin n−2 ax dx Q5. p′(5) = 6e5 Q6. x = 83 = 512
an n
Q7. Q8. integration by parts

1
sin 5 3 x dx = − sin 4 3 x cos 3 x 5
y
15
x
4 8
− sin 2 3 x cos 3 x − cos 3 x + C 3
45 45


1
23. sin 3 ax dx = − sin 2 ax cos ax
3a Q9. Q10. E
f'(x) and f (x )


2
+ sin ax dx (From Problem 22)
3 f
f'
1 2 1
= − sin 2 ax cos ax − cos ax + C x
3a 3a –1
1 3 5 6 8
1
= − cos ax (sin ax + 2) + C, Q .E.D .
2
3a

∫ sin x dx = ∫ (1 − cos x ) sin x dx


d  1
– (cos ax )(sin 2 ax + 2)
5 2 2
Or: 1.
dx  3a 
1
= − (– a sin ax )(sin ax + 2)
2 = ∫ (1 − 2 cos x + cos x ) sin x dx
2 4

3a
1 2 1
− (cos ax )(2 a sin ax cos ax ) = − cos x + cos3 x − cos 5 x + C
3a 3 5

222 Problem Set 9-5 Calculus Solutions Manual


© 2005 Key Curriculum Press
∫ cos x dx = ∫ (1 − sin x ) cos x dx ∫ sin ∫
7 2 3 1
2. 12. 2
x dx = (1 − cos 2 x ) dx
2
= ∫ (1 − 3 sin x + 3 sin x − sin x ) cos x dx
2 4 6
1 1
= x − sin 2 x + C
3 1 2 4
= sin x − sin 3 x + sin 5 x − sin 7 x + C
∫ ∫
5 7 1
13. sin 2 5 x dx = (1 − cos 10 x ) dx
∫ cos 9 x dx = ∫ (1 − sin 9 x )
7 2 3 2
3. cos 9 x dx
1 1
= x − sin 10 x + C
= ∫ (1 − 3 sin 9 x + 3 sin 9 x
2 4 2 20

∫ ∫
1
− sin6 9x) cos 9x dx 14. cos 2 6 x dx = (1 + cos 12 x ) dx
2
1 1
= sin 9 x − sin 3 9 x 1 1
9 9 = x + sin 12 x + C
1 1 2 24
+ sin 9 x − sin 7 9 x + C
5

∫ sec ∫
x dx = ( tan 2 x + 1) sec 2 x dx
4
15 63 15.
4. ∫ ∫
sin 3 10 x dx = (1 − cos 2 10 x ) sin 10 x dx 1
= tan 3 x + tan x + C
3
1 1
=− cos 10 x + cos3 10 x + C
∫ csc x dx = ∫ (cot x + 1) csc x dx
6 2 2 2
10 30 16.


1
sin 3 x cos 3 x dx = sin 5 3 x + C = ∫ (cot x + 2 cot x + 1) csc x dx
4 4 2 2
5.
15


1 1 2
6. cos8 7 x sin 7 x dx = − cos 9 7 x + C = − cot 5 x − cot 3 x − cot x + C
63 5 3

∫ cos 8x sin 8x dx ∫ csc 6 x dx = ∫ (cot 6 x + 1) csc 6 x dx


6 3 8 2 3 2
7. 17.

= ∫ cos 8 x (1 − cos 8 x ) sin 8 x dx


6 2
= ∫ (cot 6 x + 3 cot 6 x + 3 cot 6 x + 1) csc
6 4 2 2
6 x dx

= ∫ (cos 8 x − cos 8 x ) sin 8 x dx


6 8
=−
1 1 1
cot 7 6 x − cot 5 6 x − cot 3 6 x
42 10 6
1 1 1
=− cos 7 8 x + cos 9 8 x + C − cot 6 x + C
56 72 6
∫ sin 2 x cos 2 x dx ∫ sec 100 x dx = ∫ (tan 100 x + 1) sec 100 x dx
4 3
8. 18. 4 2 2

= ∫ sin 2 x (1 − sin 2 x ) cos 2 x dx


4 2 1 1
= tan 3 100 x + tan 100 x + C
300 100
= ∫ (sin 2 x − sin 2 x ) cos 2 x dx
4 6


1
19. tan10 x sec 2 x dx = tan11 x + C
1 1 11
= sin 5 2 x − sin 7 2 x + C

10 14 1
20. cot 8 x csc 2 x dx = − cot 9 x + C
∫ sin x cos x dx = ∫ sin x (1 − cos x ) cos
5 2 2 2 2 9
9. x dx
∫ sec ∫
x tan x dx = sec 9 x (sec x tan x dx )
10
21.
= ∫ (cos x − 2 cos x + cos x ) sin x dx
2 4 6
1
= sec10 x + C
1 2 1 10
= − cos3 x + cos 5 x − cos 7 x + C
∫ csc ∫
3 5 7
22. 8
x cot x dx = csc 7 x (csc x cot x dx )
∫ cos x sin x dx = ∫ cos x (1 − sin
3 2 2 2
10. x ) sin x dx
1
= − csc8 x + C
= ∫ (sin x − sin x ) cos x dx
2 4 8

∫ sec 20 dx = sec 20∫ dx = (sec 20) x + C


10 10 10
1 1 23.
= sin 3 x − sin 5 x + C
∫ csc 12 dx = csc 12∫ dx = (csc 12) x + C
3 5 8 8 8
24.
∫ ∫
1
11. cos x dx =
2
(1 + cos 2 x ) dx
∫ (cos x − sin x ) dx = ∫ cos 2 x dx = 2 sin 2 x + C
2 2 2 1
25.
1 1
= x + sin 2 x + C
2 4

Calculus Solutions Manual Problem Set 9-5 223


© 2005 Key Curriculum Press
∫ (cos x + sin x ) dx = ∫ dx = x + C
2 2 odd function between symmetrical limits is
26.
equal to zero.

∫ (sin x ) dx = ∫ csc x dx = − cot x + C 33. dV = π y2 dx = π sin2 x dx


−2 2
27.
π π π

V = π sin 2 x dx = (1 – cos 2 x ) dx ∫
∫ (cos 3x ) dx = ∫ sec 3x dx = 3 tan 3x + C
−2 1 2
28. 0 2 0
π
π π
= x − sin 2 x = π 2 /2
∫ sec x dx
3
29. 2 4 0

1 1 34. a. y = sec2 x
= sec x tan x + ln | sec x + tan x | + C
2 2 dV = π[(y + 3)2 − 32] dx

∫ csc = π (sec4 x + 6 sec2 x) dx


3
30. x dx
1
V =π ∫ (sec x + 6 sec x ) dx
4 2
1 1
= − csc x cot x − ln | csc x + cot x | + C 0
2 2 1
= π ∫ [(1 + tan x ) sec x + 6 sec x ] dx

2 2 2
31. a. cos 5 x sin 3 x dx 0
1
= π ∫ (tan x sec x + 7 sec x ) dx
u dv 2 2 2
sin 3x + cos 5x 0
1
3 cos 3x – 5 sin 5x π 1

–9 sin 3x +
1
– 25 cos 5x =tan 3 x + 7π tan x
3 0
1 3
= sin 5 x sin 3 x + cos 5 x cos 3 x π
5 25 = tan 3 1 + 7π tan 1 ( = 38.2049K)
3

9
+ cos 5 x sin 3 x dx b. dV = 2π (x + 3) y dx = 2π (x + 3)(sec2 x) dx
25
1 1
16
∫ V = 2π ∫ x sec x dx + 6π ∫ sec
2 2
cos 5 x sin 3 x dx x dx
0 0
25
1 1


1 3
= 2π x tan x − 2π tan x dx + 6π tan x
1
= sin 5 x sin 3 x + cos 5 x cos 3 x + C1 0
5 25 0 0

∫ cos 5x sin 3x dx = 8π tan 1 + 2π ln |cos x |


1
0
5 3 = 8π tan 1 + 2π ln (cos 1) (= 35.2738…)
= sin 5 x sin 3 x + cos 5 x cos 3 x + C
16 16
1 2 1
2π 35. dA = r dθ = (5 + 4 cos θ )2 dθ
b. ∫0
cos 5 x sin 3 x dx 2 2
π /4


2π 1
5 3 A= (16 cos 2 θ + 40 cos θ + 25) dθ
= sin 5 x sin 3 x + cos 3 x cos 5 x = 0 2 0
16 16 0
π /4
Because the integral finds the area above 25
= 4θ + 2 sin 2θ + 20 sin θ + θ
minus the area below, this calculation shows 2 0
the two areas are equal. 25
32. a. = π + 2 + 10 2 + π = 29.1012 … ,
y
8
1 which agrees with the numerical answer.
1 1
x 36. dA = r 2 dθ = a 2 (1 + cos θ )2 dθ
π
2 2
2 π


1
A = a2 (1 + 2 cos θ + cos 2 θ ) dθ
2 0

1 2
a θ + 2 sin θ + θ + sin 2θ 
1 1
π π =
 

1 4 2 2 4
b. A = sin 3 x dx = cos3 x − cos x = 0
0 3 0 3 3 2
= πa , which is 1.5 times A circle.
c. Numerically: A ≈ 1.3333… (Checks.) 2
d. A = 0 because sin3 x is an odd function 37. Answers will vary.
[sin3 (−x) = −sin3 x] and the integral of an

224 Problem Set 9-5 Calculus Solutions Manual


© 2005 Key Curriculum Press
Problem Set 9-6 substitution to use and the ensuing calculus, and
because algebraic techniques are of less importance now
1 1
Q1. sin 3 x + C Q2. − cos 4 x + C that technology is used for evaluating integrals, the
3 4 student is not expected to carry along the absolute value
1 1
Q3. − ln | cos 5 x | + C Q4. ln | sin 6 x | + C just to eliminate it later.
5 6
Q5.
1
7
ln | sec 7 x + tan 7 x | + C
1. ∫ 49 – x 2 dx

Q6. 5 sec2 5x v

Q7. y′ = 4 cos 4x
7
Q8. d x

Q9. See the text for the statement of fundamental θ u


theorem of calculus. √ 49 – x2

Q10. See the text for the definition of indefinite


integral, Section 5-3. x
Let = sin θ . x = 7 sin θ , dx = 7 cos θ dθ ,
Note: A radical without a sign in front of it means the 7
positive root. Because trigonometric functions can be x
49 – x 2 = 7 cos θ , θ = sin −1
positive or negative, the radical should technically be 7
replaced by the absolute value of the appropriate
trigonometric function. Fortunately, this turns out to be ∫ ∫
∴ 49 – x 2 dx = 7 cos θ (7 cos θ dθ )

∫ ∫
unnecessary. If x has been replaced by a sin θ, a tan θ, 49
= 49 cos 2 θ dθ = (1 + cos 2θ ) dθ
or a sec θ, it is assumed that θ is the corresponding 2
inverse trigonometric function. So θ is restricted to the 49 49
= θ + sin 2θ + C
range of that inverse trigonometric function. Thus, 2 4
49 49
respectively, = θ + sin θ cos θ + C
2 2
a – x = a |cos θ |, and θ ∈ Quadrant I or IV
2 2
49 −1 x 49 1 1
= sin + ⋅ x⋅ 49 – x 2 + C
a 2 + x 2 = a |sec θ |, and θ ∈ Quadrant I or IV 2 7 2 7 7
49 −1 x 1
x 2 – a 2 = a |tan θ |, and θ ∈ Quadrant I or II = sin + x 49 − x 2 + C
2 7 2


For the first two, the absolute value is unnecessary
2. 100 – x 2 dx
because cos θ ≥ 0 and sec θ ≥ 0 in the respective
quadrants. For the secant substitution, if x is negative,
v
then θ is in Quadrant II, where tan θ < 0. Thus, the
radical equals the opposite of a tan θ, and one should
10
write x

x – a = − a tan θ
2 2 θ u

Where the integral of sec θ occurs, one gets √ 100 – x2

ln x + x 2 + a 2 for x > 0
x
Let = sin θ . x = 10 sin θ , dx = 10 cos θ dθ ,
− ln x − x 2 + a 2 for x < 0 10
x
100 – x 2 = 10 cos θ , θ = sin −1
The second form can be transformed into the first by 10
taking advantage of the property −ln n = ln (1/n). Thus, ∫ ∫
∴ 100 – x dx = 10 cos θ (10 cos θ dθ )
2

1
∫ ∫
100
− ln x − a – x
2 2
= ln = 100 cos θ dθ = 2
(1 + cos 2θ ) dθ
x – a2 – x 2 2
= 50θ + 25 sin 2θ + C
= ln x + a 2 – x 2 = 50θ + 50 sin θ cos θ + C
which can be shown by rationalizing the denominator x 1 1
= 50 sin −1 + 50 ⋅ x ⋅ 100 – x 2 + C
of the fraction and incorporating the constant ln a2 10 10 10
(or 2 ln a) into the constant of integration. Because x 1
the major focus of this section is on the correct = 50 sin −1 + x 100 – x 2 + C
10 2

Calculus Solutions Manual Problem Set 9-6 225


© 2005 Key Curriculum Press
3. ∫ x 2 + 16 dx 5. ∫ 9 x 2 – 1 dx

v v

√ x 2 + 16
3x
x √ 9x 2 – 1
θ u θ u
4 1

x
Let = tan θ . x = 4 tan θ , dx = 4 sec 2 θ dθ , 3x 1
4 Let = sec θ . x = sec θ ,
x 1 3
x 2 + 16 = 4 sec θ , θ = tan −1 1
4 dx = sec θ tan θ dθ ,
∫ ∫
3
∴ x 2 + 16 dx = 4 sec θ ( 4 sec 2 θ dθ )
9 x 2 – 1 = tan θ , θ = sec −1 3 x

= 16 sec θ dθ (Compare Problem 21 in
3

∫ 9 x 2 – 1 dx = tan θ  sec θ tan θ dθ 



1

Problem Set 9-4.) 3 
16 16
= sec θ tan θ + ln | sec θ + tan θ | + C1

1
2 2 = sec θ tan 2 θ dθ
3
1 x 2 + 16 x

= x x 2 + 16 + 8 ln + + C1 1
= (sec 3 θ − sec θ ) dθ
2 4 4 3
1 1 1
= x x 2 + 16 + 8 ln x 2 + 16 + x − 8 ln 4 + C1 = sec θ tan θ + ln | sec θ + tan θ |
2 6 6
1 1
= x x 2 + 16 + 8 ln x 2 + 16 + x + C − ln | sec θ + tan θ | + C
2 3
1 1
4. ∫ 81 + x 2 dx = sec θ tan θ − ln | sec θ + tan θ | + C
6 6
1 1
v = x 9 x – 1 − ln 3 x + 9 x 2 – 1 + C
2
2 6
√ 81 + x 2

θ
x

u
6. ∫ 16 x 2 – 1 dx

9 v

4x
x √ 16x 2 – 1
Let = tan θ . x = 9 tan θ , dx = 9 sec 2 θ dθ ,
9 θ u
x
81 + x 2 = 9 sec θ , θ = tan −1
1

9
∴ ∫ ∫
81 + x 2 dx = 9 sec θ (9 sec 2 θ dθ )
Let
4x
1
1
= sec θ . x = sec θ ,
4

= 81 sec 3 θ dθ (Compare Problem 21 in 1
dx = sec θ tan θ dθ ,
Problem Set 9-4.) 4
81 81 16 x 2 – 1 = tan θ , θ = sec −1 4 x
= sec θ tan θ + ln | sec θ + tan θ | + C1
∫ 16 x 2 – 1 dx = tan θ  sec θ tan θ dθ 

2 2 1

4 
1 81 81 + x 2 x
= x 81 + x 2 + ln + + C1

1
2 2 9 9 = sec θ tan θ dθ
2
4


1 81 81 1
= x 81 + x 2 + ln 81 + x 2 + x − ln 9 + C1 = (sec 3 θ − sec θ ) dθ
2 2 2 4
1 1
1 81 = sec θ tan θ + ln | sec θ + tan θ |
= x 81 + x 2 + ln 81 + x 2 + x + C 8 8
2 2 1
− ln | sec θ + tan θ | + C
4

226 Problem Set 9-6 Calculus Solutions Manual


© 2005 Key Curriculum Press
1 1 sec 2 θ dθ
∫ ∫ ∫
= sec θ tan θ − ln |sec θ + tan θ | + C dx
∴ = = sec θ dθ
8 8 x2 +1 sec θ
1 1
= x 16 x – 1 − ln 4 x + 16 x 2 – 1 + C
2
2 8 = ln | sec θ + tan θ | + C = ln x2 +1 + x + C


dx

7. dx
10.
17 – x 2 2
x – 121
v
v

√17
x x
√x 2 – 121
θ u
θ u
√ 17 – x 2 11

Let x/ 17 = sin θ . x
Let = sec θ . x = 11 sec θ ,
x = 17 sin θ , dx = 17 cos θ dθ , 11
x dx = 11 sec θ tan θ dθ ,
17 – x 2 = 17 cos θ , θ = sin −1 x
17 x 2 – 121 = 11 tan θ , θ = sec −1
17 cos θ dθ 11
∫ ∫
dx
∴ = 11 sec θ tan θ dθ
∫ ∫
dx
17 – x 17 cos θ
2 ∴ =
x 2 – 121 11 tan θ

x
= dθ = θ + C = sin −1 +C
17 = ∫ sec θ dθ = ln | sec θ + tan θ | + C 1


dx x x 2 – 121
8. = ln + + C1
2
13 – x 11 11
v
= ln x + x 2 – 121 − ln 11 + C1
√ 13
x = ln x + x 2 – 121 + C
θ u

∫x
2
√13 – x 2 11. x 2 – 9 dx

v
Let x/ 13 = sin θ . x = 13 sin θ ,
dx = 13 cos θ dθ , x
√x 2 – 9
−1 x
13 – x = 13 cos θ , θ = sin
2
θ u
13 3
13 cos θ dθ
∫ ∫
dx
∴ =
13 – x 13 cos θ
2
x
Let = sec θ . x = 3 sec θ ,

x
= dθ = θ + C = sin −1 +C 3
13 dx = 3 sec θ tan θ dθ,
x
x 2 – 9 = 3 tan θ , θ = sec −1

dx
9. 3
x2 +1
v

∴ x 2 x 2 – 9 dx

= ∫ (9 sec θ ) (3 tan θ ) (3 sec θ tan θ dθ )


2
√ x2 + 1
x
= 81 ∫ sec θ tan θ dθ 3 2

θ u
1 = 81  ∫ sec θ dθ − ∫ sec θ dθ 
5 3

= 81  sec θ tan θ + ∫ sec θ dθ − ∫ sec θ dθ 


1 3 3 3 3
x
Let = tan θ . dx = sec 2 θ dθ , 4 4 
1
= sec θ tan θ − ∫ sec θ dθ
81 3 81 3
x 2 + 1 = sec θ , θ = tan −1 x 4 4

Calculus Solutions Manual Problem Set 9-6 227


© 2005 Key Curriculum Press
∫ (1 − x
81 3 81 2 3/2
= sec θ tan θ − sec θ tan θ 13. ) dx
4 8
81
− ln | sec θ + tan θ | + C1 v

8
81 x 3 x 2 – 9 81 x x2 – 9 1
= ⋅ ⋅ − ⋅ ⋅ x
4 27 3 8 3 3 θ u
2
81 x x –9 √1 – x2
− ln + + C1
8 3 3
1 3 2 9 Let x = sin θ. dx = cos θ dθ,
= x x –9 − x x2 – 9
4 8 1 – x 2 = cos θ , θ = sin −1 x
81 81
− ln x + x – 9
2
+ ln 3 + C1
8
1 3 2 9
8 ∫
∴ (1 − x 2 )3/2 dx = cos3 θ (cos θ dθ ) ∫
= x x –9 − x x2 – 9
4 8 = ∫ cos θ dθ 4

81
− ln x + x – 9
2
+C

1 3
8 = cos3 θ sin θ + cos 2 θ dθ
4 4
∫x
2 2
12. 9 – x dx

1 3
= cos3 θ sin θ + (1 + cos 2θ ) dθ
v
4 8
1 3 3
= cos3 θ sin θ + θ + sin 2θ + C
3
4 8 16
x
1 3 3
θ u = cos3 θ sin θ + θ + sin θ cos θ + C
4 8 8
√9 – x2
1 3 3
= x (1 − x 2 )3/2 + sin −1 x + x 1 – x 2 + C
x 4 8 8
Let = sin θ . x = 3 sin θ , dx = 3 cos θ dθ ,
∫ (x − 81) −3/2 dx
3 2
14.
x
9 – x 2 = 3 cos θ , θ = sin −1
3 v


∴ x 2 9 – x 2 dx

= ∫ (9 sin θ ) (3 cos θ ) (3 cos θ dθ )


x
2 √ x 2 – 81
θ u
= 81 ∫ sin θ cos θ dθ
2 2 9

= 81 ∫ (cos θ − cos θ ) dθ
2 4

x
Let = sec θ . x = 9 sec θ , dx = 9 sec θ tan θ dθ ,
= 81∫ cos θ dθ −
81
2
cos θ sin θ 3
9
4 1
3 ⋅ 81 x 2 – 81 = 9 tan θ , θ = sec −1 x
4 ∫
− cos θ dθ 2
9

= ∫ (1 + cos 2θ ) dθ − cos θ sin θ


81
8
81
4
3

∴ ( x 2 − 81) −3/ 2 dx

=
81 81 81
θ + sin 2θ − cos3 θ sin θ + C = ∫ (9 tan θ ) (9 sec θ tan θ dθ )
−3

8 16 4
1 sec θ dθ
81 81 81
= θ + sin θ cos θ − cos3 θ sin θ + C = ∫
8 8 4 81 tan θ 2

81 81
= θ + sin θ cos θ (1 − 2 cos 2 θ ) + C

1
8 8 = cot θ csc θ dθ
81
81 −1 x
= sin 1
8 3 = − csc θ + C
81
81 x 9 – x 2  2( 9 – x 2 ) 
− ⋅ ⋅ ⋅ 1 – +C –x
= +C
8 3 3  9  81 x 2 – 81
81 −1 x 1
= sin − x ( 2 x 2 − 9) 9 – x 2 + C
8 3 8

228 Problem Set 9-6 Calculus Solutions Manual


© 2005 Key Curriculum Press
∫ 81 + x ∫ ∫
dx x dx 1
15. 2 b. = ( x 2 + 25) −1/2 (2 x dx )
x + 25
2 2
v
= x 2 + 25 + C, which agrees with part a.
√ 81 + x2 Moral: Always check for an easy way to
x integrate before trying a more sophisticated
θ u technique!
9


x dx
18. a.
x 2 – 49
x
Let = tan θ . x = 9 tan θ , dx = 9 sec 2 θ dθ , v
9
x
81 + x 2 = 9 sec θ , θ = tan −1 x
9 √ x 2 – 49
θ
9 sec 2 θ dθ 1 u

∫ 81 + x = ∫ ∫
dx 1
∴ = dθ = θ + C 7
2
81 sec 2 θ 9 9
1 x
= tan −1 + C x
9 9 Let = sec θ . x = 7 sec θ ,
7
dx = 7 sec θ tan θ dθ,

dx
16.
25 x 2 + 1 x
x 2 – 49 = 7 tan θ , θ = sec −1
7
7 sec θ (7 sec θ tan θ dθ )
v

∫ ∫
x dx
∴ =
√ 25x 2 + 1 x 2 – 49 7 tan θ


5x
= 7 sec 2 θ dθ = 7 tan θ + C = x 2 – 49 + C
θ u
1

∫ ∫
x dx 1
b. = ( x 2 − 49) −1/2 (2 x dx )
2
x – 49 2
5x 1 1
Let = tan θ . x = tan θ , dx = sec 2 θ dθ , = x 2 – 49 + C,which agrees with part a.
1 5 5

dx
19.
25 x 2 + 1 = sec θ , θ = tan −1 5 x
9 – ( x – 5)2
sec θ dθ
2

∫ 25x ∫ 5 ⋅ sec θ ∫
dx 1 1 v
∴ = = dθ = θ + C
+1 2 2
5 5
1 −1 3
= tan 5 x + C x–5
5 θ u


x dx
17. a. √ 9 – (x – 5)2
x 2 + 25
x–5
v Let = sin θ . x = 5 + 3 sin θ , dx = 3 cos θ dθ ,
3
x–5
√ x 2 + 25 9 – ( x – 5)2 = 3 cos θ , θ = sin −1
x 3
3 cos θ dθ
∫ ∫
dx
θ u ∴ =
5 9 – ( x – 5)2 3 cos θ


x–5
= dθ = θ + C = sin −1 +C
x 3
Let = tan θ . x = 5 tan θ , dx = 5 sec 2 θ dθ ,

5 dx
20.
1
x 2 + 25 = 5 sec θ , θ = tan −1 x 36 – ( x + 2)2
5
v
5 tan θ (5 sec 2 θ dθ )
∫ ∫
x dx
∴ =
x 2 + 25 5 ⋅ sec θ 6
x+2
= 5∫ tan θ sec θ dθ = 5 sec θ + C θ u

= x + 25 + C
2 √ 36 – (x + 2)2

Calculus Solutions Manual Problem Set 9-6 229


© 2005 Key Curriculum Press
x+2 8
Let
6
= sin θ . x = 6 sin θ − 2, dx = 6 cos θ dθ , 23. ∫−3
100 – x 2 dx
x+2
36 – ( x + 2)2 = 6 cos θ , θ = sin −1 v
6
6 cos θ dθ
∫ ∫
dx
∴ =
36 – ( x + 2)2 6 cos θ 10 x

x+2 θ


u
= dθ = θ + C = sin −1 +C
6 √ 100 – x 2

∫ ∫
dx dx
21. =
x + 8 x – 20
2
( x + 4)2 – 36 Let
x
= sin θ . x = 10 sin θ , dx = 10 cos θ dθ ,
v 10
x
100 – x 2 = 10 cos θ , θ = sin −1
10
x+4
√ (x + 4) 2 –36 8

u
∴ ∫ −3
100 – x 2 dx
6
sin –1 0.8

Let
x+4
= sec θ . x = 6 sec θ − 4,
= ∫ sin –1 (–0.3)
10 cos θ ⋅ 10 cos θ dθ
6 sin –1 0.8
dx = 6 sec θ tan θ dθ, = 100 ∫ sin –1 (–0.3)
cos 2 θ dθ
x 2 + 8 x – 20 = ( x + 4)2 – 36 = 6 tan θ ,
sin –1 0.8

θ = sec −1
x+4
6
= 50 ∫sin –1 (–0.3)
(1 + cos 2θ dθ )

6 sec θ tan θ dθ sin –1 0.8


∫ ∫
dx = 50θ + 25 sin 2θ
∴ = sin –1 (–0.3)
x + 8 x – 20
2 6 tan θ
= 50 sin −1 0.8 + 25 sin (2 sin −1 0.8)
= ∫ sec θ dθ − 50 sin− 1 (−0.3) − 25 sin [2 sin− 1 (−0.3)]
= ln | sec θ + tan θ | + C 1 = 99.9084…
Numerical integration: 99.9084… (Checks.)
x+4 x 2 + 8 x – 20
= ln + + C1 4
6 6 24. ∫–1
x 2 + 25 dx

= ln x + 4 + x 2 + 8 x – 20 − ln 6 + C1
v

= ln x + 4 + x + 8 x – 20 + C
2
√ x 2 + 25
x

∫ ∫
dx dx
22. = θ u
x 2 – 14 x + 50 ( x – 7)2 + 1 5

√(x – 7) 2 + 1 x
Let = tan θ . x = 5 tan θ , dx = 5 sec 2 θ dθ ,
x–7 5
θ u
1 x 2 + 25 = 5 sec θ , θ = tan −1 0.2 x
4

x–7
∴ ∫ –1
x 2 + 25 dx
Let = tan θ . x = 7 + tan θ , dx = sec 2 θ dθ , tan –1 0.8


1
= 5 sec θ ⋅ 5 sec 2 θ dθ
x 2 – 14 x + 50 = ( x – 7) 2 + 1 = sec θ , tan –1 (–0.2 )

θ = tan− 1 (x − 7) tan –1 0.8

sec 2 θ dθ
= 25 ∫ sec 3 θ dθ
∫ ∫ ∫
dx tan –1 (–0.2 )
∴ = = sec θ dθ
x 2 – 14 x + 50 sec θ 25 25 tan −1 0.8

= ln | sec θ + tan θ | + C = sec θ tan θ + ln |sec θ + tan θ |


2 2 tan −1 ( −0.2 )

= ln x – 14 x + 50 + x − 7 + C
2

230 Problem Set 9-6 Calculus Solutions Manual


© 2005 Key Curriculum Press
x =4


25 6
= sec ( tan −1 0.8) ⋅ 0.8 ∴A= 5 cos θ ⋅ 5 cos θ dθ
2 5 x = –3
x =4
= 30 ∫
25
+ ln |sec ( tan −1 0.8) + 0.8 | cos 2 θ dθ
2 x = –3
x =4
= 15∫
25
− sec [tan −1 ( −0.2)] ⋅ ( −0.2) (1 + cos 2θ ) dθ
2 x = –3
25 x =4
− ln | sec [tan −1 ( −0.2)] − 0.2 | 15
= 15θ + sin 2θ
2 2 x =−3
= 26.9977…
= 15θ + 15 sin θ cos θ xx == 4–3
Numerical integration: 26.9977… (Checks.)
x 3 x =4
25. y = 3x 2 = 15 sin −1 + x 25 – x 2
5 5 x =−3
dL = 1 + ( y ′)2 dx = 1 + 36 x 2 dx −1 3
5 = 15 sin 0.8 + ( 4) 9
L= ∫0
1 + 36 x 2 dx
−1
5
3
− 15 sin ( −0.6) − (–3) 16
v 5
= 15[sin −1 0.8 − sin −1 ( −0.6)] + 14.4
√ 1 + 36x2
15π
6x
= + 14.4 = 37.9619…
θ u
2
1 Numerical integration: A = 37.9619…
(Checks.)
6 5

6x 1 1
Let = tan θ . x = tan θ , dx = sec 2 θ dθ , b. A = 25 – x 2 dx
1 6 6 5 –5
1 + 36 x = sec θ , θ = tan 6 x
2 −1 x = 5 ⇒ θ = π /2, x = −5 ⇒ θ = − π /2
π /2

∴L =
x =5

x =0 ∫
1
sec θ ⋅ sec 2 θ dθ
6
∴ A = 30 ∫ –π /2
cos 2 θ dθ
π /2
1 x =5 3 15
=
6 x =0 ∫
sec θ dθ = 15 θ +
2
sin 2θ
− π /2

1 1 x =5 15π 15 15π 15
= sec θ tan θ + ln |sec θ + tan θ | = + sin π + − sin ( −π )
12 12 x =0 2 2 2 2
1 1 5 = 15π = 47.1238…
= x 1 + 36 x 2 + ln 1 + 36 x 2 + 6 x The area is π (x-radius)(y-radius).
2 12 0
5 1 27. x 2 + y 2 = r 2 ⇒ y = ± r 2 – x 2 , x = 0 at y = ± r
= 901 + ln 901 + 30 = 75.3828K
2 12 Slice the region inside the circle perpendicular to
Numerical integration: L = 75.3828… (Checks.) the x-axis. Pick sample point (x, y) on the
3 positive branch of the circle, within the strip.
26. a. 9 x 2 + 25 y 2 = 225 ⇒ y = ± 25 – x 2
5 dA = 2 y dx = 2 r 2 − x 2 dx
Slice the region vertically. Pick a sample r

point (x, y) on the positive branch of the A=2 ∫–r


r 2 – x 2 dx
graph, within the strip. v
6
dA = 2 y dx = 25 – x 2 dx
5 r
x
6 4
A=
5 –3 ∫
25 – x 2 dx θ u

v √r 2 – x 2

x
5 Let = sin θ . x = r sin θ, dx = r cos θ dθ,
x r
x
θ u r 2 – x 2 = r cos θ , θ = sin −1
√ 25 – x2 r
x = r ⇒ θ = π /2, x = −r ⇒ θ = − π /2
π /2
Let
x
5
= sin θ . x = 5 sin θ , dx = 5 cos θ dθ , ∴A = 2 ∫ – π /2
r cos θ ⋅ r cos θ dθ
π /2


x = 2r cos 2 θ dθ
25 – x 2 = 5 cos θ , θ = sin −1
2

5 − π /2

Calculus Solutions Manual Problem Set 9-6 231


© 2005 Key Curriculum Press
π /2


Rotating instead about the x-axis is equivalent to
= r2 (1 + cos 2θ ) dθ
− π /2 interchanging the a and b, giving V = 43 πab 2 .
π /2
1 2
= r 2θ + r sin 2θ 30. x 2 − y 2 = 9 ⇒ y = ± x 2 – 9
2 − π /2 Slice the region perpendicular to the x-axis. Pick
π 1 π a sample point (x, y) on the positive branch of
= r 2 ⋅ + r 2 sin π + r 2 ⋅
2 2 2 the hyperbola, within the strip.
1 2 dA = 2 y dx = 2 x 2 – 9 dx
− r sin ( −π ) = πr 2
2 5
∴ A = π r 2, Q .E .D .
2 2
A=2 ∫
3
x 2 – 9 dx

28.   +   = 1 ⇒ y = ±
x y b 2
a – x2
 a  b
v
a
Slice the region inside the ellipse perpendicular
x
to the x-axis. Pick sample point (x, y) on the √x 2 – 9
positive branch of the ellipse, within the strip. θ u
2b 2 3
dA = 2 y dx = a – x 2 dx
a
2b a
A=
a –a ∫
a 2 – x 2 dx
Let
x
= sec θ . x = 3 sec θ, dx = 3 sec θ tan θ dθ,
3
v x
x 2 – 9 = 3 tan θ , θ = sec −1
3
x =5


a
x ∴ A=2 3 tan θ ⋅ 3 sec θ tan θ dθ
x =3
θ u x =5
√a 2 – x 2 = 18 ∫ tan 2 θ sec θ dθ
x =3
x =5
x = 18 ∫ (sec 3 θ – sec θ ) dθ
Let = sin θ . x = a sin θ, dx = a cos θ dθ, x =3
a
x = 9 sec θ tan θ + 9 ln | sec θ + tan θ |
a – x 2 = a cos θ , θ = sin −1
2 x =5
− 18 ln | sec θ + tan θ | x =3
a
x = a ⇒ θ = π /2, x = −a ⇒ θ = − π /2 = 9 sec θ tan θ − 9 ln | sec θ + tan θ | x =5
x =3
2 b π /2
∴A =
a – π /2 ∫
a cos θ ⋅ a cos θ dθ
=x x 2 – 9 − 9 ln
1
x+
1 2
x –9
5

π /2 3 3 3
= 2 ab ∫– π /2
cos 2 θ dθ = 20 − 9 ln 3 = 10.1124…
π /2 Numerical integration: A = 10.1124… (Checks.)
= ab ∫
– π /2
(1 + cos 2θ ) dθ
31. dV = 2πx (2 y) dx = 4πx x 2 – 9 dx
π /2
ab
= abθ + sin 2θ
5
2 − π /2
V = 4π ∫ 3
x 2 – 9 x dx
abπ ab abπ ab 5
= + sin π + − sin ( −π ) = πab = 2π ∫ x 2 – 9 (2 x dx )
2 2 2 2 3
∴ A = πab 5
2
= 2π ⋅ ( x 2 – 9)3/2
Note that if a = b = r, then πab = πr2, the area of 3 3
a circle. 4 256
a2 2 = π ⋅ 64 = π = 268.0825…
29. dV = πx 2 dy = π (b – y 2 ) dy, − b ≤ y ≤ b 3 3
b2 32. From Problems 30 and 31, A = 20 − 9 ln 3,
a2 b


256
V =π⋅ (b 2 – y 2 ) dy V= π.
b2 –b 3
b
a2  y3  128
= π ⋅ 2  b2 y –  =
4 2
πa b V = 2π x ⋅ A ⇒ x = = 4.2192 K
b  3 3 3(20 – 9 ln 3)
−b
x is a little more than halfway through the
4 2
∴ V = πa b region.
3

232 Problem Set 9-6 Calculus Solutions Manual


© 2005 Key Curriculum Press
33. x = a cos t ⇒ dx = −a sin t dt Q7. ex
y = b sin t Q8. b2 − 4ac = −1500, so x2 + 50x + 1000 is prime.
dA = 2y dx = 2(b sin t)(−a sin t dt)
Q9. b2 − 4ac = −144, so x2 + 36 is prime.
= −2ab sin2 t dt
x = −a ⇒ t = π , x = a ⇒ t = 0 Q10. B

dx = 
0 0 7 
∫ ∫ (1 – cos 2t ) dt ∫ ∫
11x – 15 4
∴ A = −2 ab sin 2 t dt = − ab 1. + dx
π π x 2 – 3x + 2  x –1 x – 2
ab 0
= 4 ln |x − 1| + 7 ln |x − 2| + C
= − abt + sin 2t = 0 + 0 + ab(π ) − 0 = πab
7 x + 25
dx = 
9 
∫ ∫
2 π –2
2. + dx
∴ A = πab, as in Problem 28. x 2 – 7x – 8  x + 1 x – 8
With this method, you get sin 2 t dt, directly. ∫ = −2 ln |x + 1| + 9 ln |x − 8| + C

= 
3/2 
∫ ∫
(5 x – 11) dx 7/2
With trigonometric substitution in Problem 28, 3. + dx
x2 – 2x – 8  x + 2 x – 4

you get cos 2 t dt, indirectly. 7 3
= ln | x + 2 | + ln | x − 4 | + C
34. r = 0.5θ ⇒ dr/dθ = 0.5 2 2

= 
dL = r 2 + ( dr/dθ )2 dθ = 0.25θ 2 + 0.25 dθ 6/5 
∫ ∫
(3 x – 12) dx 9/5
4. + dx
2
x – 5 x – 50  x+5 x – 10 
= 0.5 θ 2 + 1 dθ
9 6
6π = ln | x + 5 | + ln | x − 10 | + C
L = 0.5 ∫ 0
θ 2 + 1 dθ 5 5

= 
7 
∫ ∫
21 dx –7
v 5. + dx
x 2 + 7 x + 10  x + 5 x + 2
√ θ2 + 1 = −7 ln |x + 5| + 7 ln |x + 2| + C
θ
= 
8 
∫ ∫
10 x dx 2
φ 6. + dx
u
x 2 – 9 x – 36  x + 3 x – 12 
1
= 2 ln |x + 3| + 8 ln |x − 12| + C
9 x 2 – 25 x – 50
Let θ = tan φ ⇒ dθ = sec2 φ dφ.
θ 2 + 1 = sec φ , φ = tan −1 θ
7. ∫ ( x + 1)( x – 7)( x + 2)
dx

= 
4 

θ =6π 2 3
+ +
∴ L = 0.5 ∫ θ =0
sec φ ⋅ sec 2 φ dφ  x +1 x – 7 x + 2
dx
θ =6π = 2 ln |x + 1| + 3 ln |x − 7| + 4 ln |x + 2| + C
= 0.5 ∫ sec 3 φ dφ
7 x 2 + 22 x – 54

θ =0
= 0.25 sec φ tan φ + 0.25 ln | sec φ 8. dx
( x – 2)( x + 4)( x – 1)
θ =6π
+ tan φ |
= 
5 

3 –1
θ =0 + + dx
6π  x – 2 x + 4 x – 1
= 0.25θ θ 2 + 1 + 0.25 ln θ 2 +1 +θ = 3 ln |x − 2| − ln |x + 4| + 5 ln |x − 1| + C
0
4 x 2 + 15 x – 1
= 1.5π 36π + 1 + 0.25 ln 36π + 1 + 6π

2 2
9. dx
x 3 + 2 x 2 – 5x – 6
= 89.8589… , same as numerical
= 
3 

–1 2
integration. + + dx
 x + 3 x +1 x – 2
35. See the note preceding the solutions for this = −ln |x + 3| + 2 ln |x + 1| + 3 ln |x − 2| + C
section. For the sine and tangent substitution,
–3 x 2 + 22 x – 31
the range of the inverse sine and inverse tangent
make the corresponding radical positive. For the
10. ∫ x – 8x + 19 x – 12 dx
3 2

= ∫
–2 –4 3 
secant substitution, the situation is more + + dx
complicated but still gives an answer of the same  x –1 x – 3 x – 4
algebraic form as if x had been only positive. = −2 ln |x − 1| − 4 ln |x − 3| + 3 ln |x − 4| + C
3 x 3 + 2 x 2 – 12 x + 9
Problem Set 9-7
11. ∫ x –1
dx

=  3x 2 + 5x − 7 +
2 
Q1. (x + 5)(x − 5) Q2. x 2 + 2x − 15 ∫
 x – 1
dx
Q3. (x + 2)(x − 6) Q4. x 2 + 14x + 49 5
= x 3 + x 2 − 7 x + 2 ln | x − 1 | + C
Q5. (x + 4)2 Q6. x 2 − 64 2

Calculus Solutions Manual Problem Set 9-7 233


© 2005 Key Curriculum Press
x 3 – 7 x 2 + 5 x + 40 dy 1000 – y 1000 dy
12. ∫ x2 – 2x – 8
dx 19. a.
dt
= 2y
1000

y(1000 – y)
= 2 dt

= x −5+ 2  dx
∫ ∫
1000 dy

3x
= 2 dt
 x – 2x – 8 y(1000 – y)
1 
= x −5+
2 

1
∫  y + 1000 – y  dy = ∫ 2 dt
1
+ dx
 x + 2 x – 4
1
= x 2 − 5 x + ln | x + 2 | + 2 ln | x − 4 | + C ln |y| − ln |1000 − y| = 2t + C
2 y
ln = 2t + C
4 x + 6 x + 11
2

∫ (x
1000 – y
13. dx
2
+ 1)( x + 4) y
= e 2 t + C (Note that 0 ≤ y < 1000.)
x+2
=  2
3 

1000 – y
+ dx
 x +1 x + 4 1000 − y 1000
= e −2 t −C ⇒ − 1 = e −2 t −C
∫ ∫ ∫
1 2 x dx dx 3 dx y y
= +2 2 +
2 x2 +1 x +1 x+4 1000
1 = 1 + e −2 t −C = 1 + ke −2 t ( k = e − C )
= ln | x 2 + 1| + 2 tan −1 x + 3 ln | x + 4 | + C y
2 1000
y=
4 x 2 – 15 x – 1 1 + ke –2 t
14. ∫ x 3 – 5x 2 + 3x + 1
dx Initial condition y = 10 when t = 0 ⇒ k = 99.
1000
= 
x–2 

3 y=
+ dx 1 + 99e –2 t
 x – 1 x 2 – 4 x – 1
1 1000
= 3 ln | x − 1| + ln | x 2 − 4 x − 1| + C b. y(1) = = 69.4531… ≈ 69 students
2 1 + 99e –2
Note that have heard the rumor after one hour.
x–2 1/2 1/2 1000
= + , y( 4 ) = = 967.8567… ≈ 968
x2 – 4x – 1 x – 2 + 5 x – 2 – 5 1 + 99e –8
1 1 students have heard by lunchtime.
but ln x − 2 + 5 + ln x − 2 − 5 1000
2 2 y(8) = = 999.9888… ≈ 1000
1 1 + 99e –16
= ln | x − 4 x − 1|, so the answer comes out
2
students—everyone knows by the end of
2
the day!
the same.
c. It is quicker to analyze the original differential
4 x 2 + 18 x + 6
15. ∫ ( x + 5)( x + 1) 2 dx equation, which already refers to the derivative,
than to analyze the equation found in part a.
1000 – y
 1 –2  Maximize y′ = 2 y

3
=  + + 2  dx 1000
 x + 5 x + 1 ( x + 1)  1
= (1000 y – y ). 2
= ln |x + 5| + 3 ln |x + 1| + 2(x + 1)− 1 + C 500
1
3 x 2 – 53 x + 245 y ′′ = (1000 y′ – 2 yy′) = 0 when y = 500.
16. ∫ x 3 – 14 x 2 + 49 x
dx 500
This is the maximum point because y″ > 0
5 3 

–2 for y < 500 and y″ < 0 for y > 500 (and
=  + + 2  dx
 x x – 7 ( x – 7)  y′ > 0 for all t).
= 5 ln |x| − 2 ln |x − 7| − 3(x − 7)− 1 + C So the rate of spreading (y′) is greatest when
500 students have heard the news. This occurs
∫x ∫
dx dx
17. = when
– 6 x + 12 x – 8
3 2
( x – 2)3 1000
1 500 =
= − ( x – 2) −2 + C 1 + 99e –2 t
2 99e −2 t + 1 = 2
1
∫ ∫
1 dx
18. dx = e −2 t =
x 4 + 4x3 + 6x2 + 4x + 1 ( x + 1) 4 99
1 1
= − ( x + 1) −3 + C t = ln 99 = 2.2975… hr
3 2

234 Problem Set 9-7 Calculus Solutions Manual


© 2005 Key Curriculum Press
d. The graph follows the slope-field pattern. d. N = 1000 and P0 = 10
1000
y
1000 ⇒ P( t ) =
1 + 99e –0.3t
1000
P(7) = = 76.2010… ≈ 76 people
1 + 99e –2.1
infected after 1 week
2 4 6 t e. Solve P(t) = 990.
1000
20. a. Assume that an infected person and an 990 =
1 + 99e –0.3t
uninfected person have about the same chance 100
of meeting any other infected person (i.e., 1 + 99e −0.3t =
99
infected people are not quarantined). An e0.3 t = 992
infected person can meet N − P uninfected 2
people out of the total population, so the t= ln 99 = 30.6341K ≈ 31 days
0.3
chance of meeting an uninfected person will be
dx = 
b b –5 
∫ ∫
25 5
(N – 1)/N, so of an average infected person’s 21. A = + dx
2 x + 3x – 4 2  x –1 x + 4
2
three contacts per day, 3(N − 1)/N of them b
x –1
will be with uninfected persons. (Actually = 5 ln | x − 1| −5 ln | x + 4| b
= 5 ln
(N − P)/(N − 1) because the total population
2
x+4 2
that someone can meet is N − 1—people don’t b –1 1 b –1
= 5 ln − 5 ln = 5 ln + 5 ln 6
meet themselves outside the Twilight Zone— b+4 6 b+4
but (N − P)/N is reasonably close enough for 6
A(7) = 5 ln + 5 ln 6 = 5.9281…
now.) So there are P infected people, each 11
meeting an average of 3(N − P)/N uninfected 1
lim A(b) = lim 5 ln + 5 ln 6
people per day, for a grand total of 3P(N − P)/N b→∞ b→∞ 1
contacts between infected and uninfected (l’Hospital’s rule)
people per day. = 5 ln 6 = 8.9587…
b. If 10% of the contacts with infected people So the area does approach a finite limit.
per day result in infection, then the number 50π x
22. dV = 2π x y dx = 2 dx
of new infections per day should be 0.1 times x + 3x – 4
the number of contacts between infected and 50π x b 10π 40π 
dx = 
b
uninfected people, that is, V= ∫
2 x + 3x – 4
2
2  x –1 ∫
+
x + 4
dx
dP 3 P( N – P ) N–P
= 0.1 ⋅ = 0.3P . = 10π ln | x − 1| + 40π ln | x + 4| 2b
dt N N
dP N–P N dP = 10π ln |b − 1| + 40π ln |b + 4| − 40π ln 6
c. = 0.3P ⇒ = 0.3 dt V(7) = 40π ln 11 − 30π ln 6 = 132.4590…
dt N P( N – P )
lim V (b) = ∞ because both ln terms become
∫ ∫
N dP b→∞
= 0.3 dt
P( N – P ) infinite and are added.
(Note that if the region were rotated about the
 1 + 1  dP = 0.3 dt
∫ P N – P ∫ x-axis, the limit of the volume would be
finite. The answer would be 5π  – 2 ln 6
35
ln |P| − ln |N − P| = 0.3t + C
 6 
P
ln = 0.3t + C = 35.3400… .
N–P
dx = 
1/2 
∫ ∫
x–3 1/2
P
= e 0.3t +C (Note that 0 ≤ P < N.) 23. a. + dx
2
x – 6x + 8  x – 2 x – 4
N–P
N 1 1
= 1 + e −0.3t −C = 1 + ke −0.3t ( k = e − C ) = ln | x – 2| + ln | x – 4| + C
P 2 2
N b. x 2 – 6x + 8 = (x – 3)2 – 1
P( t ) =
1 + ke –0.3t
v

Initial condition P(0) = P0


x–3
N N √ (x – 3) 2 – 1
⇒ P0 = ⇒k= − 1.
1+ k P0 θ u
N
∴ P( t ) =
1

1 + ( N / P0 – 1)e –0.3t

Calculus Solutions Manual Problem Set 9-7 235


© 2005 Key Curriculum Press
Let x – 3 = sec θ. dx = sec θ tan θ dθ, p
d. ln = kmt + C2 (C 2 = mC 1)
( x – 3) – 1 = tan θ , θ = sec ( x – 3)
2 –1 m– p
p
= e kmt +C2 = C3e kmt (C3 = e C2 )
∫x ∫
x–3 x–3
∴ dx = dx m– p
2
– 6x + 8 ( x – 3)2 – 1
 p 
sec θ sec 2 θ  Note that m > p > 0 ⇒ > 0.
= ∫ tan 2 θ
(sec θ tan θ dθ ) = ∫ tan θ
dθ 
m– p m
m– p 

= ln | tan θ | + C = be – kmt
⇒ – 1 = be – kmt
(b = 1/C3 )
p p
= ln ( x – 3)2 – 1 + C m
= 1 + be – kmt ⇒ p =
m
p 1 + be – kmt
= ln x 2 – 6 x + 8 + C At time t = 0, p = p 0.
m
∫x ∫x
x–3 1 2x – 6
c. dx = dx ∴ p0 = ⇒ m = p0 (1 + b)
2
– 6x + 8 2 2
– 6x + 8 1+ b
p (1 + b)
1
=ln | x 2 – 6 x + 8| + C ∴ p = 0 – kmt
2 1 + be
1+ b
d. From part a, Letting K = km, p = p0 , Q .E .D .
1 1 1 + be − Kt
ln | x – 2| + ln | x – 4| + C e. Let p denote millions of people. Then
2 2
p0 = 179.3.
1
= ln |( x – 2)( x – 4)| + C Substitute p(10) = 203.2.
2 1+ b
1 203.2 = 179.3
= ln | x 2 – 6 x + 8| + C 1 + be –10 K
2 ⇒ 203.2 + 203.2be–10K = 179.3 + 179.3b
which is the answer in part c. This equals ⇒ b(203.2e–10K – 179.3) = –23.9
–23.9
ln | x 2 – 6 x + 8|1/2 + C = ln x 2 – 6 x + 8 + C, ⇒b=
203.2e –10 K – 179.3
which is the answer from part b. So all three By substituting p(20) = 226.5 and
answers are equivalent, Q.E.D. transforming,
24. a. When the population is very much smaller –47.2
than the maximum, (m – p) behaves like a b= .
226.5e –20 K – 179.3
constant, and dp/dt = k(m – p) · p is Equating the two values of b and solving
approximately proportional to p. But when p numerically for K gives K = 0.0259109… .
is approaching m, then (m – p) goes to zero, –23.9
so dp/dt = kp(m – p) goes to zero. ∴b= = 1.0630436 …
203.2e –0.259109... – 179.3
b. dp/dt = kp(m – p) = k(mp – p2). So dp/dt is a 2.063036...
∴ p = 179.3
quadratic function of p. Thus, the turning 1 + 1.063036...e –0.0259109...
point is at Check that this equation gives a good
m
p=– = m/2. approximation for 1990.
2(–1) 2.0630...
p(30) = 179.3 ⋅
If k > 0, the graph of dp/dt versus p opens 1 + 1.0630... ⋅ e –30⋅0.0259...
downward and the turning point is a = 248.4892… ≈ 248.5 million people,
maximum. which is close to the actual population,
So the population grows fastest when 248.7 million.
p = m/2. 2.0630...
f. p( 40) = 179.3 ⋅
c.
dp
= kp( m – p) ⇒
dp
= k dt 1 + 1.0630... ⋅ e –40⋅0.0259...
dt p( m – p) = 268.6144… ≈ 268.6 million people, which

∫ p(m – p) = ∫ k dt
dp is lower than the actual population by about
13 million people.
 1/m 1/m  1+ b
g. k > 0 ⇒ lim p = lim p0 = p0 (1 + b)
∫  p
+  dp = k dt
m – p ∫ t →∞ t →∞ 1 + be – kt
= 179.3 · (1 + 1.0630…)
1 1
ln | p| – ln |m – p| = kt + C1 = 369.9024… ≈ 369.9 million people
m m

236 Problem Set 9-7 Calculus Solutions Manual


© 2005 Key Curriculum Press
∫ cos
h. If p(10) had been 204.2, then K would have −1 u dv
3. x dx
been given by cos –1 x +
1
–24.9 –47.2 dx –
x
= √1 – x 2
204.2e –10 K – 179.3 226.5e –20 K – 179.3
⇒ K = 0.0343965…

x dx
–24.9 = x cos −1 x +
⇒b= –0.0343965... 1− x2
204.2e – 179.3

1
= 0.721075… = x cos −1 x − (1 − x 2 ) −1/2 ( −2 x dx )
2
So the ultimate population would have been 1
lim p = p0 (1 + b) = 179.3(1 + 0.7210 …) = x cos −1 x − ⋅ 2(1 − x 2 )1/2 + C
t →∞ 2
= 308.5888… ≈ 308.6 million people. = x cos −1 x − 1 − x 2 + C (Checks.)
An increase of 1 million in one of the initial
conditions causes a decrease of over
∫ sin
−1 u dv
61 million in the predicted maximum 4. x dx
sin –1x + 1
population! So this model does have a fairly dx –
sensitive dependence on the initial conditions. x
√ – x2
1


x dx
= x sin −1 x −
Problem Set 9-8 1− x2


1
Q1. integration by parts Q2. partial fractions = x sin −1 x + (1 − x 2 ) −1/2 ( −2 x dx )
Q3. x = tan θ or θ = tan–1 x 2
1
Q4. x = sec θ or θ = sec–1 x = x sin −1 x + ⋅ 2(1 − x 2 )1/2 + C
2
Q5. x = sin θ or θ = sin–1 x −1
= x sin x + 1 − x 2 + C
1 2
Q6. ( x + 1)8 + C
16
∫ sec
−1 u dv
5. x dx
Q7. 7 (at f (1)). Q8. 3 (at f (5)). sec –1 x 1
+
Q9. undefined Q10. B dx – x
|x| √ x 2 – 1
∫ tan
−1 u dv
1. x dx
tan –1 x 1
∫ | x|
x dx
= x sec −1 x −
+
dx –
1 + x2 x x2 −1


dx
= x sec −1 x − sgn x ( x/| x | = sgn x )
∫ x +1
x dx
= x tan −1 x − 2 x2 −1

x− ∫
1 2 x dx v
= x tan −1
2 x +1 2

1 x
√x 2 – 1
= x tan −1 x − ln | x 2 + 1| + C (Checks.)
2 θ u
1

∫ cot
−1 u dv
2. x dx
cot –1 x + 1
x
dx –
x Let = sec θ .
1 + x2 1
dx = sec θ tan θ dθ

∫ x +1
x dx x 2 – 1 = tan θ
= x cot −1 x + 2
θ = sec–1 x
x+ ∫ ∫
1 2 x dx
= x cot −1 ∴ sec –1 x dx
2 x +1 2

sec θ tan θ dθ
= x cot −1 x +
1
2
ln | x 2 + 1| + C (Checks.) = x sec –1 x – sgn x ∫ tan θ
= x sec –1 x – ∫ sgn x sec θ dθ

Calculus Solutions Manual Problem Set 9-8 237


© 2005 Key Curriculum Press
= x sec–1 x – sgn x ln |sec θ + tan θ| + C Simpson’s rule for y = sec− 1 x: n = 10 ⇒
= x sec–1 x – sgn x ln x + x 2 – 1 + C ∆x = 0.2
0.2
(Checks.) A≈ ( y1 + 4 y1.2 + 2 y1.4 + 4 y1.6 + L + 4 y2.8 + y3 )
3
Note: This answer can be transformed to
= 1.919692K
x sec –1 x – ln (| x | + x 2 – 1 ) + C. 3

∫ sec
−1
A= x dx
1

∫ csc
−1 u dv 3
6. x dx = x sec −1 x − sgn x ln ( x + x 2 – 1 )
csc –1 x +
1 1
dx – x = 3 sec −1 3 − 1 ⋅ ln (3 + 8 ) − sec −11 + 1 ⋅ ln 1
|x| √ x 2 – 1
= 1.930131… .
The Simpson’s rule answer differs from this
∫ | x|
x dx
= x csc –1 x + by 0.0104… , or about 0.5%.
x2 – 1
9. By vertical slices,

dx
= x csc –1 x + sgn x 1 π
x2 – 1 A =  – sin –1 x  dx

0 2 
π
v 1
= x − x sin −1 x − 1 – x 2
x
2 0
1
π −1
= − sin 1 − 0 − 0 + 0 + 1 = 1
θ u 2
π /2
√ x2 – 1 By horizontal slices, A = ∫
0
sin y dy

Let x = csc θ. dx = –csc θ cot θ dθ, π π /2


= − cos y = − cos
+ cos 0 = 1, which is the
0
x 2 – 1 = cot θ , θ = csc –1 x 2
same answer as by vertical slices.

∴ csc –1 x dx 10. By cylindrical shells, dV = 2π x tan− 1 x dx.
csc θ cot θ dθ 1

∫ ∫ x tan
−1
= x csc –1 x – sgn x V = 2π x dx
cot θ 0

x – ∫ sgn x csc θ dθ
= x csc –1
∫ x tan
−1 u dv
x dx
= x csc x + sgn x ln | csc θ + cot θ | + C
–1 tan –1 x +
x
dx – 1 2
1 + x2 2x
= x csc x + sgn x ln x + x – 1 + C
–1 2

x2

1 2 1
(Checks.) = x tan −1 x − dx
Note: This answer can be transformed to 2 2 1+ x2
1  1 

1
x sec −1 x + ln (| x | + x 2 – 1 ) + C. = x 2 tan −1 x − 1− dx
2 2  1+ x2 
4 4


1 1 1 1
7. tan −1 x dx = x tan −1 x −
ln | x 2 + 1| = x 2 tan −1 x − x + tan −1 x + C
1 2 1 2 2 2
∴ V = π x tan x − π x + π tan −1 x 0
−1
1 1 2 1
= 4 tan −1 4 − ln 17 − tan −11 + ln 2
2 2
π 1 17 = π tan −1 1 − π + π tan −1 1 − 0 + 0 − 0
−1
= 4 tan 4 − − ln = 3.4478K π
4 2 2 = 2π tan −1 1 − π = 2π ⋅ − π
4 4
Numerically, ∫1
tan −1 x dx = 3.4478K . 1 2
= π − π = 1.7932 K
8. 2
Compare this with a cylinder (π r2h) minus a
cone (π r2h/3), both of radius 1 and altitude π / 4,
y
π /2
which has volume 2π ( π / 4)/3 = π2/6 = 1.6449… ;
the volume is slightly less than V, which is
x expected because the cylinder minus the cone is
1 3 generated by rotating a line that lies below the
graph.

238 Problem Set 9-8 Calculus Solutions Manual


© 2005 Key Curriculum Press
Problem Set 9-9 7.
d
(csch x sin x )
dx
x
Q1. x = 5 tan θ or θ = tan −1 = −csch x coth x sin x + csch x cos x
5 d
1 8. (tan x tanh x ) = sec 2 x tanh x + tan x sech2 x
Q2. xex − ex + C Q3. tan 3 x + C dx
3

1
Q4. 2x1/ 2 + C Q5. ln |x| + C 9. sech 2 4 x dx = tanh 4 x + C
4
Q6. reduction formula Q7. False

1
10. sech 7 x tanh 7 x dx = − sech 7 x + C
Q8. dx 2 + dy 2 or dr 2 + (rd θ )2 7
d 3
Q9. (1 − x2)−1/ 2 Q10. C 11. ( x coth x ) = 3 x coth x − x 3 csch 2 x
2
dx
1.
y y
d 2.5
12. ( x csch 4 x )
dx
cosh
1 1 = 2.5x1.5 csch 4x − 4x2.5 csch 4x coth 4x
x x
3 3

∫ tanh x dx = ln (cosh x )
1 tanh 1
13.
sinh 1 1
coth = ln (cosh 3) − ln (cosh 1) = 1.875547…
4 4


y
14. sinh x dx = cosh x =0
−4 −4
sech 1
x (Note that sinh is an odd function.)
1 d sinh 5 x
15.
dx ln 3 x
csch
5 cosh 5 x ln 3 x – x –1 sinh 5 x
2. =
(ln 3 x )2
y y
d cosh 6 x
coth–1 16.
1
cosh–1
1
x
dx cos 3 x
6 sinh 6 x cos 3 x + 3 cosh 6 x sin 3 x
–1 1 x –1 1 =
tanh cos 2 3 x
sinh

y
17. ∫ x sinh x dx u
x +
dv
sinh x
1 – cosh x
–1 1 csch–1
sech 0 + sinh x
–1 1 x

= x cosh x − sinh x + C
1

d
∴ ∫ x sinh x dx
0
3. tanh 3 x = 3 tanh 2 x sec h 2 x = x cosh x − sinh x
1
= cosh 1 − sinh 1
dx 0
d =e −1
= 0.36787…
4. 5 sec h 3 x = −15 sec h 3 x tanh 3 x
dx


1
∫x
2 u dv
5. cosh 5 x sinh x dx = cosh 6 x + C 18. cosh x dx
6 x2 + cosh x
2x sinh x

1 –
6. (sinh x ) cosh x dx = − (sinh x ) −2 + C
−3
2 + cosh x
2 0 – sinh x
1
= − csch x + C
2
2 = x 2 sinh x – 2x cosh x + 2 sinh x + C

Or: (sinh x ) −3 cosh x dx = csch 2 x coth x dx ∫ ∴ ∫
b
x 2 cosh x dx
1 1 a
= − coth 2 x + C1 = − (csch 2 x + 1) + C1 b
2 2 = x 2 sinh x − 2 x cosh x + 2 sinh x a
1
= − csch x + C
2
2

Calculus Solutions Manual Problem Set 9-9 239


© 2005 Key Curriculum Press
= b2 sinh b – 2b cosh b + 2 sinh b
– a2 sinh a + 2a cosh a – 2 sinh a ∫
= 25 sinh t cosh t − 25 sinh 2 t dt − 25 dt ∫

∴ 50 sinh t dt = 25 sinh t cosh t − 25t + C1
2
d 12
19. (3 sinh –1 4 x ) =
16 x 2 + 1 ∴ 25 ∫ sinh
dx 2
t dt = 12.5 sinh t cosh t − 12.5t + C
d 15 x 2
20. (5 tanh –1 x 3 ) = x 2 – 25 x x
dx 1 – x6 = 12.5 ⋅ ⋅ − 12.5 cosh −1
5 5 5
∫ tanh
−1
21. 5x dx x
= 0.5 x x 2 – 25 − 12.5 cosh −1 + C
1 1 5
= x tanh −1 5 x + ln |1 − (5 x )2 | + C
5 10 25. a. Figure 9-9g shows that the horizontal force


is given by the vector (h, 0) and the vertical
22. 4 cosh −1 6 x dx
force is the vector (0, v), so their sum, the
4 4
= x cosh −1 6 x − [(6 x )2 – 1]1/2 + C tension vector, is the vector (h, v), which has
6 6 v
2 2 slope . Because the tension vector points
−1 h
= x cosh 6 x − 36 x 2 – 1 + C
3 3 along the graph, the graph’s slope, y ′, also
23. Let x = 3 sinh t, dx = 3 cosh t dt, v
equals .
x 2 + 9 = 9 ⋅ sinh 2 t + 9 = 3 cosh t, h
x b. v = weight of chain below (x, y) = s ⋅ w
t = sinh −1 . v s⋅w w
3 ⇒ y′ = = = ⋅s
h h h
∴ ∫ ∫
x 2 + 9 dx = 3 cosh t ⋅ 3 cosh t dt
c. ds = dx 2 + dy 2 = dx 2 [1 + ( dy/dx )2 ]

= 9 cosh 2 t dt u
cosh t +
dv
cosh t
= 1 + ( y′)2 dx
sinh t – sinh t w w
⇒ d ( y′) = ds = 1 + ( y′)2 dx
h h

= 9 cosh t sinh t − 9 sinh 2 t dt
d. ∫ [1 + ( y′) ] d( y′)
2 −1/2

= 9 cosh t sinh t − 9 ∫ (cosh t − 1) dt


2
= ∫ (1 + sinh t ) d (sinh t )
2 −1/2

= 9 cosh t sinh t − 9 ∫ cosh t + 9 ∫ dt


2
= ∫ (cosh t ) (cosh t dt )
2 −1/2

∴ 18 ∫ cosh t dt = 9 cosh t sinh t + 9t + C


2
1
= ∫ dt = t + C = sinh y′ + C −1
1
∴ 9 ∫ cosh t dt = 4.5 cosh t sinh t + 4.5t + C
2

∫ h dx = h x + C
w w
2
x2 + 9 x x
= 4.5 ⋅ ⋅ + 4.5 sinh −1 + C w
3 3 3 ⇒ sinh −1 y′ = x+C
x h
= 0.5 x x 2 + 9 + 4.5 sinh −1 + C e. At x = 0, y ′ = 0, so
3
w
24. Let x = 5 cosh t, dx = 5 sinh t dt, sinh −1 0 = 0 + C ⇒ C = 0.
h
x 2 – 25 = 25 ⋅ cosh 2 t – 25 = 5 sinh t, w w
x f. sinh −1 y′ = x ⇒ y′ = sinh x
t = cosh −1 . h h
5
∫ ∫
dy w w
= sinh x ⇒ dy = sinh x dx
∴ ∫ ∫
x 2 – 25 dx = 5 sinh t ⋅ 5 sinh t dt g.
dx
h
h
w
h
⇒ y = cosh x + C

= 25 sinh 2 t dt u
sinh t +
dv
sinh t
w h
1
cosh t – cosh t 26. a. y = 2 when x = 0 ⇒ 2 = k cosh 0 + C
k
⇒2=k+C⇒C=2−k

= 25 sinh t cosh t − 25 cosh 2 t dt
4
y = 5 when x = 4 ⇒ 5 = k cosh + 2 − k
= 25 sinh t cosh t − 25 ∫ (sinh 2
t + 1) dt k
Using the solver feature of your grapher,
k ≈ 3.0668… .

240 Problem Set 9-9 Calculus Solutions Manual


© 2005 Key Curriculum Press
1 1
y = 3.0668… cosh x + 2 − 3.0668… b. y′ = sinh x
3.0668… 500
1 150
y = 3.0668… cosh
3.0668…
x − 1.0668… L= ∫ –150
1 + sinh 2 ( x/500) dx
b. y (20) = 1040.9739… 150
=∫
1
cosh x dx
c. y = 4: −150 500
150
1 1
4 = 3.0668… cosh x − 1.0668… = 500 sinh x
3.0668… 500 −150
1 5.0668K = 500 sinh 0.3 − 500 sinh (−0.3)
cosh x=
3.0668K 3.0668K = 1000 sinh 0.3 = 304.5202… ≈ 304.5 ft
−1 5.0668…
A faster method is:
x = 3.0668… cosh = 3.3355… Half weight of cable = vertical tension at
3.0668…
(150, 110) = h ⋅ y ′ (150) (Compare
By symmetry, x = ±3.3355… . Problem 25.)
The answer can be found numerically using
1
the solver feature of your grapher. Weight = 2 ⋅ 400 sinh 150 = 800 sinh 0.3
500
1
d. y′ = sinh x; y′(3) = 1.1418… = 243.6162… ≈ 243.6 lb
k (Note: Because w ⋅ L = weight, either of these
A =  k cosh x + 2 – k  dx
3


1 methods could give both the weight and the
e.
–1  k  length.)
3
1 c. T = h 2 + v 2 ; h is constant and v is greatest
= k 2 sinh x + (2 − k ) x
k −1 at the ends, so the maximum tension is at
x = 150 ft.
= (3.0668…)2  sinh
3 –1 
− sinh
 3.0668… 3.0668… T (150) = h 2 + [hy′(150)]2
+ 4(2 − 3.0668…)
= 9.5937… = 400 1 + sinh 2 0.3 = 400 cosh 0.3
3 = 418.1354… ≈ 418.1 lb
f. L= ∫ −1
1 + ( y′)2 dx h w
d. The general equation is y = cosh x + C.
3 w h
=∫ 1 + sinh 2 ( x/k ) dx If y (0) = 100 and y (150) = 110, find h such
−1
3
that y (150) − y (0) = 10. Solve:
3


1 1
= cosh x dx = k sinh x h w h
−1 k k −1
cosh 150 − = 10, or
w h w
= k  sinh + sinh  = 4.5196 …
3 1 120 8
 cosh −1 =
k k h h
27. a. The vertex is midway between the poles, By grapher, h = 901.3301... ≈ 901.3 lb.
so y = 110 ft when x = 150 ft. 28. The answers will depend on the dimensions
h w of the chain used. Note that the answer is
y = cosh x + C independent of the kind of chain. You might
w h
400 lb 0.8 show students how a heavy chain and a light
= cosh x+C chain of equal length will hang in the same
0.8 lb / ft 400
catenary if they are suspended from the same
110 = 500 cosh  ⋅ 150 + C
1 points.
 500  Assume that the dimensions are the same as in
⇒ C = 110 − 500 cosh 0.3 Example 5.
1 a. Vertex: (0, 20). Supports: (±90, 120).
y = 500 cosh x + 110 − 500 cosh 0.3 1
500 b. y = 51.78… cosh x – 31.78K
The cable comes closest to the ground at 51.78K
x = 0. c. Note: To conserve class time, you might have
y (0) = 500 cosh 0 + 110 − 500 cosh 0.3 students plot only each 20 cm for x, as shown
= 610 − 500 cosh 0.3 = 87.3307… ≈ 87.3 ft here for Example 5. Use the TABLE feature.

Calculus Solutions Manual Problem Set 9-9 241


© 2005 Key Curriculum Press
x y c. Slice perpendicular to the y-axis.
dV = π x 2 dy = π (sinh–1 y)2 dy
0 20.0 Top of bowl is at
±20 1
23.9 y = sinh 1 − = 1.133534K .
±40 36.2 24
1.133K
±60 58.8 ∴ V =π ∫0
(sinh –1 y)2 dy
±80 95.1 = 1.25317… by numerical integration
≈ 1.253 ft3
d. A clever way to make sure the measurements
1
are vertical is to hold a book against the board 30. a. y = − k cosh x+C
with its bottom edge along the chalk tray. k
Then hold the meterstick against the vertical Inner catenary: yinner(0) = 612, yinner(260) = 0
edge of the book. It is crucial that the points 0
612 = − ki cosh + Ci ⇒ Ci = 612 + ki
be plotted accurately to get the dramatic ki
impact of “perfect fit.” 260
0 = − ki cosh + 612 + ki
e. For a quadratic function with vertex on the ki
y-axis, y = ax2 + c. Using the data for ⇒ ki = 97.1522… (numerically)
Example 5,
yinner = 97.1522…  − cosh x + 1 + 612
1
20 = a(0) + c ⇒ c = 20
1  97.1522 K 
120 = a(90 2 ) + 20 ⇒ a = Outer catenary: youter (0) = 630, youter (315) = 0
81
1 2 0
y= x + 20 630 = −ko cosh + C o ⇒ C o = 630 + k o
81 ko
y 315
parabola 0 = −ko cosh + 630 + k o
ko
catenary ⇒ ko = 127.7114… (numerically)

youter = 127.7114 K − cosh x + 1 + 630


10 1
x  127.7114 K 
20
b. The graphs are the same as in Figure 9-9k:
The parabola is more curved at the vertex. y
f. For Example 5,
1
dL = 1 + ( y′)2 dx = 1 + sinh 2 x dx
51.78K
100
1 x
= cosh x dx 100
51.78K
90


1 315 260
L=
−90
cosh
51.78K
x dx c. A = ∫−315
youter dx − ∫−260
yinner dx
90
1 315
= 51.78Ksinh x = 285.349K x
51.78K −90 = − ko2 sinh + ko x + 630 x
ko −315
≈ 285.3 cm
260
The actual length should be close to this. x
+ ki2 sinh − ki x − 612 x
29. a. y = sinh x ki −260
dS = 2π x dL = 2π x 1 + cosh 2 x dx = 54323.2729… ≈ 54,323 ft2
1
S = 2π ∫x
0
1 + cosh 2 x dx d.
dyouter
dx
1
= − sinh x, so
ko
= 5.07327… by numerical integration 315


1
≈ 5.07 ft2 L= 1 + sinh 2 x dx
−315 ko
b. Cost = 2(57)(5.07327…) = 578.3532… ≈
315
$578.35 315


1 1
= cosh x dx = ko sinh x
−315 ko ko −315

242 Problem Set 9-9 Calculus Solutions Manual


© 2005 Key Curriculum Press
315 exponential form or by transforming the second
= 2 ⋅ 127.7114 K ⋅ sinh
127.7114 K to hyperbolic form, as shown here.)
= 1493.7422… ≈ 1494 ft.
e + e + C = e x  e 2 x + e −2 x  + C
1 3x 1 − x 1 1
−315 315 6 2 6 2 
′ ( −315) = − sinh
e. youter = sinh
ko 127.7144K
= e x  e 2 x + e −2 x − e 2 x + e −2 x  + C
1 1 1 1
= 5.8481… is the spider’s starting slope. 3 3 6 6 
f. José can fly through at altitude yinner(x) if 2 x e 2 x + e –2 x 1 x e 2 x – e –2 x
x ≥ 50 + 120/2 = 110. = e − e +C
3 2 3 2
yinner(110) = 542.7829… , so the plane can 2 1
fly through at heights between 0 and 542 feet. = e x cosh 2 x − e x sinh 2 x + C
3 3
(Because of the curvature of the arch and the
vertical thickness of the plane, the closest
∫e
x u dv
distance is slightly less than 50 feet when the 34. sinh x dx
ex + sinh x
horizontal distance is 50 feet. The plane can ex cosh x

fly through at slightly higher altitudes by ex + sinh x
banking slightly.)
31. a. H(x) = csch x ⇒ H′(x) = −csch x coth x
H′(1) = −csch 1 coth 1 = −1.1172855… ∫
= e x cosh x − e x sinh x + e x sinh x dx


⇒ 0 e sinh x dx = e (cosh x − sinh x ) + C
x x
csch (1.01) – csch (0.99)
b. H′(1) ≈
0.02 The original integral reappeared with the same
= −1.11738505… . The answers differ by coefficient, so when it was added again to the left
0.0000995… , which is about 0.0089% of side, it exactly canceled out the desired integral.
the actual answer. Use the exponential form of sinh x.

∫ ∫
2 1 x x
e x sinh x dx = e (e − e − x ) dx
2
32. ∫1
sech x dx = sin −1 ( tanh x )
1
2


= sin–1 (tanh 2) − sin–1 (tanh 1) 1
= (e 2 x − 1) dx
= 0.435990… 2
2 1 1
Numerically, ∫ sech x dx = 0.435990K
1
= e2 x − x + C
4 2
(Checks.) 35. a. cosh x − sinh2 x
2

2 2
33. By parts:  ex + e– x   ex – e– x 
=  − 
 2   2 
∫e
x u dv
sinh 2 x dx
sinh 2x + ex e 2 x + 2 + e –2 x e 2 x – 2 + e –2 x
2 cosh 2x – ex = − =1
4 sinh 2x + ex 4 4
∴ cosh2 x − sinh2 x = 1, Q .E.D .

= e x sinh 2 x − 2e x cosh 2 x + 4 e x sinh 2 x dx
b.
1
(cosh 2 x – sinh 2 x ) =
1
cosh 2 x cosh 2 x

⇒ −3 e sinh 2 x dx x

⇒ 1 − tanh2 x = sech2 x
= e sinh 2 x − 2e cosh 2 x + C1 ⇒ e sinh 2 x dx ∫
x x x
1 1
c. (cosh 2 x – sinh 2 x ) =
2 1 sinh 2 x sinh 2 x
= e x cosh 2 x − e x sinh 2 x + C
3 3 ⇒ coth2 x − 1 = csch2 x
By transforming to exponential form:
36. a. Substitute 2x for x in the definition of sinh x.
∫ ∫
1 x 2 x −2 x
e x sinh 2 x dx = e (e − e ) dx 1
2 b. sinh 2 x = (e 2 x – e –2 x )
2

1 1 1
= (e 3 x − e − x ) dx = e 3 x + e − x + C 1 1
2 6 2 = 2 ⋅ (e x – e – x ) ⋅ ⋅ (e x + e − x )
2 2
Transforming to exponential form is easier!
(Note that the two answers can be shown to be = 2 sinh x cosh x
equivalent either by transforming the first to

Calculus Solutions Manual Problem Set 9-9 243


© 2005 Key Curriculum Press
1 2x d. Slice as in part b.
c. cosh 2 x = (e + e –2 x )
2
=
1 2x
4
(e
1
+ 1 + e –2 x ) + ( e 2 x – 1 + e –2 x )
4

Let l = sinh 2 t dt u
sinh t +
dv
sinh t
2
cosh t – cosh t
=  (e x + e – x ) +  (e x – e – x )
2
1 1
2  2 
= cosh x + sinh2 x
2 ∫
= sinh t cosh t − cosh 2 t dt

d. cosh2 x − sinh2 x = 1 ⇒ cosh2 x = 1 + sinh2 x = sinh t cosh t − ∫ (1 + sinh t ) dt 2

⇒ cosh 2x = cosh2 x + sinh2 x


= (1 + sinh2 x) + sinh2 x = 1 + 2 sinh2 x = sinh t cosh t − t − ∫ sinh t dt 2

e. 1 + 2 sinh2 x = cosh 2x
⇒ 2 sinh2 x = cosh 2x − 1 ∫
∴ 2 sinh 2 t dt = sinh t cosh t − t + C

∫ sinh
1
⇒ sinh 2 x = (cosh 2 x – 1) 2
t dt = 0.5 sinh t cosh t − 0.5t + C1
2
37. a. On the circle, u2 + v2 = 1 ⇒ Slicing as in part b, the area A between the
2u du + 2v dv = 0 ⇒ dv = (−u/v) du. upper and lower branches of the hyperbola is
x
dL = du 2 + dv 2 = du 2 + (u/v)2 du 2 A=2 ∫ sinh
2
t dt
0

u2 + v2 1 1 = 2(0.5 sinh x cosh x − 0.5 x ) − 0


= 2 du = du = du = sinh x cosh x − x
v v 1 – u2
Thus, the area of the sector is
1


du cosh x sinh x − (sinh x cosh x − x) = x,
L=
cos 2 1 – u2 Q .E .D .
38. a. y = sinh− 1 x ⇒ sinh y = x ⇒ cosh y y′ = 1
= − cos −1 u = − cos −1 1 + 2 = 2
1
cos 2 1 1 1
y′ = = = , Q. E . D .
The curve along the hyperbola from u = 1 to cosh y sinh y + 1
2
x +1
2
u = cosh 2 has length greater than the line
segment along the horizontal axis from (1, 0) b. y = tanh− 1 x ⇒ tanh y = x ⇒ sech2 y y′ = 1
1 1 1
to (cosh 2, 0). This segment has length y′ = 2 = 2 = , Q.E.D.
L = cosh 2 − 1 = 2.762… . So the length of sec h y 1 – tan h y 1 – x 2
the curve is greater than 2, Q.E.D. c. y = coth− 1 x ⇒ coth y = x
b. The area of the triangle that circumscribes the ⇒ −csch2 y y′ = 1
sector is 0.5(2 sinh 2 cosh 2) = sinh 2 cosh 2. 1 1 1
y′ = 2 = = ,
The area of the sector is the area of this – csch y –(coth y – 1) 1 – x 2
2

triangle minus the area of the region between Q .E .D .


the upper and lower branches of the hyperbola d. y = sech− 1 x ⇒ sech y = x
from u = 1 to u = cosh 2. ⇒ −sech y tanh y y′ = 1
Slice this region vertically. Pick sample point
1
(u, v) on the upper branch, within the strip. y′ =
Let t be the argument of sinh and cosh at the – sech y tanh y
sample point. 0 ≤ t ≤ 2. 1
=
dA = 2v du = 2 sinh t d(cosh t) = 2 sinh2 t dt – sech y 1 – sech 2 y
2
A=2 ∫ sinh t dt ≈ 11.644958K
2
1
0 =− , Q. E . D .
Thus, the area of the sector is x 1 – x2
cosh 2 sinh 2 − 11.644958… = 2, Q.E.D.
e. y = csch− 1 x ⇒ csch y = x
c. By definition of the circular functions, ⇒ −csch y coth y y′ = 1
x is the length of the arc from (1, 0) to 1 1
(cos x sin x). So the total arc has length 2x. y′ = =
– csch y coth y – csch y 1 + csch 2 y
The circumference of a unit circle is 2π, and
its area is π. Thus, 1
=− , Q .E .D .
2x | x| 1 + x 2
Asector = π = x , Q .E .D .

244 Problem Set 9-9 Calculus Solutions Manual


© 2005 Key Curriculum Press
Problem Set 9-10 4. a. y

Q1. Q2. sinh x + C


y

1
x
1 1
x
1

Q3. sinh x Q4. −sin x It might converge because the integrand


Q5. sin x + C Q6. y = x3 becomes infinite only as x approaches zero.
1 1
Q7.
Q9.
y = tan x
y = ex
Q8.
Q10.
y = sinh x or x 3 + x
A
b. ∫ (1/x ) dx = lim ∫ (1/x ) dx
0 a→ 0 +
a
1
1. a. y = lim+ ln | x | = lim+ (0 – ln a) = ∞
1 a→ 0 a a→ 0

The integral diverges.


5. a. y

x
2

It might converge because the integrand 1

approaches zero as x approaches infinity.


1 x
∞ b

∫ ∫ (x
−2
b. (1/ x ) dx = lim
2
) dx
2 b→∞ 2
It might converge because the integrand
b 1
= lim −x −1 = lim( −1/b + 1/2) = approaches zero as x approaches infinity.
b→∞ b→∞ 2 ∞ b

∫ ∫
2
1 b. 1/ x 0.2 dx = lim x −0.2 dx
Integral converges to . 1 b→∞ 1
2 b
2. a. y = lim 1.25 x 0.8
= lim (1.25b 0.8 – 1.25) = ∞
1
b→∞ 1 b→∞

The integral diverges.


6. a. y

x
3

It might converge because the integrand 1


x
approaches zero as x approaches infinity. 1
∞ b
b. ∫3
1/ x 4 dx = lim
b→∞ 3∫ x −4 dx
It might converge because the integrand
b
= lim  – 3 +  =
1 1
−3 1 1 approaches zero as x approaches infinity.
= lim − x
b→∞ 3 →∞  3b 81  81 ∞ b

∫ ∫
b
1/ x 1.2 dx = lim x −1.2 dx
3
1 b.
1 b→∞ 1
The integral converges to .
81 b
3. a. y = lim − 5 x −0.2 = lim (–5b –0.2 + 5) = 5
b→∞ 1 b→∞
The integral converges to 5.
7. a. y
1
x
1

1
It might converge because the integrand x
approaches zero as x approaches infinity. 1
∞ b
b. ∫1
(1/ x ) dx = lim ∫ (1/x ) dx
b→∞ 1
It might converge because the integrand
b
= lim ln | x | = lim (ln b – 0) = ∞ becomes infinite only as x approaches zero.
b→∞ 1 b→∞

The integral diverges.

Calculus Solutions Manual Problem Set 9-10 245


© 2005 Key Curriculum Press
1 1

∫ ∫x
−0.2 11. a. y
b. 1/ x 0.2 dx = lim+ dx 1
x
0 a→0 a
0 1
1
= lim+ 1.25 x 0.8 = lim+ (1.25 – 1.25a 0.8 )
a→ 0 a a→ 0

= 1.25
The integral converges to 1.25.
8. a. y It might converge because the integrand
becomes infinite only as x approaches 0 or 1.
b. To determine whether this converges, split the
integral into two pieces. Each piece must
1 converge in order for the integral to converge.
x
The integral can be written
1 1

∫ 1/( x ln x ) dx
0
It might converge because the integrand c 1
becomes infinite only as x approaches zero. = ∫ 1/( x ln x ) dx + ∫ 1/( x ln x ) dx
0 c
1 1

∫ 1/x ∫x
c b
= lim ∫ 1/( x ln x ) dx + lim ∫ 1/( x ln x ) dx
−1.2
b. 1.2
dx = lim+ dx + −
0 a→0 a a→0 a b→ 1 c
1 c b
= lim+ − 5 x −0.2 = lim+ (–5 + 5a –0.2 ) = ∞ = lim+ ln | ln x | + lim− ln | ln x |
a→ 0 a a→0 a→0 a b→1 c
= lim+(ln |ln c | – ln |ln a |)
The integral diverges. a→ 0

9. a. y
+ lim−(ln |ln b | – ln |ln c |)
b→1
1 =∞+∞
For the integral to converge, both limits must
exist. Because neither exists, the integral
x
diverges.
0 1
12. a.
y

It might converge because the integrand


approaches zero as x approaches infinity.
∞ b 1
b. ∫0
1/(1 + x 2 ) dx = lim
b→∞ 0 ∫ 1/(1 + x 2 ) dx x
0 1 3
b π
= lim tan −1 x = lim (tan –1 b – 0) = It might converge because the integrand
b→∞ 0 b→∞ 2 approaches zero as x approaches infinity.
The integral converges to π /2. b. The indefinite integral can be written
10. a. ∫(ln x ) −2 ( dx/ x ) = −(ln x ) −1 + C.
∞ b

∫ 1/[ x (ln x )2 ] dx = lim ∫ 1/[ x(ln x ) ] dx


1 2
3 b→∞ 3
b
= lim − (ln x ) −1
b→∞ 3
0 1 x = lim [–(ln b) –1 + (ln 3) −1 ] = (ln 3) −1
b→∞

It might converge because the integrand The integral converges to


approaches zero as x approaches infinity. (ln 3)− 1 = 0.910239… .
∞ b
13. a.
∫ 1/(1 + x ) dx = lim ∫ 1/(1 + x ) dx y
b. 1
0 b→∞ 0
b
= lim ln | x + 1| = lim [ln (b + 1) – 0] = ∞
b→∞ 0 b→∞
The integral diverges. x
2

It might converge because the integrand


approaches zero as x approaches infinity.

246 Problem Set 9-10 Calculus Solutions Manual


© 2005 Key Curriculum Press
∞ b

∫ ∫e
17. a.
b. e −0.4 x dx = lim −0.4 x
dx
2 b→∞ 2 y
b 1
−0.4 x
= lim − 2.5e
b→∞ 2
= lim(–2.5e −0.4 b + 2.5e –0.8 ) = 2.5e −0.8
b→∞
The integral converges to 2.5e− 0.8 x

= 1.1233… . 0 1

14. a.
It might converge because the integrand seems
y to approach zero as x approaches infinity.
1 b. Integrate by parts:
∫xe − x dx = − e − x ( x + 1) + C
∞ b

0 1 x
∫0
xe − x dx = lim
b→∞ 0 ∫ xe − x dx
b
= lim − e − x ( x + 1) = lim [– e – b (b + 1) + 1] = 1
b→∞ 0 b→∞
It diverges because the integrand does not
(The first term is zero by l’Hospital’s rule.)
approach zero as x approaches infinity.
Integral converges to 1.
b. (Not applicable)
18. a.
15. a. y
y

1
1
x
x
0 1 3
–1 0 2

It might converge because the integrand


It does not converge because the integrand is
becomes infinite only as x approaches 1.
undefined for x < 0. 3

∫ ( x – 1) dx
−2
b. (Not applicable) b.
0
16. a. b 3
y
= lim ∫ ( x – 1) −2
dx + lim+ ∫ ( x – 1)
−2
dx
b→1 –
0 a→1 a
b 3
= lim– − ( x − 1) −1 + lim+ − ( x − 1) −1
b→1 0 a→1 a
1 = lim– [–(b – 1) + (–1) ] –1 –1
b→1
x
+ lim+ [–2 –1 + ( a – 1) –1 ]
0 1 3 7 a→1
=∞+∞
It might converge because the integrand For the integral to converge, both limits must
becomes infinite only as x approaches 3. exist. Because neither exists, the integral
7 diverges.
∫ ( x – 3) dx
−2/3
b.
1 19. a.
b 7
= lim ∫ ( x – 3) ∫ ( x – 3)
y
−2/3 −2/3
dx + lim+ dx
b→3− 1 a→3 a 1
b 7
= lim− 3( x − 3) 1/3
+ lim+ 3( x − 3)1/3 x
b→3 1 a→3 a 20

= lim− [3(b – 3)1/3 − 3( −2)1/3 ]


b→ 3
+ lim+ [3( 4)1/3 − 3( a − 3)1/3 ]
a→3
= 3 ⋅ 21/3 + 3 ⋅ 41/3 It diverges because the integrand does not
The integral converges to 3 ⋅ 21/3 + 3 ⋅ 41/3 approach zero as x approaches infinity.
= 8.5419… . b. (Not applicable)

Calculus Solutions Manual Problem Set 9-10 247


© 2005 Key Curriculum Press
20. a. 24. y = −1/x ⇒ x = −y − 1
y Slice the vertical cross section horizontally.
1 dA = x dy = −y − 1 dy
–1 −1


x
20 A = lim − y −1 dy = lim − ln | y|
a→−∞ a a→ – ∞ a

= lim (– ln | – 1| + ln |a|) = ∞
a→−∞

The area of the bucket’s surface is greater than


It diverges because the integrand does not
the area of the cross section, and the cross-
approach zero as x approaches infinity.
sectional area diverges. Thus, the bucket has
b. (Not applicable) infinite surface area. The bucket is congruent to
b
21. As b → ∞, ∫ cos x dx oscillates between −1 and
0
the solid in Problem 23b, which has volume
approaching π. Thus, π cubic units of paint
1 and never approaches a limit. Similarly, would fill the bucket but could not coat the
b

∫ sin x dx oscillates between 0 and 2.


whole surface!
0 ∞

22. a. I = ∫ 1/ x
1.001

dx = lim ∫ x 1.001
dx
b
−1.001
25. a. f ( x ) = ∫0
t x e − t dt
1 b→∞ 1 b b

= lim −1000 x
b→∞
−0.001
b f (1) = lim
b→∞ 0 ∫ te − t dt = lim (– te – t – e – t )
b→∞ 0
1
= lim (– be – e –b –b
+ 0 + 1) = 1
= lim (–1000 b –0.001
+ 1000) b→∞
b→∞
= 1000 (converges), Q.E.D. (Using l’Hospital’s rule on be− b gives
∞ b

∫ ∫
I 0.999 = 1/ x 0.999 dx = lim x −0.999 dx b 1
b→∞ 1
lim be − b = lim b = lim b = 0. )
1 b→∞ b→∞ e b→∞ e
b ∞
= lim 1000 x 0.001
b→∞ 1
f (2) = lim
b→∞ 0 ∫ t 2 e − t dt
= lim (1000 b 0.001 – 1000) b ∞
b→∞
= ∞ (diverges), Q.E.D.
= lim − t 2 e − t
b→∞ 0
+2 ∫0
te − t dt

= lim (– b 2 e – b + 0) + 2(1) = 2
b. I1 = ∫
1
1/ x dx = ∞ (see Problem 3), so I1 b→∞

diverges. (Using l’Hospital’s rule on b2e− b gives


c. “Ip converges if p > 1 and diverges if p ≤ 1.” 2b 2
lim b 2 e − b = lim b = lim b = 0.)
b→∞ b→∞ e b→∞ e
23. a. y = 1/x = x − 1

dA = y dx = x− 1 dx

f (3) = lim ∫ t 3e − t dt

−1 b→∞ 0
A= x dx = ∞ (See Problem 3) ∞


b
1
= lim − t 3e − t +3 t 2 e − t dt
The area does not approach a finite limit. b→∞ 0 0

b. By plane slices, dV = πy2 dx = πx− 2 dx. = lim (– b 3e – b + 0) + 3(2) = 6


b→∞
b b
V = lim
b→∞ 1 ∫ π x −2 dx = lim −π x −1
b→∞ 1
(Using l’Hospital’s rule on b3e− b gives
= lim (–π b –1 + π ) = π 3b 2 6b 6
b→∞ lim b 3e − b = lim b = lim b = lim b = 0.)
b→∞ b→∞ e b→∞ e b→∞ e
The volume converges to π. b. Conjecture:
c. By cylindrical shells, dV = 2π xy dx f ( 4) = 4 f (3) = 24 = 4!
= 2π x(x− 1) dx = 2π dx.
b b f ( 5) = 5 f (4) = 120 = 5!
V = lim
b→∞ 1 ∫ 2π dx = lim 2π x
b→∞ 1
f ( 6) = 6 f (5) = 720 = 6!
= lim (2πb – 2π ) = ∞ ∞


b→∞
c. f ( x ) = t x e − t dt u dv
Volume diverges. 0
tx + e –t
d. False. The volume could approach a constant xt x–1 – –e –t
as in part b or become infinite as in part c.

248 Problem Set 9-10 Calculus Solutions Manual


© 2005 Key Curriculum Press
∞ i. (−1)! = 0!/0, which is infinite. So (−2)! and

b
= lim − t x e − t +x t x −1e − t dt
b→∞ 0 0 (−3)!, which equal (−1)!/(−1) and (−2)!/(−2),
= lim (– b x e – b + 0) + x f ( x − 1) are also infinite. However,
b→∞
(−0.5)! = 0.5!/(0.5) = 1.77245…
= 0 + 0 + x f (x − 1) (−1.5)! = (−0.5)!/(−0.5) = −3.54490…
= x f (x − 1), Q .E .D . (−2.5)! = (−1.5)!/(−1.5) = 2.36327…
( lim (– b x e – b ) = 0 can be proved by all of which are finite.
b→∞
π
mathematical induction using j. 0.5! = = 0.886226925K , which agrees
l’Hospital’s rule.) 2
with the tabulated value.
d. Part a shows that f (1) = 1 = 1!.
26. dW = F dr = 1000r− 2 dr
Part c shows that f (n) = nf (n − 1) =
At the earth’s surface, r = 1.
n(n − 1) f (n − 2) = n(n − 1)(n − 2)…(2)(1) ∞
W = 1000 r −2 dr = lim  –1000 r −1 1 

b
= n!, Q .E .D .
1 b→∞
1000

∫ = lim (–1000 b –1 + 1000)


3 −t
e. t e dt ≈ 6 b→∞
0
The value of b that makes the integral come = 1000 radius-pounds
within 0.000001 of 6 can be found numerically Thus, the amount of work does not increase
(though it will be slow), or algebraically: without bound as r goes to infinity.
b 3 x−2
∫te ∫
3 −t
2 −
x
dt 27. a.  dx
0 1  x−2
3 −b 2 −b −b −b
= − b e − 3b e − 6be − 6e + 6 2x−2 3 x−2
= ∫
2 −  dx +  2 − ∫
x x
|b3e− b + 3b2e− b + 6be− b + 6e− b| < 0.000001  dx
1  x−2 2 x−2
for b > 23.4050… , say, b ≈ 24. 2 x−2 3 x−2

∫ 2 −  dx +  2 − ∫
x x
24
 dx
∫ ∫
b.
f. 0.5! = t 0.5e − t dt ≈ t 0.5e − t dt ≈ 1  x−2 2 x−2
0 0
x−2
b
0.886227311… = lim− ∫  2 −
x
 dx
From the graphs, t0.5 e− t < t3e − t for x ≥ 24. The b→2 1 x−2
error in 0.5! from stopping at b = 24 is the 3 x−2
area under the “tail” of the graph from b = 24.
+ lim+  2 x −
a→2 a  ∫
x−2
 dx
∞ ∞
Error = ∫24
t 0.5e − t dt < ∫ 24
t 3e − t dt < 0.000001 c. lim−
b→2 1
bx−2
∫  2
x−2
x
 dx −
The difference between the tabulated value
3 x−2
of 0.5! and the value calculated here is
0.8862269255
+ lim+  2 x −
a→ 2 a  ∫
x−2
 dx
b 3
− 0.866227311… = lim− ∫ (2 + 1) dx + lim+ ∫ (2 − 1) dx
x x

= −0.000000386 b→2 1 a→2 a


b 3
which is less in absolute value than 0.000001. = lim− (2 x /ln 2 + x ) + lim+ (2 x /ln 2 – x )
b→2 1 a→2 a
Note, however, that the difference is negative
because the calculated value is larger than = lim− (2 /ln 2 + b – 2/ln 2 − 1)
b
b→2
the tabulated value. This observation means + lim+ (2 3 /ln 2 – 3 – 2 a /ln 2 + a)
that either the tabulated value is incorrect or a→ 2

there is more inaccuracy in the numerical = 4/ln 2 + 2 − 2/ln 2 − 1 + 8/ln 2 − 3


integration algorithm than there is in the error − 4/ln 2 + 2 = 6/ln 2
caused by dropping the tail of the integral. The integral converges to 6/ln 2 = 8.6561… .
(Using a smaller tolerance in the numerical d. The integral is defined by dividing the
integrator gives a value of 0.8862269252… .) interval into Riemann partitions and summing
the subintervals. But the Riemann partitions
g. Using the tabulated value of 0.5!,
may be chosen so that the discontinuities
1.5! = 1.5(0.5!) = 1.3293…
are at endpoints of subintervals. Then the
2.5! = 2.5(1.5!) = 3.3233…
subintervals corresponding to each continuous
3.5! = 3.5(2.5!) = 11.6317…

piece may be summed separately.
∫ t 0 e − t dt = lim  – e – t 0
b
h. 0! = e. False. Some discontinuous functions (notably,
0 b→∞
piecewise continuous functions) are integrable.
= lim(– e – b + 1) = 1, Q .E.D . 28. Answers will vary.
b→∞

Calculus Solutions Manual Problem Set 9-10 249


© 2005 Key Curriculum Press
Problem Set 9-11 dx =  6 −
23 
∫ ∫
6 x – 11
15. dx
x+2  x + 2
1. y = sec 3x tan 3x ⇒
= 6x – 23 ln | x + 2 | + C
y′ = (3 sec 3x tan 3x) tan 3x + sec 3x (3 sec2 3x)
5x + 9
dx =  5 +
29 
= 3 sec 3x tan2 3x + 3 sec3 3x
2. y = sinh 5x tanh 5x ⇒
16. ∫x–4  ∫
x – 4
dx

y′ = (5 cosh 5x) tanh 5x + sinh 5x (5 sech2 5x) = 5x + 29 ln | x − 4 | + C


= 5 sinh 5x + 5 sinh 5x sech2 5x or 17. f (t ) = 1 + t 2 = (1 + t 2 )1/2 ⇒
5 sinh 5x + 5 tanh 5x sech 5x 1 t
f ′(t ) = (1 + t 2 ) −1/2 (2t ) =
∫ x cosh 4 x dx 1+ t2
3. u dv 2
x cosh 4x
+
1 18. g(t ) = t 2 – 1 = (t 2 − 1)1/2 ⇒
1 – 4 sinh 4x
1 t
0 +
1
16 cosh 4x g′(t ) = (t 2 – 1) −1/2 (2t ) =
2 2
t –1
1 1
=
4
x sinh 4 x − cosh 4 x + C
16
19. ∫ 1 + t 2 dt = ∫ 1 + tan 2 θ d ( tan θ )


1
= sec 3 θ dθ = sec θ tan θ
4. ∫ x cos x dx u
x +
dv
cos x 1
2
1 – sin x + ln | sec θ + tan θ | + C
0 + –cos x 2
1 1
= t 1 + t 2 + ln 1 + t 2 + t + C
= x sin x + cos x + C 2 2
5. f (x) = (3x + 5)− 1 ⇒ f ′(x) = −3(3x + 5)− 2
6. f (x) = (5 − 2x− 1) ⇒ f ′(x) = 2(5 − 2x)− 2
20. ∫ t 2 – 1 dt = ∫ sec 2 θ – 1 d (sec θ )


1
7. (3 x + 5) −1 dx = ln |3 x + 5| + C ∫ ∫
= sec θ tan 2 θ dθ = (sec 3 θ − sec θ ) dθ
3 1 1
= sec θ tan θ + ln | sec θ + tan θ |

1
8. (5 − 2 x ) −1 dx = − ln |5 – 2 x | + C 2 2
2 − ln | sec θ + tan θ | + C
9. t(x) = tan5 4x ⇒ 1 1
= sec θ tan θ − ln | sec θ + tan θ | + C
t′(x) = 5 tan4 4x (4 sec2 4x) = 20 tan4 4x sec2 4x 2 2
1 1
10. h(x) = sech3 7x ⇒ = t t – 1 − ln t + t 2 – 1 + C
2

h′(x) = 3 sech2 7x (−7 sech 7x tanh 7x) 2 2


= −21 sech3 7x tanh 7x 21. y = x e ⇒
3 x

y′ = 3x2ex + x3ex = x2ex(3 + x)


∫ ∫
1
11. sin 2 x dx = (1 − cos 2 x ) dx
2 22. y = x4e− x ⇒
1 1 y′ = 4x3e− x − x4e− x = x3e− x(4 − x)
= x − sin 2 x + C
2 4
∫x e
3 x u dv
1 1 23. dx
= x − sin x cos x + C (or integrate by parts) x3 + ex
2 2 3x 2 – ex
ex
∫ ∫
1 6x
12. cos 2 x dx = (1 + cos 2 x ) dx
+

2 6 – ex
1 1 0 + ex
= x + sin 2 x + C
2 4 = x3ex − 3x2 ex + 6xex − 6ex + C
1 1
= x + sin x cos x + C (or integrate by parts)
∫x e
4 −x u dv
2 2 24. dx
6 x – 11 x4 + e –x
13. y = ⇒ 4x 3 – –e –x
x+2 12x 2 + e –x
6( x + 2) – (6 x – 11)(1) 23 24x –e –x
y′ = = 24

e –x
( x + 2) 2
( x + 2)2 +
0 –
–e –x
5x + 9
14. y = ⇒ = −x4 e− x − 4x3 e− x −12x2 e− x − 24xe− x − 24e− x + C
x–4
5( x – 4) – (5 x + 9)(1) –29 1
y′ = = 25. f ( x ) = sin −1 x ⇒ f ′( x ) = = (1 − x 2 ) −1/2
( x – 4) 2 ( x – 4) 2 1– x 2

250 Problem Set 9-11 Calculus Solutions Manual


© 2005 Key Curriculum Press
1 33. f (x) = tanh x ⇒ f ′(x) = sech2 x
26. g( x ) = tan −1 x ⇒ g′( x ) =
x +1
2
34. f (x) = coth x ⇒ f ′(x) = −csch2 x

∫ ∫
sinh x dx
35. tanh x dx = = ln |cosh x | + C
∫ sin
−1 u dv
27. x dx cosh x
sin –1 x + 1
(1 – x 2)–1/2 – x (Absolute value is optional.)

∫ ∫
cosh x dx
36. coth x dx = = ln | sinh x | + C

= x sin −1 x − (1 − x 2 ) −1/2 ( x dx ) sinh x
(Absolute value is necessary.)
= x sin− 1 x − (−0.5)(2)(1 − x2)1/2 + C
37. y = e2x cos 3x
= x sin −1 x + 1 – x 2 + C
y′ = (2e2x) cos 3x + e2x(−3 sin 3x)
= e2x(2 cos 3x − 3 sin 3x)
∫ tan
−1 u dv
28. x dx
tan –1 x + 1 38. y = e −3x cos 4x
1
y′ = (−3e−3x) cos 4x + e−3x(−4 sin 4x)

x
1 + x2
= −e−3x(3 cos 4x + 4 sin 4x)

∫ 1+ x
1
= x tan −1 x − ( x dx )
∫e
2 2x u dv
39. cos 3 x dx
1 e 2x cos 3x
= x tan −1 x −
+
ln |1 + x 2 | + C 2e 2x
1
2 – 3 sin 3x
1
4e 2x +
– 9 cos 3x
dx = 
–1/6 1/6 
∫ ∫
1
29. + dx
x2 + 4x – 5  x + 5 x – 1
1 1 1 2
= − ln | x + 5| + ln | x − 1| + C = e 2 x sin 3 x + e 2 x cos 3 x
6 6 3 9


4 2x
dx = 
1/8  −
∫ ∫
1 –1/8 e cos 3 x dx
30. + dx
2
x – 6x – 7  x + 1 x – 7 9


1 1 13 2 x
= − ln | x + 1| + ln | x − 7| + C 9
e cos 3 x dx
8 8
1 2
= e 2 x sin 3 x + e 2 x cos 3 x + C1
∫ ∫
1 1
31. dx = dx 3 9
x + 4x – 5
2
( x + 2)2 – 9
∫e
2x
cos 3 x dx

1
= (3 sec θ tan θ dθ )
(3 sec θ )2 – 9 3 2x 2
= e sin 3 x + e 2 x cos 3 x + C
13 13
∫ ∫
1
= (3 sec θ tan θ dθ ) = sec θ dθ
3 tan θ
∫e
−3 x u dv
40. cos 4 x dx
= ln | sec θ + tan θ | + C e –3x cos 4x
+
1 1 1
= ln ( x + 2) + ( x + 2)2 − 9 + C1 –3e –3x – 4 sin 4x
3 3 1
9e –3x + – 16 cos 4x
= ln x + 2 + x 2 + 4 x − 5 + C
1 −3 x 3
= e sin 4 x − e −3 x cos 4 x
∫ ∫
1 1
32. dx = dx 4 16
x2 − 6x − 7 ( x − 3)2 − 16

9
− e −3 x cos 4 x dx

1
= ( 4 sec θ tan θ dθ ) 16
( 4 sec θ ) – 16 2


25 −3 x
e cos 4 x dx
∫ ∫
1 16
= ( 4 sec θ tan θ dθ ) = sec θ dθ
4 tan θ 1 3
= ln |sec θ + tan θ | + C = e −3 x sin 4 x − e −3 x cos 4 x + C1
4 16

∫e
1 1 −3 x
= ln ( x − 3) + ( x – 3)2 – 16 + C1 cos 4 x dx
4 4
4 −3 x 3
= ln x − 3 + x 2 – 6 x – 7 + C = e sin 4 x − e −3 x cos 4 x + C
25 25

Calculus Solutions Manual Problem Set 9-11 251


© 2005 Key Curriculum Press
∫ ( x – 1)( x – 2)( x – 3) dx
Note: As a check for integrals such as x
48.
Problems 39 and 40, the numerators of the
coefficients equal the 3 and 4 in the arguments
= 
3/2 

1/2 2
of e−3x and cos 4x. The denominators equal − + dx
 x – 1 x – 2 x – 3
32 + 42, or 25.
41. g (x) = x 3 ln 5x ⇒ 1 3
= ln | x − 1| −2 ln | x − 2| + ln | x − 3| + C
g ′ (x) = (3x2) ln 5x + x3 (5/5x) 2 2
= x 2 (3 ln 5x + 1) 49. y = cos3 x sin x ⇒
42. h (x) = x 2 ln 8x ⇒ y′ = (−3 cos2 x sin x) sin x + cos3 x (cos x)
h ′(x) = (2x) ln 8x + x2 (8/8x) = −3 cos2 x sin2 x + cos4 x
= x(2 ln 8x + 1) 50. y = sin5 x cos x ⇒
y′ = (5 sin4 x cos x) cos x + sin5 x (−sin x)
∫ x ln 5x dx = 5 sin4 x cos2 x − sin6 x
3 u dv
43.
ln 5x x3

+ 1
1 1 4 51. cos3 x (sin x dx ) = − cos 4 x + C
x 4x
------------------------ 4
1 3

1 6
1 – 4x 52. sin x (cos x dx ) = sin x + C
5
+ 1 4 6
0 16 x
∫ cos x dx = ∫ (1 − sin x ) cos x dx
3 2
53.
1 4 1
= x ln 5 x − x 4 + C
4 16 = ∫ cos x dx − ∫ sin x (cos x dx )
2

1
∫x = sin x − sin 3 x + C
2 u dv
44. ln 8 x dx 3
ln 8x + x2
∫ ∫
1 1 3 1 2
x 3x Or: cos3 x dx = cos 2 x sin x + cos x dx
------------------------ 3 3
1 2
1 – 3x 1 2
0
1 3 = cos 2 x sin x + sin x + C
9x
+
3 3

∫ sin x dx = ∫ (1 − cos x ) (sin x dx )


5 2 2
1 1 54.
= x 3 ln 8 x − x 3 + C
= ∫ (1 − 2 cos x + cos x )(sin x dx )
3 9 2 4
x
45. y = ⇒
( x + 2)( x + 3)( x + 4) = ∫ sin x dx − 2 ∫ cos x sin x dx
2

ln y = ln x − ln (x + 2)
− ln (x + 3) − ln (x + 4) ⇒ + ∫ cos x sin x dx
4

y′ = y[x − 1 − (x + 2)− 1 − (x + 3)− 1 − (x + 4)− 1]


2 1
=
x
⋅ [ x −1 − ( x + 2) −1 = − cos x +cos3 x − cos 5 x + C
3 5
( x + 2)( x + 3)( x + 4)
∫ ∫
1 4
− (x + 3)− 1 − (x + 4)− 1] Or: sin 5 x dx = − sin 4 x cos x + sin 3 x dx
5 5
x
46. y = ⇒

1 4 8
( x – 1)( x – 2)( x – 3) = − sin 4 x cos x − sin 2 x cos x + sin x dx
5 15 15
ln y = ln x − ln (x − 1) 1 4
− ln (x − 2) − ln (x − 3) ⇒ = − sin 4 x cos x − sin 2 x cos x
5 15
y′ = y[x − 1 − (x − 1)− 1 − (x − 2)− 1 − (x − 3)− 1]
8
x − cos x + C
= ⋅ [ x −1 − ( x − 1) −1 15
( x – 1)( x – 2)( x – 3)
∫ ∫
1 3
− (x − 2)− 1 − (x − 3)− 1] 55. cos 4 x dx = cos3 x sin x + cos 2 x dx
4 4


x

47. dx 1 3 3
( x + 2)( x + 3)( x + 4) = cos3 x sin x + cos x sin x + dx
4 8 8
= 
2 

–1 3 1 3 3
+ − dx = cos3 x sin x + cos x sin x + x + C
 x + 2 x + 3 x + 4 4 8 8
= −ln |x + 2| + 3 ln |x + 3| − 2 ln |x + 4| + C

252 Problem Set 9-11 Calculus Solutions Manual


© 2005 Key Curriculum Press
2

∫ ∫ ∫
1 5
56. sin 6 x dx = − sin 5 x cos x + sin 4 x dx 68. xe − x dx u dv
6 6 0 x + e –x
1 5 1 – –e –x
= − sin 5 x cos x − sin 3 x cos x 0 + e –x
6 24


15
= − xe − x − e − x = −2e −2 − e −2 + 0 + 1
2
+ sin 2 x dx 0
24
1 5 = −3e− 2 + 1 = 0.59399…
= − sin 5 x cos x − sin 3 x cos x
6 24 69. r(x) = xex ⇒ r′(x) = xex + ex
70. s(x) = xe− x ⇒ s′(x) = −xe− x + e− x

15 15
− sin x cos x + dx
48 48 ln x + 2
71. q( x ) = ⇒
1 5 x
= − sin 5 x cos x − sin 3 x cos x
6 24 (1/ x ) ⋅ x − (ln x + 2) ⋅ 1 −1 − ln x
q ′( x ) = =
5 5 x2 x2
− sin x cos x + x+C
16 16 (ln x )3 + 4
72. r ( x ) = ⇒
57. g (x) = (x + 3) ⇒ 4 3
x
g ′(x) = 3(x4 + 3)2(4x3) = 12x3(x4 + 3)2 3(ln x )2 (1/ x ) ⋅ x − [(ln x )3 + 4] ⋅ 1
58. f (x) = (x − 1) ⇒ 3 4 r ′( x ) =
x2
f ′(x) = 4(x3 − 1)3(3x2) = 12x2(x3 − 1)3
3(ln x )2 − (ln x )3 − 4
=
∫ (x ∫
+ 3) dx = ( x + 9 x + 27 x + 27) dx
4 3 12 8 4
59. x2
ln x + 2
∫ ∫
dx
=
1 13 9 27 5
x +x + x + 27 x + C 73. dx = (ln x + 2)
13 5 x x
1
∫ ( x − 1) dx = (ln x + 2)2 + C
3 4
60. 2

= ∫ (x − 4x (ln x )3 + 4
∫ ∫ ∫ x dx
12 9
+ 6 x 6 − 4 x 3 + 1) dx dx 4
74. dx = (ln x )3 +
x x
1 13 2 10 6 7 4 1
= x − x + x − x + x+C = (ln x ) 4 + 4 ln | x | + C
13 5 7 4


1
61. ( x 4 + 3)3 x 3 dx = ( x 4 + 3) 4 + C (The absolute values are optional because ln x
16 appears in the original integrand, so only


1 positive values of x can be used.)
62. ( x 3 − 1) 4 x 2 dx = ( x 3 − 1)5 + C
15 2
75. f ( x ) = e x ⇒ f ′( x ) = 2 xe x
2


1
( x 4 + 3) dx = x 5 + 3 x + C
3 3
63. 76. f ( x ) = e x ⇒ f ′( x ) = 3 x 2 e x
5


2 1 2
xe x dx = e x + C

1 77.
64. ( x 3 − 1) dx = x 4 − x + C 2
4


3 1 3
x
x 2 e x dx = e x + C
∫ (t
78.
65. f ( x ) = 4
+ 3)3 dt ⇒ f ′( x ) = ( x 4 + 3)3 3
1
x
66. h( x ) = ∫ (t − 1) 4 dt ⇒ h ′( x ) = ( x 3 − 1) 4
∫x e
3 x2
3
79. dx u dv
5
x2 + xe x 2
1 x2
2 2x 2e
67. ∫
1
xe x dx u
x +
dv
ex
---------------------
2 –
1 x2
2 xe
1 – ex 0
1 x2
4e
+
0 + ex
1 2 x2 1 x2
= xe x − e x
2
= 2e 2 − e 2 − e + e = e 2 = 7.3890 K = x e − e +C
1 2 2

Calculus Solutions Manual Problem Set 9-11 253


© 2005 Key Curriculum Press
∫x e ∫ cos ∫ (1 + cos 2cx ) dx
5 x3 u dv 1
80. dx 84. 2
cx dx =
x3 + x 2e x 3 2
1 x3 1 1
3x 2 3e = x+ sin 2cx + C (for c ≠ 0)
---------------------
2 4c
1 x 2e x 3

0 + 1 x3 cos 2 cx dx = x + C (for c = 0)
3e
ax + b
85. f ( x ) =
1 3 1 3
= x 3e x − e x + C cx + d
3 3 a(cx + d ) – c( ax + b) ad – bc
f ′( x ) = =
(cx + d ) 2
(cx + d )2
∫e
ax u dv
81. cos bx dx (for c, d not both 0)
e ax + cos bx (undefined for c = d = 0)
1
ae ax – b sin bx 86. f (x) = (ax + b)n
f ′( x ) = na( ax + b) n−1
1
a 2e ax + –b 2 cos bx
(for a, b not both 0, or n ≥ 1)
f ′(x) = 0 (for a = b = 0 and 0 ≤ n ≤ 1)
a2
∫e
1 ax a
= e sin bx + 2 e ax cos bx − 2 ax
cos bx dx (undefined for a = b = 0 and n < 0)
b b b
a2 + b2 ax + b a b − ( a/c)d 
b2 ∫
e ax cos bx dx 87. ∫ cx + d dx = ∫  c + cx + d 
dx
1 a ax bc – ad
= e ax sin bx + 2 e ax cos bx + C1 = + ln |cx + d | + C (for c ≠ 0)
b b c c2
ax + b
∫ ∫
a 2 b
dx = x + x+C
ax
e cos bx dx
cx + d 2d d
b a (for c = 0, d ≠ 0)
= 2 e ax sin bx + 2 e ax cos bx + C
a + b2 a + b2 (undefined for c = d = 0)
(for a, b not both 0)
( ax + b) n+1
∫e cos bx dx = x + C
ax
(for a = b = 0) 88. ∫ ( ax + b) n dx =
a(n + 1)
+C

(for n ≠ −1, a ≠ 0)
∫e ∫
ax u dv 1
82. sin bx dx ( ax + b) dx = ln |ax + b| + C
n
e ax + sinbx a
ae ax
1
– b cosbx (for n = −1, a ≠ 0)


a 2e ax +
1
– 2 sinbx ( ax + b) dx = b x + C
n n
(for a = 0)
b

∫ ∫
x dx 1
1 a 89. = ( x 2 + a 2 ) −1/2 (2 x dx )
= − e ax cos bx + 2 e ax sin bx x +a
2 2 2
b b 1
a2 = ⋅ 2( x 2 + a 2 )1/2 + C = x 2 + a 2 + C

− 2 e sin bx dx
ax
2
b
∫ ∫
x dx 1
a2 + b2 90. =− ( a 2 − x 2 ) −1/2 ( −2 x dx )
b2 ∫
e ax sin bx dx a −x
1
2 2 2
1 a
= − e ax cos bx + 2 e ax sin bx + C1 = − ⋅ 2( a 2 − x 2 )1/ 2 + C = − a 2 – x 2 + C
2
b b
(for a ≠ 0)
∫e ax sin bx dx (undefined for a = 0)
a b d ( a tan θ )
∫ ∫
dx
= e ax sin bx − 2 e ax cos bx + C 91. =
a2 + b2 a + b2 x +a
2 2
a 2 tan 2 θ + a 2
(for a, b not both 0)
a sec 2 θ dθ
∫ e sin bx dx = C
ax
(for a = b = 0) = ∫ a sec θ
= sec θ dθ ∫
= ln |sec θ + tan θ | + C1
∫ sin ∫
1
83. 2
cx dx = (1 − cos 2cx ) dx
2 1 2 1
= ln x + a 2 + x + C1
1 1 a a
= x− sin 2cx + C (for c ≠ 0)
2 4c = ln x 2 + a2 + x + C
∫ sin 2 cx dx = C (for c = 0)

254 Problem Set 9-11 Calculus Solutions Manual


© 2005 Key Curriculum Press
d ( a sin θ )
∫ ∫
dx 1
92. = = x cos −1 ax − 1 – ( ax )2 + C
a −x 2 2
a − a sin θ 2 2 2 a
(for a ≠ 0)
a cos θ dθ
∫ ∫
x
= = dθ = θ + C = sin −1 + C π

−1
a cos θ a cos ax dx = x + C (for a = 0)
2
(for a ≠ 0)
(undefined for a = 0)
∫ sin
−1 u dv
100. ax dx
93. f (x) = x sin ax ⇒ f ′(x) = 2x sin ax + ax2 cos ax
2
sin –1ax + 1
94. f (x) = x2 cos ax ⇒ f ′(x) = 2x cos ax − ax2 sin ax a –
x
√ 1 – (ax) 2

∫x
2 u dv
95. sin ax dx

x2 + sinax = x sin −1 ax −
ax dx
1
2x –
– a cosax 1 – ( ax )2
1


2 – 2 sinax 1
+ a = x sin −1 ax + [1 − ( ax )2 ]−1/2 ( −2 a 2 x dx )
– 1 2a
0 a3
cosax
1
= x sin −1 ax + 1 – ( ax )2 + C (for a ≠ 0)
a
1 2 2 2
=−
a
x cos ax + 2 x sin ax + 3 cos ax + C
a a ∫sin −1 ax dx = C (for a = 0)
(for a ≠ 0)
∫ 1 + x dx
1
101. Let u = 1 + x .
∫ x 2 sin ax dx = C (for a = 0)
x = (u − 1)2
dx = 2(u − 1) du
∫x
2 u dv
96. cos ax dx
∫ ∫ ∫
2(u – 1) du
x2 + cosax = = 2 du − (2/u) du
1 u
2x – a sinax = 2u − 2 ln |u| + C
1
2 + – 2 cosax
a = 2(1 + x ) − 2 ln |1 + x | + C
1
0 –
– sinax Or: 2 x − 2 ln |1 + x | + C1
a3
Absolute values are optional because
1 2 2 2 1 + x > 0.
= x sin ax + 2 x cos ax − 3 sin ax + C

a a a 1
102. dx Let u = 1 − x .
(for a ≠ 0) 1– x


1 3 x = (1 − u)2
x cos ax dx = x + C
2
(for a = 0)
3 dx = 2(u − 1) du

∫ ∫ ∫
2(u – 1) du

1
97. sinh ax dx = cosh ax + C (for a ≠ 0) = = 2 du − (2/u) du
a u
= 2u − 2 ln |u| + C
∫ sinh ax dx = C (for a = 0)
= 2(1 − x ) − 2 ln |1 − x | + C

∫ cosh ax dx = a sinh ax + C
1 Or: −2 x − 2 ln |1 − x | + C1
98. (for a ≠ 0)


1
∫ cosh ax dx = x + C (for a = 0) 103.
1+ 4 x
dx Let u = 1 + 4 x .

x = (u − 1)4
∫ cos dx = 4(u − 1)3 du
−1 u dv
99. ax dx
cos –1ax 1 3

∫ ∫
4(u – 1) du
–a
+
= = ( 4u 2 − 12u + 12 − 4/u) du

x u
√1 – (ax) 2 4
= u 3 − 6u 2 + 12 u − 4 ln |u| + C
3


ax dx 4
= x cos −1 ax + = (1 + 4 x )3 − 6(1 + 4 x )2 + 12(1 + 4 x )
1 − ( ax )2 3
− 4 ln 1 + 4 x + C

1
= x cos −1 ax − [1 − ( ax )2 ]−1/ 2 ( −2 a 2 x dx )
2a 4 4 3
Or: ( x ) − 2( 4 x )2 + 4 4 x − 4 ln |1 + 4 x | + C1 ,
3

Calculus Solutions Manual Problem Set 9-11 255


© 2005 Key Curriculum Press
by expanding the powers or by starting with 2 du
c. u = tan ( x/2) ⇒ 2 tan −1 u = x ⇒ dx =
u = 4 x. 1 + u2
Absolute values are optional because 1 + 4 x > 0. 1 – u2 2u
cos x = 2 and sin x = from part b.
1+ u 1 + u2

1
104. dx Let u = x 1/ 6 .
x+3x
∫ 1 + cos x dx
1
d.
x = u6
=∫
dx = 6u5 du 1 2 du

5 3 1– u 1+ u 2 2

∫ ∫
6u du 6u du 1+
= 2 =
u +u
3
u +1 1+ u 2

∫ ∫
 6  2 du

= 6u − 6u + 6 − = = du
2
du (1 + u 2 ) + (1 – u 2 )
 u + 1
(by long division)
∫ ∫
1
= 2u 3 − 3u 2 + 6u − 6 ln |u + 1| + C e. dx = du = u + C = tan ( x/2) + C
1 + cos x
= 2 x − 33 x + 66 x − 6 ln (6 x + 1) + C
∫ ∫
1
108. a. sec x dx = dx

1 cos x
105. dx Let u = e x + 1.
e +1 1 + u 2 2 du
∫ ∫
x 2
= ⋅ = du
ex = u2 − 1 1 – u2 1 + u2 1 – u2
x = ln (u 2 − 1)
b. sec x dx = 
1 
∫ ∫
1
+ du
2u du
dx = 2 1– u 1+ u
u −1 = −ln | 1 − u | + ln |1 + u| + C
 1 
∫ ∫
2 du 1
= = − du 1+ u 1 + tan ( x/2)
u2 – 1  u – 1 u + 1 = ln + C = ln +C
(by partial fractions) 1– u 1 – tan ( x/2)
= ln |u − 1| − ln |u + 1| + C 1 + tan ( x/2)
= ln ( e + 1 − 1) − ln ( e + 1 + 1) + C
x x c. ∫ sec x dx = ln 1 – 1 ⋅ tan ( x/2)
+C


1 tan (π /4) + tan ( x/2)
106. dx Let u = e x – 1. = ln +C
x
e –1 1 – tan (π /4) tan ( x/2)
ex = u2 + 1 = ln |tan (π /4 + x/2)| + C
x = ln (u 2 + 1) 1 1

dx = 2
2u du
u +1
d. i. ∫ 0
sec x dx = ln | tan (π /4 + x/2) |
0
= ln |tan (π/4 + 1/2)| − ln |tan π/4|

2 du
= = 2 tan u + C = 2 tan −1 e x – 1 + C
−1
u2 + 1 = ln |tan (π/4 + 1/2)| = 1.226191…
107. a. Let t = x/2 and substitute, getting 1 1

cos x = 2 cos2 (x/2) − 1 and ii. ∫ sec x dx = ln | sec x + tan x|


0 0
sin x = 2 sin (x/2) cos (x/2).
= ln |sec 1 + tan 1| − ln |1 + 0| =
2 ln |sec 1 + tan 1| = 1.226191… , which
b. cos x = −1
sec 2 ( x/2) agrees with the answer in part i.
2 – sec 2 ( x/2)
∫ ∫
= 1 1 2 du
109. dx = ⋅
sec 2 ( x/2) 1 – cos x 1 – u 1 + u2
2
1–
2 – [1 + tan 2 ( x/2)] 1 + u2
=
1 + tan 2 ( x/2)
∫ ∫
2 du du –1
= 2 = = +C
1 – tan 2 ( x/2) (1 + u ) – (1 – u )
2
u2 u
= , Q .E .D .
1 + tan 2 ( x/2) = −cot (x/2) + C
sin ( x/2)
sin x = 2
∫ ∫
cos 2 ( x/2) 1 1 2 du
cos ( x/2) 110. dx = ⋅
1 + sin x 1+
2 u 1 + u2
1 1+ u 2
= 2 tan ( x/2)
sec 2 ( x/2)
∫ ∫
2 du du –1
= = 2 = +C
=
2 tan ( x/2)
, Q .E .D . (1 + u ) + 2u
2
(u + 1) u +1
1 + tan 2 ( x/2) –1
= +C
tan ( x/2) + 1

256 Problem Set 9-11 Calculus Solutions Manual


© 2005 Key Curriculum Press
1 – u2
∫ x cos 2 x dx
3 u dv
R3. a.
1 + u 2 ⋅ 2 du x3
∫ ∫
cos x cos 2x
111. dx = +
1
1 – cos x 1 – u2 1 + u2 3x 2 2 sin 2x
1– –
1
1 + u2 6x + – 4 cos 2x
1
1 – u2
du =  2 −
2 
∫ ∫
1 6 – 8 sin 2x
= du –
u (1 + u 2 )
2 u 1 + u2  0 + 1
16 cos 2x
1
= − − 2 tan −1 u + C
u 1 3 3
= x sin 2 x + x 2 cos 2 x
1 2 4
=− − 2 tan −1[tan ( x/2)] + C 3 3
tan ( x/2) − x sin 2 x − cos 2 x + C
4 8
= −cot (x/2) − x + C
 
∫ 1 – cos x dx = ∫  −1 + 1 – cos x  dx ∫e
cos x 1 b. 4x
sin 3 x dx u dv
Or:
e 4x + sin 3x
1
4e 4x – 3 cos 3x
∫ ∫ 1 – cos x dx
1 –
= − dx + 4x 1
– 9 sin 3x
16e +

= −x − cot (x/2) + C (using Problem 109)


1 4
= – e 4x cos 3 x + e 4x sin 3 x
3 9


16 4x
Problem Set 9-12 – e sin 3 x dx
9
1. Answers will vary.

25 4x
⇒ e sin 3 x dx
9
1 4
Problem Set 9-13 = − e 4x cos 3 x + e 4x sin 3 x + C1
3 9
Review Problems ∫
⇒ e 4x sin 3 x dx
R0. Answers will vary. 3 4x 4
=− e cos 3 x + e 4x sin 3 x + C
R1. f ( x ) = x cos x ⇒ 25 25
f ′( x ) = x(−sin x) + (1) cos x = cos x − x sin x
⇒ x cos x dx + C = f ′( x ) dx ∫ ∫ x (ln x ) dx
2 u dv
c.
(ln x)2 + x

1 1 2
= (cos x − x sin x ) dx 2 ln x · x 2x
------------------------


ln x x
= sin x − x sin x dx 1 – 1 2
x 2x


------------------------
⇒ x sin x dx = sin x − x cos x + C 1
1
+ 2x
4 4 1 2


0 –
4x
x sin x dx = sin x − x cos x
1 1
= sin 4 − 4 cos 4 − sin 1 + cos 1 = 1.5566… =
1 2 1 1
x (ln x )2 − x 2 ln x + x 2 + C
4


2 2 4
Numerically, x sin x dx ≈ 1.5566 K .
1 d. Slice parallel to the y-axis. Pick a sample
R2. ∫ 5x sin 2 x dx u = 5x dv = sin 2x dx point (x, y) on the graph, within the slice.
dV = 2π x · y · dx = 2π x(x ln x) dx
1 = 2π x 2 ln x dx
du = 5 dx v = − cos 2 x
2
= 5 x  − cos 2 x  + ∫
1 1 2

 2  2
cos 2 x (5 dx ) V = 2π ∫
1
x 2 ln x dx u
ln x +
dv
x2
5 5 1 1 3
= − x cos 2 x + sin 2 x + C x
----------------------
3x
2 4 1 2
1 – 3x
+ 1 3
0 9x

Calculus Solutions Manual Problem Set 9-13 257


© 2005 Key Curriculum Press
∫ sec
2
= 2π  x 3 ln x − x 3 
1 1 3 u dv
d. x dx
3 9  1 secx + sec 2 x
secx tanx – tanx
16 16 2 16 14
= π ln 2 − π + π = π ln 2 − π
3 9 9 3 9
= 6.7268K ∫
= sec x tan x − tan 2 x sec x dx

= sec x tan x − ∫ (sec − 1) sec x dx


2

∫ cos
30 u dv
R4. a. dx
cos 29x cosx
= sec x tan x + ∫ sec x dx − ∫ sec x dx
+ 3
– 29 cos 28x sinx – sinx
2 ∫ sec x dx 3


= cos x sin x + 29 cos x sin x dx
29 28 2
= sec x tan x + ln |sec x + tan x | + C
x sin x + 29 ∫ cos
= cos x (1 − cos x ) dx
∫ sec
29 28 2 3
x dx


⇒ 30 cos dx 30
1 1
= sec x tan x + ln |sec x + tan x | + C
2 2
= cos 29 x sin x + 29 cos 28 x dx ∫
∫ tan ∫
32 dx = ( tan 9 32) dx = ( tan 9 32) x + C
9
e.

⇒ cos dx 30

f. r = 9 + 8 sin θ

1 29
=cos 29 x sin x + cos 28 x dx 1 1
30 30 dA = r 2 dθ = (9 + 8 sin θ )2 dθ
2 2
∫ ∫
1 4
b. sec 6 x dx = sec 4 x tan x + sec 4 x dx 1 π /4
1
5
4
5 A=
2 0 ∫
(64 sin 2 θ + 144 sin θ + 81) dθ
= sec 4 x tan x + sec 2 x tan x 1 π /4
5 15 =
2 0 ∫
[32(1 − cos 2θ ) + 144 sin θ + 81] dθ

8
+ 2
sec x dx
81 π /4
= 16 θ − sin 2θ  − 72 cos θ + θ
15 1
1 4
= sec 4 x tan x + sec 2 x tan x  2  2 0
5 15 81
8 = 4π − 8 − 36 2 + π + 72
+ tan x + C 8
15 113
= π + 64 − 36 2 = 57.4633K
∫ tan x dx = ∫ tan x (tan x dx )
n n−2 2
c. 8

= ∫ tan x (sec x − 1) dx
n−2 2
R6. a. ∫ x 2 − 49 dx

= ∫ tan x sec x dx − ∫ tan x dx


n−2 2 n−2
v

tan x − ∫ tan x dx
1 n −1 n−2
=
n −1
x
√x 2 – 49

∫ cos x dx = ∫ (1 − sin x ) (cos x dx )


5 2 2 θ u
R5. a.
7

= ∫ (1 − 2 sin x + sin x )(cos x dx )


2 4

x
2 1
= sin x − sin 3 x + sin 5 x + C Let = sec θ . x = 7 sec θ ,
3 5 7
dx = 7 sec θ tan θ dθ,
∫ sec x dx = ∫ (tan x + 1) (sec x dx )
6 2 2 2
b. x
x 2 − 49 = 7 tan θ , θ = sec −1
= ∫ (tan x + 2 tan x + 1)(sec x dx )
4 2 2 7
∴ ∫ x − 49 dx
2

1 2
= tan 5 x + tan 3 x + tan x + C
5 3 = ∫ (7 tan θ )(7 sec θ tan θ dθ )

∫ ∫ = 49 ∫ sec θ tan θ dθ
1
c. sin 2 7 x dx = (1 − cos 14 x ) dx 2
2
1 1
= x − sin 14 x + C = 49  ∫ sec θ dθ − ∫ sec θ dθ 
3

2 28

258 Problem Set 9-13 Calculus Solutions Manual


© 2005 Key Curriculum Press
= 49  sec θ tan θ + ln |sec θ + tan θ |
2
1 1
2
c. ∫ 1 – 0.25 x 2 dx
v

− ln |sec θ + tan θ | + C1
 1
0.5x
49 49
= sec θ tan θ − ln |sec θ + tan θ | + C1 θ u
2 2
√ 1 – 0.25x 2
49 x x 2 − 49
= ⋅ ⋅ 0.5 x
2 7 7 Let = sin θ .
1
49 x x 2 − 49 x = 2 sin θ, dx = 2 cos θ dθ,
− ln + + C1
2 7 7 x
1 – 0.25 x 2 = cos θ , θ = sin −1
2
1 49
=
2
x x 2 − 49 −
2
ln x + x 2 − 49
∫ ∫
1 – 0.25 x 2 dx = (cos θ )(2 cos θ dθ )
49

= 2 cos θ dθ = (1 + cos 2θ ) dθ ∫
2
+ ln 7 + C1
2
1
1 49 = θ + sin 2θ + C = θ + sin θ cos θ + C
= x x 2 − 49 − ln x + x 2 − 49 + C 2
2 2 −1 x 1
= sin + x 1 − 0.25 x 2 + C
∫ ∫
b. x 2 − 10 x + 34 dx = ( x − 5)2 + 9 dx 2 2
d. Slice region vertically. Pick sample point
v
(x, y) on the upper branch of the circle,
within the strip.
√ (x – 5)2 + 9
dA = 2 y dx = 2 25 – x 2 dx
x–5
v
θ u
3
5
x

x−5 θ u
Let = tan θ .
3 √ 25 – x 2
x = 5 + 3 tan θ, dx = 3 sec2 θ dθ,
x−5 x
( x − 5)2 + 9 = 3 sec θ , θ = tan −1 Let = sin θ . x = 5 sin θ , dx = 5 cos θ dθ ,
3 5
x
∴ ∫ ( x – 5) + 9 dx 25 – x 2 = 5 cos θ , θ = sin −1
2
5
4
= ∫ (3 sec θ )(3 sec θ dθ ) = 9 ∫ sec θ dθ
2 3
A= ∫2 25 − x 2 dx
3
x =4
= 2∫
9 9
= sec θ tan θ + ln |sec θ + tan θ | + C1 5 cos θ (5 cos θ dθ )
2 2 x =3
x =4
9 ( x − 5)2 + 9 x − 5 = 25∫ (1 + cos 2θ ) dθ
= x =3
2 3 3 x =4
= 25θ + 12.5 sin 2θ x =3
9 ( x − 5) + 9 x − 5
2
x =4
+ ln + + C1 = 25θ + 25 sin θ cos θ x =3
2 3 3 4
x x 1
1 = 25 sin −1
+ 25 ⋅ ⋅ 25 − x 2
= ( x − 5)2 + 9 ⋅ ( x − 5) 5 5 5 3
2
9 9 = 25 sin −1 0.8 + 4 9 − 25 sin −1 0.6 − 3 16
+ ln ( x − 5)2 + 9 + x − 5 − ln 3 + C1 = 25(sin− 1 0.8 − sin− 1 0.6) = 7.0948…
2 2
(6 x + 1) dx (6 x + 1) dx
1
= ( x – 5) x – 10 x + 34
2
2 R7. a. ∫
x 2 – 3x – 4
=
( x + 1)( x – 4) ∫
= 
5 

1
9
+ ln x 2 – 10 x + 34 + x − 5 + C + dx
 x +1 x − 4
2
= ln |x + 1| + 5 ln |x − 4| + C

Calculus Solutions Manual Problem Set 9-13 259


© 2005 Key Curriculum Press
5 x 2 – 21x – 2 R8. a.
b. ∫ ( x – 1)( x + 2)( x – 3)
dx
y

= 
2 

3 4
+ − dx
 x – 1 x + 2 x – 3
= 3 ln |x − 1| + 4 ln |x + 2| − 2 ln |x − 3| + C
2

x
5 x 2 + 3 x + 45 5 x 2 + 3 x + 45
∫ dx = ∫
1
c. dx
x3 + 9x x ( x 2 + 9)
b. f ( x ) = sec− 1 3x
=  + 2
3 

5 x
dx = 5 ln | x | + tan −1 + C f ′( x ) =
3
=
1
 x x + 9 3
|3 x | (3 x ) – 1 | x | 9 x 2 – 1
2

(The second integral may be found by


inspection or by trigonometric substitution.)
∫ tan
−1 u dv
c. 5x dx
5 x + 27 x + 32
2
tan –1 5x 1

+
d. dx 5
x ( x + 4) 2 –
x
1 + 25x 2
2 1 

3
=  + −  dx
 x x + 4 ( x + 4) 2 
∫ 1 + 25x dx
5x
= x tan −1 5 x − 2
= 2 ln | x | + 3 ln | x + 4 | + (x + 4)− 1 + C
5x − ∫
1 1
1 = x tan −1 (50 x dx )
= ln | x 2 ( x + 4)3 | + +C 10 1 + 25 x 2
x+4 1
= x tan −1 5 x −ln |1 + 25 x 2 | + C
dy 10
e. = 0.1( y – 3)( y – 8) (Absolute values are optional because
dx
1 + 25x 2 > 0.)
∫ ∫
dy
= 0.1 dx d. “Obvious” way: Slice the region vertically.
( y – 3)( y – 8)
Pick a sample point (x, y) on the graph,
 −1/5 1/5 
∫  y − 3 + y − 8  dy = ∫ 0.1 dx within the strip.
dA = y dx = cos− 1 x dx
1 1
1 1
− ln | y − 3 | + ln | y − 8 | = 0.1x + C1
5 5
A= ∫ 0
cos −1 x dx = x cos −1 x − 1 − x 2
0
−1
−ln | y − 3 | + ln | y − 8 | = 0.5x + C = cos 1 − 0 − 0 + 1 = 1
Substituting (0, 7) gives Easier way: Slice horizontally. Pick a sample
C = –ln 4 + ln 1 = –ln 4. point (x, y) on the graph within the strip.
dA = x dy = cos y dy
y–8 π /2 π /2
= 0.5 x − ln 4

ln
y−3 A= cos y dy = sin y = 1− 0 = 1
0 0
y–8
= e 0.5 x −ln 4 = 0.25e 0.5 x R9. a.
y−3 y

y–8
− = 0.25e 0.5 x 1
y–3 x
((y – 8)/(y – 3) < 0 because (0, 7) is on the 1
graph)
5
y = 3+
1 + 0.25e 0.5 x
The graph shows that solution fits slope field. b.
y
y

1
7 x
1

260 Problem Set 9-13 Calculus Solutions Manual


© 2005 Key Curriculum Press
0 0

∫ ∫ tan x dx
c. h(x) = x2 sech x b. tan x dx = lim −
h′(x) = –x2 sech x tanh x + 2x sech x π /2 a→π /2 a

d. f ( x ) = sinh− 1 5x 0

5 = lim − ln |sec x |
f ′( x ) = a→π /2 a
25 x 2 + 1 = lim − (ln |sec 0 | − ln |sec a |) = −∞
a→π /2

∫ ∫
1
e. tanh 3 x dx = sinh 3 x dx The integral diverges.
cosh 3 x 1
1
= ln |cosh 3 x | + C
3
c. ∫−1
x −2/3 dx
b 1

∫ x −2/3 dx + lim+ ∫x
−2/3
(Absolute values are optional because = lim− dx
cosh 3x > 0.) b→ 0 −1 a→ 0 a
b 1
= lim− 3 x 1/3 + lim+ 3 x 1/3
∫ cosh
−1 u dv
f. 7x dx b→ 0 −1 a→ 0 a
cosh –1 7x 1
7
+
= lim− [3b − ( −3)] + lim+ (3 − 3a1/3 ) = 6
1/3

x b→0 a→0
√ 49x 2 – 1 The integral converges to 6.
4
 x − | x − 1|  dx
= x cosh 7 x − −1

7x
dx
d. ∫
0 x −1 
49 x 2 − 1 b 4
= lim− ∫ ( x + 1) dx + lim+ ∫( x − 1) dx

1 b→1 a→1
= x cosh −1 7 x − ( 49 x 2 − 1) −1/2 (98 x dx ) 0
b
a
4
= lim−  x 3/2 + x  + lim+  x 3/2 − x 
14 2 2
1 b→1  3   
= x cosh −1 7 x − ⋅ 2( 49 x 2 − 1)1/2 + C 0 a→1 3 a
14
= lim−  b 3/ 2 + b − 0
1 2
= x cosh −1 7 x − 49 x 2 − 1 + C b→1  3 
7
+ lim+  ⋅ 4 3/2 − 4 − a 3/2 + a
2 2
g. cosh2 x – sinh2 x
a→1  3 3 
1 1
= (e x + e – x ) 2 − (e x − e – x ) 2 2 3/2 2 3/2 2 3/2
2 2 = ⋅1 +1 + ⋅ 4 − 4 − ⋅1 +1
1 2x 1 3 3 3
= (e + 2 + e ) − (e 2 x − 2 + e –2 x )
–2 x
16 10
4 4 = 1+ − 4 +1 =
= 1, Q .E .D . 3 3
10
1 The integral converges to = 3.333K .
h. The general equation is y = k cosh x + C. 3
k ∞
y = 5 at x = 0 ⇒ 5 = k cosh 0 + C
⇒C=5–k
e. ∫1
x − p dx converges if p > 1 and diverges

3 otherwise.
y = 7 at x = 3 ⇒ 7 = k cosh + 5 − k x
k R11. a. f ( x ) = x sin −1 x ⇒ f ′( x ) = sin −1 x +
3 1− x2
⇒ 2 = k cosh − k
k
⇒ k = 2.5269… (solving numerically)
y = 2.5269K cosh
t
+ 5 − 2.5269K

b. I = x sin −1 x dx u
sin –1 x
dv
x
2.5269K 1
+
1 2
y(10) = 68.5961… 20 –
2x
√1 – x2
x
= 2.5269K cosh + 5 − 2.5269K
2.5269K x 2 dx

1 2 −1 1
⇒ x = ±6.6324… (solving numerically) = x sin x −
∞ b 2 2 1 – x2
R10. a. ∫3
( x − 2) −1.2 dx = lim
b→∞ ∫3
( x − 2) −1.2 dx
x 2 dx
= lim − 5( x − 2) −0.2
b Let I1 = ∫ 1 – x2
and x = sin θ.
b→∞ 3
= lim [–5(b – 2) –0.2 + 5] = 5 ∴ dx = cos θ dθ , 1 − x 2 = cos θ ,
b→∞
The integral converges to 5. θ = sin− 1 x

Calculus Solutions Manual Problem Set 9-13 261


© 2005 Key Curriculum Press
I1 =
sin 2 θ cos θ dθ
∫ cos θ ∫ = sin 2 θ dθ ∫
For (9 − x 2 ) −1/2 dx, transforming the dx to the
differential of the inside function, −2x dx,

1 1 1
= (1 − cos 2θ ) dθ = θ − sin 2θ + C requires multiplying by a variable. Because the
2 2 4 integral of a product does not equal the product of
1 1
= θ − sin θ cos θ + C the two integrals, you can’t divide on the outside
2 2 of the integral by −2x. So a more sophisticated
1 −1 1
= sin x − x 1 − x 2 + C technique must be used, in this case,
2 2 trigonometric substitution. As a result, an
1 2 −1 1 1
∴ I = x sin x − sin −1 x + x 1 − x 2 + C inverse sine appears in the answer:
2 4 4

x
d (9 − x 2 ) −1/2 dx = sin −1 + C
c. tanh e = e sech e
x x 2 x
3
dx

∫ ∫
1
d. ( x 3 − x ) −1 dx = 3 dx Concept Problems
x −x
= ∫
1
x ( x – 1)( x + 1)
dx
C1. ∫ sech x dx = ∫ 1 − tanh 2 x dx
v

= − +
1/2 

1 1/2
+ dx
 x x − 1 x + 1 1
tanh x
1 1
= − ln | x | + ln | x − 1| + ln | x + 1| + C θ u
2 2
√1– tanh2x
e. f ( x ) = (1 − x 2)1/2
1
f ′( x ) = (1 − x 2 ) −1/2 ( −2 x ) = − x (1 − x 2 ) −1/2 Let tanh x = sin θ.
2 ∴ x = tanh− 1 (sin θ) and θ = sin− 1 (tanh x)


1 1
f. I = (1 − x 2 )1/2 dx dx = 2 ⋅ cos θ dθ = dθ
1 − sin θ cos θ
Let x = sin θ .
1 – tanh 2 x = 1 – sin 2 θ = cos θ
∴ dx = cos θ dθ , (1 − x 2 )1/2 = cos θ ,
∫ ∫ ∫
1
θ = sin− 1 x ∴ sech x dx = cos θ ⋅ dθ = dθ
cos θ
∫ ∫
∴ I = cos θ ⋅ cos θ dθ = cos 2 θ dθ
= θ + C = sin −1 (tanh x ) + C, Q .E.D .


1 1 1
(1 + cos 2θ ) dθ = θ + sin 2θ + C
1
= 1
2
1 1
2 4 ∫0
sech x dx = sin −1 (tanh x )
0
= θ + sin θ cos θ + C = sin− 1 (tanh 1) − sin− 1 (tanh 0)
2 2
1 −1 1 = sin− 1 (tanh 1) = 0.86576948…
= sin x + x 1 − x 2 + C Numerical integration gives 0.86576948… ,
2 2
which agrees with the exact answer.
1
g. g( x ) = (ln x )2 ⇒ g′( x ) = 2 ln x ⋅ C2. From sinh 2A = 2 sinh A cosh A,
x
let A = x/2, so
sinh x = 2 sinh (x/2) cosh (x/2) ⇒ csch x
h. ∫ x ln x dx u
ln x +
dv
x =
1
=
1
1
x
1 2
2x
sinh x 2 ⋅ sinh ( x/2) ⋅ cosh ( x/2)
-------------------- 1 sech 2 ( x/2)
1
1 – 2x csch x = ⋅
+ 1 2 2 ⋅ sinh ( x/2) ⋅ cosh ( x/2) sech 2 ( x/2)
0 4x 1
sech 2 ( x/2)
sech 2 ( x/2) 2
1 2 1 = =
= x ln x − x 2 + C 2 ⋅ tanh ( x/2) tanh ( x/2)
2 4
∫ ∫ ⋅  sech 2 ( x/2) dx 
1 1
∴ csch x dx =

R12. For (9 − x 2 ) −1/2 x dx, the x dx can be tanh ( x/2)  2 
transformed to the differential of the inside = ln | tanh (x/2) | + C, Q .E .D .
2 2
function by multiplying by a constant,
∫ csch x dx = ln |tanh ( x/2)|

1
(9 − x 2 ) −1/2 ( −2 x dx ) = −(9 − x 2 )1/2 + C,
1
− 1
2 = ln |tanh 1| – ln |tanh(1/2) | = 0.49959536 K
and thus has no inverse sine. Numerical integration gives 0.49959536… .

262 Problem Set 9-13 Calculus Solutions Manual


© 2005 Key Curriculum Press
C3. From sin 2A = 2 sin A cos A, let A = x/2, so Chapter Test
sin x = 2 sin (x/2) cos (x/2)
∫ sin
1 6
1 1 T1. 5
x cos x dx = sin x + C
csc x = = 6
sin x 2 sin ( x/2) cos ( x/2)
1 sec 2 ( x/2)
∫x
3 u dv
= ⋅ T2. sinh 6 x dx
2 sin ( x/2) cos ( x/2) sec 2 ( x/2) x3 + sinh 6x
1
1 3x 2 – 6 cosh 6x
sec 2 ( x/2) 1
sec 2 ( x/2) 2 6x 36 sinh 6x
= = +
1
2 tan ( x/2) tan ( x/2) 6 – 216 cosh 6x

∫ ∫ ⋅  sec 2 ( x/2) dx 
1 1 1
∴ csc x dx =
+
0 1296 sinh 6x
tan ( x/2)  2 
= ln | tan (x/2) | + C, Q .E .D . 1 3 1
Or: = x cosh 6 x − x 2 sinh 6 x
6 12
Let u = tan (x/2), as in Problem 107 of Problem 1 1
Set 9-11. + x cosh 6 x − sinh 6 x + C
36 216
2 du 1 + u2
Then dx = 2 and csc x =
1+ u
∫ cos
2u T3. −1
x dx u dv
1 + u 2 2 du cos –1x
∫ ∫
1
∴ csc x dx = ⋅ +
2u 1 + u 2 –
1 –
x
√ x2

= (1/u) du = ln | u | + C = ln |tan ( x/2)| + C, 1 –

Q .E .D .

x
Confirmation: = x cos −1 x + dx
1 1 1 – x2
∫0.5
csc x dx = ln |tan ( x/2)|
0.5
= x cos −1 x −
1
2 ∫
(1 − x 2 ) −1/2 ( −2 x dx )
1 1 1
= ln tan − ln tan = 0.7605K = x cos −1 x − (2)(1 − x 2 )1/2 + C
2 4 2
Numerical integration gives 0.7605… . −1
= x cos x − 1 − x 2 + C
2 sin ( x/2) cos ( x/2)
Note that tan ( x/2) =
∫ sec x dx
3
2 cos 2 ( x/2) T4.
sin x 1 1 1
= = , so = sec x tan x + ln |sec x + tan x | + C
1 + cos x csc x + cot x 2 2
ln |tan ( x/2)| = − ln |csc x + cot x | .
∫e
∞ 2x u dv
T5. cos 5 x dx

1
C4. A = 2 dx e 2x cos 5x
−∞ 1 + x +
1
0 b 2e 2x 5 sin 5x
∫ ∫ 1+ x
1 1
= lim 2 dx + lim dx –
1
a→−∞ a 1 + x b→∞ 0
2
4e 2x + – 25 cos 5x
= lim tan −1 x + lim tan −1 x
0 b
a→−∞ a b→∞ 0
1 2x 2
= e sin 5 x + e 2 x cos 5 x
= lim (0 − tan −1 a) + lim (tan −1 b − 0) 5 25
a→−∞ b→∞


4
= −(− π /2) + (π /2) = π − e 2 x cos 5 x dx
25
C5. Prove that f (x) = ln x is unbounded above.

29 2 x
Proof: e cos 5 x dx
25
Assume f (x) = ln x is not unbounded above. 1 2
Then there is a number M > 0 such that = e 2 x sin 5 x + e 2 x cos 5 x + C
5 25
ln x < M for all x > 0.
Let x = eM+ 1. ∫e
2x
cos 5 x dx
Then ln x = ln eM+ 1 = M + 1.
5 2x 2
∴ ln x > M, which is a contradiction. = e sin 5 x + e 2 x cos 5 x + C
∴ the assumption is false, and ln x is unbounded 29 29
above, Q.E.D.

Calculus Solutions Manual Problem Set 9-13 263


© 2005 Key Curriculum Press
T6. ∫ ln 3x dx u
ln 3x +
dv
1
= ln ( x − 3)2 − 4 + C
1/x – x 1
= ln | x 2 − 6 x + 5 | + C
2

= x ln 3 x − dx = x ln 3 x − x + C x −3
dx = 
1/2 
∫x ∫
1/2
ii. + dx
T7. f (x) = sech (e ) ⇒
3 5x − 6x + 5
2  x − 1 x − 5
f ′(x) = 3 sech2 (e5x) ⋅ [−sech (e5x) 1 1
= ln | x – 1| + ln | x – 5 | + C
tanh (e5x)] ⋅ 5e5x 2 2
= −15e5x sech3 (e5x) tanh (e5x) 1
1 = ln | ( x – 1)( x – 5) | + C
T8. g( x ) = sin −1 x ⇒ g ′( x ) = 2
1 – x2 1
= ln | x 2 – 6 x + 5 | + C,
T9. f (x) = tanh− 1 x ⇒ tanh f (x) = x, |x| ≤ 1 2
sech2 f(x) ⋅ f ′(x) = 1 which agrees with part a.
[1 − tanh2 f (x)] ⋅ f ′(x) = 1
∫x
x–3
(1 – x2) ⋅ f ′(x) = 1 iii. dx
– 6x + 5
2
1
f ′( x ) = , |x | < 1

1 1
1 – x2 = ⋅ (2 x − 6) dx
1 1 2 x2 – 6x + 5
f ′(0.6) = = = 1.5625 1
1 – 0.36 0.64 = ln | x 2 – 6 x + 5| + C, as in parts a and b.
Numerically, f ′(0.6) ≈ 1.5625… (depending on 2
the tolerance of the calculator). b. See parts i, ii, and iii.
1
∫ cos ∫ 2 (1 + cos 2 x ) dx
T10. General equation is y = k cosh x + C. 1
T12. 2
x dx =
k
y = 1 at x = 0 ⇒ 1 = k cosh 0 + C ⇒ 1 1
C=1−k = x + sin 2 x + C
2 4
5
y = 3 at x = 5 ⇒ 3 = k cosh + 1 – k
∫ cos x dx = ∫ (1 – sin x ) cos x dx
5 2 2
k T13. a. i.
Solving numerically, k ≈ 6.5586… .
1 = ∫ (1 – 2 sin x + sin x ) cos x dx
2 4
y = 6.5586 K cosh x + 1 − 6.5586 K
6.5586 K
2 3 1
= sin x – sin x + sin 5 x + C
∫ ∫
x–3 x–3
T11. a. i. I = dx = dx 3 5
x – 6x + 5
2
( x – 3)2 – 4
∫ ∫
1 4
ii. cos 5 x dx = cos 4 x sin x + cos3 x dx
v 5 5
1 4
= cos 4 x sin x + cos 2 x sin x
x–3 5 15
√ (x – 3)2 – 4

8
+ cos x dx
θ u 15
2 1 4
= cos 4 x sin x + cos 2 x sin x
5 15
8
x–3 + sin x + C
Let = sec θ . x – 3 = 2 sec θ , 15
2 1 4 8
dx = 2 sec θ tan θ dθ, b. cos x sin x + cos 2 x sin x + sin x
4
5 15 15
x–3
( x – 3)2 – 4 = 2 tan θ , θ = sec –1 1
= (1 – sin 2 x )2 sin x
2 5
(2 sec θ )(2 sec θ tan θ dθ )
∴ I= ∫ 4 tan 2 θ
4 8
+ (1 – sin 2 x ) sin x + sin x
15 15
1 2 3 1 5
∫ tan θ ⋅ sec θ dθ = ln |tan θ | + C
1 = sin x – sin x + sin x
= 2
1 5 5 5
1 4 4 8
= ln ( x − 3)2 − 4 + C1 + sin x – sin 3 x + sin x
2 15 15 15

264 Problem Set 9-13 Calculus Solutions Manual


© 2005 Key Curriculum Press
b
=  + +  sin x −  +  sin 3 x
1 4 8 2 4 = lim (–10 xe –0.1x – 100e –0.1x )
 5 15 15   5 15  b→∞ 0

1 = lim (–10 be –0.1b


– 100e –0.1b
+ 0 + 100)
+ sin 5 x b→∞
5 10 b + 100
= lim  – + 100
2 1 b→∞  e 0.1b 
= sin x – sin 3 x + sin 5 x
= lim  – 
3 5 10
0.1b + 100 (by l’Hospital’s rule)
∞ b→∞  0.1e
T14. ∫0
xe −0.1x dx u
x +
dv
e –0.1x
= 100
1 – –10e –0.1x T15. Answers will vary.
0 – 100e –0.1x

Calculus Solutions Manual Problem Set 9-13 265


© 2005 Key Curriculum Press
Chapter 10—The Calculus of Motion—Averages,
Extremes, and Vectors

Problem Set 10-1 6


c. Displacement = ∫ (t – 10t + 16) dt = −12 ft
2

1. v(t) = 100(0.8) − 30 = 100e t


− 30 = 0 t ln 0.8 0
6

∫ |t
ln 0.3 1
⇒ e t ln 0.8
= 0.3 ⇒ t = = 5.3955K Distance = 2
− 10t + 16 | dt = 41 ft
ln 0.8 0 3
v becomes negative after t0 ≈ 5.40 min. 2 
+ −26  = −12 ft
2
d. Displacement = 14
t0 t0
3  3
2. sup = ∫ 0
v dt = ∫ 0
(100e t ln 0.8 – 30) dt
2 2 1
= 151.8341… (numerically) ≈ 151.8 ft Distance = 14 + 26 = 41 ft
3 3 3
10 10
sdown = − ∫t0
v dt = − ∫t0
(100e t ln 0.8 – 30) dt e. a(t) = v′(t) = 2t − 10
a(3) = 2(3) − 10 = −4 (ft/s)/s
= 51.8110… (numerically) ≈ 51.8 ft
Distance = sup + sdown = 203.6452… ≈ 203.6 ft 2. a. v(t) = tan 0.2t on [10, 20]
v(t) = 0 ⇔ t = … 0, 5π , 10π , … = 5π in
3. Displacement = sup − sdown = 100.0231… ≈
[10, 20]
100.0 ft
v(t) is infinite ⇔ t = … 2.5π , 7.5π , … ,
The displacement is positive, so Calvin is
none of which is in [10, 20].
upstream of his starting point.
10
v(t) < 0 for t in [10, 5π ). v(t) > 0 for t in
4. Displacement = ∫ 0
(100e t ln 0.8 – 30) dt (5π, 20].

= 100.0231… (numerically) ≈ 100.0 ft
10 10
b. For [10, 5π), displacement = ∫10
tan 0.2t dt
5. Distance = ∫ |v| dt = ∫ − 30 | dt = 5 ln | sec π | − 5 ln | sec 2 | = −4.3835…
t ln 0.8
|100e
0 0
Distance = 4.3835… ≈ 4.38 cm
= 203.6452… (numerically) ≈ 203.6 ft 20
For (5π, 20], displacement = ∫5π
tan 0.2t dt
Problem Set 10-2 = 5 ln |sec 4| − 5 ln | sec π | = 2.1259…
Distance = 2.1259… ≈ 2.13 cm
Q1. 120 mi Q2. 25 mi/h
Q4. f ′(x) = 1/x
20
Q3. 1.25 h
Q5. x ln x − x + C Q6. f ′(t) = sec2 t
c. Displacement = ∫
10
tan 0.2t dt = −2.2576 K ≈

1 3 −2.26 cm
Q7. g′(t) = sech2 t Q8. x +C 20


3
Distance = |tan 0.2t | dt = 6.5095K ≈
1 x 10
Q9. 2 +C Q10. ln 2 e x ln 2 = 2 x ln 2 6.51 cm
ln 2
1. a. v(t) = t2 − 10t + 16 on [0, 6] d. Displacement = −4.3835… + 2.1259… =
v(t) = (t − 2)(t − 8) = 0 ⇔ t = 2 or 8 s −2.2576… ≈ −2.26 cm
v(t) > 0 for t in [0, 2). v(t) < 0 for t in (2, 6]. Distance = −(−4.3835…) + 2.1259… =
6.5095… ≈ 6.51 cm
b. For [0, 2), displacement
2 e. a( t) = v′(t) = 0.2 sec2 t

2
= (t 2 – 10t + 16) dt = 14 a( 15) = 0.2 sec2 3 = 0.2040… ≈ 0.20 (cm/s)/s
0 3
π
Distance = 14 ft
2 3. a. v(t ) = sec t − 2 on [1, 11]
3 24
For (2, 6], displacement v(t) = 0 when
π
6 cos t = 0.5 ⇔ t = 8 in [1, 11]

2
= (t 2 – 10t + 16) dt = −26 24
2 3
v(t) < 0 for t in [1, 8). v(t) > 0 for t in
2
Distance = 26 ft (8, 11].
3

266 Problem Set 10-2 Calculus Solutions Manual


© 2005 Key Curriculum Press
8
 π  3 2 11
b. For [1, 8), displacement = ∫  sec 24 t – 2 dt
1
d. Displacement = −6
4
+ 24 = 17 mi
3 12
π π
Distance = − −6  + 24 = 31 mi
24 3 2 5
= ln sec + tan − 16
π 3 3  4 3 12
24 π π e. a( t) = v′(t) = 3t2 − 10t + 8
− ln sec + tan +2 a(2.5) = 1.75 (mi/min)/min
π 24 24
= −4.9420… 5. a(t ) = t 1/2, v(0) = −18, on [0, 16]
Distance ≈ 4.94 km

2
v(t ) = t 1/2 dt = t 3/2 + C; v(0) = −18 ⇒ C = −18
For (8, 11], displacement 3
π 2 3/2
=  sec t – 2 dt v(t ) = t − 18
11

8∫  24  3
16 2
 t 3/2 − 18 dt = −14 14 ft
=
24 11 11
ln sec π + tan π − 22
Displacement =
0 3 ∫  15
π 24 24 16 2


7
Distance = t 3/2 − 18 dt = 179 ft
24 π π 0 3 15
− ln sec + tan + 16
π 3 3 6. a(t ) = t −1 , v(1) = 0, on [0.4, 1.6]
= 4.7569…
Distance ≈ 4.76 km ∫
v(t ) = t −1 dt = ln t + C (t > 0); v(1) = 0 ⇒ C = 0
π
c. Displacement =  sec t – 2 dt =
11

∫  
v(t ) = ln t
1.6


1 24
Displacement = ln t dt = −0.0814 …
−0.1850 K ≈ −0.19 km 0.4
11 π ≈ −0.081 cm
Distance = ∫ sec t − 2 dt = 9.6990 K ≈ 1.6


1 24 Distance = |ln t | dt = 0.3854 …
9.70 km 0.4

d. Displacement = −4.9420… + 4.7569… = ≈ 0.385 cm


−0.1850… ≈ −0.19 km 7. a(t ) = 6 sin t, v(0) = −9, on [0, π ]
Distance = −(−4.9420…) + 4.7569… =
9.6990… ≈ 9.70 km

v(t ) = 6 sin t dt = −6 cos t + C; v(0)
= −9 ⇒ C = −3
π π π v(t) = − 6 cos t − 3
e. a(t ) = v ′(t ) = sec t tan t
24 24 24 π
a( 6) = 0.1851… ≈ 0.19 (km/h)/h Displacement = ∫ 0
(–6 cos t – 3) dt =
 exactly π 2  −9.4247… ≈ −9.42 km (exact: −3π km)
 24  π

4. a. v(t) = t3 − 5t2 + 8t − 6 on [0, 5]


Distance = ∫0
|− 6 cos t − 3| dt = 13.5338…

v(t) = (t − 3)(t2 − 2t + 2) = 0 ⇔ t = 3 in [0, 5] ≈ 13.53 km (exact: 6 3 + π )


v < 0 for t in [0, 3). v > 0 for t in (3, 5].
8. a(t) = sinh t, v(0) = −2, on [0, 5]
b. For [0, 3), displacement =
3

v(t ) = sinh t dt = cosh t + C

3
(t 3 – 5t 2 + 8t – 6) dt = −6
0 4 v(0) = −2 ⇒ C = −3
3 v(t) = cosh t − 3
Distance = 6 mi 5
4
For (3, 5], displacement =
Displacement = ∫ (cosh t – 3) dt =
0
5 59.2032… ≈ 59.20 mi (exact: sinh 5 − 15)

2
(t 3 – 5t 2 + 8t – 6) dt = 24 5
3
2
3 Distance = ∫ |cosh t − 3| dt
0
Distance = 24 mi = 64.1230 K ≈ 64.12 mi
3
c. Displacement = 9. a. v = t − 2 = 0 ⇔ t = 4 s;
1/2

5 v < 0 if t < 4, v > 0 if t > 4



11
(t 3 – 5t 2 + 8t – 6) dt = 17 mi 9

∫ (t
0 12 1
5 b. Displacement = 1/2
– 2) dt = 1 ft

5
Distance = | t − 5t + 8t − 6 | dt = 31 mi
3 2
9
1 3

∫ |t
0 12
c. Distance = 1/2
− 2| dt = 4 ft
1

Calculus Solutions Manual Problem Set 10-2 267


© 2005 Key Curriculum Press
1 9.8
1 1
10. a. v = sin 2t = 0 at t = … , − π , − π , 0, π , b. a = 0 at t = cos −1 =
2 2 0.015 40
3 88.2184 K ≈ 88.2 s
π, π, … v = 0 at t = 2,659.986…/9.8 = 271.4272… ≈
2
271.4 s
sin 2t ≥ 0 on 0, π , so
1
300
 2 
π /2
c. Displacement = ∫
0
v(t ) dt
Distance = ∫0
sin 2t dt = 1 cm
= ∫
100
 40 sin 0.015t – 9.8t  dt
4.5π  0.015 

0
b. Displacement = sin 2t dt = 1 cm 300

4.5π
0 + ∫ (–9.8t + 2659.986 K) dt

100
Distance = sin 2t dt = 9 cm
0 ≈ 116202.27… + 139997.32… ≈ 256,200 m
Or: The regions where the graph is below Distance = 116202.27… +
the x-axis cancel out the regions where the 300

graph is above the axis, leaving only one ∫


100
|− 9.8t + 2659.986 K | dt
uncancelled region above the graph, so = 116202.27… + 147998.09… ≈ 264,200 m
Displacement = area from part a = 1 cm. The The distance is greater than the displacement,
absolute values of the regions above and which agrees with the fact that the velocity
below the graph are the same, so Distance = becomes negative at t = 271.4… s.
9 times the area from part a = 9 cm.
d. v(300) = −9.8(300) + 2,659.986… =
11. a. v = 60 − 2t −280.0133… , so the rocket is moving
40
Displacement = ∫10
(60 – 2t ) dt = 300 ft
13. a.
downward (falling) at about 280 m/s.
40
b. Distance = ∫10
|60 − 2t | dt = 500 ft tend
s
aav
(mi/h)/s
vend
mi/h
vav
mi/h
send
mi
40 cos 0.015t – 9.8, t in [0, 100]
12. a. a(t ) =  0 — 0 — 0
–9.8, t > 100
5 2.95 14.75 7.375 0.0102…
For t in [0, 100],
10 3.8 33.75 24.25 0.0439…

v(t ) = ( 40 cos 0.015t − 9.8) dt 15 1.75 42.5 38.125 0.0968…
40 20 0.3 44 43.25 0.1569…
= sin 0.015t − 9.8t + C
0.015 25 0 44 44 0.2180…
v(0) = 0 ⇒ C = 0 30 0 44 44 0.2791…

For t > 100, v(t ) = −9.8 dt = −9.8t + C 35 0 44 44 0.3402…
40 40 −0.2 43 43.5 0.4006…
v(100) = sin 1.5 − 980 = 1679.986 …
0.015 45 −0.9 38.5 40.75 0.4572…
⇒ C = 1679.986… + 980 = 2659.986… 50 −2.6 25.5 32 0.5017…
 40 sin 0.015t − 9.8t, t in [0, 100] 55 −3.5 8 16.75 0.525
v(t ) =  0.015
60 −1.6 0 4 0.5305…
−9.8t + 2659.986 K , t > 100
a (t ) b. At t = 60, vend = 0, ∴ the train is at rest.
c. The train is just starting at t = 0; its
30

acceleration must be greater than zero to get it


moving, even though it is stopped at t = 0.
t
100
Acceleration and velocity are different
quantities; the velocity can be zero but
changing, which means the acceleration is
v (t ) nonzero.
d. Zero acceleration means the velocity is
1000
constant, but not necessarily zero.
e. The last entry in the last column is the
t displacement at time t = 60. Thus, it is
100
0.5305… ≈ 0.53 mi between stations.

268 Problem Set 10-2 Calculus Solutions Manual


© 2005 Key Curriculum Press
14. a. v (t )

10
tend aav v end vav send
s (mi/h)/s mi/h mi/h mi
0 — 6000 — 400 t
0 6 10
10 8.5 6085 6042.5 416.7847…
20 22 6305 6195 433.9930…
s(t )
30 33 6635 6470 451.9652…
40 39.5 7030 6832.5 470.9444…
50
50 42.5 7455 7242.5 491.0625…
60 53 7985 7720 512.5069… t
70 71 8695 8340 535.6736… 0 6 10

80 83.5 9530 9112.5 560.9861…


90 47.5 10005 9767.5 588.1180… b. The acceleration suddenly jumps from 0 to 2
at t = 0 and drops back to 0 at t = 6. (The
100 3 10035 10020 615.9513…
velocity graph has cusps in both places.)
π
b. According to these calculations, the spaceship c. a(t ) = 2 − 2 cos t
is only about 620 mi from the launchpad and 3
π
lim+ a(t ) = lim+  2 – 2 cos t 
moving at only about 10,000 mi/h. So the
specifications are definitely not met, and the t→0 t→0  3 
project should be sent back to the drawing = 2 − 2 cos 0 = 0
board.
π
lim a(t ) = lim–  2 – 2 cos t 

dv
15. a. a = ⇒ v = a dt = at + C; t →6 – t →6  3 
dt
v = v 0 when t = 0 ⇒ C = v 0 ⇒ v = v 0 + at = 2 − 2 cos 2π = 0
Because a(t) is continuous at t = 0 and 6,
∫ ∫
ds
b. v = ⇒ s = v dt = (v0 + at ) dt =
dt there are no sudden changes in acceleration.
1
v0 t + at 2 + C 2 – 2 cos π t, if 0 ≤ t ≤ 6
2
d. a(t ) =  3
s = s0 when t = 0 ⇒ C = s0 ⇒ 0, if t ≥ 6
1
s = v0 t + at 2 + s0
2 2t – 6 sin π t, if 0 ≤ t ≤ 6
16. Use s(t) for displacement. Assume v(0) = v(t ) =  π 3
s(0) = 0. 12, if t ≥ 6
2, if 0 ≤ t < 6 e.
a. a(t ) = 
0, if t ≥ 6 v (t )

2t, if 0 ≤ t < 6 10
v(t ) = 
12, if t > 6
t 2 , if 0 ≤ t < 6 t
s( t ) =  0 6 10
12 t – 36, if t > 6
−36 comes from the initial condition, There are no step discontinuities in a(t), and
s(6) = 36. thus the graph of v(t) is smooth.
6
 2t − 6 sin π t  dt =  t 2 + 18 cos π t 
6
a (t )
f. ∫
0  π 3   π2 3 0
2 18 18
= 36 + 2 − 0 − 2 = 36
π π
t
0 6 10
Elevator goes 36 ft.

Calculus Solutions Manual Problem Set 10-2 269


© 2005 Key Curriculum Press
g. The elevator will take another 36 ft to slow b. The rectangle has the same area as the shaded
down and stop. So the deceleration should region.
start where the elevator is 564 ft up, about
f (x )
the 47th floor (from part h, one floor = 12 ft).
h. The elevator takes a total of 12 s to accelerate
and decelerate. During these intervals it
50 y = 41
travels a total of 72 ft, leaving 528 ft for the
constant velocity portion. At 12 ft/s, this part x
1 5
of the trip will take 44 s. Thus, the total trip
takes 56 s.
c. 41 = c3 − c + 5
i. The elevator must start to decelerate halfway
c = 3.4028… , which is in [1, 5].
through the trip, where s(t) = 6 ft. Solving
1 9 1/2

1
b
 π  2. a. yav = ( x – x + 7) dx = 4
∫  2t – π sin 3 t dt = 6
6
8 1 6
0
b. The rectangle has the same area as the shaded
numerically for b gives b ≈ 3.1043… ≈ region.
3.1 s.
a(3.1043…) = 3.9880… ≈ 4.0 ft/s2 f (x )

By symmetry, the deceleration process must


5 y = 4.1666...
start at this time, meaning the acceleration
jumps to −3.9880… ft/s2. The graph looks
like this: x

1 9
5 a (t )

1
c. 4 = c1/2 − c + 7
t 6
0 6.2 c = 5.0892… , which is in [1, 9].
1 7
3. a. yav =
6 1 ∫
3 sin 0.2 x dx = 2.0252 …

b. The rectangle has the same area as the shaded


Thus, the passengers get a large jerk at the region.
midpoint of the trip.
One way to remedy the problem is to reduce g (x )

the acceleration so that the elevator goes only 3

6 ft instead of 36 ft in the first 6 seconds. y = 2.0252...

That is,
1 1 π
a(t ) = − cos t x
3 3 3 1 7

You may think of other ways.


c. 2.0252… = 3 sin 0.2c
c = 3.7053… , which is in [1, 7].
1 1.5
Problem Set 10-3 4. a. yav =
1 0.5 ∫
tan x dx = 2.5181…

Q1. 50 mi/h Q2. 30 mi b. The rectangle has the same area as the shaded
region.
Q3. 20 min Q4. 2π
Q5. No local maximum Q6. 1.5 h(x )

Q7. f (x) = 16 (at x = 1) Q8. infinite 10


Q9. Mean value theorem Q10. D
5 y = 2.5181...


1 1
1. a. yav = ( x 3 – x + 5) dx = (164) = 41 x
4 1 4 0.5 1.5

270 Problem Set 10-3 Calculus Solutions Manual


© 2005 Key Curriculum Press
c. 2.5181… = tan c v = 50 − 5(t − 2)2
c = 1.1927… , which is in [0.5, 1.5]. 1 4

1
vav = [50 – 5(t – 2)2 ] dt = 43 mi/h
1 9 4 0 3

1
5. a. yav = t dt = 2 1
8 1 6 This is just 13 mi/h above the speed limit.
3
b. The rectangle has the same area as the shaded
If Ida wins her appeal, her fine will be
region. 1 1
v (t)
7 ⋅ 13 = $93 ≈ $93.33, which is $46.67 less
3 3
3 than what she now faces.
y = 2.1666...
13. Consider an object with constant acceleration a,
for a time interval [t0, t1].

1 9
t

v(t ) = a dt = at + C
At t = t0, v(t) = v 0 ⇒ v 0 = at0 + C ⇒
1 C = v 0 − at0.
c. 2 = c
6 ∴ v(t) = at + v0 − at0 = v0 + a(t − t0)
t1


25 + a(t – t0 )] dt
c = 4 , which is in [1, 9]. [v 0
36 vav =
t0

1 3 t1 – t0

100
6. a. yav = 100(1 − e − t ) dt = (2 + e –3 )
1 
v0 t1 + a(t1 – t0 )2 – v0 t0 – a(t0 – t0 )2 
3 0 3 1 1
=
= 68.3262… t1 – t0  2 2 
b. The rectangle has the same area as the shaded 1
region. = v0 + a(t1 − t0 )
2
v (t ) The average of v0 and v1 is
100 1 1
(v0 + v1 ) = [v0 + v0 + a(t1 – t0 )]
y = 68.32... 2 2
1
= v0 + a(t1 − t0 )
2
t
0 3
∴ vav = the average of v0 and v1, Q.E.D.
14. Counterexample: In Problem 11, the car’s
c. 68.3262… = 100(1 − e− c) acceleration is a = 6/ t .The initial velocity is
c = 1.1496… , which is in [0, 3]. v(0) = 60 ft/s; the final velocity after 25 seconds
1 k 2 is v(25) = 120 ft/s; and the average velocity is

1
7. yav = ax dx = ak 2 vav = 100 ft/s. But the average of the initial and
k 0 3 1
1 k 3 final velocities is [v(0) + v(25)] = 90 ft/s ≠ vav .

1 3
8. yav = ax dx = ak 2
k 0 4
15. a. Integral = area = 12(100 + 70)/2 + 6(40) +
1 k x

1
9. yav = ae dx = a(e k − 1) 12(40 + 10)/2 = 1560
k 0 k yav = 1560/30 = 52, or $52,000
1 k Cost of inventory = 0.50(52000)/100 =

1
10. yav = tan x dx = ln | sec k |
k 0 k $260.00
11. a(t) = 6t− 1/2
b. At x = 12, they may have had a single, large
v(t) = 12t1/2 + C; v(0) = 60 ⇒ C = 60 sale, dropping the inventory from $70,000 to
v(t) = 12t1/2 + 60 $40,000. There is no day on which the inventory
is worth $52,000.
s(t) = 8t3/2 + 60t + s0
v(25) = 120 ft/s y (thousand dollars)

Displacement = s(25) − s(0) = 2500 ft


100

vav = 2500/25 = 100 ft/s No x where y = 52

12. The general equation of a parabola with vertex 50


(h, k) is v − k = a(t − h)2. Vertex is at
(t, v) = (2, 50), so
v − 50 = a(t − 2)2 . v = 30 when t = 0, so x (days)
−20 = a(−2)2 ⇒ a = −5. 10 20 30

Calculus Solutions Manual Problem Set 10-3 271


© 2005 Key Curriculum Press
16. 19. v = A sin 120π t and y = | A sin 120π t |
y (ft)
1 1/60
5 10
Water surface

15 20 25
x (ft)
30
yav =
1/60 0 ∫
| A sin 120π t | dt
/
1120 1/60

y av = – 6.5142
= 60 ∫
0
A sin 120π t dt − 60 ∫/
1120
A sin 120π t dt
–5
1/120 1/60
A A
=− cos 120πt + cos 120πt
2π 0 2π 1/120
–10
A 2A
= (– cos π + cos 0 + cos 2π – cos π ) =
Integral = −(area of 4 rectangles, 2 trapezoids, 2π π
and 2 quarter-circles) 2A
2(−8) + 8(−10) + 7(−3) + 1(−2) + If yav = 110, then = 110 ⇒ A = 55π
π
7[−10 + (−5)]/2 + 5[−5 + (−3)]/2 − π(22) − = 172.78… V.
π(1)2/4 = −195.4269… The average value of one arc of
yav = −195.4269…/30 = −6.5142… , or about π


1 2
6.51 feet deep. y = sin x is sin x dx = , and
π −0 0 π
The volume would equal 6.5142… times the area y = sin x has a maximum value of 1. A
of the horizontal cross section times the number horizontal stretch does not affect the average
of gallons in a cubic foot. value. Write a proportion to find the maximum
17. Integral ≈ 3(16/2 + 15 + 15 + 17/2) + of a sinusoidal curve with an average value
2(17 + 20)/2 + 1(20 + 14)/2 + 3(14/2 + 10 + 2/π 110
9 + 8 + 9/2) = 139.5 + 37 + 17 + 115.5 = 309 of 110. = , so m = 55π .
1 m
yav = 309/24 = 12.875 ≈ 12.9°C
20. a. d = k sin x
The average of the high and low temperatures is
1 2π 2 2
(20 + 8)/2 = 14°C, which is higher than the = ∫
2
dav k sin x dx
actual average. Averaging high and low 2π 0

k2  1
x – sin 2 x 
temperatures is easier than finding the average by 1
=
calculus, but the latter is more realistic for such 2π  2 4  0
applications as determining heating and air
2
conditioning needs. k k2
= (π – 0 – 0 + 0) =
18. a. At x = 3, y = 81.3139… ≈ 81.3 mg. 2π 2
1 3 ∴ rms = k/ 2 = 0.7071K k

1
yav = 200e −0.3 x dx = (395.6202 K)
3 0 3 1 1
= 131.8734 … ≈ 131.9 mg b. cos 2 x = 1 − 2 sin 2 x ⇒ sin 2 x = − cos 2 x
2 2
b. k = 81.3139… , so the equation is Thus, sin2 x is a sinusoid.
y = 281.3139…e− 0.3(x− 3 ) .
y
1 6

3 ∫
yav = 395.6202 K + 281.3139K e –0.3( x –3) dx 
6  1
x
1
= (395.6202 K + 556.4674 K) = 158.6812 K π 2π
6
≈ 158.7 mg
c. As the graph shows, there are two times in
[0, 6] at which there are 158.7 mg. So the
1
conclusion of the mean value theorem is true, c. By symmetry across the line y = , the
in spite of the discontinuity. 2
1 1
y (mg) average of y = − cos 2 x (and hence
300 2 2
y av = 158.68... 1
y = sin2 x) over [0, 2π] is . Thus, the
200 2
1
average of y = k sin x is k 2 .
2 2

100 2
Two times ∴ rms = k/ 2 , as in part a.
x (h)
1 2 3 4 5 6 7

272 Problem Set 10-3 Calculus Solutions Manual


© 2005 Key Curriculum Press
d. By symmetry, it suffices to find the average Ann should swim toward a point about 21.8 m
and rms for one arch of the graph, that is, downstream.
over [0, π]. 2.
1 π 100
yav =
π 0 ∫
|sin x | dx 100 – x x

30
1 π
=
π 0∫ sin x dx (because sin x ≥ 0 in [0, π ])
π
1 2
= − cos x = 1 1
π 0 π T = (100 – x ) + 30 2 + x 2
13 12
1 π
dav = ∫
(|sin x | – 2/π ) 2 dx ≈ 0.094715K The graph shows a minimum T at x ≈ 72.
2
π 0
∴ rms ≈ 0.094715…1/2 = 0.3077… T

The maximum distance between high and low 10


points for this curve is 1; a sinusoidal curve
with maximum distance 1 between high and
1
low points has equation y = sin x, with x
2 100
rms = 2 /4 = 0.3535K(using part a). This
number is greater than the rms for |sin x|, Algebraic solution:
so |sin x| is smoother. 1 1
T ′ = − + (30 2 + x 2 ) −1/2 ⋅ 2 x
13 24
Problem Set 10-4 T′ = 0 ⇔
1
=
1
x (30 2 + x 2 ) −1/ 2
Q1. x = 81 Q2. y′ = −x(100 − x 2)− 1/ 2 13 12
13x = 12(302 + x2)1/2
1
Q3. − (100 – x 2 )3/2 + C Q4. y′ = 3 ⋅ (1 − 9x2)− 1/ 2 169x2 = 144 · 302 + 144x2
3 x = ±72
Q5.
1 2x 1 2x
xe − e + C Q6. y′ = sech2 x The diver should swim for 100 − 72 = 28 m,
2 4 then dive.
Q7. 1.5 Q8. t = 1 and t = 4 3.
Q9. t = 4 Q10. A x
1000
1000 – x

1. 300

50
x 100 – x
C = 40(1000 − x ) + 50 300 2 + x 2
100
The graph shows a minimum C at x ≈ 400
1 1
T= 50 2 + x 2 + (100 – x ) (exactly x = 400).
2 5
The graph shows a minimum T at x ≈ 22 m. C
100,000
T 100

x
1000
x
100
The pipeline should be laid 600 m along the road
Algebraic solution: from the storage tanks, then straight across the
1 1
T ′ = (50 2 + x 2 ) −1/2 ⋅ 2 x − field to meet the well.
4 5 4.
1 2 −1/2 1
T ′ = 0 ⇔ (50 + x ) x =
2
2 5 120
5x = 2(502 + x 2)1/2 400 – x x
25x 2 = 4 · 502 + 4x 2 400
x = ±100/ 21 = ±21.8217K

Calculus Solutions Manual Problem Set 10-4 273


© 2005 Key Curriculum Press
W = 3000( 400 − x ) + 4000 120 2 + x 2 9.
The graph shows a minimum W at x ≈ 136 x C(x), approximate
(exactly x = 360/ 7 = 136.067K).
300 49,213
W 390 49,002
2,000,000
400 49,000
410 49,002
500 49,155
x
400
The table shows that a near miss will have
virtually no effect on the minimal cost. For
instance, missing the optimal value of x by
The walkway should go 400 −136.067…
10 m will make about a $2 difference in cost,
≈ 263.9 m parallel to the street, then cross
and missing by 100 m makes only a $150 to
the street.
$200 difference.
5. a. For minimal path, x = 100/ 21. 1 1
10. T ( x ) = (500 – x ) + 1200 2 + x 2
x 5 3
∴ sin θ = = 0.4 = 2/5, Q .E.D .
50 + x 2
2
The graph shows a local minimum at x ≈ 900 ft
b. For minimal path, x = 400. (exact: 900 ft), which is out of the domain.
T (x )
x
sin θ = = 0.8 = 40/50, Q .E.D . 500
300 2 + x 2
6. Distance swimming = p 2 + x 2 . Distance
walking = k − x. x
1 2 1 500 900
T= p + x 2 + (k – x )
s w The minimum occurs at an endpoint of the
x 1 1 1
T′ = − = sin θ − domain. Because Calvin can walk entirely along
s p +x
2 2 w s w pavement when x = 0, there is a removable
s discontinuity in the above function and
T ′ = 0 ⇔ sin θ = , Q .E .D . T(0) = 100 + 240 = 340 s. Because T(500) =
w
12 433.333… , which is greater than 340, the
7. sin θ = minimum time is at x = 0. Calvin’s time is
13
minimized by staying on the sidewalks. If road
x = 30 tan  sin −1  = 72
12
construction (for instance) prevented Calvin from
 13  walking on Heights Street, his time would be
The diver should swim 100 − 72 = 28 m, then minimized by walking directly to Phoebe’s
dive. The algebraic solution is easier than before house.
because no algebraic calculus needs to be done.
11.
Mathematicians find general solutions to gain
insight, and to find patterns and methods to allow
easier solution of similar problems. 120

x 3000 3
8. sin θ = = = x
300 – x

120 + x
2 2 4000 4 70
16x2 = 9(1202 + x2)
7 x 2 = 9 ⋅ 120 2 ⇔ x = ±360/ 7 = 136.067… 1 1
The walkway should go 400 − 136.067… ≈ T= 120 2 + x 2 + 70 2 + (300 – x )2
50 130
263.9 m parallel to the street, then cross the The graph shows a minimum T at x ≈ 48 yd.
street.
The algebraic solution is easier than before T

because no algebraic calculus needs to be done.


Mathematicians find general solutions to gain 5
insight, and to find patterns and methods to allow
easier solution of similar problems. x
300

274 Problem Set 10-4 Calculus Solutions Manual


© 2005 Key Curriculum Press
Algebraically: b. Light always takes the path requiring the least
x 300 – x time between two points.
T′ = −
50 120 + x
2 2
130 70 2 + (300 – x )2 c. When you look at the object, your mind tells
you that the light rays go straight. Actually,
Setting T ′ = 0 and simplifying leads to a
they are bent, as shown in the diagram. So
fourth-degree equation, which must be solved
the object is deeper than it appears to be.
numerically. Minimum is at x = 47.8809…
Because θ water < θ air, v water < v air.
≈ 47.9 yd.
Apparent path
12. θAir of light rays

θ1
θ Apparent
120 1
Actual path depth
of light rays
300 – x
x θ Water Actual
θ2 depth
70
θ2
15. Answers will vary.
From Problem 11,
x 300 – x Problem Set 10-5
T′ = −
50 120 + x 2 2
130 70 2 + (300 – x )2 Q1. f ′(x) = sin x + x cos x Q2. g″(x) = x− 1
By trigonometry, Q3. xex − ex + C Q4. Snell’s law
x
sin θ1 = , Q5. x
120 2 + x 2 Q6. Q7.
300 – x y
sin θ 2 = 1
70 + (300 – x )2
2
x
1
1 1
∴ T′ = sin θ1 − sin θ 2
50 130
For minimal path, T ′ = 0. Thus, Q8. Total distance
1 1
sin θ1 = sin θ 2 Q9. Newton and Leibniz Q10. C
50 130 1
sin θ1 50 1. D = t + D′ = 1 − t −2
= , Q. E . D . t
sin θ 2 130 The graphs show zero derivative and local
1 1 minimum of D at t = 1, and maximum of D at
13. T = a2 + x 2 + b 2 + (k – x )2
v1 v2 t = 3.
x k–x 3 D or D'
T′ = −
D

v1 a + x
2 2
v2 b + ( k – x ) 2
2 D'
t
x
sin θ1 = , 3

a2 + x 2
k–x
sin θ 2 =
b 2 + (k – x )2 D′ = 0 ⇔ t 2 = 1 ⇔ t = ±1, confirming the graph.
1 1 Minimum is D(1) = 2, or 2000 mi.
∴ T′ = sin θ1 − sin θ 2 1
v1 v2 Maximum is D(3) = 3 , or about 3333 mi.
3
For minimal path, T ′ = 0. Thus, 2. Fuel cost per mile = k · v2.
1 1 At v = 30, cost = 0.18.
sin θ1 = sin θ 2
v1 v2 1
0.18 = k ⋅ 30 2 ⇒ k =
5000
sin θ1 v1
= , Q .E .D . 100 2000
sin θ 2 v2 Driver cost is 20t = 20 ⋅ = .
v v
14. a. The light rays take the minimal time to get 2000 v2 2000 v 2
∴ C= + ⋅ 100 = +
from one point to another, just as Robinson v 5000 v 50
Crusoe wanted to take the minimal time to 2000 v
C′ = − 2 +
get from hut to wreck. v 25

Calculus Solutions Manual Problem Set 10-5 275


© 2005 Key Curriculum Press
The graphs show minimum C at v ≈ 37 mi/h; ii. G(2) = 54.5454… , ≈ 55, about 1 point
C′ is multiplied by 10 so that it is easier to see less.
its behavior around C′ = 0. 6. a. µ = 130 − 12T + 15T 2 − 4T 3 , 0 ≤ T ≤ 3
C ′ = 0 at v = 103 50 = 36.8403… dµ
= −12 + 30T − 12T 2 = −6(2T − 1)(T − 2)
C or C' times 100 dT
C dµ
100 = 0 at T = 0.5 or T = 2
dT
µ(0) = 130
v
µ(0.5) = 127.25
37 85 100
C' times 10 µ(2) = 134
µ(3) = 121
3. Maximize f(x) = x − x2.
f ′(x) = 1 − 2x; f ′(x) = 0 at x = 0.5; Maximum viscosity occurs at T = 2, or 200°.
f ″(x) = −2, so graph is concave down b. Minimum viscosity = 121 centipoise at
everywhere. T = 3, or 300°.
Maximum of f (x) is at x = 0.5. dµ dµ dT
4. Maximize f (x) = x − x 2 for x ≥ 2. c. = ⋅
dt dT dt
The graph shows maximum at endpoint x = 2.
dT 1 1
1
f (x ) Because T = t , = = .
x dt 2 t 2T
2
dT dµ
When T = 1, = 0.5 and = 6.
dt dT

∴ = 0.5(6) = 3
dt
Because f (2) is the maximum and it is negative, Viscosity is increasing at 3 centipoise/min.
there is no number greater than 2 that exceeds its 7. a. Put a coordinate system with origin at the
square. center of the cone’s base. Pick a sample point
100t 9 900t
5. a. S = ;F= ;G = (x, y) where the cylinder touches the element
t +1 t+9 (t + 1)(t + 9) of the cone. Thus, x is the radius of the
The graph shows a maximum of G at t = 3 cylinder and y is its altitude. The volume and
hours. surface area are
y
V = π x 2y
S A = 2π x 2 + 2π x y
G
The cone element has equation y = −0.6x + 6.
50 V = π x 2(−0.6x + 6) = π (−0.6x3 + 6x2)
F times 20 A = 2π x 2 + 2π x(−0.6x + 6)
t = π (0.8x2 + 12x)
10

y
900t 900t 600
b. G = = 2
(t + 1)(t + 9) t + 10t + 9 A

900(t 2 + 10t + 9) – 900t (2t + 10)


G′ = V
(t 2 + 10t + 9)2
x
900(9 – t 2 )
= 2 10
(t + 10t + 9)2
G ′ = 0 ⇔ t = ±3 b. From the graphs, the maximum volume
Because G′ changes from positive to negative occurs where the radius x ≈ 6.7 in. The
at t = 3, there is a local maximum there, as maximum area occurs at x = 10, where all of
in the graph. the area is in the two bases of the cone.
Fran should study for 3 hours. Algebraically,
c. Optimum grade = G(3) = 56.25 ≈ 56 (Not V′ = π ( −1.8x2 + 12x)
good!) 2
V ′ = 0 ⇔ x = 0 or x = 6
i. G(4) = 55.3846… ≈ 55, about 1 point 3
less.

276 Problem Set 10-5 Calculus Solutions Manual


© 2005 Key Curriculum Press
Maximum V is at x = 6
2
in., as shown on When the water is 3 m deep, y = 8.
3 Because y = x 4 + 5, x = 4 3 .
the graph, and y = 2 in. dy – 0.7
A′ = π (1.6x + 12) = = − 0.1286 K ≈ 0.129 m/min
dt π 3
A′ = 0 ⇔ x = −7.5, which is out of the 10. a. Pick a sample point (x, y) where the cylinder
domain. touches the parabola. Thus, the radius of the
A(0) = 0 and A(10) = 200π, so maximum A cylinder is x and its altitude is y.
is at x = 10 in., as shown on the graph, and dx dV
y = 0 in. Know: = 0.3. Want: .
dt dt
The maximum volume and maximum area do V = π x 2y = π x 2(4 − x 2) = π (4x 2 − x 4)
not occur at the same radius. dV dx
= π (8 x − 4 x 3 ) ⋅ = 1.2π (2 x − x 3 )
Note that the radius of the cone is large dt dt
compared to its altitude. Thus, the increase in dV
When x = 1.5, = −0.45π .
areas of the two bases of the cylinder offsets dt
the decrease in its lateral area as x increases, dV
b. = 4π (2 x − x 3 )
making the maximum area that of the dx
degenerate cylinder of altitude zero. dV
= 0 ⇔ x = 0, ± 2 (– 2 is out of domain)
8. a. Put a coordinate system with origin at the dx
center of the cone’s base. Pick a sample point V (0) = V (2) = 0; V ( 2 ) = 4π
(x, y) where the cylinder touches the element Maximum volume = 4π ≈ 12.6 units3 at
of the cone. Thus, x is the radius of the radius = 2 units.
cylinder and y is its altitude. 11. a. w = 1000 + 15t (lb); p = 0.90 − 0.01t ($/lb)
Know:
dy
= 2 in./min. Want:
dV
. A = (1000 + 15t)(0.90 − 0.01t)
dt dt = 900 + 3.5t − 0.15t2 ($)
V = π x 2y dA 35 2
The cone element has equation y = −3x + 18, b. = 3.5 − 0.3t = 0 at t = = 11 days
dt 3 3
1 2
from which x = 6 − y. Maximum A at t = 11 , not a minimum,
3 3
2
V = π  6 − y ⋅ y = π  36 y − 4 y 2 + y 3 
1 1 dA
because goes from positive to negative
 3   9  dt
= π  36 − 8 y + y 2  = π ( y − 6)( y − 18)
dV 1 1 there.
 3  3
c. A 11  = 920 ≈ $920.42
dy 2 5
dV dV dy 2  3 12
= ⋅ = π ( y − 6)( y − 18)
dt dy dt 3 12. a. 0 ≤ D ≤ 130 ⇒ 0 ≤ 20x + 10 ≤ 130 ⇒
dV 1
When y = 12, = −24π . − ≤x≤6
dt 2
V is decreasing at 24π = 75.3982… ≈ 0 ≤ W ≤ 310 ⇒ 0 ≤ 10(x 2 − 8x + 22) ≤
75.4 in.3/min. 310 ⇒ −1 ≤ x ≤ 9
b. If t ∈ [0, 9], then y ∈ [0, 18]. Given x ≥ 1, the domain of x is [1, 6].
dV
= 0 ⇔ y = 6 or y = 18 b. Minimize/maximize W on x ∈ [1, 6].
dt dW
V(0) = 0; V(6) = 96π ; V(18) = 0 = 10(2 x − 8) = 0 at x = 4.
dx
Maximum V is 96π in.3 at t = 3 min. W(1) = 150; W(4) = 60; W(6) = 100
c. If t ∈ [4, 6], then y ∈ [8, 12]. Minimum: W = 60 ft (at x = 4 mi)
No critical points for V are in [8, 12]. Maximum: W = 150 ft (at x = 1 mi)
V(8) = 88.8888…π; V(12) = 48π c. C = k · D · W
Maximum V is 88.8888…π ≈ 279.3 in.3 at = k · (20x + 10) · 10(x 2 − 8x + 22)
t = 4 min. = 100k(2x3 − 15x2 + 36x + 22) (k > 0)
dV dy dC
9. Know: = −0.7 m 3 /min. Want: . = 100 k (6 x − 30 x + 36)
2

dt dt dx
dV = πx2 dy = 600 k ( x – 2)( x – 3)
dV dy dy – 0.7 dC
∴ = πx 2 ⇒ = = 0 ⇔ x = 2 or x = 3
dt dt dt π x 2 dx

Calculus Solutions Manual Problem Set 10-5 277


© 2005 Key Curriculum Press
C(1) = 4500k; C(2) = 5000k; C(3) = 4900k; (Note that the distance traveled is less than the
C(6) = 13,000k distance from the origin because the particle
Cheapest bridge at x = 1 mi. started at (1, 0), not at (0, 0).)
r r r
d. No. The shortest bridge at x = 4 mi would c. a = ( −2e t sin t )i + (2e t cos t ) j .
cost C(4) = 5400k, which is 900k more than r r r
a (1) = ( −4.5747...) i + (2.9373...) j
the cheapest bridge at x = 1. r r r
4. a. r1 = (t − 2)i + (t − 2) 2 j and
r r r
Problem Set 10-6 r2 = (1.5t − 4)i + (1.5t − 2) j
r r r r r r
Q1. −x cos x + sin x + C Q2. 2xe3x + 3x2e3x v1 = 1i + 2(t − 2) j and v2 = 1.5i + 1.5 j
r r r r r r
2x a1 = 0i + 2 j and a2 = 0i + 0 j
Q3. +C Q4. 53 = 125 r r r r r r
ln 2 v1 (1) = i − 2 j and v2 (1) = 1.5i + 1.5 j
r r r r r r
1 4 6 sec 2 6t 2 a1 (1) = 0i + 2 j and a2 (1) = 0i + 0 j
Q5. x +7 Q6. = sec 2 ln x 2
4 3e 3t x s1 (1) = 12 + 2 2 ≈ 2.24 cm/s and
Q7. parametric Q8. x ln x − x + C
s2 (1) = 1.52 + 1.52 ≈ 2.12 cm/s
Q9. x2 Q10. E
4
1. The velocity is tangent to the path, the
acceleration is toward the concave side of the
b. Distance = ∫
1
12 + [2(t – 2)]2 dt ≈
6.1257… ≈ 6.13 m
path, and there is an obtuse angle between
acceleration and velocity. c. The paths cross at (x, y) = (−1, r 1) and (2, 4).
By tracing on the grapher, r1 is at (−1, 1)
y r

v when t = 1, but r2 is not at (−1, 1) until
t = 2.
→ →
By further tracing, both paths are at (2, 4)
r a
when t = 4.
x So the particles collide only at (x, y) = (2, 4)
when t = 4.
r r r
2. The velocity is tangent to the path, the 5. a. r (t ) = (10 sin 0.6t )i + ( 4 cos 1.2t ) j
r r r
acceleration is toward the concave side of the v (t ) = (6 cos 0.6t )i + ( −4.8 sin 1.2t ) j
path, and there is an acute angle between the r r r
a (t ) = ( −3.6 sin 0.6t )i + ( −5.76 cos 1.2t ) j
acceleration vector and the velocity vector. r r
r
y b. r (0.5) = (10 sin 0.3)i + ( 4 cos 0.6) j

v r r
→ = 2.9552 …i + 3.3013… j
a
r r r
v (0.5) = (6 cos 0.3)i + ( −4.8 sin 0.6) j

r r r
= 5.7320 …i − 2.7102 … j
r r r
x
a (0.5) = ( −3.6 sin 0.3)i + ( −5.76 cos 0.6) j
r r
r r r = −1.0638…i − 4.7539… j
3. a. r = (e t cos t ) i + (e t sinr t ) j r r r
r r The graph shows r , v , and a at t = 0.5.
v = (e t cos t − e t sin t ) i + (e t sin t + e t cos t ) j
r r r
v (1) = −0.8186 Ki + 3.7560 … j y

x is decreasing at t = 1 because dx/dt is


negative. t = 0.5
at
Speed = 0.8186 K2 + 3.7560...2 = v
a
3.8442… ≈ 3.84 cm/s x
an a
b. L = t=7
1 v

∫ (e cos t − e sin t ) + (e sin t + e cos t ) dt


t t 2 t t 2
0
1
=∫ 2e 2 t dt = 2 (e – 1) = 2.4300 K ≈ 2.43
0
These vectors make sense because the head of
Distance from origin r r
r is on the graph, v is tangent to the graph,
= (e1 cos1)2 + (e1 sin 1)2 r
and a points to the concave side of the graph.
= e = 2.7812… ≈ 2.78 cm c. The object is speeding up because the angle
r r
between a and v is acute.
278 Problem Set 10-6 Calculus Solutions Manual
© 2005 Key Curriculum Press
r r r r
d. | v(0.5) | = (6 cos 0.3)2 + (–4.8 sin 0.6)2 The graph shows r , v , and a at t = 1.
y
= 6.3404…
r r
a (0.5) ⋅ v (0.5) = ( −3.6 sin 0.3)(6 cos 0.3) t = 1.25π
a
+ (−5.76 cos 0.6)(−4.8 sin 0.6) v at

= 6.7863… , so the angle is acute. a t=1


r r
a (0.5) ⋅ v (0.5)
r an x
P= r = 1.0703…
|v (0.5)|
r r
r v (0.5) a
at (0.5) = P r
| v (0.5)| v
r r t = 10.5
(6 cos 0.3)i + (–4.8 sin 0.6) j
=P r
| v (0.5)|
r r These vectors make sense because the head of
r r
= 0.9676 …i − 0.4575… j r is on the graph, v is tangent to the graph,
r r r r
an (0.5) = a (0.5) − at (0.5) and a points to the concave side of the graph.
r r
= −2.0314 …i − 4.2964 … j c. The object is speeding up because the angle
r r
See the graph in part b. between a and v is acute.
r r
e. The object is speeding up at | at (0.5)| = P d. | v(1) | = (–6.4 sin 0.8)2 + (2.4 cos 0.4)2
= 1.0703… ≈ 1.07 (ft/s)/s. = 5.0955…
r r r
f. r (7) = (10 sin 4.2)ri + ( 4 cos 8.4r) j r r
a (1) ⋅ v (1) = ( −5.12 cos 0.8)( −6.4 sin 0.8)
= −8.7157…i − 2.0771… j + (−0.96 sin 0.4)(2.4 cos 0.4)
r r r
v (7) = (6 cos 4.2)i + ( −4.8 sin 8.4) j = 15.5506… , so the angle is acute.
r r r r
= −2.9415…i − 4.1020 … j a (1) ⋅ v (1)
r r P= r = 3.0518…
r | v (1)|
a (7) = ( −3.6 sin 4.2)i + ( −5.76 cos 8.4) j r
r r r v (1)
= 3.1376 …i + 2.9911… j at (1) = P r
See the graph in part b. | v (1)|
r r
The object is slowing down because the
r r (–6.4 sin 0.8)i + (2.4 cos 0.4) j
angle between ra andr v is obtuse. =P r
| v (1)|
a (7) ⋅ v (7) r r
(Note that P = r = −4.2592 … , so = −2.7496 …i + 1.3239… j
| v (7)| r r r
the object is slowing down at 4.2592… an (1) = a (1) − at (1)
r r
≈ 4.26 (ft/s)/s.) = −0.8174 …i − 1.6977… j
r r r r r r r
g. r (0) = (10 sin 0)i + ( 4 cos 0) j = 0i + 4 j See the graph in part b, showing at and an
r r r r r
v (0) = (6 cos 0)i + ( −4.8 sin 0) j = 6i + 0 j at t = 1.
r r r
a (0) = ( −3.6 sin 0)i + ( −5.76 cos 0) j e. The
r object is speeding up at
r r | at (1)| = P(1) = 3.0518… ≈ 3.05 (ft/s)s.
= 0i − 5.76 j r r r
r r r (10.5) = (8 cos 8.4)i + (6 sin 4.2) j
a (0) ⋅ v (0) = (0)(6) + ( −5.76)(0) = 0 f.
r r
r r
∴ a(0) and v(0) are perpendicular, Q.E.D. = −4.1543…i − 5.2294 … j
r r r
This means the object is neither slowing v (10.5) = ( −6.4 sin 8.4) i + (2.4 cos 4.2) j
down nor speeding up at t = 0. r r
r r r = −5.4694 …i − 1.1766 … j
6. a. r (t ) = (8 cos 0.8t )i + (6 sin 0.4t ) j r r r
r r r a (10.5) = ( −5.12 cos 8.4)i + ( −0.96 sin 4.2) j
v (t ) = ( −6.4 sin 0.8t )i + (2.4 cos 0.4t ) j r r
r r r = 2.6587…i + 0.8367… j
a (t ) = ( −5.12 cos 0.8t )i + ( −0.96 sin 0.4t ) j r r r
r r See the graph in part b, showing r , v , and a
r
b. r (1) = (8 cos 0.8)i + (6 sin 0.4) j at t = 10.5.
r r
= 5.5736 …i + 2.3365… j The object is slowing downr at t =r10.5
r r r because the angle between a and v is obtuse
v (1) = ( −6.4 sin 0.8)i + (2.4 cos 0.4) j
r r at that time. (Note that
= −4.5910 …i + 2.2105… j r r
r r r a (10.5) ⋅ v (10.5)
P= r = −2.7552 … ,
a (1) = ( −5.12 cos 0.8)i + ( −0.96 sin 0.4) j |v (10.5)|
r r
= −3.5671…i − 0.3738… j so the object is slowing down at about
2.8 (ft/s)/s.)

Calculus Solutions Manual Problem Set 10-6 279


© 2005 Key Curriculum Press
π π
g. The object is stoppedrwhen 2 2
r r ∴   +   = cos 2 t + sin 2 t
r x y
v (t ) = ( −6.4 sin 0.8t )i + (2.4 cos 0.4t ) j = 0  10   6  6 6
⇔ −6.4 sin 0.8t = 0 and 2.4 cos 0.4t = 0.
π π π π
2 2
Using −6.4 sin 0.8t = −1.28 rsin 0.4t
r +   sin 2 t +   cos 2 t
cos 0.4t, you see that v (t ) = 0 exactly when  6 6  6  6
cos 0.4t = 0; the first time this happens is at  x  +  y = 1+  π 
2 2 2

t = 1.25π s. r r  10   6   6
r
r (1.25π ) = (8 cos π )i + (6 sin 0.5π ) j
r r x2 y2
= −8i + 6 j + =1
r r r 100(1 + π 2 /36) 36(1 + π 2 /36)
a (1.25π ) = ( −5.12 cos π )i + ( −0.96 sin 0.5π ) j
r r This is the equation of an ellipse centered at
= 5.12i − 0.96 j the origin with x-radius 11.2878… and
r
See the graph in part b, showing a at y-radius 6.7727… .
t = 1.25π . r  10π 2 π  r  6π 2 π r
The acceleration vector points along the path d. a (t ) =  − cos t  i +  − sin t  j
 36 6   36 6 
at t = 1.25π. So the object is stopped, but it r r
has a nonzero acceleration. At first glance, r (t ) + a (t )
this fact may be surprising to you!  10π 2  π r  6π 2  π r
r π r π r = 10 −  cos t i +  6 −  sin t j
7. a. r (t ) = 10 cos t  i +  6 sin t  j  36  6  36  6
 6   6 
r See the graph in part a, showing an elliptical
r 10π π 6π π r
v (t ) =  − sin t  i +  cos t  j path followed by the heads of the acceleration
 6 6   6 6  vectors.
r r π 10π π r
r (t ) + v (t ) = 10 cos t − sin t  i e. The direction of each acceleration vector is the
 6 6 6  opposite of the corresponding position vector
π 6π π r
+  6 sin t + cos t  j
and is thus directed toward the origin.
 6 6 6  r π2 r
r r Note that a (t ) = − r (t ).
The graph shows the path of r + v . 36
r r r
8. a. r (t ) = (0.5t cos t )i + (0.5t sin t ) j
y

r r
v (t ) = (0.5 cos t − 0.5t sin t )i
3 r
4 2
+ (0.5 sin t + 0.5t cos t ) j
5 r r
a (t ) = ( − sin t − 0.5t cos t )i
1
0 x r
6 12 + (cos t − 0.5t sin t ) j
7 11
r r r
b. r (8.5) = 4.25 cos 8.5 i + 4.25 sin 8.5 j
8 10 r r
= −2.5585…i + 3.3935… j
9

r r
v (8.5) = (0.5 cos 8.5 − 4.25 sin 8.5)i
r r
b. See the graph in part a, showing vectors v . + (0.5 sin 8.5 + 4.25 cos 8.5) j
r r r r
c. For r + v , = −3.6945…i − 2.1593… j
r r
x π π π a (8.5) = ( − sin 8.5 − 4.25 cos 8.5)i
= cos t − sin t r
10 6 6 6 + (cos 8.5 − 4.25 sin 8.5) j
y π π π r r
= sin t + cos t = 1.7600 …i − 3.9955… j
6 6 6 6 r r r
r (12) = 6 cos 12i + 6 sin 12 j
 x  = cos 2 π t − π cos π t sin π t
2 r r
= 5.0631…i − 3.2194 … j
 10  6 3 6 6 r r
v (12) = (0.5 cos 12 − 6 sin 12)i
π π
2 r
+   sin 2 t + (0.5 sin 12 + 6 cos 12) j
 6 6 r r
= 3.6413…i + 4.7948… j
 y  = sin 2 π t + π sin π t cos π t
2
r r
a (12) = ( − sin 12 − 6 cos 12)i
 6 6 3 6 6 r
+ (cos 12 − 6 sin 12) j
π π r r
2
+   cos 2 t = −4.5265…i + 4.0632 … j
 6 6

280 Problem Set 10-6 Calculus Solutions Manual


© 2005 Key Curriculum Press
r r r
The graph shows that r (8.5) and r (12) really At x = 2, speed = | v (2) | = 153 =
do terminate on the path. 12.3693… ≈ 12.4 cm/s.
y
r r
c. The graph shows r (2) and v(2).
r
This is reasonable because v(2) points along
the curve to the left, indicating that x is
t = 8.5
v r decreasing.
v
a x y
a a
20 at
r
at
t = 12

an
r x=2
c. See the graph in part b, showing v(8.5), x
r r r 5
v (12), a (8.5), and a(12). The velocity
vectors point along the path as it spirals v

outward, and the acceleration vectors point r r r


inward to the concave side of the graph. d. From part b, v ( x ) = −3i − 6 xj ,
r r r r dx r r
d. In both cases, the angle between a and v ∴ a ( x ) = 0i − 6 j = 18 j
r dt
appears to be acute. Check using dot r
products. a (2) = 18 j . See the graph in part c.
r r r r r
a (8.5) ⋅ v (8.5) = 2.125, which is positive. r a (2) ⋅ v (2) v (2)
r r e. at (2) = r ⋅ r
a (12) ⋅ v (12) = 3, which is also positive. | v (2)| | v (2)|
Thus, the angles are acute, and the object is r r
–216(–3i – 12 j )
speeding up at both times. =
r r 153
e. At t = 12 h, a (12) ⋅ v (12) = 3.
r 72 r 288 r r r
= i+ j = 4.2352 …i + 16.9411… j
| v(12) | = 36.25 = 6.0207…
r r r r 17 r 17 r
r a (12) ⋅ v (12) v (12) an (2) = a (2) − at (2)
at (12) = r ⋅ r
| v (12)| | v (12)| 72 r 18 r r r
=− i + j = −4.2352 …i + 1.0588… j
3 r r r 17 17 r
= (3.6413...i + 4.7948... j ) at (2) is parallel to the curve. an (2) is
36.25
r r normal to the curve and points inward to the
= 0.3013…i + 0.3968… j
r r r concave side.
an (12) = a (12) − at (12)
r r f. The object is slowing downr when xr= 2
= −4.8279…i + 3.6664 … j because the angle between a(2) and v(2) is
r r obtuse, as shown by the graph and by the
See the graph in part b, showing an and at at r fact
t = 12. that the dot product is negative. Also, at (2)
r
r points in the opposite direction of v(2).
f. Speed = | v(12) | = 36.25 = 6.0207…
≈ 6.02 mi/h g. dL = 1 + ( dy/dx )2 dx = 1 + 4 x 2 dx
3 dL dx
Speed is increasing by P(12) = = = 1 + 4x2 =5
36.25 dt dt
0.4982… ≈ 0.498 (mi/h)/h. dx dx
r r At x = 2, 5 = 1 + 4(2)2 = 17
g. See the graph in part b, showing r (t ) + a (t ). dt dt
dx 5
The heads seem to lie on a unit circle. ⇒ = = 1.2126 … ≈ 1.21 cm/s.
Algebraic verification: dt 17
r r r r r r r
r (t ) + a (t ) = − sin t i + cos t j , which is a 10. r (1) = 8.8615…i + 4.8410 … j
circle. r r r
r r r r q (2) = −2.9659…i + 4.3406 … j
r r r r
9. a. r ( x ) = xi + yj = xi + x 2 j q (1.5) = −0.2065…i + 6.6362 … j
r dx r dy r dx r dx r r r r
v( x) = i+ j= i + 2x j q (1.1) = 2.5579…i + 7.4880 … j
dt dt dt dt r r
r r v (t ) = (12 cos t cos 0.5t − 6 sin t sin 0.5t )i
dx r r
b. = −3 ⇒ v ( x ) = −3i − 6 xj + (12 cos t sin 0.5t + 6 sin t cos 0.5t ) j
dt r r r r r
r v (1) = 3.2693…i + 7.5391… j
v (2) = −3i − 12 j

Calculus Solutions Manual Problem Set 10-6 281


© 2005 Key Curriculum Press
The graph shows average velocity vectors e. When x = 400, t = 2 sec 15° = 2.0705… s
approaching the instantaneous velocity vector y(2.0705…) = 41.5846… .
r
v(1) as t approaches 1. The instantaneous Phyllis makes the home run because
velocity vector is tangent to the graph and points 41.5… > 10.
in the direction of motion. y

100
t = 1.1 v
t = 1.5
x
10
t=2 400

r r r
14. a. a (t ) = 3i + y ′′(t ) j
r r r
v (t ) = (3t + C1 )i + ( y′(t )) j
r r r
v (0) = 0i + 0 j ⇒ C1 = 0
x
r r r
10 r (t ) = (1.5t 2 + C2 )i + ( y(t )) j
r r r
r r r r (0) = 0i + 0 j ⇒ C2 = 0
11. r (t ) = (10 sin 0.6t )i + ( 4 cos 1.2t ) j r r r r r
∴ r (t ) = (1.5t 2 )i + ( y(t )) j = (1.5t 2 )i + (sin x (t )) j
r r r
dL = ( dx/dt )2 + ( dy/dt )2 dt r (t ) = (1.5t 2 )i + (sin 1.5t 2 ) j
r r r
= (6 cos 0.6t )2 + (–4.8 sin 1.2t )2 dt b. v (t ) = (3t )i + (3t cos 1.5t 2 ) j
2 If x = 6, t = 2.
L= ∫ dL ≈ 12.0858… ft (numerically)
0
r r
v (2) = 6i + 5.7610 K j
r

r π r π r
12. r (t ) = 10 cos t  i +  6 sin t  j
r
 Speed = | v(2) | = 6 2 + 5.76 K2 = 8.3180 …
6   6 
≈ 8.32 m/ min
dL = ( dx/dt )2 + ( dy/dt )2 dt
15. a. d = a + b cos t
–10π π 6π π
2 2 t = 0: 240 = a + b cos 0 = a + b
=  ⋅ sin t  +  ⋅ cos t  dt t = π : −60 = a + b cos π = a − b
 6 6   6 6 
2a = 180 ⇒ a = 90
One complete cycle of the curve is 0 ≤ t ≤ 12, so
12 2b = 300 ⇒ b = 150
L= ∫ dL = 51.0539… (numerically) ≈ 51.1 ft.
0
∴ d = 90 + 150 cos t
r r
r r r b. r (t ) = (90 cos t + 150 cos 2 t ) i
13. a. a (t ) = 0i − 32 j r
r r r r r + (90 sin t + 150 sin t cos t ) j
r

v (t ) = (0i − 32 j ) dt = C1i + ( −32t + C2 ) j
r r r
v (t ) = ( −90 sin t − 300 cos t sin t ) i
r
v (0) = −130i + 0 j ⇔ C1 = −130 and C2 = 0 r
r r r
∴ v (t ) = −130i − 32tj + (90 cos t + 150 cos 2 t − 150 sin 2 t ) j
r r
r r r v (t ) = ( −90 sin t − 150 sin 2t ) i

b. r (t ) = ( −130i − 32tj ) dt
r r + (90 cos t + 150 cos 2t ) j
r
= ( −130t + C3 )i + ( −16t 2 + C4 ) j r r
r r r r
r (0) = 60.5i + 8 j ⇔ C3 = 60.5 and C4 = 8 v (1) = −212.1270 Ki − 13.7948K j
r r r
∴ r (t ) = ( −130t + 60.5)i + ( −16t 2 + 8) j Speed = (–212.1K)2 + (–13.7K)2
c. When the ball passes over the plate, x(t) = 0, = 212.5750 K ≈ 212.6 cm/s
so t = 60.5/130 = 0.4653… . At that time, r r
c. a (t ) = ( −90 cos t − 300 cos 2t ) i
y(t) = 4.5346… , which is slightly above the r
strike zone. + ( −90 sin t − 300 sin 2t ) j
r r r
d. At t = 0, dx/dt = 200 cos 15°, a (1) = 76.2168Ki − 348.5216 K j
dy/dt = 200 sin 15°. r r r
r r a (1) ⋅ v (1)
As in part a, v (t ) = C1i + ( −32t + C2 ) j = P(1) = r = −53.4392 K
r r | v (1)|
(200 cos 15°)i + ( −32t + 200 sin 15°) j r
r r r v (1)
As in part b, r (t ) = (200t cos 15°)i + at (1) = P(1) r
r | v (1)|
( −16t 2 + 200t sin 15° + 3) j . r r
= 53.3266 Ki + 3.4678K j

282 Problem Set 10-6 Calculus Solutions Manual


© 2005 Key Curriculum Press
r r r r r
an (1) = a (1) − at (1) The graph shows v (2.5) and a (2.5).
r r
= 22.8902 Ki − 351.9894 K j y

Annie is slowing down. The angle between


the acceleration and velocity vector is obtuse,
as revealed by the negative dot product. She is
slowing down at 53.4392… ≈ 53.4 cm/s. a

r r r
16. a. r = (0.5t + sin t ) i + ( 4 cos 0.5t ) j
r r r 5

v (t ) = (0.5 + cos t ) i + ( −2 sin 0.5t ) j x

r r r 5 v
a (t ) = ( − sin t )i + ( − cos 0.5t ) j
r r r r
r a (2.5) ⋅ v (2.5)
v (14) = 0.6367…i − 1.3139… j c. P(2.5) = r
r r r | v (2.5)|
a (14) = −0.9906 …i − 0.7539… j 60 sin 2.5
r = = 2.2052 …
Speed = | v(14) | = 1.4601… mi/h 169 – 120 cos 2.5
r r r
a (14) ⋅ v (14) r
P(14) = r at (2.5) = P(2.5) r
v (2.5)
| v (14)| | v (2.5)|
0.3598K r r
= = 0.2464 K = 1.9791…i − 0.9726 … j
1.4601K r r r
r an (2.5) = a (2.5) − at (2.5)
r v (14) r r
at (14) = P(14) r = 5.2024 …i + 10.5863… j
| v (14)|
r r r
d. v(2.5) is reasonable because its graph
= 0.1074 …i − 0.2217… j
r r r points alongr the path in the direction of
an (14) = a (14) − at (14) motion. a(2.5) is reasonable because it points
r r toward the concave side of therpath. The
= −1.0980 …i − 0.5312 … j
roller coaster is traveling at | v(2.5) | =
The log is speeding up at t = 14. You can tell 16.2830… ft/s. Its speed is increasing at
by the fact that rP(14) is positive,
r and thus the P(2.5) = 2.2052… ft/s2, as shown by the fact
angle between a(14) and v(14) is acute, which
r that P(2.5) r is positive, r meaning that the angle
means that at (14) points in the same direction between a(2.5) and v(2.5) is acute.
r
as v(14) . It is speeding up at 0.2464… ≈ e. The path is at a high
0.246 (mi/h)/h. r point when the
y-component of r is a maximum. This
happens when cos t = 1, or t = 0 + 2π n.
b. dL = dx 2 + dy 2 r r r
a (0 + 2π n) = 0i − 12 j , pointing straight
= (0.5 + cos t )2 + (–2 sin 0.5t )2 dt down.
14 Similarly, the path is at a low point when
L= ∫
0
dL ≈ 22.7185… (numerically) cos t = −1, or t = π + 2π n.
r r r
≈ 22.7 mi a (π + 2π n) = 0i + 12 j , pointing straight up,
Q .E .D .
1
Average speed ≈ (22.7185K)
14 f. dL = dx 2 + dy 2
= 1.6227… ≈ 1.62 mi/h
r r r = (5 – 12 cos t )2 + (–12 sin t )2 dt
17. a. r (t ) = (5t − 12 sin t ) i + (15 + 12 cos t ) j 2π
r r
v (t ) = (5 − 12 cos t ) i + ( −12 sin t ) j
r L= ∫0
dL ≈ 78.7078… (numerically)

r r r ≈ 78.7 ft
a (t ) = (12 sin t ) i + ( −12 cos t ) j r r
r r 18. Recall that |i | = | j | = 1.
r r r
b. v (2.5) = 14.6137…i − 7.1816 … j The angle between i and i is 0, so
r r r r r r
r i ⋅ i = |i ||i | cos 0 = 1.
a (2.5) = 7.1816 Ki + 9.6137K j r r
Similarly, j ⋅ j = 1.

Calculus Solutions Manual Problem Set 10-6 283


© 2005 Key Curriculum Press
r r π dφ 15 sin 2 t + 15 cos 2 t
The angle between i and j is , so ∴ =
2 dt (25 sin 2 t + 9 cos 2 t )3/ 2
r r r r π
i ⋅ j = | i | | j | cos = 0. 15
r 2 r r r =
r r (16 sin 2 t + 9)3/ 2
∴ v1 ⋅ v2 = ( x1i + y1 j ) ⋅ ( x2 i + y2 j ) =
r r r r r r r r which is maximized for sin2 t = 0 at (± 5 , 0),
x1 x 2 i ⋅ i + x1 y2 i ⋅ j + y1 x 2 j ⋅ i + y1 y2 j ⋅ j
= x1 x 2 (1) + x1 y2 (0) + y1 x 2 (0) + y1 y2 (1) the ends of the major axis, and is minimized
for sin2 t = 1 at (0, ±3), the ends of the minor
= x1 x 2 + y1 y2 , Q .E.D .
r r axis, Q.E.D.
r
19. r (t ) = (10 sin 0.8t )i + (10 cos 0.6t ) j e. x = r cos t ⇒ x′ = −r sin t ⇒ x′′ = −r cos t
r
+ (6t 0.5 )k y = r sin t ⇒ y′= r cos t ⇒ y′′ = −r sin t
r r r
v (t ) = (8 cos 0.8t )i + ( −6 sin 0.6t ) j r
r |v | = r 2 cos 2 t + r 2 sin 2 t = r
+ (3t −0.5 )k
r dφ r 2 sin 2 t + r 2 cos 2 t 1
r ∴ = =
a (t ) = ( −6.4 sin 0.8t )i ds | r |3 |r |
r r
+ ( −3.6 cos 0.6t ) j + ( −1.5t −1.5 )k a constant equal to the reciprocal of the radius.
r r r r
v (1) = (8 cos 0.8)i + ( −6 sin 0.6) j + 3k f. x = 5 cos2 t ⇒ x′ = −10 cos t sin t = −5 sin 2t
r r r r
a (1) = ( −6.4 sin 0.8)i + ( −3.6 cos 0.6) j − 1.5k ⇒ x′′ = −10 cos 2t
y = 3 sin2 t ⇒ y′ = 6 sin t cos t = 3 sin 2t
To determine whether the object is speeding up ⇒ y′′ = 6 cos 2t
or slowing down, find the dot product. r
r r |v | = 25 sin 2 2t + 9 sin 2 2t = 34 | sin 2t |
a (1) ⋅ v (1) = ( −6.4 sin 0.8)(8 cos 0.8) +
( −3.6 cos 0.6)( −6 sin 0.6) + ( −1.5)(3) dφ –30 sin 2t cos 2t + 30 sin 2t cos 2t
∴ =
= −20.0230… ds 34 |sin 2t |
∴ the object is slowing down because the angle = 0, Q.E.D.
r r
between a(1) and v(1) is obtuse. dφ 15 5
20. a. This is an example of the chain rule. g. At (5, 0), sin t = 0, so = 3/ 2 = .
ds 9 9
b. dy/dx equals the slope of the velocity vector, 9
and tan φ also equals the slope of this vector. Radius of curvature = = 1.8
5
Thus, tan φ = dy/dx.
h. The osculating circle has radius 1.8 and center
By the chain rule, dy/dx = (dy/dt)/(dx/dx), on the x-axis at x = 4 − 1.8 = 3.2. Equations
Q .E .D . are
dy/dt y′(t ) x = 3.2 + 1.8 cos t
c. tan φ = =
dx/dt x ′(t )
y = 1.8 sin t
d d  y ′(t ) 
⇒ (tan φ ) =   The graph shows the osculating circle. The
ds ds  x ′(t )  name is appropriate because the circle “kisses”
dφ x ′ y′′ – x ′′ y′ dt the ellipse at the point (5, 0).
⇒ sec 2 φ =
ds x′2 ds 3 y
dφ x ′ y′′ – x ′′ y′
∴ =
ds x ′ 2 (sec 2 φ )( ds/dt ) x
3.2 5
But sec φ = 1 + tan φ = 1 + y′ /x′ , so
2 2 2 2
r
ds/dt = | v |.
dφ x ′ y′′ – x ′′ y′
∴ = 2 r
ds x ′ (1 + y′ 2 / x ′ 2 ) |v |
x ′ y′′ – x ′′ y′ x ′ y′′ – x ′′ y′ Problem Set 10-7
= 2 r = r , Q .E .D .
( x ′ + y ′ ) |v |
2
| v |3 Review Problems
d. x = 5 cos t ⇒ x′ = −5 sin t ⇒ x″ = −5 cos t R0. Answers will vary.
y = 3 sin t ⇒ y′ = 3 cos t ⇒ y′′ = −3 sin t R1. v = t − 3 = 0 at t = 9 s
r v > 0 for t > 9 s
|v | = 25 sin 2 t + 9 cos 2 t

284 Problem Set 10-7 Calculus Solutions Manual


© 2005 Key Curriculum Press
Displacement from t = 0 to t = 9 is ii. The rectangle has the same area as the
9 shaded region.
∫(
0
t – 3) dt = −9.
f (x )

They have moved 9 ft closer to the sawmill.


18
From t = 0 to t = 25:
25


1
Displacement = ( t – 3) dt = 8 ft 5
x
0 3 6
25


1
Distance = | t − 3| dt = 26 ft
0 3 iii. The average of the two values of f(x) at the
R2. a. i. endpoints is 0, not 18.
v (t ) R4. a.

5
θ θ 200
t 700 – x x
3 700

Let x = distance from intersection to cutoff.


0 ≤ x ≤ 700
4 Let T = total time taken.
ii. Displacement = ∫ (2 – 8) dt ≈ −3.8022 K
t
1 1 1
T= 200 2 + x 2 + (700 – x )
≈ −3.8 cm (exactly 14/ln 2 − 24) 5.7 6.2
4
iii. Distance = ∫ |2 − 8| dt ≈ 10.8853…
t T
1
≈ 10.9 cm (exactly 2/ln 2 + 8)
100
b.
tend a aav
x
700
0 2 — 30 speeding up
1 1 1
5 8 5 55 speeding up T′ = ⋅ (200 2 + x 2 ) −1/2 (2 x ) −
10 1 4.5 77.5 speeding up 5.7 2 6.2
1 2 −1/2 1
15 0 0.5 80 neither = ⋅ x (200 + x ) −
2
5.7 6.2
20 −10 −5 55 slowing down T′ = 0 ⇔ 6.2x = 5.7(2002 + x 2)1/2
25 −20 −15 −20 slowing down 38.44x2 = 32.49(2002) + 32.49x2
32.49(200 2 )
(Note that the object is speeding up, slowing x2 = ⇔ x = ±467.3544 K
5.95
down, or neither, exactly when aend > 0,
aend < 0, or aend = 0, respectively, in the Or: Let θ = angle of incidence.
original table.) Minimal path occurs for θ = sin− 1 (5.7/6.2).
3
x = 200 tan θ = 467.3544…
∫ sin (π t/6) dt = 2/π = 0.6366K
1 Note that at x = 0, Juana goes entirely along
R3. a. i. vav =
3 0 the sidewalk.
9 1
∫ sin (π t/6) dt = 0
1 T(0) = ⋅ 900 = 145.1612 K
ii. vav =
6 3 6.2
12 T(467.3544…) = 126.7077…

1
iii. vav = sin (π t/6) dt = 0 T(700) = 127.7212…
12 0
Turning at a point about 467 ft from the
b. i. f ( x ) = 6 x 2 − x 3 = x 2 (6 − x ) = 0 at x = 0, 6 intersection of the two sidewalks takes the
6 minimum time, although it takes only a
∫ (6 x
1
Average = 2
– x 3 ) dx = 18 second longer to head straight for the English
6 0
building.

Calculus Solutions Manual Problem Set 10-7 285


© 2005 Key Curriculum Press
b. Because s(t) measures distance from the
6 starting point, s(0) = 0, which implies that
θ
x C = 0.
θ 1
10 ∴ s( t ) = t 3 − t 4
12
1
s ′(t ) = v(t ) = t 2 ( 9 − t )
10 – x

3
s′(t) = 0 ⇔ t = 0 or t = 9
1 2
Let x = distance from closest point on the s(0) = 0; s(9) = 182 ; s(10) = 166
beach to the cutoff point, 0 ≤ x ≤ 10. 4 3
1
Let C = total cost of the road. Maximum displacement = 182 at t = 9.
4
C = 5(10 − x ) + 13 36 + x 2 Minimum displacement = 0 at t = 0.
The graph shows a minimum C at x ≈ 2.5.
b. i. Let t = number of days Dagmar has been
C
saving, P(t) = number of pillars in
150 Dagmar’s account, and V(t) = real value
(in constant day-zero pillars) of money in
account after t days.
x
P(t) = 50t (assuming continuous
10 depositing)
V(t) = P(t)(0.50.005 t ) = 50t(0.50.005 t )
Let θ be the angle of incidence. ii. The graph shows a maximum V(t) at t ≈
By the minimal path property, the cost is 289 days.
minimized when
V (t )
x 5
sin θ = = 6000
bridge length 13
x = 6 tan [sin− 1 (5/13)] = 2.5
C(2.5) = 122
C(10) = 151.6047… t
500
The minimum cost is $122,000, obtained by
going 7.5 km along the beach, then cutting
V′(t) =
across to the island. This path saves about
50(0.50.005 t ) + 50t[0.005(0.50.005 t )] ln 0.5
$29,600 over the path straight to the island.
V′(t) = 0 ⇔ 1 = −0.005t ln 0.5
R5. a. i. a(t) = 6t − t2, t in [0, 10] 200
t=− = 288.5390 K
a′(t) = 6 − 2t ln 0.5
a′(t) = 0 ⇔ t = 3 Dagmar’s greatest purchasing power will
a(0) = 0; a(3) = 9; a(10) = −40 be after about 289 days.
Maximum acceleration = 9 at t = 3.
R6. a. i. and ii.
Minimum acceleration = −40 at t = 10.
v


1
ii. v(t ) = (6t − t 2 ) dt = 3t 2 − t 3 + C
3
v(0) = 0 ⇒ C = 0 a
1 Speeding up
∴ v(t ) = 3t 2 − t 3
3 Slowing down
v′(t) = a(t) = t(6 − t) a

r r r
v′(t) = 0 ⇔ t = 0 or t = 6 b. i. r (t ) = (5 cosh t )i + (3 sinh t ) j
1 r r r
v(0) = 0; v(6) = 36; v(10) = −33 v (t ) = (5 sinh t )i + (3 cosh t ) j
3 r r r
Maximum velocity = 36 at t = 6. a (t ) = (5 cosh t )i + (3 sinh t ) j
r r r
1 r (1) = (5 cosh 1)i + (3 sinh 1) j
Minimum velocity = −33 at t = 10. r r
3 = 7.7154 Ki + 3.5256 K j
r r r

1
iii. s(t ) = v(t ) dt = t 3 − t 4 + C v (1) = 5.8760 Ki + 4.6292 K j
12 r r r
a (1) = 7.7154 Ki + 3.5256 K j

286 Problem Set 10-7 Calculus Solutions Manual


© 2005 Key Curriculum Press
r r r
ii. The graph shows r (1), v (1), and a(1). Concept Problems
r r
Note that a (1) = r (1) so that the
acceleration vector points directly away C1. a.
from ther origin when it is drawn from the
v (t )
head of r (1). (For an elliptical path, the 5

acceleration vector points directly toward t


the origin.) 0 4

y –5
v
a
5 Asymptote
r

x
b. v = t3 − 7t2 + 15t − 9 = (t − 1)(t − 3)2
5 10 v = 0 ⇔ t = 1, 3
Particle is stopped at 1 s and 3 s.
c. v′ = 3t2 − 14t + 15 = (3t − 5)(t − 3)
v′ = 0 at t = 5/3, 3
v(0) = −9; v   =
5 32 5
=1 ;
r  3  27 27
iii. Speed = | v (1) | = 25 sinh 2 1 + 9 cosh 2 1 =
v(3) = 0; v( 4) = 3
7.4804… ≈ 7.48 units/min
r r Maximum velocity at t = 4, minimum
a (1) ⋅ v (1) = 34 sinh 1 cosh 1 = 61.6566 K
velocity at t = 0.
The object is speeding up at t = 1, as
shown by the positive dot product and by d. v″(t) = 6t − 14
r r
the acute angle between a(1) and v(1). v″(t) = 0 ⇔ t = 7/3
r r r
r a (1) ⋅ v (1) v (1) v″(t) changes from negative to positive at
at (1) = r r t = 7/3, so there is a point of inflection at
| v (1)| | v (1)|
that point.
34 sinh 1 cosh 1 r
= v (1) e. At t = 7/3, the particle’s acceleration stops
25 sinh 2 1 + 9 cosh 2 1
r decreasing and starts increasing. Thus, the
= 1.1018… v (1) minimum acceleration is at that time.
r r
= 6.4744 …i + 5.1007… j

1 7 15
f. y = v(t ) dt = t 4 − t 3 + t 2 − 9t + C
r r r 4 3 2
( an (1) = 1.2409…i − 1.5751… j )
r r y(0) = 4 ⇒ C = 4
r | a (1) ⋅ v (1)| 61.6566 … 1 7 15
| at (1)| = r = ∴ y = t 4 − t 3 + t 2 − 9t + 4
| v (1)| 7.4804 … 4 3 2
= 8.2423… g.
r r
at (1) points in the same direction as v(1), y (t )
as indicated by the positive dot product and
r r 4
by the acute angle between a(1) and v(1),
so the object is speeding up at about
8.24 units/min2.
1 t
iv. Distance = ∫ 0
ds 0 4

1
= ∫
0
25 sinh 2 t + 9 cosh 2 t dt h. y′(t) = v(t) = 0 when t = 1, 3.
5 7 8
= 4.5841… (numerically) ≈ 4.58 units y(0) = 4; y(1) = ; y(3) = ; y( 4) =
r r r 12 4 3
v. r (t ) + v (t ) = (5 cosh t + 5 sinh t )i Maximum y at t = 0, minimum y at t = 1.
r
+ (3 sinh t + 3 cosh t ) j 5
i. y ′′ = v ′ = (3t − 5)(t − 3) = 0 at t = , 3
Note that the y-coordinate is 0.6 times the 3
5
x-coordinate, so the head lies on y = 0.6x, y″ changes sign at t = and at t = 3, so there
one asymptote of the hyperbola. 3
are points of inflection at these values of t.

Calculus Solutions Manual Problem Set 10-7 287


© 2005 Key Curriculum Press
j. At t = 5/3, the particle stops accelerating and By the law of cosines,
starts decelerating, so the velocity at that time L 2 = x 2 + 252 − 2 · x · 25 · cos θ
is a local maximum. At t = 3, the particle
dL dx dx dθ
stops decelerating and starts accelerating, so 2L = 2x − 50 cos θ + 50 x sin θ
the velocity is a local minimum. dt dt dt dt
π 1
k. y is never negative because its minimum At t = 10, x = 18 cm, θ = , cos θ = ,
value is 5/12 at t = 1. 3 2
4

∫ v ( t ) = y( 4 ) − y( 0 )
3
l. Displacement = sin θ = .
0 2
8 4
= − 4 = − ft So L = 182 + 252 – 25 ⋅ 18 = 499
3 3
4
1 

5 dL 1
m. Distance = | v(t )| dt = 5 ft = −18 ⋅ 0.7 + 25 ⋅ ⋅ 0.7
0 6 dt 499  2
1 4

1 1
n. vav = v dt = ⋅ (displacement) = − ft/s 3 π
4 0 4 3 + 25 ⋅ 18 ⋅ ⋅
4 2 30 

1 1 35
o. |v|av = |v| dt = ⋅ (distance) = ft/s
4 0 4 24 = 1.6545… cm/s
C2. Assume that the maximum g a human can C4. a. Let t = time since vertex of cone touched
withstand is A and that the distance from water, y = distance from vertex of cone to
New York to Los Angeles is D km. bottom of cylinder (0 ≤ y ≤ 15), h = altitude
Recall that 1 g = 9.81 (m/s)/s. of submerged part of cone, r = radius of
For the fastest trip, the passenger accelerates at submerged part of cone, and D = depth of
A g for the first D/2 km, then decelerates at −A g water in cylinder.
for the last D/2 km. dy dD
Starting at rest, the velocity t seconds after Know: = −2 cm/min . Want: .
dt dt
leaving New York, when accelerating at
the maximum rate, is v(t) = A · 9.81 t Volume of water is 15 · 72π = 735π cm3.
1
and the distance from New York is Volume of submerged part of cone is πr 2 h.
1 3
s(t ) = A ⋅ 9.81 t 2 .
2 Volume of submerged part of cone plus water
The passenger reaches the halfway point of is π · 72 · D.
1
the trip when s(t ) = 1000 ⋅ D (because D is km 1 2
2 ∴ 49πD = 735π + πr h
and s is m), so the first half of the trip takes 3
1 2
t=
1000 D
seconds. By symmetry, the second 49 D = 735 + r h
9.81 A 3
5
half takes exactly as long, so the minimum time But D = h + y, and r = h, so
1000 D 12
for the trip is t = 2 seconds. 1 25
9.81 A 49 D = 735 + ⋅ ( D – y)3
For example, suppose that it is 4000 km from 3 144

( D – y)2 
New York to Los Angeles and that the human dD 25 dD dy 
49 = −
body can withstand A = 5 g. Then the minimal dt 144  dt dt 

time is t = 2
1000( 4000)
= 571.1372 … , or Find D when y = 10.
9.81(5) 25
about 9.5 min. 49 D = 735 + ( D – 10)3
432
C3. Let x = distance from center along clock hand, Solving numerically gives D ≈ 15.1624… .
L = length of web, and θ = central angle.
dx dθ π Substitute this for D, 10 for y, and −2 for
Know: = −0.7 cm/s; = rad/s. dy/dt.
dt dt 30
(5.1624 K)2 
dL dD 25 dD 
Want: at t = 10 s. 49 = +2
dt dt 144  dt 

288 Problem Set 10-7 Calculus Solutions Manual


© 2005 Key Curriculum Press
Solving algebraically or numerically gives T′(θ ) = 0 ⇔ cos θ ( 1 − 4 sin θ ) = 0
dD cos θ = 0 only for values of θ outside the
= 0.2085… ≈ 0.21 cm/min . domain.
dt
∴ 1 − 4 sin θ = 0 ⇔ θ = sin− 1 0.25
b. When the cone is completely submerged, the
T(θ ) approaches positive infinity as θ approaches
total volume is
either end of the domain. So T(θ ) is a minimum
1
735π + π ⋅ 52 ⋅ 12 = 835π . for θ = sin− 1 0.25.
3 Optimal trip takes T(sin− 1 0.25)
835π
In this case, D = = 17.0408… . 100 2
49π = [0.25 – 2(0.25)2 ]−1 = 266 s,
3 3
When the cone first becomes completely 2
submerged, or 4 minutes 26 seconds.
3
y = 17.0408… − 12 = 5.0408… .
Thus when y = 1 the cone is already Chapter Test
completely submerged and the depth D is not
T1. If the acceleration and velocity have the same
changing.
sign, the object is speeding up. If the acceleration
C5. Let h(t) = f(t) − g(t), so h′(t) = f ′(t) − g′(t). Then and velocity have opposite signs, the object is
h(a) = h(b) = 0 because f(a) = g(a), f(b) = g(b). slowing down.
By the mean value theorem (or Rolle’s theorem), 60
there exists an x = c in (a, b) such that T2. Displacement = ∫ 0
y dx
h( b ) – h( a ) 20 + 10 25 + 15
h ′( c ) = = 0. = 8⋅ + 15 ⋅ 0 − 10 ⋅
b–a 2 2
But because h′(c) = f ′(c) − g′(c) at time c, = 120 + 0 − 200 = −80 ft
f ′(c) = g′(c), so the knights have the same 60

velocity at this time. Distance = ∫ 0


| y| dx = 120 + 0 + 200 = 320 ft
C6. Let L = length of track, z = vertical coordinate of T3. Average value of f ( x) ≈ 2.8. The graph shows
a point on the track, and T( θ ) = number of equal areas above and below y = 2.8, and the
minutes to reach the top. point x = c where f ( c) ≈ 2.8 by the mean value
v = 30 − 60 sin θ theorem for integrals.
Domain of θ is 0 ≤ θ ≤ π / 6 because v would be f (x )
negative for acute angles θ > π /6. Equal areas

Because θ is constant, v is constant. Thus, 5


Average ¯ 2.8
L L
T (θ ) = = .
v 30 – 60 sin θ
To find L for any value of θ, consider z to be an
independent variable. By trigonometry, x
c 5 10
dL
= csc θ , and thus dL = csc θ dz. r r r
dz T4. r = 7i + 3 j
1000
L= ∫
z =0
csc θ dz = 1000 csc θ (θ is constant) The velocity vector points in the negative
x-direction and the acceleration vector makes an
(Another way to find L is to “unroll” the track obtuse angle with the velocity vector, indicating
into a vertical plane. Because the track always that the object is slowing down.
makes an angle of θ with the horizontal, this y

will result in a right triangle with hypotenuse = →


v

L, altitude = 1000, and base angle θ. Then 5 →


sin θ = 1000/L so that L = 1000 csc θ.) a

1000 csc θ
∴ T (θ ) =
30 – 60 sin θ
100
(sin θ – 2 sin 2 θ ) −1
x
=
3 5 10

100
T′(θ ) = − (sin θ − 2 sin 2 θ ) −2 ⋅ T5. v = t + 60
3 25


1
(cos θ − 4 sin θ cos θ ) Displacement = ( t + 60) dt = 1583 ft
0 3

Calculus Solutions Manual Problem Set 10-7 289


© 2005 Key Curriculum Press
T6. Because v(0) is negative and a(0) is positive,
the object is slowing down.
3 θ θ
x 7–x
T9.
7
t a aav vend vav displ end
By the minimal path property, 0 4 — 50 — 0
360
sin θ = = 0.45. 7 6 5 85 67.5 472.5
800
14 10 8 141 113 1263.5
∴ θ = sin− 1 0.45
21 13 11.5 221.5 181.25 2532.25
x = 3 tan θ = 3 tan (sin− 1 0.45) = 1.5117…
The object traveled about 2532.25 cm.
So the cheapest path is 7 − x = 5.4882… mi
along the road, then turning toward Ima’s house. Average velocity was about
1
This path costs ⋅ (2532.25) = 120.583K ≈ 120.6 cm/s.
21
360 ⋅ 5.4882 K + 800 ⋅ 9 + 1.5117K2 r r r
= 4663.2685… ≈ $4663, which is about $257 T10. r (t ) = (10 cos 0.4t )i + (10 sin 0.6t ) j
r r r
cheaper than the proposal. v (t ) = ( −4 sin 0.4t )i + (6 cos 0.6t ) j
r r r
T7. f(x) = x 3 − 4x + 5, x ∈ [1, 3] T11. a (t ) = ( −1.6 cos 0.4t )i + ( −3.6 sin 0.6t ) j
r r r
f ′(x) = 3x 2 − 4 T12. r (2) = (10 cos 0.8)i + (10 sin 1.2) j
f ′( x ) = 0 ⇔ x = ± 4/3 = ±1.1547K = 6.9670 Ki + 9.3203K j
r
(The negative value is out of the domain.) The graph shows r (2).
f(1) = 2 y
v
f(1.1547…) = 1.9207… , the minimum.
t=2
f(3) = 20, the maximum. r

1 3 3
Average =
2 1 ∫
( x – 4 x + 5) dx = 7
5
a

The graph shows a minimum at x = 1.1547… ,


a maximum at x = 3, and an average of 7. The
x
area of the rectangle of altitude 7 equals the area 0 5
of the region under the graph. r r r
T13. v (2) = ( −4 sin 0.8)i + (6 cos 1.2) j
f (x) r r
20 = −2.8694 Ki + 2.1741K j
See the graph in Problem T12.
7 The velocity vector is tangent to the path,
x pointing in the direction of motion.
r r r
T14. a (2) = ( −1.6 cos 0.8)i + ( −3.6 sin 1.2) j
1 3
r r
= −1.1147Ki − 3.3553K j
T8. a. v(t) = 10(0.5 − 2− t)
2 See the graph in Problem T12.
Distance = ∫ 0
| v(t ) | dt = 3.6067K ≈ 3.61 ft r r
T15. a (2) ⋅ v (2)
(exactly 2.5/ln 2) = 6.4 cos 0.8 sin 0.8 − 21.6 cos 1.2 sin 1.2
2 = −4.0963… ≈ −4.10 (mi/h)2/h
Displacement = ∫ v(t ) dt
0
r
| v (2) | = 16 sin 2 0.8 + 36 cos 2 1.2
= −0.8202 K ≈ −0.82 ft = 3.6000… mi/h
(exactly 10 − 7.5/ln 2) r r
a (2) ⋅ v (2)
P= r = −1.1378K ≈ −1.14 (mi/h)/h
b. a(0) = v′(0) = 10 ⋅ 2− 0 ln 2 = 10 ln 2 | v (2) |
= 6.9314… ≈ 6.93 (ft/s)/s r P( 2 ) r r r
at (2) = r v (2) = 0.9069Ki − 0.6871K j
c. v(0) = −5 | v (2) |

290 Problem Set 10-7 Calculus Solutions Manual


© 2005 Key Curriculum Press
r r r r
an (2) = a (2) − at (2) T17. Object is slowing down at | at (2)| = | P(2)| =
r r 1.1378K (mi/h)/h.
= −2.0216 Ki − 2.6681K j r
T18. an (2) points inward to the concave side because
v r
an is the component of acceleration that pulls the
object out of the straight path into a curve.
r
T19. dL = dx 2 + dy 2 = | v (t ) | dt
t=2

at
= 16 ⋅ sin 2 0.4t + 36 ⋅ cos 2 0.6t dt
2

an
L= ∫ dL = 10.0932K (numerically) ≈ 10.09 mi
0
a
T20. Answers will vary.
r
T16. The tangential component at (2) has direction the
r r
opposite of v(2), so v is decreasing and the object
is slowing down at t = 2.

Calculus Solutions Manual Problem Set 10-7 291


© 2005 Key Curriculum Press
Chapter 11—The Calculus of Variable-Factor Products

Problem Set 11-1 dW =  30 − y dy


2
 7 
1.
70
 30 – 2 y dy = 1400 mi-tons

F
W=
0  7 
(x , F )
10
b. W total = 90 tons · 70 mi = 6300 mi-tons
∆x = 0.2
Excess energy becomes kinetic energy of
x rocket and spent fuel.
4
3. Hooke’s law: F = k · s
F ≈ F(4) = 80e− 2 = 10.8268… ≈ 10.83 lb dW = ks ds
in the strip. 10

W ≈ F(4) · ∆x = 16e− 2 = 2.1653… ≈ 2.17 ft-lb W= ∫0


ks ds = 50 k

2. dW = 20xe− 0.5 x dx 4. a. F = −x 3 + 6x 2 − 12x + 16


7
3. W = ∫
0
20 xe −0.5 x dx = 69.1289K The graph starts at the high force of 16 lb,
levels off, then drops to F = 0 at x = 4.
(exactly 80 − 360e− 3.5 ) F
4. W ≈ 69.13 ft-lb 15

5. The amount of work done from x = 0 to x = b is


b
W= ∫
0
20 x −0.5 x dx
x

= −40 be −0.5b − 80e −0.5b + 80 4

lim W = 0 + 0 + 80 = 80 ft-lb b. dW = F dx = (−x3 + 6x2 − 12x + 16) dx


b→∞
− 0.5 b 4


(Use l’Hospital’s rule for be .)
W= (– x 3 + 6 x 2 – 12 x + 16) dx = 32 ft-lb
0
5.
Problem Set 11-2
2 17
Q1. 2 Q2. 10
3 10

Q3. v(t) = ln | sec t | + C Q4. a( t) = t− 1 (x, y )


15
Q5. Fundamental theorem of calculus
Q6. Riemann dV = π x2 dy
Q7. Integration by parts ∴ dW = (17 − y)(62.4)(π x2 dy)
Q8. Implicit differentiation By similar triangles, x = 1.5y.
Q9. Heaviside method Q10. A ∴ dW = (17 − y)(62.4)(π · 2.25y2 dy)
= 140.4π (17y2 − y3) dy
1. Ignore the weight of the rope. 10
Let y = the distance from the bottom of the well.
Slope of linear function is −8/50 = −0.16.
W= ∫0
dW = 1,396, 752.0937...

Weight = 20 − 0.16y ≈ 1.4 million ft-lb (exactly 444,600π)


dW = (weight) dy = (20 − 0.16y) dy 6. dV = π x2 dy
50 At x = 4, y = 16.
W= ∫
0
(20 – 0.16 y) dy = 800 ft-lb ∴ dW = (26 − y)(54.8)(π x2 dy)
2. a. Let y = number of miles up. Because y = x2,
2 dW = (26 − y)(54.8)(π y dy)
Slope of linear function is −20/70 = − . = 54.8π (26y − y2) dy
7
2 16
Weight = 30 − y (tons)
7
W= ∫0
dW = 337,891.2751… ≈ 337,891 ft-lb

292 Problem Set 11-2 Calculus Solutions Manual


© 2005 Key Curriculum Press
7. a. Draw x- and y-axes with the origin at the Initial condition V = 1 at p = 1000 ⇒
center of the sphere. To fill the sphere half k1 = 1000.
full, the water must be pumped from −120 to ∴ dW = 1000V − 1.4 dV
y, where y is negative. Integration is from 10
y = −20 to y = 0. W= ∫ 1
1000V −1.4 dV ≈ 1504.7320 K
dV = π x2 dy ≈ 1504.7 in.-lb (exactly 2500(1 − 10− 0.4 ))
x 2 + y 2 = 202 ⇒ x 2 = 400 − y 2
b. Initial condition p = 15 at V = 10 ⇒
∴ dW = [y − (−120)](62.4)[π (400 − y2) dy]
k2 = 15 ⋅ 101.4
= 62.4π (y + 120)(400 − y2) dy
0
dW = 15 ⋅ 101.4 V − 1.4 dV
W= ∫−20
dW = 117, 621, 229
W= ∫
1

10
15 ⋅ 101.4 V −1.4 dV ≈ − 566.9574 K
≈ 117.6 million ft-lb So about 567 in.-lb of work is done in
(exactly 62.4π · 600000) compressing the gases (exactly
b. For filling the tank, the limits of 37.5 ⋅ 101.4 (10− 0.4 − 1)).
integration are from −20 to 20. (Mathematically, the work is negative because
20 the force is positive and dx is negative.
∫−20
dW = 250, 925, 288.4 K Physically, the work is negative because
≈ 250.9 million ft-lb energy is taken out of the surroundings to put
(exactly 62.4π · 1,280,000) into the gases. Positive work indicates that
This answer can be found quickly by lifting energy is put into the surroundings by the
the entire weight of the water through the expanding gases.)
distance the center of the sphere moves, c. Net amount of work ≈ 1504.7320… −
namely 120 ft. 566.9574 = 937.7746… ≈ 937.8 in.-lb

W = (62.4)  (π ⋅ 20 3 )(120)
4 d. Carnot (kar-NO), Nicolas Léonard Sadi,
 3 1796−1832, was a French physicist and a
= 62.4π ⋅ 1, 280, 000 pioneer in the field of thermodynamics.

(Note that the amount of work to fill the


entire tank is more than twice the amount Problem Set 11-3
needed to half-fill it. The work to fill the top
Q1. 2 cm3 Q2. 3 cm3
hemisphere is greater than that to fill the
bottom hemisphere because the same amount Q3. Q4.
y
of water has to be lifted through a greater
displacement.) y x
1 x 1
8. Slice the water horizontally. Pick sample point
π
(x, y) on the curve y = 0.0002x4 within the
slice.
dV = 15 · 2x · dy = 30(5000y)1/4 dy Q5. Q6.
= 300(0.5y)1/4 dy y y

∴ dW = (30 − y)(67)[300(0.5y)1/4 dy]


= 20100(30 − y)(0.5y)1/4 dy 1
16


x x
W= dW ≈ 9,134, 602 ft-lb
0
 1/4  4 Q7. (mass)/(volume) Q8. (force)(displacement)
 exactly (20100)(0.5)  540  
 9  1
Q9. Q10. B
9. a. If x is the distance between the piston and the 1 – x2
cylinder head and F is the force exerted by the
1. a. The graph shows y = ln x, rotated about
hot gases, then dW = F dx.
x = 0, showing back half of solid only.
F = pA, where p is the pressure and A is the
area of the piston. y

∴ dW = pA dx 1 (x, y)
A dx = dV x

∴ dW = p dV 1 3

p = k 1V − 1.4

Calculus Solutions Manual Problem Set 11-3 293


© 2005 Key Curriculum Press
Slice the region parallel to the axis of b. Slice perpendicular to the axis of rotation.
rotation, generating cylindrical shells. Pick ρ = ky 2
sample point (x, y) on the curve, within the dm = ρ dV = ky 2 ⋅ π ( 9 − y) dy
9


slice.
m= dm = 546.75π k
ρ = kx− 1 0
dm = ρ dV = (kx− 1)(2π x ln x dx)
c. Slice parallel to the axis of rotation.
= 2π k ln x dx
3
ρ = k(1 + x)
m= ∫1
2π k ln x dx ≈ 8.1419K k dm = ρ ⋅ 2 π xy dx = 2π kx(1 + x)(9 − x2) dx
3
(exactly 2π k(3 ln 3 − 2)) m= ∫0
dm = 105.3π k
b. Slice perpendicular to the axis of rotation,
generating plane washers. d. The solid in part b has the largest mass.
ρ = 5 + 2y 4. a. The graph shows y1 = x and y2 = 0.5x,
dm = ρ dV = (5 + 2y) ⋅ π ( 32 − x2) dy intersecting at (0, 0) and (4, 2), rotated about
= π ( 5 + 2y)(9 − e2y) dy the x-axis, showing back half of solid only.
ln 3
m= ∫0
dm ≈ 108.1103K
2
y

(exactly π [36 ln 3 + 9 (ln 3) − 16]) 2 y1


y2
2. The graph shows y = sin x, rotated about the x
4
y-axis, showing back half of solid only.
y

1
x
π Slice perpendicular to the axis of rotation,
generating plane washers.
Pick sample points (x, y1) and (x, y2).
Slice the region parallel to the axis of rotation, ρ = kx
generating cylindrical shells. dm = ρ dV = kx ⋅ π ( y12 − y22 ) dx
ρ = kx
dm = ρ dV = kx ⋅ 2π xy dx = 2π kx 2 sin x dx = kxπ ( x − 0.25 x 2 ) dx
π


m= dm ≈ 36.8798K k (exactly 2π (π 2 − 4)k ) 16
m= π k = 16.7551K k
0 3
3. a. The graph shows y = 9 − x2, rotated about the b. Slice parallel to the axis of rotation,
y-axis. generating cylindrical shells.
9 y Pick sample points (x1, y) and (x2, y).
ρ = ky 2
(x, y )
dm = ρ dV = ky 2 ⋅ 2π y(x2 − x1) dy
= 2 π ky3(2y − y2) dy
2


64
x
m = dm = π k = 13.4041K k
3 0 15
5. a. Prediction: The cone on the left, with higher
Slice the region perpendicular to the axis of density at its base, has greater mass because
rotation, generating plane disks. higher density is in the larger part of the cone.
ρ = k,
b. Set up a coordinate system with the origin
dm = k dV = k ⋅ π x2 dy = k ⋅ π ( 9 − y) dy
9
at the center of the base. Slice each cone
m= ∫ dm = 40.5πk
0
perpendicular to its axis, generating plane
disks.
Or: Slice parallel to the axis of rotation. Pick sample point (x, y) on the element of
dm = k ⋅ 2π xy dx = 2π kx(9 − x2) dx the cone, y = 6 − 2x.
3
m= ∫0
dm = 40.5π k dV = π x 2 dy = π ( 3 − 0.5y)2 dy
For the cone on the left, ρ = 80 − 5y.
Or: Volume of paraboloid is half the volume dm = (80 − 5y) · π(3 − 0.5y)2 dy
of the circumscribed cylinder, or 6

0.5(π ⋅ 32)(9) = 40.5π, so m = 40.5π k. m= ∫ dm = 1305π oz


0

294 Problem Set 11-3 Calculus Solutions Manual


© 2005 Key Curriculum Press
For the cone on the right, ρ = 50 + 5y. ρ is given in the table in the text.
dm = (50 + 5y) · π ( 3 − 0.5y)2 dy π
6
dV = π 0.52 dy = dy

4
m= dm = 1035π oz
π
dm = ρ dV = ρ dy
0
∴ the cone on the left has the greater mass, as 4
predicted in part a. π 2
6. a. Prediction: The cylinder on the left, with
m= ∫
4 0
ρ dy
higher density at walls, has greater mass Simpson’s rule cannot be used because there is
because higher density is in the larger part an odd number of increments. Use the trapezoidal
of the cylinder. rule.
π
m ≈ (0.4) (10) + 9.9 + 9.8 + 9.6 + 9.4 + (9.0)
b. Set up a coordinate system with the origin at 1 1
the center of the bottom base. Slice each 4 2 2 
cylinder parallel to its axis, generating = 4.82π ≈ 15.14 g
cylindrical shells. Pick sample point (x, 6). 10. a. The graph shows y1 = 4 − x2 and y2 =
dV = 2π x · 6 · dx = 12π x dx 4x − x2, intersecting at (1, 3), rotated about
For the cylinder on the left, ρ = 50 + 10x. the y-axis, showing back half of solid only.
dm = (50 + 10x) · 12π x dx
4 y
= 12π (50x + 10x2) dx (x, y 1)

3
m= ∫
0
dm = 3780π oz (x, y2)

For the cylinder on the right, ρ = 80 − 10x. x


dm = (80 − 10x) · 12π x dx 1
= 12π ( 80x − 10x2) dx
3 Slice parallel to the axis of rotation,
m= ∫
0
dm = 3240π oz generating cylindrical shells.
Pick sample points (x, y1) and (x, y2).
∴ the cylinder on the left has the greater
mass, as predicted in part a.
ρ = kx, dV = 2π x (y1 − y2) dx
= 2π x (4 − 4x) dx
7. y1 = 4 − 2x2 and y2 = 3 − x2, rotated about dm = ρ dV = 2π k x2(4 − 4x) dx
the x-axis. 1


2
The graphs intersect at (1, 2) in Quadrant I. m = dm = π k = 2.0943K k
0 3
Slice perpendicular to the axis of rotation, b. The graph shows the curves in part a, rotated
generating plane washers. about the x-axis, showing back half of solid
Pick sample points (x, y1) and (x, y2). only.
ρ = kx 2, dV = π ( y12 − y22 ) dx y
4
dm = ρ dV = π kx2(7 − 10x2 + 3x4) dx
(x, y )
1


16
m = dm = π k = 2.3935K k (x, y 2)
0 21
8. Rotate the region in Problem 7 about the y-axis. x
1
Slice the region parallel to the axis of rotation,
generating cylindrical shells.
Pick sample points (x, y1) and (x, y2).
ρ = e − x , dV = 2π x(y1 − y2) dx = 2π x (1 − x2) dx
dm = ρ dV = 2πxe − x (1 − x 2 ) dx
1

∫ dm = 0.9444K (exactly 2π (14e


−1
m= − 5)) Slice perpendicular to the axis of rotation,
0 generating plane washers.
9. Set up axes with the origin at the center of the Pick sample points (x, y1) and (x, y2).
lower base and the y-axis coaxial with the ρ = kx, dV = π ( y12 − y22 ) dx
cylinder’s axis.
Slice perpendicular to the axis of the cylinder, dm = ρ dV = π kx(16 − 24x2 + 8x3) dx
1
generating plane disks of constant radius 0.5. m= ∫ dm = 3.6πk = 11.3097Kk
0

Calculus Solutions Manual Problem Set 11-3 295


© 2005 Key Curriculum Press
c. The region is rotated about the y-axis as Let x = r sin θ.
in part a. Then dx = r cos θ dθ , r 2 – x 2 = r cos θ .
Slice perpendicular to the axis of rotation,
generating plane disks. x = r ⇒ θ = sin −1 1 = π /2
π /2


Pick sample points (x1, y) and (x2, y). ∴ m = 4π k r 2 sin 2 θ ⋅ r cos θ ⋅ r cos θ dθ
Below y = 3, disks have radius x2. 0
Above y = 3, disks have radius x1. π /2

ρ = ky
= 4π r 4 k ∫ 0
sin 2 θ cos 2 θ dθ
π /2
For x in [0, 3], dV = π x 22 dx
= π (2 − 4 – y ) dy. 2
= πr 4 k ∫ 0
sin 2 2θ dθ

(half-argument property)
For x in [3, 4], dV = π x12 dx = π ( 4 − y) dy. π /2


1 4
3 = πr k (1 – cos 4θ ) dθ
m= ∫ ky ⋅ π (2 − 4 – y ) dy 2
2 0
0
(half-argument property)
4
+ ∫ 3
ky ⋅ π ( 4 − y) dy
=
1 4 
π r k θ − sin 4θ 
1
π /2
=
1 2 4
π r k
2  4  0 4
14 2
=1 π k + 1 π k = 3.6π k = 11.3097K k
15 3 c. Slice into spherical shells. Pick a sample
point on the x-axis within the shell.
(Coincidentally, this answer equals the answer
Then x is the radius of the shell and 4π x 2
to part b.)
is the area of the shell at the sample point.
11. a. The graph shows a sphere with origin at its
∴ dV = 4π x2 dx
center.
ρ = kx
r y
(x, y )
dm = ρ dV = 4π kx3 dx
r


r
x m = 4π k x 3 dx = π kx 4 0 = π kr 4
0
r
12. Assume Earth is spherical, with radius
3960 mi = 3960 ⋅ 5280 ⋅ 12 ⋅ 2.54 cm =
637,300,224 cm, and slice into spherical
Slice the upper semicircular region shells with radius x and dV = 4π x2 dx.
perpendicular to the x-axis and rotate it to get 8x
plane disks. ρ = 12 − g/cm 3
637300224
Pick a sample point (x, y).
 32π x 3 
Equation of the circle in the xy-plane is dm = ρ dV =  48π x 2 −  dx
x 2 + y 2 = r2.  637300224 
ρ = k|x|, dV = π y 2 dx = π (r2 − x2) dx 637300224

dm = ρ dV = k|x| π (r2 – x2) dx m= ∫0


dV
r
m = πk ∫ | x | (r 2 − x 2 ) dx
637300224
 8π x 4 
–r = 16π x 3 − 
r  637300224  0
= 2π k ∫ 0
(r 2 x – x 3 ) dx = 8π ⋅ 6373002243 ≈ 6.505 × 1027 g
r Mass is about 6.505 × 1021 metric tons!
= 2π k  r 2 x 2 − x 4 
1 1 1 4
πr k = 13. The graph shows y = ex, from x = 0 to π/2,
2 4  0 2
rotated about the y-axis, showing back half of
b. Slice the right semicircular region parallel to
solid only.
the y-axis, and rotate it to get cylindrical
y
shells coaxial to the y-axis.
ρ = kx, dV = 2π x ⋅ 2 y ⋅ dx = 4π x r 2 – x 2 dx
. dm = ρ dV = 4π kx 2 r 2 – x 2 dx (x, y )

1
r x
m = 4π k ∫ 0
x 2 r 2 – x 2 dx 1

296 Problem Set 11-3 Calculus Solutions Manual


© 2005 Key Curriculum Press
Slice parallel to the axis of rotation, generating –1/3 1/3
Q5. +
cylindrical shells. ( x – 2) ( x – 5)
Pick a sample point (x, y). 1 1
ρ = cos x, dV = 2π x ⋅ y ⋅ dx = 2π xex dx Q6. ln | x − 5| − ln | x − 2| + C
3 3
dm = ρ dV = cos x 2π xex dx
1/3 –1/3
π /2  π  +
∫ dm ≈ 8.6261K  exactly πeπ /2  − 1 
Q7.
m= ( x – 2) 2 ( x – 5)2
0   2 
14. a = 4 mi, b = 1 mi, c = 0.5 mi
2 2 2
Q8. g( x ) = ∫ f ( x ) dx ⇔ g′( x ) = f ( x )
 x   y  z Q9. f ′(2) > 0: increasing Q10. E
a.   + + = 1,

 a   b   c  1. a. The graph shows y = 9 − x2, rotated about
where a = 4, b = 1, c = 0.5. the y-axis.
The cross section at z = z0 < c has equation 9 y
2 2 2
 x  +  y  = 1 −  z0  . (x, y )

 a  b  c
This is the equation of an ellipse with
x
x -radius a 1 – ( z0 /c)2 and 3

y -radius b 1 – ( z0 /c)2 .
b. Slice horizontally into plane elliptical disks. Slice the region perpendicular to the axis of
The area of the cross section is rotation, generating plane disks.
π (x-radius)(y-radius) = π ab(1 − (z/c)2) dV = π x 2 dy = π (9 − y) dy
= 4π (1 − (z/0.5)2) = 4π (1 − 4z2). 9

ρ = 0.08(5280)3 e− 0.2 z lb/mi3


V= ∫0
π (9 − y) dy = 40.5π

dm = ρ dV = 0.08 ⋅ 52803e− 0.2 z ⋅ 4π (1 – 4z2) dz b. Each point in a disk is about y units from the
= 0.32 ⋅ 52803π e− 0.2 z(1 – 4z2) dz xz-plane, where y is at the sample point (x, y).
0.5
m= ∫
0
dm ≈ 0.32 ⋅ 5280 3 ⋅ 1.008953K dMxz = y dV = π (9y − y2) dy
9
≈ 4.7525… × 1010 lb, or about
23,762,540 tons
M xz = ∫
0
π (9 y − y 2 ) dy = 121.5π

121.5π
(exactly 0.32 · 52803π (1100e− 0.1 − 995)) c. y ⋅ V = M xz ⇒ y = =3
40.5π
2 4 x = z = 0 by symmetry.
c. Volume of semi-ellipsoid = π abc = π mi 3
3 3 The centroid is at (0, 3, 0).
3 4
Weight = 0.08 ⋅ 5280 ⋅ π 2 2
2. a.   +   = 1 ⇒ y 2 = 25 1 −
x y 1 2
3 x
 12   5   144 
≈ 49,326,507,160 lb,
1,801,427,783 lb more (≈ 3.8% more than Slice the ellipsoidal region above the x-axis
actual) perpendicular to the x-axis, generating plane
c disks as the region rotates.
d. V = 2 ∫0
πab[1 − ( z/c)2 ] dz Pick sample point (x, y).

dV = πy 2 dx = 25π 1 −
c 1 2
= 2πab  z − ⋅ c( z/c)3 
1 x dx
 144 
 3  0
12
1 – 1 x 2  dx
= 2πab  c = πabc, Q .E.D .
2
3  3
4 V = 25π ∫ 0  144 
12
= 25π  x −
1 3
x = 200π
 432  0
Problem Set 11-4
2
This answer equals ⋅ π ⋅ 12 ⋅ 52 , which is
Q1. −5 2 = −25 Q2. (−11)2 = 121 3
Q3. sin 2x = 2 sin x cos x expected because the volume of a (whole)
1 4
Q4. cos 2 x = (1 + cos 2 x ) ellipsoid is V = πabc.
2 3

Calculus Solutions Manual Problem Set 11-4 297


© 2005 Key Curriculum Press
b. Each point in a disk is about x units from the b. Each point in a disk is about x units from the
yz-plane, where x is at the sample point yz-plane, where x is at the sample point
(x, y). (x, y).

dM yz = x dV = 25π x 1 −
1 2
dM yz = x dm = 25πkx 2 1 −
1 2
x dx x dx
 144   144 
12


12
M yz = ∫ 0
dM yz = 900π M yz =
0
dM yz = 5760πk

900π 5760πk
c. x ⋅ V = M yz ⇒ x = = 4.5 c. x ⋅ m = M yz ⇒ x = = 6.4
200π 900πk
y = z = 0 by symmetry. y = z = 0 by symmetry.
The centroid is at (4.5, 0, 0). Center of mass is at (6.4, 0, 0).
3. a. See the graph in Problem 1. d. False. The centroid is at (4.5, 0, 0), but the
Each point in a disk is about y units from the center of mass is at (6.4, 0, 0).
xz-plane, where y is at the sample point 5. a. y = ex
(x, y), so each point has about the same
Slice the region parallel to the y-axis.
density.
dA = y dx = ex dx
ρ = ky 1/3, dm = ρ dV = k π (9y 1/3 − y 4/3) dy 2
9 A= ∫ e x dx = e 2 − 1 = 6.3890 K
m= ∫0
dm = 170.1375K k 0
Each point in a strip is about x units from the
 93 3  y-axis, where x is at the sample point (x, y).
 exactly = π 9k ∴ dMy = x dA = xex dx
 28  2

b. Each point in a disk is about y units from the My = ∫ 0


xe x dx = e 2 + 1 = 8.3890 K
xz-plane, where y is at the sample point x ⋅ A = My ⇒
(x, y). e2 + 1
dMxz = y dm = k π (9y 4/3 − y 7/3) dy x= 2 = 1.3130… (= coth 1)
9 e –1
M xz = ∫ 0
dM xz b. Strips in part a generate plane disks. Each
point in a disk is about x units from the
 94 3 
= 612.4952 K k  exactly π 9k yz-plane, where x is at the sample
 28  point (x, y).
612.4952 K k dV = πy2 dx = πe2x dx
c. y ⋅ m = M xz ⇒ y =
170.1375K k 2 π
= 3.6 (exactly)

V = πe 2 x dx = (e 4 – 1) = 84.1917K
0 2
x = z = 0 by symmetry. dMyz = x dV = πxe2x dx
2


The center of mass is at (0, 3.6, 0). 3 1
M yz = dM yz = πe 4 + π = 129.4292 K
d. False. The centroid is at (0, 3, 0), but the 0 4 4
center of mass is at (0, 3.6, 0). 3e 4 + 1
x ⋅ V = M yz ⇒ x = = 1.5373K
4. a. Slice the ellipsoid as in Problem 2. 2(e 4 – 1)
Each point in a disk is about x units from c. False. For the solid, x is farther from the
the yz-plane, where x is at the sample point yz-plane.
(x, y), so each point has about the same 6. a. Slice the region parallel to the y-axis.
density as at the sample point. dA = sec x dx
ρ = kx, dV = π y 2 dx = 25π 1 −
1 2 π /3
 144 
x dx A= ∫
0
sec x dx = ln (2 + 3 )
= 1.3169…
dm = ρ dV = 25πkx 1 −
1 2
 144 
x dx dM y = x dA = x sec x dx
π /3
12 My = ∫ x sec x dx
m= ∫0
dm = 2827.4333K k 0
= 0.7684… (numerically)
(exactly 900π k) 0.7684 K
x ⋅ A = My ⇒ x = = 0.5835K
1.3169K

298 Problem Set 11-4 Calculus Solutions Manual


© 2005 Key Curriculum Press
b. Strips in part a generate plane disks. Each c. Each point in a strip of part a is
point in a disk is about x units from the approximately x units from the y-axis, where
yz-plane, where x is at the sample x is the value in the sample point (x, y).
point (x, y). dM y = x dA = x 5/3 dx
dV = π y2 dx = π sec2 x dx 8
π /3 My = ∫ x 5/3 dx = 96
V= ∫ π sec x dx = π 3 = 5.4413K
2 0
0 96
d. x ⋅ A = M y ⇒ x = =5
dM yz = x dV = π x sec2 x dx 19.2
π /3
M yz = ∫ 0
π x sec 2 x dx = 3.5206 K y ⋅ A = Mx ⇒ y =
27.4285K
19.2
= 1.4285…
 π2 3 1  exactly 1 3 
 exactly 3 + π ln 2   7
 
Centroid is at (5, 1.4285…).
x ⋅ V = M yz ⇒ x = 0.6470 K
e. The balance point is shown on the graph.
 π ln 2 
 exactly −  y
 3 3
4
c. False. For the solid, x is farther from the
yz-plane.
7. Construct axes with the origin at a vertex and the x
8
x-axis along the base, b.
Slice the triangle parallel to the x-axis.
b 9. a. Slice the region parallel to the y-axis so that
The width of a strip is b − y.
h each point in a strip will be about x units

dA =  b − y dy
b from the y-axis, where x is at the sample
 h  point (x, y).
dA = y dx = sin x dx
dM x = y dA =  by − y 2  dy
b
π
 h  A= ∫ sin x dx = 2 (exactly)
M x =  by – y 2  dy
h 0


b
(This may be “well-known” by now.)
0  h 
dM y = x dA = x sin x dx
h
1 b 1 π
= by 2 − y 3 = bh 2
2 3h 0 6 My = ∫ 0
x sin x dx = 3.1415… = π (exactly)
1 2 π
bh
1 x ⋅ A = My ⇒ x = , Q .E .D .
y ⋅ A = Mx ⇒ y = 6 = h, Q.E.D. 2
1
bh 3 (Or just note the symmetry.)
2 b. dM 2y = x2 dA = x2 sin x dx
8. a. y = x 2/3 from x = 0 to x = 8. π
Slice the region vertically. Pick a sample M2y = ∫
0
x 2 sin x dx = 5.8696 …
point (x, y) on the graph within the strip. (exactly π 2 − 4)
(See the graph in part e.)
dA = y dx = x 2/3 dx π2 – 4
c. x 2 ⋅ A = M2y ⇒ x = = 1.7131…
8 2
A= ∫ 0
x 2/3 dx = 19.2
10. a. Set up axes with the x-axis along the base, B.
b. Slice the region parallel to the x-axis so that dM 2B = y2 dA = y2B dy
each point in a strip is about y units from the H


1
x-axis, where y is at the sample point (x, y). M2B = BH 3 y 2 B dy =
03
dM x = y(8 − x) dy = (8y − y5/2) dy b. Set up axes with the x-axis through the
4
Mx = ∫ 0
(8 y – y 5/2 ) dy = 27.4285K centroid.
dM 2c = y2 dA = y2B dy
 exactly 27 3  0.5 H


 7 1
M2c = y 2 B dy = BH 3
–0.5 H 12

Calculus Solutions Manual Problem Set 11-4 299


© 2005 Key Curriculum Press
π /2


c. Set up axes with the x-axis along the base, B. M = 4π R3 sin 3 θ ⋅ R cos θ ⋅ R cos θ dθ
dM2B = y 2 dA = y 2  B − y dy
B 0
 H  π /2
= 4π R 5 ∫ (cos 2 θ – cos 4 θ ) sin θ dθ
M2B =  By 2 – y 3  dy =
H


B 1 0
BH 3
0  H  π /2
= 4πR 5  − cos3 θ + cos 5 θ 
12 1 1
d. Use the axes in part c. The distance from the  3 5  0
centroidal axis to a sample point (x, y) is
= 4πR 5  −0 + 0 + −  = πR 5
1 1 1 8
y − H.  3 5  15
3
2 2
dM2 c =  y − H  dA =  y − H   B − y dy
1 1 B 8
πR 5
 3   3   H  r ⋅V = M ⇒ r =
2 2 15 ⇒ r = 0.4 R
4 3
2 H/3
 – 1 y 3 + 5 y 2 – 7 Hy + 1 H 2  dy πR
M2 c = B ∫– H/3  H 3 9 9  3
H 12. Assume the clay has uniform density ρ.
B 4 5 3 7 1
=− y + By − BHy 2 + BH 2 y Cylinder: H = 2 RC , V = π RC2 H = 2π RC3 = 1000
4H 9 18 9 0
1/3
RC = 
1 500 
= BH 3
cm
36  π 
11. a. Slice into cylindrical shells so that each point 1
in a shell will be about r units from the axis. Second moment of volume = π (2 RC ) RC4 = π RC5
2
The altitude of a shell is a constant, H. (from Problem 11a)
dM = r2 dV = r2 2πrH dr
R Second moment of mass

1
M = 2πHr 3 dr = πHR 4 2/3
= ρπRC5 = 500 ρ 
500 
0 2 = 14, 684.1932 … ρ
1  π 
πHR 4 1
⋅ = ⇒ = 2 ⇒r =
1/3
πRS = 1000 ⇒ RS = 
2
r V M r 2
R 4 3 750 
πR 2 H 2 Sphere: V = cm
3  π 
b. Slice the cone into cylindrical shells so that
8
each point in a shell will be about r units Second moment of volume = πRS5
from the axis. 15
H (from Problem 11c)
The altitude of a shell is H − r.
R Second moment of mass
2/3
= ρ πRS5 = 400 ρ 
750 
dM = r 2 dV = r 2 2πr  H − r dr
H 8
 R  15  π 
= 15,393.3892…ρ
Mh = 2πH  r 3 – r 4  dr = πHR 4
R


1 1
0  R  10 The sphere has higher moment of mass.
1 13. a. Set up axes with the x-axis through the
πHR 4 centroid.
r ⋅V = M ⇒ r =
2 2 10 ⇒ r = 0.3 R
1 2 dM2 = y dA = y2 · B dy
πR H 0.5 H
3 0.5 H


1 3
M2 = B y 2 dy = y
c. Slice the sphere into cylindrical shells so that −0.5 H 3 −0.5 H
each point in a shell is about r units from the 1
axis. = BH 3 , Q.E.D.
12
The equation of the sphere is r2 + y2 = R2. (Same answer as in Problem 10b)
The altitude of a shell is 2y. b. i. B = 2, H = 12; M 2 = 288;
dM = r 2 dV = r 2 2πr ⋅ 2 R 2 – r 2 dr stiffness = 288k
R ii. B = 12, H = 2; M 2 = 8; stiffness = 8k
M = 4π ∫
0
r 3 R 2 – r 2 dr A board on its edge is 36 times stiffer.
Let r = R sin θ. c. i. Set up axes with the x-axis through the
dr = R cos θ dθ , R 2 – r 2 = R cos θ centroid.
π From y = 0 to y = 2, dM 2 = y2 · 2 dy.
r = 0 ⇒ θ = 0, r = R ⇒ θ = From y = 2 to y = 4, dM 2 = y2 · 4 dy.
2

300 Problem Set 11-4 Calculus Solutions Manual


© 2005 Key Curriculum Press
4


y
By symmetry, M2 = 2 dM2 r
0
2 4
=2 ∫
0
2 y 2 dy + 2 ∫ 2
4 y 2 dy = 160. f (r )

Stiffness = 160k r
ii. From y = 0 to y = 4, dM 2 = y2 · 1 dy. a b

From y = 4 to y = 6, dM 2 = y2 · 4 dy.
6 Let f(r) be the length of the strip or the sum of
By symmetry, M2 = 2 ∫
0
dM2 the lengths if the region has S-shaped parts.
4 6 Let A be the area of the region.
=2 ∫ y dy + 2 ∫ 4 y dy = 448.
2 2
dMy = r dA = r f (r) dr
0 4 b
Stiffness = 448k (2.8 times stiffer!) My = ∫ a
r f (r ) dr
d. Increasing the depth does seem to increase Rotate the region about the y-axis. The strips
stiffness greatly, but making the beam very generate cylindrical shells.
tall would also make the web very thin,
dV = 2π r f (r) dr
perhaps too thin to withstand much force. b b
14. a. dA = y dx = x 3 dx
2
V= ∫ a
2πr f (r ) dr = 2π ∫
a
r f (r ) dr

A= ∫ x dx = 4 = 2π M y
3
0
But My also equals r ⋅ A.
b. dV = 2π x · y · dx = 2π x4 dx ∴ V = 2πrA = (2πr )( A)
= (distance traveled by centroid)(area of region),
2
V= ∫
0
2π x 4 dx = 12.8π
Q .E .D .
c. dM y = x dA = x4 dx
2 Problem Set 11-5
My = ∫ 0
x 4 dx = 6.4
Q1. centroid Q2. center of mass
The volume integral is 2π times the moment
integral. Q3. radius of gyration Q4. definite integration
6.4 Q5. indefinite integration (or antidifferentiation)
d. x ⋅ A = M y ⇒ x = = 1.6
4 Q6. ρ = (mass) ÷ (volume)
e. The centroid travels 2π x = 3.2π . Q7. x1/ 2 Q8. ln | sec x + tan x | + C
(Area)(Distance traveled by centroid) = Q9. y′ = (x + 1)
2 −1
Q10. A
(4)(3.2π) = 12.8π, which equals the volume. 1. a. Slice the trough face horizontally so that each
Thus, the theorem of Pappus is confirmed. point in a strip is about the same distance
15. a. Area of a small circle = π r2 below the surface as at the sample point
The centroid of the small circle is its center, (x, y).
so the distance from the axis of rotation to y = 2x 4 ⇒ x = (0.5y)1/4
the centroid is R. Thus, the theorem of p = k(2 − y), dA = 2x dy = 2(0.5y)1/4 dy
Pappus implies dF = p dA = 2k(2 − y)(0.5y)1/4 dy
V = 2π RA = 2π R(π r2) = 2π 2r2R
F = dF = 2.8444 K k  exactly
2 128 
1
b. Area of a semicircle = πr 2
0 ∫  45 
k

2 b. dM x = y dF = y · 2k(2 − y)(0.5y)1/4 dy
4 3
M x = dM x = 2.1880 K k  exactly
2 256 
Volume of a sphere = πr
3 0 ∫  117 
k
4 3
πr 2.1880 K k
4 c. y ⋅ F = M x ⇒ y =
2πr ⋅ A = V ⇒ r = 3 = r 2.8444 K k
1 2 3π
2π ⋅ πr
= 0.7692 K  exactly 
2 10
16. Pick a closed region that does not lie on both  13 
sides of the y-axis. x = 0 by symmetry.

The center of pressure is at  0,


Slice the region parallel to the y-axis so that each 10 
.
point in the strip will be about r units from the  13 
y-axis (see graph).
2. a. The graph shows y = x2, between y = 0 and
y = 100.

Calculus Solutions Manual Problem Set 11-5 301


© 2005 Key Curriculum Press
32


100 y
Mx = dM x ≈ 13, 992, 028.2564 K
0
≈ 13.992 million lb-ft
(x, y ) d. y ⋅ F = M x ⇒ y ≈ 11.6668K ft
x = 0 by symmetry.
x
Center of pressure is at about (0, 11.67) ft.
10 e. Moment of area:
Width at y = 100 ft is 2 y = 20 ft, Q .E .D .  4

1/ 4

dM = y dA = y ⋅ 40 1 −  y − 1 
1
dy
b. Slice the dam face horizontally so that each
  32  
point in a strip is the same distance below the 32
surface as the sample point (x, y).
dA = 2x dy = 2y1/2 dx
M= ∫ 0
dM ≈ 20, 071.5364 K
100 ≈ 20.07 thousand ft3

1
A= 2 y1/2 dx = 1333 ft 2 y ⋅ A = M ⇒ y ≈ 16.9150 K ft
0 3
(2/3 the area of the circumscribed rectangle) x = 0 by symmetry.
c. p = k(100 − y) with k = 62.4 lb/ft3 The centroid is at about (0, 16.92) ft.
dF = p dA = 2k(100 − y)(y1/2) dy The centroid is different from center of
100 pressure.

1
F= dF = 53, 333 k = 3, 328, 000 f. Area below waterline:
0 3
16


Force is 3,328,000 lb, or 1664 tons. Aw = dA ≈ 548.6345K ≈ 548.6 ft 2
d. dMx = y dF = 2ky(100 − y)(y1/2) dy 0
100 First moment of area below waterline:

16000000
Mx = dM x = k 16
0

= 142, 628, 571.4285K


7 Mw = ∫ 0
dM ≈ 4749.3398K ≈ 4749.3 ft 3

≈ 142.6… million lb-ft y ⋅ Aw = Mw ⇒ y ≈ 8.6566 K ft


e. y ⋅ F = M x ⇒ x = 0 by symmetry.
300 7 The center of buoyancy is at about
y= = 42 = 42.8571… ≈ 42.86 ft (0, 8.66) ft.
7 8 2 2
4. a. Equation of ellipse is   +   = 1.
3. a. Slice the bulkhead horizontally so that each x y
point in a strip is the same distance below  6   3
the surface as the sample point (x, y). b. Slice the ellipse horizontally so that each
4 4 point in a strip is y units from the surface
 x  +  y – 32  = 1
 20   32  where y is at the sample point and y is
4 1/4
negative.
  1   Surface of oil is at y = 0 ⇒ p = −50y.
x = 20 1 − y −1 
  32   x = 2 9 – y 2 , dA = 2 x dy = 4 9 – y 2 dy
4 1/ 4
 
dA = 2 x dy = 40 1 −  y − 1 
1
 32 
dy dF = p dA = −50 y ⋅ 4 9 – y 2 dy
 
= −200 y 9 – y 2 dy
32
A= ∫
0
dA ≈ 1186.6077K ≈ 1186.6 ft 2
0
b. p = 67(32 − y) F= ∫ −3
dF = 1800 lb (exactly)
dF = p dA 5. a.
1/ 4
 4

= 67(32 − y) ⋅ 40 1 −  y − 1 
1 y
dy
 
60
 32 
32
F= ∫
0
dF ≈ 1,199, 294.1645K (x, y )

≈ 1.199 million lb
c. dMx = y dF
1/ 4
 4
 x
= y ⋅ 67(32 − y) ⋅ 40 1 −  y − 1 
1
dy 10

  32  

302 Problem Set 11-5 Calculus Solutions Manual


© 2005 Key Curriculum Press
Slice the wing parallel to the y-axis. Pick 7. a. Slice the region as shown in Figure 11-5f.
sample point (x, y) within the strip. At a sample point (x, t), d(dM2x) = t2 dx dt.
π t=y
 t=y

dA = y dx = 60 cos
20
x dx dM2 x = ∫ t =0
t 2 dx dt = 
 ∫
t =0
t 2 dt  dx

10 t=y


2400 1 1
A= dA = = 763.9437K = t3 dx = y 3 dx
−10 π 3 t =0 3
≈ 763.9 ft2 1
= [0.25( x – 4) – ( x – 4)1/ 3 ]3 dx
π 3
b. dF = p dA = k (10 − | x |) ⋅ 60 cos x dx x =4

∫ dM2 x = 0.5333K  exactly 


20 8
b. M2 x =
10 x = –4  15 
F= ∫ −10
dF = 4, 863.4168K k
8. a. Slice the region parallel to the y-axis so that
 exactly 48000 k  each point in a strip will have about the same
 π2  pressure as at the sample point (x, y).
y = e− x
c. Make 4863.4168…k ≥ 96. p = kx 2, dA = (1 − e − x) dx
k ≥ 0.0197… tons/ft2 (exactly 0.002π 2 ) dF = p dA = kx2(1 − e− x) dx
ln 5
6. a. y = 100 − x2 intersects the x-axis at x = ±10.
Slice the wing parallel to the y-axis. Pick
F= ∫ 0
dF = 0.9514 K k

sample point (x, y) within the strip.  1 3 1 2 8 


 exactly  (ln 5) + (ln 5) + ln 5 −  k 
2
p = 90 − 7x   3 5 5 5 
dA = y dx = (100 − x2) dx b. Slice the region parallel to the x-axis so that
dF = p dA = (90 − 7x)(100 − x ) dx 2 each point in a strip will have about the same
10
pressure as at the sample point (x, y).
F= ∫−10
dF = 120, 000 lb (exactly) x = −ln y, p = ky − 1
dA = (ln 5 − x) dy = (ln 5 + ln y) dy
b. dMy = x dF = x(90 − 7x)(100 − x2) dx = ln (5y) dy
10 dF = p dA = ky− 1ln (5y) dy

560000
My = dM y = − lb-ft x = ln 5 ⇒ y = e − l n 5 = 0.2
−10 3
dF = 1.2951K k  exactly (ln 5)2 k 
1


1
5 F=
c. x ⋅ F = M y ⇒ x = −1 ft 0.2  2 
9
c. Slice the region parallel to the y-axis. Then
6 slice a strip parallel to the x-axis as shown in
d. p = ky = y (because p = 60 at y = 50)
5 Figure 11-5g.
dA = 2 x dy = 2 100 – y dy At sample point (x, t), p = kx2t− 1.
d(dF) = p dA = kx2t− 1 dx dt
t =1
12  t =1 −1  2
dF = p dA =
5
y 100 – y dy dF =
t=y ∫
kx 2 t −1 dx dt = 
 t=y
kt dt  x dx
 ∫
= ( k ln t tt ==1y ) x 2 dx = k (0 − ln y) x 2 dx = kx 3 dx
100
F= ∫ 0
dF = 64,000 lb (exactly)
ln 5


1
F= kx 3 dx =
k (ln 5) 4 = 1.6774 K k
12 2 0 4
e. dM x = y dF = y 100 – y dy
5 9. The integrals in Problems 7 and 8 can be written
100 in the form
Mx = ∫ dM x = 3, 657,142.8K x =b t =d
0
∫ ∫
x =a t =c
f ( x, t ) dt dx

≈ 3.657 million lb-ft  exactly


25600000 
The result is called a double integral because two
 7  integrals appear. (Hiding inside each integral is a
f. y ⋅ F = M x ⇒ second integral!)
π
10. a. y = 5 tan 2 x
y = 57.1428K ≈ 57.14 ft  exactly
400 
 8
7  π
y = 5 ⇒ tan 2 x = 1 ⇒ x = ±2
8

Calculus Solutions Manual Problem Set 11-5 303


© 2005 Key Curriculum Press
Slice the floodgate parallel to the y-axis. 2. a. v(t) = 55 + 6 t − t2
π v(0) = 55 + 6 · 0 − 02 = 55 mi/h
dA =  5 − 5 tan 2 x  dx
 8  v(3) = 55 + 6 · 3 − 32 = 64 mi/h
2 v(6) = 55 + 6 · 6 − 62 = 55 mi/h, Q .E .D .
A= ∫ −2
dA = 14.5352 K ≈ 14.54 ft 2
b. Cost of a short time, dt, at speed v is
 exactly 40 − 80  dC = 3(v − 55) dt = 18t − 3t2 dt.
 π Total ticket cost is
6
b. Slice the floodgate parallel to the x-axis so
that each point in a strip has about the same
C= ∫0
dC = 108 (exactly).
Fine should be $108.00.
pressure as at the sample point (x, y).
p = k(20 − y) with k = 62.4 lb/ft2 3. a. Cost per foot, P, = ax2 + bx + c
π 8 a · 02 + b · 0 + c = 500 ⇒ c = 500
y = 5 tan 2 x ⇒ x = tan −1 0.2 y
8 π a ⋅ 100 2 + b ⋅ 100 + 500 = 820  a = 0.002
16 ⇒
dA = 2 x dy = tan −1 0.2 y dy a ⋅ 200 2 + b ⋅ 200 + 500 = 1180  b = 3
π
16 P(x) = 0.002x2 + 3x + 500
dF = p dA = k (20 − y) ⋅ tan −1 0.2 y dy
π b. P(700) = 0.002 · 7002 + 3 · 700 + 500
5
= $3580/ft
F= ∫ 0
dF = 248.2628K k
c. Cost to dig a short distance, dx, is
= 15491.6027… ≈ 15.49 thousand lb dC = P dx = (0.002x2 + 3x + 500) dx.
  5200   Cost to dig 1000 feet is
 exactly  800 − k
 3π   1000


8
(0.002 x 2 + 3 x + 500) dx = ⋅ 1000 2.
(The force can also be found by slicing 0 3
parallel to the y-axis as in part a, then slicing Cost is about $2,666,667.
the strip horizontally and using a double d. Cost to dig 500 feet twice (once from
integral. In this case, the pressure at a each end) is
500


sample point (x, t) is 17
C=2 (0.002 x 2 + 3 x + 500) dx = ⋅ 500 2.
p = k(20 − t) 0 3
d(dF) = p dA = k(20 − t) dt dx Cost is about $1,416,667.
The first integration is from t = y to t = 5. Savings is about $1,250,000!
The second integration is from x = −2 to in.
x = 2.) 4. a. velocity · area has the units ⋅ in.2 ,
s
c. Let µ (Greek letter mu) = coefficient of which is in.3/s, correct for flow rate.
friction. b. v = 4 − x 2 ⇒ v′ = −2x
10000 v′ changes from positive to negative at x = 0.
µ ⋅ F = 10000 ⇒ µ =
15491.6027K ∴ there is a maximum flow rate at the center
= 0.6455K of the pipe where x = 0.
(Or simply observe that the graph of v is
a parabola opening downward with vertex
Problem Set 11-6 at x = −b/(2a) = 0.)
1 101 v(2) = 4 − 22 = 0, Q .E .D .
Q1. x +C Q2. 0
101 c. Slice the water in the pipe into cylindrical
Q3. x ln x − x + C Q4. 2 sin x cos x = sin 2x shells.
Q5. (force)(displacement) Q6. y′ = 3(1 + 9x2)− 1 Each point in a shell has about the same
Q7. x = −2 Q8. 2 sec2 x tan x water velocity as at the sample point x units
from the axis.
Q9. y ′′ = −9 cos 3 x Q10. D
Let F = flow rate in in.3/s.
1. Partition the interval into small subintervals dF = v dA = (4 − x2) · 2π x · dx
of width dT so that C is about the same at any = 2π (4x − x3) dx
point in a subinterval. The amount of heat, 2

dH, to raise the temperature by dT is F= ∫0


2π ( 4 x − x 3 ) dx = 8π = 25.1327…
dH = C dT = (10 + 0.3T1/2) dT. ≈ 25.13 in.3/s
900
H= ∫100
dH = 13, 200 calories (exactly) d. 25.1327… in.3/s · 60 s/min · 1 gal/231 in.3 =
6.5279… ≈ 6.53 gal/min

304 Problem Set 11-6 Calculus Solutions Manual


© 2005 Key Curriculum Press
e. 4 in./s · π · 22 in.2 = 16π = 13.0559… 7. Slice the solid into cylindrical shells so that each
≈ 13.06 gal/min (exactly twice the actual point in a shell is about the same distance from
rate) the y-axis as the sample point (x, y).
f. The problem is equivalent to finding the dM 2y = x 2 dm = x 2 · ρ dV = x 2 · k · 2π xy dx
volume of a solid of rotation by cylindrical = 2π kx3(4 − x2) dx
2


shells. The velocity takes the place of the 32
M2 y = dM2 y = π k = 33.5103K kg-cm 2
altitude of a shell. 0 3
5. a. 8. a. T(D) = 20 sin 2π D
F T(0) = 20 sin 0 = 0
600 T(1/4) = 20 sin π /2 = 20, which checks.
b. Partition the time interval into short
increments of width dD so that T is about the
same at any time in the increment as it is at
x
5
the sample point (D, T ) .
Let H = number of degree-days.
dH = T dD = 20 sin 2π D dD
b. F has a step discontinuity at x = 2. 1/4


10
c. dW = F dx H= dH = = 3.1830 K
0 π
Because the graph is linear on [0, 2], the work ≈ 3.18 degree-days
equals the area of the triangle.
9. a. m = 2000 − 5t (mass in kilograms, time in
1
W = ⋅ 2 ⋅ 600 = 600 in.-lb seconds)
2
b. a = F/m = 7000(2000 − 5t)− 1
= 1400(400 − t)− 1
5
d. W = ∫ 2
F dx
dv 1400
c. a = =
By Simpson’s rule, dt 400 – t
1
∫ ∫
1400
W ≈ (0.5)( 450 + 4 ⋅ 470 + 2 ⋅ 440 + 4 ⋅ 420 dv = dt
3 400 – t
+ 2 ⋅ 410 + 4 ⋅ 390 + 330) v = −1400 ln | 400 − t | + C
2 Assume the car starts at rest at t = 0.
= 1266 in.-lb
3 0 = −1400 ln 400 + C ⇒ C = 1400 ln 400
400
2
e. Total work ≈ 600 + 1266 = 1866 in.-lb
2 v(t ) = 1400 ln
3 3 |400 – t |
20
f. Yes, a piecewise continuous function such as d. v(20) = 1400 ln = 71.8106 K ≈ 71.81 m/s
this one can be integrable. See Problem 27 in 19
ds 400
Problem Set 9-10. v= = 1400 ln
6. a. Slice the solid into disks parallel to the dt 400 – t
20


400
xz-plane so that each point in a disk has about s= 1400 ln dt = 711.9673K
the same density as at the sample point (x, y). 0 400 – t
 20 
y = 4 − x2 ⇒ x2 = 4 − y ≈ 712.0 m  exactly 28000 1 − 19 ln  
dm = ρ dV = k · π x 2 dy = kπ (4 − y) dy   19  
4 10. Slice the tract parallel to the tracks so that each
m= ∫ 0
kπ ( 4 − y) dy = 8π k g point in the strip will have about the same
value per square kilometer as at the sample
b. Each point in a disk of part a is also about the point (x, y).
same distance from the xz-plane as the sample Let v = thousands of dollars per square kilometer
point (x, y). and W = thousands of dollars the land is worth.
Let K stand for the constant. v = kx = 200x (v = 200 at x = 1)
dF = K · dm · y− 1/2 = K · k π (4 − y) dy · y− 1 / 2 dW = v dA = 200x[(4 − x2) − (4x − x2)] dx
= K kπ (4y− 1/2 − y 1/2) dy = 800(x − x2) dx
4 The curves intersect at x = 1.

32
F = dF = Kkπ = 33.5103K Kk 1


3 1
0
W = 800( x − x 2 ) dx = 133
0 3

Calculus Solutions Manual Problem Set 11-6 305


© 2005 Key Curriculum Press
The land is worth about $133,333. 1
b. v = 10e kr , v(3) = 1 ⇒ k = − ln 10 ⇒
If all the land were worth $200,000 per km2, 3
1 v = 10e − (ln 10 ) r/3
W = 200 A = 200 ∫ (4 – 4 x ) dx = 400.
0 dW = v dA = 10e − ( ln 10 ) r/3 ⋅ 2πr dr
3


The land would be worth $400,000. W= dW = 71.4328K ≈ 71.4 million
Actual value is about $267,000 less. 0

11. a. Slice the tract parallel to the y-axis so that  18π 


dollars  exactly (9 – ln 10)
each point in a strip will be about the same  (ln 10)2 
value per square unit as at the sample
c. By Simpson’s rule,
point (x, y). 3


1
y = cos x W= v ⋅ 2πr dr ≈ (0.3)(2π ) (10 + 4 ⋅ 12
Let v = value of land per square unit and 0 3
+ 2 ⋅ 15 + 4 ⋅ 14 + 2 ⋅ 13 + 4 ⋅ 10
W = worth of the land.
+ 2 ⋅ 8 + 4 ⋅ 5 + 2 ⋅ 3 + 4 ⋅ 2 + 1)
v = kx, dA = y dx = cos x dx
= 52.2π = 163.9911… ≈ 164.0 million
dW = v dA = kx cos x dx
dollars
π /2 π
W= ∫ dA =  − 1 k = 0.5707K k d. This problem is equivalent to volume by
0 2 
cylindrical shells, where the value of the land
b. Slice the tract parallel to the x-axis so that per square unit takes the place of the altitude
each point in a strip will be about the same of the cylinder. It is also equivalent to the
value per square unit as at the sample water flow in Problem 4 of this problem set.
point (x, y). e. Answers will vary.
v = ky
dW = v dA = v · x dy = ky · cos− 1 y dy 14. a. p = 100[(x − 8)1/2 − 0.5(x − 8)]
1 π dF = p dA = 2p dx

W = dW = k = 0.3926 K k
0 8 = 200[(x − 8)1/2 − 0.5(x − 8)] dx
10
12.
y
F= ∫
8
dF = 177.1236 K ≈ 177 lb
9
 800 2 
 exactly 3 − 200
(x, y )  
b. Average pressure = total force/total area
177.1236 K
x
= = 44.2809K ≈ 44.3 lb/ft 2
3 4
 200 2 
Slice the wall parallel to the ground so that  exactly 3 − 50
each point in the slice will cost about the  
same to paint per square meter. c. dMyz = x dF = 2px dx
Let r = rate in dollars per square meter and = 200x[(x − 8)1/2 − 0.5(x − 8)] dx
C = cost in dollars to paint the wall. 10

r = ky2 = 3y2 (r = 12 when y = 2) M yz = ∫8


dM yz = 1602.8706 K ≈ 1603 lb-ft
dA = 2 x dy = 2 9 – y dy  7360 2 5600 
 exactly −
dC = r dA = 3 y ⋅ 2 9 – y dy 3 
2
 3
dC ≈ $1999.54  exactly 1999 
9


19 1602.8706 K
C= d. x ⋅ F = M yz ⇒ x =
 35 
0
177.1236 K
13. a. Let v = value of land per square kilometer, = 9.0494… ft
W = worth of the land in dollars, and Calvin should stand about 10 − 9.0494 ft
r = distance from center of town. 1
≈ 11 in. from the end.
Slice the city into circular rings of width dr so 2
that each point in a ring will be about r units 15. a. f (x) = 9 − x2 = (3 − x)(3 + x) = 0 only at
from the center. x = ±3.
v = 10 − 3r, dA = 2π r dr 1
dW = v dA = (10 − 3r ) ⋅ 2π r dr g( x ) = − x 3 − x 2 + 3 x + 9
3
3
W= ∫ 0
dW = 36π = 113.0973K 1
= − ( x – 3)( x + 3)2 = 0 only at x = ±3.
3
≈ 113.1 million dollars

306 Problem Set 11-6 Calculus Solutions Manual


© 2005 Key Curriculum Press
3


The “skewness” being zero reflects the
b. A f = (9 – x 2 ) dx = 36
–3 symmetry of this region. It is not skewed
Ag =∫  – 1 x 3 – x 2 + 3 x + 9 dx = 36
3 at all.
–3  3  i. For example, graph
To simplify algebraic integration, you could 1
g( − x ) = x 3 − x 2 − 3 x + 9.
use 3
3
Af = 2 ∫ (9 – x ) dx
2 y
New
0 graph g
3
Ag = 2 ∫ (9 – x ) dx, where the odd terms
2
0
integrate to zero between symmetrical limits.
x
Thus, the two integrals are identical. –3 3
c. The high point of f comes at x = 0.
The high point of g comes where g′(x) = 0. 16. a. y = x 2
g′(x) = −x 2 − 2x + 3 = −(x + 3)(x − 1) dL = dx 2 + dy 2 = 1 + 4 x 2 dx
g′(x) = 0 ⇔ x = −3 or x = 1
The high point is at x = 1. dM y = x dL = x 1 + 4 x 2 dx
d. Slice the region under the g graph parallel to 2

the y-axis so that each point in a strip will be b. M y = ∫ 0


dM y = 5.7577K
about the same distance from the y-axis as the  exactly 1 (17 17 – 1)
sample point (x, y).  12 
dM y = x dA = x g(x) dx 2

=  − x 4 − x 3 + 3 x 2 + 9 x  dx
1 c. L = ∫ 0
1 + 4 x 2 dx = 4.6467K
 3 
3
 exactly 1 ln ( 17 + 4) + 17 
My = ∫
–3
dM y = 21.6  4
5.7577K

x ⋅ A = My ⇒ x =
21.6
= 0.6 d. x ⋅ L = M y ⇒ x = = 1.2390 K
4.6467K
36
e. False. For the symmetrical region under the e. dS = 2π x dL = 2π x 1 + 4 x 2 dx
π
dS = 36.1769K  exactly (17 17 – 1)
2


graph of f, the centroid is on the line through
S=
the high point. But for the asymmetrical 0  6 
region under the graph of g, the high point is f. Integral for S is 2π times the integral for My!
at x = 1 and the centroid is at x = 0.6.
17. In Problem 16, R = x = 1.2390 K and
f. False
0.6
L = 4.6467… .
Area to left = ∫
–3
g( x ) dx = 17.1072 (exactly) 2π RL = 2π (1.2390…)(4.6467…) = 36.1769… ,
which equals S, Q.E.D.
3
Area to right = ∫0.6
g( x ) dx = 18.8928 (exactly) 18. The centroid of the small circle is at its center,
R units away from the axis.
(or 36 − 17.1072 = 18.8928)
The arc length L of the small circle is 2πr.
g. Let S stand for skewness. Surface area S = 2π R(2πr) = 4π 2rR
3 3
dS =  x −  dA =  x −  g( x ) dx
3 3
 5  5
3
Problem Set 11-7
 x – 3   − 1 x 3 − x 2 + 3 x + 9 dx
3
S= ∫
–3  5  3  Review Problems
R0. Answers will vary.
 –64 ⋅ 35 
= −17.7737K  exactly  R1. Slice the region parallel to the force axis so that
 7 ⋅ 125  each point in a strip has about the same force as
h. By symmetry, the centroid of the area under f at the sample point (x, F ) .
is on the y-axis, so x = 0. Then
dW = F dx = 30e− 0.2 x dx
dS = x3 dA = x3(9 − x2) dx 10

S= ∫
3
x (9 − x ) dx = 0 (odd function
3 2 W= ∫0
30e −0.2 x dx = 150(1 − e −2 )
–3
= 129.6997… ≈ 129.7 ft-lb
integrated between symmetrical limits)

Calculus Solutions Manual Problem Set 11-7 307


© 2005 Key Curriculum Press
R2. a. dW = F dx = kx− 2 dx R4. a. The width of a strip at the sample
1 point (x, y) is

2
W = kx −2 dx = − k ft-lb
y ⇒ dA =  b − y dy
3 3 b b
w =b−
(Mathematically, the answer is negative h  h 
because dx is negative. Physically, the answer
dM x = y dA =  by − y 2  dy
b
is negative because the magnets absorb
energy from their surroundings rather than  h 
releasing energy to their surroundings.) h
 2
h

∫  by – h y  dy = 2 by
b 1 b 3
Mx = 2
− y
b. Construct axes with the origin at the vertex of 0 3h 0
the cone. An element of the cone in the xy-
7 3 1 2 b 3 1
plane has the equation y = x or x = y. = bh − h − 0 + 0 = bh 2
3 7 2 3h 6
Slice the water horizontally into disks so that 1
y ⋅ A = y ⋅ bh = M x
each point in a disk is lifted about the same 2
distance as the sample point (x, y) on the 1 2
element of the cone. bh
1
⇒y= 6 = h, Q .E.D .
F = 0.036 dV = 0.036 ⋅ π x2 dy 1
bh 3
9 2 2
= 0.036 ⋅ π y dy
49 b. The graph shows the region under y = ex
Each disk is lifted (10 − y) cm. rotated about the y-axis, showing back half of
dW = (10 − y) dF solid only.
9 2
= (10 − y)(0.036)π y dy y
49 (x, y )
7
W= ∫ 0
dW = 3.591π = 11.2814 …
≈ 11.28 in.-lb 1
x
R3. a. The graph shows the region in Quadrant I 1
under the graph of y = 8 − x3 rotated about
the y-axis.
Slice the region parallel to the y-axis,
y
8 generating cylindrical shells, so that each
point in a shell will be about the same
(x, y )
distance from the y-axis as the sample
point (x, y).
dV = 2π x ⋅ y ⋅ dx = 2π xex dx
x dM2y = x2 dV = 2πx3ex dx
1

∫ 2πx e
2
M2 y = 3 x
dx = 3.5401K
0
Slice the region parallel to the x-axis,
generating disks, so that each point in a disk (exactly 12π − 4πe)
is about the same distance from the xz-plane R5. Draw axes with the x-axis at ground level and the
as the sample point (x, y). y-axis through the upper vertex of the triangle.
ρ = ky, dV = π x2 dy = π (8 − y)2/3 dy Slice the face of the building horizontally so that
8 the wind pressure at any point in a strip is about
m= ∫ kπy(8 − y) dy = 57.6π k
2/3
0
equal to the pressure at the sample point (x, y).

dA = 150 −
b. Slice the region parallel to the y-axis, 150 
y dy
generating cylindrical shells, so that each  400 
point in a shell is about the same distance
dF = p dA = 200 ⋅ 150(1 − e −0.01y )1 −
1 
from the y-axis as the sample point (x, y). y dy
 400 
ρ = ex, dV = 2π xy dx = 2π (8x − x4) dx 400

2
F= ∫ dF = 3736263.2708…
∫ 2π e x (8 x − x 4 ) dx = 64π
0
m=
0 ≈ 3.736 million lb
(exactly 30000(125 − 25e− 4))

308 Problem Set 11-7 Calculus Solutions Manual


© 2005 Key Curriculum Press
R6. a. Let x = number of feet at which drill is 8


768
V= 2π y 4/3 dy =
π = 344.6775K
operating, and r(x) = number of dollars per 0 7
foot to drill at x feet. With slices perpendicular to the x-axis,
r(x) = a · bx
dV = π (82 − y2) dx = π (64 − x6) dx
r(0) = 30 ⇒ a = 30 2


768
/
110000
V = π (64 − x 6 ) dx = π = 344.6775K
50 = 30 ⋅ b10000 ⇒ b =  
5
0 7
 3
ii. With slices parallel to the y-axis,
x/10000
∴ r ( x ) = 30 
5 dV = 2π x · (8 − y) · dx = 2π x(8 − x3) dx
 3 2

(or r ( x ) = 30e − ln 0.6⋅x /10000


= 30e 0.00005108256Kx
)
V= ∫ 0
2π (8 x − x 4 ) dx = 19.2π
x/10000
= 60.3185…
b. dC = r ( x ) dx = 30  
5
dx
 3 With slices perpendicular to the y-axis,
x/10000
dV = π x 2 dy = π y 2/3 dy
30 
50000


5
C= dx = 6965243.17K
 3 8
V= ∫ πy 2/3 dy = 19.2π = 60.3185K
0

≈ 6.965 million dollars 0

iii. With slices parallel to the line x = 3,


 10000  5 
5

 exactly − 30 ⋅ ln 0.6  3  – 1 dV = 2π (3 − x) · (8 − y) · dx
   = 2π (3 − x)(8 − x3) dx
2

Concept Problems
V= ∫ 0
2π (3 − x )(8 − x 3 ) dx = 52.8π

C1. a. Either slice the region parallel to the y-axis, = 165.8760…


dA = (8 − y) dx = (8 − x3) dx With slices perpendicular to the line
2 x = 3,
A= ∫
0
(8 – x 3 ) dx = 12 dV = π [32 − (3 − x)2] dy
or slice parallel to the x-axis, = π [9 − (3 − y1/3)2] dy
8


8
A= ∫
0
y1/3 dy = 12 V=
0
π [9 − (3 − y1/3 )2 ] dy = 52.8π

b. i. Use slices parallel to the x-axis so that = 165.8760…


each point in a strip will be about the e. i. The centroid is 32/7 units from the x-axis.
same distance from the x-axis as the 32 768
sample point (x, y). ∴ V = 2π ⋅ ⋅ 12 = π
7 7
dM x = y dA = y(y1/3 dy) = 344.6775… (Checks.)
8


384 ii. The centroid is 0.8 unit from the y-axis.
M x = y 4/3 dy = = 54.8571K
0 7 V = 2π · 0.8 · 12 = 19.2π
ii. Use slices parallel to the y-axis so that = 60.3185… (Checks.)
each point in a strip will be about the
iii. The centroid is 3 − 0.8 = 2.2 units from
same distance from the y-axis as the
the line x = 3.
sample point (x, y).
V = 2π · 2.2 · 12 = 52.8π = 165.8760…
dM y = x dA = x(8 − x3) dx (Checks.)
2
My = ∫0
(8 x – x 4 ) dx = 9.6 f. Use horizontal slices so that each point in a
disk will be about the same distance from the
9.6 xz-plane as the sample point (x, y).
c. x ⋅ A = M y ⇒ x = = 0.8
12 dMxz = y dV = y(π x 2 dy) = y · π y 2/3 dy
384/7 32
y ⋅ A = Mx ⇒ y = = = 4.5714 K
8
12 7 M xz = ∫
0
πy 5/3 dy = 96π = 301.5928K
Centroid is at (0.8, 4.5714…). 96π
g. y ⋅ V = M xz ⇒ y = =5
d. i. With slices parallel to the x-axis, 19.2π
dV = 2π y · x · dy = 2π y 4/3 dy x = z = 0 by symmetry.
Centroid is at (0, 5, 0).

Calculus Solutions Manual Problem Set 11-7 309


© 2005 Key Curriculum Press
h. No. For the solid, y = 5, but for the region, Note that V = 2π M y, thus showing that the two
y = 4.5714 … . problems are mathematically equivalent, Q.E.D.
i. Use slices of the region parallel to the y-axis C3. dMxz = y dVxz = y π x 2 dy = π y(9 − y) dy
9
so that each point in a resulting cylindrical
shell will be about the same distance from the
M xz = π ∫ 0
π y(9 − y) dy = 121.5π
y-axis as the sample point (x, y). dM 2y = x 2 dV y = x 2 2π xy dx = 2π x3(9 − x2) dx
3
ρ = kx 2,
dV = 2π x(8 − y) dx = 2π (8x − x4) dx
M2 y = ∫ 0
2π x 3 (9 − x 2 ) dx = 121.5π , Q .E.D .
This is not true in general. Counterexample:
dm = ρ dV = kx2 · 2π (8x − x4) dx Rotate the region under y = 2 − 2x2.
= 2π k(8x3 − x6) dx
dM xz = y dVxz = y π x 2 dy = π y1 − y dy
1
2  2 

192
m = 2πk (8 x 3 − x 6 ) dx = πk 2
 1 

7 2
M xz = π y 1 − y dy = π
0

= 86.1693…k 0  2  3
j. Use cylindrical shells as in part i so that each dM 2y = x 2 dV y = x 2 2π xy dx = 2π x3(2 − 2x2) dx
1


1 2
point in a shell will be about the same M2 y = 2π x 3 (2 − 2 x 2 ) dx = π , not π .
distance from the y-axis as the sample 0 3 3
General proof: For any paraboloid of height H
point (x, y).
and base radius R, let h = distance (along the
dM 2 = x2 · dm = 2π k(8x5 − x8) dx axis) from the base and r = radius. Then a
2


512 H
M2 = 2πk (8 x 5 − x 8 ) dx = πk generating parabola is given by h = H − 2 ⋅ r 2.
0 9 R
= 178.7217… R2
dMbase = h dV = h π r 2 dh = h π ( H − h) dh
k. Use vertical slices of the region so that each H
R2 h= H
point in a strip will have about the same
pressure acting on it as at the sample
Mbase =
H
π
h=0 ∫
( Hh – h 2 ) dh
h= H
point (x, y). 2
R  H 2 1 3 1 2 2
= π h − h = πR H
p = 3 − x, dA = (8 − y) dx = (8 − x3) dx H 2 3  h=0 6
dF = p dA = (3 − x)(8 − x3) dx dM2axis = r dV = r 2π rh dr
2 2

∫ = r 2 2π r  H − 2 r 2  dr
F= (3 – x )(8 – x ) dx = 26.4
3 H
0  R 
r= R
 r3 – r 5  dr

(Note the similarity to the integral in 1
M2 axis = 2πH
part d.iii.) r =0  R2 
r= R
l. F = kz− 2
= 2πH  r 4 − 2 r 6 
1 1 1 4
= πR H
F = 26.4 at z = 1 ⇒ k = 26.4 ⇒ F = 26.4z− 2 4 6R  r =0 6
dW = F dz = 26.4z− 2 dz In the original example, H = R , so the two 2


moments turned out to be equal.
W= 26.4 z −2 dz = 17.6
1 C4. a. Assume m ≠ 0.
m. Use horizontal slices so that each point in a The area of the trapezoid is
resulting disk will be at about the same b +b ma + mb
A= 1 2 ⋅h = (b – a)
temperature as the sample point (x, y). 2 2
dH = CT dm = 0.3(10 − y)(5.8π y2/3 dy) 1
= m( b 2 − a 2 )
8

∫ 1.74(10 − y)(π y
2
H= 2/3
) dy = 167.04π b b b

∫ ∫
0 1
Integrating, y dx ≈ mx dx = mx 2
= 524.7716… ≈ 524.8 cal a a 2 a
C2. Let f (x) be the height of a vertical strip at x (or 1
= m(b − a ) = A, Q .E.D .
2 2
combined heights if the region being rotated is 2
not convex). Let x = a and x = b be the left and The length is L = (b − a) 1 + m 2 .
right boundaries of the region. b b

dV = 2π x dA = 2π x f ( x ) dx ⇒ V = 2π ∫
b
x f ( x ) dx
Integrating, ∫
a
dL ≈ ∫
a
dx = (b − a) ≠ L,
a Q .E .D .
b
dM y = x dA = x f ( x ) dx ⇒ M y = ∫a
x f ( x ) dx

310 Problem Set 11-7 Calculus Solutions Manual


© 2005 Key Curriculum Press
b. Note that r = mh. T2. dW = (40x − 10x2) dx
4


The volume of the cone is W= ( 40 x − 10 x 2 ) dx = 90
1 1
V = πr 2 h = πm2 h3 . 1
3 3 T3. y ⋅ m = M xz ⇒ y ⋅ 200 = 3000
Integrating dV ≈ π y 2 dx = π m 2x2 dx, ∴ y = 3000/200 = 15 cm
h


1
π m 2 x 2 dx = π m 2 h 3 = V , Q.E.D. T4. The center of the circle is (8, 9) and the radius
0 3
is 7, so the circle is on just one side of the axis
The surface area is S = π r r 2 + h 2 = of rotation (the y-axis). So the solid satisfies the
π mh 2 1 + m 2 . hypothesis of the theorem of Pappus.
Integrating dS ≈ 2π y dx = 2π mx dx, The centroid of the circle is (8, 9), the
h displacement from the y-axis is R = 8, and the
∫0
2π mx dx = π mh 2 ≠ S, Q.E.D. area of the circle is 49π.
∴ V = 2π RA = (2π )(8)( 49π ) = 784π =
c. Exact area of a strip:
1 1 2463.0086…
∆ A = ( mx + m( x + ∆x ))∆x = y∆x + ∆y∆x
2 2 T5. Using exponential regression,
Exact volume of frustum: F ≈ 29.9829… (1.0626…)x
π dW = F dx
∆V = ( m 2 ( x + ∆x )2 + m 2 x ( x + ∆x ) + m 2 x 2 )∆x 10

π
3 W= ∫
0
F dx ≈ 412.4652 … ≈ 412.5 ft-lb
= m 2 ∆x (3 x 2 + 3 x ( ∆x ) + ( ∆x )2 ) (By the trapezoidal rule, W ≈ 413 ft-lb.
3
Simpson’s rule cannot be used because there is
= π  y 2 ∆x + y∆y∆x + ( ∆y)2 ∆x 
1 an odd number of increments.)
 3 
T6. a.
d. dA − y dx =  y dx + ∆y dx  − y dx
1 y
 2 
1
= ∆y dx (x, y )
2
dV − π y dx2

= π  y 2 dx + y ∆y dx + ∆y 2 dx  − π y 2 dx
1
 3 
1
1 2
= π y ∆y dx + ∆y dx x
3 2
Both differences contain only higher-order
Slice the region parallel to the y-axis so that each
infinitesimals.
point in a strip will be about the same distance
e. If dQ = ∆Q leaves out only infinitesimals of from the y-axis as the sample point (x, y).
b
higher order, then ∫ dQ is exactly equal to Q. dMy = x dA = xex dx
2


a
My = xe x dx = e 2 + 1 = 8.3890 … ≈ 8.39 in.3
f. Reasons: 0

i. 0.5 and ∆y are constant with respect to the b. dM2y = x2 dA = x2ex dx


2


summation, so they can be pulled out.
M2 y = x 2 e x dx = 2e 2 − 2 = 12.7781…
ii. The sum of all the subsegments ∆x of 0

[a, b] must be b − a, the whole interval. ≈ 12.78 in.4


iii. ∆y has limit zero as ∆x goes to zero. 2
c. A = ∫ 0
e x dx = e 2 − 1 = 6.3890 … ≈ 6.39 in.2

Chapter Test e2 + 1
x ⋅ A = My ⇒ x = = 1.3130 … ≈ 1.31 in.
T1. a. force · displacement e2 – 1
b. mass T7. The graph shows y = x1/2 from x = 0 to x = 16,
rotated about the x-axis.
c. force
y
d. area · displacement 4 (x, y )

x
e. second moment of volume 16
f. x

Calculus Solutions Manual Problem Set 11-7 311


© 2005 Key Curriculum Press
Slice the region parallel to the x-axis so that each b. dM x = y dF = 62.4(8 − y) · 2y4/3 dy
8 9 ⋅ 210

point in a resulting cylindrical shell will be
F = 62.4(8 − y) ⋅ 2 y 4/3 dy = ⋅ 62.4
about the same distance from the x-axis as the 0 35
sample point (x, y). = 16430.8114… ≈ 16.43 thousand lb-ft
ρ = 3 y, dm = ρ dV = 3 y ⋅ 2π (16 − x ) y dy 16430.8114 K
y ⋅ F = Mx ⇒ y = = 3.2 ft
= 6π y2(16 − y2) dy 5134.6285K
4 x = 0 by symmetry.
m= ∫
0
6π (16 y 2 − y 4 ) dy = 819.2π Center of pressure is at (0, 3.2).
= 2573.5927… ≈ 2573.6 g T9. a. Slice the seating area into concentric rings of
T8. a. Slice the end of the trough parallel to the width dr. Each point in a ring will be about
x-axis so that each point in a strip has about the same distance from the center as the
the same pressure acting on it as the sample sample point.
point (x, y), where x ≥ 0. Let W = worth of the seating and v = value
p = 62.4(8 − y), dA = 2x dy = 2y1/3 dy per square foot.
dF = p dA = 62.4(8 − y) · 2y1/3 dy dW = v dA = 150r− 1 · 2π r dr = 300π dr
b


8
F= ∫
0
62.4(8 − y) ⋅ 2 y1/3 dy W=
30
300π dr = 300π (b − 30) dollars

64 ⋅ 9  b. 300π (b − 30) = 60000 ⇒


= 62.4  = 5134.6285… ≈ 5134.6 lb 200
 7  b = 30 + = 93.6619… ≈ 93.7 ft
π

312 Problem Set 11-7 Calculus Solutions Manual


© 2005 Key Curriculum Press
Chapter 12—The Calculus of Functions Defined
by Power Series

Problem Set 12-1 The values differ by 0.0625, and


6(−0.5)6 = −0.09375.
6
1. f ( x ) = , For P5(0.5), the difference is greater than the
1– x value of the next term of the series. This result is
P5 ( x ) = 6 + 6 x + 6 x 2 + 6 x 3 + 6 x 4 + 6 x 5 to be expected because the rest of the series
is formed by adding more positive terms.
100 y
f P5 For P5(−0.5), the difference is less in absolute
value than the absolute value of the next term.
f x
–1 1
This result is reasonable because the terms
alternate in sign so that you are adding and
P5
subtracting ever smaller quantities.
–100
7. A geometric series; the common ratio
The graph of P5 fits the graph of f reasonably
well for about −0.8 < x < 0.6. Problem Set 12-2
The graph of P5 bears no resemblance to the
Q1. . . . for any ε > 0 there is a D > 0 such that
graph of f at x = 2 and at x = −2, for example.
if x > D, then f (x) is within ε units of L.
2. P6(x) = P5(x) + 6x6
Q2. the fundamental theorem of calculus
100 y
f P5
Q3. the fundamental theorem of calculus
P6
Q4. the mean value theorem
f x Q5. derivative Q6. cos x − x sin x
–1 1 1
Q7. x sin x + cos x + C Q8. dA = r 2 dθ
P5 2
–100 Q9. f (x) = e− x Q10. D
1. Series: 200 − 120 + 72 − 43.2 + 25.92 −
The graph of P5 fits the graph of f slightly better, 15.552 + L
perhaps for −0.9 < x < 0.7. Sums: 200, 80, 152, 108.8, 134.72, 119,
3. P5(0.5) = 11.8125, P6(0.5) = 11.90625, 168, …
f (0.5) = 12 Sn
∴ P6(0.5) is closer to f (0.5) than P5(0.5) is.
P5(2) = 378, P6(2) = 762, f (2) = −6 125
∴ P6(2) is not closer to f (2) than P5(2) is. 100

4. Possible conjecture: P(x) converges to f (x) for


n
−1 < x < 1, or perhaps for −1 ≤ x ≤ 1. 10
5. P0(1) = 6 P0(−1) = 6
1
P1(1) = 12 P1(−1) = 0 S = 200 ⋅ = 125
P2(1) = 18 P2(−1) = 6 1 + 0.6
The series converges to 125.
P3(1) = 24 P3(−1) = 0
P4(1) = 30 P4(−1) = 6 1 – (–0.6) n
|125 − Sn | = 125 − 200 ⋅
For x = 1, the sums just keep getting larger and 1 + 0.6
larger as more terms are added. For x = −1, the 1 – (–0.6) n
sums oscillate between 0 and 6. In neither case = 125 1 − 1.6 ⋅
1.6
does the series converge. If the answer to
Problem 4 includes x = 1 or x = −1, the = 125|1 − (1 − (−0.6)n)|
conjecture will have to be modified. = 125|−0.6n| = 125(0.6n)
6. P5(0.5) = 11.8125 and f (0.5) = 12 125(0.6 n ) = 0.0001 ⇔ n =
ln 0.0001125
/
= 27.48K
The values differ by 0.1875, and ln 0.6
6(0.5)6 = 0.09375. Make n ≥ 28.
P5(−0.5) = 3.9375 and f (−0.5) = 4 Sn will be within 0.0001 unit of 125 for all
values of n ≥ 28.

Calculus Solutions Manual Problem Set 12-2 313


© 2005 Key Curriculum Press
2. Series: 30 + 33 + 36.3 + 39.93 + 43.923 + c. The total perimeter converges to
48.3153 + K 1
16 ⋅ = 54.6274 K cm.
Sums: 30, 63, 99.3, 139.23, 183.153, 1 – 0.51/2
231.4683, … d. The sum of the areas is 16 + 8 + 4 + 2 + L ,
Sn
which is a convergent geometric series with
r = 0.5.
1
S = 16 ⋅ = 32 cm2
100 1 – 0.5
5. a. The interest rate for one month is
n 0.09/12 = 0.0075.
10
Months Dollars
The graph shows divergence. 0 1,000,000.00
1 – 1.1100
S100 = 30 ⋅ = 4,133,883.70 K (Wow!) 1 1,007,500.00
1 – 1.1
2 1,015,056.25
The formula S = t1/(1 − r) gives −300 for S, but
it has no meaning because the series does not 3 1,022,669.17
converge. b. Worth is (1,000,000)(1.007512) =
3. a. Series: $1,093,806.90; interest is $93,806.90.

∑ 7(0.8
n =1
n −1
) = 7 + 5.6 + 4.48 + 3.584 + L c. The first deposit is made at time t = 0, the
second at time t = 1, and so forth, so at time
Sums: 7, 12.6, 17.08, 20.664, 23.5312, … t = 12, the term index is 13.
S 4 = 20.664, so the amount first exceeds d. Meg earned $93,806.90 the first year.
20 µg at the fourth dose. 93806.90
APR = ⋅ 100 = 9.3806 K%
1 1000000
S = 7⋅ = 35, so the total amount
1 – 0.8 e. (1,000,000)(1.0075n) = 2,000,000
never exceeds 40 µg. ln 2
n= = 92.7657K
The graph confirms that the partial sums of ln 1.0075
the series approach 35 asymptotically and After 93 months
first exceed 20 µg at the fourth dose.
6. a. The interest rate for one month is
0.108/12 = 0.009.
40
S5 = 100 + 100(1.009) + 100(1.009)2
30 Asymptote + 100(1.009)3 + 100(1.009)4
Goes > 20 + 100(1.009)5
20 1 – 1.009 6
= 100 ⋅ = $613.66
10
Stays > 20 1 – 1.009
n
b. There are six terms because the term index of
1 2 3 4 5 6 7 8 9 10 the first term is zero.
c. 10 years equals 120 months. There will have
b. tn = Sn − 7, so the sequence is 0, 5.6, 10.08, been 121 deposits after 10 years because the
13.664, 16.5312, … . initial deposit was made at time 0. So there
See the graph in part a. The open circles show are 121 terms.
the partial sums just before a dose. 1 – 1.009121
S120 = 100 ⋅ = $21,742.92
t7 = 20.6599… . The amount remains 1 – 1.009
above 20 µg for n ≥ 7. The principal is 121(100) = $12,100.
c. See the graph in part a. The interest is 21,742.92 − 12,100 =
$9,642.92.
4. a. Perimeters are 16, 16 0.5 , 16(0.5), … ,
7. a. Sequence: 20, 18, 16.2, 14.58, 13.122, …
which is a geometric sequence with t1 = 16
and r = 0.5 . b. S 4 = 20 + 18 + 16.2 + 14.58 = 68.78 ft
1
1 – 0. 5 5 c. S = 20 ⋅ = 200
b. S10 = 16 ⋅ = 52.9203K cm 1 – 0.9
1 – 0.51/2 So the ball travels 200 ft before it comes to
rest.

314 Problem Set 12-2 Calculus Solutions Manual


© 2005 Key Curriculum Press
d. For the 10-ft first drop, 10 = 0.5(32.2)t2. 6
9. f ( x ) =
t = (10/16.1)1/2 = 0.7881… 1– x
The total time for the 20-ft first cycle is P( x ) = 6 + 6 x + 6 x 2 + 6 x 3 + 6 x 4 + 6 x 5 + L
2(0.7881…) = 1.5762… s. P ′( x ) = 6 + 12 x + 18 x 2 + 24 x 3 + 30 x 4 + L
For the 18-ft second cycle, P ′′( x ) = 12 + 36 x + 72 x 2 + 120 x 3 + L
t = 2(9/16.1)1/2 = 1.4953… s.
P ′′′( x ) = 36 + 144 x + 360 x 2 + L
e. The times form a geometric series with first f ′(x) = 6(1 − x)− 2
term 1.5762… and common ratio equal to f ″(x) = 12(1 − x)− 3
0.91/2 = 0.9486… . So the series of times f ″′(x) = 36(1 − x)− 4
converges to P′(0) = 6 and f ′(0) = 6
1
S = 1.5762 K ⋅ = 30.7155K P″(0) = 12 and f ″(0) = 12
1 – 0.91/2 P″′(0) = 36 and f ″′(0) = 36
The model predicts that the ball comes to rest Conjecture: P (n)(0) = f (n)(0) for all values of n.
after about 30.7 s.
8. a.
Iteration Total Length Problem Set 12-3
0 27 Q1. 0.3333… Q2. 0.4444…
2 4
1 36 Q3. Q4.
3 9
2 48
Q5. 13 Q6. 125
3 64
Q7. mass ⋅ displacement Q8. centroid
Because each segment is divided into four Q9. ln x + C Q10. D
pieces, each of which is 1/3 of the original 1. f (x) = 5e2x
length, the length at the next iteration can be f ′(x) = 10e2x
calculated by multiplying the previous length f ″(x) = 20e2x
by 4/3. f ′′′(x) = 40e2x
f (4)(x) = 80e 2x
b. The sequence of lengths diverges because the
common ratio, 4/3, is greater than 1. Thus, 2. P1(x) = c0 + c1x and P1′ ( x ) = c1
the total length of the snowflake curve is P1(0) = c0 and f (0) = 5 ⇒ c0 = 5
infinite! P1′ (0) = c1 and f ′(0) = 10 ⇒ c1 = 10
c. From geometry, the area of an equilateral ∴ P1(x) = 5 + 10x
3 2 3. P2(x) = c0 + c1x + c2x2
triangle of side s is A = s . P2′ ( x ) = c1 + 2c2 x and P2′′( x ) = 2c2
4
The number of triangles added is 3, 12, 48, P2(0) = c0 and f (0) = 5 ⇒ c0 = 5
192, … . P2′ (0) = c1 and f ′(0) = 10 ⇒ c1 = 10
The side of each added triangle is 3, 1, 1/3, P2′′(0) = 2c2 and f ′′(0) = 20 ⇒ 2c2 = 20
1/9, … . ⇒ c2 = 10
The added areas form the series ∴ P2(x) = P1(x) = 5 + 10x + 10x2
3 c0 and c1 are the same as for P1(x).
[3(3) 2 + 12(1) 2 + 48(1/3)2 + 192(1/9)2 + L ] 4. P 3(x) = c0 + c1x + c2x2 + c3x3
4
P 4(x) = c0 + c1x + c2x2 + c3x3 + c4x4
3
= ⋅ 3 ⋅ 9 2 [ 4 0 (1/3)2 + 41 (1/3) 4 + 4 2 (1/3)6 P4′′′ ( x ) = 6c3 + 24c4 x and P4( 4 ) ( x ) = 24c4
4
P4′′′ (0) = 6c3 and f ′′′(0) = 40
+ 4 3 (1/3)8 + L ]
20
3 ⇒ 6c3 = 40 ⇒ c3 =
= ⋅ 3 ⋅ 9 2 [ 4/9 + ( 4/9)2 + ( 4/9)3 + ( 4/9) 4 + L ] 3
16 P (44 ) (0) = 24c4 and f ( 4 ) (0) = 80
3 1 10
= ⋅ 3 ⋅ 9 2 ( 4/9) ⋅ = 12.15 3 ⇒ 24c4 = 80 ⇒ c4 =
16 1 – 4/9 3
The area of the pre-image is c0, c1, and c2 are the same as before.
3 2 20 3
⋅ 9 = 20.25 3 . 5. P3 ( x ) = 5 + 10 x + 10 x 2 + x
4 3
The total area is 32.4 3 = 56.1184 K cm 2 . 20 3 10 4
P4 ( x ) = 5 + 10 x + 10 x 2 + x + x
3 3

Calculus Solutions Manual Problem Set 12-3 315


© 2005 Key Curriculum Press
y
100 f P4 1. a.
P3 n f (n) ( x ) f (n) (0) P (n) (0) cn

P4 0 ex 1 c0 1
x 1 ex 1 c1 1
1
P3
2 ex 1 2!c2 1
2!
6. P4 is indistinguishable from f for about 3 ex 1 3!c3 1
−1 < x < 0.9. 3!
7. P3(1) = 31.6666666… 1 2 1 3
P4(1) = 35.0000000… ∴ P( x ) = 1 + x +
x + x + L , Q .E .D .
2! 3!
f (1) = 5e2 = 36.9452804…
1 4 1 5
∴ P4(1) is closer to f (1) than P3(1), Q.E.D. b. Next two terms: L + x + x + L
4! 5!
80 5 ⋅ 2 4 ∞
8. c4 = =

1 n
24 4! c. x
n=0
n!
The 5 is the coefficient in 5e2x.
The 2 is the exponential constant. d.
y
The 4 is the exponent of x in the last term. ex
S3
20 5 ⋅ 2 20 5 ⋅ 2
3 2
9. c3 = = , c2 = = , 5
6 3! 2 2!
10 5 ⋅ 21 5 ⋅ 20
c1 = = , c0 = 5 = (0! = 1) x
1 1! 0! 3

10. Conjecture:
5 ⋅ 2 5 160 4 5 ⋅ 2 6 320 4 e. The two graphs are indistinguishable for
c5 = = = , c6 = = = approximately −1 < x < 1.
5! 120 3 6! 720 9
∞ f. Solve ex − S3(x) = 0.0001 for x close to 1.
5 ⋅ 2n n
11. P( x ) = ∑
n=0
n!
x x ≈ 0.2188…
Solve ex − S3(x) = 0.0001 for x close to −1.
x = −0.2237…
The interval is −0.2237… < x < 0.2188… .
Problem Set 12-4 g. The ninth partial sum is S8(x).
Solve ex − S8(x) = 0.0001 for x close to 1.
Q1. Q2.
y y
x ≈ 1.4648…
x x Solve S8(x) − ex = 0.0001 for x close to −1.
x = −1.5142…
The interval is −1.5142… < x < 1.4648… .
2. a. By equating derivatives:
Q3. Q4.
y y n f (n) ( x ) f (n) (0) P (n) (0) cn

x 0 cos x 1 c0 1
1 −sin x 0 c1 0
x
1
2 −cos x −1 2!c2 −
2!
Q5. Q6. 3 sin x 0 3!c3 0
y y
1
4 cos x 1 4!c4
x 4!
5 −sin x 0 5!c5 0
x
1
6 −cos x −1 6!c6 −
6!
Q7. exponent Q8. coefficient 7 sin x 0 7!c7 0
Q9. power Q10. D 1
8 cos x 1 8!c8
8!

316 Problem Set 12-4 Calculus Solutions Manual


© 2005 Key Curriculum Press
1 2 1 4 1 6 1 8 1 3 1 5 1 7
∴ P( x ) = 1 − x + x − x + x −L , 4. a. P( x ) = x + x + x + x +L
2! 4! 6! 8! 3! 5! 7!
Q .E .D . b. By equating derivatives:
1 10 1 12 1 14
b. L − x + x − x +L
10! 12! 14! n f (n)( x ) f (n)(0) P (n) (0) cn

∑ (–1) n 1 0 sinh x 0 c0 0
c. x 2n
n=0
(2 n)! 1 cosh x 1 c1 1
d. y = cos x 2 sinh x 0 2!c2 0
y
1
S4 3 cosh x 1 3!c3
3!
cos
x 4 sinh x 0 4!c4 0
1
5 cosh x 1 5!c5
S7
5!
6 sinh x 0 6!c6 0
e. See the graph in part d, showing S7(x) (eighth 1
7 cosh x 1 7!c7
partial sum). 7!
The graphs are indistinguishable for
1 3 1 5 1 7
approximately −5.5 < x < 5.5. ∴ P( x ) = x + x + x + x + L , Q .E .D .
3! 5! 7!
f. Solve S7(x) − cos x = 0.0001 for x close
c. S3(0.6) = 0.636653554…
to 5.5.
sinh 0.6 = 0.636653582…
x ≈ 4.5414…
(Note that some solvers may give an error ∴ S3(0.6) ≈ sinh 0.6, Q.E.D.
message. In this case, zoom in by table, d. Solve S3(x) − sinh x = 0.0001 for x
starting at x = 5 and using increments of 0.1; close to 1.
then x = 4.5, and increments of 0.01, and so x ≈ 1.4870…
forth.) By symmetry, the interval is
By symmetry, the interval is −1.4870… < x < 1.4870… .
−4.5414… < x < 4.5414… . 1 1 1
e. P ′( x ) = 1 + ⋅ 3 x 2 + ⋅ 5 x 4 + ⋅ 7 x 6 + L
g. Both functions are even. P(x) is even because 3! 5! 7!
it has only even powers of x. 1 2 1 4 1 6
= 1+ x + x + x +L
1 1 1 2! 4! 6!
3. a. S3 (0.6) = 0.6 − (0.6 3 ) + (0.6 5 ) − (0.6 7 )
3! 5! 7! f. Find S3(0.6) for the P′ series.
= 0.564642445… S3(0.6) = 1.1854648
sin 0.6 = 0.564642473… cosh 0.6 = 1.18546521…
∴ S3(0.6) ≈ sin 0.6, Q.E.D. ∴ S3(0.6) ≈ cosh 0.6, and thus the P′(x)
b. sin 0.6 = 0.564642473… series seems to represent cosh x, Q.E.D.
Tail = sin 0.6 − Sn(0.6)
First term of the tail is tn+1 .
g. ∫ P( x ) dx
sin 0.6 − S1(0.6) = 0.0006424733… 1 2 1 1 4 1 1 6
= x + ⋅ x + ⋅ x +L+ C
t2 = 0.000648 2 3! 4 5! 6
sin 0.6 − S2(0.6) = −0.00000552660… Simplifying and letting C = 1 gives
t3 = −0.00000555428… 1 1
1+ x2 + x4 + x6 + L ,
1
sin 0.6 − S3(0.6) = 0.0000000276807… 2! 4! 6!
t4 = 0.0000000277714… which is the series for cosh x, Q.E.D.
In each case, the tail is less in magnitude than 5. a. f (x) = ln x f (1) = 0
the absolute value of the first term of the tail, f ′(x) = x − 1 f ′(1) = 1
Q .E .D . f ′′( x ) = − x −2 f ′′(1) = −1
c. Make | tn+1 | < 0.5 × 10 −20. f ′′′( x ) = 2 x −3 f ′′′(1) = 2
1 1 1
(0.6 2 n+3 ) < 5 × 10 −21 P( x ) = ( x − 1) − ( x – 1)2 + ( x – 1)3
(2 n + 3)! 2 3
Inequality is first true for n = 8. 1
− ( x – 1) + L
4
Use at least nine terms (n = 8). 4

Calculus Solutions Manual Problem Set 12-4 317


© 2005 Key Curriculum Press
P′( x ) = 1 − ( x − 1) + ( x − 1)2 − ( x − 1)3 + K 2n ∞
b. lim | tn | = lim →
P ′′( x ) = −1 + 2( x − 1) − 3( x − 1) 2 + K x →∞ x →∞ n ∞
P ′′′( x ) = 2 − 6( x − 1) + K = lim
2 n ln 2


P(1) = 0 = f (1) x →∞ 1 1
P′(1) = 1 = f ′(1) =∞
P″(1) = −1 = f ″(1) The series cannot possibly converge because
P′′′(1) = 2 = f ′′′(2), Q .E .D . the terms do not approach zero as n
1 1 approaches infinity.
b. L + ( x – 1)5 − ( x – 1)6 + L c.
5 6
∞ n tn(1.2)

1
c. P( x ) = (–1) n+1 ⋅ ( x – 1) n 1 0.2
n
n =1 2 −0.02
d. 3 0.0026666…
y 4 −0.0004
ln x 5 0.000064
1
x
6 −0.00001066…
1
The absolute values of the terms are
S10
approaching zero as n increases.
d. Tail = ln 1.2 − Sn(1.2)
First term of the tail is tn+1 .
e. S10(1.2) = 0.182321555…
ln 1.2 − S1(1.2) = −0.01767…
ln 1.2 = 0.182321556…
t2 = −0.02
S10(1.95) = 0.640144911…
ln 1.2 − S2(1.2) = 0.002321…
ln 1.95 = 0.667829372…
t3 = 0.002666…
S10(3) = −64.8253968…
ln 1.2 − S3(1.2) = −0.0003451…
ln 3 = 1.0986122…
t4 = −0.0004
S 10(x) fits ln x in about 0 < x < 2.
In each case, the tail is less in magnitude than
This is a wider interval of agreement than that the absolute value of the first term of the tail.
for the fourth partial sum, which looks like 7. a. f (x) = tan− 1 x
about 0.3 < x < 1.7. ∞


1
S10(1.2) and ln 1.2 agree through the eighth P( x ) = (–1) n x 2 n+1
decimal place. The values of S10(1.95) and n=0
2 n + 1
ln 1.95 agree only to one decimal place. The 1 1 1
= x − x3 + x5 − x7 + L
values of S10(3) and ln 3 bear no resemblance 3 5 7
to each other. b. y = tan− 1 x, y = S 5(x), and y = S 6(x) (sixth
1 1 and seventh partial sums)
6. a. P( x ) = ( x − 1) − ( x – 1)2 + ( x – 1)3
2 3 y S6
1 4 K S5
− ( x – 1) + f (x )
4 1
x
1
n tn(3) S5
S6
1 2
2 −2
Both partial sums fit the graph of f very well
3 2.6666… for about −0.9 < x < 0.9. For x > 1 and
4 −4 x < −1, the partial sums bear no resemblance
5 6.4 to the graph of f.
6 −10.6666…

The absolute values of the terms are getting Problem Set 12-5
larger as n increases. Q1. 4! = 24 Q2. 3! = 6
Q3. 4!/4 = 6 Q4. n = 3

318 Problem Set 12-5 Calculus Solutions Manual


© 2005 Key Curriculum Press
Q5. n = m − 1 Q6. m = 1 1 1
13. ln x 2 = ( x 2 − 1) − ( x 2 – 1)2 + ( x 2 – 1)3 − L
Q7. 0! = 1!/1 = 1 Q8. (−1)! = 0!/0 = 1/0 = ∞ 2 3
(Or: ln x = 2 ln x = 2(x − 1) − (x − 1)2
2
Q9. x/ x 2 – 7 Q10. A
2
1. f (u) = e u + ( x – 1)3 − L )
3
1 2 1 3 1 4 1 5 K
= 1+ u + u + u + u + u + 1 1
14. e − x = 1 + (– x 2 ) + (– x 2 )2 + (– x 2 )3
2

2! 3! 4! 5! 2! 3!
2. f (u) = ln u 1
+ (– x 2 ) 4 + L
1 1 1
= (u − 1) − (u – 1)2 + (u – 1)3 − (u – 1) 4 + K 4!
2 3 4 1 1 1
= 1 − x2 + x 4 − x6 + x8 − L
3. f (u) = sin u 2! 3! 4!
x x

t – t + t – L dt
∫e ∫ 1 – t
1 3 1 5 1 7 1 9 1 11 −t2 1 4 1 6 1 8
=u− u + u − u + u − u +L 15. dt = 2
+
3! 5! 7! 9! 11! 0 0 2! 3! 4! 
4. f (u) = cos u 1 3 1 1 5 1 1 7 1 1 9
= x − x + ⋅ x − ⋅ x + ⋅ x −L
3 5 2! 7 3! 9 4!
1 2 1 4 1 6 1 8 1 10
= 1− u + u − u + u − u +L x
(3t − 1) − 1 (3t – 1)2
x
2! 4! 6! 8! 10! 16. ∫ 1/3
ln (3t ) dt = ∫
1/3 
 2
5. f (u) = cosh u
+ (3t – 1)3 − (3t – 1) 4 + … dt
1 1
1 2 1 4 1 6 1 8 1 10
= 1+ u + u + u + u + u +L 3 4 
2! 4! 6! 8! 10!
1 1
6. f (u) = sinh u = (3t – 1)2 − (3t – 1)3
3 ⋅ 2 ⋅1 3⋅3⋅ 2
1 3 1 5 1 7 1 9 1 11 x
=u+ u + u + u + u + u +L 1 1
3! 5! 7! 9! 11! + (3t – 1) 4 − (3t – 1)5 + L
3⋅ 4 ⋅3 3⋅5⋅ 4 1/3
7. f (u) = (1 − u) −1 = 1 + u + u 2 + u 3 + u 4 + u 5 + L
1 1 1
= (3 x – 1)2 − (3 x – 1)3 + (3 x – 1) 4
8. f (u) = tan −1 u 6 18 36
1 1 1 1 1 1
= u − u 3 + u 5 − u 7 + u 9 − u11 + L − (3 x – 1)5 + L
3 5 7 9 11 60

9. x sin x = x  x − x 3 + x 5 − x 7 + x 9 − L
1 1 1 1 1
17. = 1 − x 4 + x 8 − x 12 + x 16 − L
 3! 5! 7! 9!  x +1 4

1 4 1 6 1 8 1 10 9 3
= x2 − x + x − x + x −L 18. =
3! 5! 7! 9! x 2 + 3 1 + ( x 2 /3)
10. x sinh x = x  x + x 3 + x 5 + x 7 + x 9 + L
1 1 1 1
= 3 1 − x 2 + 2 x 4 − 3 x 6 + L
1 1 1
 3! 5! 7! 9!   3 
3 3
1 4 1 6 1 8 1 10
= x2 + x + x + x + x +L 1 4 1 6
3! 5! 7! 9! = 3 − x2 + x − 2 x +L
3 3
11. cosh x 3 x x

∫ ∫ (1 – t
1
1 1 1 1 19. dt = 4
+ t 8 – t 12 + t 16 – K) dt
= 1 + (x 3)2 + (x 3) 4 + (x 3)6 + (x 3)8 + L 0 t +1
4
0
2! 4! 6! 8! 1 5 1 9 1 13 1 17 K
1 1 1 1 = x− x + x − x + x −
= 1 + x 6 + x 12 + x 18 + x 24 + L 5 9 13 17
2! 4! 6! 8!
x x
 + t 4 – 2 t 6 + K dt
∫ ∫ 3 – t
9 1 1
12. cos x 2 20. dt = 2
0 t +3
2
0 3 3 
1 1 1 1
= 1 − ( x 2 ) 2 + ( x 2 ) 4 − ( x 2 ) 6 + ( x 2 )8 − L 1 1 5 1 1
2! 4! 6! 8! = 3x − x 3 + x − 2 x7 + 3 x9 −K
3 3⋅5 3 ⋅7 3 ⋅9
1 4 1 8 1 12 1 16
= 1− x + x − x + x −L
2! 4! 6! 8!

Calculus Solutions Manual Problem Set 12-5 319


© 2005 Key Curriculum Press
25.a. f (0.4) ≈ 2 + 0.5(1.4) − 0.3(1.4)2 − 0.18(1.4)3
d + 0.02(1.4)4 = 1.694912
21. (sinh x 2 ) We must assume that the series converges for
dx
x = 0.4.
d  2 1 6 1 10 1 14 K
= x + x + x + x +
dx  3! 5! 7!  b. f (−1) = c 0 = 2
6 10 14 f ′(−1) = c 1 = 0.5
= 2 x + x 5 + x 9 + x 13 + K f ″(−1) = 2!c2 = 2(−0.3) = −0.6
3! 5! 7!
f ″′(−1) = 3!c3 = 6(−0.18) = −1.08
2 5 2 9 2 13 K
= 2x + x + x + x + f (4)(−1) = 4!c4 = 24(0.02) = 0.48
2! 4! 6!
1 9 1 13 … c. g(x) = f (x2 − 1) ≈ P 4(x2 − 1)
= 2x + x + x +
5
x + = 2 + 0.5(x2 − 1 + 1) − 0.3(x2 − 1 + 1)2
12 360
− 0.18(x2 − 1 + 1)3 + 0.02(x2 − 1 + 1)4
Alternate solution: = 2 + 0.5x2 − 0.3x4 − 0.18x6 + 0.02x9
d
(sinh x 2 ) = 2 x cosh x 2 Sixth-degree polynomial:
dx
g(x) ≈ 2 + 0.5x2 − 0.3x6
= 2 x 1 + x 4 + x 8 + x 12 + K
1 1 1 g(1) ≈ 2.2
 2! 4! 6! 
d. g′(x) = x − 1.2x3 + terms in higher powers
2 2 2
= 2 x + x 5 + x 9 + x 13 + … of x.
2! 4! 6! ∴ g′(0) = 0
d g″(x) = 1 − 3.2x2 + terms in higher powers
22. (cos x 0.5 )
dx of x.
d  ∴ g″(0) = 1 > 0. ∴ (0, g(0)) is a local
1 – x + x 2 – x 3 + x 4 – K
1 1 1 1
= minimum.
dx  2! 4! 6! 8! 

∫ ∫
x x
1 2 3 4 g (t ) dt ≈ (2 + 0.5t − 0.3t ) dt
2 4
= − + x − x2 + x3 −K e. 0 0
2! 4! 6! 8!
1 1 1 2 1 x
0.5 0.3
=− + x − x + x3 − … = 2t + t −
3
t
5

2 12 240 10080 3 5 0
Alternate solution:
1
= 2x + x − 0.06 x
3 5
d 1
(cos x 0.5 ) = − x −0.5 sin x 0.5 6
dx 2
1 −0.5  0.5 1 1.5 1 2.5 1 3.5 26. a. f (1) ≈ P4(1)
=− x x − x + x − x + L = −4 + 3(1 − 2) + 0.5(1 − 2)2
2  3! 5! 7! 
1 1 1 2 1 3 − 0.09(1 − 2)3 − 0.06(1 − 2)4
=− + x− x + x −L = −6.47
2! 2 ⋅ 3! 2 ⋅ 5! 2 ⋅ 7!
We must assume that the series converges for
1 2 3 4
= − + x − x2 + x3 −L x = 1.
2! 4! 6! 8!
Multiply by 1/1, 2/2, 3/3, 4/4, … and simplify. b. f (2) = c 0 = −4
0.7 f ′(2) = c 1 = 3
23. P4 ( x ) = −8 + 3( x − 2) + ( x − 2)2 f ″(2) = 2!c2 = 2(0.5) = 1
2!
f ′″(2) = 3!c3 = 6(−0.09) = −0.54
0.51 0.048
+ ( x − 2)3 − ( x − 2) 4 f (4)(2) = 4!c4 = 24(−0.06) = −1.44
3! 4!
= −8 + 3(x − 2) + 0.35(x − 2)2 c. g(x) = f (x2 + 2) ≈ P(x2 + 2)
+ 0.085(x − 2)3 − 0.002(x − 2)4 = −4 + 3(x2 + 2 − 2) + 0.5(x2 + 2 − 2)2
−0.09(x2 + 2 − 2)3 − 0.06(x2 + 2 − 2)4
0.48 0
24. P5 ( x ) = 7 + 2( x + 1) − ( x + 1) 2 + ( x + 1)3 = −4 + 3x2 + 0.5x4 − 0.09x6 − 0.06x8
2! 3!
0.36 0.084 Fourth-degree polynomial:
+ ( x + 1) −
4
( x + 1)5 g(x) ≈ −4 + 3x2 + 0.5x4
4! 5!
= 7 + 2(x + 1) − 0.24(x + 1)2
+ 0.015(x + 1)4 − 0.0007(x + 1)5

320 Problem Set 12-5 Calculus Solutions Manual


© 2005 Key Curriculum Press
d. g′(x) = 6x + 2x3 + terms in higher powers 33. By equating derivatives:
of x.
∴ g′(0) = 0 n f (n)( x ) f (n)(0) P (n)(0) cn
g″(x) = 6 + 6x2 + terms in higher powers
of x. 0 cos 3x 1 c0 1
∴ g″(0) = 6 > 0 1 −3 sin 3x 0 c1 0
∴ (0, g(0)) = (0, 4) is a local minimum. 9
x x 2 −9 cos 3x −9 2!c2 −
e. h( x ) = ∫ g(t ) dt ≈ ∫ (−4 + 3t + 0.5t 4 ) dt
2 2!
0 0
x 3 27 sin 3x 0 3!c3 0
= −4t + t + 0.1t 5 0 = −4 x + x 3 + 0.1x 5
3
81
4 81 cos 3x 81 4!c4
27. f (x) = sin x, about x = π/4: 4!
2 π π
2 5 −243 sin 3x 0 5!c5 0
x– −
2 
x– 
2
f ( x) = + 729
2 2  4  2 ⋅ 2!  4 6 −729 cos 3x −729 6!c6 −
6!
π π
3 4
2 
x–  +
2 
− x– 
2 ⋅ 3!  4 2 ⋅ 4!  4 ∴ cos 3 x = 1 −
9 2 81 4 729 6
x + x − x +L
2! 4! 6!
π
5
2 
+ x –  −L By substitution:
2 ⋅ 5!  4 1 1 1
28. f (x) = cos x, about x = π/4: cos 3 x = 1 − (3 x )2 + (3 x ) 4 − (3 x )6 + L
2! 4! 6!
π π
2
2
x– −
2 
x– 
2 9 2 81 4 729 6
f ( x) = − = 1− x + x − x +L
2 2  4  2 ⋅ 2!  4 2! 4! 6!
π π
3 4
2 
x–  +
2 
x– 
The two answers are equivalent. Substitution
+
2 ⋅ 3!  4 2 ⋅ 4!  4 gives the answer much more easily in this case.
π
5
2 
x –  −L
34. By equating derivatives:

2 ⋅ 5!  4
n f (n)( x ) f (n)(0) P (n)(0) cn
29. f (x) = ln x, about x = 1:
1 1 0 ln (1 + x) 0 c0 0
f ( x ) = ( x − 1) − ( x – 1)2 + ( x – 1)3
2 3 1 (1 + x)− 1 1 c1 1
1 2 −(1 + x)− 2 −1 2!c2 1
− ( x – 1) + L
4 −
4 2!
3 2(1 + x)− 3 2 3!c3 2
30. f (x) = log x, about x = 10:
3!
1 ( x – 10) 1 ( x – 10) 2 4 −6(1 + x)− 4 −6 4!c4 6
f ( x) = 1 + ⋅ − ⋅ −
ln 10 10 2 ln 10 10 2 4!
1 ( x – 10)3 1 ( x – 10) 4
+ ⋅ − ⋅ +L 1 2 1 3 1 4
3 ln 10 10 3
4 ln 10 10 4 ∴ ln (1 + x ) = x − x + x − x +L
2 3 4
31. f (x) = (x − 5)7/3 , about x = 4: By substitution, substitute (1 + x) for u in
7 7⋅4
f ( x ) = −1 + ( x – 4) − 2 ( x – 4)2 1 1 1
ln u = (u − 1) − (u – 1)2 + (u – 1)3 − (u – 1) 4 + L
3 3 2! 2 3 4
7 ⋅ 4 ⋅1 7 ⋅ 4 ⋅ 1 ⋅ (–2)
+ 3 ( x – 4)3 − ( x – 4) 4 1 2 1 3 1 4
3 3! 34 4! ln (1 + x ) = x − x + x − x + L .
7 ⋅ 4 ⋅ 1 ⋅ (–2)(–5) 2 3 4
+ ( x – 4) 5 − L The two answers are equivalent. Substitution
355!
gives the answer much more easily in this case.
32. f (x) = (x + 6)4.2, about x = −5:
35. S4(1.5) = 0.40104166… ;
4.2 ⋅ 3.2
f ( x ) = 1 + 4.2( x + 5) + ( x + 5)2 ln 1.5 = 0.40546510…
2! Error = 0.00442344…
4.2 ⋅ 3.2 ⋅ 2.2
+ ( x + 5)3 1
3! Fifth term = (1.5 – 1)5 = 0.00625
5
4.2 ⋅ 3.2 ⋅ 2.2 ⋅ 1.2
+ ( x + 5) 4 + L The error is smaller in absolute value than the
4! first term of the tail.

Calculus Solutions Manual Problem Set 12-5 321


© 2005 Key Curriculum Press
36. Solve numerically for x close to 2: The answer differs from π by only 1 in the
S 4(x) − ln x = 0.0001 seventh decimal place. The improvement in
x ≈ 1.2263… accuracy is accounted for by the fact that the
Solve numerically for x close to 0.1: inverse tangent series converges much more
ln x − S 4(x) = 0.0001 rapidly for x = 1/2 and x = 1/3 than it does for
x ≈ 0.7896… x = 1. In Problem 17 of Problem Set 12-6,
Interval is about 0.7896… < x < 1.2263… . you will see that the interval of convergence
1 1 1 for the inverse tangent series is −1 ≤ x ≤ 1.
37. a. tan −1 x = x − x 3 + x 5 − x 7 In general, power series converge slowly at
3 5 7
1 9 1 11 the endpoints of the convergence interval.
+ x − x +L
9 11 1 3 1 5 1 7
38. sin x = x − x + x − x +L
1 1 1 1 1 3! 5! 7!
∴ tan −1 1 = 1 − + − + − + L
3 5 7 9 11 1 2 1 4 1 6
The tenth partial sum is S9(1). cos x = 1 − x + x − x +L
2! 4! 6!
9

∑ (–1)
1
S9 (1) = n
= 0.760459904 K x+
1
x3 +
2
x 5+
17
x 7+ L
n=0
(2 n + 1) 3 15 315
4S9(1) = 3.04183961… 1 2 1 4 1 6 1 1 1
π = 3.14159265… 1− x + x − x + L x – x 3 + x 5 – x 7 +L
2! 4! 6! 3! 5! 7!
The error is about 3%.
1 3 1 5 1 7
b. The fiftieth partial sum is S49(1). x – x + x – x +L
2! 4! 6!
4S49(1) = 3.12159465…
π = 3.14159265… 1 3 4 5 6 7
x − x + x −L
The error is about 0.6%. 3 5! 7!
(It is merely an interesting coincidence that 1 3 1 5 1 7
x – x + x –L
although 4S49(1) differs from π in the second 3 6 72
decimal place, several other decimal places 2 5 64 7
later on do match up!) x − x −L
15 7!
c. By the composite argument properties 2 5 1 7
x – x +L
from trig, 15 15
tan  tan −1 + tan −1 
1 1 17 7
 2 3 x −L
315
tan  tan –1  + tan  tan –1 
1 1 1 3 2 5 17 7
 2  3 ∴ tan x = x + x + x + x +L
= 3 15 315
1 – tan  tan –1  ⋅ tan  tan –1 
1 1 S4(0.2) = 0.202710024…
 2   3 tan 0.2 = 0.202710035…
1 1
+ f (i )
( a)
= 2 3 =1 39. Define ai ( x ) = ( x − a)i, the ith term of the
1 1 i!
1– ⋅ ∞
2 3
1 1
general Taylor series. So, f ( x ) = ∑ a ( x ).
i

∴ tan −1 1 = tan −1 + tan −1 , Q .E.D . i=0


2 3 ∞ ∞
dn dn
 9
1  1
2 n +1 We must assume ∑ ai ( x ) = ∑ dx ai ( x );
∑ dx n n
p = 4 S9 = 4  (–1) n i=0 i=0
 n=0 2n + 1 2  that is, the nth derivative of an infinite series is
9 2 n +1  the infinite sum of the nth derivatives of the
1  1
+ ∑ (–1)
n=0
n
2n + 1 3 


individual terms.
dn f ( i ) ( a) d n
For i < n, n ai ( x ) = ⋅ n ( x − a)i
9
(−1)  1 n 2 n +1 2 n +1

∑ 2n + 1  2  + 
1 dx i! dx
=4 
 3 
(i )
f ( a)
n=0 = ⋅0 = 0
= 3.14159257… i!
π = 3.14159265…

322 Problem Set 12-5 Calculus Solutions Manual


© 2005 Key Curriculum Press
dn dn Q5. (1 − x)− 1 (−1 < x ≤ 1)
For i = n, a i ( x ) = an ( x )
dx n dx n Q6.
1
sin 2 x + C
f ( n ) ( a) d n 2
= ⋅ n ( x − a) n
n! dx Q7. 3 sec2 3x Q8. 1
f ( n ) ( a) Q9. e Q10. B
= ⋅ n!( x − a) 0
n! ∞

∑4
dn f ( i ) ( a) d n n 1 2 3 3 4 4
For i > n, n ai ( x ) = ⋅ n ( x − a)i 1. a. n xn = x + x2 + x + x
dx i! dx n =1
4 16 64 256
f (i ) ( x ) 5 5
= i ⋅ (i − 1)(i − 2)K(i − n + 1)( x − a)i −n = 0 + x +L
i! 1024
for x = a.
dn
tn+1 (n + 1) x n+1 4 n
b. L = lim = lim ⋅ n
So n ai ( a) = 0 for i < n and i > n, and n→∞ tn n→∞ 4 n+1 nx
dx
dn x n +1 x
an ( a) = f ( n ) ( a). =lim =
dx n 4 n→∞ n 4

dn
Thus, n ∑
dx i =0
ai ( x ) evaluated at x = a L <1⇔
x
4
x
< 1 ⇔ −1 < < 1 ⇔ −4 < x < 4
4
dn Open interval of convergence is (−4, 4).
is an ( a) = f ( n ) ( a).
dx n c. Radius of convergence = 4.
40. Brook Taylor: 1685−1731 ∞
xn
∑ n⋅2
1 1 2 1 3 1 4
Colin Maclaurin: 1698−1746 2. a. = x+ x + x + x
Sir Isaac Newton: 1642−1727 n =1
n
2 2⋅4 3⋅8 4 ⋅ 16
Gottfried Wilhelm von Leibniz: 1646−1716 1
+ x5 +L
(–1) n+2
1
( x – 1) n+1 5 ⋅ 32
41. a.
tn+1
= n + 1 =
n
| x − 1|
1 n +1 tn+1 x n+1 n ⋅ 2n
tn (–1) n+1 ( x – 1) n b. L = lim = lim n +1 ⋅
n n→∞ tn n→∞ ( n + 1) ⋅ 2 xn
2
b. r10 = for x = 1.2 x n x
11 = lim =
9.5 2 n→∞ n + 1 2
r10 = for x = 1.95
11 x x
20 L <1⇔ < 1 ⇔ −1 < < 1 ⇔ −2 < x < 2
r10 = for x = 3 2 2
11 Open interval of convergence is (−2, 2).
c. r = lim
n
| x − 1| = | x − 1| lim
n
= | x − 1| c. Radius of convergence = 2.
n→∞ n + 1 n→∞ n + 1

(2 x + 3) n (2 x + 3)2
d. r = 1.1 for x = −0.1
r = 1 for x = 0
3. a. ∑
n =1
n
= (2 x + 3) +
2
r = 0.9 for x = 0.1
(2 x + 3)3 (2 x + 3) 4
r = 0.9 for x = 1.9 + + +L
r = 1 for x = 2 3 4
e. Possible conjecture: The series converges to (2 x + 3) n+1 n
b. L = lim ⋅
ln x whenever the value of x makes r < 1, and n→∞ n +1 (2 x + 3) n
diverges whenever the value of x makes r > 1.
n
f. The series should converge for r < 1. = | 2 x + 3 | lim = | 2 x + 3|
x →∞ n + 1
r = |x − 1| < 1 ⇒ −1 < (x − 1) < 1 ⇒
0<x< 2 L < 1 ⇔ |2x + 3| < 1 ⇔ −1 < 2x + 3 < 1
⇔ −2 < x < −1
42. Answers will vary.
Open interval of convergence is (−2, −1).
1
c. Radius of convergence = .
Problem Set 12-6 2
Q1. sin x Q2. sinh x
Q3. e− x Q4. ex

Calculus Solutions Manual Problem Set 12-6 323


© 2005 Key Curriculum Press

(5 x – 7) n x 2 n+3 (2 n + 1)!
4. a. ∑
n=1
2n
L = lim
n→∞ ( 2 n + 3)!
⋅ 2 n+1
x
(5 x – 7) (5 x – 7)2 (5 x – 7)3 1
= + + = x 2 lim = x2 ⋅ 0
n→∞ ( 2 n + 3)(2 n + 2 )
2 4 6
(5 x – 7) 4 ∴ L < 1 for all x and the series converges for
+ +L all x.
8 ∞
(–1) n 2 n
b. L = lim
(5 x – 7) n+1

2n 8. cos x = ∑
n=0
(2 n)!
x
n→∞ 2(n + 1) (5 x – 7) n Note that |(−1)n| can be left out of the ratio.
= | 5 x − 7 | lim
n
= | 5x − 7| x 2 n+2 (2 n)!
n→∞ n + 1
L = lim ⋅ 2n
n→∞ ( 2 n + 2 )! x
L < 1 ⇔ |5x − 7| < 1 ⇔ −1 < 5x − 7 < 1
1
⇔ 1.2 < x < 1.6 = x 2 lim = x2 ⋅ 0
n→∞ ( 2 n + 2 )(2 n + 1)
Open interval of convergence is (1.2, 1.6).
∴ L < 1 for all x and the series converges for
c. Radius of convergence = 0.2. all x.
∞ ∞
n3

1
5. a. ∑
n =1
n!
( x – 8) n 9. sinh x =
n=0
( 2 n + 1 )!
x 2 n+1

8 27 x 2 n+3 (2 n + 1)!
= ( x − 8) + ( x – 8)2 + ( x – 8)3 L = lim ⋅ 2 n+1
2 6 n→∞ ( 2 n + 3)! x
64 1
+ ( x – 8) 4 + L = x 2 lim = x2 ⋅ 0
24 n→∞ ( 2 n + 3)(2 n + 2 )

(n + 1)3 ( x – 8) n+1 n! ∴ L < 1 for all x and the series converges for
b. L = lim ⋅ 3 all x.
n→∞ (n + 1)! n ( x − 8) n ∞


1
10. cosh x = x 2n
 n + 1 3 1 
= | x − 8 | lim 
( 2 n )!
⋅  n=0
n→∞  n  n + 1
 x 2 n+2 (2 n)!
= | x − 8| ⋅ 1 ⋅ 0 = 0 L = lim ⋅ 2n
n→∞ ( 2 n + 2 )! x
L < 1 for all values of x. 1
Series converges for all values of x. = x 2 lim = x2 ⋅ 0
n→∞ ( 2 n + 2 )(2 n + 1)
c. Radius of convergence is infinite. ∴ L < 1 for all x and the series converges for

∑n
n! all x.
6. a. ( x + 2) n ∞


4
1 n
n =1 11. e x = x
2 6 n!
= ( x + 2) + ( x + 2) 2 + ( x + 2)3 n=0
16 81 x n+1 n! 1
24 L = lim ⋅ n = | x | lim = | x| ⋅ 0
+ ( x + 2) 4 + L n→∞ (n + 1)! x n→∞ n +1
256
∴ L < 1 for all x and the series converges for
(n + 1)! ⋅ ( x + 2) n+1 n4
b. L = lim ⋅ all x.
n→∞ (n + 1) 4
n! ⋅ ( x + 2) n ∞
(–1) n n
 n  
4
12. e − x = ∑ n!
x
= | x + 2| lim (n + 1) ⋅  n=0
n→∞
  n + 1  x n+1 n! 1
L = lim ⋅ = | x | lim = | x| ⋅ 0
= | x + 2| lim [(n + 1) ⋅ 1] = ∞ n→∞ (n + 1)! x n n→∞ n + 1
n→∞
The series converges only for |x + 2| = 0 ∴ L < 1 for all x and the series converges for all x.
⇔ x = −2. 13. tn = xnn!
c. Radius of convergence = 0. x n+1 (n + 1)!
L = lim = | x | lim (n + 1) = | x | ⋅ ∞
∞ n→∞ x n n! n→∞
(–1) n
7. sin x =
n=0
∑( 2 n + 1)!
x 2 n+1
L = ∞ for all x ≠ 0; L = 0 at x = 0.
Note that |(−1)n| can be left out of the ratio. ∴ the series converges only for x = 0.

324 Problem Set 12-6 Calculus Solutions Manual


© 2005 Key Curriculum Press
n! n b.
14. tn = x
100 n y
S4
S3
n +1
(n + 1)! x 100 n
L = lim ⋅ 1
n→∞ 100 n+1 n! x n x
1
n +1
= | x | lim → | x| ⋅ ∞ tan–1
S4
S3
n→∞
100
L = ∞ for all x ≠ 0; L = 0 at x = 0.
∴ the series converges only for x = 0. The graphs of the partial sums of P(x) and
∞ tan− 1 x fit very well for −1 < x < 1. The
∑ (2n)!10
1
15. cosh 10 = 2n
partial sums diverge from tan− 1 x for x outside
n=0
this interval.
10 2 n+2 (2 n)!
L = lim ⋅ 2n c. S3(0.1) = 0.09966865238095…
n→∞ ( 2 n + 2 )! 10
d. tan− 1 0.1 = 0.09966865249116… ;
1 Tail = 0.00000000011021…
= 10 2 lim =0
n→∞ ( 2 n + 2 )(2 n + 1)
1
L = 0 < 1 ⇒ series converges. e. First term of tail = (0.1)9 =
9

∑ (–1) n +1 1 0.00000000011111… ,
16. ln 0.1 = (–0.9) n
n which is larger than the tail.
n =1
tn = −(0.9)n/n 18. a. y = x2 sin 2x, from x = 0 to x = 1.5, rotated
about the y-axis.
n tn tn+1 /tn A slice of the region parallel to the axis of
rotation generates a cylindrical shell.
1 −0.9 0.45 dV = 2π x · y · dx = 2πx3 sin 2x dx
1.5
2
3
−0.405
−0.243
0.6
0.675
V= ∫ 0
2π x 3 sin 2 x dx
Integrate by parts.
4 −0.164025 0.72 u dv
5 −0.118098 0.75 x 3 + sin 2x
1
9 −0.043046721 0.81 3x 2 – – 2 cos 2x
1
15 −0.0137260754… 0.84375 6x + – 4 sin 2x
1
35 −0.0007151872… 0.875 6 – 8 cos 2x
1
0 + 16 sin 2x
Ratio seems to approach 0.9.
V = 2π  − x 3 cos 2 x + x 2 sin 2 x
1 3
Proof:
 2 4
(–0.9) n+1 n
L = lim ⋅
1.5
+ x cos 2 x − sin 2 x 
3 3
n→∞ n +1 (–0.9) n  0
4 8
n
= 0.9 lim = 2π  − cos 3 + sin 3
9 21
n +1
n→∞
 16 16 
= 0.9(1) = 0.9, Q .E .D . = 4.662693947…


1.5


1
17. a. P( x ) = (–1) nx 2 n+1 V= 2πx 3 sin 2 x dx ≈ 4.662693947…
n=0
2 n + 1 0

Note that |(−1)n| = 1 for all n. The answers are the same to at least nine
decimal places.
x 2 n +3 2 n + 1
L = lim ⋅ 2 n+1 b. Omitting the 2π , ∫ x sin 2 x dx
3
n→∞ 2 n + 3 x

2n + 1
= x 3 (2 x ) − (2 x )3 + (2 x )5

1 1
= x 2 lim = x 2 ⋅ 1 by l’Hospital’s rule
n→∞ 2 n + 3  3! 5!
L < 1 ⇔ x 2 < 1 ⇔ −1 < x < 1 1 
Open interval of convergence is (−1, 1). − (2 x ) + L dx
7
7! 

Calculus Solutions Manual Problem Set 12-6 325


© 2005 Key Curriculum Press
 4 2 3 6 2 5 8 2 7 10  (–1) n
= ∫ 2x −
 3!
x +
5!
x −
7!
x + L dx

c. tn =
(2 n + 1)n!
x 2 n+1

2 5 23 7 25 9 Note that | (−1)n | = 1.


= x − x + x
5 7 ⋅ 3! 9 ⋅ 5! x 2 n +3 (2 n + 1)n!
L = lim ⋅
2 7 11 n→∞ (2 n + 3)(n + 1)! x 2 n+1
− x +L+C
11 ⋅ 7! 2n + 1 1
∞ = x 2 lim ⋅ lim
2 2 n+1
= ∑
n=0
(–1) n
( 2 n + 5) ⋅ ( 2 n + 1)!
x 2 n+5 + C
n→∞ 2 n + 3 n→∞ n + 1

= x2 ⋅ 1 ⋅ 0 < 1 for all x.


For this series, d. Erf x does seem to be approaching 1 as x
increases, as shown by the following table
2 2 n +3 ⋅ x 2 n + 7 (2 n + 5) ⋅ (2 n + 1)!
L = lim ⋅ generated by numerical integration.
n→∞ ( 2 n + 7) ⋅ ( 2 n + 3)! 2 2 n+1 ⋅ x 2 n+5
2n + 5 1 x erf x
= 4 x 2 lim ⋅ lim
n→∞ 2 n + 7 n→∞ ( 2 n + 3)(2 n + 2 )
1 0.8427007929…
= 4x2 ⋅ 1 ⋅ 0
∴ the series converges for all x and thus 2 0.9953222650…
converges for x = 1.5. 3 0.9999779095…
So, 4 0.9999999845…
2 23 25
V ≈ 2π  (1.5)5 − (1.5) 7 + (1.5)9 5 0.9999999999…
 5 7 ⋅ 3 ! 9 ⋅ 5!
27 29  20. a. Assume this series can be integrated term by
− (1.5)11 + (1.5)13  term.
11 ⋅ 7! 13 ⋅ 9! 
= 4.67164363… sin t 1  1 3 1 5 1 7
= ⋅ t− t + t − t
The answer is within 0.01 of the answer t t  3! 5! 7!
found in part a. 1 9 1 11 K
+ t − t +
2π ⋅ 2 2 n+3 9! 11! 
c. tn+1 = ( −1) n (1.5)2 n+ 7
(2 n + 7) ⋅ (2 n + 3)! = 1−
1 2 1 4 1 6 1 8 1 10 K
t + t − t + t − t +
3! 5! 7! 9! 11!
By table search, | tn+ 1 | < 0.5 × 10− 10 for

n ≥ 10. t 2n
Because n starts at zero for the first term, you
= ∑ (–1)
n=0
n
(2 n + 1)!
would need 11 terms to estimate the volume
Si x =
to 10 decimal places.
x
1 − 1 t 2 + 1 t 4 − 1 t 6 + 1 t 8 − 1 t 10 + K dt
19. a. Assume this series can be integrated term by
term.

0  3! 5! 7! 9! 11! 
x 1 3 1 5 1 7
∫e = x− x + x −
−t 2
f ( x) = dt x
0 3 ⋅ 3! 5 ⋅ 5! 7 ⋅ 7!
= 1 − t 2 + t 4 − t 6 + t 8 − t 10 + K dt
x


1 1 1 1 1 9 1
+ x − x 11 + K
0  2! 3! 4! 5!  9 ⋅ 9! 11 ⋅ 11!
1 3 1 5 1 7 1 9 ∞
= x− x + x − x +
∑ (–1)
x 1
3 5 ⋅ 2! 7 ⋅ 3! 9 ⋅ 4! = n
x 2 n+1
1 n=0
(2 n + 1)(2 n + 1)!
− x 11 + K
11 ⋅ 5! sin t
b. Ratio for is
b. t
t 2 n+2 (2 n + 1)!
y

f (x ) L = lim ⋅
1 n→∞ ( 2 n + 3)! t 2n
x
1
1 = t 2 lim = t2 ⋅ 0
S5 n→∞ ( 2 n + 3)(2 n + 2 )

∴ L < 1 for all values of t.


Series for (sin t)/t converges for all values
The partial sum is reasonably close for of t.
approximately −1.5 < x < 1.5.

326 Problem Set 12-6 Calculus Solutions Manual


© 2005 Key Curriculum Press
Ratio for Si x is ( L + ε ) n+1− k [1 + ( L + ε ) + ( L + ε )2 + K ] =
x 2 n +3 (2 n + 1)(2 n + 1)! ( L + ε ) n+1– k
L = lim ⋅
n→∞ (2 n + 3)(2 n + 3)! x 2 n+1 1 – (L + ε)
So the series converges, Q.E.D.
2n + 1 1
= x 2 lim ⋅ lim 22. L = lim n n
n→∞ 2 n + 3 n→∞ (2 n + 3)(2 n + 2) n→∞

= x ⋅1 ⋅ 0
2 Because ln is a continuous function,
∴ L < 1 for all values of x. ln L = ln ( lim n n ) = lim ln n n
n→∞ n→∞
Series for Si x converges for all values of x. 1 ln n ln x ∞
The radii of convergence for both series are = lim ln n = lim = lim →
n→∞ n n→∞ n x →∞ x ∞
infinite. 1/ x
= lim by l’Hospital’s rule
c. The third partial sum is S2(x). x →∞ 1

1 1 =0
S2 (0.6) = 0.6 − (0.6)3 + (0.6)5 ∴ L = e0 = 1, Q .E.D .
3 ⋅ 3! 5 ⋅ 5!

= 0.5881296 (–1) n+1
Si 0.6 = 0.5881288…
23. ln x = ∑
n =1
n
( x – 1) n
The answers are quite close!
( x – 1) n
d. L = lim n |tn | = lim n
n→∞ n→∞ n
| x – 1| | x – 1|
Tenth
y
= n
= = | x − 1|
Partial Sum
1 lim n 1
n→∞
x L < 1 ⇔ |x − 1| < 1 ⇔ 0 < x < 2
−3π −2π −π π 2π 3π
Open interval of convergence is (0, 2).
−1

∑n
1
Si (x) 24. n xn
n=1

|x n | | x|
S9(x) is reasonably close to Si x for L = lim n = lim = | x | ⋅ 0
−3 π < x < 3π . n→∞ nn n→∞ n

∴ L < 1 for all values of x, and thus the series


21. a. Given L = lim n tn where L < 1. converges for all values of x, Q.E.D.
n→∞
By the definition of limit as n → ∞, there ∞

is a number k > 0 for any ε > 0 such that 25. ∑n x n n

if n > k, then n tn is within ε units of L. n=1

L = lim n |n n x n | = lim |nx | = | x | ⋅ ∞


Thus, n tn < L + ε , Q .E .D . n→∞ n→∞
L = 0 if x = 0 and is infinite if x ≠ 0.
b. L < 1 ⇒ 1 − L > 0
∴ the series converges only if x = 0.
So take any ε < 1 − L

⇒L+ε<L+1−L
∑n
n!
26. xn
⇒ L + ε < 1. n =1
n

c. For all integers n > k, Use the ratio technique.


0 ≤ n tn < L + ε ⇒ 0 ≤ tn < ( L + ε ) n and (n + 1)! ⋅ x n+1 nn
L = lim n +1 ⋅
n→∞ (n + 1) n! ⋅ x n
(L + ε )n < (L + ε )n− k for all n > k because
L + ε < 1, so 0 ≤ tn < (L + ε ) n− k, Q .E .D . nn 1 1
= | x | lim n = | x | lim n = | x| ⋅
n→∞ ( n + 1) n→∞ (1 + 1/n)
d. Because 0 ≤ tn < (L + ε)n− k for all n > k, it e
follows that the tail after tn satisfies 1
L < 1 ⇔ | x | ⋅ < 1 ⇔ −e < x < e
0 ≤ tn+1 + tn+2 + tn+3 + L e
Open interval of convergence is (−e, e).
< (L + ε)n+ 1− k + (L + ε)n+ 2− k + (L + ε)n+ 3− k + K
= (L + ε)n+ 1− k[1 + (L + ε ) + (L + ε )2 + K ],
which converges because L + ε < 1. Problem Set 12-7
e. The tail of the series is increasing and is Q1. geometric Q2. multiplying by 2
bounded above by Q3. common ratio Q4. ln x

Calculus Solutions Manual Problem Set 12-7 327


© 2005 Key Curriculum Press
4 2 16 4 The series converges because the sequence of
Q5. 1 − x + x Q6. −1/3!
2! 4! partial sums is increasing and bounded above
Q7. 2.5 Q8. 1 < x < 7 by 0.2.
r r ∞ ∞
Q9. (2e )i + (3 cos 3t ) j
∫ ∫
2t
c. (1/ x 2 ) dx < R1000 < (1/ x 2 )
1001 1000
Q10. D
1/1001 < R 1000 < 1/1000
5
R1000 ≈ 0.5(1/1001 + 1/1000)

6 6 6 6 6
n +1
1. a. S5 = (–1) =6− + − +
n! 2! 3! 4! 5! = 0.00099950049…
n =1
1 1 S = S1000 + R1000
= 6 − 3 +1− + = 3.8 ≈ 1.643934… + 0.00099950049…
4 20

= 1.644934…
∑ (–1)
6 6 6 6
b. Tail = R5 = n +1
= − + − +K The answer is correct to at least three decimal
n=6
n! 6! 7! 8! places.
(The exact answer is π 2/6 = 1.64493406… .
c. Hypotheses: (1) signs are strictly alternating,
The estimate is actually correct to nine
(2) |tn| are strictly decreasing, and
decimal places.)
(3) lim tn = 0.
n→∞
d. 0.5[1/(n + 1) + 1/n] = 0.0000005 (six
| R5 | < | t6 | = 6/6! = 1120
/
decimal places)

n ≈ 1,999,999.5, so use about 2 million
d. Absolute convergence means that ∑ |t |
n=1
n terms.
5

∑ n = 1+ 2 + 3 + 4 + 5
converges. 1 1 1 1 1
4. a. S5 =
If the convergent series were not absolutely n =1
convergent, it would be called “conditionally 17
convergent.” =2 = 2.2833…
60
e. When you show absolute convergence, you p = 1, a harmonic series.
find the partial sums of |tn|. The partial sums ∞

∑ n = 6 + 7 + 8 + L . The
1 1 1 1
must be increasing because |tn| is positive. b. Tail = R5 =
|tn| is decreasing because the series is n=6

convergent. graph shows the tail for R5 bounded below by



2. a. Comparison test (direct) ∫6
(1/ x ) dx.
b. Integral test tn
c. nth term test 0.2

d. Geometric series test


0.1
e. Ratio test (or ratio technique)
f. Limit comparison test n
5 6 7 8 9
g. p-series test


5
(1/ x ) dx = lim (ln b – ln 6) = ∞

1 1 1 1 1
3. a. S5 = 2 = 1+ + + + 6 b→∞
n 4 9 16 25
n =1 ∴ the series diverges because a lower bound is
1669 infinite.
=1 = 1.463611…
3600 c. Graphing the rectangles to the left of the

n-values leads to R5 < ∞, which does not
∑n
1 11 1
b. Tail = R5 = + 2 += +L
36 49 64 imply that the tail is finite.
n=6
n +1 n +1
The graph shows the tail bounded above by
∞ d. Sn > 1000 if ∫ (1/ x ) dx > 1000 ⇔ ln x =
∫5
(1/ x 2 ) dx = 0.2. 1

ln (n + 1) > 1000 ⇔ n > e1000 − 1


1

tn ≈ 1.97 · 10434 s. It would take approximately


6.24 · 10420 yr.
1 1 1
0.03 5. a. 1 − + − + L
2 3 4
n The series converges because
5 6 7 8 9 (1) strictly alternating signs, (2) strictly
decreasing |tn|, (3) tn → 0.

328 Problem Set 12-7 Calculus Solutions Manual


© 2005 Key Curriculum Press


d. The terms are strictly alternating in sign, the
b. (1/ x ) dx = lim (ln b – ln 1) = ∞
1 b→∞ terms are strictly decreasing in absolute value,

and the terms approach zero for a limit as
Τhe series ∑ |t | diverges, so the given series
n=1
n n → ∞. Thus, the series converges by the
alternating series test.
does not converge absolutely.
Or:
c. S1000 = 0.692647… , S1001 = 0.693646… ,
| Rn | < |tn+1 | for all n ≥ 1, as shown by example
ln 2 = 0.693147… in part c.
| S 1000 − ln 2| = 0.0004997… , |S 1001 − ln 2| = 0
lim |tn+1 | = 0 because it takes the form .
0.0004992… , |t1001| = 1/1001 = n→∞ ∞
0.00009900… ∴ lim | Rn | = 0, and thus the series converges.
n→∞
∴ both partial sums are within |t1001| of ln 2.
Or: Use the ratio technique.
d. No term is left out. No term appears more
than once. 0.6 2 n+3 (2 n + 1)!
L = lim ⋅
1 1 1 1 1 1 n→∞ (2 n + 3)! 0.6 2 n+1
Series is − + − + − + K
2 4 6 8 10 12 1
= 0.36 lim =0
= 1 – + – + K .
1 1 1 1 n→∞ (2 n + 3)(2 n + 2)
2 2 3 4 
1 Because L < 1, the series converges.
∴ the series converges to ln 2.
2 8. The sequence converges because lim tn = 2, a
n→∞
Conditional convergence means that whether (finite) real number.
the series converges, and, if so, what value it The series does not converge because lim tn ≠ 0.
converges to, depends on the condition that n→∞

you do not rearrange the terms. 1 1 1 1 K


1 1 1 9. a. + + + +
6. a. 1 − + − + K 3 8 15 24
4 9 16 Compare with the p-series with p = 2:
The series converges because 1/(n 2 − 1) n2
(1) strictly alternating signs, (2) strictly lim = lim = 1, a positive
1/n 2 n→∞ n − 1
2
n→∞
decreasing |tn|, (3) tn → 0. real number.

∴ the series converges by the limit
b. ∑ |t | converges by the p-series test
n=1
n
comparison test.
because p > 1. b. The p-series with p = 2 begins
1 n2 1 1 1 1
c. L = lim ⋅ = 1, so the ratio test + + + . These terms form a lower
n→∞ ( n + 1)
2
1 4 9 16 25
fails because L is not less than 1. bound, not an upper bound, so the direct
∞ comparison test fails.
∑ (–1)
1
7. a. sin x = n
x 2 n+1 c. If n started at 1, the first term would be 1/0,
n=0
( 2 n + 1 )! which is infinite.
1 ∞
t3 = ( −1)3 ⋅ ⋅ 0.6 2⋅3+1
∑ n! 0.6
1
(2 ⋅ 3 + 1)!
n
10. a. The seventh term of is
1 n=0
= − 0.6 7 = −0.00000555428571K 1
7! t6 = 0.6 6 = 0.0000648.
1 6!
b. S1 (0.6) = 0.6 − 0.6 3 = 0.564
3! 4

∑ n! 0.6
1
1 1 b. S4 = n
= 1.8214
S2 (0.6) = 0.6 − 0.6 3 + 0.6 5 = 0.564648
3! 5! n=0
e0.6 = 1.8221188…
c. R 1 = sin 0.6 − S1(0.6) = 0.0006424…
S4 differs from e0.6 by 0.00071880… , which
R 2 = sin 0.6 − S2(0.6) = −0.0000055266…
is greater than t5 = 0.000648, but not much
|R1| = 0.0006424…
greater. The difference is greater than t5
|t2| = 0.000648
because all subsequent terms are added, not
∴ |R 1| < |t2|
subtracted. It is not much greater than t5
|R2| = 0.0000055266…
because the subsequent terms are very small.
|t3| = 0.0000055542…
∴ |R 2| < |t3|

Calculus Solutions Manual Problem Set 12-7 329


© 2005 Key Curriculum Press
c. The terms of V are bounded below by the
corresponding terms of G, and so the direct
n 5 6 7 comparison test fails in this case.
Tail: 0.000648 0.0000648 0.000005554… 2 n /(3n – 1) 3n
b. L = lim n n = lim n
Geometric n→∞ 2 /3 n→∞ 3 – 1
series: 0.000648 0.0000648 0.00000648 n
3 ln 3
= lim n =1
n→∞ 3 ln 3
Terms of the e0.6 series are formed by
multiplying the previous term: (using l’Hospital’s rule)
0.6 ∴ the V series converges because the G series
tn = tn−1 converges and L is a (finite) positive number.
n
Terms of the geometric series are formed by 13. Divergent harmonic series
multiplying the previous term by 0.1: 14. Convergent p-series, p > 1
tn = 0.1tn−1 15. Convergent alternating series meeting the three
For n ≥ 7, 0.6/n is smaller than 0.1, so the hypotheses
terms of the e0.6 series are smaller than the
16. Divergent p-series, p ≤ 1
corresponding terms of the geometric series.

∑4
Thus, the geometric series forms an upper 3 3 3 3 K
17. = 3+ + + +
bound for the tail of the e0.6 series after n
4 16 64
n=0
term t6.
Converges because it is a geometric series with
d. Geometric series converges to common ratio 1/4, which is less than 1 in
1
0.0000648 ⋅ = 0.000072. absolute value
1 – 0.1

3n

3 9 27
n = 1+ + + +L
e. The tail of the series after t6 is bounded by 18.
0.000072. n=0
4 4 16 64
The entire series is bounded by Converges because it is a geometric series with
S6(0.6) + 0.000072 = 1.8221128 + 0.000072 common ratio 3/4, which is less than 1 in
= 1.8221848. absolute value
e0.6 = 1.8221188… ∞

∑ (2n + 1)! = 1! + 3! + 5! + 7! + K
So the upper bound is just above e0.6 , Q .E .D . 1 1 1 1 1
19.
1 2 3 4 5 K n=0
11. a. + + + + +
1 1 1
1 1 2 6 24 + = 1+
+ +K
n + 1 (n – 1)! n +1 1
= lim 
6 120 5040
L = lim ⋅ ⋅ =0
n→∞ n! n n→∞  n n Converges by comparison with geometric
∴ the series converges because L < 1. series with t0 = 1 and r = 1/6
1 2 3 4 K 1 1 ∞
+ + + + = 1+1+ + + K
∑ (–3)
b. 1 1 1 1 K
1 2 6 24 2! 3! 20. n = 1− + − +
n=0
3 9 27
This is the Maclaurin series that converges
to e1. Converges by the alternating series test. (Terms
n/(n – 1)! n! are strictly alternating. Terms are strictly
c. L = lim = lim decreasing in absolute value. tn approaches zero
n→∞ n/n! n→∞ (n – 1)!
as n approaches infinity.)
= lim n = ∞
n→∞ ∞
n3
∑n
∴ the test fails because the limit of the ratio 8 27 64 125 K
21. =
4 + + + +
is infinite. n=2
– 1 15 80 255 624
2 4 8 16 K Diverges. Use the integral test.
12. a. U: + + + + b
4 10 28 82 ∞ x3

1
2 4 8 16 K dx = lim ln | x 4 − 1|
G: + + + + 2
4
x –1 b→∞ 4
2
3 9 27 81
= lim  ln (b 4 − 1) − 0  = ∞
2 4 8 16 K 1
V: + + + +
2 8 26 80 b→∞  4 
The terms of U are bounded above by the Or: Compare with a harmonic series.
∞ ∞
corresponding terms of G, and so U converges n3
∑ ∑
1
by the direct comparison test. 4 > →∞
n=2
n – 1 n=2 n

330 Problem Set 12-7 Calculus Solutions Manual


© 2005 Key Curriculum Press
1 2 3 4 K 1 1 1 1 K
22. + + + + , divergent because tn does x = −2: + + + +
2 3 4 5 1 2 3 4
not approach zero Diverges. p series with p = 0.5, which is less
–1 1 –1 1 K than 1.
23. + + + + , a convergent alternating
1 2 3 4 Complete interval is [−4, −2).
series meeting the three hypotheses 1
∞ 36. x = −2 : − 1 + 1 − 1 + 1 − 1 + K
∑ sin n = sin 0 + sin 1 + sin 2 + sin 3 + K
3
24. Diverges by the nth term test
n=0 2
= 0 + 0.8414 … + 0.9092 … + 0.1411… + K x = −1 : 1 + 1 + 1 + 1 + 1 + K
3
Diverges. tn does not approach 0 as n → ∞. Diverges by the nth term test
( 4/3) n+1 /(n + 1)
Complete interval is  −2 , – 1  .
tn+1 1 2
25. L = lim = lim = 4/3 > 1.
n→∞ t n→∞ ( 4/3) n /n  3 3
n

Diverges by the ratio test
26. Convergent geometric series with | r| = 7/11 < 1
37. ∑ n( x − 3)
n =1
n

27. Diverges because tn does not approach zero


(n + 1)( x – 3) n+1
t L = lim
28. L = lim n+1 n→∞ n( x – 3) n
n→∞ t
n +1
n

[(n + 1)2 – 1]/2 n+1 = | x − 3 | lim = | x − 3| ⋅ 1


= lim n→∞
n
n→∞ [n 2 – 1]/2 n L < 1 ⇔ |x − 3| < 1 ⇔ 2 < x < 4
1 (n + 1)2 + 1 1 At x = 2 the series is −1 + 2 − 3 + 4 − L , which
=
lim = <1 diverges because the terms do not approach zero.
2 n→∞ n 2 + 1 2
Converges by the ratio test At x = 4 the series is 1 + 2 + 3 + 4 + L , which
∞ b diverges because the terms do not approach zero.
∫ ∫
1
29. dx = lim (ln x ) −1 ( dx/ x ) Interval of convergence is (2, 4).
2 x ln x b→∞ 2

= lim [ln (ln b) − ln (ln 2)] = ∞ 5n ⋅ x n
b→∞
Diverges by the integral test
38. ∑
n =1
n2
30. Converges by direct comparison with the 5n+1 ⋅ x n+1 n2
∞ L = lim ⋅
(n + 1)2 5n ⋅ x n

2 n→∞
convergent p-series 2
n 2
= 5 | x | lim 
n=3 n 
= 5 | x | ⋅1
1 2 6 24 K n→∞  n + 1
31. 1 + 2 + 3 + 4 +
e e e e L < 1 ⇔ 5| x | < 1 ⇔ −0.2 < x < 0.2
Diverges because tn does not approach zero 1 1 1
32. Converges to e by the definition of e At x = −0.2 the series is −1 + − + − L ,
4 9 16
33. x = 1: − 1 + 1 − 1 + 1 − 1 + K which is a convergent alternating series.
Diverges by the nth term test 1 1 1
At x = 0.2 the series is 1 + + + + K ,
x = 9: 1 + 1 + 1 + 1 + 1 + K 4 9 16
Diverges by the nth term test which is a convergent p-series with p = 2.
Complete interval is (1, 9). Interval of convergence is [−0.2, 0.2].
1 1 1 1 K ∞
xn
34. x = −1: − + − +
3 18 81 324
− 39. ∑ n
Converges by the alternating series test n=1
1 1 1 1 K x n+1 n
x = 5: + + + + L = lim ⋅ n
3 18 81 324 n→∞ n + 1 x
Converges by comparison with the geometric
n
1 1 1 1 = | x | lim = | x| ⋅ 1
series + + + +K n→∞ n + 1
3 9 27 81
Complete interval is [−1, 5]. L < 1 ⇔ | x | < 1 ⇔ −1 < x < 1
1 1 1
1 1 1 1 K At x = −1 the series is −1 + − + − K ,
35. x = −4: − + − + − 2 3 4
1 2 3 4
which is a convergent alternating series.
Converges by the alternating series test

Calculus Solutions Manual Problem Set 12-7 331


© 2005 Key Curriculum Press

1 1 1 K ln (n + 1) n
At x = 1 the series is 1 + + + + ,
2 3 4 43. ∑
n=0
n +1
x
which is a divergent harmonic series (p-series
with p = 1). ln (n + 2) ⋅ x n+1 n +1
L = lim ⋅
Interval of convergence is [−1, 1). n→∞ n+2 ln (n + 1) ⋅ x n
∞ n n

∑ ln (n + 2) n +1
(–1) ( x – 6)
40. = | x | lim ⋅ lim
n= 4
n ⋅ 2n n→∞ ln (n + 1) n→∞ n + 2
( x – 6) n+1 n ⋅ 2n 1/(n + 2) n +1
L = lim = | x | lim ⋅ lim
n +1 ⋅ n→∞ 1/( n + 1) n→∞ n + 2
n→∞ (n + 1) ⋅ 2 ( x – 6) n
1 n 1 (by l’Hospital’s rule)
= | x − 6 | lim = | x − 6| ⋅1 n +1 n +1
2 n→∞ n +1 2 = | x | lim ⋅ lim = | x | ⋅1⋅1
1 n→∞ n + 2 n→∞ n + 2
L < 1 ⇔ | x − 6| < 1 ⇔ 4 < x < 8
2 L < 1 ⇔ | x | < 1 ⇔ −1 < x < 1
1 1 1 1 At x = −1 the series is
At x = 4 the series is + + + + K ,
4 5 6 7 ln 1 ln 2 ln 3 ln 4 K
− + − + .
which is a divergent harmonic series (p-series 1 2 3 4
with p = 1). By l’Hospital’s rule,
1 1 1 1
At x = 8 the series is − + − + K , ln (n) 1/n
4 5 6 7 lim = lim = 0.
n→∞ n n→∞ 1
which is a convergent alternating series.
Interval of convergence is (4, 8]. Because the terms decrease in absolute value and
∞ approach zero for a limit, the series converges by
(–1) n+1 ⋅ ( x + 5)2 n
41. ∑
n =1
2n
the alternating series test.
At x = 1 the series is
( x + 5)2 n+2 2n ln 1 ln 2 ln 3 ln 4
L = lim ⋅ + + + +L .
n→∞ 2(n + 1) ( x + 5)2 n 1 2 3 4
n ∞ 1 b 

ln ( x )
= ( x + 5)2 lim = ( x + 5)2 ⋅ 1 dx = lim  (ln x )2 
n→∞ n + 1 x b→∞ 2
1
 1 
L < 1 ⇔ (x + 5)2 < 1 ⇔ −6 < x < −4
= lim  (ln b)2 − 0  = ∞
1
1 1 1 1
At x = −6 the series is − + − + K , b→∞  2 
2 4 6 8
which is a convergent alternating series. Thus, the series diverges by the integral test.
1 1 1 1 Interval of convergence is [−1, 1].
At x = −4 the series is − + − + K , ∞
2 4 6 8
which is a convergent alternating series. 44. ∑ 5( x − 3) n

Interval of convergence is [−6, −4]. n =1

∞ 5( x – 3) n+1
( x + 1) n L = lim = | x − 3|
42. ∑
n =1
n2
n→∞ 5( x – 3)
n

L < 1 ⇔ |x − 3| < 1 ⇔ 2 < x < 4


( x + 1) n+1 n2 At x = 2 the series is −5 + 5 − 5 + 5 − L , which
L = lim 2 ⋅
n→∞ (n + 1) ( x + 1) n diverges because the terms do not approach zero.
2 At x = 4 the series is 5 + 5 + 5 + 5 + L , which
= | x + 1| lim 
n 
= | x + 1| ⋅ 1 diverges because the terms do not approach zero.
n→∞  n + 1
Interval of convergence is (2, 4).
L < 1 ⇔ | x + 1 | < 1 ⇔ −2 < x < 0

4n

1 1 1
At x = −2 the series is −1 + − + − K , 45.
4 9 16 n=0
xn
which is a convergent alternating series.
4 n+1 x n 4
1 1 1
At x = 0 the series is 1 + + + + K , which L = lim ⋅ =
4 9 16
n→∞ x n+1 4 n |x|
is a convergent p-series with p = 2. 4 |x|
Interval of convergence is [−2, 0]. L <1⇔ <1⇔ > 1 ⇔ x < −4 or x > 4
|x| 4

332 Problem Set 12-7 Calculus Solutions Manual


© 2005 Key Curriculum Press
At x = −4 the series is 1 − 1 + 1 − 1 + L , which The first n for which
diverges because the terms do not approach zero. 1 1 1 1
At x = 4 the series is 1 + 1 + 1 + 1 + L , which 1< + + +L+ is n = 5.
2 4 6 2(n – 1)
diverges because the terms do not approach zero.
Intervals of convergence are (−∞, −4) and (4, ∞). c. To make a pile with overhang H, find an n
(Note that the series in Problems 45 and 46 have 1 1 1 2H
such that 1 + + + L + > (this is
negative powers of x and are called Laurent series 2 3 n –1 L
rather than Taylor series.) possible because the harmonic series diverges
∞ to infinity). Then a stack of n blocks will
∑x
1
46. n have total overhang
n=1 H1 + L + Hn−1
xn 1
L = lim = 1 1 1 1
n→∞ n +1 = L + L + L +K+ L
x | x| 2 4 6 2(n – 1)
1 | x| 1  1 1 K 1  1
L <1⇔ <1⇔ > 1 ⇔ x < −1 or x > 1 = L 1+ + + + < L⋅
2H
=H
| x| 1 2  2 3 n + 1 2 L
At x = −1 the series is −1 + 1 − 1 + 1 − L ,
(The achieved overhang is greater than H, so
which diverges because the terms do not approach
one may pull blocks slightly back—moving
zero.
blocks back can only make the pile more
At x = 1 the series is 1 + 1 + 1 + 1 + L ,
stable—until the overhang equals H exactly.)
which diverges because the terms do not approach
zero. d. The theoretical overhang for a stack of 52
Intervals of convergence are (−∞, −1) and (1, ∞). objects is
47. a. Assume all the blocks have equal mass = m, H = H1 + L + H51
with the center of mass at the center of the 1 1 1 1
block, and equal length = L. = L + L + L +L+ L
2 4 6 102
Write Hn = the distance the nth block
= L + + +K+
overhangs the (n + 1)th block. (n = 1 for 1 1 1 1 
the top block.) 2 4 6 102 
Note that according to the rule, Hn = the = 2.2594 K L
distance between the rightmost edge of the slightly more than two-and-a-quarter card
nth block and the center of mass of the pile lengths.
of the top n blocks.
48. The least upper bound postulate says that any
Now, the center of mass of the nth block is non-empty set of real numbers that has an upper
1
2 L units from its rightmost edge, and the bound has a least upper bound. In particular, any
center of mass of the pile of the top n − 1 number less than this least upper bound cannot
blocks is 0 units from (i.e., right on top of ) be an upper bound for the set.
the edge of the nth block according to the The set of real numbers {t1, t2, t3, …} is non-
rule. empty and is bounded above (given). Therefore,
Therefore, the center of mass of the pile of the this set has a least upper bound L. Any number
1 1
top n blocks is L ⋅ m + 0 ⋅ (n − 1)m  less than L is also less than some tD in the set.
nm  2  Claim: L = lim tn
units from the edge of the nth block; that is, b→ ∞

1
Hn = L , Q .E .D . Proof:
2n
Pick a number ε > 0.
b. The total distance the top (first) block Because L is an upper bound for tn, L + ε is also
overhangs the nth block is H1 + H2 + L + an upper bound.
Hn−1 . So for a pile of n blocks, the top block Because L is the least upper bound for tn,
will project entirely beyond the bottom block L − ε is not an upper bound.
if ∴ there exists an integer D > 0 such that
L < H1 + L + Hn−1 = tD > L − ε .
1 1 1 1 But the values of tn are increasing.
L + L + L +L+ L ∴ tn > tD > L − ε for all n > D.
2 4 6 2(n – 1)

Calculus Solutions Manual Problem Set 12-7 333


© 2005 Key Curriculum Press
Keep n > D. c. sinh 5 = 74.2032105…
Then L − ε < t n < L + ε . S9(5) = 74.2032007…
Thus, tn is within ε units of L for all n > D. sinh 5 − S9(5) = 0.00000981… , which is
∴ L = lim tn by the definition of limit as well within the 0.001138… upper bound
n→∞
found by Lagrange form.
n → ∞, Q .E.D .

xn
Problem Set 12-8
3. a. e x = ∑
n=0
n!
Q1. ratio Q2. |common ratio| < 1 The fifteenth partial sum is
Q3. for all values of x Q4. radius = 1 S14(3) = 20.0855234… .
1 1 1 f (15) (c) 15
Q5. x − x 3 + x 5 − x 7 + K b. R14 (3) = ⋅3
3 5 7 15!
Q6. f (15)(x) = e x
S4
∴ M = e3 < 33 = 27
27 15
sin | R14 (3)| ≤ ⋅ 3 = 0.0002962 K
15!
S14(3) is within 3 units of e3 in the fourth
decimal place.
Q7. ln |sec x + tan x| + C
Q8. y′ = sec2 x c. e3 = 20.085536923…
Q9. tan x + C Q10. Newton and Leibniz S14(3) = 20.085523458…
∞ e3 − S14(3) = 0.00001346… , which is within
∑ (2n)! x
1
1. a. cosh x = 2n the 0.0002962… found by Lagrange form.

∑ (–1)
n=0 1
n +1
S5 ( 4) = 27.2699118K 4. a. ln x = ⋅ ( x – 1) n
n =1
n
f ( 2.5+2 ) (c) 2⋅5+2 S8(0.7) = −0.356671944…
b. R5 ( 4) = ⋅4
(2 ⋅ 5 + 2)! f ( 9 ) (c )
(12 ) b. R8 (0.7) = ⋅ (0.7 − 1)9
f ( x ) = cosh x 9!
1 4 –4 f (9)(x) = 8!x − 9
∴ M = cosh 4 < (3 + 2 ) ∴ M = 8!(0.7)− 9
2
= 40.5312… < 41 8!(0.7) –9 1
| R8 (0.7)| ≤ ⋅ (0.3) 9 = (3/7)9
41 12 9! 9
| R5 ( 4) | ≤ ⋅ 4 = 1.4360 K = 5.4195… × 10− 5
12!
S5(4) is within 2 of cosh 4 in the units digit. S8(0.7) is within 6 units of ln 0.7 in the fifth
decimal place.
c. cosh 4 = 27.3082328…
(Note that for ln x, the Lagrange form of the
S5(4) = 27.2699118…
remainder simplifies to
cosh 4 − S5(4) = 0.0383… , which is well n +1
within the 1.4360… upper bound found by 1  | x – 1| 
| Rn ( x )| ≤ ⋅
Lagrange form. n +1  x 
∞ For x < 0.5, the fraction |x − 1|/x is greater
∑ (2n + 1)! x
1 2 n +1
2. a. sinh x = than 1.
n=0 The Lagrange form of the remainder becomes
S9(5) = 74.2032007… infinite as n → ∞ and is thus not useful.)
f ( 2⋅9+3) (c) 2⋅9+3 c. ln 0.7 = −0.356674943…
b. R9 (5) = ⋅5
(2 ⋅ 9 + 3)! S8(0.7) = −0.356671944…
f (21)(x) = cosh x |ln 0.7 − S8(0.3)| = 2.9998 × 10− 6, which is
within the 5.4195… × 10− 5 found by
1
∴ M = cosh 5 < (35 + 2 –5 ) Lagrange form.
2
= 121.5156… < 122 5. For sinh 2, all derivatives are bounded by cosh 2.
1
| R9 (5)| ≤
122 21
⋅ 5 = 0.001138K cosh 2 < (32 + 2 –2 ) = 4.625
21! 2
1
S9(5) is within 2 units of sinh 5 in the third The general term is tn = ⋅ 2 2 n+1.
decimal place. (2 n + 1)!

334 Problem Set 12-8 Calculus Solutions Manual


© 2005 Key Curriculum Press
For six-place accuracy, 1 n
The general term is 5.
4.625 n!
| Rn (2)| ≤ ⋅ 2 2 n+3 < 0.5 × 10 −6.
(2 n + 3)! f ( 20 ) (c) 20 520
R19 (5) = ⋅ 5 = ec ⋅
The second inequality is first true for n = 6. 20! 20!
Use at least seven terms (n = 6). 520
6. For cosh 3, all derivatives are bounded by cosh 3. ∴ ec ⋅ = 5.1234 K × 10 −5
20!
1 ec = 1.30702776…
cosh 3 < (33 + 2 –3 ) = 13.5625
2 c = ln 1.3070… = 0.2677… ,
1
The general term is tn = ⋅ 32 n . which is between 0 and 5.
(2 n)! 1 1 1
For eight-place accuracy, 11. cos 2.4 = 1 − (2.4)2 + (2.4) 4 − (2.4)6
2! 4! 6!
13.5625 2 n+2
| Rn (3)| ≤ ⋅3 < 0.5 × 10 −8. 1
+ (2.4) − 8 1
(2.4) + L
10
(2 n + 2)! 8! 10!
The second inequality is first true for n = 10. = 1 − 2.88 + 1.3824 − 0.2654208
Use at least 11 terms (n = 10). + 0.0273004 L − L
7. For ln x, the nth derivative (n ≥ 1) on [x, 1] The terms are strictly alternating. They are
is bounded by M = (−1)n+ 1(n − 1)!(0.6)− n. decreasing in absolute value after t1, and they
approach zero for a limit as n → ∞.
n!(0.6) –( n+1)
| Rn (0.6)| ≤ ⋅ | 0.6 − 1|n+1 Therefore, the hypotheses of the alternating series
(n + 1)! test apply, and | Rn(2.4) | < | tn+ 1| =
n +1 1
1  2 (2.4)2 n+2 .
= ⋅
n +1  3 (2 n + 2)!
For seven-place accuracy, For six-place accuracy, make
n +1 | Rn(2.4) | < 0.5 × 10− 6.
1  2 The inequality is first true for n = 7.
⋅ < 0.5 × 10 −7.
n +1  3 Use 8 terms (n = 7).
This inequality is first true for n = 32. 1 1 1
Use at least 32 terms. 12. e −2 = 1 + ( −2) + (–2)2 + (–2)3 + (–2) 4
2! 3! 4!
8. For e10, all derivatives are bounded by e10. 1 1
+ (–2) + (–2) + L
5 6
e10 < 310 = 59049 5! 6!
For five-place accuracy, = 1 − 2 + 2 − 1.3333… + 0.6666 … − 0.2666 … + L
59049 The terms are strictly alternating. They are
| Rn (10)| ≤ ⋅ 10 n+1 < 0.5 × 10 −5. decreasing in absolute value after t2, and they
(n + 1)!
approach zero for a limit as n → ∞.
The second inequality is first true for n = 43.
Therefore, the hypotheses of the alternating series
Use 44 terms (n = 43). 1
9. cosh 2 = 3.76219569… test apply, and | Rn ( −2)| < |tn+1 | = ⋅ 2 n+1.
(n + 1)!
S4(2) = 3.76190476… For seven-place accuracy, make
cosh 2 − S4(2) = 0.000290929…
| Rn(−2) | < 0.5 × 10− 7.
1 The inequality is first true for n = 14.
The general term is tn = ⋅ 2 2 n.
(2 n)! Use 15 terms (n = 14).
f ( 2⋅4+2 ) (c) 2⋅4+2 210 10

∑n
1
R4 (2) = ⋅2 = cosh c ⋅ 13. a. S10 = = 1.19753198…
(2 ⋅ 4 + 2)! 10! 3
n =1

210
∫x −3 dx = lim  –0.5 x –2 10  =
b
∴ cosh c ⋅ = 0.000290929K R10 <
10! 10 b →∞

cosh c = 1.03098027… 0.5(10− 2) = 0.005



c = cosh− 11.0309… = 0.2482… ,
∫x −3 dx = lim  –0.5 x –2 11 =
b
R10 >
11 b→∞
which is between 0 and 2.
0.5(11− 2) = 0.00413223…
10. e5 = 148.413159…
R10 ≈ 0.5(0.005 + 0.00413223…) =
S19(5) = 148.413107…
0.00456611…
e5 − S19(5) = 5.1234… × 10− 5

Calculus Solutions Manual Problem Set 12-8 335


© 2005 Key Curriculum Press
S ≈ 1.19753198… + 0.00456611… = R10 ≈ 0.5(0.0996686… + 0.0906598…) =
1.20209810… 0.0951642…
Error < 0.5(0.005 − 0.00413223…) = S ≈ 1.9817928… + 0.0951642… =
0.00043388… (about three decimal places) 2.0769570…
b. Using both the upper and lower bounds, Error < 0.5(0.0996686… − 0.0906598…) =
 ∞ ∞
 0.004504…
error < 0.5  x 3 dx −
 n n +1∫x 3 dx 
 ∫ S ≈ 2.0769570… is correct to at least two
−2 −2 decimal places.
= 0.25n − 0.25(n + 1) .
Make R n < 0.00005.
Solve 0.25n− 2 − 0.25(n + 1)− 2 = 0.000005 to
Rn ≈ 0.5[(π/2 − tan− 1 n) − (π /2 − tan− 1 (n + 1))]
get n = 45.9194… .
= 0.5(tan− 1 (n + 1) − tan− 1 n)
Use 46 terms.
Solve 0.5(tan− 1 (n + 1) − tan− 1 n) = 0.00005
Using only the upper bound, Rn < 0.000005
∞ to get n = 99.4962… .
if ∫
n
x 3 dx < 0.000005. Use 100 terms.
Set 0.5n− 2 = 0.000005n = (0.000005/0.5)− 1 / 2 16. In this p-series, p = 0.5, which is not greater
= 316.2277… , which rounds up to 317 than or equal to 1. Thus, the series diverges and
terms, considerably more that the 46 terms to the remainder is infinite.
give this accuracy by comparing upper and ∞

∑ n! ⋅ 2
1
lower bounds of Rn. 17. e 2 = n

100 n=0
∑n
1
14. a. S100 = 1.05 = 4.698244 … From Example 1, S10 = 7.38899470… .
n =1

By Lagrange form, | R 10 | < 0.0004617… .
x −1.05 dx = lim  –20 x –0.05 100  =

b
R100 < Use a geometric series as an upper bound.
100 b→∞
1 11 1
20(100− 0.05 ) = 15.886564… t11 = ⋅ 2 and t12 = ⋅ 212
∞ 11! 12!
x −1.05 dx = lim  –20 x –0.05 101 =

b
R100 > t 1
101 b→∞
Common ratio r = 12 =
20(101− 0.05 ) = 15.878662… t11 6
R100 ≈ 0.5(15.886564… + 15.878662…) = 1 11 1
15.882613… ∴ | R10 | < ⋅2 ⋅ = 0.00006156 …
11! 1
S ≈ 4.698244… + 15.882613… = 1–
6
20.580858… The geometric series gives a better estimate of
Error < 0.5(15.886564… − 15.878662…) = the remainder than does the Lagrange form.
0.003950… (about two decimal places)

 ∞
∑ (–1)

 1 n
b. Error < 0.5  x 1.05 dx − ∫ ∫ 18. e −2 = ⋅2 n
x 1.05 dx 
 n n +1  n=0
n!
= 20n− 0.05 − 20(n + 1)− 0.05 S10 = 0.135379188…
Solve 20n− 0.05 − 20(n + 1)− 0. 05 = 0.000005 to 211
get n = 111840.2309… . | R10 | < |t11 | = = 0.000051306 …
11!
Use 111,841 terms.
With a value of p such as 1.05, which is This number appears to be a better estimate of
the error. However, it represents an error of ≈
closer to 1 than 3 is, it takes more terms
| R 10 |/S(10) = 0.03789…%.
because the terms approach zero more slowly.
e2 ≈ 1/S10 = 7.38665971…
10

∑n
1 A 0.037…% error for this value would be
15. a. S10 = = 1.9817928…
n=0
2
+1 0.002799… , which is a worse estimate of the
The series converges because the terms of the error than that by Lagrange or by geometric
tail starting at t1 are bounded above by the series.
(In general, an error of ε% in1/f (x) gives a
convergent p-series with p = 2.
ε
b maximum error of in the value of f (x).
∫ dx = lim  tan –1 x 10  =
1 b
b. R10 < 1 – ε /100
10 x + 1
2
b →∞
So an error of 0.03789…% in 1/e2 means an
π /2 − tan− 110 = 0.0996686… 0.03789K
b error of = 0.03788K% in e 2 .)
∫ dx = lim  tan –1 x 11 =
1 b
R10 > 1 – 0.0003789K
11 x + 1
2
b→∞

π /2 − tan− 111 = 0.0906598…

336 Problem Set 12-8 Calculus Solutions Manual


© 2005 Key Curriculum Press
250 The second inequality is first true when n is
19. a. = 39.7887357K
2π 348. Because both numerator and denominator
Thus, 250 radians is 39 complete cycles plus may overflow most computers, you can
0.7887… additional cycle, or calculate values of ln | Rn(250) | as follows:
b = (2π)(0.7887…) = 4.9557730… radians. 2 n +3
sin b = −0.970528019… ln | Rn (250)| = (2 n + 3) ln 250 − ∑ ln i
sin 250 = −0.970528019… (Checks.) i =2

The value of b can be calculated efficiently Then compare the values with ln (0.5 ·10− 10).
using the fraction part command. For a So you would need to use 349 terms.
typical grapher, b = f Part(250/(2π))2π. Unfortunately, even this procedure would not
be practical because the terms themselves
b. From Figure 12-8c, you can tell that the value
would have to be calculated to ten or more
of c is one cycle back from the value of b.
decimal places, and they are so large that each
c = b − 2π = −1.32741228…
term overflows most computers’ capacities.
Check:
sin c = −0.970528019… e. The program will have the following steps.
sin 250 = −0.970528019… (Checks.) The particular commands will depend on the
In general: grapher or computer used.
If b is in [0, π/2], then c = b. • Put in a value of x.
If b is in (π /2, 3π /2], then c = π − b. • Find b, as shown in part a.
If b is in (3π /2, 2π ], then c = 2π − b.
• Find c, as shown in part b.
c. From Figure 12-8c, you can tell that the
• Find d, as shown in part c.
value of d is a quarter-cycle ahead of the value
of c. The value of the sine is the opposite of • Choose the function and sign, as shown
the corresponding value of cos d. in part c.
π • Calculate and display the answer.
d = + c = 0.243384039K
2 1
Check: 20. For sin 1, | Rn (1)| < | tn+1 | = ⋅ 12 n+3 =
(2 n + 3)!
−cos d = −0.970528019… 1
sin 250 = −0.970528019… (Checks.) .
(2 n + 3)!
In general:
1
π π π < 0.5 × 10 −23.
c ∈ − , −  : d = + c and sin x = − cos d Set
(2 n + 3)!
 2 4 2
π This inequality is first true for n = 11.
c ∈ − , 0 : d = − c and sin x = − sin d Use at least 12 terms (n = 11).
 4  Using the technique in Problem 19,
π
c ∈ 0,  : d = c and sin x = sin d 1
 4 | Rn (1)| < (π /4)2 n+2 < 0.5 ⋅ 10 −23.
(2 n + 2)!
π π π
c ∈  ,  : d = − c and sin x = cos d
4 2 2 The second inequality is first true for n = 10.
You would save only one term by the method of
d. For x in [0, π/4], both the sine and cosine
Problem 19.
series meet the hypotheses of the alternating
series test. Thus, the error in S5(x) is bounded 21. a. Apply the mean value theorem to f ′(x) on
by | t6 |, the first term of the tail. | t6 | is greater [a, x]. There is a number x = c in (x, a) such
for the cosine series than for the sine series. that
The maximum of | t6 | in the interval is at f ′( x ) – f ′( a )
f ′′(c) =
x = π /4. x–a
1 ⇒ f ′(x) = f ′(a) + f ″(c)(x − a), Q.E.D.
∴ | R5 ( x )| < | t6 (π /4)| = (π /4)2⋅6+2

= 3.8980… × 10− 13,


(2 ⋅ 6 + 2)! b. ∫ f ′( x ) dx = ∫ f ′(a) dx + ∫ f ′′(c)( x − a) dx
1
which is small enough to guarantee that sin x f ( x ) = f ′( a) x + f ′′(c) ⋅ ( x – a)2 + C
will be correct to ten decimal places. 2
Substituting the initial condition (a, f (a))
For direct calculation, gives
1
| Rn (250)| < ⋅ 250 2 n+3 < 0.5 ⋅ 10 −10 f (a) = f ′(a)a + f ″(c)(0) + C ⇒
(2 n + 3)! C = f (a) − f ′(a)a

Calculus Solutions Manual Problem Set 12-8 337


© 2005 Key Curriculum Press
f ( x) = 2!c2 = f ″(0) = 0 ⇒ c2 = 0
1 3!c3 = f ″′(0) = 0 ⇒ c3 = 0…
f ′( a) x + f ′′(c) ⋅ ( x – a)2 + f ( a) − f ′( a)a
2 ∴ series is 0 + 0x + 0x2 + 0x3 + … , Q .E .D .
1
f ( x ) = f ( a) + f ′( a)( x − a) + f ′′(c)( x − a) 2 , b. Each partial sum of the Maclaurin series
2 equals zero for any value of x. Thus, the
Q .E .D . sequence of partial sums converges to zero
c. Apply the mean value theorem to f ″(x) for all x. But f (x) does not equal zero except
on [a, x]. at x = 0. Thus, the series converges to f (x)
There is a number x = c in (a, x) such that only at x = 0.
f ′′( x ) – f ′′( a) −2 1 1
f ′′′(c) = c. e − x = 1 + ( − x −2 ) + (– x –2 )2 + (– x –2 )3 + L
x–a 2! 3!
⇒ f ′′( x ) = f ′′( a) + f ′′′(c)( x − a) −2 1 −4 1 −6
Integrate once to get f ′(a). = 1− x + x − x +L
2! 3!

∫ f ′′( x ) dx = ∫ f ′′(a) dx + ∫ f ′′′(c)( x − a) dx d. The fourth partial sum, S3(2) =


0.7786458333… .
1 −2
f ′( x ) = f ′′( a) x + f ′′′(c) ⋅ ( x – a)2 + C f (2) = e −2 = e −0.25 = 0.7788007830 K
2
Use (a, f ′(a)) as an initial condition. The partial sum is close to f (2), so it is
1 reasonable to make the conjecture that the
f ′( a) = f ′′( a)a + f ′′′(c)(0) + C ⇒ Laurent series converges to f (2).
2
C = f ′( a) − f ′′( a)a 23. Using the Lagrange form of the remainder, the
1 value of ex is given exactly by
f ′( x ) = f ′′( a) x + f ′′′(c)( x − a)2 k


2 1 n
ex = x + Rk ( x ), where
+ f ′( a) − f ′′( a)a n!
1 n=0
f ′( x ) = f ′( a) + f ′′( a)( x − a) + f ′′′(c)( x − a) 2 f ( k +1) (c) k +1
2 Rk ( x ) = x and c is between 0 and x.
( k + 1)!
Integrate again to get f(x).
M
| Rk ( x )| ≤ | x |k +1
∫ f ′( x ) dx = ∫ f ′(a) dx + ∫ f ′′(a)( x − a) dx ( k + 1)!
Because all derivatives of e x equal e x, the value of
+ ∫ f ′′′(c)( x − a) dx
1 2
2 M for any particular value of x is also e x, which
is less than 3x, if x ≥ 0; or 1, if x < 0.
1
f ( x ) = f ′( a ) x + f ′′( a)( x − a) 2 3x
2 lim | Rk ( x )| < lim | x |k +1
k →∞ k →∞ ( k + 1)!
1
+ f ′′′(c)( x − a)3 + C which approaches 0 as k → ∞ by the ratio
6
Use (a, f ″(a)) as an initial condition. technique.
1 1 Because the remainder approaches zero as n
f ( a) = f ′( a)a + f ′′( a)(0) + f ′′′(c)(0) + C approaches infinity, ex is given exactly by
2 6 ∞


⇒ C = f ( a ) − f ′( a ) a 1 n
ex = x , Q.E.D.
1 n=0
n !
f ( x ) = f ′( a) x + f ′′( a)( x − a) 2
2!
1
+ f ′′′(c)( x − a)3 + f ( a) − f ′( a)a Problem Set 12-9
3!
1 Review Problems
f ( x ) = f ( a) + f ′( a)( x − a) + f ′′( a)( x − a) 2 R0. Answers will vary.
2!
1 9
+ f ′′′(c)( x − a) , Q .E.D .
3 R1. f ( x ) = and P( x ) = 9 + 9 x + 9 x 2 + 9 x 3 + L
3! 1– x
d. The technique is mathematical induction. y
P6
e – x –2 , if x ≠ 0
22. a. f ( x ) =  f
20

 0, if x = 0 x
1
It is given that f (n)(0) = 0 for all n > 0.
P5
c0 = f (0) = 0
c1 = f ′(0) = 0

338 Problem Set 12-9 Calculus Solutions Manual


© 2005 Key Curriculum Press
The graph shows that P5(x) and P6(x) are close to R4. a. e0.12 = 1.127496851…
f (x) for x between about −0.7 and 0.6, and bear S3(0.12) = 1.127488, which is close to e0.12 .
little resemblance to f (x) beyond ±1. b. cos 0.12 = 0.9928086358538…
P5(0.4) = 14.93856 S3(0.12) = 0.9928086358528, which is close.
P6(0.4) = 14.975424 c. sinh (0.12) = 0.1202882074311…
f (0.4) = 15 S3(0.12) = 0.1202882074310… , which is
∴ P6(0.4) is closer to f (0.4) than P5(0.4) is, close.
Q .E .D . d. ln 1.7 = 0.530628251…
P 5(x) = 9 + 9x + 9x 2 + 9x 3 + 9x 4 + 9x 5; S20(1.7) = 0.530612301… , which is close.
P5(0) = 9 ln 2.3 = 0.83290912…
P5′ ( x ) = 9 + 18 x + 27 x 2 + 36 x 3 + 45 x 4 ; P5′ (0) = 9 S20(2.3) = −4.42067878… , which is not
P5′′( x ) = 18 + 54 x + 108 x 2 + 180 x 3 ; P5′′(0) = 18 close.
P5′′′( x ) = 54 + 216 x + 540 x 2 ; P5′′′(0) = 54 R5. a. A Maclaurin series is a Taylor series expanded
f (x) = 9(1 − x)− 1; f (0) = 9 about x = 0.
f ′(x) = 9(1 − x)− 2; f ′(0) = 9 b. Substitute t = x + 1 into
f ″(x) = 18(1 − x)− 3; f ″(0) = 18 1 1
f ′′′( x ) = 54(1 − x ) −4 ; f ′′′(0) = 54 ln t = (t − 1) − (t – 1)2 + (t – 1)3
2 3
∴ P5′ (0) = f ′(0), P5′′(0) = f ′(0), 1
and P5′′′(0) = f ′′′(0) − (t – 1) 4 + L .
4
Pn(x) is a subseries of a geometric series. 1 1 1
ln ( x + 1) = x − x 2 + x 3 − x 4 + L
R2. a. Series is 3 + 2.7 + 2.43 + 2.187 + L . 2 3 4
1 – 0.910 ∞
After 10 days, S10 = 3 ⋅ = (–1) n+1 n
1 – 0.9 = ∑
n =1
n
x
19.5396… .
About 19.5 mm increase in 10 days c. Assume one may integrate this function term
1 by term.
S = 3⋅ = 30

1 1 3 1 4
1 – 0.9 ln ( x + 1) dx = x 2 − x + x −L+ C
About 30 mm increase eventually 2 3⋅2 4⋅3

(–1) n+1
b. Let x be the amount invested to have
0.5 million dollars at the end of 19 years.
= ∑ (n + 1)n x
n =1
n +1
+C

Interest rate is 10% per year, so the amount


at the end of a year is 1.1 times the amount at d. ∫ ln ( x + 1) dx = ( x + 1) ln ( x + 1) − ( x + 1) + C
1
the beginning of the year. = x ln ( x + 1) + ln ( x + 1) − x + C (C = C1 − 1)
x(1.119) = 0.5 1 1 1
x = 0.5(1.1− 19) = 0.081753995… = x2 − x3 + x4 − x5 + L
2 3 4
They must invest $81,754.00 now in order 1 2 1 3 1 4
to make the last payment. + x − x + x − x +L− x + C
2 3 4
The total to invest is the sum 1 2 1 3 1 4
0.5(1.1−1 ) + 0.5(1.1−2 ) + L + 0.5(1.1−19 ). = x − x + x − L + C,
2 3⋅2 4⋅3
This is the nineteenth partial sum of the which is the same as the series in part c.
geometric series with first term 0.5(1.1)− 1
t cos t 2 dt = t 1 − (t 2 )2 + (t 2 ) 4 − L dt
x x

∫ ∫
1 1
and common ratio 1.1− 1. e.
0 0  2! 4! 
1 – 1.1–19
S19 = 0.5(1.1−1 ) ⋅ = 4.182460045…
=  t – t 5 + t 9 – t 13 + L dt
x


1 1 1
1 – 1.1–1
They must invest $4,182,460.05 now to 0  2! 4! 6! 
make all 19 payments. 1 2 1 6 1 1
= x − x + x 10 − x 14 + L
R3. P(x) = c0 + c1x + c2x2 + c3x3 + c4x4 + L 2 6 ⋅ 2! 10 ⋅ 4! 14 ⋅ 6!
f (x) = 7e3x ⇒ f (0) = 7 ⇒ c0 = 7 (Note that the series can be transformed to
f ′(x) = 21e3x ⇒ f ′(0) = 21 ⇒ c1 = 21
=  x 2 − ( x 2 )3 + ( x 2 )5 − ( x 2 ) 7 + L
1 1 1 1
f ″(x) = 63e3x ⇒ f ″(0) = 63 ⇒ 2!c2 = 63 ⇒ 2 3! 5! 7! 
c2 = 31.5 x


1 1
f ′′′( x ) = 189e 3 x ⇒ f ′′′(0) = 189 ⇒ = sin x 2 , and t cos t 2 dt = sin x 2. )
2 0 2
c3 = 189/3! = 31.5

Calculus Solutions Manual Problem Set 12-9 339


© 2005 Key Curriculum Press
x
interval. Above x = 2 the partial sums

1
f. tan −1 x = dt
0 1+ t2 diverge rapidly to ±∞. Below x = 0 the partial
x

∫ [1 − t
sums give answers, but there are no real
= 2
+ (t 2 )2 − (t 2 )3 + (t 2 ) 4 − L ] dt
0 values for ln x.
(| t | ≤ 1)
1 – 0.810
1 1 1 1 R7. a. S10 = 1000 ⋅ = 4463.129088 (exactly)
= x − x3 + x5 − x7 + x9 − L 1 – 0.8
3 5 7 9 ∞

∑ 0.8
g. f(3) = 5 ⇒ c0 = 5 1000
b. S = 1000 n
= = 5000
f ′(3) = 7 ⇒ c1 = 7 n=0
1 – 0. 8
f ″(3) = −6 ⇒ c2 = −6/2! = −3 S − S10 = 536.870912, which differs from the
f ′′′(3) = 0.9 ⇒ c3 = 0.9 / 3! = 0.15 limit by about 10.7%.
∴ f (x) = 5 + 7(x − 3) − 3(x − 3)2 + c. “Tail”
0.15(x − 3)3 + L d. “Remainder”

∑ = (−3)

R10 < x −3 dx = lim − b −2 + (10 –2 )

R6. a. −n
( x − 5) n 1 1
e.
n =1 10 b→∞  2 2 
1 1 1 = 0.005
= − ( x – 5) + ( x – 5)2 − ( x – 5)3 + L ∞
R10 > x −3 dx = lim − b −2 + (11–2 )

3 9 27 1 1
(–3) –( n+1) ( x – 5) n+1 1 11 b→∞  2 2 
b. L = lim = | x − 5| = 0.004132…
n→∞ (–3) – n ( x – 5) n 3
Series converges because the tail is bounded
L < 1 ⇔ |x − 5| < 3 ⇔ 2 < x < 8 above by 0.005.
Open interval of convergence is (2, 8). S = S 10 + R 10 ≈ 1.197531… + 0.5(0.005 +
Radius of convergence = 3 0.004132…) = 1.202098…

R10 is approximately 0.5(0.005 − 0.004132…)
∑ (2n)! x
1
c. cosh x = 2n
= 0.0004338… , so S10 is correct to about
n=0
three decimal places.
x 2 n+2 (2 n)!
L = lim ⋅ 2n 1 1 1 1
n→∞ ( 2 n + 2 )! x f. − + + + +L
4 3 22 59
1
= x 2 lim = x2 ⋅ 0 1/(n 3 – 5) n3
n→∞ ( 2 n + 2 )(2 n + 1) L = lim = lim 3 =1
3
n→∞ 1/n n→∞ n – 5
L < 1 for all x.
Series converges for all x, Q.E.D. (Apply l’Hospital’s rule three times.)
∴ the series converges because L is a positive
1 1
d. e1.2 = 1 + 1.2 + (1.2)2 + (1.2)3 real number.
2! 3! The terms of the F series begin
1
+ (1.2) + L 4
1 1 1 1
4! + + + +L .
S4(1.2) = 3.2944 (the fifth partial sum) 1 8 27 64
e1.2 = 3.32011692… Although the F series converges, its terms
Error = e1.2 − S4(1.2) = 0.02571692… (after t1) are less, not greater, than the
1 corresponding terms of the S series, so
The first term of the tail is t5 = (1.2)5 =
5! the comparison test is inconclusive.
0.020736. g. 2/1! + 4/2! + 8/3! + 16/4! + 32/5! + L
The error is greater than t5, but not much = 2 + 2 + 1.3333… + 0.6666…
greater. ∞
e. + 0.2666 … + L = ∑ 2 /n!
n =1
n

y S11
The terms are decreasing starting at t2, which
ln
1 can be seen numerically, above, or
x
algebraically by the fact that the next term is
1 2
S10
formed by multiplying the numerator by 2
and the denominator by more than 2.
R1 is bounded by the geometric series with
first term 2 and common ratio
The open interval of convergence is (0, 2).
1.3333…/2 = 2/3.
Both partial sums fit ln well within this

340 Problem Set 12-9 Calculus Solutions Manual


© 2005 Key Curriculum Press
Because | common ratio | is less than 1, which converges because it is a p-series
the geometric series converges with p = 2.
(to 2/(1 − 2/3) = 6). Interval of convergence is [2.9, 3.1].
Thus, the tail after the first partial sum is ∞ n
(–1) n ( x + 1)
bounded above by a convergent geometric ii. ∑
n =1
n ⋅ 2n
series, Q.E.D.
h. The given series converges because, as ( x + 1) n+1 n ⋅ 2n
L = lim n +1 ⋅
written, it meets the three hypotheses of the n→∞ (n + 1) ⋅ 2 ( x + 1) n
alternating series test. It does not converge
1 n 1
absolutely because replacing all minus signs = | x + 1| lim = | x + 1| ⋅ 1
2 n→∞ n + 1 2
with plus signs gives the divergent harmonic
series. 1
L <1⇔ | x + 1| < 1 ⇔ | x + 1| < 2
The given series is the Taylor series for ln x 2
expanded about x = 1 and evaluated at x = 2. ⇔ −3< x <1
The remainders approach zero, so the series At x = −3 the series is
converges to ln 2.
1 1 1
Rearrange the series this way: 1 + + + + L , which is a divergent
2 3 4
1 – 1  − 1 +  1 – 1  − 1 +  1 – 1  harmonic series.
 2  4  3 6  8  5 10  At x = 1 the series is
1 1 1 1
− +L −1 + − + − L , which converges
12 2 3 4
Each term of the series appears exactly once. by the alternating series test.
Simplifying inside the parentheses and Interval of convergence is (−3, 1].
factoring out 1/2 gives ∞

∑ n! = 10 + 10 + 5 + 1.6666K
10
1 1 1 1 1 1 k. i.
− + − + − +L
2 4 6 8 10 12 n=0
+ 0.4166 K + L
=  – + – + – + L
1 1 1 1 1 1 1
2 1 2 3 4 5 6  The tail after S0 is bounded above by the
convergent geometric series with first
The series in parentheses is the original series term 10 and common ratio 0.5. Thus, the
that converges to ln 2. So the series as series converges.
rearranged converges to 0.5 ln 2, Q.E.D.
(Other justifications are possible.)
i. |R 10000| < |t10001| = 1/10001

= 0.0000999900…
Upper bound is 1/10001.
ii. ∑ (n
n =1
–3
+ 5 –1 ) = 1.2 + 0.325 + 0.2370… +

∞ n 0.215625 + 0.208 + 0.2046… +


10 n ( x – 3)
j. i. ∑
n =1
n2
0.2029... + L
Diverges because tn → 0.2, not 0,
10 n+1 ( x – 3) n+1 n2 as n → ∞
L = lim ⋅
n→∞ (n + 1)2 10 n ( x − 3) n iii. Converges. The general term can be
2
rewritten 3(2/5)n, so the series is a
= 10 | x − 3 | lim 
n  convergent geometric series with common
= 10 | x − 3 | ⋅ 1
n→∞  n + 1 ratio r = 2/5.
L < 1 ⇔ 10 |x − 3| < 1 ⇔ 2.9 < x < 3.1 iv. Diverges. p-series with p = 1/3, which is
At x = 2.9 the series is not greater than 1.
1 1 1 v. Converges by the ratio technique
−1 + − + − L ,
4 9 16 (n + 4)! 3! ⋅ n! ⋅ 3n
which converges by the alternating series lim ⋅
n→∞ 3! ⋅ (n + 1)! ⋅ 3n+1 (n + 3)!
test.
At x = 3.1 the series is 1 n+4 1
= lim = ⋅1 < 1
1 1 1 3 n→∞ n +1 3
1+ + + + L ,
4 9 16

Calculus Solutions Manual Problem Set 12-9 341


© 2005 Key Curriculum Press

sinh 0.6 − S3(0.6) = 2.7862… × 10− 8

1
R8. a. cosh 2 = ⋅ 22n
(2 n)! f ( 2⋅3+3) (c) cosh c
n=0
R3 (0.6) = ⋅ 0.6 2⋅3+3 = ⋅ 0.6 9
Fourth partial sum is S3(2). 2⋅3+3 9!
The (2n)th derivative of cosh x is cosh x, = 2.7862… × 10− 8
so all derivatives are bounded by cosh c = 1.00328…
1 c = cosh− 1 1.00328… = 0.0809… , which is
cosh 2 < (32 + 2 –2 ) = 4.625. in the interval (0, 0.6).
2
4.625 8 ∞

∑ (–1)
| R3 (2)| ≤ ⋅ 2 = 0.02936 K n +1 1
8! e. ln x = ( x – 1) n
n =1
n
Error is less than 0.03. | Rn(1.3) | < | tn+ 1 |

∑ n! ⋅ 3
1 For 20-place accuracy, make
b. e 3 = n
1
n=0 (1.3 – 1) n+1 < 0.5 × 10 −20.
n +1
All derivatives of ex are equal to ex, so all
This inequality is first true for n = 35.
derivatives are bounded by e3 < 33 = 27.
Use at least 35 terms.
For 20-place accuracy,
50

∑n
27 1
| Rn (3)| ≤ ⋅ 3n+1 < 0.5 × 10 −20. f. S50 = = 1.08232064 K
(n + 1)! n =1
4

The second inequality is first true for n = 33. ∞

∫ x −4 dx = lim −(1/3) x −3 =
b
Use at least 34 terms (n = 33). R50 < 
50 b→∞  50 

c. Using the Lagrange form of the remainder, the (1/3)(50− 3) = 0.000002666…


value of cosh 4 is given exactly by ∞

∫ x −4 dx = lim −(1/3) x −3 51  =
b
k R50 >

1 b→∞  
cosh 4 = ⋅ 4 2 n + Rk ( 4), where 51

n=0
(2 n)! (1/3)(51− 3) = 0.000002512…
f ( 2 k + 2 ) (c ) 2 k + 2 The series converges because the sequence of
Rk ( 4) = ⋅4 and c is between partial sums is increasing, and the tail after
(2 k + 2)!
0 and 4. S50 is bounded above by 0.000002512… .
R50 ≈ 0.5(0.000002666… + 0.000002512…)
M
| Rk ( 4)| ≤ ⋅ 4 2 k +2 = 0.000002589…
(2 k + 2)! S ≈ 1.082232064… + 0.000002589…
Because all even derivatives of cosh x equal = 1.082323235…
cosh x, for any value of x between 0 and 4 we Error < 0.5(0.000002666… − 0.000002512…)
can use cosh 4 for M, and cosh 4 is less than = 0.0000000769… (about seven decimal
1 4 –4 places)
(3 + 2 ) = 40.53125.
2
Use M = 41.
41 Concept Problems
lim | Rk ( 4)| < lim ⋅ 4 2 k +2
k →∞ k →∞ ( 2 k + 2 )!
C1. Recall that i = –1, i 2 = −1, i 3 = −i, i 4 = 1,
4 2 k +2
= 41 lim =0 so i4n = 1 and i4n+ 2 = −1 for all n.
k →∞ ( 2 k + 2 )!
1 1 1
By the ratio technique, this fraction a. cos ix = 1 − (ix )2 + (ix ) 4 − (ix )6
2! 4! 6!
approaches zero as k approaches infinity. 1
Therefore, because the remainder approaches + (ix )8 − L
8!
zero as k approaches infinity, cosh 4 is given
i2 i4 i6 i8
∞ = 1 − x2 + x 4 − x6 + x8 − L

1
exactly by cosh 4 = ⋅ 4 2 n , Q .E .D . 2! 4! 6! 8!
n=0
( 2 n )! –1 2 1 4 –1 6 1 8
= 1− x + x − x + x −L
∞ 2! 4! 6! 8!
∑ (2n + 1)! ⋅ 0.6
1 2n +1
d. sinh 0.6 = 1 2 1 4 1 6 1 8
n=0
= 1+ x + x + x + x +L
2! 4! 6! 8!
S3(0.6) = 0.636653554… = cosh x, Q .E .D .
sinh 0.6 = 0.636653582…

342 Problem Set 12-9 Calculus Solutions Manual


© 2005 Key Curriculum Press
1 1 1 Substituting this value gives
b. sin ix = ix − (ix )3 + (ix )5 − (ix ) 7 + L
3! 5! 7! 120 1

i2 i4 5 i6 7 
tan 4 tan −1 1
− tan −1 1 
= 119 239 =1
= ix − i x 3 + i x −i x +L  5 239  1 + 120 ⋅ 1
3! 5! 7!
119 239
–1 3 1 5 –1 7
= ix − i x + i x − i x + L −1 1 −1 1 π
3! 5! 7! Thus, 4 tan − tan = tan −1 1 = , Q .E.D .
5 239 4
= i  x + x 3 + x 5 + x 7 + L
1 1 1 The two series are
  3 5 7
4 tan −1 = 4 ⋅ −   +   −   + K
3! 5! 7! 1 1 4 1 4 1 4 1
= i sinh x, Q .E .D . 5 5 3  5 5  5 7  5
3 5
− 
4 1 
+ 
1 1 4 1 
1 1 1 tan −1 = 4⋅ −L
c. e ix = 1 + ix + (ix )2 + (ix )3 + (ix ) 4 239 239 3  239  5  239 
2! 3! 4! 2 n +3
Rn   <
1 1 4  1
+ (ix )5 + L  5  2n + 3  5 
5!
2 n +3
i2 i3 i4 i5
Rn 
1  1  1 
= 1 + ix + x 2 + x 3 + x 4 + x 5 + L <
2! 3! 4! 5!  239  2 n + 3  239 
–1 2 – i 3 1 4 i 5
= 1 + ix + x + x + x + x +L | Total remainder |
2 n +3 2 n +3
2! 3! 4! 5! 1   1 
+
1 
1 2 1 4 < 4 ⋅   
= 1− x + x − L 2n + 3  5  239  
2! 4!
To get π accurate to 50 places, as shown, the
+ i  x − x 3 + x 5 − L
1 1
remainder for π must be less than 0.5 × 10− 50.
 3! 5! 
For π/4, the remainder must be less than
= cos x + i sin x, Q .E .D . 0.125 × 10− 50.
d. Using the formula in part c (Euler’s formula): The inequality
2 n +3 2 n +3
e i π = cos π + i sin π = −1 + i ⋅ 0 = −1, 1   1  1   −50

  
4 +  < 0.125 × 10
Q .E .D . 2n + 3  5  239  
is first true for n = 34.
C2. tan  4 tan −1 − tan −1
1 1 
Use at least 35 terms.
 5 239 
C3. a. y″ + 9xy = 0; y = 5 and y′ = 7 when x = 0.
tan  4 tan –1  – tan  tan –1
1 1 
y = c 0 + c 1x + c 2x 2 + c 3x 3 + c 4x 4 + c 5x 5
 5  239 
= + c6 x 6 + L
1 + tan  4 tan –1  ⋅ tan  tan –1
1 1 
   y′ = c1 + 2c2x + 3c3x2 + 4c4x3 + 5c5x4
5 239 
+ 6c6 x 5 + L
tan  4 tan –1  –
1 1 y″ = 2c2 + 6c3x + 12c4x2 + 20c5x3
 5  239 + 30c6 x 4 + L
=

1 + tan 4 tan –1 1 

1
 b. Substitute y = 5 into the y-equation: c0 = 5
5  239
Substitute y′ = 7 into the y′-equation: c1 = 7
To evaluate tan  4 tan −1  , recall that
1
c. Constant term: 2c2 = 0 ⇒ c2 = 0
 5 1
2 tan A x -term: 6c3 + 9c0 = 0 ⇒ c3 = (–9 ⋅ 5) = −7.5
tan 2 A = . 6
1 – tan 2 A x 2 -term: 12c4 + 9c1 = 0 ⇒
Therefore, 1
c4 = (–9 ⋅ 7) = −5.25
2 12
tan  2 tan −1  =
1 5 5 1
2 = x -term: 20c5 + 9c2 = 0 ⇒ c5 = (–9 ⋅ 0) = 0
3
 5
1–  
1 12 20
 5 x -term: 30c6 + 9c3 = 0 ⇒
4

10 1
c6 = (–9)(–7.5) = 2.25

and tan 2 ⋅ 2 tan −1 1 
= 12 120 30
 2 =
5 d. y = 5 + 7x + 0x 2 − 7.5x 3 − 5.25x 4 + 0x 5
1–  
5 119
 12  + 2.25 x 6 + L
S6(0.3) = 5 + 7(0.3) − 7.5(0.3)3 − 5.25(0.3)4
+ 2.25(0.3)6 = 6.85661525

Calculus Solutions Manual Problem Set 12-9 343


© 2005 Key Curriculum Press
e. To ascertain convergence or divergence, notice 1
f ′′ ( x ) = − x −2 , f ′′(1) = −1, 2! c2 = −1, c2 = −
that y can be written as three separate series. 2
(–9) (–9)2 f ′′′( x ) = 2 x −3 , f ′′′(1) = 2, 3! c3 = 2, c3 =
1
y = c0 + c0 x 3 + c0 x 6 + L
2⋅3 2 ⋅3⋅5⋅6 3
(–9) (–9)2 f ( 4 ) ( x ) = −3! x −4 , f ( 4 ) (1) = −3!, 4! c4 = −3!,
+ c1 x + c1 x 4 + c1 x 7 + L 1
3⋅ 4 3⋅ 4 ⋅6⋅ 7 c4 = − L
4
(–9) (–9)2
+ c2 x 2 + c2 x 5 + c2 x 8 + L 1
∴ ln x = ( x − 1) − ( x – 1)2 + ( x – 1)3
1
4⋅5 4⋅5⋅7⋅8 2 3
If 0.3 is substituted for x, the first and second 1
− ( x – 1) + L , Q .E.D .
4
series have terms that are strictly alternating, 4
decreasing in absolute value, and approaching T8. 1000 + 999 + L converges because
zero for a limit as n approaches infinity. r = 0.999 < 1.
Thus, these series converge by the alternating (It converges to 1000/(1 − 0.999) = 1,000,000.)
series test. The third series is zero because 0.0001 + 0.0002 + L diverges because r = 2 ≥ 1.
c2 = 0. Thus, the entire series for y converges

when x = 0.3. (2 x – 5) n
T9. ∑
n=1
3n
Chapter Test (2 x – 5) n+1 3n
L = lim ⋅
1 2 1 3 1 n→∞ 3n + 3 (2 x – 5) n
T1. e − x = 1 − x + x − x + L + ( −1) n ⋅ x n + L
2! 3! n! n
= |2 x − 5| lim = |2 x − 5| ⋅ 1
Note: The last ellipsis mark is necessary or this n→∞ n + 1
would stand for a Taylor polynomial (finite L < 1 ⇔ |2x − 5| < 1 ⇔ −1 < 2x − 5 < 1
number of terms), not a Maclaurin series ⇔ 4 < 2x < 6 ⇔ 2 < x < 3
(infinite number of terms). Open interval of convergence is (2, 3).
5 Radius of convergence is 0.5.
T2. = 5 − 5x + 5x 2 − 5x 3 + L
1+ x T10. At x = 2 the series is
Geometric series, common ratio r = −x 1 1 1 1
− + − + − L , which converges by
1 1 3 6 9 12
T3. cos x = −1 + ( x – π )2 − ( x – π ) 4
2! 4! the alternating series test.
1 At x = 3 the series is
+ (x – π ) − L 6
1 1 1 1
6! + + + + L , which is a divergent
f ( 6 ) (c ) 6 3 6 9 12
T4. R5 ( x ) = x , where c is between 0 and x. harmonic series (or 1/3 of p-series with
6! p = 1).
f (n)( x ) = ( −1) n (n!)(1 + x ) − (n+1) ⇒ f ( 6 ) (c) = 6! c −7 Converges at x = 2, diverges at x = 3
6! c –7 6 x 6
∴ R5 ( x ) = x = 7 , where 0 < c ≤ x.
x


1
6! c T11. f ( x ) = 2 dt
0 1+ t
1 1 1
sin ( x 2 ) = x 2 − ( x 2 )3 + ( x 2 )5 − ( x 2 ) 7 + L
x
T5.
3! 5! 7! = ∫ (1 – t 2
+ t 4 – t 6 + L) dt
0
1 6 1 10 1 14
= x − x + x − x +L
2
1 1 1
3! 5! 7! = x − x3 + x5 − x7 + L
∞ 3 5 7

1
= (–1) n x 4 n+2 ∞

∑ (–1)
+ 1
n=0
( 2 n 1)! = n
x 2 n+1
T6. The alternating harmonic series n=0
2n + 1
1 1 1
1 − + − + L converges conditionally, but (The same as tan− 1 x)
2 3 4
x 2 n +3 2 n + 1
not absolutely. The condition is that the terms T12. L = lim ⋅
remain in the order presented and not be n→∞ 2 n + 3 x 2 n+1
rearranged. 2n + 1
T7. f(x) = ln x, f(1) = 0, c 0 = 0 = x 2 lim = x2 ⋅1
2n + 3
n→∞
f ′(x) = x − 1 , f ′(1) = 1, c1 = 1 L < 1 ⇔ x 2 < 1 ⇔ −1 < x < 1

344 Problem Set 12-9 Calculus Solutions Manual


© 2005 Key Curriculum Press
At x = −1 the series is R100 ≈ 0.5(0.2 + 0.19900743…) =
1 1 1
−1 + − + − L , which converges by the 0.19950371…
3 5 7
S = S 100 + R 100 ≈ 2.41287409… +
alternating series test.
0.19950371… = 2.61237781…
1 1 1
At x = 1 the series is 1 − + − + L , which (As additional information, the error in R100
3 5 7
converges by the alternating series test. is less than 0.5(0.2 − 0.1900743…) =
Interval of convergence is [−1, 1]. 0.0004962… , making S correct to about two
decimal places.)
T13. f (0.6) ≈ S19(0.6) (the 20th partial sum)
T20. Answers will vary.
= 0.540419500…
T14. f (0.6) ≈ 0.540419500… numerically
Problem Set 12-10
T15. f (0.6) = tan− 1 0.6 = 0.540419500… exactly
The answers to Problems T13 and T14 are correct Cumulative Review Number 1
to at least ten decimal places. 1. Limit: See Sections 2-2 and 2-5.
T16. The first term of the tail for S19(0.6) is Derivative: See Sections 3-2 and 3-4.
1 Indefinite integral: See Section 5-3.
t20 = (0.6 41 ) = 1.9562 … × 10 −11 , which agrees Definite integral: See Section 5-4.
41
with the observation that S19(0.6) is correct to at 2. a. Continuity at a point: See Section 2-4.
least ten decimal places. b. Continuity on an interval: See Section 2-4.
1 1 1
T17. cosh x = 1 + x 2 + x 4 + x 6 + L c. Convergence of a sequence: A sequence
2! 4! 6! converges if and only if lim tn exists.
n→∞


1
= x 2n d. Convergence of a series: A series converges
n=0
(2 n)! if and only if the sequence of partial sums
T18. All even derivatives of cosh x equal cosh x. converges.
Derivatives are bounded by e. Natural logarithm: See Section 3-9.
1 1 f. Exponential: ax = ex ln a
cosh 3 = (e 3 + e –3 ) < (33 + 2 –3 ) = 13.5625 = M.
2 2 3. a. Mean value theorem: See Section 5-5.
For ten-place accuracy, b. Intermediate value theorem: See Section 2-6.
13.5625 2 n+2 c. Squeeze theorem: See Section 3-8.
| Rn (3)| ≤ ⋅3 < 0.5 × 10 −10.
(2 n + 2)! d. Uniqueness theorem for derivatives: See
The second inequality is first true for n = 11. Section 6-3.
Use at least 12 terms (n = 11). e. Limit of a product property: See Section 2-3.
T19. a. p-series, with p = 1.5 f. Integration by parts formula: See Section 9-2.

b. ∫1
x −1.5 dx = lim [ −2 b −0.5 + 2(1−0.5 )] = 2
b→∞
g. Fundamental theorem of calculus: See
Section 5-6.
∴ the series converges because the integral h. Lagrange form of the remainder: See
converges, Q.E.D. Section 12-8.
(As additional information, this calculation
i. Parametric chain rule: See Section 4-7.
also proves that R1 is bounded above by 2
and thus that S is bounded above by 3.) j. Polar differential of arc length: See
Section 8-7.
c. S100 = 2.41287409…
x

d. R100 < ∫
100

x −1.5
dx 4. a. f ( x ) = ∫
3
1 + sech t dt ⇒

= lim [ −2 b −0.5 + 2(100 −0.5 )] = 0.2 f ′( x ) = 1 + sech x


b→∞
∞ b. f (x) = ax ⇒ f ′(x) = ax ln a

−1.5
R100 > x dx c. f (x) = x a ⇒ f ′(x) = axa− 1
101

= lim [–2 b –0.5 + 2(101–0.5 )] d. f (x) = xx ⇒ ln f (x) = x ln x


b→∞ 1/f (x) · f ′(x) = ln x + x · (1/x)
= 0.19900743… f (x) = (ln x + 1) – f (x)
f ′(x) = x x ln x + x x

Calculus Solutions Manual Problem Set 12-10 345


© 2005 Key Curriculum Press
e. e6x cos 3x dx u dv 40 40


2 2
e 6x + cos 3x A= 2(10 y)1/2 dy =
⋅ (10 y)3/2
1 0 10 3 0
6e 6x 3 sin 3x
= 3200/3 = 1066.6666… yd
– 2
1
36e 6x +
– 9 cos 3x (Or: Area = 2/3 of circumscribed rectangle =
(2/3)(1600) = 3200/3, etc.)
1 2
= e 6 x sin 3 x + e 6 x cos 3 x c. dF = p dA = k(40 − y) ⋅ 2(10y)1/2 dy
3 3
40


− 4 e cos 3 x dx
6x F= ∫ 0
dF = 17066.6 … k lb (exactly


256,000k/15)
5 e 6 x cos 3 x dx
d. dM = y dF = y ⋅ k(40 − y) ⋅ 2(10y)1/2 dy
1 2 40
= e 6 x sin 3 x + e 6 x cos 3 x + C1
3 3 M= ∫ 0
dM = 292, 571.4 … k lb-yd


6x
e cos 3 x dx (exactly 10,240,000k/35)
1 6x 2 M 10240000 k/35 1
= e sin 3 x + e 6x cos 3 x + C e. F ⋅ y = M , y = = = 17 yd
15 15 F 256000 k/15 7
By symmetry, x = 0.

1
f. cosh 5 x sinh x dx = cosh 6 x + C
Center of pressure is at  0, 17  .
6 1
 7
∫ sec
3
g. x dx
7. a. z = 30 − 0.5y
1 1 b. For a cross section,
= sec x tan x + ln | sec x + tan x | + C
2 2 A = 2xz = 2(10y)1/2(30 − 0.5y).
A = 101/2(60y 1/2 − y 3/2)

−1 1
h. (sin 5 x ) cos 5 x dx = ln | sin 5 x | + C A′ = 101/ 2(30y− 1/ 2 − 1.5y1/ 2)
5
cos 7 x – 1 –7 sin 7 x = (101/ 2)(y− 1/ 2)(30 − 1.5y)
i. lim = lim A′ = 0 ⇔ 30 − 1.5y = 0 ⇔ y = 20
x→0 13 x 2 x→0 26 x
A′ is infinite ⇔ y = 0.
–49 cos 7 x 49
= lim =− A(0) = 0
x→0 26 26 A(20) = 565.6854… (exactly 400 2 )
j. L = lim (1 – x )3/ x A(40) = 400
x→0
3 ln (1 – x ) –3 Maximum at y = 20; minimum at y = 0
ln L = lim = lim = −3
x→0 x x→0 1 – x c. dV = 2xz dy = 101/2(60y 1/2 − y 3/2) dy
L = e− 3 = 0.0497… 40

5. a.
dy
= 0.2 x − 0.3 y + 0.3, (1, 8)
V= ∫ 0
dV = 19200 (exactly)
dx Use 19200/5 = 3840 truckloads.
10 y d. dL = dx 2 + dy 2 = 1 + 0.04 x 2 dx
20
L= ∫dL = 92.9356 … ≈ 92.9 yd
−20
r r r
8. r = (100 cos 0.03t )i + (50 sin 0.03t ) j
r r r
v = ( −3 sin 0.03t )i + (1.5 cos 0.03t ) j
r
Speed = | v | = (–3 sin 1.5)2 + (1.5 cos 1.5) 2
x
10
= 2.9943… ≈ 2.99 ft/s
t sin u

b. If x = 9, y ≈ 5.413… , which agrees with the


9. Si t = ∫
0 u
du

1
u – u + L du
graph. t

∫ u  u – 3! u
1 1 5 1 7
= 3
+
6. a. p = k(40 − y) 0 5! 7! 
b. y = 0.1x 2 ⇒ x = (10y)1/2  u – u + L du
t

∫ 1 – 3! u
1 1 4 1 6
dA = 2x dy = 2(10y)1/2 dy = 2
+
0 5! 7! 

346 Problem Set 12-10 Calculus Solutions Manual


© 2005 Key Curriculum Press
∫ x sinh 2 x dx
1 3 1 5 1 7
=t− t + t − t +L 6. u dv
3 ⋅ 3! 5 ⋅ 5! 7 ⋅ 7! x + sinh 2x
1
(–1) n 1 – 2 cosh 2x
+ t 2 n+1 + L 1
(2 n + 1)(2 n + 1)! 0 +
4 sinh 2x

t 2 n +3 (2 n + 1)(2 n + 1)! 1 1
L = lim ⋅ = x cosh 2 x − sinh 2 x + C
n→∞ (2 n + 3)(2 n + 3)! t 2 n+1 2 4
3 x + 14
dx = 
4 
∫ ∫
(2 n + 1) –1
= t 2 lim = t2 ⋅ 0 7. + dx
n→∞ ( 2 n + 3)(2 n + 3)(2 n + 2 ) ( x + 3)( x – 2)  x + 3 x – 2
∴ L < 1 for all values of t, and the series = −ln | x + 3 | + 4 ln | x − 2 | + C
1
x + x 3 + x 5 + x 7 + L dx
converges for all values of t.
∫ ∫
sinh x 1 1 1
8. dx =
Third partial sum is x x 3! 5! 7! 
1 1
= 1 + x 2 + x 5 + x 6 + K dx

S2 (0.6) = 0.6 − (0.6 3 ) + (0.6 5 ) 1 1 1
3 ⋅ 3! 5 ⋅ 5!  3! 5! 7! 
= 0.5881296 1 3 1 5 1 7 …
= x+ x + x + x + +C
| R2 | < | t3 | =
1
(0.6 7 ) = 0.0000007934 … 3 ⋅ 3! 5 ⋅ 5! 7 ⋅ 7!
7 ⋅ 7! ∞
n( x − 5) n
The answer is correct to within ±1 in the sixth
decimal place.
9. ∑
n =1
3n
0.6 sin u
(n + 1)( x – 5) n+1 3n
Si 0.6 =
0 ∫ u
du ≈ 0.588128809… L = lim
n→∞ 3n+1

n( x – 5) n
Note that this answer agrees with the third partial
1 n +1 1 1
sum to within 1 in the sixth decimal place. = | x − 5 | lim = | x − 5| ⋅ 1 = | x − 5|
3 n→∞ n 3 3
10. r = 5 + 4 cos θ
1
1 L < 1 ⇔ | x − 5| < 1 ⇔ − 3 < x − 5 < 3
dA = (5 + 4 cos θ )2 dθ 3
2 Open interval of convergence is 2 < x < 8.

A= ∫ dA ≈ 103.6725… ≈ 103.7 ft 2 (exactly 33π )
1 1

∫x ∫x
−0.998 −0.998
0
10. dx = lim+ dx
0 a→ 0 a
dV dV
11. = kV ⇒ = k dt 1 1
dt V = lim+ x 0.002
a→ 0 0.002
ln | V | = kt + C a

V = C 1e kt = lim+ (500 − 500 a 0.002


)
a→0
At t = 0, V = 300.
= 500
300 = C 1
dV 11. y = x 2
= −5 when V = 300 9
dt
−5 = 300 k ⇒ k = −
1
y=
∫ 3
x 2 dx
= 39 =
1 1 3
⋅ x
9

60 9–3 3 6 3
∴ V = 300e− ( 1 / 6 0 ) t 12. f ( x) = x 2
At t = 10, V = 300e− 1/6 = 253.9445… ≈ f ( 4) = 16
253.9 million gal. f ( 3.99) = 15.9201, which is within 0.08 unit
of 16.
f ( 4.01) = 16.0801, which is not within 0.08
Cumulative Review Number 2 unit of 16.
1. Derivative: See Sections 3-2 and 3-4. Thus, δ = 0.01 is not small enough to keep
f (x) within 0.08 unit of 4.
2. Definite integral: See Section 5-4. 10
3. Mean value theorem: See Section 5-5.
x
13. V = ∫
2
A dx

4. f ( x ) = ∫ 3
g(t ) dt ⇒ f ′( x ) = g( x ) 2
≈ [153 + 4(217) + 2(285) + 4(319) + 343]
3

∫ tanh
1
5. 5
x sech 2 x dx = tanh 6 x + C = 2140 ft3
6

Calculus Solutions Manual Problem Set 12-10 347


© 2005 Key Curriculum Press
14. r = 4 sin 2θ 1 1
23. ln x = ( x − 1) − ( x – 1)2 + ( x – 1)3 − L
1 2 2 3
dA = r dθ = 8 sin 2 2θ dθ
2 (–1) n+1
+ ( x – 1) n + L
π /2 n
A= ∫ 0
8 sin 2 2θ dθ ≈ 6.2831 ≈ 6.28 ft 2
L = lim
( x – 1) n+1

n
(exactly 2π) n→∞ (n + 1) ( x – 1) n
15. (x/5)2 + (y/3)2 = 1 n
= | x − 1| lim = | x − 1| ⋅ 1 = | x − 1|
n→∞ n + 1
y = ± 0.6 25 – x ( Use + .) 2

L < 1 ⇔ | x − 1 | < 1 ⇔ −1 < x − 1 < 1


dA = 2 y dx = 1.2 25 – x 2 dx
⇔ 0<x< 2
5
A= ∫ dA ≈ 17.6021K ≈ 17.6 square units
1
1 1 1
At x = 0 the series is −1 − − − − L ,
2 3 4
(exactly 15(sin −1 1 − sin −1 0.2) − 1.2 6 ) which is a divergent harmonic series.
16. y = 0.0016x4 1 1 1
At x = 2 the series is 1 − + − + L , which
dA = (16 − 0.0016x4) dx 2 3 4
10 converges because it meets the three hypotheses
A= ∫ −10
(16 – 0.0016 x 4 ) dx = 256 ft 2
of the alternating series test.
∴ interval of convergence is 0 < x ≤ 2, Q.E.D.
17. dL = dx 2 + dy 2 = 1 + (0.0064 x 3 )2 dx
1
24. | Rn | < | tn+1 | = ( x – 1) n+1
10
n +1
L= ∫ −10
dL ≈ 42.5483… ≈ 42.55 ft
For ln 1.4 to 20 places, make
18. p = 62.4(16 − y) 0.4 n+1
dA = 2x dy = 10y1/4 dy < 0.5 × 10 −20.
n +1
dF = p dA = 62.4(16 − y) ⋅ 10y1/4 dy
Solving numerically gives n > 45.817… .
16
F= ∫ 0
dF ≈ 113595.73… ≈ 113, 600 lb Use 46 terms.
25. If the velocity is 0 ft/s at time t = 0, the ship
 exactly 113595 11  speeds up, approaching approximately 34 ft/s
 15  asymptotically as t increases.
ln x ∞ If the velocity is 50 ft/s at time t = 0, the ship
19. lim y = lim →
x →∞ x →∞ x ∞ slows down, again approaching 34 ft/s
1/ x asymptotically as t increases.
= lim =0
x →∞ 1 (The graphs are shown here. The differential
equation is dv/dt = 0.7(34 − v).)
(1/ x )( x ) – (ln x )(1) 1 – ln x
20. y′ = =
x2 x2 v
y′ = 0 ⇔ ln x = 1 ⇔ x = e = 2.718… ft
There is a maximum at x = e because y′ goes
from positive to negative there.
(–1/ x )( x 2 ) – (1 – ln x )(2 x ) –3 + 2 ln x
21. y′′ = =
x4 x3
y″ = 0 ⇔ ln x = 1.5 ⇔ x = e = 4.4816… 1.5

≈ 4.48 ft
There is a point of inflection at x ≈ 4.48 ft t

because y″ changes sign there.


22. r r r
26. r = (ln t )i + (sin 2t ) j
y
r r r
1
v = (1/t )i + (2 cos 2t ) j
r r r
a = ( −1/t 2 )i + ( −4 sin 2t ) j
x
40

348 Problem Set 12-10 Calculus Solutions Manual


© 2005 Key Curriculum Press
Cumulative Review Number 3
dx = 
–2 
∫x ∫
3 x – 11 5
11. + dx
1. δ is clearly smaller than necessary. + 2x – 3
2  x + 3 x – 1
f (x ) = 5 ln | x + 3 | − 2 ln | x − 1| + C
L+ε

∫ sin
−1 u dv
L 12. x dx
sin –1 x + 1
L–ε 1 –
x
x √1 – x 2
c – δ c c+δ

2. See Sections 3-2 and 3-4 for definitions of ∫


= x sin −1 x − (1 − x 2 ) −1/2 ( x dx )


derivative. 1
= x sin −1 x + (1 − x 2 ) −1/2 ( −2 x dx )
Graphical meaning: slope of tangent line 2
Physical meaning: instantaneous rate of change
= x sin −1 x + 1 − x 2 + C
3. g( x ) = ∫ f ( x ) dx if and only if g′( x ) = f ( x ). 13. Fundamental theorem of calculus
s See Section 5-6 for statement.
4. ∫r
f (t ) dt = lim Ln = lim Un , where Ln and Un are
∆t →0 ∆t →0 14. See Figure 5-5b.
lower and upper Riemann sums, respectively, x

provided the two limits are equal. 15. f ( x ) = ∫


3
h(t ) dt ⇒ f ′( x ) = h( x )

5. l’Hospital’s rule 16. f (x) = xe− x


x cos x 0 f ′(x) = e− x − xe− x
lim 5x → f ″(x) = −e− x − e− x + xe− x = e− x(x − 2)
x→0 1 – e 0
cos x – x sin x 1 f ″(x) = 0 ⇔ x = 2
= lim 5x → f ″(x) changes sign at x = 2.
x→0 –5e –5
= −0.2 ∴ the only point of inflection is at x = 2.
17. y = sin x from x = 0 to x = 2.
6. y = tan (sin 5x)
y′ = sec2 (sin 5x) ⋅ 5 cos 5x dL = dx 2 + dy 2 = 1 + cos 2 x dx
2

∫ dL ≈ 2.3516K
Chain rule
L=
7. y = (5x − 3)(2x + 7)4(x − 9) 0

ln y = ln (5x − 3) + 4 ln (2x + 7) + ln (x − 9) 16 16

y′ = y 
5
+
8
+
1 
18. a. ∫ 0
x −3/4 dx = lim+ 4 x 1/4
a→0 a
 5x – 3 2 x + 7 x – 9 
= lim+ (8 – 4 a1/4 ) = 8
a→ 0
8. y = tan x −1

tan y = x, sec2 y y′ = 1 8 1
b. Average value = =
1 1 16 – 0 2
y′ = =
sec y 1 + tan –1 y
2
19. r = 10 cos θ
1 dA = 50 cos2 θ dθ
y′ = 1
1+ x2 A= ∫ 50 cos 2 θ dθ ≈ 13.3478…


1 0.5
9. sin 7 x cos x dx = sin 8 x + C (exactly 12.5(1 + sin 2 − sin 1))
8 r r
r
20. r = t 2 i + 3t −1 j
10. ∫ x 2 + 9 dx x = 3 tan θ r r
v = 2ti − 3t −2 j
r
dx = 3 sec2 θ dθ r r r
v (1) = 2i − 3 j
x 2 + 9 = 3 sec θ
Speed = 13 = 3.6055…

= 9 sec 3 θ dθ r
Distance from origin is |r | = t 4 + 9t –2 .
9 9 r
= sec θ tan θ + ln | sec θ + tan θ | + C1 d |r | 1 4
2 2 = (t + 9t –2 ) −1/2 ( 4t 3 − 18t −3 )
9 x2 + 9 x 9 dt 2
x2 + 9 x
= ⋅ + ln + + C1 = −7/ 10 at t = 1
2 3 3 2 3 3
Distance is decreasing at 2.2135… .
1 9
= x x 2 + 9 + ln x2 + 9 + x + C
2 2

Calculus Solutions Manual Problem Set 12-10 349


© 2005 Key Curriculum Press
21. y = cos x f ( x + h) – f ( x )
dV = 2π x ⋅ y ⋅ dx 3. f ′( x ) = lim
h→0 h
π /2
V= ∫ 0
2π x cos x dx ≈ 3.5864 …
f ′(c) = lim
x →c
f ( x ) – f (c )
(exactly 2π (π/2 − 1)) x–c
22. a. y = −1.5x + 6 4. f (x) = ex
A = xy = −1.5x 2 + 6x lim f ( x ) = e 2
x →2
A′ = −3x + 6 If f (x) = e 2 + 0.1, x = ln (e 2 + 0.1) =
A′ = 0 ⇔ −3x + 6 = 0 ⇔ x = 2 2.01344… .
A(0) = 0, A(4) = 0, A(2) > 0 If f (x) = e 2 − 0.1, x = ln (e 2 − 0.1) =
Thus, maximum area is at x = 2, Q.E.D. 1.98637… .
b. V = π x 2y = π (−1.5x 3 + 6x 2) On the left, keep x within 0.01362… unit of 2.
V′ = π (−4.5x 2 + 12x) On the right, keep x within 0.01344… unit of 2.
2
V ′ = 0 ⇔ x = 0 or x = 2 So you must keep x within 0.01344… unit of 2.
3 5. L = lim f ( x ) if and only if for any ε > 0 there is
V (0) = V ( 4) = 0 and V 2  > 0
 2 x →c
 3 a δ > 0 such that if x is within δ units of c but
2 not equal to c, then f (x) is within ε units of L.
Thus, maximum volume is at x = 2 .
3 6. ε = 0.1, δ = 0.01344…
1 7. See Figure 1-3a.
23. V ≈ (2)[51 + 4(37) + 2( 41) + 4(63) + 59]
3 8. Distance ≈ 17.4 m
2
= 394 ft 3
3 r
x
24. a. erf x = (2π −1/2 ) ∫e
−t 2
dt
0
x 10

∫ e − t dt
2
f ( x) =
0
t

= 1 – t 2 + t 4 – t 6 + t 8 – L dt
x


1 1 1 1 1.3 1.6 1.9 2.2 2.5 2.8
0  2! 3! 4! 
1 3 1 5 1 7 1 9
= x− x + x − x + x −L 1
3 5 ⋅ 2! 7 ⋅ 3! 9 ⋅ 4! 9. Distance ≈ (0.3)[7 + 4(9) + 2(13) + 4(12) +
3

2(10) + 4(8) + 5] = 17.4, which agrees with
∑ (–1)
1
b. f ( x ) = n
x 2 n+1 Problem 8.
n=0
(2 n + 1)n!
10. v(t) = te− t
x 2 n +3 (2 n + 1)n!
L = lim ⋅ Distance ≈ 0.4[v(0.2) + v(0.6) + v(1) + v(1.4) +
n→∞ ( 2 n + 3)( n + 1)! x 2 n+1 v(1.8)] = 0.601474…
(2 n + 1)
= x 2 lim = x2 ⋅ 0 2 2
n→∞ ( 2 n + 3)( n + 1)
∫ te
−t
11. Distance = dt = −te − t − e − t
L < 1 for all values of x, and thus the series 0 0

converges for all values of x, Q.E.D. = −2e − e + 0 + 1 = 1 − 3e − 2 = 0.593994…


−2 −2

The difference is 0.00748… , which is about


1.26%.
Final Examination 12. If f is integrable on [a, b] and g( x ) = ∫ f ( x ) dx,
sin 1.1 – sin 1 b
1.
0.1
= 0.497363752… then ∫a
f ( x ) dx = g(b) − g( a).
sin 1.01 – sin 1
= 0.536085981… 13. f (x) = x 2/3
0.01 2
sin 1.001 – sin 1 f ′( x ) = x −1/3
= 0.539881480… 3
0.001 f is differentiable everywhere except at x = 0. But
2. f ′(1) = cos 1 = 0.540302305… f is continuous at x = 0 because the limit of f (x)
The quotients in Problem 1 are converging to as x → 0 is zero, the same as f (0). Thus,
cos 1.

350 Problem Set 12-10 Calculus Solutions Manual


© 2005 Key Curriculum Press
f meets the hypotheses of the mean value 17. Cross section of solid at any point in the slice is
theorem because it is differentiable on (0, 1) and essentially the same as at the sample point.
continuous at 0 and 1. y
Slope of the secant line from (0, 0) to (1, 1) is 1. 4

(x, y )
2 −1/3 8
f ′( c ) =
c =1⇔ c =
3 27
Tangent at x = 8/27 is parallel to secant. x
2

f (x )
1
18. Height at any point in the slice is essentially the
same as at the sample point.
y
4
x
8/27 1 (x, y)

∫ ( x – 2)( x + 3) dx
5x – 3 x
14. a. Example: 2

 1 4 
= ∫  x – 2 + x + 3 dx 19. dM y = x dA = x(4 − x2) dx
2

= ln | x − 2 | + 4 ln | x + 3 | + C
My = ∫ 0
x ( 4 − x 2 ) dx = 4
2

∫ (4 – x
16
∫ A= ) dx =
2
b. Example: 9 – x 2 dx
0 3
x = 3 sin θ 4 3
xA = M y ⇒ x = =
dx = 3 cos θ dθ 16/3 4
9 – x 2 = 3 cos θ 20. Let H = number of calories added.
dH = C dT = (10 + 0.3T1/2) dT
∫ ∫ (1 + cos 2θ ) dθ
9
= 9 cos 2 θ dθ = 900

9 9
2 H= ∫ 100
(10 + 0.3T 1/2 ) dT = 13, 200 cal
= θ + sin 2θ + C x


2 4 sin u sin x
21. a. Si x = du ⇒ Si ′ x =
9 9 u
0 x
= θ + sin θ cos θ + C
b. Si x = 1 – u 2 + u 4 – u 6 + K du
t


2 2 1 1 1
9 x 1 0  3! 5! 7! 
= sin −1 + x 9 – x 2 + C
2 3 2 1 3 1 5 1 7 K
=t− t + t − t +
3 ⋅ 3! 5 ⋅ 5! 7 ⋅ 7!
∫ sec
3 u dv
15. x dx 1
sec x + sec 2 x c. Si 0.7 ≈ S1 (0.7) = 0.7 − (0.73 ) =
sec x tan x –
tan x 3 ⋅ 3!
0.68094444…


= sec x tan x − sec x tan 2 x dx 1
d. | R1 (0.7)| < | t2 (0.7)| = (0.75 ) =
5 ⋅ 5!
= sec x tan x − ∫ sec x dx + ∫ sec x dx
3 0.0002801…
S1(0.7) equals Si 0.7 correct to three decimal
places and is within ±0.3 in the fourth
2 ∫ sec x dx = sec x tan x + ∫ sec x dx
3
decimal place.
e. See Cumulative Review Number 1,
∫ sec
1 1
3
x dx = sec x tan x + ln | sec x + tan x | + C Problem 9.
2 2 r r r
22. r = (t 3 )i + (t 2 ) j
r r r r r
∫ ∫
dy dy r
16. = ky ⇒ = k dx ⇒ ln | y | = kx + C ⇒ v = (3t 2 )i + (2t ) j ⇒ v (0.5) = 0.75i + 1 j
dx y r r r r r r
| y | = ekx+ C = ekxeC ⇒ y = C1ekx a = (6t )i + (2) j ⇒ a (0.5) = 3i + 2 j

Calculus Solutions Manual Problem Set 12-10 351


© 2005 Key Curriculum Press
y →
a 23. r = cos θ
1 1

v
dA = r 2 dθ = cos 2 θ dθ
2 2
1 π /6 1
A= ∫ cos θ dθ ≈ 0.2391K
2
x 0 2
1
 π 3
 exactly 24 + 16 
The object is speeding up at t = 0.5 because the  
angle between the velocity and acceleration
vectors is acute, indicating that the tangential
component of acceleration acts in the same
direction as the rvelocity.
r
(Algebraically, v ⋅ a = 4.25, which is positive,
again indicating an acute angle.)

352 Problem Set 12-10 Calculus Solutions Manual


© 2005 Key Curriculum Press
Calculus: Concepts and Applications
Correction/Comment Form
Please help us correct and improve Calculus: Concepts and Applications. If you find mistakes in the
text or the teacher support materials, use this form to let us know. If you have general comments or
suggestions about the materials, we’d like to hear those as well. Once you’ve filled out this form, all
you have to do is fold it and drop it in the mail. We’ll pay the postage. Thank you!
Your name ____________________________________________________________
School _______________________________________________________________
School address _________________________________________________________
City/State/Zip _________________________________________________________
Phone ________________________________________________________________
Email ________________________________________________________________

Calculus: Concepts and Applications student text


Page _______ Comment ____________________________________________________________
Page _______ Comment ____________________________________________________________
Page _______ Comment ____________________________________________________________
Page _______ Comment ____________________________________________________________

Instructor’s Guide
Section ________________ Page _____ Comment ______________________________________
Section ________________ Page _____ Comment ______________________________________
Section ________________ Page _____ Comment ______________________________________
Section ________________ Page _____ Comment ______________________________________

Instructor’s Resource Book


Section ________________ Page _____ Comment ______________________________________
Section ________________ Page _____ Comment ______________________________________
Section ________________ Page _____ Comment ______________________________________
Section ________________ Page _____ Comment ______________________________________

Solutions Manual
Section ________________ Page _____ Comment ______________________________________
Section ________________ Page _____ Comment ______________________________________
Section ________________ Page _____ Comment ______________________________________
Section ________________ Page _____ Comment ______________________________________

Do you have any other comments about Calculus: Concepts and Applications or any suggestions for
improving the student text or the teacher’s material? ______________________________________
_________________________________________________________________________________
_________________________________________________________________________________
_________________________________________________________________________________
Fold carefully along this line.

Attn: Editorial Department


1150 65th Street
Emeryville, CA 94608-9740

Attention Editorial—Foerster, Calculus 2nd ed.

Fold carefully along this line.

Potrebbero piacerti anche